Download as pdf or txt
Download as pdf or txt
You are on page 1of 2273

VIKAS® PUBLISHING HOUSE PVT LTD

 
Preface

Scheme of the Test

Scheme and Syllabus

Chapter 1 Geomorphology

Chapter 2 Climatology

Chapter 3 Oceanography and Bio-Geography

Chapter 4 Geographic Thought

Chapter 5 Population and Settlement Geography

Chapter 6 Economic Geography

Chapter 7 Political, Social and Cultural Geography

Chapter 8 Regional Planning

Chapter 9 Geography of India

Chapter 10 Cartography

Chapter 11 Statistics

Chapter 12 Images

Chapter 13 Reading Comprehension


Chapter 14 Special Practice Zone for Paper - II

Chapter 15 Special Practice Zone for Paper - III

Chapter 16 Previous Years' Papers - Paper II

Chapter 17 Previous Years' Papers - Paper III

 
The present work is an attempt to provide a focused, well-structured
resource that is devised in a way so as to become a natural supplement to
any teaching methodology or preparation plan for National Eligibility Test
(NET) conducted by the University Grants Commission (UGC). It is vital to
mention that compiling and developing such a resource has required a
comprehensive research and a very methodical planning.

This book is an attempt to help the UGC-NET aspirants realize their


goals, by providing them with a good book containing both: (i)
comprehensive practice material, and (ii) original questions from previous
years' papers.

As with any preparation plan for a test, it is important to first know about
the scheme of the test, the format, the marking scheme and the syllabus.
These are available on the UGC website; however, these are also provided
in the next few pages for convenience of the students. Students are advised
to visit the UGC website regularly for the notified changes in these aspects
for the upcoming tests.

ACKNOWLEDGEMENTS

It is not easy to bring out a compendium of Multiple Choice Questions,


where collection, classification, sorting as per syllabus, their verification
and assessment of material is a huge task. The team behind this effort is a
gifted squad of researchers, who went through the entire process
meticulously, without losing steam at any phase, and kept in mind the
timelines and all qualitative aspects of publishing a book. Despite our best
efforts at the desk-level, there may still be a few errors that may have
unintentionally crept into the book. I request that all errors be brought
forward to us so we rectify these in our subsequent editions. Suggestions for
improving the book in any way are most welcome.

We wish all the aspirants the very best!

 
There are 3 papers, all of objective type assessment, conducted as follows:

PAPER - I [60 MCQs out of which 50 question to be attempted/100 marks;


Duration: 1'/4 hours]

Paper - I shall be of general nature, intended to assess the teaching/research


aptitude of the candidate. It will primarily be designed to test reasoning
ability, comprehension, divergent thinking and general awareness of the
candidate. Sixty (60) multiple choice questions of two marks each will be
given, out of which the candidate would be required to answer any fifty
(50). In the event of the candidate attempting more than fifty questions, the
first fifty questions attempted by the candidate would be evaluated.

We have a great resource, UGC-NET/JRF/SET Teaching and Research


Aptitude (General Paper - I) published by Vikas Publishing House, that was
released in January 2014 to help you prepare for this paper.

PAPER - II [50 MCQs/100 marks; Duration: 1'/4 hours]

Paper - II shall consist of 50 objective type compulsory questions based on


the subject selected by the candidate. Each question will carry 2 marks. All
questions of Paper - II will be compulsory, covering entire syllabus
(including all electives, without options).

PAPER - III [75 MCQs/150 marks; Duration: 2'/Z hours]

Paper - III will consist of 75 objective type compulsory questions from the
subject selected by the candidate. Each question will carry 2 marks. All
questions of Paper - III will be compulsory, covering entire syllabus
(including all electives, without options).

The candidate will have to mark the responses for questions of Paper - I,
Paper - II and Paper - III on the Optical Mark Reader (OMR) sheet
provided along with the Test Booklet. The detailed instructions for filling
up the OMR Sheet will be uploaded on UGC website (www.ugcnetonline.in
or www.ugc.ac.in). It may be noted that instructions to candidates will not
be sent to the candidate by post.

PROCEDURE & CRITERIA FOR DECLARATION OF RESULT

This will comprise of following steps:

Step I: Minimum marks to be obtained in NET for considering a candidate


for the award of JRF and eligibility for Assistant Professor:

The candidates are required to obtain following minimum marks


separately in Paper - I, Paper - II and Paper - III as given below:

Step II: Amongst those candidates who have cleared Step I, a merit list will
be prepared subject-wise and category-wise using the aggregate marks of
all the three papers secured by such candidates.

Step III: Top 15% candidates (for each subject and category), from the
merit list mentioned under Step II, will be declared NET qualified for
eligibility for Assistant Professor only.
Step IV: A separate merit list for the award of JRF will be prepared from
amongst the NET qualified candidates figuring in the merit list prepared
under Step III.

It may be noted that the above qualifying criteria decided by UGC is final
and binding.

For Persons with Disability (Visually Challenged candidates) thirty minutes'


compensatory time shall be provided separately for Paper - I and Paper - II.
For Paper - III, forty five minutes' compensatory time shall be provided.
They will also be provided the services of a scribe who would be a graduate
in a subject other than that of the candidate. UGC will also provide the Test
Booklets of Paper - I, II & III in Braille alongwith usual Test Booklets as
provided to other candidates.

Those Persons with Disability (Physically Challenged) candidates who are


not in a position to write in their own hand-writing can also avail these
services by making prior request (at least one week before the date of UGC-
NET) in writing to the Co-ordinator of the NET Coordinating Institution.
Compensatory time and facility of scribe would not be provided to other
Persons with Disability (Physically Challenged) candidates.

The facility of Scribe is applicable only for candidates with disability of


40% or more.

The candidate has the discretion of opting for his/her own scribe or has to
request the Co-ordinator of the Coordinating Institution for the same in
writing at least one week in advance of the test. In such instances the
candidate is allowed to meet the scribe a day before the examination so as
to verify whether the scribe is suitable or not. Those candidates who opt for
their own scribe have to produce the scribe before the Co-ordinator along
with her/his certificates of educational qualifications at least one day before
the test.

Syllabus of Test: All questions of Paper - II and Paper - III will be


compulsory, covering entire syllabus (including all electives, without
options). Syllabi for all NET subjects can be downloaded from the UGC
Website www. ugc. ac. in and are also available in the libraries of all Indian
Universities. UGC will not send the syllabus to individual candidates.

In case of any discrepancy found in the English and Hindi versions, the
questions in English version shall be taken as final.

 
Subject: GEOGRAPHY

Code No: 80

SYLLABUS FOR PAPER - II

1.Geomorphology: Fundamental concepts; Endogenetic and Exogenetic


forces; Denudation and weathering; Geosynclines, continental drift and
plate tectonics; Concept of geomorphic cycle; Landforms associated with
fluvial, glacial, arid, coastal and karst cycles.

2.Climatology: Composition and structure of the atmosphere; Heat budget


of the earth; Distribution of temperature; Atmospheric pressure and
general circulation of winds; Monsoon and jet stream; Tropical and
temperate cyclones; Classification of world climates; Koppen's and
Thornthwaite's schemes.

3.Oceanography: Ocean deposits; Coral reefs; Temperature and salinity of


the oceans; Density of sea water; Tides and ocean currents.

Bio-Geography: World distribution of plants and animals; Forms and


functions of ecosystem; Conservation and management of ecosystems;
Problems of pollution.

4.Geographic Thought: General character of Geographic knowledge during


the ancient and medieval period; Foundations of Modern Geography;
Determinism and possibilism; Areal differentiation and spatial
organisation.
5.Population Geography: Patterns of world distribution; Growth and density
of population; Patterns and processes of migration; Demographic
transition.

Settlement Geography: Site, situation, types, size, spacing and internal


morphology of rural and urban settlements; City-region; Primate city;
Rank-size rule; Settlement hierarchy; Christaller's Central Place theory;
August Losch's theory of market centres.

6.Economic Geography: Sectors of economy: primary, secondary, tertiary


and quaternary; Natural resources: renewable and non-renewable.

Measurement of agricultural productivity and efficiency; Crop


combination and diversification; Von Thunen's Model.

Classification of industries: Weber's and Losch's approaches; Resource


based and footloose industries.

Models of transportation and transport cost: Accessibility and


connectivity.

7.Political Geography: Heartland and Rimland theories; Boundaries and


frontiers; Nature of administrative areas and Geography of public policy
and finance.

Social Geography: Ethnicity; tribe; dialect; language, caste and religion;


Concept of social well-being.

Cultural Geography: Culture-areas and cultural regions; Human races;


Habitat; Economy and Society of tribal groups.
8.Regional Planning: Regional concept in Geography; Concept of planning
regions; Types of regions; Methods of regional delineation; Regional
planning in India; Indicators of development; Regional imbalances;
Evolution, nature and scope of town planning with special reference to
India, and Fundamentals of Town and Country planning.

9.Geography of India: Physiographic divisions; Climate: Its regional


variations; Vegetation types and vegetation regions; Major soil types;
Irrigation and agriculture; Population distribution and growth; Settlement
patterns; Mineral and power resources; major industries and industrial
regions.

10.Cartography: Types of maps: Techniques for the study of spatial patterns


of distribution; Choropleth; Isopleth and Chorochromatic maps and pie
diagrams; Mapping of location-specific data; Accessibility and flow
maps.

Remote sensing and Computer application in mapping; Digital mapping;


Geographic Information System (GIS).

Statistical Methods: Data sources and types of data; Frequency


distribution and cumulative frequency; Measures of central tendency;
Selection of class intervals for mapping; Measures of dispersion and
concentration; Standard deviation; Lorenz Curve; Methods of measuring
association among different attributes; Simple and Multiple correlation;
Regression.

Nearest-neighbour analysis; Scaling techniques; Rank score; Weighted


score; Sampling techniques for Geographical analysis.

SYLLABUS FOR PAPER - III


Unit-I

Geomorphology: Fundamental concepts; Factors controlling landform


development; Endogenetic and Exogenetic forces; Denudation process:
weathering and erosion, Geosynclines, mountain building, continental drift
and plate tectonics; Concept of Geomorphic Cycle; Landforms associated
with fluvial, glacial, arid, coastal and karst cycles, Slope forms and
processes; Environmental and Applied Geomorphology.

Unit-II

Climatology: Composition and structure of the atmosphere; Insolation; Heat


budget of the earth; Distribution of temperature, atmospheric pressure and
general circulation of winds; Monsoons and jet streams; Stability and
instability of the atmosphere; Air-masses; Fronts, temperate and tropical
cyclones; Types and distribution of precipitation; Classification of world
climates; Koppen's and Thornthwaite's schemes; Hydrological Cycle;
Global warming.

Unit-III

Oceanography: Origin of ocean basins; Bottom relief of Indian, Atlantic


and Pacific Oceans; Ocean deposits; Coral reefs; Temperature and salinity
of the Oceans; Density of sea water; Tides and ocean currents; Sea-level
changes.

Bio-Geography: Physical factors influencing world distribution of plants


and animals; Forms and functions of ecosystem: Forest, grassland, marine
and mountain ecosystem; Bio-diversity and its depletion through natural
and man induced causes; Conservation and management of ecosystems;
Environmental hazards and problems of pollution; Ozone depletion.
Unit-IV

History of Geographic Thought: General character of Geographic


knowledge during the ancient and medieval period; Foundations of Modern
Geography: Contribution of German, French, British and American
schools; Conceptual and methodological developments during the 20th
century; Changing paradigms; Man and Environment, determinism and
possibilism, areal differentiation and spatial organisation; Quantitative
revolution; Impact of positivism, humanism, radicalism and behaviouralism
in Geography.

Unit-V

Population Geography: Nature, scope, subject matter and recent trends;


Patterns of world distribution, growth and density of population; Policy
issues; Patterns and processes of migration; Demographic transition;
Population-resource regions.

Settlement Geography: Site, situation, types, size, spacing and internal


morphology of rural and urban settlement; Ecological processes of urban
growth; Urban fringe; City-region; Settlement systems; Primate city; Rank-
Size rule; Settlement hierarchy; Christaller's Central Place theory; August
Losch's theory of market centres.

Unit-VI

Economic Geography: Location of economic activities and spatial


organization of economies; Classification of economies; Sectors of
Economy: primary, secondary, tertiary and quaternary; Natural resources:
Renewable and non-renewable; Conservation of resources.
Agricultural Geography: Concept and techniques of delimitation of
agricultural regions; Measurement of agricultural productivity and
efficiency; Crop combinations and diversification; Von Thunen's Model;
Agricultural systems of the world.

Industrial Geography: Classification of industries: Weber's and Losch's


approaches; Resource based and footloose industries.

Geography of Transport and Trade: Models of transportation and transport


cost; Accessibility and connectivity: Inter-regional and Intra-regional:
Comparative cost a dvantages.

Unit-VII

Political Geography: Definition and scope of Political Geography;


Geopolitics; Global strategic views (Heartland and Rimland theories);
Concept of nation, state and Nation-State; Boundaries and frontiers; Politics
of world resources; Geography and Federalism.

Social Geography: Nature and scope of social geography; Social structure


and social processes; Elements of Social Geography-ethnicity, tribe, dialect,
language, caste and religion; Concept of Social well-being.

Cultural Geography: Nature and scope of Cultural Geography; Environment


and culture; Concept of culture-areas and cultural regions; Theories of tribal
groups; Dwelling places as cultural expressions.

Unit-VIII

Regional Planning: Regional concept in Geography; its application to


planning; Concept of planning region; Regional hierarchy; Types of regions
and methods of regional delineation; Conceptual and theoretical framework
of regional planning; Regional planning in India: Concept of development;
Indicators of development; Regional imbalances.

Unit-IX

Geography of India: Physiographic divisions; Climate: Its regional


variations; Vegetation types and vegetation regions; Major soil types;
Coastal and Marine resources; Water resources; Irrigation; Agriculture;
Agroclimatic regions; Mineral and power resources; Major industries and
industrial regions; Population distribution and growth; Settlement patterns;
Regional disparities in social and economic development.

Unit-X

Cartography: Map as a tool in Geographical studies; Types of maps:


Techniques for the study of spatial patterns of distribution; Single purpose
and composite maps; Choropleth, Isopleth and Chorochromatic maps and
pie diagrams; Mapping of location specific data; Accessibility and flow
maps.

Remote sensing and computer application in mapping; Digital mapping;


Geographic Information System (GIS): Thematic maps.

Statistical Methods: Data sources and types of data; Statistical diagrams;


study of frequency distribution and cumulative frequency; Measures of
central tendency; Selection of class intervals for mapping; Measures of
dispersion and concentration; Standard deviation; Lorenz curve; Methods of
measuring association among different attributes; Simple and multiple
correlation; Regression.

Measurement of spatial patterns of distribution: Nearest-neighbour analysis;


Scaling techniques, rank score, weighted score; Sampling techniques for
geographical analysis.

SAMPLE QUESTIONS

PAPER-II

1.Who, among the following, is associated with the concept of `spatial


diffusion'?

(a)Hartshorne

(b)Hagerstrand

(c)Humboldt

(d)Harvey

2.What is the thrust of the journal Antipode?

(a)Physical Geography

(b)Quantitative Geography

(c)Radical Geography

(d)Behavioural Geography

3.Who wrote the book Spatial Organisation?

(a)Abler

(b)Johnson

(c)Haggett
(d)Johnston

PAPER - III (A)

Identify Landforms associated with `Karst Topography'.

Or

Differentiate between Endogenetic and Exogenetic forces in


Geomorphology.

PAPER - III (B)

Write an essay on `Environmental and Applied Geomorphology'.

Or

Which are the parameters used for forecasting of Indian monsoons?

Or

Write a note on the conservation and management of Ocean-resources.

 
1.Delta kames are the outcome of

(a)glacial erosion.

(b)wind deposition.

(c)river deposition.

(d)glacial deposition.

[UGC D-2010]

2.A ramp of sand deposited by streams along foothills in the and land is called

(a)hamada.

(b)bajada.

(c)desert wash.

(d)pediment.

[UGC D-2010]

3.Arrange the following mountain building theories in order in which they


were published using the codes given.

1.Continental Drift Theory

2.Convection Current Theory

3.Sea Floor Spreading Theory


4.Plate Tectonic Theory

Codes:

[UGC D-2010]

4.Given below are two statements. One is labelled as Assertion (A) and the
other is labelled as Reason (R), Select your answers from the codes given.

Assertion (A): Remains of older mountain belts lie within the shields in few
places.

Reason (R): Older mountain belts in shield areas are composed of soft
sedimentary rocks.

(a)(A) is correct, but (R) is wrong.

(b)Both (A) and (R) are correct.

(c)(A) is wrong, but (R) is correct.

(d)Both (A) and (R) are wrong.

[UGC J-2010]

5.Flat-topped, steep-sided upland capped by a resistant rock layer normally


formed in and lands is called

(a)erg.
(b)cuesta.

(c)mesa

(d)escorponant.

[UGC D-2010]

6.Given below are two statements, one labelled as Assertion (A) and the other
labelled as Reason (R). Select your answer from the codes given.

Assertion (A): Flowing as a sheet across a land surface, running water picks
up particles and moves them down slope into a stream channel.

Reason (R): Stream flow always transports all kinds of sediment load
downstream.

(a)Both (A) and (R) are true.

(b)Both (A) and (R) are true, but (R) is not the correct explanation.

(c)(A) is true, but (R) is false.

(d)(A) is false, but (R) is true.

[UGC D-2010]

7.Match List I with List II and select the correct answer using the codes given.
Codes:

[UGC D-2010]

8.are both deposited by meltwater streams.

(a)Terminal moraines and cirques

(b)Outwash plains and valley trains

(c)Recessional kettles and erratics

(d)Valley moraines and ice sheet trains

9.are both dry land, erosional features cut from bedrock.

(a)Pediments and inselbergs

(b)Pediments and playas


(c)Bajadas and blowouts

(d)Inselbergs and barcanoids

10.are built more or less parallel to the beach.

(a)Seawalls

(b)Breakers

(c)Jetties

(d)Groins

11.are coastal structures designed to keep tidal inlets from shifting location or
filling with sand.

(a)Jetties

(b)Breakwaters

(c)Seawalls

(d)Groins

12.are erosional features produced by valley/Alpine glaciers.

(a)Eskers

(b)Moraines

(c)Cirques

(d)Drumlins

13.denotes the exposed, crescentshaped rupture surface at the head of a slump.


(a)Sole

(b)Scarp

(c)Scoop

(d)Toe

14.dunes are long, high sand dunes parallel with the prevailing wind direction.

(a)Transversal

(b)Longitudinal

(c)Transducinal

(d)Latitudinal

15.dunes result from persistent, onshore winds in certain coastal areas.

(a)Barchanoid

(b)Staroid

(c)Parabolic

(d)Diabolic

16.first related the symmetrical magnetic patterns in seafloor basalts toseafloor


spreading at a mid-ocean ridge.

(a)Wegener and Wilson

(b)Vine and Matthews

(c)Evans and Novak


(d)Matthews and Marks

17.has/have speeds comparable to those of a flowing Alpine glacier.

(a)Soil creep

(b)Basaltic lava flows

(c)Streams

(d)Mudflows

18.have rainfall amounts and soil moisture contents between those of true
deserts and humid lands.

(a)Steppes

(b)Sundras

(c)Tundras

(d)Sabkhas

19.involveso movement on a zone f compressed air.

(a)Soil creep

(b)A slump

(c)A mudflow

(d)A rock avalanche

20.is an irregular, usually thin till layer laid down by a retreating glacier.

(a)Kame sheet
(b)Outwash blanket

(c)Terminal moraine

(d)Ground moraine

21.is one of the two major flow mechanisms in a glacier.

(a)Basal slip

(b)Morainal sliding

(c)Frost heaving

(d)Crevassal slip

22.The best explanation for a glacial surge.

(a)The climate cools suddenly and a retreating glacier begins to advance.

(b)Heavy snowfalls resulting in avalanches in the zone of accumulation.

(c)Crevasses opening suddenly near the snout of a glacier.

(d)Melting at the base of the glacier resulting in increased rates of basal slip.

23. is the process by which rocks breakdown in place to produce soils and
sediments.

(a)Lithification

(b)Metamorphism

(c)Weathering

(d)Subduction
24.The following most effectively outlines the edges of the lithospheric plates.

(a)The locations of deep mantle hot spots

(b)Lines of earthquake epicentres

(c)Margins of the continental shelves

(d)Lines of active stratovolcanoes

25.not the direct result of longshore current action.

(a)Cracking and abrasion of rock at the base of a wave-cut cliff are

(b)Closing off of small estuaries by bay mouth bars is

(c)Transport of sand along the beach is

(d)Building and extension of spits are

26.plate boundaries have the largest magnitude earthquakes.

(a)Divergent and convergent

(b)Transform and convergent

(c)Divergent and transform

(d)All plate boundaries have large magnitude earthquakes.

27.refers to the `bouncing' mode of sand transport in a windstorm or stream.

(a)Deflation

(b)Ventifaction
(c)Siltation

(d)Saltation

28.was a flora that was widely distributed throughout the once contiguous late
Palaeozoic, southern hemisphere landmass.

(a)Mesosaurus

(b)Glossopteris

(c)Monastarious

(d)Granopteris

29.was a very large, pluvial lake in Utah during the Pleistocene epoch.

(a)Lake Bonneville

(b)Lake Aral

(c)Lake Mead

(d)Lake Caspian

30.This was never proposed as evidence supporting the existence of Pangaea.

(a)Glossopteris flora

(b)Geometrical fit between South America and Africa

(c)Late Palaeozoic glacial features

(d)Islands along the mid-Atlantic ridge with rocks dating back from 542
million to 2,500 million years ago
31.Water table refers to

(a)the lower limit of the zone of saturation.

(b)the upper limit of the zone of saturation.

(c)seepage of water into fissures lying underground.

(d)the contact zone of the permeable and impermeable rocks.

[UPSC-1996]

32.A extends partway across the mouth of a bay or estuary.

(a)sand arch

(b)barrier island

(c)spit

(d)jetty

33.A forms when stagnant glacial ice melts after being buried by drift.

(a)kettle

(b)drumlin

(c)tam

(d)pluvial delta

34.A is a crescent-shaped dune whose tips point downwind.

(a)parabarcal
(b)barchan

(c)star

(d)transverse

35.A is a glacier-cut valley that partly flooded as sea levels increased.

(a)hanging cirque

(b)fiord

(c)kettle trough

(d)till crevasse

36. Ac is an intermittent stream hannel in the dry land areas of the western
USA.

(a)rill

(b)wash

(c)playa

(d)rivulet

37. A is an isolated remnant of bedrock standing above a wave-cut platform.

(a)sea spit

(b)sea rampart

(c)sea span

(d)sea stack
38.A is formed by abrasion of rocks by windblown sand.

(a)playa

(b)pediment

(c)ventifact

(d)desert pavement

39.A migrates inland as a wavecut platform is extended.

(a)wave-cut barrier stack

(b)wave-cut arch

(c)wave-cut cliff

(d)wave-cut barrier dune

40.A bay mouth bar is

(a)a sand bar extending across the former inlet to a bay or estuary

(b)a sand deposit on the seaward side of a tidal inlet to a large estuary

(c)a sand deposit on the estuary side of an inlet through a barrier island

(d)a sand barrier extending partway across the entrance to a bay or estuary

41.A drumlin is a

(a)bowl-shaped depression eroded largely by frost action and glacial


plucking

(b)smooth, striated bedrock ridgeshaped and polished by a glacier


(c)till mound of outwash deposited by meltwater streams at the snout of a
glacier

(d)smooth, tapering ridge of till formed and shaped beneath a continental


ice sheet

42.A fiord is

(a)a glacier-cut valley which sinks below the sea level due to glacial
rebound after the ice melts

(b)a stream valley, deepened by glacial erosion, that floods as the sea level
rises

(c)a large, kettle-pocked moraine left as an island when sea level rises
following melting of the ice

(d)a glacier-cut valley that is dammed by an end moraine and a large lake is
formed

43.A geologist wants to locate a horizontal coal bed on a soil-covered slope


where soil creep is active. A distinctive sandstone bed underlies the coal
bed. On the basis of weathered sandstone fragments in the soil, where is the
coal bed?

(a)Just below the highest piece of sandstone seen in the soil

(b)Just above the highest piece of sandstone seen in the soil

(c)Just below the lowest piece of sandstone seen in the soil

(d)Just above the lowest piece of sandstone seen in the soil


44.A landform developed by the process of deposition occurring towards the
centre of intermountain basis in and or semiarid regions is known as

(a)pediment.

(b)bajada.

(c)playa.

(d)peneplain.

[UPSC-2009]

45.A lens-shaped igneous intrusion situated beneath an anticlinal fold or in the


base of syncline is called

(a)lapolith.

(b)lacolith.

(c)phacolith.

(d)batholith.

[UPSC-2009]

46.A mountainous coast having presence of large numbers of narrow, steep-


sided elongated and inundated coastal valleys is called

(a)Ria coast.

(b)fiord coast.

(c)Haff coast.

(d)Dalmatian coast.
[UPSC-2002]

47.A natural sand bar or low sand ridge that connects one island to another
island or to the mainland is called a

(a)tombolo

(b)sand groin

(c)spit

(d)jetty barrier

48.A transform plate boundary is characterized by

(a)a deep, vertical fault along which two plates slide past one another in
opposite directions

(b)two converging oceanic plates meeting head-on and piling up into a mid-
ocean ridge

(c)a divergent boundary where the continental plate changes to an oceanic


plate

(d)stratovolcanoes on the edge of a plate and shield volcanoes on the


adjacent plate

49.A typical rate of seafloor spreading in the Atlantic Ocean is

(a)2 feet per year

(b)20 feet per year

(c)2 centimetres per year

(d)0.1 inches per year


50.A weathering process in which layers of rocks peel off as expansion
alternates with contraction is known as

(a)shattering.

(b)block separation.

(c)granular disintegration.

(d)exfoliation.

[UPSC-2004]

51.A(n) is a depositional feature composed of till and only associated with


continental glaciation, not with Alpine glaciers.

(a)till

(b)solid rock

(c)outwash deposit

(d)outwash

52.A(n) is likely to host a waterfall or steep rapids today.

(a)hanging valley

(b)striated drumlin

(c)outwash plain

(d)horn peak

53.A(n) is similar in appearance to a sinkhole of a karst area.


(a)kettle

(b)cirque

(c)moraine

(d)esker

54.A(n) represents a former meltwater channel or tunnel in glacial ice that was
filled with sand and gravel.

(a)esker

(b)valley plain

(c)yazoo ridge

(d)kettle

55.A(n) would logically be situated next to a large end moraine or terminal


moraine.

(a)cirque

(b)fiord

(c)outwash plain

(d)pluvial lake

56.Abrasion and plucking generally involve what part of a glacier'?

(a)The internal, flowage zone

(b)The snout zone


(c)The basal, sliding zone

(d)The surface, brittle zone

57.Active mountain belts are most likely to be found

(a)along the margins of continents

(b)scattered throughout continents

(c)in the interior regions of continents

(d)along only the eastern margins of continents

58.All of the following are evidence supporting the theory of plate tectonics
except for

(a)ocean floor drilling

(b)measurements of plate motions

(c)hot spots

(d)changes in the Moon's orbit due to shifting plates

59.All of the following are factors affecting mass wasting except for

(a)water

(b)gravity

(c)geologic age

(d)slope angle
60.All of the following are possible indicators that creep is occurring except
for

(a)an extremely thick soil profile

(b)tilted fences or power line poles

(c)curved tree trunks

(d)cracks in roads or sidewalks

61.All of the following are possible steps of scientific investigation except for

(a)development of observations and experiments to test the hypotheses

(b)the collection of scientific facts through observation and measurement

(c)the development of one or more working hypotheses or models to


explain facts

(d)assumption of conclusions without prior experimentation or observation

62.All of the following are thought to possibly contribute to the formation of


glaciers except for

(a)comets

(b)plate tectonics

(c)precession

(d)eccentricity

63.All of the following descriptions apply to stratified drift (not glacial till)
except for
(a)sand and gravel beds

(b)deposited directly from melting ice

(c)deposits are often layered and wellsorted

(d)deposited by glacial meltwater streams

64.All of the following provide evidence or clues to the composition of Earth's


interior except for

(a)slivers of crustal and mantle rocks now exposed at Earth's surface

(b)comets

(c)meteorites

(d)diamond-bearing rocks

65.Alluvial cone is predominant in

(a)coastal belt.

(b)piedmont zone.

(c)delta region.

(d)pediment region.

[UPSC-1999]

66.Among the following elements, which one is in the highest amount in the
composition of basalt rock?

(a)Aluminium
(b)Calcium

(c)Iron

(d)Silicon

[UPSC-2008]

67.An area which has not been glaciatedmay show

(a)hanging valleys.

(b)corries and pyramidal peaks.

(c)drumlins.

(d)V-shaped valleys.

[UPSC-2006]

68.Approximately how long ago did the last of the great North American ice
sheets melt'?

(a)1,500 years

(b)1,50,000 years

(c)1.5 million years

(d)15,000 years

69.Aretes are sharp ridges which develop between the adjacent

(a)cirques.

(b)Roche moutonnee.
(c)medial moraines.

(d)U-shaped valleys.

[UPSC-2009]

70.As an erosional process, how is mass wasting unique from wind, water and
ice'?

(a)Mass wasting affects particles of all sizes whereas the others affect only
smaller particles.

(b)Mass wasting affects much larger geographic areas than does wind, water
and ice.

(c)Mass wasting does not require a transporting medium.

(d)All of the above make mass wasting unique compared to wind, water and
ice.

71.Given below are two statements, one labelled as Assertion (A) and the other
labelled as Reason (R). Select your answer from the codes given.

Assertion (A): Laterite soils are red in colour and coarse in texture.

Reason (R): Abundant phosphoric acid and silica give these soils the
particular colour and texture.

(a)Both (A) and (R) are individually true and (R) is the correct explanation
of (A).

(b)Both (A) and (R) are individually true but (R) is not the correct
explanation of (A).

(c)(A) is true but (R) is false.


(d)(A) is false but (R) is true.

[UPSC-2007]

72.Given below are two statements, one labelled as Assertion (A) and the other
labelled as Reason (R). Select your answer from the codes given.

Assertion (A): The difference in the geomorphic features of different areas


of the Earth's surface is known as area differentiation.

Reason (R): This definition of geography is based on interrelation,


differential character and areal expression of different phenomena.

(a)Both (A) and (R) are individually true and (R) is the correct explanation
of (A).

(b)Both (A) and (R) are individually true but (R) is not the correct
explanation of (A).

(c)(A) is true but (R) is false.

(d)(A) is false but (R) is true.

[UPSC-1999]

73.Given below are two statements, one labelled as Assertion (A) and the other
labelled as Reason (R). Select your answer from the codes given.

Assertion (A): The most important single agent of physical weathering is


the freeze/that action of water in open spaces in rock.

Reason (R): Repetition is the key to understanding the force of frost


shattering.
(a)Both (A) and (R) are individually true and (R) is the correct explanation
of (A).

(b)Both (A) and (R) are individually true but (R) is not the correct
explanation of (A).

(c)(A) is true but (R) is false.

(d)(A) is false but (R) is true.

[UPSC-2000]

74.Given below are two statements, one labelled as Assertion (A) and the other
labelled as Reason (R). Select your answer from the codes given.

Assertion (A): Vertisols are most extensive in Australia, India and Sudan.

Reason (R): The clay-producing materials are available in mesothermal or


tropical climates with periodic dry and wet seasons

(a)Both (A) and (R) are individually true and (R) is the correct explanation
of (A).

(b)Both (A) and (R) are individually true but (R) is not the correct
explanation of (A).

(c)(A) is true but (R) is false.

(d)(A) is false but (R) is true.

[UPSC-2007]

75.Given below are two statements, one labelled as Assertion (A) and the other
labelled as Reason (R). Select your answer from the codes given.
Assertion (A): Akasi Chin is desolate and the direst part of the Ladakh
region.

Reason (R): The whole are bears a stamp of excessive Aeolian erosion.

(a)Both (A) and (R) are individually true and (R) is the correct explanation
of (A).

(b)Both (A) and (R) are individually true but (R) is not the correct
explanation of (A).

(c)(A) is true but (R) is false.

(d)(A) is false but (R) is true.

[UPSC -2001]

76.Given below are two statements, one labelled as Assertion (A) and the other
labelled as Reason (R). Select your answer from the codes given.

Assertion (A): Bajadas or piedmont alluvial plains extend for several miles
away from a mountain front.

Reason (R): A series of adjacent alluvial fans sometimes coalesce to form


an extensive piedmont alluvial plain which is also calledbajadas.

(a)Both (A) and (R) are individually true and (R) is the correct explanation
of (A).

(b)Both (A) and (R) are individually true but (R) is not the correct
explanation of (A).

(c)(A) is true but (R) is false.

(d)(A) is false but (R) is true.


[UPSC-2003]

77.Given below are two statements, one labelled as Assertion (A) and the other
labelled as Reason (R). Select your answer from the codes given.

Assertion (A): Basalt is a fine-grained and dark-coloured igneous rock


which is formed below the Earth's surface.

Reason (R): The rate of cooling and solidification of magma inside the
Earth is very slow.

(a)Both (A) and (R) are individually true and (R) is the correct explanation
of (A).

(b)Both (A) and (R) are individually true but (R) is not the correct
explanation of (A).

(c)(A) is true but (R) is false.

(d)(A) is false but (R) is true.

[UPSC-2001]

78.Given below are two statements, one labelled as Assertion (A) and the other
labelled as Reason (R). Select your answer from the codes given.

Assertion (A): The circum-Pacific belt accounts for major percentage share
of world's total earthquakes of varying magnitude and deep focus.

Reason (R): Maximum occurrence of earthquakes along the circum-Pacific


belt is caused by divergent motion of tectonic plates and consequent
formation of faults.

(a)Both (A) and (R) are individually true and (R) is the correct explanation
of (A).
(b)Both (A) and (R) are individually true but (R) is not the correct
explanation of (A).

(c)(A) is true but (R) is false.

(d)(A) is false but (R) is true.

[UPSC-2002]

79.Given below are two statements, one labelled as Assertion (A) and the other
labelled as Reason (R). Select your answer from the codes given.

Assertion (A): Estuaries were never considered important for human


settlement.

Reason (R): They provided a good food source.

(a)Both (A) and (R) are individually true and (R) is the correct explanation
of (A).

(b)Both (A) and (R) are individually true but (R) is not the correct
explanation of (A).

(c)(A) is true but (R) is false.

(d)(A) is false but (R) is true.

80.Given below are two statements, one labelled as Assertion (A) and the other
labelled as Reason (R). Select your answer from the codes given.

Assertion (A): Explosive type of volcanic eruptions are associated with the
destructive or convergent plate boundaries in which the heavier plate is sub
ducted beneath the lighter plate
Reason (R): Material of the upper mantle lying below the mid-oceanic
ridges are melted and move upwards in the form of lava during volcanic
eruptions.

(a)Both (A) and (R) are individually true and (R) is the correct explanation
of (A).

(b)Both (A) and (R) are individually true but (R) is not the correct
explanation of (A).

(c)(A) is true but (R) is false.

(d)(A) is false but (R) is true.

[UPSC-2001]

81.Given below are two statements, one labelled as Assertion (A) and the other
labelled as Reason (R). Select your answer from the codes given.

Assertion (A): Fjords are the features produced by glacial erosion.

Reason (R): Glaciers descending from coastal mountains may reach the sea
and continue their erosion on the seafloor.

(a)Both (A) and (R) are individually true and (R) is the correct explanation
of (A).

(b)Both (A) and (R) are individually true but (R) is not the correct
explanation of (A).

(c)(A) is true but (R) is false.

(d)(A) is false but (R) is true.

[UPSC-2000]
82.Given below are two statements, one labelled as Assertion (A) and the other
labelled as Reason (R). Select your answer from the codes given.

Assertion (A): For formation of inland lagoons, offshore currents must be


strong.

Reason (R): Strong currents carry material away before it can be deposited.

(a)Both (A) and (R) are individually true and (R) is the correct explanation
of (A).

(b)Both (A) and (R) are individually true but (R) is not the correct
explanation of (A).

(c)(A) is true but (R) is false.

(d)(A) is false but (R) is true.

83.Given below are two statements, one labelled as Assertion (A) and the other
labelled as Reason (R). Select your answer from the codes given.

Assertion (A): Graben is a major relief feature resulting from the faulting
activities. It is, in fact, a valley or narrow trough bounded by one more
parallel faults.

Reason (R): Block of the Earth's crust may be relatively raised or lowered
between more or less parallel faults. The lower part known as graben is
formed due to the subsidence of middle part between two normal faults.

(a)Both (A) and (R) are individually true and (R) is the correct explanation
of (A).

(b)Both (A) and (R) are individually true but (R) is not the correct
explanation of (A).
(c)(A) is true but (R) is false.

(d)(A) is false but (R) is true.

[UPSC-2003]

84.Given below are two statements, one labelled as Assertion (A) and the other
labelled as Reason (R). Select your answer from the codes given.

Assertion (A): Hydraulic action is the work of turbulence in the water.

Reason (R): Running water causes hydraulic squeeze and releases action.

(a)Both (A) and (R) are individually true and (R) is the correct explanation
of (A).

(b)Both (A) and (R) are individually true but (R) is not the correct
explanation of (A).

(c)(A) is true but (R) is false.

(d)(A) is false but (R) is true.

85.Given below are two statements, one labelled as Assertion (A) and the other
labelled as Reason (R). Select your answer from the codes given.

Assertion (A): In a resequent fault line scarp, the down throw side will form
lower ground.

Reason (R): The down throw side possesses the softer rock at the level of
present erosion surface.

(a)Both (A) and (R) are individually true and (R) is the correct explanation
of (A).
(b)Both (A) and (R) are individually true but (R) is not the correct
explanation of (A).

(c)(A) is true but (R) is false.

(d)(A) is false but (R) is true.

[UPSC-2005]

86.Given below are two statements, one labelled as Assertion (A) and the other
labelled as Reason (R). Select your answer from the codes given.

Assertion (A): Our knowledge of Earth's inner differentiation is acquired


entirely through indirect evidence.

Reason (R): We are unable to drill more than a few kilometres into Earth's
crust.

(a)Both (A) and (R) are individually true and (R) is the correct explanation
of (A).

(b)Both (A) and (R) are individually true but (R) is not the correct
explanation of (A).

(c)(A) is true but (R) is false.

(d)(A) is false but (R) is true.

87.Given below are two statements, one labelled as Assertion (A) and the other
labelled as Reason (R). Select your answer from the codes given.

Assertion (A): Pediments are the bed rock portions of piedmont slopes
which extend downwards to neighbouring basin floors.
Reason (R): It is commonly believed that the dissected pediments are
formed by erosion of original flat pediments under the second cycle of
erosion.

(a)Both (A) and (R) are individually true and (R) is the correct explanation
of (A).

(b)Both (A) and (R) are individually true but (R) is not the correct
explanation of (A).

(c)(A) is true but (R) is false.

(d)(A) is false but (R) is true.

[UPSC-1999]

88.Given below are two statements, one labelled as Assertion (A) and the other
labelled as Reason (R). Select your answer from the codes given.

Assertion (A): Plutonic rocks are intrusive rocks of deep-seated origin.

Reason (R): Plutonic rocks get cooled slowly at great depth with large
crystals.

(a)Both (A) and (R) are individually true and (R) is the correct explanation
of (A).

(b)Both (A) and (R) are individually true but (R) is not the correct
explanation of (A).

(c)(A) is true but (R) is false.

(d)(A) is false but (R) is true.

[UPSC-1996]
89.Given below are two statements, one labelled as Assertion (A) and the other
labelled as Reason (R). Select your answer from the codes given.

Assertion (A): River system can continue to cut down its valleys at
approximately the same rate as uplift and so maintains its general pattern
and direction.

Reason (R): In case of an antecedent drainage system, the river is said to


have originated before a period of uplift and folding of land as a result of
Earth's movement.

(a)Both (A) and (R) are individually true and (R) is the correct explanation
of (A).

(b)Both (A) and (R) are individually true but (R) is not the correct
explanation of (A).

(c)(A) is true but (R) is false.

(d)(A) is false but (R) is true.

[UPSC-2005]

90.Given below are two statements, one labelled as Assertion (A) and the other
labelled as Reason (R). Select your answer from the codes given.

Assertion (A): Straight slope is a type recognized by W.Penck.

Reason (R): Straight slope develops due to increasing intensity of erosion.

(a)Both (A) and (R) are individually true and (R) is the correct explanation
of (A).

(b)Both (A) and (R) are individually true but (R) is not the correct
explanation of (A).
(c)(A) is true but (R) is false.

(d)(A) is false but (R) is true.

[UPSC-1996]

91.Given below are two statements, one labelled as Assertion (A) and the other
labelled as Reason (R). Select your answer from the codes given.

Assertion (A): The beaches act to stabilize a shoreline by absorbing wave


energy-

Reason (R): The sediments on the beaches are almost always in motion.

(a)Both (A) and (R) are individually true and (R) is the correct explanation
of (A).

(b)Both (A) and (R) are individually true but (R) is not the correct
explanation of (A).

(c)(A) is true but (R) is false.

(d)(A) is false but (R) is true.

92.Given below are two statements, one labelled as Assertion (A) and the other
labelled as Reason (R). Select your answer from the codes given.

Assertion (A): The continually forming oceanic crust records each magnetic
reversal and reorientation of Earth's polarity.

Reason (R): The seafloor is remarkably young, not exceeding 208 million
years.

(a)Both (A) and (R) are individually true and (R) is the correct explanation
of (A).
(b)Both (A) and (R) are individually true but (R) is not the correct
explanation of (A).

(c)(A) is true but (R) is false.

(d)(A) is false but (R) is true.

93.Given below are two statements, one labelled as Assertion (A) and the other
labelled as Reason (R). Select your answer from the codes given.

Assertion (A): The location of the epicentre of an earthquake may be


estimated using the time-tag between the arrival of P-wave cannot start until
the P-wave hits the surface.

Reason (R): The L-wave cannot start until the P-wave hits the surface.

(a)Both (A) and (R) are individually true and (R) is the correct explanation
of (A).

(b)Both (A) and (R) are individually true but (R) is not the correct
explanation of (A).

(c)(A) is true but (R) is false.

(d)(A) is false but (R) is true.

[UPSC-2003]

94.Given below are two statements, one labelled as Assertion (A) and the other
labelled as Reason (R). Select your answer from the codes given.

Assertion (A): The origin of Himalayas is due to the collision of the Indian
subcontinent with Eurasian landmass.
Reason (R): The convergence of crystal plates is often associated with
mountain building.

(a)Both (A) and (R) are individually true and (R) is the correct explanation
of (A).

(b)Both (A) and (R) are individually true but (R) is not the correct
explanation of (A).

(c)(A) is true but (R) is false.

(d)(A) is false but (R) is true.

[UPSC-1998]

95.Given below are two statements, one labelled as Assertion (A) and the other
labelled as Reason (R). Select your answer from the codes given.

Assertion (A): The outer core of the Earth's interior is in molten state while
the inner core is in solid state.

Reason (R): The Pseismic waves disappear in outer core while S-waves
penetrate up to the inner core.

(a)Both (A) and (R) are individually true and (R) is the correct explanation
of (A).

(b)Both (A) and (R) are individually true but (R) is not the correct
explanation of (A).

(c)(A) is true but (R) is false.

(d)(A) is false but (R) is true.

[UPSC-2005]
96.Given below are two statements, one labelled as Assertion (A) and the other
labelled as Reason (R). Select your answer from the codes given.

Assertion (A): The structure of the interior of the Earth is deduced with the
help of seismic waves.

Reason (R): The S-waves travel in solid, liquid and gas while the P-waves
travel only in solid.

(a)Both (A) and (R) are individually true and (R) is the correct explanation
of (A).

(b)Both (A) and (R) are individually true but (R) is not the correct
explanation of (A).

(c)(A) is true but (R) is false.

(d)(A) is false but (R) is true.

[UPSC-2001]

97.Given below are two statements, one labelled as Assertion (A) and the other
labelled as Reason (R). Select your answer from the codes given.

Assertion (A): When floods occur, the floodplain is inundated.

Reason (R): When the water recedes, it takes with it alluvial deposits.

(a)Both (A) and (R) are individually true and (R) is the correct explanation
of (A).

(b)Both (A) and (R) are individually true but (R) is not the correct
explanation of (A).

(c)(A) is true but (R) is false.


(d)(A) is false but (R) is true.

98.Given below are two statements, one labelled as Assertion (A) and the other
labelled as Reason (R). Select your answer from the codes given.

Assertion (A): When the longitudinal profile of a stream shows an abrupt


change in gradient, the point of interruption is termed knickpoint.

Reason (R): The interruption is temporary in nature.

(a)Both (A) and (R) are individually true and (R) is the correct explanation
of (A).

(b)Both (A) and (R) are individually true but (R) is not the correct
explanation of (A).

(c)(A) is true but (R) is false.

(d)(A) is false but (R) is true.

99.Assume that the central slip face of a barchan dune slopes downhill towards
the east. What is the direction of the prevailing wind'?

(a)East to west

(b)South to north

(c)North to south

(d)West to east

100.Barrier islands are

1.unwise choice for home sites or commercial building.


2.common choice for home sites or commercial building.

(a)Only 1

(b)Only 2

(c)Both 1 and 2

(d)Neither 1 nor 2

101.Calcium carbonate deposited by precipitation form carbonate-saturated


waters around hot spring is called

(a)stalactite.

(b)stalagmite.

(c)travertine.

(d)lapis.

[UPSC-2009]

102.Caledonian organic movements are related to geological history of

(a)late Jurassic and early Cretaceous period.

(b)late Silurian and early Devonian period.

(c)middle Triassic period.

(d)late Devonian and early Carboniferous period.

[UPSC-2009]

103.Chemical weathering is most prominent in


(a)and region.

(b)semi-arid region.

(c)humid tropical region.

(d)Mediterranean region.

[UPSC-1999]

104.Consider a steep highway cut made by removing slightly weathered to


fresh, fractured granite bedrock. Which of the following situations is most
stable against rockslides?

(a)One set of widely spaced, sub-horizontal fractures

(b)Two sets of fractures, one inclined towards the road cut and the other
away

(c)One set of widely spaced fractures inclined towards the road cut

(d)Two sets of fractures, one widely spaced and sub-horizontal, the other
inclined away from the highway cut

105.Consider a weathered rock or soil particle lying on a slope. How will the
gravitational force pulling the particle downwards along the land surface
vary with the inclination of the slope'?

(a)It will increase as the slope angle is lessened.

(b)It will possibly increase or decrease as slope angle is lessened, depending


upon other factors.

(c)It is not affected by the slope angle.


(d)It will decrease as the slope angle is lessened.

106.Consider the following are formed due to collision of continental plates:

1.The Alps

2.The Himalayas

3.The Rockies

4.The Caucasus mountains

Which of these are correct'?

(a)1, 2 and 3

(b)2,3 and 4

(c)1, 2 and 4

(d)1, 3 and 4

[UPSC-2004]

107.Consider the following landform characteristics:

1.Stream divides are sharp and ridge like resulting in a minimum of


interstream uplands.

2.Extensive areas are at or near the base level of erosion.

3.There are a few consequent trunk streams but few large tributaries.
Numerous short tributaries and gullies are extending themselves by head
ward erosion and developing valley systems.
Which one of the following is the orderly above in an idealized fluvial
cycle'?

(a)1, 3, 2

(b)2, 3, 1

(c)3, 1, 2

(d)2, 1, 3

[UPSC-2005]

108.Consider the following minerals:

1.Corudum

2.Quartz

3.Topaz

What is the correct sequence of the above in terms of descending order of


their hardness'?

(a)1, 2, 3

(b)2, 3, 1

(c)1, 3, 2

(d)3, 1, 2

[UPSC-2009]

109.The correct sequence of the following features towards sea from the coast
is
(a)berm, low-tide terrace, longshore trough, longshore bar.

(b)longshore trough, longshore bar, berm, low-tide terrace.

(c)low-tide terrace, berm, longshore bar, longshore trough.

(d)longshore bar, low-tide terrace, berm, longshore trough.

[UGC J-2010]

110.Consider the following statements:

1.The Rhine rift valley stretches for more than 800 km.

2.The floor of Dead sea is more than 1,100 m below the sea level.

Which of the statements given above is/ are correct

(a)1 only

(b)2 only

(c)Both I and 2

(d)Neither 1 nor 2

[UPSC-2009]

111.Consider the following statements:

1.The granite and gneisses give rise to red soil through weathering.

2.The laterite soil contains the hydrated oxides of iron and aluminium

Which of the statements given above is/are correct?

(a)land 2 only
(b)2 only

(c)1 and 2 only

(d)1,2 and 3

[UPSC-2006]

112.Consider the following statements:

1.In an overturned fold, the axial plane is inclined and both limbs dip in the
same direction.

2.A recumbent fold is the one in which the axial plane is essentially
horizontal.

Which of the statements given above is/ are correct'?

(a)1 only

(b)2 only

(c)Both 1 and 2

(d)Neither 1 nor 2

[UPSC-2006]

113.Consider the following statements:

1.Youthful topography is characterized comparatively a few streams but


usually with low gradients.

2.In mature topography, the main streams have the valleys cut to as level
and are at grade.
Which of the statements given above is/ are correct'?

(a)1 only

(b)2 only

(c)Both 1 and 2

(d)Neither 1 nor 2

[UPSC-2006]

114.Consider the following statements:

1.Glacial valley floors are flat and their walls steep; in contrast, the V-
shaped valleys of many mountain rivers.

2.Unlike rivers, valley glaciers at coastlines may erode their valley floors
far deeper than sea level.

Which of the statements given above is/ are correct'?

(a)1 only

(b)2 only

(c)Both 1 and 2

(d)Neither 1 nor 2

[UPSC-2008]

115.Consider the following statements:

1.P-waves travel through a solid rock faster than sound waves travelling
through air.
2.S-waves travel at about half the speed of P-waves.

Which of the statements given above is/are correct?

(a)1 only

(b)2 only

(c)Both 1 and 2

(d)Neither 1 nor 2

[UPSC-2008]

116.Consider the following statements:

1.When feldspar undergoes chemical weathering, kaolinite is produced.

2.Water is essential for the chemical weathering of feldspar.

Which of the statements given above is/ are correct'?

(a)1 only

(b)2 only

(c)Both 1 and 2

(d)Neither 1 nor 2

[UPSC-2008]

117.Consider the following statements:


Which of the statements given above is/ are correct'?

(a)1 only

(b)2 only

(c)Both 1 and 2

(d)Neither 1 nor 2

[UPSC-2008]

118.Consider the following statements:

1.Laolinite is the weathered silicate clay.

2.Kaolinization process occurs due to hydrothermal alternation of feldspar.

Which of the statements given above is/ are correct?

(a)1 only

(b)2 only

(c)Both 1 and 2

(d)Neither 1 nor 2
[UPSC-2009]

119.Consider the following statements:

1.Pyramidal peak is a gentle-sided mountain summit.

2.Tarn is formed often in the cirques.

3.The long axis of drumlins is parallel to the direction of glacial movement.

Which of the statements given above are correct?

(a)1 and 2

(b)2 and 3

(c)1 and 3

(d)1, 2 and 3

[UPSC-2009]

120.Consider the following statements axioms regarding Tidal Hypothesis on


origin of the Earth:

1.The Solar System was formed from the primitive Sun and another
intruding star.

2.The intruding star was smaller in size than the primitive Sun.

3.The primitive Sun was rotating on its axis.

4.The tidal force of intruding star was greater than the primitive Sun.

Which of the axioms given above were presented by Sir James Jeans'?
(a)1, 2 and 3

(b)2, 3 and 4

(c)1, 2 and 4

(d)1, 3 and 4

[UPSC-2005]

121.Consider the following statements in relation to Earth's topmost layer:

1.Transverse earthquake waves move at a speed of 3.5 km/second in that


layer.

2.Specific gravity of Earth is 5.5 g/cm3.

3.It is shallowest under continents and deepest under oceans.

4.Abundance of magnesium

Which of the above statements are correct?

(a)1 and 2

(b)2 and 3

(c)3 and 4

(d)1, 2 and 4

[UPSC-2001]

122.Which of the following statements are correct'?

Larger bodies of intrusive rocks called Batholiths are


1.very commonly formed of granites though not invariably.

2.lens-shaped masses of rocks occupying the saddles of anticlines or the


keels of synclines of mountains.

3.larger dome-shaped masses; the sides of whose plunge steeply to


unknown depths.

(a)1, 2 and 3

(b)1 and 2

(c)2 and 3

(d)1 and 3

[UPSC-2005]

123.Consider the following statements made about the sedimentary rocks:

1.Sedimentary rocks are formed at Earth's surface by the hydrological


system.

2.The formation of sedimentary rocks involves the weathering of


preexisting rocks.

3.Sedimentary rocks contain fossils.

4.Sedimentary rocks typically occur in layers.

Which of these statements are correct?

(a)1 and 2

(b)1 and 4
(c)2, 3 and 4

(d)1, 2, 3 and 4

[UPSC-2001]

124.Consider the following statements regarding earthquakes:

1.In earthquakes, shocks are generated at a point known as the epicentre.

2.The point of origin of shock may be within the crust or mantle.

3.The more earthquake-prone areas are at the margin of tectonic plates.

Which of the above statements are correct'?

(a)1, 2 and 3

(b)1 and 2

(c)1 and 3

(d)2 and 3

[UPSC-2000]

125.Consider the following statements regarding peninsular India:

1.It is largely composed of Archaean crystalline peninsular India.

2.Most of the mountains here are of relict type.

3.The interior parts have post-Cambrian marine deposits

4.Deccan basalts are formed at the end of Cretaceous.

Which of the statements given above correct?


(a)1, 2, 3 and 4

(b)1, 2 and 4

(c)3 and 4

(d)1 and2

[UPSC-2005]

126.Consider the following statements:

1.The Richter scale is a logarithmic scale and so an increase of 1 magnitude


unit represents a factor of 10 times in amplitude.

2.Each integer reading of the Richter scale has energy 100 times that the
previous integer reading.

Which of the statement given above is/ are correct?

(a)1 only

(b)2 only

(c)Both 1 and 2

(d)Neither 1 nor 2

[UPSC-2006]

127.Consider the following statements:

1.A tectonic scarp is a steep slope that results from differential movement of
the Earth's surface.
2.The typical angle of repose for scarps of all origins is between 25° and
40°.

3.A scarp produced by structurally controlled erosion at an ancient fault is


known as fault line scarp.

4.A capable fault is the one that shows movement at or near the ground
surface at least one within the past 35,000 years.

Which of the above statements are correct'?

(a)1, 2, 3 and 4

(b)2, 3 and 4

(c)1 and 3

(d)2 and 3

[UPSC-2001]

128.Consider the following statements:

1.P-waves are the first seismic waves to reach a place.

2.S-waves travel only through liquids.

3.Arrival time of P-and S-waves is used to locate the epicentre.

4.Richter scale is a logarithmic scale.

Which of the statements given above are correct?

(a)1, 2 and 3

(b)1, 3 and 4
(c)1 and 4

(d)3 and 4

[UPSC-2004]

129.Consider the following statements:

`Vulcanicity' refers to

1.all those processes in which molten rock material or magma rises into the
crust.

2.the greater bulk of the volcanic rocks of the Earth's surface that was
erupted from volcanoes.

3.the process of solidification of rock into crystalline or semi-crystalline


form molten rock material after being poured out on the surface.

Which of the statements given above are correct'?

(a)1, 2 and 3

(b)1 and 2

(c)2 and 3

(d)land 3

[UPSC-2005]

130.Consider the following statements:

The circum-Pacific belt is highly prone to the occurrence of earthquakes


because it is a zone of
1.young folded mountains.

2.active volcanoes.

3.divergent plate boundaries.

4.convergent plate boundaries.

Which of the statements given above are correct?

(a)1, 2 and 3

(b)1, 2 and 4

(c)2, 3 and 4

(d)1, 3 and 4

[UPSC-2005]

131.Consider the following statements:

1. Canyon is an extreme type ofshaped valley with very steep sides and no
valley floor.

2.Grand Canyon in associated with the Colorado river.

Which of the statements given above is/ are correct?

(a)1 only

(b)2 only

(c)Both 1 and 2

(d)Neither 1 nor 2
[UPSC-2005]

132.Consider the following statements:

1.Contour ploughing promotes erosion and the formation of gullies.

2.Gully erosion is the removal of top soil and creation of many steep-sided
cuttings in a hill side.

Which of the statements given above is/ are correct?

(a)1 only

(b)2 only

(c)Both 1 and 2

(d)Neither 1 nor 2

[UPSC-2005]

133.Consider the following statements:

1.Weathering encompasses a group of processes disintegrate.

2.Extreme dryness reduces most weathering to a minimum.

3.Physical weathering dominates in drier and cooler climates.

4.In equatorial rainforest climate, most rocks weather slowly.

Which of these are correct?

(a)1, 2 and 3

(b)1 and 4
(c)2, 3 and 4

(d)1, 2, 3 and 4

[UPSC-2002]

134.Continental crust is basically

(a)molten lava.

(b)granite.

(c)solidified iron.

(d)basalt.

135.Cooler, older oceanic lithosphere sinks into the mantle at

(a)sites of long-lived, hot spot volcanism in the ocean basins

(b)transform fault zones along divergent plate boundaries

(c)rift zones along mid-ocean ridges

(d)subduction zones along convergent plate boundaries

136.Dead Sea is situated in which one of the following'?

(a)A rift valley

(b)An intermundane plateau

(c)Intermontane plains

(d)Canyons

[UPSC-2006]
137.Deep ocean trenches are surficial evidence for

(a)rising of hot as thenosphere from deep in the mantle

(b)transform faulting between an oceanic plate and a continental plate

(c)rifting beneath a continental plate and the beginning of continental drift

(d)sinking of oceanic lithosphere into the mantle at a subduction zone

138.Deep-focus earthquakes, those between 300 and 700 kilometres below the
surface, occur only in association with

(a)hot spots

(b)transform faults

(c)subduction zones

(d)mid-ocean ridges

139.Deepoceanic trenches are most abundant around the rim of the ocean
basin.

(a)Arctic

(b)Atlantic

(c)Pacific

(d)Indian

140.Deposition of glacial rock flour from blowing winds is responsible for

(a)blowout pavement
(b)loess deposits

(c)star steppes

(d)deflation ventifacts

141.Desert and steppe lands cover about what percentage of Earth's land area?

(a)30 per cent

(b)66 per cent

(c)10 per cent

(d)3 per cent

142.Desert pavements are the result of

(a)deflation.

(b)abrasion.

(c)both (a) and (b).

(d)neither (a) nor (b).

143.Desertification has been particularly well documented over the past 50


years in

(a)the steppe lands of southern Russia, Ukraine and Kazakhstan.

(b)the Sahel along the southern margin of the Sahara Desert.

(c)the Dust Bowl states of the Great Plains.

(d)the Empty Quarter of the Arabian Peninsula.


144.Diorite is an example of

(a)chemically formed sedimentary rock.

(b)extrusive igneous rock.

(c)plutonic igneous rock.

(d)metamorphic rock.

[UPSC-2009]

145.Down faulted blocks are also known as

1.horst.

2.graben.

(a)Only 1

(b)Only 2

(c)Either 1 or 2

(d)Neither 1 nor 2

146.During a typical sandstorm, saltating sand grains reach a maximum height


of above the land surface.

(a)2 meters

(b)40 feet

(c)10 centimetres

(d)1 inch
147.During wet weather or times when snow is melting, sometimes the
downhill toe of a slump

(a)turns into a rock avalanche.

(b)stops moving because water makes the clay sticky.

(c)becomes an earth flow.

(d)collapses and is deposited as talus at the base of the slope.

148.Early results of the Deep Sea Drilling Project clearly justified the
conclusion that

(a)the youngest sediments were deposited directly on the oldest seafloor


basalts.

(b)the oceans have not always contained most of Earth's water.

(c)Proterozoic rocks are found only as seamounts in the deepest parts of the
ocean basins.

(d)the ocean basins are relatively young; most ocean basin rocks and
sediments are Cretaceous or younger in age.

149.Earthquakes along the Himalayas and the foothills are best explained in
terms of

(a)place tectonics.

(b)foothills are young to the Himalayas.

(c)fragile and largely unconsolidated rocks.

(d)divergence of plates.
[UPSC-2002]

150.Emergent coastlines of Scandinavia (Norway and Sweden) and the


Hudson Bay region of Canada result from which one of the following
combinations'?

(a)The rate of tectonic uplift exceeds the rate of sea level fall.

(b)The rate of glacial rebound is less than the rate of sea level rise.

(c)The rate of glacial rebound exceeds the rate of sea level rise.

(d)The tectonic subsidence rate exceeds the rate of sea level rise.

151.Erosional retreat of aleadse to enlargement and xtension of a wavecut


platform in the inland direction.

(a)wave-cut barrier beach

(b)wave-cut tombolo

(c)offshore, wave-cut, breakwater bar

(d)wave-cut cliff

152.Eskers are of glacio-fluvial origin. Which one of the following statements


characterizes them'?

(a)They are long, narrow and sinuous ridges of sands.

(b)They are small, irregular mounds of bedded sands.

(c)They are small alluvial cones.

(d)They are flat-topped terraces.


[UPSC-2009]

153.European Alps are the result of

(a)oceanic plate and continental plate collision.

(b)oceanic plate and oceanic plate collision.

(c)continental plate and continental plate collision.

(d)none of these.

154.Exogenic process develops

(a)initial landscapes.

(b)sequential landscapes.

(c)both (a) and (b).

(d)neither (a) nor (b).

155.Fiords are very typical of the

(a)Alaskan coast.

(b)Spanish coast.

(c)Norwegian coast.

(d)Peruvian coast.

[UPSC-1996]

156.Folds in which one limb inclines moderately with regular slope while the
other inclines steeply at right angle and the slope is almost vertical are
known as

(a)isoclinals.

(b)monoclonal.

(c)recumbent.

(d)asymmetrical.

[UPSC-2005]

157.Gondwanaland does not include

(a)India.

(b)Vietnam.

(c)China.

(d)Australia.

[UPSC-1996]

158.Hawaii Islands are known f-or active volcanoes because

(a)they are located in the subduction zone of covering plates.

(b)faults and fractures are found there.

(c)they are situated over a hot plume.

(d)they are situated on a mild oceanic ridge with rising convective currents.

[UPSC-2002]

159.Himalayan mountain range is the result of


(a)oceanic plate and continental plate collision.

(b)oceanic plate and oceanic plate collision.

(c)continental plate and continental plate collision.

(d)none of these.

160.How are sand grains transported by the wind'?

(a)By deflation of abraded desert pavement

(b)High in the moving air column as suspended load

(c)By saltation in the first few meters above the land surface

(d)By being picked up in swirling dust clouds and carried to distant


blowouts

161.How are spits, hooks and bay mouth bars formed'?

(a)A headland is eroded and the sand is deposited in an offshore basin.

(b)Wave erosion cuts away both sides of a long sand bar, leaving a sand
ridge aligned parallel to the shore.

(c)Sand eroded from a wave-cut diff is deposited around sea stacks and
arches.

(d)Sand is deposited from longshore currents.

162.How can crashing, collapsing storm waves generate explosive forces and
stresses on rocky outcrops and manmade structures'?
(a)Backwash breaks out blocks of rock or concrete and carries them out to
deeper water.

(b)Pressurized water and compressed air are driven into cracks and fissures.

(c)Oscillating, refractive waves shake the hard materials into small


fragments.

(d)All of the above.

163.How do freezing, thawing, wetting and drying contribute to soil creep'?

(a)Eventually, these cause the soil and regolith to suddenly slide down the
slope.

(b)The soil becomes much weaker when dry and frozen.

(c)Gravity exerts a much stronger force when the soil is wet and thawed.

(d)The soil expands and contracts, lifting particles and dropping them a
slight distance down slope.

164.How do icebergs in the North Atlantic Ocean originate'?

(a)As masses of sea ice that float southwards from the Arctic Ocean

(b)By calving of large piedmont glaciers in Greenland

(c)As calved blocks of glacial ice that float northwards from Antarctica

(d)As large masses of sea ice that float northwards from Antarctica

165.How do the strength and cohesion of clay-rich regolith or soil change with
the addition of water'?
(a)Water increases the strength and cohesion.

(b)Water reduces the strength of clays but raises the cohesion of the soil.

(c)Water does not affect the cohesion but lowers the strength.

(d)Water lowers the strength and cohesion.

166.How is desert pavement formed?

(a)Groundwater in an alluvial fan evaporates, leaving behind a surface layer


of hard-baked mud.

(b)Deflation removes the coarse fragments leaving behind a layer of loess.

(c)Alluvial fans are eroded to form inselbergs with rocky surfaces.

(d)Runoff and deflation carry off the silt and clay, leaving coarser particles
behind.

167.How is desert pavement formed?

(a)Intense chemical weathering removes the sand - and silt-sized particles,


leaving coarse rock debris covering the land surface.

(b)Deflation and sheet wash remove fine-sized materials leaving coarse,


weathered rock fragments concentrated at the surface.

(c)Blowing wind removes fine-size soil particles; coarser particles abrade to


sand size.

(d)Running water deposits gravel and sand over the finersized soil particles.

168.What is the amount of horizontal displacement in a normal fault called'?


(a)Throw

(b)Tear

(c)Heave

(d)Strike

[UPSC-2008]

169.What is the term given to the rectilinear drainage pattern that is formed
where two sets of structural controls occur at right angle'?

(a)Rectangular

(b)Radial

(c)Dendrite

(d)Trellis

[UPSC-2008]

170.Identify the correct order of the process of soilerosion from the following.

(a)Splash erosion, sheet erosion, rill erosion, gully erosion

(b)Sheet erosion, splash erosion, gully erosion, rill erosion

(c)Rill erosion, gully, erosion, sheet erosion, splash erosion

(d)Gully erosion, rill erosion, sheet erosion, splash erosion

[UPSC-2001]

171.In correct order from the centre outwards, Earth includes which units'?
(a)Inner core, crust, mantle, hydrosphere

(b)Core, crust, mantle, hydrosphere

(c)Core, inner mantle, outer mantle, crust

(d)Inner core, outer core, mantle, crust

172.In isostatic rebound,

(a)weight of ice is removed.

(b)magnetic field of the Earth changes.

(c)seafloor is formed.

(d)None of these.

173.In sedimentary rocks, lithification includes

(a)compaction and cementation

(b)cementation and weathering

(c)crystallization and cooling

(d)compaction and transportation

174.In the early part of the 20th century, argued forcefully for continental drift.

(a)Alfred Wegener

(b)Karl Wagner

(c)Peter Rommel
(d)Bill Kohl

175.In waves,

(a)energy is converted into water movement.

(b)energy travels and water itself stays in place.

(c)energy carries the water with itself.

(d)energy stays in place and the water travels.

176.In weathering, frost action happens because

(a)when water freezes, its volume expands.

(b)when snow thaws, its volume expands.

(c)when water freezes, it gains weight.

(d)when snow thaws, it gains weight.

177.In which area would surface water most effectively infiltrate into the local
groundwater system'?

(a)A stream in a steep-sided, bedrock canyon in the mountains

(b)A playa lake with a thick mud bottom

(c)Streams flowing in the numerous channels of an alluvial fan

(d)All of the above would promote infiltration.

178.Inselbergs are

(a)blowouts cut from bedrock in mountainous areas


(b)insulated icebergs floating in a hot spring

(c)lithified rock formed by cementation of wind-deposited, dune sands

(d)bedrock hills in a highly eroded desert landscape

179.Karkatau, the most famous volcano of Indonesia, is located between

(a)Sumatra and Java.

(b)Sumatra and Malaysia.

(c)Java and Borneo.

(d)Bali and Lombok.

[UPSC-1995]

180.Large estuaries are more common on a e.

(a)stable

(b)submergent

(c)retreating

(d)emergent

181.Linear, magnetic patterns associated with mid-ocean ridges are configured


as

(a)normal and reversed magnetized strips roughly perpendicular to the ridge


axis

(b)normal and reversed magnetized strips roughly parallel to the ridge


(c)concentric circles about a rising plume of hot, mantle rocks and magma

(d)reversed magnetizations along the rift valleys and normal magnetizations


along the ridge

182.Loess deposits in the central United States

(a)originated as rock flour in Pleistocene glacial streams and rivers

(b)blew in from the dry areas in the Great Plains and south-western desert
areas

(c)were originally deposited as barchanoid dunes and later redeposited by


glaciers

(d)accumulated from flooding of the Mississippi river

183.Match List I with List II and select the correct answer using the codes
given.

Codes:
[UPSC-2005]

184.Match List I with List II and select the correct answer using the codes
given.

Codes:

[UPSC-2004]

185.Match List I with List II and select the correct answer using the codes
given.
Codes:

[UPSC-2005]
186.Match List I with List II and select the correct answer using the codes
given.

Codes:

[UPSC-2004]

187.Match List I with List II and select the correct answer using the codes
given.

Codes:
[UPSC-2009]

188.Match List I with List II and select the correct answer using the codes
given.

Codes:

[UPSC-1995]

189.Match List I with List II and select the correct answer using the codes
given.
Codes:

[UPSC-1995]

190.Match List I with List II and select the correct answer using the codes
given.

Codes:
[UPSC-1996]

191.Match List I with List II and select the correct answer using the codes
given.

Codes:

[UPSC-1996]

192.Match List I with List II and select the correct answer using the codes
given.
Codes:

[UPSC-1996]

193.Match List I with List II and select the correct answer using the codes
given.
Codes:

[UPSC-1997]

194.Match List I with List II and select the correct answer using the codes
given.

Codes:

[UPSC-1997]

195.Match List I with List II and select the correct answer using the codes
given.
Codes:

[UPSC-1997]

196.Match List I with List II and select the correct answer using the codes
given.

Codes:
[UPSC-1998]

197.Match List I with List II and select the correct answer using the codes
given.

Codes:

[UPSC-1998]

198.Match List I with List II and select the correct answer using the codes
given.
Codes:

[UPSC-1999]

199.Match List I with List II and select the correct answer using the codes
given.

Codes:
[UPSC-1999]

200.Match List I with List II and select the correct answer using the codes
given.

Codes:

[UPSC-1999]

201.Match List I with List II and select the correct answer using the codes
given.

Codes:
[UPSC-1999]

202.Match List I with List II and select the correct answer using the codes
given.

Codes:

[UPSC-2000]
203.Match List I with List II and select the correct answer using the codes
given.

Codes:

[UPSC-2001]
204.Match List I with List II and select the correct answer using the codes
given.

Codes:

[UPSC-2001]

205.Match List I with List II and select the correct answer using the codes
given.
Codes:

[UPSC-2001]

206.Match List I with List II and select the correct answer using the codes
given.

Codes:

[UPSC-2002]

207.Match List I with List II and select the correct answer using the codes
given.
Codes:

[UPSC-2002]

208.Match List I with List II and select the correct answer using the codes
given.
Codes:

[UPSC-2004]

209.Match List I with List II and select the correct answer using the codes
given.
Codes:

[UPSC-2005]

210.Match List I with List II and select the correct answer using the codes
given.
Codes:

[UPSC-2008]

211.Match List I with List II and select the correct answer using the code
given.

Codes:
[UPSC-2006]

212.Match List I with List II and select the correct answer using the code
given.

Codes:

[UPSC-2009]

213.Match List I with List II and select the correct answer using the code
given.
Codes:

[UPSC-2009]

214.Mohodiscontinuity is found between the

(a)crust and mantle.

(b)upper mantle and lower mantle.

(c)mantle and core.


(d)inner core and outer core.

[UPSC-2000]

215.Mohorovic discontinuity separates

(a)core and mantle.

(b)crust and mantle.

(c)inner core and outer core.

(d)sima and nife.

[UPSC-1995]

216.Mohorovicic discontinuity separates

(a)upper mantle and lower crust.

(b)mantle and astonisher.

(c)inner solid core and outer liquid core.

(d)core and mantle.

[UPSC-2005]

217.Moraine type of landforms is associated with

(a)fluvial cycle.

(b)glacial cycle.

(c)karst cycle.

(d)Aeolian cycle.
218.Most dry lands lie between degrees north and south of the equator.

(a) and 5

(b)5 and 10

(c)40 and 50

(d)20 and 30

219.Mount St. Helens and the other Cascade volcanoes are

(a)a row of young, active volcanoes built as western North America moved
over a hot spot deep in the mantle

(b)old, deeply eroded volcanoes built before the Pacific Ocean existed

(c)old, deeply eroded, basaltic volcanoes built when western North


Americawas over the present-day site of the Hawaiian hot spot

(d)young, active volcanoes built on a continental margin above a sinking


slab of oceanic lithosphere

220.New oceanic crust and lithosphere are formed at

(a)convergent boundaries by submarine eruptions and intrusions of basaltic


magma

(b)convergent boundaries by submarine eruptions and intrusions of rhyolitic


magma

(c)divergent boundaries by submarine eruptions and intrusions of basaltic


magma
(d)divergent boundaries by submarine eruptions and intrusions of rhyolitic
magma

221.Niagara Falls is located between which of the following pairs of lakes'?

(a)Lake Superior and Lake Huron

(b)Lake Huron and Lake Erie

(c)Lake Erie and Lake Ontario

(d)Lake Ontario and Lake Huron

[UPSC-2007]

222.Oceanic crust is basically

(a)molten lava.

(b)granite.

(c)solidified iron.

(d)basalt.

223.Of the following, which one would most likely be triggered by an


earthquake'?

(a)Slump

(b)Rock avalanche

(c)Solifluction

(d)Soil creep
224.On anaverage, lithospheric plates are thick.

(a)1 km

(b)10 km

(c)1,000 km

(d)100 km

225.Open folds are those in which the angle between the two limbs of the fold
is

(a)more than 90° but less than 180°.

(b)more than 75° but less than 150°.

(c)more than 45° but less than 90°.

(d)more than 120° but less than 2 10°.

[UPSC-2009]

226.Playas are depressions formed due to

(a)deflating action of winds.

(b)glacial erosion.

(c)river erosion.

(d)marine erosion.

[UPSC-1997]

227.Pre-Cambrian rocks in India are mostly found in the


(a)Aravallis.

(b)Western Ghats.

(c)Gondwanas.

(d)Siwaliks.

[UPSC-2002]

228.Pull-apart, rift zones are generally associated with

(a)a divergent plate boundary

(b)a transform plate boundary

(c)a convergent plate boundary

(d)all plate boundaries

229.Rainshadow deserts are common in

(a)Europe, north of the Alps

(b)vast, dry, steppe lands like the Great Plains

(c)the dry valleys of eastern California and Nevada

(d)north central Africa

230.Residual hill in the desert region is known as

(a)inliers.

(b)inselberg.
(c)playa.

(d)pediment.

[UPSC-2004]

231.Salt marches and mangrove swamps are types of

(a)coral reefs.

(b)coastal wetlands.

(c)beaches.

(d)none of these.

232.Sedimentary rocks are formed when sediments become compressed and


cemented together in a process known as

(a)crystallization.

(b)sedimentation.

(c)solidification.

(d)lithification.

[UPSC-2005]

233.Spits develop due to

(a)circular movement of water currents along the bays.

(b)formation of long shore drifts towards the coast.

(c)formation of large-scale deposits of fine-grained dust.


(d)oblique movement of water currents along the shores.

[UPSC-2006]

234.Swash and backwash describe

(a)the forward and backward water movements as storm waves reflect from
a seawall or groin

(b)the swirling action and sand movements produced when a shallow water
wave impinges on the bottom

(c)the oscillatory movement of water beneath a passing wave

(d)movements of water and sand as waves break along a beach

235.Tertiary period had five epochs. Select the correct order in which these
epochs appeared in the geological time scale form ancient to more recent
times.

(a)Palaeocene, Oligocene, Miocene, Eocene, Pliocene

(b)Palaeocene, Miocene, Oligocene, Eocene, Pliocene

(c)Oligocene, Palaeocene, Miocene, Eocene, Pliocene

(d)Palaeocene, Eocene, Oligocene, Miocene, Pliocene

[UPSC-2003]

236.The division of the geologic time scale is an era of the Phanerozoic eon.

(a)Palaeocene

(b)Palaeozoic
(c)Proterozoic

(d)Permian

237.The forms the relatively cool, brittle plates of plate tectonics.

(a)lithosphere

(b)asthenosphere

(c)astrosphere

(d)eosphere

238.The is/are a logical evolutionary analogue of the African rift valleys


10million years from now.

(a)Ural Mountains

(b)Red Sea

(c)Peru-Chile trench

(d)San Andreas fault

239.The is an example of an active, continent-continent collision.

(a)Arabian Peninsula slamming into North Africa under the Red Sea

(b)northward movement of India into Eurasia

(c)northern movement of Baja California and a sliver of western California


towards the Hawaiian Islands

(d)westward movement of the South American plate over the Nazca plate
240.The is not a part of the Earth's physical environment.

(a)hydrosphere

(b)solid Earth

(c)astrosphere

(d)atmosphere

241. Theist not the name of a Pleisocene glacial episode in North America.

(a)Kansan

(b)Nebraskan

(c)Illinoian

(d)Dakotan

242.The is the thinnest layer of the Earth.

(a)mantle

(b)outer core

(c)inner core

(d)crust

243.The is thought to be a liquid, metallic region in the Earth's interior.

(a)outer core

(b)lithosphere
(c)mantle

(d)inner core

244.The of the geologic time scale represents the time of the most recent `Ice
Age'.

(a)Pliocene epoch

(b)Pleistocene epoch

(c)Pliocene era

(d)Pleistocene era

245. Therefers to the sum total of all life on Earth.

(a)biosphere

(b)atmosphere

(c)hydrosphere

(d)asthenosphere

246 .Thewas the most recent Pleistocene glacial episode in North America.

(a)Wisconsinan

(b)Kansan

(c)Indianan

(d)Dakotan
247. ThewereP formed in the late alaeozoic when sediments in a marine basin
were squeezed and crumpled between converging continents.

(a)Urals in Russia

(b)northern Rockies in Canada and the United States

(c)Alps in Europe

(d)Andes in South America

248.The Aleutian islands occur at a

(a)transform boundary where North America has moved towards Alaska

(b)convergent boundary on a volcanic arc above a northward-subducting


Pacific plate

(c)divergent boundary where shield volcanoes are forming

(d)convergent, continental margin with uplifted fault blocks, much like


those of the Basin and Range Province

249.The arrangement of planets with smallest on either ends and big planet in
the middle supports which one of the following theories of origin of the
solar system?

(a)Big Bang theory

(b)Tidal hypothesis

(c)Binary Star theory

(d)Cepheid theory

[UPSC-2002]
250.The asthenosphere is actually a part of the of the Earth.

(a)crust

(b)inner core

(c)mantle

(d)outer core

251.The bottom or downward tip of a crevasse marks the

(a)boundary between parts of the glacier moving by basal sliding and


crevassal slip

(b)bottom of the basal sliding zone

(c)top of the internal deformation and flowage zone

(d)boundary between the basal sliding and external flowage zones

252.The composition of the core of Earth is thought to be

(a)peridotite

(b)solid iron-nickel alloy

(c)basalt

(d)granite

253.The concept of Ice Age was first put forward by

(a)Louis Agassiz.

(b)Jean de Charpentier.
(c)James Geike.

(d)De Geer.

[UPSC-1996]

254.The concept of uniformitarianism was first purposed by

(a)Harry H.Hess.

(b)Abraham Ortelius.

(c)John Hutton.

(d)Charles Lyell.

255.The continental drift hypothesis was rejected primarily because Alfred


Wegener could not

(a)disprove competing theories that were more accepted by scientists

(b)identify a mechanism capable of moving continents

(c)find geologic similarities on different continents

(d)all of the above

256.The continental shelf is located

(a)seaward of the continental slope

(d)between the continental rise and the abyssal plains

(d)between the continental slope and continental rise

(d)landward of the continental slope


257.The correct sequence (form the oldest to the youngest) of different periods
of tertiary epoch is

(a)Pliocene, Miocene, Eocene, Oligocene.

(b)Eocene, Oligocene, Pliocene, Miocene.

(c)Pliocene, Eocene, Miocene, Pliocene.

(d)Eocene, Oligocene, Miocene, Pliocene.

[UPSC-2000]

258.The depositional feature formed due to the coalescence of several alluvial


cones in an and region is called

(a)pediment.

(b)bolson.

(c)deflation basin.

(d)bajada.

[UPSC-2000]

259.The development of synclinal ridges and anticlinal valleys in a folded


region is named as

(a)parallel belt of valleys and ridges.

(b)abrasion platform.

(c)topography of inversion of relief.

(d)ridge and valley landscape.


[UPSC-2000]

260.The drainage pattern developed on folded sedimentary rocks is termed as

(a)rectangular.

(b)radial.

(c)dendritic.

(d)trellis.

[UPSC-2004]

261.The expression of earliest volcanic activity of India is found in

(a)Dalma hill of Bihar.

(b)northwestern Deccan plateau.

(c)Buldana district of Maharashtra.

(d)Kumaon hill region.

[UPSC-1995]

262.The extensive flood plain with level to gentle slopes (2°-5°) and very low
channel gradient makes the river flow so sluggish that it becomes a tangled
network of interconnected diverging and converging shallow channels. Such
a flood plain is said to be

(a)braided.

(b)graded.

(c)entrenched.
(d)meandered.

[UPSC-2000]

263.The former, late Palaeozoic supercontinent is known as

(a)Pandomonia

(b)Pancakea

(c)Panatopia

(d)Pangaea

264.The glaciated valleys are

(a)U-shaped.

(b)V-shaped.

(c)W-shaped.

(d)Y-shaped.

265.The main difference between paternoster lake and tarn is that paternoster
lake is

(a)formed by glacial action whereas tam is not.

(b)associated with cirque whereas tarn is associated with glacial stairways.

(c)associated with glacial stairways whereas tarn is associated with cirque.

(d)a lake whereas tam is not.

[UPSC-1999]
266.The modern-day Red Sea is explained by plate tectonics theory because it
is

(a)a rare example of a two-continent subduction zone where the African


continental plate is sinking under the Arabian continental plate

(b)a rift zone that may eventually open into a major ocean if Arabia and
Africa continue to separate

(c)the site of a transform fault along which Arabia is moving away from
Africa

(d)a tiny remnant of a once immense ocean that was closed as Africa moved
towards Asia

267.The most abundant gas emitted from volcanoes is

(a)water vapour.

(b)helium.

(c)sulphur dioxide.

(d)carbon dioxide.

[UPSC-2003]

268.The most rapid type of mass movement is a

(a)lahar

(b)slump

(c)rock avalanche

(d)debris flow
269.The most recent geological epoch of the quaternary period covering the
10,000 years or so from the end of the Pleistocene epoch to the present day
is known as

(a)Pliocene.

(b)Miocene.

(c)Oligocene.

(d)Holocene.

[UPSC-2002]

270.The oldest sea floor is situated in

(a)south Pacific west of South America.

(b)western Pacific, near Japan.

(c)mid-Atlantic ridge south of Iceland.

(d)south of Indian ocean.

271.The palaeomagnetic results obtained from India indicate that in the past,
the Indian land mass has moved

(a)northwards.

(b)southwards.

(c)eastwards.

(d)westwards.

[UPSC-1995]
272.The plain formed due to coalescence of series of alluvial fans in the
piedmont zone is known as

(a)pediment.

(b)bajada.

(c)pediplain.

(d)hamada.

[UPSC-2009]

273.The predominant landform of and climate is

(a)alluvial fan.

(b)continuous apron.

(c)bajada.

(d)playa.

274.The process that results in the breakdown of rocks and mineral in situ is
known as

(a)attrition.

(b)erosion.

(c)weathering.

(d)corrosion.

[UPSC-2002]
275.The profile of equilibrium refers to the

(a)profile of a glaciated valley.

(b)transverse profile of a graded stream.

(c)longitudinal profile of a graded stream.

(d)profile along the shoreline.

[UPSC-1995]

276.The relatively stable interior portion of a continent is known as a

(a)craton

(b)shield

(c)belt

(d)platform

277.The rivers of Africa are as large as those of North or South America, but
they do not attract much transport because:

(a)they pass through dense forests.

(b)the people there are not commercial minded.

(c)their courses are broken by cataracts and waterfalls.

(d)they traverse through resource-poor area.

[UPSC-1995]

278.The rocks receiving strain in form of shortening results in


(a)tension fault.

(b)reverse fault.

(c)strike slip fault.

(d)none of these.

279.The strike slip fault is result of stress in the form of

(a)tension on the rock.

(b)compression on the rock.

(c)shear on the rock.

(d)none of these.

280.The temperature below which magnetic material can retain a permanent


magnetization is called the

(a)Darcy temperature

(b)Bullard point

(c)Vine temperature

(d)Curie point

281.The top of the asthenosphere is closest to Earth's surface

(a)above a deep mantle hot spot

(b)along a transform fault

(c)along a subduction zone


(d)along a mid-ocean ridge

282.The Ural mountains were formed in the

(a)Hercynian period.

(b)Permean period.

(c)Tertiary period.

(d)Archaean period.

[UPSC-1998]

283.The volcanoes and deep valleys of east Africa are related to a

(a)continental rift along which parts of the African continent are beginning
to slowly separate

(b)continental collision zone between Africa and the Zagros Mountains


along the southern margin of Eurasia

(c)fault allowing Arabia to slip westwards past east Africa and penetrate
into Turkey

(d)transform fault aligned with the Red Sea carrying the Arabian and
African blocks in opposite directions

284.Thu subtropical and landscapes are dominated by

(a)sand dunes.

(b)desert pavements.

(c)star dunes.
(d)none of these.

285.Today, is in about the same geographic position as during late Palaeozoic


time.

(a)Australia

(b)India

(c)South America

(d)Antarctica

286.Towards which direction does the magnetic North Pole show a declination
form the geographic North Pole'?

(a)East

(b)West

(c)North

(d)South

[UPSC-2005]

287.Transform boundaries occur

(a)where plates slide laterally past one another at right angles to a seafloor.

(b)where up welling material from the mantle forms new seafloor and
lithospheric plates spread apart.

(c)where areas of continental and oceanic lithosphere collide.

(d)None of these.
288.Very long-lived magma source located deep in the mantle is called a

(a)melt well.

(b)magma welt.

(c)hot spot.

(d)basalt spout.

289.Volcanism eruptions do not occur in the

(a)Baltic Sea

(b)Black Sea

(c)Caribbean Sea

(d)Caspian Sea

[UPSC-2001]

290.Water movement and sand transport parallel to the beach are


fundamentally caused by

(a)deep-water waves breaking offshore.

(b)a long fetch parallel to the beach.

(c)strong, offshore winds creating a pileup of water along the beach front.

(d)waves impinging obliquely onto a beach.

291.What are geosynclines'?

(a)Arched unfolds in the strata of the Earth's crust.


(b)Major structural down folds in the Earth's crust.

(c)Large-scale rises in the Earth's crust.

(d)Sliding of plate margins inside the Earth's crust.

[UPSC-2006]

292.What caused the mudflows on the Nevado del Ruiz volcano in 1985'?

(a)Warm weather caused rapid melting of snow and ice near the summit.

(b)A crater lake was broken during an eruption.

(c)Hot ash fell onto snow near the summit.

(d)All of the above.

293.What is seismic zone extending at an angle of about 45° forming the base
of an ocean trench down through lithosphere to the asthenosphere known
as?

(a)Appleton layer

(b)Benioff zone

(c)Conard discontinuity

(d)Convergence zone

[UPSC-2006]

294.What type of moraine is formed by the merging of two lateral moraines at


a junction of two valley glaciers'?

(a)Kettle
(b)Ground

(c)Recessional

(d)Medial

295.What type of moraine would be most useful for tracing diamond-bearing


kimberlite fragments directly to their bedrock source area'?

(a)Terminal, continental ice sheet

(b)Terminal valley or Alpine glacier

(c)Recessional, continental ice sheet

(d)Lateral valley or Alpine glacier

296.When does escarpment develop'?

(a)When a block steps along a fault plane.

(b)When a block moves down vertically along a fault plane

(c)When a block moves horizontally along a fault plane.

(d)When a block moves upwards along a fault plane.

[UPSC-2006]

297.Where is the world's largest ice sheet located today'?

(a)Iceland

(b)Russia, Siberia

(c)Greenland
(d)Antarctica

298.Where is the world's second largest continental ice sheet?

(a)Iceland

(b)Antarctica

(c)Greenland

(d)Siberian Russia

299.Where would you drill to recover samples of the oldest basalts of the
oceanic crust, which are Jurassic in age'?

(a)Mid-Atlantic ridge under Iceland

(b)Crest of the East Pacific, mid-ocean ridge

(c)Oceanic side of the Aleutian trench, along the southern coastline


ofAlaska

(d)Just offshore from the Hawaiian Islands

300.Which disaster was triggered by a huge mass of soil and rock that
suddenly slid into a water-filled reservoir'?

(a)Nevado del Ruiz, Colombia, 1985

(b)Yungay-Ranrahirca, Peru, 1970

(c)Sherman Glacier rock avalanche, AK, 1964

(d)Vaiont Canyon, Italy, 1963

301.Which mass wasting process has the slowest rate of movement'?


(a)Creep

(b)Rock avalanche

(c)Slump

(d)Solifluction

302.Which of the following are driving forces of fluvial systems'?

1.Insolation

2.Gravity

3.Thermal energy

(a)1 and 2

(b)2 and 3

(c)3 and 1

(d)1, 2 and 3

303.Which of the following are examples of peneplains?

1.Central Russia

2.Paris Basin

3.Eastern England

4.Upper Mississippi Basin

Select the correct answer using the codes given.


(a)2 and 3

(b)1, 3 and 4

(c)1, 2 and 4

(d)1, 2, 3 and 4

[UPSC-1996]

304.Which of the following best describes the climatic factors that cause
lowlatitude deserts like the Sahara in Africa'?

(a)Warm, humid air aloft is descending; surface winds blow away from the
equator.

(b)Warm, humid air is rising; surface winds are calm.

(c)Cool, dry air at the surface is rising causing winds to blow away from the
equator.

(d)Cool, dry air aloft is descending; surface winds are blowing towards the
equator.

305.Which of the following best describes the term glacial drift'?

(a)Floating of icebergs southwards from the north polar seas

(b)Slow, plastic flow movement in the brittle zone of a glacier

(c)The slow, southwards advance of the continental ice sheets over Canada
and North America during the Pleistocene

(d)The sedimentary materials outwash and till


306.Which of the following characteristics are associated with great masses of
metamorphic rocks generally resulting from regional metamorphism'?

1.Such rocks become wholly crystalline.

2.Such rocks become partially crystalline.

3.A streaking or parallel arrangement of the constituent crystals which lie


with their longer axes pointing in the same direction.

Select the correct answer using the codes given.

(a)1, 2 and 3

(b)1 and 2

(c)2 and 3

(d)1 and 3

[UPSC-2005]

307.Which of the following characteristics would suggest geologically recent,


fault uplift of a desert mountain range?

(a)Flat, upland surfaces, steep slopes and small alluvial fans

(b)Inselbergs, extensive pediments and flat valley floors

(c)Steep playas with extensive, bedrock alluvial fans and numerous sand
dunes

(d)Extensive pediments and bajadas and small, deep playas

308.Which of the following conditions need to be satisfied for a limestone


landscape to develop into a krast landscape'?
1.The landscape formation must contain 70 per cent or more calcium
carbonate.

2.There must not be an aerated zone between the ground surface and water
table.

(a)Only 1

(b)Only 2

(c)Both 1 and 2

(d)Neither 1 nor 2

309.Which of the following conditions need to be satisfied for a limestone


landscape to develop into a krast landscape'?

1.Complex patterns of joints in the otherwise impermeable limestone must


be present.

2.Vegetation cover must be present.

(a)Only 1

(b)Only 2

(c)Both 1 and 2

(d)Neither 1 nor 2

310.Which of the following correctly describes a fiord (fjord)?

(a)Deep, stream-cut canyon flooded as the coastal lands subsided during the
last lmillion years.
(b)Deep, glacier-cut valley flooded by rising sea level over the last 10,000
years.

(c)Deep, glacier-cut valley flooded as the coastal lands subsided during the
last lmillion years.

(d)Deep, stream-cut canyon flooded by rising sea level over the last 10,000
years.

311.Which of the following depositional landforms are produced by glaciers'?

(a)Roche mountains

(b)Outwash plains

(c)Eskers

(d)U-shaped valley

[UPSC-1997]

312.Which of the following does not produce cliffs?

(a)Compression fault

(b)Reversal fault

(c)Thrust fault

(d)Strike-slip fault

313.Which of the following energy sources is thought to drive the lateral


motions of Earth's lithospheric plates'?

(a)Electrical and magnetic fields localized in the inner core


(b)Gravitational attractive forces of the Sun and the Moon

(c)Export of heat from deep in the mantle to the top of the asthenosphere

(d)Swirling movements of the molten iron particles in the outer core

314.Which of the following factors influence the type and rate of weathering'?

1.Glacier

2.Climate

3.Vegetation cover

4.Rock structure

5.Topography

Select the correct answer using the codes given.

(a)1, 2, 3and 4

(b)2, 3, 4 and 5

(c)1, 2, 3 and 5

(d)1, 2, 4 and 5

[UPSC-2001]

315.Which of the following glacial features would typically be found in close


proximity'?

(a)Cirque and terminal moraine

(b)Drumlin and fjord


(c)Outwash plain and horn

(d)Hanging valley and truncated spur

316.Which of the following is NOT an example of chemical weathering'?

(a)Hydrolysis

(b)Exfoliation

(c)Carbonation

(d)None of these

317.Which of the following is often associated with a cirque basin in high,


mountainous terrain'?

(a)Tarn lake

(b)Loon lake

(c)Kettle pond

(d)Arete pond

318.Which of the following is the correct listing of the North American glacial
stages from older to younger'?

(a)Nebraskan, Indianan, Illinoian, Wisconsinan

(b)Indianan, Kansan, Nebraskan, Ohioan

(c)Nebraskan, Kansan, Illinoian, Wisconsinan

(d)Kansan, Illinoian, Iowan, Dakotan


319.Which of the following is/are the correct characteristics of primary seismic
wave'?

(a)It is a longitudinal and compressional wave.

(b)It is analogous to sound waves.

(c)It travels with faster speed through solids but slowly through liquids.

(d)All the above.

[UPSC-2004]

320.Which of the following mass movements is most likely to occur in a


geologic setting where the rock strata are inclined'?

(a)Debris flow

(b) Slump

(c)Rockslide

(d)Creep

321.Which of the following pairs are correctly matched'?

(a)Karst topography: Blind valley

(b)Humid topography: Stone lattice

(c)Aeolian topography: Ventifacts

(d)Periglacial topography: Pingo

[UPSC-1998]
322.Which of the following pairs are correctly matched?

1.Tienshan: Fold mountain

2.Vosges: Structural dome

3.Ferghana: Deep-seated fault

4.Mauna Loa: Volcanic cone

Select the correct answer using the codes given.

(a)1 and 2

(b)1 and 3

(c)1, 3 and 4

(d)2, 3 and 4

[UPSC-1999]

323.Which of the following pairs are correctly matched'?

1.Granites: Batholiths

2.Marbles: Hogback ridges

3.Quartzite: Ridge crests

4.Hard rocks: Monadnocks

Select the correct answer using the codes given.

(a)1, 2, 3 and 4
(b)1, 3 and 4

(c)2 and 4

(d)1, 2 and 3

[UPSC-1999]

324.Which of the following palaeoclimatic evidence supports the idea of the


late Palaeozoic supercontinent in the Southern Hemisphere?

(a)Thick sediments in the Amazon and Congo deltas of South America and
Africa

(b)Lithified loess (wind-blown) deposits in the deserts of Chile, Australia


and Africa

(c)Tillites (rocks formed by glaciers) in South Africa and South America

(d)Cold water fossils in the deep-water sediments of the South Atlantic


abyssal plain

325.Which of the following statements about glaciers are NOT true'?

1.Most of the glaciers are moving.

2.Worldwide glacial ice is in retreat.

3.Approximately 11 per cent of Earth's land is dominated by them.

(a)1 and2

(b)2 and 3

(c)3 and 1
(d)1, 2 and 3

326.Which of the following statements about the glaciers is not true?

(a)A glacier is an open system with inputs of snow and outputs of ice,
meltwater and water vapour

(b)Snow that survives the summer and into the following winter begins a
slow transformation into glacier ice

(c)In Antarctica glacier ice formation may take 1,000 of years

(d)Greenland ice sheets do not become isostatically depressed

327.Which of the following statements appliesto the asthenosphere, but not the
lithosphere?

(a)Cool, rigid layer of crust and upper mantle that forms the tectonic plates.

(b)Zone in the upper mantle that deforms by plastic flowage.

(c)Deforms mainly by brittle fracturing and faulting.

(d)Partial melting of rising granitic plumes produces huge volumes of


basaltic magma.

328.Which of the following statements are true?

(a)Regolith is the parent rock from which bedrock is formed.

(b)The fractures and separation in the rock are called sediment.

(c)Bedrock is the parent rock from which regolith is formed.

(d)None of these.
329.Which of the following statements are true'?

1.Tombola occurs when the spit grows to completely cut off the bay.

2.Technically, a beach is that place along a coast where sediment is in


motion.

3.The tsunamis in deep ocean water can occasionally reach the speed of 700
km per hour.

(a)1 and 2

(b)2 and 3

(c)3 and 1

(d)1, 2 and 3

330.Which of the following statements are true

1.Almost 75 per cent of Earth's freshwater is frozen.

2.Approximately 25 per cent of Earth's land area is subject to freezing


conditions.

(a)Only 1

(b)Only 2

(c)Both 1 and 2

(d)Neither 1 nor 2

331.Which of the following statements are true'?

1.Moderate earthquakes are caused along constructive plate boundaries.


2.Low-magnitude earthquakes are caused along conservative plate
boundaries.

3.Disastrous earthquakes are caused along destructive plate boundaries

4.Severe earthquakes are caused along the conservative plate boundaries


with the creation of transform faults.

Select the correct answer using the codes given.

(a)1 and 2

(b)1 and 3

(c)1, 3 and 4

(d)2, 3 and 4

[UPSC-1999]

332.Which of the following statements concerning dry lands is not true?

(a)Storms are infrequent and rainfall amounts are highly variable.

(b)Precipitation totals are low; dew points are lower in summers than in
winters.

(c)Wind is the dominant agent of erosion and sediment transport.

(d)Evaporation potential exceeds actual precipitation.

333.Which of the following statements concerning mudflows is not true'?

(a)Mudflows may be caused by heavy rains or melting snow.

(b)Mudflows deposit talus slopes.


(c)Mudflows can move and carry very large boulders and other coarse
debris.

(d)In hilly areas, mudflows move down the canyons and stream valleys.

334.Which of the following statements concerning the Mississippi delta lands


is not true'?

(a)The thick sediment pile is slowly compacting and the delta is slowly
subsiding.

(b)Artificial levees help spread riverborne sediment uniformly over the


delta swamps and wetlands.

(c)The sediment supply has dropped since large reservoirs were constructed
on the Missouri and Arkansas rivers.

(d)Coastal erosion and loss of delta lands will continue as sea level
continues to rise.

335.Which of the following statements holds true'?

(a)Glacial erratics were an early clue that blankets of iceonce covered the
land.

(b)The size-sorted sediments deposited by glacial meltwater are called till.

(c)Eroded debris that is dropped at the glacier's farthest extent is called a


medial moraine.

(d)None of these.

336.Which of the following statements is true?

(a)Glaciers range from 10 to 100 km in size.


(b)Several cirque glaciers may jointly feed a valley glacier.

(c)Glaciers form in areas of permanent snow or permafrost.

(d)A glacier which forms in a cirque is called piedmont glacier.

337.Which of the following was a major factor leading to the Gros Ventre, WY
slide'?

(a)Dry weather caused the clay-rich strata to compact, allowing the


overlying sandstone to slide into the valley.

(b)A strong earthquake caused the unstable rock layers to move.

(c)Soils and shallow bedrock were very wet and locally saturated.

(d)Erosion by the river had undercut the slope-forming shale and exposed
the underlying sandstone.

338.Which one among the following rivers is the longest'?

(a)Amazon

(b)Amur

(c)Congo

(d)Lena

[UPSC-2007]

339.Which one of the following applies to a valley glacier that lengthens


(extends its terminus downslope) over a period of many years'?

(a)Accumulation and wastage are about equal.


(b)Wastage exceeds accumulation.

(c)Accumulation exceeds wastage.

(d)None of the above.

340.Which one of the following coastlines would typically have wave-cut


cliffs, sea stacks, sea arches and wave-cut platforms?

(a)One where unconsolidated sediments are being rapidly eroded as sea


level rises.

(b)One where unconsolidated sediments are being eroded as sea level falls.

(c)One where bedrock is vigorously eroded as sea level rises.

(d)One where a bedrock, wave-cut cliff is rapidly retreating inland as sea


level falls.

341.Which one of the following concerning desert lands is false?

(a)Less than 30 per cent is covered with dunes and drifting sand.

(b)Wind erosion and deposition are important processes.

(c)Most desert areas are characterized by descending wind patterns.

(d)Running water has little effect on shaping the landscape.

342.Which one of the following could not have significantly affected climatic
variations and advances and retreats of ice sheets during the Pleistocene
epoch'?

(a)Variations in the Sun's energy output


(b)Precession of Earth's orbit

(c)Movements of Earth's tectonic plates

(d)Wobbling of Earth's rotational axis

343.Which one of the following is a lowlatitude desert characterized by high


atmospheric pressures and descending air masses (not rain shadow deserts)'?

(a)Sahara Desert; northern Africa

(b)Desert valleys of the Great Basin, USA

(c)Atacama Desert in Chile, South America

(d)Gobi Desert, China and Mongolia

344.Which one of the following is a thirdorder relief?

(a)Abyssal plains

(b)Cliffs

(c)Plateau

(d)Trench

[UPSC-2002]

345.Which one of the following is a true example of median mass?

(a)Hungarian Plain

(b)Indo-Gangetic Plain

(c)Hwangho Plain
(d)Danube Plain

[UPSC-1998]

346.Which one of the following is an artificial coastal feature'?

(a)Sea arch

(b)Barrier island

(c)Sand spit

(d)Breakwater

347.Which one of the following is an important fundamental assumption


underlying the plate tectonic theory?

(a)Earth's diameter has been essentially constant over time.

(b)Radioactive decay slows down at the extreme pressures of the inner core.

(c)Earth's magnetic field originates in the outer core.

(d)Earth's ocean basins are very old and stable features.

348.Which one of the following is determined by the angle of repose for dry
sand'?

(a)Avertical cut bank in loess

(b)Longitudinal dune gradient angle

(c)Slope of an alluvial fan

(d)Inclination angle of a dune slip face


349.Which one of the following is not a landform, which results due to
rejuvenation of a river?

(a)Incised meander

(b)Knickpoint

(c)Oxbow lake

(d)River terrace

[UPSC-2007]

350.Which one of the following is not a soilforming process'?

(a)Weathering

(b)Translocation

(c)Gleying

(d)Hydration

[UPSC-1996]

351.Which one of the following is not directly related to wave refraction'?

(a)Ebb tidal currents carry large quantities of sand from estuaries to the
seacoast side of a barrier island system.

(b)Speeds decrease as waves enter shallow water.

(c)Deposition is concentrated in bays and protected waters.

(d)Erosional energy is focused on headland areas along the beach.


352.Which one of the following is the correct chronological order of the
mountain building'?

(a)Alpine, Caledonian, Hercynian, PreCambrian

(b)Pre-Cambrian, Hercynian, Caledonian, Alpine

(c)Pre-Cambrian, Caledonian, Hercynian, Alpine

(d)Caledonian, Pre-Cambrian, Alpine, Gercynian

[UPSC-2004]

353.Which one of the following is the correct sequence of landforms


developed under fluvial cycle of erosion?

(a)Gorge, flood plain, piedmont plain, delta

(b)Gorge, piedmont plain, flood plain, delta

(c)Gorge, piedmont plain, delta, flood plain

(d)Piedmont plain, gorge, flood plain, delta

[UPSC-1999]

354.Which one of the following is the best measure of the wetness or dryness
of a region?

(a)Difference between annual precipitation and evaporation potential

(b)Total annual precipitation

(c)Percentage of precipitation that falls during the summer months

(d)Mean annual temperature


355.Which one of the following materials has the maximum, sustainable slope
angle, as determined by the angle of repose'?

(a)Moist, clay-rich shale layers

(b)Moist, sandy soil with a clay-rich matrix

(c)Dry sand

(d)Dry silt

356.Which one of the following most accurately describes the volcanoes of the
Hawaiian Islands'?

(a)Shield volcanoes associated with a mid-Pacific ridge and spreading


centre

(b)Stratovolcanoes associated with a mid-Pacific transform fault

(c)Shield volcanoes fed by a longlived hot spot below the Pacific


lithospheric plate

(d)Stratovolcanoes associated with subduction and a convergent plate


boundary

357.Which one of the following operates primarily in areas of permafrost?

(a)Soil creep

(b)Solifluction

(c)Rock avalanche

(d)Mudflow
358.Which one of the following pairs is correctly matched?

(a)A.N. Strahler: Pediplain

(b)W.M. Davis: Peneplain

(c)L.C. King: Panplain

(d)C.H. Crickmay: Slope equilibrium

[UPSC-1998]

359.Which one of the following pairs is not correctly matched?

(a)Himalayas: Tertiary fold mountain

(b)Deccan Trap: Volcanic cone eruption

(c)Western Ghats: Palaeozoic fold mountains

(d)Aravalli: Pre-Cambrian relict mountain

[UPSC-1995]

360.Which one of the following pairs is not correctly matched'?

(a)Jefferson: Primate city

(b)Monte Carlo: Simulation model

(c)Penck: Trepen concept

(d)Strahler: Waxing slope

[UPSC-1995]

361.Which one of the following pairs is not correctly matched'?


Type of waterfall: Name of waterfalls

(a)Caprock fall: Gersoppa

(b)Scarp fall: Dasam

(c)Knickpoint fall: Dhunwadhar

(d)Hanging valley fall: Jahna

[UPSC-2006]

362.Which one of the following pairs is not correctly matched?

(a)01 Doinyo Lengai: Northern Tanzania

(b)Mount Egmont: Hawaii

(c)Volcan Poas: Costa Rica

(d)Mount Klyuchevskaya: Kamchatka Peninsula

[UPSC-2007]

363.Which one of the following periods is largely associated with extinction of


dinosaurs and the increase in flowering plants and reptiles'?

(a)Jurassic

(b)Triassic

(c)Cretaceous

(d)Permian

[UPSC-2005]
364.Which one of the following plate movements is responsible for the
formation of mid-ocean ridge'?

(a)Divergent movement

(b)Convergent movement

(c)Transform fault movement

(d)Parallel movement

[UPSC-2004]

365.Which one of the following prominent landforms, located from the


Wasatch Range, UT, westward to the Sierra Nevada, CA, were formed
during the Pleistocene glacial episodes'?

(a)Old shorelines of large, pluvial lakes

(b)Terminal moraines of continental ice sheets that moved south from


Canada

(c)Erosional features produced by continental glaciation

(d)Subaerial deltas submerged by the post-glacial rise in sea level

366.Which one of the following represents wall-like formation of solidified


magma andis mostly perpendicular to the beds of the sedimentary rock'?

(a)Dyke

(b)Sills

(c)Lalpilli

(d)Breccia
[UPSC-2005]

367.Which one of the following rivers does not represent antecedent drainage'?

(a)Chenab

(b)Sutlej

(c)Ravi

(d)Subansiri

[UPSC-1999]

368.Which one of the following rivers has a cuspate delta?

(a)Mississippi

(b)Niger

(c)Nile

(d)Tiber

[UPSC-2006]

369.Which one of the following samples and experimental measurements


would provide the most information about the Earth's climate over the last
1,00,000 years or so'?

(a)Sediment cores from a Pleistocene loess deposit in the state of


Mississippi; SiO2 content

(b)Cores from sea ice in the Arctic Ocean; NaCl content

(c)Sediment cores from a terminal moraine in Ohio; sand-to-clay ratio


(d)cores from the Greenland ice sheet; ratios of the oxygen isotopes (0-
18/0-16)

370.Which one of the following sets indicates the Aeolian process'?

(a)Corrosion, transportation, deposition

(b)Nivation, down-cutting, solifluction

(c)Corrosion, attrition, deflation

(d)Ablation, deposition, oxidation

[UPSC-1995]

371.Which one of the following sets of lakes is to tectonic origin'?

(a)Baikal, Tanganyika and Rudolf

(b)Chilka, Pulicat and Dal

(c)Ladoga, Onega and Superior

(d)Chand, Biwa and Sambar

[UPSC-1999]

372.Which one of the following types of actions occurs in and topography'?

(a)Deflation, attrition, corrosion

(b)Attrition, corrosion, abrasion

(c)Deflation, abrasion

(d)Deflation, attrition
[UPSC-1998]

373.Which one of the following statements about sand dunes is correct'?

(a)Sand is blown up the slip face and rolls down the more gently sloping
flank of the dune.

(b)The more gently sloping surface is the leeward slope of the dune.

(c)A dune migrates in the direction of inclination of the slip face.

(d)In a sand dune, the more gently inclined strata lie parallel to the slip face.

374.Which one of the following statements concerning glacial deposits is not


true'?

(a)Outwash is mainly stratified sand and gravel.

(b)Glacial erratics are blocks of rock that are too large for the glacier to
move.

(c)Till is deposited directly from the ice; outwash is deposited by meltwater


streams.

(d)Tills are poorly sorted and the fragments are mostly angular.

375.Which one of the following statements concerning glaciers is not true'?

(a)Long, extendedAlpine glaciers occupied valleys in most high


mountainous areas in the USA, Canada and Europe at one or more times
during the past 2 million years.

(b)Ice sheets are larger than ice caps.

(c)Piedmont glaciers form on lowlands at the base of mountainous terrain.


(d)The volume of water tied up in the Antarctic ice sheet is about the same
as the total discharge of the Amazon River in lyear.

376.Which one of the following statements concerning rock weathering is


true'?

(a)Low temperatures and high soil moisture contents accelerate chemical


weathering but inhibit mechanical weathering.

(b)Temperature has no effect on rock weathering.

(c)Warm temperatures and low soil moisture contents both promote rapid
rates of mechanical weathering.

(d)Warm temperatures and high soil moisture contents accelerate chemical


weathering.

377.Which one of the following statements is correct'?

(a)Alluvial fans typically rim desert valleys; playas form in the lowest,
interior parts of the valleys.

(b)Saline sediments and evaporites are common in inselbergs and pediments


of desert landscapes.

(c)Playas are typically covered with gravel-sized desert pavement and loess
deposits.

(d)Inselbergs are low, circular depressions on gently sloping pediments and


bajadas.

378.Which one of the following statements is correct'?

Plateaus are product of


(a)mature stage of cycle of erosion.

(b)youth stage of cycle of erosion.

(c)old stage of cycle of erosion.

(d)rejuvenation of the old mountain.

[UPSC-2005]

379.Which one of the following statements is correct?

Monocline folds are normally associated with

(a)faulting.

(b)distorted beds.

(c)inverted beds.

(d)inclined beds.

[UPSC-2005]

380.Which one of the following statements is not correct'?

(a)Igneous rocks can undergo metamorphism.

(b)Magmas crystallize to form igneous rocks.

(c)Sedimentary rocks may weather to igneous rocks.

(d)Metamorphic rocks may melt to magma.

381.Which one of the following statements is NOT correct?


(a)The largest known flood basalt region in the world is the Deccan trap
areas of peninsular India.

(b)The largest flood basalt region in the USA is the Columbia plateau.

(c)The two shield volcanoes Mauna Loa and Mauna Kea have summit
elevations of more than 6,500 m above sea level.

(d)The island of Hawaii includes at least seven coalesced volcanic domes.

[UPSC-2006]

382.Which one of the following statements is true'?

(a)Desert landscapes are monotonous, relatively flat areas covered to


various depths with sand.

(b)Despite infrequent rainfalls, erosional and depositional features of


running water are important in desert landscapes.

(c)Deserts and dry lands are concentrated in areas of ascending air masses
and relatively low atmospheric pressures.

(d)Rainshadow deserts occur where air masses descend after first having
risen to cross a mountain range.

383.Which of the following statements regarding a fjord are correct'?

1.Most of the fords were formed due to a rise in sea level after the melting
of Pleistocene ice sheets.

2.It is an emergent coast which originally was a submerged glaciated valley.

3.It is a long, narrow inlet of the sea bounded by steep mountain slopes
extending to considerable depths.
4.It is a glaciated valley that has been inundated by the sea and forms a
deep-sided coastal inlet.

Select the correct answer using the codes given.

(a)1, 2, 3 and 4

(b)1, 3 and 4

(c)2 and 4

(d)1, 2 and 3

[UPSC-1999]

384.Which one of the following terms best defines South Asia as a region?

(a)Physiographic

(b)Economic

(c)Geographic

(d)Political

[UPSC-2006]

385.Which one of the following types of spurs is typically associated with


valley glaciers'?

(a)Interlocking

(b)Juxtaposed

(c)Overlapping
(d)Truncated

[UPSC-2008]

386.Which one of the following types of volcanic eruptions is not usually


explosive?

(a)Pelean

(b)Hawaiian

(c)Strombolian

(d)Vulcanian

[UPSC-2007]

387.Which one of the following will effectively limit further deflation in a


given area?

(a)Ahanging valley

(b)Desert pavement

(c)Sea level

(d)The repose level

388.Which one of the following would not be a likely effect of a breakwater?

(a)Increased sand deposition between the beach and the breakwater

(b)Increased, long shore current velocities between the breakwater and the
beach

(c)Increased erosion of the beach on one side of the breakwater


(d)Dissipation of storm wave energy on the seaward side of the breakwater

389.Which one of the following would probably not affect the size and depth
of a blowout'?

(a)The near-surface water table

(b)Areas of desert pavement

(c)Arise in sea level

(d)Type and density of vegetation

390.Which one of the following would prove that a coastline was emergent'?

(a)Extensive barrier islands

(b)Elevated, wave-cut terraces

(c)Many small bedrock islands

(d)Numerous large estuaries

391.Which process occurs whena glacier enters the sea?

(a)Calving

(b)Surging

(c)Kaming

(d)Drowning

392.Which statement best describes slumping, a mass wasting process?

(a)Blocks of hard bedrock rapidly slide downhill along fracture surfaces.


(b)A mass of soil or regolith becomes saturated with water and suddenly
flows downhill to the base of the slope.

(c)The soil and regolith move downhill very slowly.

(d)A block or blocks of unconsolidated regolith slide downhill along a


curved slip surface.

393.Why is so much basaltic magma erupted along mid-ocean ridges'?

(a)Lowered pressures decrease the temperatures at which basalt magma can


partially melt from a rising plume of mantle peridotite.

(b)Basaltic mantle under the ridge is hot enough to completely melt if


seawater is added.

(c)The subducting, oceanic slab sinks so deep that eventually it melts,


producing massive quantities of basalt magma.

(d)The mantle beneath the ridges is enriched in thorium, uranium and


potassium, causing strong heating due to energy from radioactive decay.

394.Why was the Alaskan pipeline built above ground and insulated'?

(a)To protect the pipeline against damage due to thawing ground and to
keep the crude oil hot and fluid

(b)To protect against damage from roots of large trees uprooted in


windstorms

(c)To keep the crude oil cool and highly fluid

(d)To protect against damage from herds of elk and deer

395.Willy-Willy is
(a)a type of tree grown in temperate regions.

(b)a wind that blows in a desert.

(c)a tropical syncline of the north-west Australia.

(d)a kind of common fish found near Lakshadweep islands.

[UPSC-1995]

396.With reference to earthquakes, which one of the following statements is


NOT correct'?

(a)L waves travel along the surface of the Earth's crust.

(b)As the waves propagate, they move material in a path parallel to the
direction of movement.

(c)S waves objects at right angles to their direction of motion.

(d)S waves propagate even through liquid material.

[UPSC-2006]

397.Nazca Plate lies

(a)west of South America.

(b)east of South America.

(c)east of North America.

(d)west of North America.

[UGCD-2011]
398.A narrow block elevated between two normal faults is called

(a)graben.

(b)horst.

(c)monocline.

(d)anticline.

[UGCD-2011]

399.Match List I with List II and select the correct answer from the codes
given.

Codes:

[UGC D-2011]

400.Given below are two statements, one labelled as Assertion (A) and the
other labelled as Reason (R). Select your answer from the codes given.
Assertion (A): Spheroidal weathering is the result of chemical weathering.

Reason (R): Spheroidal weathering occurs when water penetrates joints and
dissolves the rock cementing material.

(a)Both (A) and (R) are true and (R) is correct explanation of (A).

(b)Both (A) and (R) are true, but (R) is not correct explanation of (A).

(c)(A) is true, but (R) is false.

(d)(A) is false, but (R) is true.

[UGC D-2011]

401.Given below are two statements, one labelled as Assertion (A) and the
other labelled as Reason (R). Select your answer from the codes given.

Assertion (A): The islands along the mid-Atlantic ridge are highly prone to
volcanic activity.

Reason (R): The sea floor spreading is the main cause of earthquakes in the
Atlantic Ocean.

(a)Both (A) and (R) are true and (R) is correct explanation of (A).

(b)Both (A) and (R) are true, but (R) is not correct explanation of (A).

(c)(A) is true, but (R) is false.

(d)(A) is false, but (R) is true.

[UGC D-2011]

402.The `firn' is associated with


(a)fluvial deposition.

(b)glacial compaction and recrystallization.

(c)wind accumulation.

(d)coastal accumulation.

[UGCJ-2011]

403.The `Moho' discontinuity is the surface of demarcation between

(a)crust and mantle.

(b)mantle and core.

(c)mantle and outer core.

(d)outer core and inner core.

[UGC J-2011]

404.Which one of the following rivers does not represent antecedent drainage?

(a)Chenab

(b)Sutlej

(c)Ravi

(d)Subansiri

[UPSC-1999]

405.The climatic conditions under which erosion is most rapid are

(a)hot and moist.


(b)cold and dry.

(c)hot and dry.

(d)cold and moist.

[UGC D-2005]

406.Obsequent streams generally flow

(a)along the consequent stream.

(b)opposite to the consequent stream.

(c)right angle of the consequent stream.

(d)diagonal to the stream.

[UGC D-2005]

407.Which is irrelevant with the rock conglomerate?

(a)Transportation

(b)Fluvial

(c)Metamorphism

(d)Consolidation

[UGC D-2005]

408.Arrange the following major soil types in ascending order in terms of the
thickness of A horizon in each of them.

1.Alluvial soil
2.Chestnut soil

3.Chernozem soil

4.Laterite soil

(a)3, 1, 2, 4

(b)3, 2, 4, 1

(c)3, 2, 1, 4

(d)3, 4, 1, 2

[UGC D-2005]

409.Given below are two statements, one labelled as Assertion (A) and the
other labelled as Reason (R). Select your answer from the codes given.

Assertion (A): The lithosphere is distinguished from the asthenosphere


beneath it by a differencein their behaviour under stress.

Reason (R): The lithosphere is rigid whereas the asthenosphere is capable of


plastic flow.

(a)Both (A) and (R) are true and (R) is the correct explanation of (A).

(b)Both (A) and (R) are true but (R) is not the correct explanation of (A).

(c)(A) is true but (R) is false.

(d)(A) is false but (R) is true.

[UGC D-2005]
410.The North Pole lies within the Arctic Ocean whereas the South Pole is
located within the Antarctic continent. Which of the following theory uses
this phenomenon as a supportive argument'?

(a)Theory of Isostasy

(b)Theory of Plate Tectonics

(c)Convective Current Theory

(d)Tetrahedral Theory

[UGC D-2006]

411.Given below are two statements, one labelled as Assertion (A) and the
other labelled as Reason (R). Select your answer from the codes given.

Assertion (A): The ultimate source of energy that makes possible the
erosion of landscapes is Sun.

Reason (R): Solar energy evaporates surface water, some of which


subsequently falls as rain and snow on high grounds.

(a)Both (A) and (R) are true and (R) is the correct explanation of (A).

(b)Both (A) and (R) are true and (R) is not a correct explanation of (A).

(c)(A) is true but (R) is false.

(d)(A) is false but (R) is true.

[UGC D-2006]

412.Given below are two statements, one labelled as Assertion (A) and the
other labelled as Reason (R). Select your answer from the codes given.
Assertion (A): The magnitude of an earthquake is greatest near the epicentre
and decreases with distance.

Reason (R): The energy released by an earthquake can be established from


its magnitude.

(a)Both (A) and (R) are true and (R) is the correct explanation of (A).

(b)Both (A) and (R) are true and (R) is not a correct explanation of (A).

(c)(A) is true but (R) is false.

(d)(A) is false but (R) is true.

[UGC D-2006]

413.Which one of the following forces is responsible for syncline formation'?

(a)Crustal faulting

(b)Crustal rifting

(c)Crustal compression

(d)Crustal sinking

[UGC D-2007]

414.Pediplains and inselbergs are features of the old stage of which cycle of
erosion'?

(a)Glacial

(b)Karst

(c)Arid
(d)Marine

[UGC D-2007]

415.Which of the following terms does not indicate similar processes'?

(a)Diastrophism

(b)Folding

(c)Exfoliation

(d)Warping

[UGC D-2007]

416.In which of the following the direction of flow of streams is controlled by


rock structure'?

(a)Consequent

(b)Subsequent

(c)Insequent

(d)Obsequent

[UGC D-2007]

417.Match List I with List II and mark the correct answer using the codes
given.
Codes:

[UGC D-2007]

418.What is the point of origin of an earthquake called?

(a)Quake centre

(b)Epicentre

(c)Seismic focus

(d)Tectonic point

[UGC D-2007]

419.The process of weathering refers to

(a)lateral expansion of valleys.

(b)spread of continentalshelf.

(c)breakdown of rock materials.


(d)building of rock-cut platform.

[UGC D-2008]

420.Stalactites and stalagmites are depositional landscape of

(a)glaciated region.

(b)limestone region.

(c)coastal topography.

(d)fluvial topography.

[UGC D-2008]

421.The correct sequence of landforms evolved during the cycle of fluvial


erosion is

(a)rapids, waterfalls, alluvial fans, peneplain.

(b)waterfalls, rapids, peneplain, alluvial fans.

(c)peneplain, alluvial fans, rapids, waterfalls.

(d)waterfalls, peneplain, alluvial fans, rapids.

[UGC D-2008]

422.Given below are two statements, one labelled as Assertion (A) and the
other labelled as Reason (R). Select your answer from the codes given.

Assertion (A): In and and semi-arid regions, sheet flood is an important land
forming process.
Reason (R): Arid and semi-arid regions experience high-intensity, short-
duration rainfall.

(a)Both (A) and (R) are true and (R) is correct explanation of (A).

(b)Both (A) and (R) are true, but (R) is not correct explanation of (A).

(c)(A) is true but (R) is false.

(d)(R) is true but, (A) is false.

[UGC D-2008]

423.The soils which originate in the humid region due to the accumulation of
large amounts of organic matter are known as

(a)desert soils.

(b)peat soils.

(c)red soils.

(d)laterite soils.

[UGC D-2008]

424.When endogenetic forces act horizontally from the opposite directions


causing compression and tension, the resultant landform is a large dome-
shaped structure, the process is called

(a)folding.

(b)faulting.

(c)fracturing.
(d)warping.

[UGC D-2009]

425.Which one of the following explanations of the origin of pediments was


first propounded by A.C.Lawson (1915)?

(a)Sheet flood deposition

(b)Horizontal erosion

(c)Recession of mountain front

(d)Lateral erosion

[UGC D-2009]

426.Given below are two statements, one labelled as Assertion (A) and the
other labelled as Reason (R). Select your answer from the codes given.

Assertion (A): By far, the most common and widespread topographic form
in a Karst terrain is the sinkhole.

Reason (R): Topographically, a sinkhole is a depression that varies in depth


from less than a metre to few hundred metres.

(a)Both (A) and (R) are true and (R) is the correct explanation of (A).

(b)Both (A) and (R) are true, but (R) is not the correct explanation of (A).

(c)(A) is true but (R) is false.

(d)(A) is false but (R) is true.

[UGC D-2009]
427.Match List 1 with List II and select the correct answer from the codes
given.

Codes:

[UGC D-2009]

428.Arrange the following concepts in the order in which they appeared. Use
the codes given.

1.Principle of Uniformitarianism

2.Theory of Continental Drift.

3.Dynamic Equilibrium Theory

4.Convection Current Theory

(a)1,3,4,2
(b)2, 1, 4, 3

(c)4, 1, 2, 3

(d)1, 2, 4, 3

[UGC D-2009]

429.Given below are two statements, one labelled as Assertion (A) and the
other labelled as Reason (R). Select your answer from the codes given.

Assertion (A): In temperate regions, the grasslands have been converted into
fertile agricultural lands at places.

Reason (R): Low temperatures in these lands make the rainfall more
effective.

(a)Both (A) and (R) are correct and (R) explains (A).

(b)Both (A) and (R) are correct but (R) does not explain (A).

(c)(A) is true but (R) is false.

(d)(A) is false but (R) is true.

[UGC D-2009]

430.Which one of the following is an artificial coastal feature'?

(a)Sea arch

(b)Barrier island

(c)Sand spit

(d)Breakwater
431.Stone lattices are produced by

(a)wind action.

(b)water action.

(c)wave action.

(d)glacial action.

[UGC J-2005]

432.Eskers are

(a)a Tundra tribe.

(b)glacial landforms.

(c)a type of and vegetation.

(d)a type of animal found in the tropical forests.

[UGC J-2005]

433.Given below are two statements, one labelled as Assertion (A) and the
other labelled as Reason (R). Select your answer from the codes as given.

Assertion (A): Undercutting of base of soil or rock leads to headward


erosion in a stream.

Reason (R): Undercutting of channels is caused by underground percolation.

(a)Both (A) and (R) are true and (R) is the correct explanation of (A).

(b)Both (A) and (R) are true and (R) is not the correct explanation of (A).
(c)(A) is true but (R) is false.

(d)(A) is false but (R) is true.

[UGC J-2005]

434.Whaleback dunes are characteristic features of

(a)Thar desert.

(b)Gobi desert.

(c)Australian desert.

(d)Sahara desert.

[UGC J-2007]

435.Given below are two statements, one labelled as Assertion (A) and the
other labelled as Reason (R). Select your answer from the codes given.

Assertion (A): In and and semi-arid regions, sheet flood is an important land
forming process.

Reason (R): Arid and semi-arid regions experience high-intensity, short-


durationrainfall.

(a)Both (A) and (R) are true, and (R) is the correct explanation of (A).

(b)Both (A) and (R) are true, but (R) is not the correct explanation of (A).

(c)(A) is true and (R) is false.

(d)(R) is true but (A) is false.

[UGC J-2007]
436.Given below are two statements, one labelled as Assertion (A) and the
other labelled as Reason (R). Select your answer from the codes given.

Assertion (A): Glacial troughs often exhibit remarkably flat floors.

Reason (R): The flat floor in a glacial trough is produced by uniform glacial
erosion.

(a)Both (A) and (R) are true, and (R) is the correct explanation of (A).

(b)Both (A) and (R) are true, but (R) is not the correct explanation of (A).

(c)(A) is false but (R) is true.

(d)(R) is true but (A) is false.

[UGC J-2007]

437.Match List with List II and select the correct answer from the codes given.

Codes:
[UGC J-2007]

438.Who propounded the concept of dynamic equilibrium in geomorphology?

(a)Davis

(b)King

(c)Hack

(d)Crickmay

[UGC J-2007]

439.Obsequent streams generally flow

(a)along the consequent stream.

(b)opposite to the consequent stream.

(c)at right angle to the consequent stream.

(d)diagonal to the consequent stream.

[UGC J-2007]

440.The principle of uniformitarianism in geomorphology was first advocated


by

(a)Davis.
(b)Playfair.

(c)Gilbert.

(d)Hutton.

[UGC J-2007]

441.V-shaped valley is formed during the

(a)mature stage of glacial erosion.

(b)old age of fluvial cycle of erosion.

(c)youthful stage of fluvial cycle of erosion.

(d)old age of and cycle.

[UGC J-2008]

442.The term `Pangea' was coined by

(a)Alfred Wegener.

(b)A.N. Strahler.

(c)Sharp.

(d)W.M. Smith.

[UGC J-2008]

443.The concept `landscape is a function of structure, process and stage' was


evolved by

(a)Griffith Taylor
(b)Davis

(c)Penck

(d)Thornbury

[UGC J-2008]

444.Given below are two statements, one labelled as Assertion (A) and the
other labelled as Reason (R), select your answer from the codes given.

Assertion (A): The river valleys are usually narrow shaped in the youthful
stage.

Reason (R): Pothole drilling is a dominant erosional process responsible for


down cutting of river valleys.

(a)Both (A) and (R) are true and (R) is the correct explanation of (A).

(b)Both (A) and (R) are true but (R) is not the correct explanation of (A).

(c)(A) is true, but (R) is false.

(d)(A) is false, but (R) is true.

[UGC J-2008]

445.The statement `The present is the key to the past' was given by

(a)G.K. Gilbert.

(b)James Hutton.

(c)W.H. Davis.

(d)Walter Penk.
[UGC J-2010]

446.Which process of chemical weathering causes rusting of iron?

(a)Carbonation

(b)Oxidation

(c)Hydration

(d)Dissilication

[UGC J-2010]

447.Sea floor spreading theory was propounded by

(a)Harry Hess.

(b)Tuzo Wilson.

(c)A. Hobbes.

(d)D.L. Holms.

[UGC J-2010]

448.Match List I with List II and select correct answer from the codes given.
Codes:

[UGC J-2010]

ANSWER KEYS
 
1.refers to the broad dome of water moving with the eye and frontal portion of
a hurricane.

(a)Sea dome

(b)Eyewall ridge

(c)Storm surge

(d)Cyclonic mound

2.A cold local wind draining occasionally down the Rhone valley is known as

(a)chinook.

(b)mistral.

(c)bora.

(d)blizzard.

[UPSC-1999]

3.A general term for particulate matter found in the atmosphere is

(a)albedo

(b)aerosols

(c)proxy data

(d)pollen
4.A region with summer temperature of 20°C, winter temperature of 15°C and
annual rainfall of about 50cmmainly in spring and early summer with
coniferous vegetation would lie in the

(a)cool temperate marginal-type environment.

(b)Prairie-type environment.

(c)cool temperate central or Siberiantype environment.

(d)Manchurian-type environment.

[UPSC-1998]

5.A relatively small rotating funnel that extends downwards from the base of a
towering cloud with thunder, lighting and strong busty wind is known as

(a)hurricane.

(b)tornado.

(c)tropical cyclone.

(d)extra tropical cyclone.

[UPSC-2003]

6.A surface'sis the amount of total radiation that it reflects.

(a)albedo

(b)isotope ratio

(c)lapse rate

(d)greenhouse effect
7.A temperature inversion is most likely to occur under which of the following
conditions'?

(a)Mountain top in late evening

(b)Windy but cloud night

(c)Calm, cloudy and humid night

(d)Calm, clear and cool winter night

[UPSC-2004]

8.According to Koppen, the best expression of overall climate was

(a)the natural animal kingdom.

(b)the soil which the place had

(c)the temperature difference.

(d)the natural vegetation.

9.According to Koppen's classification, Brazil can be classified as

(a)Af, Am.

(b)Aw.

(c)Cw.

(d)BWh.

10.According to polar front theory of cyclone development, the cyclone is


fully developed in stage
(a)I.

(b)II.

(c)III.

(d)IV.

11.Aerosols are most numerous in

(a)troposphere.

(b)thermosphere.

(c)mesosphere.

(d)stratosphere.

12.All other factor being equal cold air will

(a)be denser than warm air.

(b)be less dense compared to warm air.

(c)will be as dense as warm air.

(d)Cannot be scientifically proven

13.Altocumulus clouds are NOT

(a)known as sheep clouds.

(b)known as wool-pack clouds.

(c)characterized by wavy layers of globular form.


(d)generally dark grey in colour.

14.Altostratus clouds

(a)are generally white in colour.

(b)do not form halos around the Sun and the Moon.

(c)are transparent.

(d)are associated with dry weather.

15.An observer in the Northern Hemisphere who stands with his back to the
wind will have lower pressure to his left and higher pressure to his right.
This is known by which one of the following?

(a)Bergeron-Findeisen theory

(b)Buys Ballot's law

(c)Walker circulation

(d)Stoke's law

[UPSC-2008]

16.Arctic Sea smoke is

(a)a result of pollution.

(b)steam fog.

(c)advection fog.

(d)frontal fog.
17.Areas near oceans have than areas in the interior of continents because of
the great storage capacity of water.

(a)warmer winters, warmer summers

(b)cooler winters, cooler summers

(c)cooler winters, warmer summers

(d)warmer winters, cooler summers

18.As you ascend in elevation within the troposphere, the air temperature
decreases by

(a)5.3 °C/km

(b)3.5 °C/km

(c)6.5 °C/km

(d)4.2 °C/km

19.Assertion (A): Hot deserts in the trade wind belts occur invariably on the
western side of the continents.

Reason (R): In the tropical belt where the trade winds blow, there are
practically no highlands on the western side to obstruct the moisture-laden
winds for causing rainfall.

(a)Both (A) and (R) are true and (R) is the correct explanation of (A).

(b)Both (A) and (R) are true but (R) is not the correct explanation of (A).

(c)(A) is true but (R) is false.


(d)(A) is false but (R) is true.

[UPSC-2000]

20.Assertion (A): Polar areas have high atmospheric pressure.

Reason (R): These areas receive minimum sunshine throughout the year.

(a)Both (A) and (R) are true and (R) is the correct explanation of (A).

(b)Both (A) and (R) are true but (R) is not the correct explanation of (A).

(c)(A) is true but (R) is false.

(d)(A) is false but (R) is true.

[UPSC-2002]

21.Assertion (A): Seas near the Equator receive rainfall throughout the year.

Reason (R): High temperature and high humidity near the Equator cause
convectional rain in most afternoons.

(a)Both (A) and (R) are true and (R) is the correct explanation of (A).

(b)Both (A) and (R) are true but (R) is not the correct explanation of (A).

(c)(A) is true but (R) is false.

(d)(A) is false but (R) is true.

[UPSC-1999]

22.Assertion (A): 60° to 65° latitudes in both the hemispheres have a low
pressure.
Reason (R): The low-pressure areas are permanent over oceans rather than
on land.

(a)Both (A) and (R) are true and (R) is the correct explanation of (A).

(b)Both (A) and (R) are true but (R) is not the correct explanation of (A).

(c)(A) is true but (R) is false.

(d)(A) is false but (R) is true.

[UPSC-2002]

23.Assertion (A): A person feeling the air in thermosphere will get his hand
burned due to high temperature.

Reason (R): The thermosphere absorbs very shortwave, high-energy solar


radiations.

(a)Both (A) and (R) are true and (R) is the correct explanation of (A).

(b)Both (A) and (R) are true but (R) is not the correct explanation of (A).

(c)(A) is true but (R) is false.

(d)(A) is false but (R) is true.

24.Assertion (A): Airplanes collect ice when they pass through liquid clouds.

Reason (R): Supercooled water will readily freeze if it impacts an object.

(a)Both (A) and (R) are true and (R) is the correct explanation of (A).

(b)Both (A) and (R) are true but (R) is not the correct explanation of (A).

(c)(A) is true but (R) is false.


(d)(A) is false but (R) is true.

25.Assertion (A): Anti-cyclonic areas are ideal for the formation of air masses.

Reason (R): Convergence of air happens in the anti-cyclonic areas.

(a)Both (A) and (R) are true and (R) is the correct explanation of (A).

(b)Both (A) and (R) are true but (R) is not the correct explanation of (A).

(c)(A) is true but (R) is false.

(d)(A) is false but (R) is true.

26.Assertion (A): Apart from large-scale climatic aberrations in the Tropics, El


Nino has a disastrous consequence on the fishing industry of Peru.

Reason (R): El Nino leads to massive displacements of the rainfall bringing


droughts over large parts of the normal raining areas and torrential rains to
and regions of the Tropics.

(a)Both (A) and (R) are true and (R) is the correct explanation of (A).

(b)Both (A) and (R) are true but (R) is not the correct explanation of (A).

(c)(A) is true but (R) is false.

(d)(A) is false but (R) is true.

[UPSC-2002]

27.Assertion (A): Baltic Sea remains open for international trade throughout
the year.

Reason (R): Baltic Sea lies in the warm temperature zone.


(a)Both (A) and (R) are true and (R) is the correct explanation of (A).

(b)Both (A) and (R) are true but (R) is not the correct explanation of (A).

(c)(A) is true but (R) is false.

(d)(A) is false but (R) is true.

[UPSC-2000]

28.Assertion (A): Beyond the upper limit of the ionosphere, the concentration
of ions is low.

Reason (R): Beyond the upper limit of ionosphere, the density of the air is
extremely low.

(a)Both (A) and (R) are true and (R) is the correct explanation of (A).

(b)Both (A) and (R) are true but (R) is not the correct explanation of (A).

(c)(A) is true but (R) is false.

(d)(A) is false but (R) is true.

29.Assertion (A): Cirrus clouds primarily contain ice particles.

Reason (R): Cirrus clouds occur at very high altitudes.

(a)Both (A) and (R) are true and (R) is the correct explanation of (A).

(b)Both (A) and (R) are true but (R) is not the correct explanation of (A).

(c)(A) is true but (R) is false.

(d)(A) is false but (R) is true.


[UPSC-2006]

30.Assertion (A): Convergence of air helps in the formation of air masses.

Reason (R): Divergence of air flow causes air to stay over the region for a
longer period of time.

(a)Both (A) and (R) are true and (R) is the correct explanation of (A).

(b)Both (A) and (R) are true but (R) is not the correct explanation of (A).

(c)(A) is true but (R) is false.

(d)(A) is false but (R) is true.

31.Assertion (A): Convergence of two contrasting air masses is a prerequisite


condition for frontogenesis.

Reason (R): When two thermally contrasting air masses meet, they try to
penetrate into the region of one another.

(a)Both (A) and (R) are true and (R) is the correct explanation of (A).

(b)Both (A) and (R) are true but (R) is not the correct explanation of (A).

(c)(A) is true but (R) is false.

(d)(A) is false but (R) is true.

32.Assertion (A): Cyclone is developed in a region of extremely low pressure


of air with steep gradient of pressure.

Reason (R): It has outward spiralling air at low elevations and inward air
flowing at the high elevations.

(a)Both (A) and (R) are true and (R) is the correct explanation of (A).
(b)Both (A) and (R) are true but (R) is not the correct explanation of (A).

(c)(A) is true but (R) is false.

(d)(A) is false but (R) is true.

[UPSC-2003]

33.Assertion (A): Easterlies are stronger in the Northern Hemisphere than in


the Southern Hemisphere.

Reason (R): The Northern Hemisphere has more land area than the Southern
Hemisphere.

(a)Both (A) and (R) are true and (R) is the correct explanation of (A).

(b)Both (A) and (R) are true but (R) is not the correct explanation of (A).

(c)(A) is true but (R) is false.

(d)(A) is false but (R) is true.

[UPSC-2002]

34.Assertion (A): Equatorial region is an area of sluggish development.

Reason (R): Extreme climate conditions prevail here.

(a)Both (A) and (R) are true and (R) is the correct explanation of (A).

(b)Both (A) and (R) are true but (R) is not the correct explanation of (A).

(c)(A) is true but (R) is false.

(d)(A) is false but (R) is true.


[UPSC-1999]

35.Assertion (A): Generally, Delhi receives around 700 mm of rain and Shimla
receives 1,520mm.

Reason (R): In convectional rainfall, the areas near the mountains receive
more rain than the areas further from it.

(a)Both (A) and (R) are true and (R) is the correct explanation of (A).

(b)Both (A) and (R) are true but (R) is not the correct explanation of (A).

(c)(A) is true but (R) is false.

(d)(A) is false but (R) is true.

36.Assertion (A): Generally, temperature decreases form the Equator towards


the poles which is temperature gradient. But the highest temperature is
never recorded at the Equator; instead it is recorded near both the tropics.

Reason (R): A sizeable portion of the incoming solar radiation is reflected


by clouds and a large part of heat is spent in the process of evaporation.

(a)Both (A) and (R) are true and (R) is the correct explanation of (A).

(b)Both (A) and (R) are true but (R) is not the correct explanation of (A).

(c)(A) is true but (R) is false.

(d)(A) is false but (R) is true.

[UPSC-2004]

37.Assertion (A): Greenland and Antarctica regions are permanent centres of


thermal anti-cyclones.
Reason (R): The high pressure is the common feature of Greenland and
Antarctica because of the higher latitude and free radiation.

(a)Both (A) and (R) are true and (R) is the correct explanation of (A).

(b)Both (A) and (R) are true but (R) is not the correct explanation of (A).

(c)(A) is true but (R) is false.

(d)(A) is false but (R) is true.

[UPSC-2007]

38.Assertion (A): Hurricanes do not originate in South Atlantic and south-


eastern Pacific Ocean.

Reason (R): In South Atlantic and southeastern seldom occurs.

(a)Both (A) and (R) are true and (R) is the correct explanation of (A).

(b)Both (A) and (R) are true but (R) is not the correct explanation of (A).

(c)(A) is true but (R) is false.

(d)(A) is false but (R) is true.

[UPSC-2007]

39.Assertion (A): In temperate cyclones, wind blows from the periphery


towards the centre.

Reason (R): They have high-pressure centres.

(a)Both (A) and (R) are true and (R) is the correct explanation of (A).

(b)Both (A) and (R) are true but (R) is not the correct explanation of (A).
(c)(A) is true but (R) is false.

(d)(A) is false but (R) is true.

40.Assertion (A): Mangalore receives more rain than Bangalore.

Reason (R): Windward slope receives more rain than the rain shadow zone.

(a)Both (A) and (R) are true and (R) is the correct explanation of (A).

(b)Both (A) and (R) are true but (R) is not the correct explanation of (A).

(c)(A) is true but (R) is false.

(d)(A) is false but (R) is true.

41.Assertion (A): Ozone is formed in stratosphere.

Reason (R): Temperature is very high in this layer.

(a)Both (A) and (R) are true and (R) is the correct explanation of (A).

(b)Both (A) and (R) are true but (R) is not the correct explanation of (A).

(c)(A) is true but (R) is false.

(d)(A) is false but (R) is true.

42.Assertion (A): Rajasthan receives very low amount of rainfall.

Reason (R): Rajasthan is a desert area.

(a)Both (A) and (R) are true and (R) is the correct explanation of (A).

(b)Both (A) and (R) are true but (R) is not the correct explanation of (A).
(c)(A) is true but (R) is false.

(d)(A) is false but (R) is true.

43.Assertion (A): Some layers of ionosphere disappear during the night and
reappear during the day.

Reason (R): The density of the atmosphere in these layers is very low.

(a)Both (A) and (R) are true and (R) is the correct explanation of (A).

(b)Both (A) and (R) are true but (R) is not the correct explanation of (A).

(c)(A) is true but (R) is false.

(d)(A) is false but (R) is true.

44.Assertion (A): Sometimes in stationary form, sky becomes overcast with


cumulonimbus clouds and thunderstorms are formed.

Reason (R): The cold air is moist and unstable.

(a)Both (A) and (R) are true and (R) is the correct explanation of (A).

(b)Both (A) and (R) are true but (R) is not the correct explanation of (A).

(c)(A) is true but (R) is false.

(d)(A) is false but (R) is true.

45.Assertion (A): Sometimes sky is overcast by thick clouds but there is no


rainfall.

Reason (R): Rainfall does not occur unless cloud droplets become so large
due to coalescence that the air becomes unable to hold them.
(a)Both (A) and (R) are true and (R) is the correct explanation of (A).

(b)Both (A) and (R) are true but (R) is not the correct explanation of (A).

(c)(A) is true but (R) is false.

(d)(A) is false but (R) is true.

46.Assertion (A): The annual range of temperature is greater over land areas.

Reason (R): Land area warm and cool more rapidly than water bodies.

(a)Both (A) and (R) are true and (R) is the correct explanation of (A).

(b)Both (A) and (R) are true but (R) is not the correct explanation of (A).

(c)(A) is true but (R) is false.

(d)(A) is false but (R) is true.

[UPSC- 2001]

47.Assertion (A): The boiling point of water drops at higher altitudes.

Reason (R): The less gravity causes water to boil faster.

(a)Both (A) and (R) are true and (R) is the correct explanation of (A).

(b)Both (A) and (R) are true but (R) is not the correct explanation of (A).

(c)(A) is true but (R) is false.

(d)(A) is false but (R) is true.

48.Assertion (A): The concentration of charged particles in the ionosphere


changes from day to night.
Reason (R): The production of ions requires radiation reflected from Earth.

(a)Both (A) and (R) are true and (R) is the correct explanation of (A).

(b)Both (A) and (R) are true but (R) is not the correct explanation of (A).

(c)(A) is true but (R) is false.

(d)(A) is false but (R) is true.

49.Assertion (A): India and Southeast Asia are often characterized by the
alternating periods of rainfall and drought.

Reason (R): The monsoons substantially affect these areas.

(a)Both (A) and (R) are true and (R) is the correct explanation of (A).

(b)Both (A) and (R) are true but (R) is not the correct explanation of (A).

(c)(A) is true but (R) is false.

(d)(A) is false but (R) is true.

50.Assertion (A): The mesosphere is the least explored regions of the


atmosphere.

Reason (R): It can only be reached by lowest orbiting satellites.

(a)Both (A) and (R) are true and (R) is the correct explanation of (A).

(b)Both (A) and (R) are true but (R) is not the correct explanation of (A).

(c)(A) is true but (R) is false.

(d)(A) is false but (R) is true.


51.Assertion (A): The moist adiabatic lapse rate is lower than the dry adiabatic
lapse rate.

Reason (R): When condensation begins, the latent heat of condensation is


released in the parcel of air.

(a)Both (A) and (R) are true and (R) is the correct explanation of (A).

(b)Both (A) and (R) are true but (R) is not the correct explanation of (A).

(c)(A) is true but (R) is false.

(d)(A) is false but (R) is true.

[UPSC-2000]

52.Assertion (A): The rate of growth of cloud droplets quickly diminishes.

Reason (R): The available water vapours are quickly consumed.

(a)Both (A) and (R) are true and (R) is the correct explanation of (A).

(b)Both (A) and (R) are true but (R) is not the correct explanation of (A).

(c)(A) is true but (R) is false.

(d)(A) is false but (R) is true.

53.Assertion (A): The temperature in the Southern Hemisphere is considerably


lower than that in the Northern Hemisphere.

Reason (R): The large mass of icecovered Antarctic continent is an


important source of cold in the South.

(a)Both (A) and (R) are true and (R) is the correct explanation of (A).
(b)Both (A) and (R) are true but (R) is not the correct explanation of (A).

(c)(A) is true but (R) is false.

(d)(A) is false but (R) is true.

[UPSC-2002]

54.Assertion (A): The tropical cyclones are thermal in origin.

Reason (R): The tropical cyclones develop due to Coriolis force.

(a)Both (A) and (R) are true and (R) is the correct explanation of (A).

(b)Both (A) and (R) are true but (R) is not the correct explanation of (A).

(c)(A) is true but (R) is false.

(d)(A) is false but (R) is true.

[UPSC-1998]

55.Assertion (A): Tropical cyclones only affect the coastal areas of the land.

Reason (R): The centres of these cyclones have extremely low pressure.

(a)Both (A) and (R) are true and (R) is the correct explanation of (A).

(b)Both (A) and (R) are true but (R) is not the correct explanation of (A).

(c)(A) is true but (R) is false.

(d)(A) is false but (R) is true.

56.Assertion (A): Troposphere is the chief focus of meteorologist.


Reason (R): in this layer, essentially all important weather phenomena
occur.

(a)Both (A) and (R) are true and (R) is the correct explanation of (A).

(b)Both (A) and (R) are true but (R) is not the correct explanation of (A).

(c)(A) is true but (R) is false.

(d)(A) is false but (R) is true.

57.Assertion (A): When the air ascends hurriedly with great speed, heavy
downpour results.

Reason (R): The speed of air helps in forming very large raindrops.

Codes:

(a)Both (A) and (R) are true and (R) is the correct explanation of (A).

(b)Both (A) and (R) are true but (R) is not the correct explanation of (A).

(c)(A) is true but (R) is false.

(d)(A) is false but (R) is true.

58.Assertion (A): Considering the world's wind systems, the westerlies


normally blow between 40° and 60° latitudes.

Reason (R): The flow of air from subtropical high pressure leads to the
formation of mid-latitudinal planetary wind system.

(a)Both (A) and (R) are true and (R) is the correct explanation of (A).

(b)Both (A) and (R) are true but (R) is not the correct explanation of (A).
(c)(A) is true but (R) is false.

(d)(A) is false but (R) is true.

[UPSC-1998]

59.Assertion (A): The surface winds spiral inwards upon the centre of the
cyclone.

Reason (R): Air descends in the centre of the cyclone.

(a)Both (A) and (R) are true and (R) is the correct explanation of (A).

(b)Both (A) and (R) are true but (R) is not the correct explanation of (A).

(c)(A) is true but (R) is false.

(d)(A) is false but (R) is true.

[UPSC-2002]

60.Assertion (A): Anti-cyclonic conditions are formed in winter season when


atmospheric pressure is high and air temperatures are low.

Reason (R): Winter rainfall in Northern India causes development of


anticyclone conditions with low temperatures.

(a)Both (A) and (R) are true and (R) is the correct explanation of (A).

(b)Both (A) and (R) are true but (R) is not the correct explanation of (A).

(c)(A) is true but (R) is false.

(d)(A) is false but (R) is true.

[UPSC-2001]
61.Assertion (A): Areas lying within 5 to 8° latitude on either side of the
Equator receive rainfall throughout the year.

Reason (R): High temperatures and high humidity afternoons are


experienced near the Equator.

(a)Both (A) and (R) are true and (R) is the correct explanation of (A).

(b)Both (A) and (R) are true but (R) is not the correct explanation of (A).

(c)(A) is true but (R) is false.

(d)(A) is false but (R) is true.

[UPSC-2003]

62.Assertion (A): In the Mediterranean climate, a pronounced winter


precipitation is observed.

Reason (R): The Mediterranean regime is controlled by subtropical anti-


cyclone in winter.

(a)Both (A) and (R) are true and (R) is the correct explanation of (A).

(b)Both (A) and (R) are true but (R) is not the correct explanation of (A).

(c)(A) is true but (R) is false.

(d)(A) is false but (R) is true.

[UPSC-2008]

63.Assertion (A): In western Europe, rainfall decreases form the coastal areas
towards the interior.

Reason (R): Most of the western Europe lies in the easterlies belt.
(a)Both (A) and (R) are true and (R) is the correct explanation of (A).

(b)Both (A) and (R) are true but (R) is not the correct explanation of (A).

(c)(A) is true but (R) is false.

(d)(A) is false but (R) is true.

[UPSC-2000]

64.Assertion (A): Large deserts are located at low latitudes.

Reason (R): Continental drift has moved them to lower latitudes from
higher latitudes.

(a)Both (A) and (R) are true and (R) is the correct explanation of (A).

(b)Both (A) and (R) are true but (R) is not the correct explanation of (A).

(c)(A) is true but (R) is false.

(d)(A) is false but (R) is true.

[UPSC-2008]

65.Assertion (A): The amount of moisture in the atmosphere is related to


latitude.

Reason (R): The capacity to hold moisture in the form of water vapour is
related to temperature.

(a)Both (A) and (R) are true and (R) is the correct explanation of (A).

(b)Both (A) and (R) are true but (R) is not the correct explanation of (A).

(c)(A) is true but (R) is false.


(d)(A) is false but (R) is true.

[UPSC-2003]

66.Assertion (A): The thickness of the atmosphere is maximum over the


Equator.

Reason (R): High isolation and strong convection currents occur over the
Equator.

(a)Both (A) and (R) are true and (R) is the correct explanation of (A).

(b)Both (A) and (R) are true but (R) is not the correct explanation of (A).

(c)(A) is true but (R) is false.

(d)(A) is false but (R) is true.

[UPSC-2004]

67.Assertion (A): The westerlies of the Southern Hemisphere are stronger and
more constant in direction than those of the Northern Hemisphere.

Reason (R): The broad expanses of ocean in the Southern Hemisphere rule
out the development of stationary pressure systems.

(a)Both (A) and (R) are true and (R) is the correct explanation of (A).

(b)Both (A) and (R) are true but (R) is not the correct explanation of (A).

(c)(A) is true but (R) is false.

(d)(A) is false but (R) is true.

[UPSC-2008]
68.Assertion (A): Wind patterns are clockwise in the Northern Hemisphere and
anticlockwise in the Southern Hemisphere.

Reason (R): The directions of wind patterns in the Northern and the
Southern hemispheres are governed by the Coriolis effect.

(a)Both (A) and (R) are true and (R) is the correct explanation of (A).

(b)Both (A) and (R) are true but (R) is not the correct explanation of (A).

(c)(A) is true but (R) is false.

(d)(A) is false but (R) is true.

[UPSC-2005]

69.At the geostrophic balance

(a)there are no winds.

(b)winds generated are called geostrophic winds.

(c)winds are at high speed.

(d)winds are at low speed.

70.At the Tropic of Cancer, the Sun is directly overhead at noon on the

(a)summer solstice.

(b)winter solstice.

(c)autumnal equinox.

(d)vernal equinox.
71.On what date will 50°N receive the largest amount of daylight'?

(a)December 21

(b)March 21

(c)September 22

(d)June 21

72.At which angle (of incidence) will the intensity of energy striking Earth be
greatest?

(a)30°

(b)60°

(c)0°

(d)90°

73.Atmospheric condition of absolute stability occurs when

1.dry adiabatic lapse rate is lower than normal lapse rate.

2.dry adiabatic lapse rate is higher than normal lapse rate.

3.condensation point has been reached.

(a)Only 1

(b)Only 2

(c)1 and 3

(d)2 and 3
74.Atmospheric instability occurs when

(a)dry adiabatic lapse rate is lower than normal lapse rate.

(b)dry adiabatic lapse rate is higher than normal lapse rate.

(c)dry adiabatic lapse rate is equal to normal lapse rate.

(d)Either (a) or (b)

75.Boreal forest climates are associated with

(a)sub-Arctic climates.

(b)Mediterranean climates.

(c)humid low-latitude climates.

(d)Tundras.

76.Burning which of the following materials contributes carbon dioxide to


Earth's atmosphere'?

(a)Wood

(b)Coal

(c)Petroleum

(d)All of these.

77.Climate over the past 2 million years has

(a)stayed the same.

(b)become progressively warmer.


(c)shifted between warm and cool periods.

(d)become progressively cooler.

78.Clouds

(a)scatter the solar radiation.

(b)backscatter the solar radiation.

(c)reflect the solar radiations back.

(d)Both (a) and (b)

79.Cloudy nights are warmer compared to clear cloudless nights because


clouds

(a)prevent clod waves from the sky from descending on Earth.

(b)reflect back the heat given off by Earth.

(c)produce heat and radiate it towards Earth.

(d)absorb heat from the atmosphere and send it towards Earth.

[UPSC-2001]

80.Collision coalescence process of precipitation is applicable to

(a)cirrus.

(b)clouds that extend beyond freezing level.

(c)clouds that do not extend beyond freezing level.

(d)all types of clouds.


[UPSC-2006]

81.Condensation is a process which occurs when

(a)temperature falls below 0°C.

(b)absolute humidity becomes equal to relative humidity.

(c)temperature in the air parcel remains static.

(d)relative humidity becomes 100 per cent and more.

[UPSC-2009]

82.Consider the following climate date:

The above climate date resembles that of which one of the following cities'?
(a)Buenos Aires

(b)Tokyo

(c)Melbourne

(d)San Francisco

[UPSC-2008]

83.Consider the following climatic and geographical phenomena:

1.Condensation

2.High temperature and humidity

3.Orography

4.Vertical wind

Thunder cloud development is due to which of these phenomena?

(a)1 and2

(b)2, 3 and 4

(c)1, 3 and 4

(d)1, 2, 3 and 4

[UPSC-2002]

84.Consider the following climatic conditions (Northern Hemisphere):


These are most likely to be found in the natural regions of

(a)China type.

(b)equatorial type.

(c)hot desert type.

(d)monsoon type.

[UPSC-1998]

85.Consider the following conditions for the formation of temperature


inversion:

1.Cloud sky
2.Strong winds

3.Long winter nights

4.Cold dry air

Which of the above conditions are ideal'?

(a)1, 2 and 3

(b)1 and 2

(c)2, 3 and 4

(d)3 and 4

[UPSC-2001]

86.Consider the following statements:

1.General characteristics of westerlies are largely modified by cyclones and


anti-cyclones.

2.Westerlies are the most dominant wind systems in middle latitudes.

3.Westerlies become more vigorous in the Southern Hemisphere.

Which of the statements given above is/are correct?

(a)1 and 2 only

(b)2 only

(c)1 and 3 only

(d)1, 2 and 3
[UPSC-2006]

87.Consider the following statements:

1.The decrease of temperature with increasing latitude is more pronounced


in North Atlantic Ocean than the South Atlantic Ocean.

2.The vertical distribution of temperature is enclosed seas of higher


latitudes registers inversion of temperature.

Which of the statements given above is/ are correct?

(a)1 only

(b)2 only

(c)Both 1 and 2

(d)Neither 1 nor 2

[UPSC-2006]

88.Consider the following statements:

1.Strong winds

2.Long winter night

3.Cloudy sky

4.Cold dry air

Which of the above are the ideal conditions of the formation of temperature
inversions'?

(a)1, 2 and 3
(b)3 and 4

(c)1, 2 and 4

(d)2 and 4 only

[UPSC-2006]

89.Consider the following statements:

1.Most rainfall and snowfall in the mid-latitudes are formed by the ice
crystal process.

2.The base of cirrus clouds invariably occurs at an altitude below 5,000 m.

Which of the statements given above is/ are correct?

(a)1 only

(b)2 only

(c)Both 1 and 2

(d)Neither 1 nor 2

[UPSC-2007]

90.Consider the following statements:

1.The variation in average annual range of temperature at 15° latitude of


Southern Hemisphere is much less than that at 150 latitude of Northern
Hemisphere.

2.At latitudes pole wards of 35°, the incoming radiation exceeds the
outgoing radiation in Northern Hemisphere, whereas in Southern
Hemisphere, the outgoing radiation exceeds the incoming radiation.
Which of the statements given above is/ are correct?

(a)1 only

(b)2 only

(c)Both 1 and 2

(d)Neither 1 nor 2

[UPSC-2007]

91.Consider the following statements:

1.In the tropical rain forest of Brazil, the average diurnal range of
temperature is much greater than the average annual range of
temperature.

2.In the humid subtropical climate of Kyushu island of Japan, the rainfall
for the year almost totals that of the rainfall in tropical rainforest.

Which of the statements given above is/ are correct?

(a)1 only

(b)2 only

(c)Both 1 and 2

(d)Neither 1 nor 2

[UPSC-2007]

92.Consider the following statements:

1.A hurricane acquires its spin from the Coriol is effect.


2.The diameter of the hurricane decreases as it moves away from low
latitudes.

3.The diameter of a hurricane is never below 150 km.

Which of the statements given above is/ are correct'?

(a)1 only

(b)2 only

(c)Both 1 and 2

(d)Neither 1 nor 2

[UPSC-2007]

93.Consider the following statements:

1.The subtropical jet stream is evident throughout the year, but the polar
front jet stream is strong only during the half-year centred on winter.

2.The migration of equatorial low (ITCZ) into the Southern Hemisphere in


January is far less pronounced than its migration into the Northern
Hemisphere in July.

Which of the statements given above is/ are correct'?

(a)1 only

(b)2 only

(c)Both 1 and 2

(d)Neither 1 nor 2
[UPSC-2007]

94.Consider the following statements:

1.Tropical cyclones occur only in oceanic areas where the sea temperature
exceeds 33°C.

2.In order to achieve the rotational component, the tropical cyclone must be
spawned at least 5°N or 5°S of the Equator.

Which of the statements given above is/ are correct'?

(a)1 only

(b)2 only

(c)Both 1 and 2

(d)Neither 1 nor 2

[UPSC-2008]

95.Consider the following statements

1.Convective type of precipitation is associated with towering cumulus and


cumulonimbus clouds.

2.In tropical cyclones, cumulonimbus cells become organized around the


centre in spiralling bands

Which of the statements given above is/ are correct?

(a)1 only

(b)2 only
(c)Both 1 and 2

(d)Neither 1 nor 2

[UPSC-2008]

96.Consider the following statements:

1.Shifting of the subtropical highpressure belt towards the Equator.

2.Lower seasurfaces temperature in the eastern North Atlantic.

3.Expansion of the circumpolar westerly vortex.

Which of the statements given above is/ are correct'?

(a)1 only

(b)2 only

(c)Both 1 and 2

(d)Neither 1 nor 2

[UPSC-2008]

97.Consider the following statements:

1.The jet streams are located near the altitude of the tropopause.

2.The polar front jet stream is evident throughout the year, but the
subtropical jet stream is strong only during the half-year centred on
winter.

Which of the statements given above is/ are correct?


(a)1 only

(b)2 only

(c)Both 1 and 2

(d)Neither 1 nor 2

[UPSC-2008]

98.Consider the following statements:

1.Nearly 99 per cent of the total mass of the atmosphere lies within 30 km
of the Earth's surface.

2.The proportion of ozone in the atmosphere increased to a maximum at


about 60 km from Earth's surface.

Which of the statements given above is/ are correct?

(a)1 only

(b)2 only

(c)Both 1 and 2

(d)Neither 1 nor 2

[UPSC-2008]

99.Consider the following statements:

1.Mistral is a cold wind blowing in North America.

2.Sirocco is a hot dry wind blowing across the North African coast.
Which of the statements given above is/ are correct?

(a)1 only

(b)2 only

(c)Both 1 and 2

(d)Neither 1 nor 2

[UPSC-2009]

100.Consider the following statements about the Roaring Forties:

1.They blow uninterrupted in the Northern and Southern hemispheres.

2.They blow with great strength and constancy.

3.Their direction is generally from north-west to east in the Southern


Hemisphere.

4.Overcast skies, rain, and raw weather are generally associated with them.

Which of these statements are correct?

(a)1, 2 and 3

(b)2, 3 and 4

(c)1, 3 and 4(d)

1,2 and 4

[UPSC-2000]
101.Consider the following statements regarding general characteristics
associated with tropical cyclones:

1.Tropical cyclones move with very low velocity over the oceans but
become strong while moving over the land areas.

2.They are confined to a particular period of the year (mainly summer) and
normally move from east to west.

Which of the statements given above is/ are correct?

(a)1 only

(b)2 only

(c)Both 1 and 2

(d)Neither 1 nor 2

[UPSC-2009]

102.Consider the following statements regarding lapse rate:

1.An unsaturated air mass cools at dry adiabatic lapse rate when it rises in
the atmosphere.

2.The dry adiabatic lapse rate amounts to 1°C per 100 meters.

3.The wet adiabatic lapse rate may vary between 0.4°C and 0.9°C per 100
meters.

4.Dry adiabatic lapse rate is higher because of the release of latent heat of
condensation.

Which of the above statements are correct'?


(a)1, 2 and 4

(b)1, 3 and 4

(c)2, 3 and 4

(d)1, 2 and 4

[UPSC-2001]

103.Consider the following statements regarding the distribution and pattern of


isotherms:

1.In the Northern Hemisphere, isotherms bend sharply Equator-ward over


the continents during winter.

2.Temperature gradients are greater in summer than in winter in the


Northern Hemisphere.

Which of the statements given above is/ are correct'?

(a)1 only

(b)2 only

(c)Both 1 and 2

(d)Neither 1 nor 2

[UPSC-2009]

104.Consider the following statements:

1.In equatorial regions, the year is divided into four main seasons.

2.In Mediterranean region, summer receives more rain.


3.In China-type climate, rainfall occurs throughout the year.

4.Tropical highlands exhibit vertical zonation of different climates.

Which of these statements are correct'?

(a)1, 2, 3 and 4

(b)1, 2 and 3

(c)1, 2 and 4

(d)3 and 4

[UPSC-2002]

105.Consider the following statements:

1.Either of the two belts over the oceans at about 30° to 35°N and S
latitudes is known as horse latitude.

2.Horse latitudes are low-pressure belts.

Which of the statement given above is/ are correct?

(a)1 only

(b)2 only

(c)Both 1 and 2

(d)Neither 1 nor 2

[UPSC-2007]

106.Consider the following statements:


1.The albedo of an object determines its visual brightness when viewed with
reflected light.

2.The albedo of Mercury is much greater than the albedo of the Earth.

Which of the statement given above is/ are correct'?

(a)1 only

(b)2 only

(c)Both 1 and 2

(d)Neither 1 nor 2

[UPSC-2008]

107.Consider the following statements:

1.In the world, the tropical desert occurs along the western margins of
continent within the trade wind belt.

2.In India, the east Himalaya region gets high rainfall from north-east
winds.

Which of the statements given above is/ are correct'?

(a)1 only

(b)2 only

(c)Both 1 and 2

(d)Neither 1 nor 2

[UPSC-2009]
108.Consider the following statements:

1.In the high latitudes, eastcoast regions are wetter than westcoast.

2.Precipitation is abundant on the windward slopes but sparse on the


leeward slopes.

3.In the subtropics, westcoast is wetter than the eastcoast.

4.Precipitation is abundant in the middle latitudes.

Which of the above statements is/are correct?

(a)1, 2 and 3

(b)2 only

(c)2 and 4

(d)3 and 4

[UPSC-2001]

109.Consider the following statements:

1.Ozone is found mostly in the stratosphere.

2.Ozone layer lies 55 to 75 km above the surface of the Earth.

3.Ozone absorbs ultraviolet radiation from the Sun.

4.Ozone layer has no significance for life on the Earth.

Which of the above statements are correct'?

(a)1 and3
(b)2 and 4

(c)2 and 3

(d)1 and 4

[UPSC-2000]

110.Consider the following temperature and rainfall data:

The climate to which this data pertains is

(a)St. Lawrence type.

(b)China type.

(c)West European type.


(d)Mediterranean type.

[UPSC-1999]

111.Continental polar air masses tend to be

(a)warm and dry.

(b)cool and wet.

(c)cool and dry.

(d)warm and wet.

112.Coriolis force

1.affects wind speed.

2.is affected by wind speed.

3.is strongest at poles.

(a)1 and 2

(b)2 and 3

(c)3 and 1

(d)1, 2 and 3

113.Coriolis force is the result of

(a)gravity.

(b)rotation of Earth.
(c)revolution of Earth.

(d)None of these.

114.Cumulus clouds are NOT

(a)very dense and widespread.

(b)generally associated with fair weather.

(c)dome-shaped.

(d)None of these.

115.During the passage of a tropical cyclone at sea coast, the following


atmosphere conditions are observed:

1.Torrential rainfall with gusty winds

2.Barometric fall with dark clouds

3.Total clam and clear skies

The correct sequence of the occurrence of these conditions is:

(a)2, 1, 3

(b)1, 2, 3

(c)3, 2, 1

(d)2, 3, 1

[UPSC-1999]
116.During the 20thcentury, the Earth's atmospheric temperature increased by
about

(a)3°C

(b)0.3°C

(c)2°C

(d)0.8°C

117.During the vernal equinox, which location receives the greatest amount of
insolation?

(a)Tropic of Cancer

(b)Equator

(c)poles

(d)Tropic of Capricorn

118.Eccentricity, obliquity and precession are considered to be causes of


climate change.

(a)natural

(b)astrological

(c)volcanic

(d)solar

119.Environmental lapse rate is associated with

(a)stratosphere.
(b)ionosphere.

(c)heterosphere.

(d)troposphere.

120.Explosive volcanic eruptions can act to global temperatures.

(a)raise

(b)lower

(c)moderate

(d)Volcanic eruptions do not affect temperature.

121.Fifty per cent of Earth's atmosphere is found below an altitude of

(a)7.4 km

(b)5.6 km

(c)4.2 km

(d)7.9 km

122.Fohn and chinook winds are experienced in

(a)the valleys of the northern Alps in Switzerland and on the eastern slopes
on the Rockies in North America.

(b)the eastern slopes of southern Alps in Switzerland and on the western


slopes on Andes and South America.

(c)Po valley of France and in Newfoundland.


(d)Netherlands and New Zealand.

[UPSC-1997]

123.For short-term climate predictions, which one of the following events,


detected in the last decade, is associated with occasional weak monsoon
rains in the India subcontinent?

(a)La Nina

(b)Movement of j et streams

(c)El Nino and southern oscillations

(d)Greenhouse effect on global level

[UPSC-2002]

124.From September to March, the Southern Hemisphere receives

(a)less radiation than the Northern Hemisphere.

(b)more radiation than the Northern Hemisphere.

(c)24 hours of daylight.

(d)exactly 12 hours of daylight and darkness.

125.T hew describes the movement of ater through Earth's hydrosphere.

(a)precipitation cycle

(b)hydrologic cycle

(c)aqualytic cycle
(d)aquatic cycle

126.Generally which of the following fogs will be least dense?

(a)Advection fog

(b)Radiation fog

(c)Steam fog

(d)Generally, all of them are dense.

127.Geostrophic balance is established between (a) vertical and horizontal


pressure gradients.

(b)vertical pressure gradient and gravity.

(c)pressure gradient force and Coriolis force.

(d)horizontal pressure gradient and gravity.

128.Geostrophic winds flow

1.in straight lines.

2.parallel to the isobars.

3.perpendicular to isobars.

(a)1 and 2

(b)2 and 3

(c)3 and 1

(d)1, 2 and 3
129.Given below are two statements one labelled as Assertion (A) and the
other labelled as Reason (R). Select the correct answers from the codes
given.

Assertion (A): Areas near the Equator receive rainfall throughout the year.

Reason (R): High temperatures and high humidity cause convectional rain
in most afternoons near the Equator.

(a)Both (A) and (R) are true and (R) is the correct explanation of (A).

(b)Both (A) and (R) are true and (R) is not the correct explanation of (A).

(c)(A) is true but (R) is false.

(d)(A) is false but (R) is true.

[UPSC-1996]

130.Global warming is likely to have a greater adverse effect on regions.

(a)boreal

(b)tropical

(c)polar

(d)coastal

131.Guano is a unique fertilizer product of

(a)Atacama.

(b)Grand Banks.

(c)Paraguay.
(d)coastal areas and inlands off Peru.

[UPSC-1995]

132.Heat budget

1.remains constant year to year.

2.varies from year to year.

3.on an average causes us to lose all the energy received.

(a)Only 1

(b)Only 2

(c)1 and 3

(d)2 and 3

133.Heat can be transferred through the vacuum by the method of

(a)convection.

(b)radiation.

(c)conduction.

(d)none of these.

134.Heterosphere is equivalent to

(a)stratosphere.

(b)mesosphere.
(c)troposphere.

(d)thermosphere.

135.High temperature and low pressure over the Indian subcontinent during
the summer season draws air from the Indian Ocean leading to the in-
blowing of the

(a)south-east monsoon.

(b)south-west monsoon.

(c)trade winds.

(d)westerlies.

[UPSC-1996]

136.How many primary groups of Koppen's climate classification use


precipitation as primary criteria for definition'?

(a)One

(b)Two

(c)Three

(d)Four

137.Hydrostatic equilibrium is established between

(a)vertical and horizontal pressure gradients.

(b)vertical pressure gradient and gravity.

(c)horizontal pressure gradient and Coriolis force.


(d)horizontal pressure gradient and gravity.

138.Ice crystal theory was postulated by

(a)Longmuir.

(b)Tor Bergeron.

(c)Jameson.

(d)Belington.

139.Identify the climatic type which has cool summer (10°C to 15°C) and cold
winter (0°C to 5°C) and rainfall throughout the year with no dry season.

(a)Df

(b)CW

(c)CS

(d)Cf

[UPSC-2001]

140.Identify the correct sequence of the given processes regarding rainfall.

(a)Unsaturated air, condensation, dew point, precipitation

(b)Dew point, condensation, unsaturated air, precipitation

(c)Unsaturated air, dew point, condensation, precipitation

(d)Dew point, precipitation, condensation unsaturated air

[UPSC-2001]
141.If cold ocean currents hit the coast of a continent, it is likely that the
coastal area will

(a)have higher levels of evaporation than with warm currents.

(b)be relatively arid.

(c)have extensive precipitation.

(d)be subtropical.

142.If Earth's atmosphere contained no greenhouse gases, the average surface


temperature would be

(a)non-existent

(b)unchanged

(c)lower

(d)incalculable

143.If there was no inclination of the axis of Earth we will

(a)Not have seasons

(b)Not have day and night

(c)Not have atmosphere

(d)All of these

144.If you are told you are in a BWh climate, in which of the following
locations might you be?

(a)Poland
(b)Florida

(c)Sahara Desert

(d)Iceland

145.If you were at a location that just experienced a thunderstorm with heavy
downpours, what could you predict about a location 25 miles to the
southwest?

(a)It received more rainfall.

(b)It is not predictable.

(c)It received approximately the same rainfall.

(d)It received less rainfall.

146.In Koppen's classification of world climate, Aw climates refer to

(a)humidtropical wetmonsoon climate with a short dry season.

(b)tropical wet and dry climate with dry season in winter.

(c)tropical desert climate.

(d)humid mesothermal climates.

[UPSC-2009]

147.In the, the temperature remains constant to an altitude of about 20 km.

(a)stratosphere

(b)mesosphere
(c)stratopause

(d)troposphere

148.In the case of saturated air, the column of air tends to be in stable
equilibrium when the prevailing lapse rate

(a)is equal to the wet-adiabatic lapse rate.

(b)is more than the wet-adiabatic lapse rate.

(c)is less than the wet-adiabatic lapse rate.

(d)does not vary with the wet-adiabatic lapse rate.

[UPSC-1999]

149.In the Hadley cell, rising air is found in the

(a)subtropical high-pressure zone.

(b)trade winds.

(c)easterlies.

(d)intertropical convergence zone.

[UPSC-2004]

150.In the low latitudes, the average temperature is between°C.

(a)35 and 37

(b)65 and 72

(c)26 and 28
(d)50 and 54

151.In the Northern Hemisphere, the Sun is lowest in the sky with less
radiation in the

(a)spring.

(b)summer.

(c)winter.

(d)fall.

152.In the Northern Hemisphere, tropical cyclones and middle latitude


cyclones are similar in that both

(a)originate over warm ocean.

(b)intensify with increasing height above the ground.

(c)generally move from east to west.

(d)have winds that blow anticlockwise around their centres.

[UPSC-2003]

153.In the south-eastern United States, which climate is dominant?

(a)BS

(b)Dwc

(c)Cfa

(d)AW
154.In Tricellular Meridional Circulation model of the atmosphere, which of
the following cell is also called Hardey Cell'?

(a)Polar Cell

(b)Mid-latitude Cell

(c)Tropical Cell

(d)None of these

155.In Tricellular Meridional Circulation model of the atmosphere, which of


the following cell is also known as Ferrel Cell?

(a)Polar Cell

(b)Mid-latitude Cell

(c)Tropical Cell

(d)None of these

156.In tropical climates, the combination of th creates intense daily onvective


storms.

(a)low temperatures, dry air

(b)high temperatures, dry air

(c)low temperatures, humidity

(d)high temperatures, humidity

157.In which of the following layers of ionosphere does the concentration of


ions and electrons change very slowly?
(a)Layer A

(b)Layer B

(c)Layer C

(d)None of these

158.In which of the following regions monthly mean temperature does not
exceed 0°C?

1.ET

2.EF

(a)Only 1

(b)Only 2

(c)Both 1 and 2

(d)Neither 1 nor 2

159.In which one of the following regions are the hurricanes apparently
absent?

(a)Coast of Texas

(b)Gulf of Mexico

(c)South-eastern coast of USA

(d)South-eastern coast of Pacific

[UPSC-2009]
160.In which zone would temperature vary more in a single day than between
months?

(a)polar areas

(b)low latitudes

(c)highlands

(d)mid-latitudes

161.Increased levels of atmospheric CO2 will ultimately the pH of global


oceans.

(a)raise

(b)suspend

(c)lower

(d)have no effect on

162.Increasing quantities of will eventually produce global warming.

(a)volcanic aerosols

(b)ozone

(c)carbon dioxide

(d)water vapour

163.Inversion of rainfall occurs

(a)due to marked decrease in pressure of air.


(b)due to marked decrease in temperature of air.

(c)due to marked decrease in moisture content of air.

(d)None of these

164.ITCZ stands for

(a)Intertropical Convergence Zone.

(b)International Convergence Zone.

(c)Intertropical Catalyst Zone.

(d)International Catalyst Zone.

165.Koppen's scheme divides Climate into principal climate groups.

(a)four

(b)five

(c)six

(d)seven

166.La Nina

(a)only affects the Northern Hemisphere.

(b)always lasts for the same period of time as the previous El Nino.

(c)has nothing to do with ocean temperatures and circulation.

(d)can lead to drought in the southern USA.


167.Large areas in India are subjected to considerable rainfall variability.
Rainfall variability is likely to be the greatest in the areas of

(a)very high rainfall.

(b)high rainfall.

(c)medium rainfall.

(d)very low rainfall.

[UPSC-2001]

168.Latent heat is

(a)heat left behind by a resting object.

(b)heat stored in water and water vapour.

(c)detectable by sense of touch.

(d)heat reflected into the atmosphere.

169.Layers D, E and F are found in

(a)stratosphere.

(b)ionosphere.

(c)homosphere.

(d)troposphere.

170.Lines of indirect evidence that scientists use to reconstruct past climates


are termed
(a)inferences

(b)hypotheses

(c)paradigms

(d)proxy data

171.Marine west-coast climates are NOT

(a)mild climates with a small annual temperature range.

(b)commonly found in low latitudes.

(c)plentifully moist year round.

(d)found in places like Kodiak, Alaska.

172.Match List I with List II and select he correct answer using the codes
given.
Codes:

[UPSC-2005]

173.Match List I with List II and select the correct answer using the codes
given.
Codes:

[UPSC-1995]

174.Match List I with List II and select the correct answer using the codes
given.
Codes:

[UPSC-1996]

175.Match List I with List II and select the correct answer using the codes
given.

Codes:

[UPSC-1997]
176.Match List I with List II and select the correct answer using the codes
given.

Codes:

[UPSC-1997]

177.Match List I with List II and select the correct answer using the codes
given.
Codes:

[UPSC-1997]

178.Match List I with List II and select the correct answer using the codes
given.
Codes:

[UPSC-1999]

179.Match List I with List II and select the correct answer using the codes
given.
Codes:

[UPSC-2001]

180.Match List I with List II and select the correct answer using the codes
given.

Codes:
[UPSC-2003]

181.Match List I with List II and select the correct answer using the codes
given.

Codes:
[UPSC-2003]

182.Match List I with List II and select the correct answer using the codes
given.

Codes:

[UPSC-2003]

183.Match List I with List II and select the correct answer using the codes
given.
Codes:

[UPSC-2007]

184.Match List I with List II and select the correct answer using the codes
given.
Codes:

[UPSC-2007]

185.Match List I with List II and select the correct answer using the codes
given.

Codes:
[UPSC-2009]

186.Mean monthly temperature means

(a)the sum of daily temperature in a month.

(b)the sum of daily temperature in a year in that month.

(c)the sum of daily means temperature in a month divided by the number of


days in that month.

(d)the sum of daily moon temperature divided by the number of days in that
month.

[UPSC-1995]

187.Melting of permafrost will release as a result of decomposing vegetation.

(a)hydrogen and helium

(b)nitrous oxide

(c)carbon dioxide and methane

(d)CFCs and ozone

188.Mesopause is located at approximate height of

(a)20 km.
(b)40 km.

(c)60 km.

(d)80 km.

189.Methane (CH4), nitrous oxide (N 20) and chlorofluorocarbons (CFCs) are


considered (greenhouse) gases.

(a)constituent

(b)volatile

(c)trace

(d)negligible

190.Methane is produced by bacteria.

(a)polymorphic

(b)anaerobic

(c)aerobic

(d)oxidized

191.Most of the weather phenomena take place in troposphere only because

(a)temperature remains almost constant in troposphere.

(b)it contains electrically charged particles.

(c)it absorbs harmful ultraviolet radiation from the Sun.

(d)it contains almost all the water vapour and most dust particles.
[UPSC-2002]

192.Most of the world's deserts occur in which zone?

(a)Mid-latitude low-pressure zone

(b)Subtropical high-pressure zone

(c)Intertropical convergence zone

(d)Polar high-pressure zones

193.Most of the world's tropical rainforests lie within

(a)30°S and 60°S.

(b)60°S and 80°S.

(c)30°N and 60°N.

(d)10°N and 10°S.

194.Most part of Central Africa is forested because

(a)most of the countries are thinly populated.

(b)of the rugged terrain.

(c)the sub-Saharan soil is most suitable for forest growth.

(d)Equator passes through the middle of the region.

[UPSC-1995]

195.Movement of a fluid when part of it is heated is

(a)convection.
(b)conduction.

(c)advection.

(d)sublimation.

196.Nimbostratus clouds are NOT

(a)low clouds having dark colour.

(b)associated with lightning, thunder or hailstorm.

(c)associated with snow and sleet.

(d)compact and thick.

197.Ocean water movement accounts for approximately of the total heat


transport on Earth

(a)10 per cent

(b)25 per cent

(c)40 per cent

(d)60 per cent

198.Off the coast of South America, as a result of a circulation change called


El Nino,

(a)the fisheries off the coast increase in productivity.

(b)the normal cool-water flow is replaced by warm waters.

(c)no effect has been observed.


(d)the normal warm-water flow is often replaced by cool waters.

199.On a daily basis, the Sun is most intense at

(a)12 noon.

(b)6.00 a.m.

(c)6.00 p.m.

(d)3.00 p.m.

200.On the Pacific Islands of Oceania, which climate would you expect to
find?

(a)Tundra

(b)Humid tropical

(c)Semi-arid

(d)Humid continental

201.Ozone is concentrated in the

(a)stratosphere.

(b)thermosphere.

(c)troposphere.

(d)cryosphere.

202.Ozone is created when

(a)two molecules of oxygen (Oz) collide.


(b)when two single atoms of oxygen (0) collide.

(c)when a single atom of oxygen (0) collides with molecule of oxygen (O2).

(d)None of these

203.Permafrost is a feature of

(a)ET.

(b)EF.

(c)M.

(d)Cfb.

204.Permafrost occurs in what kind of climate'?

(a)Dfa

(b)Csa

(c)Aw

(d)ET

205.Permanent ice and snow are found in

(a)ET.

(b)EF.

(c)Dfc.

(d)Cfb.
206.The phrase `growing degree days' is related to

(a)agriculture economies.

(b)temperature.

(c)pressure.

(d)wind speed.

207.Polar high-pressure belt is

(a)thermally induced.

(b)dynamically induced.

(c)Both (a) and (b)

(d)Neither (a) nor (b)

208.Potential evapotranspiration exceeds precipitation in

(a)continental climates.

(b)the tropics.

(c)the Tundra.

(d)dry lands.

209.Radiation fog occurs

(a)due to nuclear population.

(b)when warm and moist air lies over cold ground.


(c)when cold air and warm moist air mix.

(d)when cold air moves from land over oceanic surface.

210.Rain shadows in mountainous areas are a result of

(a)orographic uplift.

(b)adiabatic cooling.

(c)frontal uplift.

(d)convection.

211.Regional large-scale storms where winds converge on areas of low


pressure are

(a)hurricanes.

(b)fronts.

(c)cyclones.

(d)tornados.

212.Regions on the rain shadow (leeward) side of mountains (such as the


Rockies and Andes) are generally

(a)Tundra.

(b)humid continental.

(c)Mediterranean.

(d)semi-arid.
213.Relative humidity changes

1.when the amount of moisture in the air changes.

2.when the air temperature changes.

(a)Only 1

(b)Only 2

(c)Both 1 and 2

(d)Neither 1 nor 2

214.Relative humidity refers to

(a)absolute amount of water vapour in the atmosphere.

(b)the amount of hygroscopic nuclei in the atmosphere.

(c)saturated vapour pressure.

(d)ratio of actual amount of water vapour in the air to the amount it could
hold at the temperature.

[UPSC-1995]

215.Rotational speed of Earth is

(a)constant.

(b)highest at the poles.

(c)highest at the Equator.

(d)highest at the Tropic of Cancer.


216.Sea level has risen over the last 100 ears.

(a)10 and 15 cm

(b)10 and 23 cm

(c)5 and 10 cm

(d)15 and 23 cm

217.Single-cell circulation model was purposed by

(a)Sir James McGill.

(b)George Hadley.

(c)Arthur Daniel.

(d)Charles Davidson.

218.Some scientists believe that these huge magnetic storms are responsible
for variations in temperature and precipitation.

(a)monsoons

(b)typhoons

(c)hurricanes

(d)sunspots

219.Some water that infiltrates the ground is absorbed by the plants. They then
release it into the atmosphere. This process is called

(a)hydrologic cycle.
(b)transpiration.

(c)emancipation.

(d)hydroxylation.

220.Status clouds are

(a)low-lying white-colour clouds.

(b)always close to ground surface.

(c)composed of several uniform layers.

(d)not dense.

221.Stratocumulus clouds are

(a)generally associated with fair of clearing weather.

(b)dark grey in colour.

(c)known as wool-pack clouds.

(d)All of these

222.Sun is closet to Earth at

(a)Perihelion

(b)Aphelion

(c)Both (a) and (b)

(d)Neither (a) nor (b)


223.Sun's halo is produced by the refraction of light in

(a)water vapour in stratus clouds.

(b)ice crystals in cirrocumulus clouds.

(c)ice crystals in cirrus clouds.

(d)dust particles in stratus clouds.

[UPSC-2002]

224.Temperate cyclones are characterized by

(a)fronts.

(b)diverging winds.

(c)dry winds.

(d)hot winds.

[UPSC-2006]

225.Temperature generally decrease towards the poles because

(a)air movement is generally towards the Equator.

(b)cold polar air masses prevent surface heating of the land.

(c)cold surface do not absorb solar energy as readily as warm surfaces.

(d)progressively lesser solar energy per unit area falls on the Earth's surface
as we more to polar regions

[UPSC-2004]
226.Temperature inversion happens in troposphere when

(a)the temperature decreases with increase in altitude.

(b)the temperature increases with increase in altitude.

(c)the layers of troposphere are not shallow.

(d)the average height is approximately 12 km.

227.Temperatures in cities are higher as a result of

(a)increased number of trees.

(b)pavement and buildings storing heat during the day and releasing it at
night.

(c)increased evapotranspiration in the city.

(d)greater insolation.

228.Temperatures rise to more than 1,000 °C in the

(a)cryosphere

(b)troposphere

(c)stratosphere

(d)thermosphere

229.The actual moisture content of a sample of air as a percentage of that


contained in the same volume of saturated air at the same temperature is
called

(a)absolute humidity.
(b)relative humidity.

(c)specific humidity.

(d)vapour pressure.

[UPSC-2002]

230.The advocates of thermal school of the mechanism of general circulation


of the atmosphere

(a)consider horizontal component of the atmospheric circulation.

(b)consider vertical component of the atmospheric circulation.

(c)divide the Earth into three cells.

(d)Both A and B

231.The albedo for Earth as a whole is X per cent. This means that

(a)X per cent of Earth's radiation is lost and does not play any role in
heating the atmosphere.

(b)X per cent of the solar radiation is lost and does not play any role in
heating the atmosphere.

(c)X per cent of the solar radiation is used for heating the atmosphere.

(d)None of these

232.The albedo of which one among the following is the highest'?

(a)Cirrus cloud

(b)Cumulonimbus cloud
(c)Melting snow

(d)Sand

[UPSC-2008]

233.The amount of radiation reflected by a surface is termed its

(a)reflectivity constant

(b)albedo

(c)infrared quotient

(d)electromagnetic spectrum

234.The amount of solar energy intercepted at a particular area on the Earth's


surface is

(a)convection.

(b)insolation.

(c)angle of incidence.

(d)sensible heat.

235.The Asian monsoons in winters are dominated by

(a)subpolar low.

(b)Bermuda high.

(c)Siberian high

(d)Aleutian low.
236.The average value of environmental lapse rate is

(a)3.5°C per kilometre.

(b)5.5°C per kilometre.

(c)6.5°C per kilometre.

(d)7.5°C per kilometre.

237.The axis of rotation of the Earth is inclined how many degrees away from
being perpendicular to the Sun's rays?

(a)23.5°

(b)45-

(c)66.5°

(d)90°

238.The B climate under the Koppen's system of climate classification covers

(a)10 per cent of Earth's surface area.

(b)20 per cent of Earth's surface area.

(c)30 per cent of Earth's surface area.

(d)40 per cent of Earth's surface area.

239.The blueness of the sky is mainly due to

(a)absorption of blue light by the atmosphere.

(b)scattering of sunlight by air molecules.


(c)emission of blue light by air.

(d)presence of water vapour.

[UPSC-2003]

240.The characteristics of troposphere:

1.The temperature decreases with increase in altitude.

2.The temperature increases with decrease in altitude.

3.Environment lapse rate is constant.

(a)Only 1

(b)Only 2

(c)1 and 3

(d)2 and 3

241.The climate system involves the exchanges of energy and moisture that
occur among the spheres.

(a)three

(b)six

(c)seven

(d)five

242.The clouds appear red because

(a)they absorb all light except red.


(b)they are illuminated by sunlight in which most of the blue light has been
lost due to scattering.

(c)they are illuminated by sunlight in which all the lights except red have
been lost due to scattering.

(d)they scatter red light.

243.The coldest temperature in the Earth's atmosphere is found in

(a)mesopause.

(b)thermosphere.

(c)stratosphere.

(d)tropopause.

244.The conversion of water from vapour to liquid is the process of

(a)evaporation.

(b)convection.

(c)saturation.

(d)condensation.

245.The correct sequence of different layers of the atmosphere form the


surface of the Earth upwards is

(a)troposphere, stratosphere, ionosphere, mesosphere

(b)stratosphere, troposphere, ionosphere, mesosphere

(c)troposphere, stratosphere, mesosphere, ionosphere


(d)stratosphere, troposphere, mesosphere, ionosphere

[UPSC-1998]

246.The cyclones of temperate latitudes differ from those of the tropics in a


number of ways. Which one of the following is the most prominent of them
all?

(a)Their size is far greater.

(b)They are caused by frontal system whereas the tropical cyclones are not.

(c)These occur more.

(d)The precipitation is more evenly distributed in a temperate cyclone.

[UPSC-1995]

247.The energy with wavelengths between 0.4 and 0.7 micrometres is known
as

(a)X-rays.

(b)radio waves.

(c)visible light.

(d)infrared.

248.The environment of wet tropics (Af, Am) cover almost

(a)10 per cent of the Earth's land area.

(b)20 per cent of the Earth's land area.

(c)30 per cent of the Earth's land area.


(d)40 per cent of the Earth's land area.

249.The ET classification of Koppen's system stands for

(a)extraterritorial climate.

(b)Tundra climate

(c)ice cap climate.

(d)highland climate.

250.The flow or influx of radiation is proportional to the fourth power of the


absolute temperature of the radiating body. This is known as

(a)Wien's Displacement Law.

(b)Stefan-Boltzmann Law.

(c)Planck's Law.

(d)Kirchhoff's Law.

251.The following are the features of precipitation:

1.It is of very short duration but occurs in form of heavy showers.

2.It is accompanied by cloud thunder and lighting.

3.It occurs through thick dark and extensive cumulonimbus clouds.

4.In hot deserts, it is irregular and sudden.

These features are found in

(a)orographic rainfall.
(b)frontal rainfall.

(c)convectional rainfall.

(d)sleet.

252.The following table contains temperature and rainfall data of a station in


the Southern Hemisphere

The data is indicative of

(a)subtropical dry summer.

(b)tropical wet dry climate.

(c)subtropical humid climate.


(d)marine west coast climate.

[UPSC-2000]

253.The fuel driving the weather is

(a)convection.

(b)inertia of motion.

(c)gravity.

(d)solar energy.

254.The glow of the auroras is caused by

(a)solar flares.

(b)nuclear fission.

(c)nuclear fusion.

(d)None of these

255.The heating of Earth's atmosphere mainly happens as the result of

(a)insolation.

(b)ground radiation.

(c)counter radiation.

(d)None of these

256.The highest daytime temperature is generally recorded


(a)at noon.

(b)in the mid to late morning.

(c)in the mid to late afternoon.

(d)before the sunset.

257.The hot dust-laden wind blowing from Sahara desert to Mediterranean


region is called the

(a)chinook.

(b)fohn.

(c)mistral.

(d)sirocco.

[UPSC-1996]

258.The increase in atmospheric nitrous oxide ('laughing gas') results primarily


from

(a)automobiles

(b)fertilizers

(c)locomotives

(d)factories

259.The increase in global temperature during the 20th century was


approximately

(a)0.7°C
(b)0.5°C

(c)0.9°C

(d)0.8°C

260.The inversion of temperature

(a)encourages rainfall.

(b)is responsible for causing frost.

(c)discourages aridity.

(d)All of these

261.The main cause of Kalahari being a desert is

(a)the cold currents near the coast.

(b)its location on the west coast in the tropics.

(c)its location in the tropics.

(d)that its people are primitive.

[UPSC-1996]

262.The mass of water vapour in grams contained in a kilogram of air is


known as

(a)absolute humidity.

(b)specific humidity.

(c)relative humidity.
(d)None of these

263.The maximum amount of water vapour the air can hold is

(a)latent heat.

(b)saturation vapour pressure.

(c)supersaturated.

(d)relative humidity.

264.The most distinguishing feature between Aw andAf climate is

(a)the difference between annual temperature variation.

(b)the seasonal character of rainfall.

(c)the amount of rainfall.

(d)the difference between daily temperature variations.

265.The most significant atmospheric window exists between the wavelengths


of

(a)0.3 and 0.4 micrometres.

(b)1.5 to 1.7 micrometres.

(c)8 and 12 micrometres.

(d)32 and 35 micrometres.

266.The normal lapse rate of temperature in the troposphere is 1 °C for every

(a)146 m.
(b)156 m.

(c)166 m.

(d)176 m.

[UPSC-1998]

267.The percentage of radiant energy reflected back by surface is called

(a)albedo.

(b)greenhouse effect.

(c)isolation.

(d)refraction.

[UPSC-2001]

268.The process whereby light bounces back from an object at the same angle
and intensity at which it encounters the surface is called

(a)absorption.

(b)reflection.

(c)scattering.

(d)backscattering.

269.The progressive wave theory regarding the origin of tides was put forth by

(a)R.A. Harris.

(b)Isaac Newton.
(c)William Whewell.

(d)G.B. Airy.

[UPSC-2006]

270.The radiation emitted by Earth is in form of

(a)shortwave radiation.

(b)long wave infrared radiation

(c)visible light.

(d)None of these

271.The region with grasslands, extensive rolling plains, dull uniformity and
monotony of scene, no trees extreme cold and dryness in winter, can be
described as having

(a)Steppe type of climate.

(b)Tundra type of climate.

(c)China type of climate.

(d)warm temperate eastern margin type of climate.

[UPSC-1995]

272.The regions under the influence of cyclones will not generally produce air
masses because

(a)air will stay over the region influenced by cyclones for a considerable
period of time.
(b)the winds are constantly bringing air with unlike temperature and
humidity properties.

(c)humidity in such areas is high.

(d)All of these

273.The seasonal reversal of pressure and wind over a large continent, known
as monsoon circulation, is characterized by

(a)wind blowing towards the poles.

(b)wind blowing towards the ocean in the winter and towards the
continental interior in the winter.

(c)wind blowing towards the Equator.

(d)wind blowing towards the continental interior in the summer and towards
the ocean in the winter.

274.The seasonal temperature changes occur due to

(a)variation in distance between Earth and Sun.

(b)the rotation of the Earth.

(c)the variation in angle of the Sun above the horizon.

(d)the variation in the axis of Earth.

275.The temperature and rainfall of a meteorological station are given below


Average temperature: 12.8°C

Average Rainfall: 45.9 cm per annum

Identify the region having the above climatic patterns from amongst the
following:

(a)Mediterranean region

(b)Monsoon region

(c)Steppe region
(d)N.W. European region

[UPSC-2001]

276.The temperature decrease in the troposphere is termed the

(a)normal lapse rate.

(b)thermocline.

(c)environmental lapse rate.

(d)tropocline.

277.The thermal concept of origin of monsoons was first propounded by

(a)Chang-Chia Chengch.

(b)Halley.

(c)Nieuwolt.

(d)Herbert Smith.

278.The thermal upper air inversion is caused by

(a)horizontal or vertical movement of air.

(b)presence of ozone layer.

(c)temperate cyclones.

(d)non-temperate cyclones.

279.The thick clouds


1.may help in increasing the temperature in night time.

2.may help in increasing the temperature in day time.

3.may help in decreasing the temperature in night time.

(a)1 and 2

(b)2 and 3

(c)3 and 1

(d)1, 2 and 3

280.The tropical cyclones do not occur close to the Equator because of

(a)weak Coriolis force.

(b)light and variable winds.

(c)excessive humidity.

(d)convective activity

[UPSC-2006]

281.The true zonal pattern(s) of pressure exists

(a)in the Northern Hemisphere.

(b)in the Southern Hemisphere.

(c)in both Southern and Northern hemispheres.

(d)neither in Southern nor in Northern hemispheres.


282.The wavelength of the radiation is inversely proportional to the absolute
temperature of the radiation. This is known as

(a)Wien's Displacement Law.

(b)Stefan-Boltzmann Law.

(c)Planck's Law.

(d)Kirchhoff's Law.

283.The wind speeds in jet streams range from

(a)50 to 100 km per hour.

(b)100 to 300 km per hour.

(c)200 to 400 km per hour.

(d)250 to 500 km per hour.

284.The winds in high-pressure centres are called

(a)geostrophic winds.

(b)gradient winds.

(c)cyclones.

(d)anti-cyclones.

285.The worldwide jet stream that occurs in winter above the troposphere
resulting from a very steep stratospheric thermal gradient is the

(a)Arctic jet stream.


(b)subpolar jet stream.

(c)Poplar-night jet stream.

(d)subtropical jet stream.

[UPSC-1998]

286.Thermo pause occurs at the approximate height of

(a)80 km.

(b)100 km.

(c)1,000 km.

(d)None of these

287.Thunderstorms originate from

(a)nimbostratus clouds.

(b)altocumulus clouds.

(c)stratus clouds.

(d)cumulonimbus clouds.

[UPSC-2000]

288.To a large extent, the degree of scattering is determined by

1.the wavelength of the radiation received.

2.the size of intervening gas molecules.

3.the size of intervening dust particles.


(a)1 and 2

(b)2 and 3

(c)3 and 1

(d)None of these

289.Traim Basin of Sinkang experiences strong heat wave causing discomfort


during early spring. The wind is popularly known as

(a)karaburan.

(b)katabatic.

(c)chinook.

(d)fohn.

[UPSC-2001]

290.Tropical cyclones do NOT have

(a)low-pressure centres.

(b)different rainfall cells.

(c)almost same temperature in different parts.

(d)mobility.

291.Vertical changes in temperature, pressure, wind and humidity are recorded


by

(a)barographs.
(b)radiosondes.

(c)aerometers.

(d)climoprobes.

292.Warm mid-latitude climates have seasonal variations of insolation that


profoundly influence temperature, which is a result of

(a)the tilt of the Earth's axis.

(b)hilly terrain.

(c)the distribution of oceans.

(d)El Nino.

293.Warming in the Earth's stratosphere is mainly caused by

(a)release of latent heat energy during condensation.

(b)frictional heating caused by meteorites.

(c)chemical reaction between ozone and chlorofluorocarbon.

(d)absorption of solar radiation by ozone.

[UPSC-2003]

294.What are chinooks?

(a)Cold local winds which blow from north-west to south-east direction in


Spain.

(b)Extremely dry winds blowing from north-east and east to west in eastern
parts of Sahara desert.
(c)Warm and dry winds which blow down the eastern slopes of Rocky
Mountains of North America.

(d)Warm and dry winds along the northern slopes of Alps Mountains.

[UPSC-2009]

295.What are the causes for adiabatic temperature changes in the atmosphere?

(a)Convection and advection

(b)Latent heat of condensation

(c)Expansion and compression of the air

(d)Partial absorption of solar radiation by the atmosphere

[UPSC-2005]

296.What best explains the movement of heat from tropical areas towards the
poles?

(a)Polar fronts

(b)Orographic fronts

(c)Specific heat

(d)Advection

297.What climate region is designated as humid and tropical'?

(a)A

(b)B
(c)C

(d)D

298.What is the correct sequence of the following zonal pressure belts'?

1.Subpolar low

2.Equatorial low

3.Polar high

4.Subtropical high

Selectthe correct answer using the codes given below:

(a)2, 3, 1, 4

(b)1, 4, 2, 3

(c)2, 4, 1, 3

(d)1,3,2,4

[UPSC-2004]

299.What is the difference between a fog and a cloud?

(a)Place of formation

(b)Their appearance

(c)Their structure

(d)There is no difference
300.When a descending air contracts and its volume decreases, what happens
to its temperature'?

(a)Its temperature decreases.

(b)Its temperature increases

(c)Its temperature remains constant.

(d)Its temperature first increases, then decreases.

[UPSC-2009]

301.When atmospheric CO2 dissolves in seawater, it forms

(a)chloroform

(b)carbonic acid

(c)carbolic acid

(d)nitric acid

302.When climbing a mountain, what would you predict about temperatures'?

(a)The temperature will begin to drop above 5,000 feet.

(b)The temperature will drop about 3.5°F for every 1,000 feet climbed.

(c)The temperature will drop about 5.6°F for every 1,000 feet climbed.

(d)The temperature will be the same as the temperature at the base of the
mountain.

303.Where would higher levels of potential evapotranspiration occur on an


average?
(a)Tropics

(b)Mid-latitudes

(c)Continental interiors

(d)Polar regions

304.Which area receives the most seasonal variation in incoming radiation'?

(a)Equator

(b)Tropics

(c)Low latitudes

(d)High latitudes

305.Which climate is influenced by the ITCZ most of the year'?

(a)Marine westcoast

(b)Desert

(c)Humid tropical

(d)Polar

306.Which gas, contributed in part by burning fossil fuels, is used by scientists


as an indicator of future global warming'?

(a)Hydrogen

(b)Nitrogen

(c)Oxygen
(d)CO2

307.Which gaseous element composes the greatest percentage of the air'?

(a)Hydrogen

(b)Nitrogen

(c)Oxygen

(d)Carbon dioxide

308.Which of the following are functions of aerosols'?

1.They absorb solar radiation.

2.They absorb ultraviolet rays.

3.They act as a surface on which water vapours can condense.

(a)1 and 2

(b)2 and 3

(c)3 and 1

(d)1, 2 and 3

309.Which of the following are prerequisites for advectional inversion of


temperature'?

1.Strong wind movement

2.Unstable atmosphere

3.Snow-covered ground
(a)1 and 2

(b)2 and 3

(c)3 and 1

(d)1, 2 and 3

310.Which of the following are significant absorbers in the atmosphere of


incoming solar radiation'?

1.Nitrogen

2.Oxygen

3.Ozone

4.Water vapours

(a)1, 2 and 3

(b)2, 3 and 4

(c)3, 4 and 1

(d)None of these

311.Which of the following are significant absorbers in the atmosphere of


Earth's longwave radiation'?

1.Nitrogen

2.Oxygen

3.Carbon dioxide
4.Ozone

5.Water vapours

(a)1, 3 and 5

(b)1, 2 and 4

(c)2, 3 and 4

(d)III and V

312.Which of the following are source region of polar continental air masses'?

1.Northern Eurasia

2.Australia

3.North Atlantic

4.Antarctica

Select the correct answer using the codes given below.

(a)1 and 2

(b)2 and 3

(c)1 and 4

(d)2 and 4

[UPSC-2004]

313.Which of the following atmospheric conditions helps in formation of


tornados'?
(a)Absolute stability

(b)Neutral equilibrium

(c)Conditional instability

(d)Mechanical instability

314.Which of the following classification of the atmosphere is not based on


vertical variation in temperature'?

(a)Troposphere

(b)Heterosphere

(c)Stratosphere

(d)Mesosphere

315.Which of the following climates would you expect to find in coastal


southern California?

(a)Mediterranean

(b)Humid continental

(c)Desert

(d)Humid tropical

316.Which of the following conditions characterizes the equatorial climates of


the world'?

1.Cold winter

2.Hot summer
3.Afternoon thunder shower

4.Small diurnal range of temperature

Select the correct answer using the codes given below:

(a)1 and 3

(b)2 and 4

(c)1 and 2

(d)3 and 4

[UPSC-1996]

317.Which of the following conditions must be met for radiation inversion to


occur'?

(a)Winter nights must be short.

(b)Sky must be cloudless and clear.

(c)There must be high movement of air.

(d)There must be absence of snow on the ground.

318.Which of the following conditions must be satisfied for a source region for
the originating air mass to be considered as cold air mass?

(a)Temperature must be very low.

(b)Specific humidity must be very low.

(c)Normal lapse of temperature must be low.


(d)All of these.

319.Which of the following conditions must be satisfied for convectional


rainfall to occur?

1.Abundant supply of moisture so that relative humidity becomes high.

2.Intense heating of ground surface

3.Freezing temperature in clouds

(a)1 and 2

(b)2 and 3

(c)3 and 1

(d)1, 2 and 3

320.Which of the following criteria must be met for a source region of air
masses'?

(a)It must be extensive and physically uniform area.

(b)It must have a highly irregular topology.

(c)It must have a surface consisting of both land and water.

(d)It must not be situated over desert.

321.Which of the following criteria must be met for a source region of air
masses'?

1.The area must be characterized by a general stagnation of atmospheric


circulation.
2.The area must not be under the influence of anti-cyclones.

(a)Only 1

(b)Only 2

(c)Both 1 and 2

(d)Neither 1 nor 2

322.Which of the following factors plays a relatively small role in the amount
of solarenergy received at a particular place on the Earth'?

(a)The season of year

(b)Distance of the Earth and the Sun

(c)The length of day

(d)The angle of the Sun

323.Which of the following in not a greenhouse gas'?

(a)Carbon dioxide

(b)Methane

(c)Nitrous oxide

(d)Argon

[UPSC-2003]

324.Which of the following is also known as `belt of calm"?

(a)Equatorial low-pressure belt


(b)Subtropical high-pressure belt

(c)Subpolar low-pressure belt

(d)Polar high-pressure belt

325.Which of the following is also known as horse latitude'?

(a)Equatorial low-pressure belt

(b)Subtropical high-pressure belt

(c)Subpolar low-pressure belt

(d)Polar high-pressure belt

326.Which of the following is normally known as savannah climate'?

(a)Af

(b)Aw

(c)Cw

(d)Csa

327.Which of the following is NOT considered when classifying climates'?

(a)Unique weather events

(b)Terrain

(c)Weather patterns over several years

(d)Vegetation
328.Which of the following pairs are correctly matched'?

1.Cwa: Hot summer, dry winter

2.Cwb: Hot, dry summer

3.Csb: Warm, dry summer

4.Cfb: Warm summer

Select the correct answer using the codes given below:

(a)1, 2, 3 and 4

(b)1, 3 and 4

(c)1 and 4

(d)2 and 3

[UPSC-1996]

329.Which of the following sequences of clouds in ascending order would one


meet 500 meters and above the mean sea level'?

(a)Stratus, nimbus, cirrostratus, cirrus

(b)Nimbus, cirrostratus, cirrocumulus, stratus

(c)Cirrostratus, stratus, nimbus, cumulonimbus

(d)Stratus, cirrostratus, cumulonimbus, cirrocumulus

[UPSC-2009]

330.Which of the following sources of energy power the Earth's system'?


1.Sun

2.Heat from Earth's interior

3.Energy generated by oceans

(a)1 and 2

(b)2 and 3

(c)3 and 1

(d)1, 2 and 3

331.Which of the following spheres does not have a well-defined upper limit'?

(a)Stratosphere

(b)Mesosphere

(c)Troposphere

(d)Thermosphere

332.Which of the following spheres is also known as weather sphere'?

(a)Stratosphere

(b)Mesosphere

(c)Troposphere

(d)Thermosphere

333.Which of the following spheres is the largest in size?


(a)Atmosphere

(b)Geosphere

(c)Hydrosphere

(d)Biosphere

334.Which of the following statement holds true for the cold air mass after it
moves from its source region to other region?

(a)The usual visibility in the air mass increases.

(b)The temperature of the area which it reaches starts decreasing.

(c)Specific humidity becomes very low.

(d)All of these.

335.Which of the following statement if true raises the best objections to


concept of thermal origin of monsoon'?

(a)They produce rain.

(b)The low-pressure centres developed do not remain stationary at their


places for some time.

(c)ITC is static.

(d)None of these.

336.Which of the following statement is true regarding the effect of friction on


airflow?
(a)At higher heights, the effect is less but never small enough to be
disregarded.

(b)The effect can be disregarded above a height of few kilometres.

(c)At higher heights, the effect is more.

(d)None of these.

337.Which of the following statements about the clouds is NOT true'?

(a)Cirrus clouds are indicative of dry weather.

(b)Cirrus clouds are composed of tiny ice crystals.

(c)Cirrocumulus is a common type of high clouds.

(d)Cirrostratus clouds cause halos to form around the Sun and the Moon.

338.Which of the following statements is true for relative humidity'?

(a)It cannot reach 100 per cent.

(b)It cannot be more than 100 per cent.

(c)It cannot be more than 75 per cent.

(d)None of these.

339.Which of the following statements is true'?

1.The layers of air are lighter near the Earth's surface.

2.The marine environment moderates the weather conditions of the coastal


areas.
(a)Only 1

(b)Only 2

(c)Both 1 and 2

(d)Neither 1 nor 2

340.Which of the following statements are true'?

1.The Sun rays penetrate up to the depth of 5 feet on land.

2.The specific amount of heat required to raise the temperature of water is


more than what is required to raise the temperature of equal amount of
land.

3.The albedo of oceanic water is more than albedo of land.

4.Evaporation helps in cooling the water in the oceans of the Earth.

5.Sunlight is slowly redistributed by conduction because land surface is


static.

(a)1, 3 and 4

(b)1, 2, 4 and 5

(c)3, 4 and 5

(d)1, 2 and 5

341.Which of the following statements are true?

1.In open systems, energy moves in and out freely.

2.In closed systems, energy moves in and out freely.


3.In open systems, matter moves in and out freely.

4.In closed systems, matter moves in and out freely.

(a)1 and3

(b)2 and 4

(c)1, 2 and 3

(d)2, 3 and 4

342.Which of the following statements is true?

(a)High dew point corresponds to dry air.

(b)High relative humidity corresponds to dry air.

(c)Hygrometer is used to measure dew point.

(d)Psychrometer is used to measure relative humidity.

343.Which of the following statements holds true for albedo?

1.It varies from place to place.

2.It varies from time to time.

3.The albedo for the Earth as a whole is 30 per cent.

(a)1 and 2

(b)2 and 3

(c)3 and 1
(d)1, 2 and 3

344.Which of the following statements holds true for Group D in Koppen's


system of climate classification'?

(a)Average temperature of the coldest month is - 3°C or below.

(b)Average temperature in warmest month is 18°C or below.

(c)Average temperature of the warmest month is - 10°C.

(d)Average temperature of the coldest month is - 20°C or below.

345.Which of the following statements holds true for stratosphere'?

(a)There are contrasting opinions about its height and thickness.

(b)A few scientists believe that this layer is isothermal.

(c)The lower portion of this layer having maximum concentration of ozone


is called ozonosphere.

(d)All of these.

346.Which of the following statements holds true for the heat balance of the
Earth?

(a)Ocean currents and winds equalize latitudinal energy imbalances.

(b)There is a net gain of energy at higher latitudes.

(c)There is a net gain of energy at both lower and higher latitudes.

(d)Winds are the only significant means of equalizing latitudinal


imbalances.
347.Which of the following statements holds true for the incoming solar
radiations'?

(a)Atmosphere blocks 95 per cent of the radiation reaching Earth.

(b)Atmosphere is almost transparent to the incoming solar radiation.

(c)Atmosphere reflects back 50 per cent of the incoming solar radiation.

(d)None.

348.Which of the following statements holds true for the radiation inversion'?

(a)It can extend up to half a mile in the low-latitude areas.

(b)The inversion disturbs the stability of lower portion of the atmosphere.

(c)It causes dense fogs.

(d)The duration of the inversion decreases polewards.

349.Which of the following statements hold true for the scattering of sunlight'?

1.Gas molecules more effectively scatter shorter wavelengths of visible


light.

2.Gas molecules more effectively scatter longer wavelengths of visible


light.

3.Large particles scatter light more equally in all wavelengths.

(a)Only 1

(b)Only 3

(c)1 and 3
(d)2 and 3

350.Which of the following statements holds true for the solar radiation
received by Earth'?

(a)The shorter the length, the higher the wave frequency.

(b)Electromagnetic radiations travel at the approximate speed of 3,00,000


km per second.

(c)They travel approximately 150 million km to reach Earth.

(d)All of these.

351.Which of the following statements hold true for the temperate cyclones?

(a)After their formation, they move in westerly direction under the


influence of westerly winds.

(b)The pressure difference between the centre and the periphery is generally
from 10 mb to 20 mb.

(c)They are sometimes large enough to cover the area of 10,00,000 square
kilometres.

(d)All of these.

352.Which of the following statements holds true for tropical cyclones?

(a)They move with very high velocity over the land.

(b)They are comparatively of almost same size.

(c)They are regular and uniform.


(d)They move with very high velocity over the ocean.

353.Which of the following statements holds true for tropopause?

(a)It is approximately 5-kilometre thick.

(b)The height of tropopause changes with seasons.

(c)The word tropopause literally means `zone or region of mixing'.

(d)All of these.

354.Which of the following statements holds true for wind'?

(a)Wind is the result of vertical difference in air pressure.

(b)Wind is the result of horizontal difference in air pressure.

(c)The air pressure is highest in vacuum due to absence of gravity.

(d)None of these.

355.Which of the following statements holds true for wind'?

(a)Pressure differences are not influenced by temperature variations.

(b)Pressure gradient force is the only force acting on wind.

(c)Greater temperature differences will result in higher speed of wind.

(d)All of these.

356.Which of the following statements holds true regarding the temperature in


the wet tropics (Af, Am)'?

(a)The annual temperature range is high.


(b)The monthly temperature is high.

(c)The daily temperature range is high.

(d)None of these.

357.Which of the following statements holds true regarding the wet tropics
(Af, Am)?

(a)These areas lay outside the influence of ITCZ.

(b)The total precipitation rarely increases beyond 200 centimetres.

(c)The annual variability in rain is higher than annual variability in


temperature.

(d)All of these.

358.Which of the following statements holds true with regard to Tricellular


Meridional Circulation'?

1.There is cellular circulation of air at each meridian.

2.Surface winds flow from highpressure areas to low-pressure areas.

3.In upper atmosphere, the winds move opposite to the direction of surface
winds.

(a)1 and 2

(b)2 and 3

(c)3 and 1

(d)1, 2 and 3
359.Which of the following statements holds true for temperature gradient'?

1.The temperature gradient is insignificant from the tropics towards poles.

2.The temperature gradient is significant from the tropics towards poles.

3.The temperature gradient is significant between the tropics of Cancer and


Capricorn.

4.The temperature gradient is insignificant between the tropics of Cancer


and Capricorn.

(a)1 and 3

(b)1 and 4

(c)2 and 3

(d)2 and 4

360.Which of the following statements is NOT true about the upper air
westerly jet streams'?

(a)They are located at an approximate height of 12 km.

(b)They are located in troposphere.

(c)They are bifurcated due to the obstruction of Himalayas.

(d)All of these.

361.Which of the following statements is/are true?

1.Climate is an aggregate of weather conditions.

2.Weather is an aggregate of climate conditions.


3.The nature of both weather and climate is expressed in terms of same
basic elements.

(a)1 and3

(b)2 and 3

(c)Only 1

(d)Only 2

362.Which of the following statements is true about precipitation'?

(a)Over the oceans precipitation exceeds evaporation.

(b)Over the land mass (excluding poles), the precipitation exceeds


evaporation.

(c)North America receives precipitation equivalent to six times the water


transported by all the rivers of the North America together.

(d)All of these.

363.Which of the following statements is true for equatorial low-pressure belt'?

(a)They are not thermally induced.

(b)This zone is not stationary.

(c)This zone does not represent the zone of convergence of NE and SE trade
winds.

(d)The conditions in this zone are never calm.

364.Which of the following statements is true for feedback of natural systems'?


1.Negative feedback mechanism works to maintain status quo.

2.Positive feedback mechanism works to maintain status quo.

3.Most of the Earth's systems contain wide variety of both negative and
positive feedback systems.

(a)Only 1

(b)Only 2

(c)1 and 3

(d)2 and 3

365.Which of the following statements is true with regard to hydrologic cycle?

(a)The water balance is a quantitative view of the hydrologic cycle.

(b)The water balance is qualitative view of the hydrologic cycle.

(c)The hydrologic cycle is predominantly powered by Earth's radiation.

(d)The hydrologic cycle is not influenced by winds.

366.Which of the following statements is/are true for the atmospheric


pressure?

1.It will be higher in areas where convergence occurs.

2.It will be higher in areas where divergence occurs.

3.Humid air will be lighter.

(a)Only 1
(b)Only 2

(c)1 and 3

(d)2 and 3

367.Which of the following typically does NOT cause summer thunderstorms'?

(a)Orographic uplift

(b)Frontal uplift

(c)Adiabatic cooling

(d)Convection

368.Which one of the following climates is found in coastal areas of California


in North America, central Chile in South America and south-west tip of
West Australia'?

(a)Tropical savannah

(b)Mediterranean

(c)Humid continental

(d)Low-latitude steppe

[UPSC-2007]

369.Which one of the following is the correct sequence in terms of descending


values ofAlbedo?

(a)Forest, snow, water

(b)Snow, forest, water


(c)Water, snow, forest

(d)Forest, water, snow

[UPSC-1998]

370.Which one of the following is the correct sequence of cloud forms in


ascending order of their heights from the ground'?

(a)Cirrus, cumulus, stratus

(b)Cumulus, stratus, cirrus

(c)Stratus, cirrus, cumulus

(d)Stratus, cumulus, cirrus

[UPSC-1999]

371.Which one of the following layers of the atmosphere is responsible for the
deflection of radio waves'?

(a)Troposphere

(b)Stratosphere

(c)Mesosphere

(d)Ionosphere

[UPSC-1996]

372.Which one of the following pairs is correctly matched'?

(a)Cs: Mediterranean
(b)Am: N.W.Europe

(c)DW: Tropical monsoon

(d)ET: Siberia

[UPSC-1999]

373.Which one of the following precipitation conditions is not characteristic of


frontal rainfall type'?

(a)Precipitation falls during the passage of a depression on zones facing the


passage.

(b)It occurs in the middle and high latitudes.

(c)It is distinguished from the cyclonic rain because of the prolonged


drizzling rain which falls from the warm front of the depression.

(d)Cloud burst with longer duration of rain is associated with the squally
passage of the cold front.

[UPSC-2000]

374.Which one of the following provides the force needed to drive the
atmospheric circulation'?

(a)Higher biotic content of the tropical latitudes and lower biotic content of
the polar latitudes

(b)The energy contrasts between high insulation polar latitudes.

(c)Higher rotational speed of lower latitudes and lower rotational speed of


higher latitudes
(d)Equatorial radius of the Earth is longer than the polar radius.

[UPSC-2005]

375.Which one of the following regions has a Mediterranean type of climate'?

(a)Cape Town region of South Africa

(b)Great Plains of North America

(c)North-eastern Australia

(d)Southern Algeria

[UPSC-2004]

376.Which one of the following regions receives the rainfall throughout the
year due to westerly winds'?

(a)South-west Australia

(b)Iran and Iraq

(c)South Africa

(d)South Chile

[UPSC-2002]

377.Which one of the following statements is correct'?

Hurricanes

(a)form at the Equator.

(b)are not as large as tornadoes.


(c)develop over warm ocean areas.

(d)tend to intensify when they move over land surface.

[UPSC-2004]

378.Which one of the following statements is correct'?

Collision-coalescence process of precipitation is applicable to

(a)clouds which extend beyond freezing level.

(b)all type of clouds.

(c)cumulonimbus clouds.

(d)those clouds which do not extend beyond the freezing level.

[UPSC-2005]

379.Which one of the following statements is correct'?

The innermost region of a hurricane is known as hurricane eye where

(a)temperature is abnormally low and pressure is the lowest.

(b)sky is clear and temperature is the lowest.

(c)temperature is abnormally high and pressure is the lowest.

(d)sky is under dense cloud cover and pressure is the lowest.

[UPSC-2005]

380.Which one of the following weather conditions is indicated by a sudden


fall in barometer reading'?
(a)Stormy

(b)Calm weather

(c)Cold and dry weather

(d)Hot and sunny weather

[UPSC-2001]

381.Which side of China, South America, the USA and Australia has the driest
climate'?

(a)East

(b)West

(c)North

(d)South

382. White frost occurs

(a)when dew is frozen due to low temperature.

(b)when dew point of the air is below freezing.

(c)when stream fog fails to evaporate.

(d)none of these.

383.Who introduced the term anti-cyclone for the first time?

(a)Fitzroy

(b)J. Bjerknes
(c)F. Galton

(d)V. Bjerknes

[UPSC-2009]

384.Winds are deflected by, caused by the rotation of Earth.

(a)the Milankovitch cycles

(b)the Coriolis effect

(c)the greenhouse effect

(d)gyres

385.Winds blowing at a constant speed parallel to curved isobars are called

(a)geostrophic winds.

(b)gradient winds.

(c)cyclones.

(d)anti-cyclones.

386.With reference to conventional type of rainfall, consider the following


statements:

1.It is a warm weather phenomenon.

2.This rainfall is associated with cumulonimbus clouds.

Which of the statements given above is/ are correct'?

(a)1 only
(b)2 only

(c)Both 1 and 2

(d)Neither 1 nor 2

[UP S C -2 009]

387.With reference to equatorial climate, consider the following statements:

1.Thermally induced low-pressure belt develops around the Equator but the
pressure gradient is very low.

2.Temporal variability in the amount of rainfall is not found.

Which of the statements given above is/ are correct'?

(a)1 only

(b)2 only

(c)Both 1 and 2

(d)Neither 1 nor 2

[UPSC-2009]

388.With reference to Koppen's classification of climate types, consider the


following statements:

1.The minor/sub type indicated through small letter `f' means rains all the
year round.

2.The minor/sub type indicated through small letter `s' means dry/summer.

Which of the statements given above is/ are correct'?


(a)1 only

(b)2 only

(c)Both 1 and 2

(d)Neither 1 nor 2

[UPSC-2006]

389.With reference to the subtropical pressure belt, consider the following


statements:

1.It is affected by Earth's rotation and descent of winds from higher


altitudes.

2.It is dynamically induced and characterized with anti-cyclonic conditions

Which of the statements given above is/ are correct'?

(a)1 only

(b)2 only

(c)Both 1 and 2

(d)Neither 1 nor 2

[UPSC-2009]

390.World Environment Conference was held in December 1997 at

(a)Stockholm.

(b)Riode Janeiro.
(c)Paris.

(d)Kyoto.

[UPSC-1998]

391.Condensation of water vapour in the atmosphere on particles of dust leads


to

(a)fog.

(b)frost.

(c)hail.

(d)cloud formation.

[UGC D-2005]

392.The mechanism by which energy of solar origin is transported around the


Earth is

(a)ocean waves.

(b)winds.

(c)ocean tides.

(d)tornado.

[UGC D-2005]

393.Idealize global pattern of surface wind from Equator to poles is

(a)doldrum, trade wind, westerlies, easterlies.


(b)doldrum, trade wind, easterlies, westerlies.

(c)doldrum, westerlies, trade wind, easterlies.

(d)doldrum, easterlies, trade wind, westerlies.

[UGC D-2005]

394.Atmospheric pressure at sea level is

(a)1,034 millibars/sq cm.

(b)1,024 millibars/sq cm.

(c)1,014 millibars/sq cm.

(d)1,044 millibars/sq cm.

[UGC D-2005]

395.Given below two statements, one labelled as Assertion (A) and the other
labelled as Reason (R). Select your answer from the codes given.

Assertion (A): The temperature of the stratosphere increases with altitude.

Reason (R): The stratosphere is heated largely through the absorption by


ozone molecules of ultraviolet radiation from the Sun.

(a)Both A and R are true and R is the correct explanation of A.

(b)Both A and R are true but R is not the correct explanation of A.

(c)A is true but R is false.

(d)A is false but R is true.


[UGC D-2005]

396.Given below are two statements, one labelled as Assertion (A) and the
other labelled as Reason (R). Select your answer from the codes given.

Assertion (A): The troposphere is the dense lower part of the atmosphere in
which metrological phenomenon such as clouds and storms occur.

Reason (R): In the troposphere, temperature decreases with altitude.

(a)Both (A) and (R) are true and (R) is the correct explanation of (A).

(b)Both (A) and (R) are true but (R) is not a correct explanation of (A).

(c)(A) is true but (R) is false.

(d)(A) is false but (R) is true.

[UGC D-2006]

397.What is recorded with the help of a hydrograph?

(a)Wind velocity

(b)Vapour retention

(c)Relative humidity

(d)Water discharge

[UGC D-2007]

398.The increase in the chlorofluorocarbon in the atmosphere is associated


with

(a)ozone depletion.
(b)sea level rise.

(c)aquatic life depletion.

(d)intense sonic pollution.

[UGC D-2007]

399.Surface inversion of temperature is a phenomenon related to

(a)increase in temperature with increase in elevation.

(b)decrease in temperature with increasing elevation.

(c)decrease in temperature with decreasing elevation.

(d)increase in temperature with decreasing elevation.

[UGC D-2007]

400.Who had used the concept of evapotranspiration in climate classification'?

(a)Thornthwaite

(b)Koppen

(c)Kendrew

(d)Austin Miller

[UGC D-2007]

401.Heat budget refers to

(a)solar radiation.

(b)balance between incoming and outgoing radiations.


(c)temperature distribution over the continents.

(d)heat distribution over land and sea.

[UGC D-2008]

402.Match List I with List II and select the correct answer from the codes
given.

Codes:

[UGC D-2008]

403.Given below are two statements, one labelled as Assertion (A) and the
other labelled as Reason (R). Select your answer from the codes given.

Assertion (A): The water droplets are found to be high in the troposphere.

Reason (R): The temperature variation in the troposphere is not alarming


compared to stratosphere.

(a)Both (A) and (R) are true and (R) is correct explanation of (A).
(b)Both (A) and (R) are true, but (R) is not correct explanation of (A).

(c)(A) is true but (R) is false.

(d)(A) is false but (R) is true.

[UGC D-2008]

404.The winter position of subtropical jet stream fluctuates between

(a)20° and 35°N.

(b)35 ° and 65 °N.

(c)5° and 20°N.

(d)30° and 50°N.

[UGC D-2008]

405.Given below are two statements, one labelled as Assertion (A) and the
other labelled as Reason (R). Select your answer from the codes given.

Assertion (A): Stable atmospheric condition prevails when environmental


lapse rate is less than dry and wet adiabatic lapse rates.

Reason (R): The parcel of air is warmer than the surrounding environment
at all the levels as it ascends.

(a)Both (A) and (R) are true and (R) is the correct explanation of (A).

(b)Both (A) and (R) are true, but (R) is not the correct explanation of (A).

(c)(A) is true but (R) is false.

(d)(A) is false but (R) is true.


[UGC D-2009]

406.The mature stage of thunderstorm is characterized by

(a)strong updrafts and downdrafts.

(b)lack of electrical charges.

(c)end of precipitation.

(d)lack of ice crystals.

[UGC D-2009]

407.Match List 1 with List II and select the correct answer using codes given.

Codes:
[UGC D-2009]

408.The idealized global pattern of surface wind from the Equator to pole is

(a)doldrums, trade winds, westerlies, easterlies.

(b)doldrums, westerlies, trade winds, easterlies.

(c)doldrums, easterlies, trade winds, westerlies.

(d)doldrums, trade winds, easterlies, westerlies.

[UGC D-2009]

409.The marine influence upon climate results into

(a)increase in annual rainfall.

(b)increase in aridity.

(c)increase in annual temperature range.

(d)decrease in annual temperature range.

[UGC D-2009]

410.Given below are two statements, one labelled as Assertion (A) and the
other labelled as Reason (R) Select your answer from the codes given.

Assertion (A): Ozone layer protects the Earth from excessive quantities of
severely changing solar radiation.

Reason (R): Ozone hole has been discovered over the continent of
Antarctica.

(a)Both (A) and (R) are correct and (R) explains (A)
(b)Both (A) and (R) are correct but (R) does not explain (A)

(c)(A) is correct but (R) is wrong

(d)(A) is wrong but (R) is correct

[UGC J-2005]

411.Given below are two statements, one labelled as Assertion (A) and the
other labelled as Reason (R). Select your answer from the codes given.

Assertion (A): Continental tropical air masses are dry and hot.

Reason (R): Continental tropical air masses are not localized.

(a)Both (A) and (R) are correct and (R) explains (A)

(b)Both (A) and (R) are correct but (R) does not explain (A)

(c)(A) is correct but (R) is wrong

(d)(A) is wrong but (R) is correct.

[UGC J-2005]

412.Which one of the following is not a greenhouse gas?

(a)Nitrogen

(b)Carbon dioxide

(c)Methane

(d)Chlorofluorocarbon

[UGC J-2005]
413.In which of the following weather conditions does the negative lapse rate
of temperature operate?

(a)Surface inversion of temperature

(b)Adiabatic change of temperature

(c)Isothermal change of temperature

(d)Advectional inversion of temperature

[UGC J-2005]

414.Seasonal contrasts are maximum in

(a)low latitudes.

(b)mid-latitudes.

(c)high latitudes.

(d)subtropics.

[UGC J-2005]

415.The polar front is associated with

(a)origin of tropical cyclones.

(b)origin of wind belts.

(c)origin of temperate cyclones.

(d)origin of tornadoes.

[UGC J-2005]
416.Global pattern of surface temperature is generally controlled by

(a)latitude, longitude and altitude.

(b)latitude, longitude and continentocean contrast.

(c)continent ocean contrast, longitude and altitude.

(d)continent ocean contrast, latitude and altitude.

[UGC J-2007]

417.The criteria used by Thornthwaite in his classification of world climates


are

1.precipitation effectiveness.

2.thermal efficiency.

3.potential evapotranspiration.

4.radiation efficiency.

Select your answer from the codes given.

(a)1, 2 and 4

(b)1, 2 and 3

(c)1, 3 and 4

(d)2, 3 and 4

[UGC J-2007]

418.The Earth's average albedo is estimated as


(a)32 per cent

(b)40 per cent

(c)21 per cent

(d)50 per cent

[UGC J-2007]

419.Which of the following types of clouds occupy the lowest layer'?

(a)Stratus

(b)Cumulus

(c)Cumulonimbus

(d)Cirrus

[UGC J-2008]

420.Pollution plume is the result of

(a)a regional wind caused by a pressure gradient.

(b)unequal distribution of temperature.

(c)abnormalities caused by latent heat of condensation.

(d)interaction among different types of air masses.

[UGC J-2008]

421.Polar frontal theory is associated with

(a)origin of trade wind tract.


(b)origin of tropical cyclones.

(c)origin of tornadoes.

(d)origin of jet stream.

[UGC J-2008]

422.Given below are two statements one labelled as Assertion (A) and the
other labelled as Reason (R). Select your answer from the codes given.

Assertion (A): The equatorial area at which the trade winds converge is
known as Intertropical Convergence Zone (ITCZ).

Reason (R): The ITCZ usually lies at or near the meteorological Equator
(doldrums).

(a)Both (A) and (R) are true and (R) is correct explanation of (A).

(b)Both (A) and (R) are true, but (R) is not correct explanation of (A).

(c)(A) is true, but (R) is false.

(d)(A) is false, but (R) is true.

[UGC J-2011]

423.Cirrus clouds are at about a height of

(a)0-1.5 km.

(b)1-3 km.

(c)4-6 km.

(d)8-14 km.
[UGC J-2011]

424.The standard air pressure at the sea level is

(a)1,010.25 mb.

(b)1,013.25 mb.

(c)1,015.25 mb.

(d)1,017.25 mb.

[UGC J-2011]

425.Match List I with List II and select the correct answer using the codes
given.

Codes:
[UGC J-2011]

426.Which of the following factors is responsible for the planetary wind belts?

(a)The tilt of Earth's axis

(b)The rotation of Earth

(c)High - and low-pressure belts

(d)Differential rate of heating of land and water

[UGC J-2010]

427.The correct sequence of Koppen's climatic type from Nile delta towards
Congo basin is

(a)BWh-BSh-Aw-A£

(b)Aw-Af-BSh-BWh.

(c)BWh-Aw-BWh-A£

(d)Af-Aw-BSh-BWh.

[UGC J-2010]

428.Given below are two statements, one is labelled as Assertion (A) and the
other is labelled as Reason (R). Select your answers from the codes given.
Assertion (A): There is a little mixing of air between the troposphere and
stratosphere.

Reason (R): The stratosphere holds large volumes of water vapour dust.

(a)Both (A) and (R) are true and (R) is the correct explanation of (A).

(b)Both (A) and (R) are true, but (R) is not the correct explanation of (A).

(c)(A) is true, but (R) is false.

(d)(A) is false, but (R) is true.

[UGC J-2010]

429.Which of the following area's observation motivated Davis to develop his


concept on `Cycle of Erosion"?

(a)Missouri

(b)Grinoco

(c)Montana

(d)Red River

[UGC J-2010]

430.The position of thermal Equator is normally at

(a)0°.

(b)5°N.

(c)5°S.
(d)10°N.

[UGC D-2011]

431.Which of the following is not relevant to Ice Age hypothesis?

(a)Thermal contraction theory

(b)Carbon dioxide hypothesis

(c)Ozone depletion hypothesis

(d)Volcanic dust hypothesis

[UGC D-2011]

432.Given below are two statements, one labelled as Assertion (A) and other
labelled as Reason (R). Select your answer from the codes given.

Assertion (A): Pressure in the troposphere falls with increasing height.

Reason (R): The upper atmosphere has relatively low temperature.

(a)Both (A) and (R) are correct and (R) is correct explanation of (A).

(b)Both (A) and (R) are true, but (R) is not correct explanation of (A).

(c)(A) is true, but (R) is false.

(d)(A) is false, but (R) is true.

[UGC D-2011]

433.Given below are two statements, one labelled as Assertion (A) and the
other labelled as Reason (R). Select your answer from the codes given.
Assertion (A): The Indian monsoon is influenced by El Nino.

Reason (R): At the occurrence of El Nino, there develops a high-pressure


area along the coast of Peru.

(a)Both (A) and (R) are true and (R) is correct explanation.

(b)Both (A) and (R) are true, but (R) is not correct explanation of (A).

(c)(A) is true, but (R) is false.

(d)(A) is false, but (R) is true.

[UGC D-2011]

434.Given below are two statements, one labelled as Assertion (A) and the
other labelled as Reason (R). Select your answer from the codes given.

Assertion (A): A characteristic feature of the tropical cyclone is its central


eye.

Reason (R): Tropical cyclone is not associated with cloud-free vortex


produced by the intense spiralling of the storm.

(a)Both (A) and (R) are true and (R) is the correct explanation of (A).

(b)Both (A) and (R) are true, but (R) is not the correct explanation of (A).

(c)(A) is false, but (R) is true.

(d)(A) is true, but (R) is false.

[UGC D-2010]

435.Rating curve is useful for the estimation of


(a)river discharge.

(b)stream velocity.

(c)dissolved load.

(d)hydraulic radius.

[UGC D-2010]

436.Sublimation describes the direct transition from

(a)solid to vapour.

(b)vapour to solid.

(c)liquid to vapour.

(d)vapour to liquid.

[UGC D-2010]

437.The savannah biome is usually associated with

[UGC D-2010]

(a)tropical wetdry climate.

(b)equatorial wet climate.

(c)tropical dry climate.

(d)monsoon climate.

438.The only substance that can exist as a gas, liquid and solid at common
Earth temperatures is
(a)carbon.

(b)nitrogen.

(c)oxygen.

(d)water.

439.Water constantly circulates through

(a)the atmosphere.

(b)all four spheres.

(c)the biosphere.

(d)the lithosphere.

440.What is the process that converts liquid water in lakes, oceans and streams
into vapour, delivering it to the atmosphere?

(a)Evaporation

(b)Advection

(c)Precipitation

(d)Runoff

441.Water

(a)is not a very good solvent.

(b)exists in three different states on Earth.

(c)is less abundant on Earth than on Venus.


(d)requires very little energy to change from one state to another.

442.Which of the following would be logical sequential step for a drop of


water in the hydrologic cycle'?

(a)Evapotranspiration, storage, runoff, precipitation

(b)Precipitation, runoff, storage, evapotranspiration

(c)Storage, evapotranspiration, runoff, precipitation

(d)Runoff, precipitation, evapotranspiration, storage

443.Drawing water from the soil by plant roots is known as

(a)transpiration.

(b)condensation.

(c)evaporation.

(d)infiltration.

444.Evapotranspiration occurs faster in

(a)windy conditions.

(b)warm, windy and dry conditions.

(c)warm conditions.

(d)dry conditions.

445.Evapotranspiration

(a)includes the processes of plants.


(b)decreases immensely in the summer.

(c)takes water vapour from the atmosphere and stores it in the soil.

(d)takes place only in the winter

446.Carbon and hydrogen are NOT

(a)important elements for sustaining life.

(b)key factors in the hydrologic cycle.

(c)the major components in fossil fuels.

(d)major components of the foods that plants produce (carbohydrates) and


that animals consume.

447.Deforestation (cutting and burning) is known to cause

(a)a decrease of carbon in the atmosphere.

(b)an increase in forested land.

(c)relatively large amounts of carbon to enter the atmosphere.

(d)long-term storage of carbon in the soil.

448.Which one of the following statements is correct?

(a)Sea level drops when evaporation rates increase over the oceans and
when this extra atmospheric moisture falls on land as rain.

(b)Sea level rises when water is added to the oceans through increased
rainfall and increased inflow from rivers.
(c)Sea level rises when water is stored in expanding ice sheets and
continental glaciers.

(d)Sea level drops when water is stored in expanding ice sheets and
continental glaciers.

ANSWER KEYS
 
1.A class of animals known as marsupials is a characteristic feature of

(a)Africa.

(b)Australia.

(c)South America.

(d)Southeast Asia.

[UPSC-2001]

2.Assertion (A): During the neap tides, the high tide is lower and the low
tide is higher than usual.

Reason (R): The neap tide, unlike the spring tide, occurs on new moon
instead of on full moon.

(a)Both (A) and (R) are true and (R) is the correct explanation of (A).

(b)Both A and (R) are true but (R) is not the correct explanation of (A).

(c)(A) is true but (R) is false.

(d)(A) is false but (R) is true.

[UPSC-2001]

3.Assertion (A): There are no tea plantations in any African country.


Reason (R): Tea plants need fertile soil with high humus.

(a)Both (A) and (R) are true and (R) is the correct explanation of (A).

(b)Both A and (R) are true but (R) is not the correct explanation of (A).

(c)(A) is true but (R) is false.

(d)(A) is false but (R) is true.

[UPSC-2007]

4.Assertion (A): Unlike temperate forest, the tropical rainforest, if cleared,


can yield productive farmland that can support intensive agriculture for
several years even without chemical fertilizers.

Reason (R): The primary productivity of the tropical rainforest is very


high when compared to that of temperate forests.

(a)Both (A) and (R) are true and (R) is the correct explanation of (A).

(b)Both A and (R) are true but (R) is not the correct explanation of (A).

(c)(A) is true but (R) is false.

(d)(A) is false but (R) is true.

[UPSC-2003]

5.Coffee cultivation in Sri Lanka was abandoned because of the disease

(a)leaf blight.

(b)leaf spot.
(c)leaf rust.

(d)rut.

[UPSC-1995]

6.Consider the following ecosystems:

1.Taiga

2.Tropical evergreen

3.Tropical deciduous

4.Tundra

The correct sequence in decreasing order of the albedo values of these


ecosystems is

(a)1, 4, 3, 2.

(b)4, 1, 3, 2.

(c)4, 1, 2, 3.

(d)1, 4, 2, 3.

[UPSC-2002]

7.Consider the following regions:

1.Eastern Himalayas

2.Eastern Mediterranean region


3.North-western Australia

Which of the above is/are Biodiversity Hotspots?

(a)1 only

(b)2 only

(c)2 and 3

(d)1, 2 and 3

[UPSC-2009]

8.Consider the following statements:

1.The annual range of temperature is greater in the Pacific Ocean than


that in the Atlantic Ocean.

2.The annual range of temperature is greater in the Northern Hemisphere


than that in the Southern Hemisphere.

Which of the statements given above is/ are correct'?

(a)1 only

(b)2 only

(c)Both 1 and 2

(d)Neither 1 nor 2

[UPSC-2007]

9.Dalbergia species is associated with which one of the following'?


(a)Cashew nut

(b)Coffee

(c)Tea

(d)Rosewood

[UPSC-2007]

10.Estuaries possess distinct blooms of excessive growth of pigmented


dinoflagellates. These blooms are called

(a)red tides.

(b)sea tides.

(c)black tides.

(d)sea flowers.

[UPSC-1998]

11.Given below are two statements one labelled as Assertion (A) and the
other labelled as Reason (R). Select your answers from the codes given.

Assertion (A): Mangroves are very specialized forest ecosystem of


tropical and subtropical regions bordering certain sea coasts.

Reason (R): They stabilize the shoreline and act as bulwark against
encroachments by sea.

(a)Both (A) and (R) are true and (R) is the correct explanation of (A).

(b)Both A and (R) are true but (R) is not the correct explanation of (A).
(c)(A) is true but (R) is false.

(d)(A) is false but (R) is true.

[UPSC-1996]

12.Match List I with List II and select the correct answer using the codes
given.

Codes:

[UPSC-1999]

13.Open-stunted forest with bushes and small trees having long roots and
sharp thorns or sharp thorns or spines are commonly found in

(a)eastern Orissa.

(b)north-eastern Tamil Nadu.

(c)Siwaliks and Terai regions.


(d)western Andhra Pradesh.

[UPSC-2002]

14.Out of all the Biosphere Reserves in India, four have been recognized on
the World Network by UNESCO. Which one of the following is not one
of them?

(a)Gulf of Mannar

(b)Kanchenjunga

(c)Nanda Devi

(d)Sunderbans

[UPSC-2008]

15.The alpine vegetation in the western Himalayas is found only up to a


height of 3,000 metres, while in the Eastern Himalayas it is found up to a
height of 4,000 metres. The reason for this variation in the same mountain
range is that

(a)eastern Himalayas are higher than western Himalayas.

(b)eastern Himalayas are nearer to the Equator and sea coast than western
Himalayas

(c)eastern Himalayas get more monsoon rainfall than the western


Himalayas.

(d)the rocks of eastern Himalayas are more fertile than the rocks of
western Himalayas.
[UPSC-1995]

16.The largest coral reef in the world is the found near the coast of which
one of the following countries?

(a)Australia

(b)Cuba

(c)Ghana

(d)Philippines

[UPSC-2007]

17.Where is Bundala Biosphere Reserve which has been recently added to


the UNESCO's Man and Biosphere (MAB) network located'?

(a)Russia

(b)India

(c)Sri Lanka

(d)Bangladesh

[UPSC-2006]

18.Which one of the following countries is the first country in the world to
propose a carbon tax for its people to address global warming'?

(a)Australia

(b)Germany
(c)Japan

(d)New Zealand

[UPSC-2006]

19.Which one among the following covers the highest percentage of forest
area in the world'?

(a)Temperate coniferous forests

(b)Temperate deciduous forests

(c)Tropical monsoon forests

(d)Tropical rainforests

[UPSC-2003]

20.Which one of the following does not belong to biosphere reserves set-up
so far'?

(a)Great Nicobar

(b)Sunderbans

(c)Nanda Devi

(d)Gulf of Kutch

[UPSC-1995]

21.Which one of the following factors is responsible for the charge in the
regular direction of the ocean currents in the Indian Ocean'?
(a)Indian Ocean is half an ocean.

(b)Indian Ocean has monsoon drift.

(c)Indian Ocean is a land-locked ocean.

(d)Indian Ocean has greater variation in salinity.

[UPSC-1997]

22.Which one of the following is not essentially a species of the Himalayan


vegetation'?

(a)Juniper

(b)Mahogany

(c)Silver fir

(d)Spruce

[UPSC-2008]

23.Which one of the following regions of India is now regarded as an


`ecological hotspot"?

(a)Western Himalayas

(b)Eastern Himalayas

(c)Western Ghats

(d)Eastern Ghats

[UPSC-1996]
24.Which one of the following statements is not correct'?

(a)Gulf with narrow fronts and wider rears experience high tides.

(b)Tidal currents take place when a gulf is connected with the open sea by
a narrow channel.

(c)Tidal bore occurs when a tide enters the narrow and shallow estuary of
a river.

(d)The tidal nature of the mouth of the river Hooghly is of crucial


importance to Kolkata as port.

[UPSC-2004]

25.Find out the sequence of vegetation zones from Equator to pole.

(a)Taiga, tundra, selva, savannah

(b)Selva, savannah, taiga, tundra

(c)Savannah, taiga, selva, tundra

(d)Tundra, taiga, savannah, selva

[UGC J-2011]

26.Given below are two statements, one labelled as Assertion (A) and the
other labelled as Reason (R). Select your answers from the codes given.

Assertion (A): The sea water is more saline in tropical areas.

Reason (R): Salinity of seawater is dependent on temperature and fresh-


water mixing.
(a)(A) is true, (R) is false.

(b)(A) is false, (R) is true.

(c)Both (A) and (R) are true.

(d)Both (A) and (R) are wrong.

[UGC J-2011]

27.Which of the following is different from the other three on the basis of
their locations?

(a)Canaries current

(b)Benguela current

(c)Guinea current

(d)Labrador current

[UGC J-2011]

28.Red Sea is an example of

(a)synclinal valley.

(b)volcanic structure.

(c)rift valley.

(d)eroded valley.

[UGC J-2010]

29.Most of the salt in the sea water is derived from


(a)seafloors.

(b)atmosphere.

(c)remains of sea life.

(d)continents.

[UGC D-2005]

30.Plants that behave as xerophyte at one season and as hydrophyte at


another are known as

(a)heterotrophytes.

(b)tropophytes.

(c)ornniphytes.

(d)tropornniphytes.

[UGC D-2005]

31.In open ocean, the difference in height between high and low tide is

(a)1.0 metre.

(b)1.5 metre.

(c)0.5 metre.

(d)2.5 metre.

[UGC D-2005]

32.Which one of the following is not a woody plant'?


(a)Liana

(b)Herb

(c)Shrub

(d)Tree

[UGC D-2005]

33.Match List I with List II and select the correct answer from the codes
given.

Codes:

[UGC D-2005]

34.Which one of the following oceans currents is called Gulf Stream of the
Pacific?

(a)Oyashio
(b)Kuroshio

(c)Aleutian

(d)Kurile

[UGC D-2005]

35.The average time interval between two successive high tides is

(a)6 hours, 13 minutes.

(b)12 hours, 26 minutes.

(c)24 hours, 50 minutes.

(d)18 hours, 39 minutes.

[UGC D-2006]

36.Assertion (A): The ocean floors are much younger than continents.

Reason (R): Owing to their low density and consequent buoyancy, the
ocean floors are not forced down into the mantle in subduction zone.

(a)Both (A) and (R) are true and (R) is the correct explanation of (A).

(b)Both (A) and (R) are true and (R) is not a correct explanation of (A).

(c)(A) is true but (R) is false.

(d)(A) is false but (R) is true.

[UGC D-2006]

37.Consider the following seas:


1.Red Sea

2.Black Sea

3.Dead Sea

4.Baltic Sea

[UGC D-2006]

The correct sequence of those seas in descending order of salinity is

(a)1, 2, 3, 4

(b)2, 1, 4, 3

(c)4, 2, 3, 1

(d)3, 1, 2, 4

38.A plant adapted to a wet environment is called

(a)hygrophyte.

(b)xerophyte.

(c)mesophyte.

(d)tropophyte.

[UGC D-2006]

39.Ecological succession normally leads to

(a)habitat.
(b)biome.

(c)climatic frontier.

(d)climax.

[UGC D-2006]

40.Eustatic changes in the sea levels are caused by

(a)ice ages.

(b)epirogenic movements.

(c)orogenic movements.

(d)isostatic changes.

[UGC D-2007]

41.Which of the following is a coral island?

(a)Tasmania

(b)Lakshadweep

(c)Maldives

(d)Rameswaram

[UGC D-2007]

42.Which one of the following is Taiga biome?

(a)Sub-Arctic biome
(b)Sub-Saharan biome

(c)Savannah grass

(d)None of the above.

[UGC D-2007]

43.Match List I with List II and select the correct answer from the codes
given.

Codes:

[UGC D-2007]
44.Urban environmental degradation is caused by various factors

1.Suspended Particulate Matter (SPM)

2.Polluted water

3.Carbon dioxide

4.Vehicular emission

Choose the correct answer

(a)1, 2, 3 and 4

(b)2 and 4

(c)2 and 3

(d)1 and 4

[UGC D-2007]

45.The soil-water budget was developed by

(a)Thornthwaite.

(b)Kendrew.

(c)Koppen.

(d)Austin Miller.

[UGC D-2008]

46.The correct sequence of the great oceans in term of area is


(a)Indian, Pacific, Arctic, Atlantic.

(b)Pacific, Atlantic, Indian, Arctic.

(c)Indian, Pacific, Atlantic, Arctic.

(d)Arctic, Indian, Atlantic, Pacific.

[UGC D-2008]

47.Bay of Bengal is a

(a)large marine ecosystem.

(b)small marine ecosystem.

(c)a subset of small marine ecosystem.

(d)a subset of large coastal ecosystem.

[UGC D-2008]

48.Ozone depletion may lead to

(a)water shortage.

(b)forest fire.

(c)occurrence of skin cancer.

(d)resurgence of waterborne diseases.

[UGC D-2008]

49.Plants that grow under high intensity of disturbance and low intensity of
stress are called
(a)competitors.

(b)stress tolerators.

(c)coniferous forests.

(d)ruderals.

[UGC D-2009]

50.Match List 1 with List II and select the correct answer by using the codes
given.

Codes:

[UGC D-2009]

51.Given below are two statements, one labelled as Assertion (A) and the
other labelled as Reason (R). Select your answer from the codes given.

Assertion (A): Water temperature over the ocean decreases with depth.
Reason (R): The decrease in temperature is well marked from 370 to 730
m.

(a)Both (A) and (R) are true and (R) is the correct explanation of (A).

(b)Both (A) and (R) are true but (R) is not the correct explanation of (A).

(c)(A) is true but (R) is false.

(d)(A) is false but (R) is true.

[UGC D-2009]

52.Given below are two statements one labelled as Assertion (A) and the
other labelled as Reason (R). Select your answer from the codes given.

Assertion (A): The Indian Ocean currents have a circulation pattern very
different from those of the Atlantic and Pacific oceans.

Reason (R): The Indian Ocean is influenced by the seasonal rhythm of the
monsoon winds and is landlocked in the north.

(a)Both (A) and (R) are correct and (R) explains (A).

(b)Both (A) and (R) are correct but (R) does not explain (A).

(c)(A) is correct but (R) is wrong.

(d)(A) is wrong but (R) is correct.

[UGC J-2005]

53. The convergence of ocean currents takes place in

(a)equatorial region.
(b)tropical region.

(c)polar region.

(d)mid-latitude regions.

[UGC J-2005]

54.The North equatorial current in the northern Atlantic Ocean and the
South equatorial current in the southern Atlantic Ocean are caused by

(a)centrifugal force at the equator.

(b)trade winds.

(c)Coriolis force.

(d)centripetal force towards the poles.

[UGC J-2005]

55.Given below are two statements, one labelled as Assertion (A) and the
other labelled as Reason (R). Select your answer from the codes given.

Assertion (A): Ferrel's Law explains the clockwise direction of ocean


currents in the Northern Hemisphere.

Reason (R): Coriolis force is responsible for anticlockwise direction of


ocean currents in the Southern Hemisphere.

(a)Both (A) and (R) are correct and (R) explains (A).

(b)Both (A) and (R) are correct but (R) does not explain (A).

(c)(A) is correct but (R) is wrong.


(d)(A) is wrong but (R) is correct.

[UGC J-2005]

56.Match List I with List II and select the correct answer from the codes
given.

[UGC J-2005]

57.Which of the following arrangements of biomes in order of availability of


soil, water and heat is correct?

(a)Savannah, grassland, forest, tundra and desert

(b)Savannah, forest, grassland, tundra and desert

(c)Tundra, forest, grassland, savannah and desert

(d)Forest, savannah, grassland, desert and tundra

[UGC J-2007]

58.Which one of the following is known as the `Carbon Sink' of the globe?
(a)Pacific Ocean

(b)Equatorial rainforests

(c)Antarctica

(d)Savannah

[UGC J-2007]

59.Deep seafloor comprises which of the following deposits?

(a)Muds and sands

(b)Sands and gravels

(c)Muds and gravels

(d)Muds and red clay

[UGC J-2007]

60.Which one of the following is the most important air pollutant?

(a)Carbon dioxides

(b)Sulphur dioxides

(c)Hydrocarbons

(d)SPM

[UGC J-2007]

61.Siachen glacier is located in


(a)Greater Himalayas.

(b)Trans Himalayas.

(c)Lesser Himalayas.

(d)sub-Himalayas.

[UGC J-2007]

62.The ocean current Benguela is noticed in

(a)Pacific Ocean.

(b)Indian Ocean.

(c)Atlantic Ocean.

(d)Arctic Ocean.

[UGC J-2008]

63.Acid rain is caused by

(a)excess carbon dioxide in the atmosphere.

(b)washout of sulphuric acid by precipitation.

(c)excess in greenhouse gas emission from tropical forests.

(d)abnormal variations in composition of gases in the atmosphere.

[UGC J-2008]

64.Atoll is one of the stages of


(a)a natural reservoir.

(b)a pressure system.

(c)a coral development.

(d)development of the carbon sink in the ocean.

[UGC J-2008]

65.The most prominent features on the ocean floor are t

(a)lava plateaus.

(b)deepocean trenches.

(c)seamounts.

(d)oceanic ridges.

66.The shells of numerous marine microorganisms are composed of

(a)H2504

(b)CO2

(c)NaCl

(d)CaCO3

67.When does a deep-water wave change to a shallow water wave'?

(a)Only when the wavelength is about one-half the water depth.

(b)Only when the wavelength is about twice the water depth.


(c)Only when the wave period is greater than one-half the water depth.

(d)Only when the wave period is greater than twice the water depth.

68.Submarine canyons found on the continental slope and rise are believed
to have been created

(a)by rivers during the Ice Age

(b)because of a plate plunging into the mantle

(c)by faulting

(d)none of these

69.Which one of the following would most likely be covered with thick
turbidite layers?

(a)Axial rift zone of an active midocean spreading centre

(b)Deepsea fan at the base of a continental slope

(c)Ocean floor around a conical-shaped seamount

(d)Upper part of a steep, narrow submarine canyon

70.Which of the following is NOT true of deep ocean trenches'?

(a)They are long and narrow depressions.

(b)They are geologically very stable.

(c)They are sites where plates plunge back into the mantle.

(d)They may act as sediment traps.


71. Seamounts

(a)are volcanoes that form on the ocean floor

(b)are a special type of oceanic trench

(c)form only in the Pacific Ocean basin

(d)are submarine canyons found near Australia

72.`Black smokers' are associated with

(a)oceanic ridges

(b)hot water

(c)metal-rich sulphide deposits

(d)all of these

73.marks the edge of a continent.

(a)The centre of an abyssal plain

(b)The lowest part of the continental slope

(c)An offshore barrier island system

(d)The highest part of the continental shelf

74.The gently sloping submerged surface extending from the shoreline


towards the deep ocean is termed the

(a)submarine canyon
(b)continental rise

(c)continental shelf

(d)continental slope

75.Which one of the following is not connected in any way with submarine,
hot spring vents?

(a)Black smokers on a mid-ocean ridge

(b)Ecological communities living without photosynthesis

(c)Sediment rich in metallic sulphides

(d)Thick turbidites on the continental rise

76.Which one of the following would typically have the widest continental
shelf?

(a)A tectonically passive mid-ocean ridge and trench system

(b)A tectonically active continental margin next to a deep ocean trench

(c)A tectonically passive trailing continental margin

(d)All of the above would roughly equal continental shelves.

77. areo not associated with a midcean ridge.

(a)Very thin ocean floor, sediment layer

(b)Deep ocean trenches

(c)Submarine basaltic lava eruptions


(d)Shallower depths than abyssal plains

78.Seafloor spreading occurs along relatively narrow areas at the crests of


oceanic ridges called

(a)submarine canyons

(b)rift zones

(c)trenches

(d)seamounts

79.Oceanic ridges are elevated compared to the surrounding ocean floor


because

(a)newly formed lithosphere is hotter and therefore less dense than the
surrounding rocks

(b)the mantle is pushing up the lithosphere along the ridge

(c)the older, colder lithosphere is less dense and tends to rise

(d)of the shield volcanoes that develop due to seafloor spreading

80.Where in the oceans are biological communities thriving without


sunlight'?

(a)Around fissure vents for flood basalts

(b)Around seafloor hot spring vents

(c)In shallow water coral reefs


(d)In muds of deep ocean trenches

81.Which one of the following statements concerning submarine canyons is


not true'?

(a)They extend from the continental shelf to the base of the continental
slope.

(b)They channel turbidity currents down the continental slope to deeper


waters.

(c)They were cut by streams and rivers during the Pleistocene when sea
level had dropped to the base of the continental slope.

(d)They generally conned across the continental shelf to specific river


valleys on land.

82.develop where oceanic lithosphere bends downwards and sinks into the
mantle.

(a)Rift valleys on mid-ocean ridges

(b)Deep ocean trenches

(c)Abyssal seamounts

(d)Submarine canyons

83.Spreading rates of are common along the mid-Atlantic and the mid-
Indian ridges.

(a)1 to 5 metres per year

(b)1 to 5 kilometres per year


(c)1 to 5 centimetres per year

(d)1 to 5 millimetres per year

84.All of the following are layers that comprise oceanic crust except for

(a)gabbro

(b)turbidites

(c)pillow basalts

(d)sheeted dike complexes

85.Basaltic rocks of the oceanic crust are altered by heated seawater moving
through fractures in a process known as

(a)contact metamorphism

(b)regional metamorphism

(c)diagenesis

(d)hydrothermal metamorphism

86.East Africa, the Rhine Valley and the Baikal in Siberia are all examples
of

(a)passive continental margins

(b)ophiolites

(c)continental rifts

(d)continental margins
87.is the oceanward edge of a continental shelf.

(a)The base of the continental rise

(b)The top of the continental slope

(c)The deepest portion of a deepocean trench

(d)The top of the abyssal plain

88.Plate subduction occurs because the , not the oceanic crust, is denser than
the underlying asthenosphere.

(a)seafloor sediment

(b)outer core

(c)trailing continent

(d)lithospheric mantle

89.The idea that rifting and dispersal of one large landmass is followed by a
long period during which the fragments are gradually reassembled into a
new single landmass is called

(a)the super continent cycle

(b)uniformitarianism

(c)plate assemblage

(d)plate tectonics

90.The continental rise is located


(a)at the top of a mid-ocean ridge

(b)at the seaward edge of a deep ocean trench

(c)at the top of the continental slope

(d)between an abyssal plain and continental slope

91.An echo sounder operates by measuring the time required for a

(a)sound pulse to travel from a ship to the seafloor and back

(b)radar beam to travel from a harbour patrol boat to a fuzz-buster on a


speeding yacht

(c)light beam to travel from a satellite at a known altitude to the sea


bottom and back

(d)radar beam to travel from a ship to the seafloor and back

92.Which one of the following concerning mid-ocean ridges is false?

(a)Terrigenous sediment coverings are very thin or absent.

(b)They are sites for submarine eruptions of basaltic lava.

(c)Sediments include thick siliceous ooze deposits and sandy turbidite


beds.

(d)They are where young lithosphere is added to the edges of spreading


oceanic plates.

93.Oceans cover approximately of Earth's surface area.


(a)50 per cent

(b)10 per cent

(c)70 per cent

(d)90 per cent

94.The Nino which influenced the Indian monsoon is

(a)a cold ocean current that flows northwards along the coast of Chile.

(b)a warm ocean current that flows westwards along the coast of Ecuador
and Peru.

(c)a low-pressure system over western coast of Spain.

(d)none of the above.

95.Salinity in ocean water decreases when

(a)evaporation is more.

(b)wind velocity is high.

(c)rainfall is heavy.

(d)humidity is high.

96.Assertion (A): Though Britain and Labrador have the same latitudinal
location, the climate of Britain is temperate while that of Labrador is cold.

Reason (R): Warm ocean currents skirt Britain while cold currents flow
along the Labrador coast.
(a)Both(A) and (R) are true and (R) is the correct explanation of (A).

(b)Both (A) and (R) are true but (R) is not the correct explanation of (A).

(c)(A) is true but (R) is false

(d)(A) is false but (R) is true.

97.Match the List I with List II and select the correct answer using the codes
given.

Codes:

98.Following relief zones are the characteristics of ocean basins:

1.Continental slopes

2.Continental shelf
3.Deep sea plain

4.Oceanic trench

The correct sequence of their seaward appearance from the coast is

(a)2, 1, 4, 3

(b)1, 2, 4, 3

(c)2, 1, 3, 4

(d)1, 2, 3, 4

99.Assertion (A): Corals are not found near the shores where rivers meet the
sea.

Reason (R): Corals do not thrive in regions where water contain a lot of
silt.

(a)Both (A) and (R) are true and (R) is the correct explanation of (A).

(b)Both (A) and (R) are true but (R) is not the correct explanation of (A).

(c)(A) is true but (R) is false.

(d)(A) is false but (R) is true.

100.Assertion (A): The temperature removed at the surface of enclosed seas


of high latitudes is lower than the temperature recorded at relatively
greater depth.

Reason (R): The surface of ocean directly receives insulation and heat is
transmitted to the lower part of the ocean through the process of
conduction.

(a)Both (A) and (R) are true and (R) is the correct explanation of (A).

(b)Both (A) and (R) are true but (R) is not the correct explanation of (A).

(c)(A) is true but (R) is false.

(d)(A) is false but (R) is true.

101.Which of the following factors supports the hypothesis of the


permanency of ocean basins and continents'?

1.The cross section of the ocean basin, comprising the broad abyssal
through flanked by rim like continental shelves.

2.Occurrence of shallow-water sands and muds is limited to the


continental shelf and slope, while that of pelagic oozes being limited to
the abyssal plain.

3.Absence of normal sedimentary rocks on the oceanic islands

Select the correct answer using the codes given below:

(a)2 and 3

(b)1 and 3

(c)1 and 2

(d)1, 2 and 3

102.Match the List I with List II and select the correct answer using the
codes given.
Codes:

103.Which one of the following statements regarding El Nino is NOT true'?

(a)It is an extension of the equatorial current towards the western coast of


South America.

(b)It is an occasional warm current leading to an increase of about 10°C


in surface water temperatures.

(c)It develops as a temporary replacement of the usual cold Peruvian


current.
(d)It causes an increase in planktons which thrive in the cold Peruvian
current.

104.Assertion (A): Average ocean salinity generally decreasing from


Equator to the pole tropical zone (20°-30°) is characterized by maximum
salinity.

Reason (R): The tropical zone owes its high salinity to high temperature,
high evaporation and significantly low rainfall.

(a)Both (A) and (R) are true and (R) is the correct explanation of (A).

(b)Both (A) and (R) are true but (R) is not the correct explanation of (A).

(c)(A) is true but (R) is false.

(d)(A) is false but (R) is true.

105.Match the List I with List II and select the correct answer using the
codes given.

Codes:
106.Consider the following conditions:

1.Anti-cyclone conditions with static air and high temperature

2.Cyclonic conditions with unstable air

3.High precipitation with low evaporation

4.Low precipitation with high evaporation

High salinity of ocean waters is associated with

(a)1 and 4.

(b)2 and 3.

(c)1 and 3.

(d)2 and 4.

107.Radiolarian oozes occur in the depths of

(a)600-1,200 fathoms.

(b)1,500-2,000 fathoms.

(c)2,000-5,000 fathoms.

(d)5,000-6,000 fathoms.
108.Consider the following factors:

1.Rotation of the Earth

2.Air pressure and wind

3.Ocean water density

4.Revolution of the Earth

Which of these factors modify ocean currents?

(a)1 and 2

(b)1, 2 and 3

(c)1 and 4

(d)2, 3 and 4

109.Tsunamis are produced by

(a)shrinking of the Earth's crust.

(b)cyclones.

(c)submarine earthquakes.

(d)tides.

110.Water bodies which have low concentration of nutrients are in

(a)littoral zones.

(b)oligotrophic areas.
(c)neritic province.

(d)benthic zone.

111.Assertion (A): Tropical seas in India are rich in fish species but there are
few important fishing grounds around the country.

Reason (R): The quantity of fish of a particular species is lacking and


hence not commercially exploitable.

(a)Both (A) and (R) are true and (R) is the correct explanation of (A).

(b)Both (A) and (R) are true but (R) is not the correct explanation of (A).

(c)(A) is true but (R) is false.

(d)(A) is false but (R) is true.

112.The layer of water in the oceans and lakes that separates the warmer
surface layer from the deeper colder layer is called

(a)epilimnion.

(b)hypolimnion.

(c)thermocline.

(d)hypothermia.

113.Consider the following statements:

1.Continental shelves are wider where high mountains are very close and
parallel to the coast.
2.Continental shelves are formed by prolonged deposition of detritus
under sea water in those areas where sea conditions are calm.

3.Extensive continental shelves are formed by marine erosion of the


continental margins during the time of negative change of the sea level.

4.Continental shelves are formed by the submergence of continental


margins due to tilting of land towards sea.

Which of the above statements are correct?

(a)1, 2 and 3

(b)2, 3 and 4

(c)1, 3 and 4

(d)1, 2 and 4

114.The first research project programme which focused on the El Nino-


Southern Oscillation and its predictability was called

(a)Climate Variability and Predictability.

(b)Global Energy and Water Cycle Experiment (GEWEX).

(c)Global Change and Terrestrial Ecosystem (GCTE).

(d)Tropical Ocean and Global Atmosphere (TOGA).

115.Assertion (A): The temperature in the Southern Hemisphere is


considerably lower than that in the Northern Hemisphere.
Reason (R): The large mass of ice-covered Antarctic continent is an
important source of cold in the Southern Hemisphere.

(a)Both (A) and (R) are true and (R) is the correct explanation of (A).

(b)Both (A) and (R) are true but (R) is not the correct explanation of (A).

(c)(A) is true but (R) is false.

(d)(A) is false but (R) is true.

116.Assertion (A): The Great Barrier Reef is located near Australia.

Reason (R): The coral polyps grow in shallow, saline and warm waters
grow.

(a)Both (A) and (R) are true and (R) is the correct explanation of (A).

(b)Both (A) and (R) are true but (R) is not the correct explanation of (A).

(c)(A) is true but (R) is false.

(d)(A) is false but (R) is true.

117.Assertion (A): Apart from large-scale climatic aberrations in the tropics,


El Nino has a disastrous consequence on the fishing industry of Peru.

Reason (R): El Nino leads to massive displacements of the rainfall


bringing droughts over large parts of the normal raining areas and
torrential rains to and regions of the tropics.

(a)Both (A) and (R) are true and (R) is the correct explanation of (A).

(b)Both (A) and (R) are true but (R) is not the correct explanation of (A).
(c)(A) is true but (R) is false.

(d)(A) is false but (R) is true.

118.Which one of the following combination is responsible for higher


salinity in ocean water'?

(a)High evaporation, high temperature and high rainfall

(b)High evaporation, high temperature and low rainfall

(c)Low evaporation, low temperature and high rainfall

(d)Low evaporations, low temperature and low rainfall

119.Which one of the following is not a factor modifying ocean current?

(a)Direction and shape of the coastline

(b)Tidal wave in the ocean

(c)Seasonal variations in winds

(d)Bottom topography

120.Consider the following statements:

The horizontal distribution of temperature of ocean water is largely


affected by

1.depth of water in the ocean.

2.ocean currents.

3.prevailing winds.
4.latitude.

Which of these are correct'?

(a)1, 2 and 3

(b)2, 3 and 4

(c)1, 2 and 4

(d)1, 2 and 4

121.Consider the following statements:

1.Hydrogenous deposits are formed very slowly.

2.Phosphorites are found either in the form of nodules or in the form of


thin crust.

3.Glauconitic is a bigamous sediment.

Which of these are correct'?

(a)land 2

(b)2 and 3

(c)1 and 3

(d)1, 2 and 3

122.Which one of the following is the correct system in the South Atlantic
Ocean?
(a)South equatorial current, Brazil current, Antarctic drift, Benguela
current

(b)Benguela current, Brazil current, South equatorial current, Antarctic


drift

(c)Antarctic drift, Brazil current, Benguela current, South equatorial


current

(d)South equatorial current, Benguela current, Brazil current, Antarctic


drift

123.Oceanic waves by an earthquake are known as

(a)S-waves.

(b)L-waves.

(c)P-waves

(d)tsunamis.

124.Pelagic deposits are mainly found below the depth of

(a)1,000 fathoms.

(b)100 fathoms.

(c)600 fathoms.

(d)200 fathoms.

125.The ocean relief is generally


(a)more diverse than that of the continents.

(b)more uniform than that of the continents.

(c)showing minor variations only.

(d)not much of a significance.

126.Consider the following statements:

1.Among the major oceans, Indian Ocean is warmer than the Atlantic and
Pacific oceans in both the hemispheres.

2.The average surface temperature of Pacific Ocean is higher than that of


the Atlantic Ocean in the Southern Hemisphere.

3.Among the major oceans, the Atlantic Ocean is the coldest in the
equatorial region.

Which of these statements are correct'?

(a)1 and 2

(b)2 and 3

(c)1 and 3

(d)1, 2 and 3

127.Consider the following statements:

Salinity of water in the equatorial oceans decrease because

1.large rivers like Amazon and Congo enter into the sea.
2.evaporation is less.

3.waters from Arctic and Antarctic mix with the water of the Equator.

4.of heavy rainfall.

Which of these statements are correct'?

(a)1 and 2

(b)3 and 4

(c)1 and 3

(d)1 and 4

128.Which one of the following sequence in the increasing order of salinity


concentration in their waters is correct'?

(a)Gulf of California, Baltic Sea, Red Sea, Arctic Ocean

(b)Baltic Sea, Arctic Ocean, Gulf of California, Red Sea

(c)Red Sea, Gulf of California, Arctic Ocean, Baltic Sea

(d)Arctic Ocean, Gulf of California, Baltic Sea, Red Sea

129.Oozes are associated with

(a)volcanic deposits.

(b)terrigenous deposits.

(c)pelagic deposits.
(d)coral reefs.

130.Match the List I with List II and select the correct answer using the
codes given.

Codes:

131.Which of the following is a warm current'?

(a)South Pacific current

(b)Kuroshio current

(c)Labrador current

(d)Rip current

132.Which of the following conditions are associated with El Nino


phenomena'?
(a)Heavy rains in South America and droughts in Australia

(b)Droughts in South America and heavy rains in Australia

(c)Heavy rains in both South America and Australia

(d)Droughts in both South America and Australia

133.Spring tide means

(a)higher high tide, higher low tide.

(b)higher high tide, lower low tide.

(c)lower high tide, lower low tide.

(d)lower high tide, higher low tide.

134.Consider the following statements:

1.Nearly 30 per cent of the petroleum production in the world is from the
offshore areas.

2.The exclusive economic zone in the oceans extends up to 100 nautical


miles from the coast.

3.Only less than 1 per cent of marine areas is declared as protected area.

4.The largest marine-protected area is the Great Barrier Reef.

Which of these statements are correct?

(a)l and 2

(b)1, 3 and 4
(c)1 and 4

(d)2 and 3

135.The favourable percentage of sea water salinity from the point of view
of fish production is

(a)2.0-2.5.

(b)2.5-3.0.

(c)3.0-3.5.

(d)3.5-4.0.

136.Assertion (A): The North Atlantic Ocean Route is the world's most
important ocean route.

Reason (R): The North Atlantic Ocean Route brings together the
developing and developed nations.

(a)Both (A) and (R) are true and (R) is the correct explanation of (A).

(b)Both (A) and (R) are true but (R) is not the correct explanation of (A).

(c)(A) is true but (R) is false.

(d)(A) is false but (R) is true.

137.Assertion (A): In the deep ocean, primary production occurs near


hydrothermal vents.

Reason (R): Over there, microbes produce food through chemosynthesis


which forms the basis for a food chain.
(a)Both (A) and (R) are true and (R) is the correct explanation of (A).

(b)Both (A) and (R) are true but (R) is not the correct explanation of (A).

(c)(A) is true but (R) is false.

(d)(A) is false but (R) is true.

138.Consider the following statements:

1.Where fold mountains run parallel and close to the coast, continental
shelf is narrow or absent.

2.The average depth for continental shelf is 100 metres.

3.Submarine canyons are mostly found in continental slope.

4.The submarine flat-topped mountains are called guyots.

Which of the statements given above are correct'?

(a)1, 2, 3 and 4

(b)1, 2 and 4

(c)1, 3 and 4

(d)2 and 3

139.Match the List I with List II and select the correct answer using the
codes given.
Codes:

140.The coastline formed by the submergence of mountain ridges running


parallel to the coast is known as

(a)Dalmatian coast.

(b)Ria coast.

(c)Fiord coast.

(d)Haff coast.

141.Coral reefs are formed by

(a)volcanic rocks.

(b)marine sediments.
(c)chlorine materials precipitated form sea water.

(d)tiny colonial marine animals which construct limestone skeleton


material.

142.Match the List I with List II and select the correct answer using the
codes given.

Codes:

143.Assertion (A): Oceanic salinity is higher along the tropics of Cancer and
Capricorn.

Reason (R): Increased evaporation coupled with relatively more


insulation along these latitudes cause higher oceanic salinity.
(a)Both (A) and (R) are true and (R) is the correct explanation of (A).

(b)Both (A) and (R) are true but (R) is not the correct explanation of (A).

(c)(A) is true but (R) is false.

(d)(A) is false but (R) is true.

144.Assertion (A): A composite profile includes only the lowest parts of a


series of parallel profiles.

Reason (R): A composite profile is constructed to represent the overall


relief of an area as viewed form distance.

(a)Both (A) and (R) are true and (R) is the correct explanation of (A).

(b)Both (A) and (R) are true but (R) is not the correct explanation of (A).

(c)(A) is true but (R) is false.

(d)(A) is false but (R) is true.

145.Which one of the following sequence correctly indicates the increasing


order of the distance from the coast?

(a)Abyssal plain, continental shelf, continental slope, continental rise

(b)Continental shelf, continental slope, continental rise, abyssal plain

(c)Continental slope, continental shelf, abyssal plain, continental rise

146.Match the List I with List II and select the correct answer using the
codes given.
Codes:

147.Which one of the following sequences correctly represents the


percentage of given salts in sea water in decreasing order'?

(a)Magnesium chloride, sodium chloride, magnesium sulphate, calcium


sulphate

(b)Magnesium sulphate, magnesium chloride, calcium sulphate, sodium


chloride

(c)Sodium chloride, magnesium chloride, magnesium sulphate, calcium


sulphate
(d)sodium chloride, magnesium sulphate, magnesium chloride, calcium
sulphate

148.Match List I with List II and select the correct answer using the codes
given.

Codes:

149.Kuroshio is a warm ocean current which runs from

(a)Philippines to Japan.

(b)Indonesia to Philippines.

(c)Japan to China.

(d)Sri Lanka to Indonesia.


150.Assertion (A): The amount of salinity in the tropical zone is recorded
higher in the eastern than the western coasts of continents.

Reason (R): The trade wind causes up welling of water in the western
coasts and the water is piled up in the eastern coasts of continents.

(a)Both (A) and (R) are true and (R) is the correct explanation of (A).

(b)Both (A) and (R) are true but (R) is not the correct explanation of (A).

(c)(A) is true but (R) is false.

(d)(A) is false but (R) is true.

151.Match List I with List II and select the correct answer using the codes
given.

Codes:
[UPSC-1995]

152.Which one of the following statements is not true of coniferous forests'?

(a)Spruce is the important tree of this jungle which is used in preparing


pulp and artificial silk.

(b)They are evergreen forests of soft wood trees with short and pointed
leaves.

(c)Such jungles are also called `boreal'.

(d)Tree trunks of this region are quite thick and are covered with thick
barks so as to protect them from the dryness of the summer season.

[UPSC-1995]

153.Match List I with List II and select the correct answer using the codes
given.

Codes:
[UPSC-1995]

154.Match List I with List II and select the correct answer using the codes
given.

Codes:

[UPSC-1996]

155.Assertion (A): The boundary marking the termination of tree growth is


known as the timber line.

Reason (R): Its location is determined by the amount of moisture,


exposure, evaporation and depth of snow.
(a)Both (A) and (R) are true and (R) is the correct explanation of (A).

(b)Both (A) and (R) are true but (R) is not the correct explanation of (A).

(c)(A) is true but (R) is false.

(d)(A) is false but (R) is true.

[UPSC-1997]

156.Match List I with List II and select the correct answer using the codes
given.

Codes:

[UPSC-1997]

157.Match List I with List II and select the correct answer using the codes
given.
Codes:

-1-1 [UPSC-1997]

158.The daily and annual range of temperature is maximum in the

(a)Savannah grassland.

(b)temperate grassland.

(c)hot desert.

(d)equatorial region.

[UPSC-1997]

159.Which of the following leaf modifications occur in the desert areas to


inhibit water loss?

1.Hard and waxy leaves


2.Tiny leaves or no leaves

3.Drip-tip leaves

4.Thorns instead of leaves

Select the correct answer using the codes given.

(a)1, 2 and 3

(b)2 and 3

(c)1, 2 and 4

(d)1 and4

[UPSC-1998]

160.Soils which have undergone gleying and are associated with marshes,
swamps or poorly drained upland are called

(a)calcimorphous soils.

(b)hydro-orphic soils.

(c)halomorphic soils.

(d)sierozems.

[UPSC-1999]

161.Which one of the following factors caused the recent depletion of


rainforests in Indonesia?

(a)Growth of forest-based industries


(b)Encroachment by urban areas

(c)Forest fire in peat-bogs in marshy areas

(d)Lack of proper management of forests

[UPSC-1999]

162.Which one of the following commercially important groups of trees


belongs to cool temperate hardwood species'?

(a)Maple, mahogany and oak

(b)Mahogany, ebony and maple

(c)Oak, popular and maple

(d)Ebony, oak and popular

[UPSC-1999]

163.Assertion (A): Laterite soils are reddish in colour and poor in humus
and plant nutrients.

Reason (R): In humid tropical climate, excessive leaching takes place


under alternative and and humid conditions.

(a)Both (A) and (R) are true and (R) is the correct explanation of (A).

(b)Both (A) and (R) are true but (R) is not the correct explanation of (A).

(c)(A) is true but (R) is false.

(d)(A) is false but (R) is true.


[UPSC-1999]

164.Consider the following pairs of projects and related environmental


problems:

1.River Valley project: water body induced

2.Mining project: Groundwater and surface water pollution

3.Thermal power project: Degradation of land

Which of the above pairs is/are correctly matched'?

(a)1, 2 and 3

(b)2 and 3

(c)1 and 2

(d)1 alone

[UPSC-2000]

165.The endangered Ridley turtles have the world's largest aggregation in

(a)Gahirmatha.

(b)Sagarmatha.

(c)Lakshadweep islands.

(d)Andaman and Nicobar group of islands.

[UPSC-2000]
166.`Rally for Valley' programme in India was organized to highlight the
problem of

(a)environmental degradation.

(b)biodiversity.

(c)resettlement of displaced people.

(d)loss of agricultural land.

[UPSC-2000]

167.Match List I with List II and select the correct answer using the codes
given.

Codes:

[UPSC-2000]
168.Match List I with List II and select the correct answer using the codes
given.

Codes:

[UPSC-2000]

169.The state of inactivity adopted by desert animals during the hottest and
driest season is known as

(a)basal metabolism.

(b)dormancy.

(c)aestivation.

(d)hibernation.
[UPSC-2000]

170.Which one of the following sets of biomes represents the right sequence
in terms of increasing water deficiency?

(a)Tropical scrub, tropical savannah, desert

(b)Tropical savannah, mid-latitude grassland, desert

(c)Mediterranean woodland, mid-latitude grassland, mid-latitude


deciduous forest

(d)Desert, tropical savannah, tropical scrub

[UPSC-2000]

171.Bamboo and pine trees are generally found in which one of the
following types of climates'?

(a)Taiga

(b)Tropical monsoon

(c)Mediterranean

(d)Savannah

[UPSC-2000]

172.The correct sequence of the natural regions of Brazil from north to south
is

(a)Amazon Basin, Mato Grosso Plateau, Brazilian Highlands, Campos.

(b)Amazon Basin, Brazilian Highlands, Campos, Mato Grosso Plateau.


(c)Brazilian Highlands, Mato Grosso Plateau, Amazon Basin, Campos.

(d)Mato Grosso Plateau, Compos, Brazilian High lands, Amazon Basin.

[UPSC-2001]

173.In eastern Himalayas, the subtropical broad-leaved forests are


commonly found between the altitudes ranging from

(a)500 to 1,000 metres.

(b)750 to 1,000 metres.

(c)1,000 to 2,000 metres.

(d)2,000 to 2,500 metres.

[UPSC-2001]

174.Assertion (A): Oligotrophics lack in high concentration of nutrients


resulting in algae blooms.

Reason (R): A higher concentration of nutrients results in higher organic


productivity.

(a)Both (A) and (R) are true and (R) is the correct explanation of (A).

(b)Both (A) and (R) are true but (R) is not the correct explanation of (A).

(c)(A) is true but (R) is false.

(d)(A) is false but (R) is true.

[UPSC-2001]
175.The first research project programme which focused on the El Nino-
Southern Oscillation and its predictability was called

(a)Climate Variability and Predictability.

(b)Global change and Terrestrial Ecosystem.

(c)Tropical Ocean and Global Atmosphere.

(d)Tropical Ocean and Global Atmosphere.

[UPSC-2001]

176.Match List I with List II and select the correct answer using the codes
given.

Codes:

[UPSC-2002]
177.Assertion (A): The Tundra environment is characterized by a few
important climate and environmental elements.

Reason (R): The Tundra biomes are found in the Arctic or Alpine regions.

(a)Both (A) and (R) are true and (R) is the correct explanation of (A).

(b)Both (A) and (R) are true but (R) is not the correct explanation of (A).

(c)(A) is true but (R) is false.

(d)(A) is false but (R) is true.

[UPSC-2002]

178.Assertion (A): The habitat of date palms is warm deserts with little or no
rainfall.

Reason (R): Date palm is a tree of waterless desert.

(a)Both (A) and (R) are true and (R) is the correct explanation of (A).

(b)Both (A) and (R) are true but (R) is not the correct explanation of (A).

(c)(A) is true but (R) is false.

(d)(A) is false but (R) is true.

[UPSC-2002]

179.Assertion (A): Laterite, the typical soil of tropical region, is rich in


oxides of iron and aluminium compounds.

Reason (R): Heavy rainfall promotes leaching of lime and silica content
of the soil.
(a)Both (A) and (R) are true and (R) is the correct explanation of (A).

(b)Both (A) and (R) are true but (R) is not the correct explanation of (A).

(c)(A) is true but (R) is false.

(d)(A) is false but (R) is true.

[UPSC-2002]

180.Match List I with List II and select the correct answer using the codes
given.

Codes:

[UPSC-2002]
181.Match List I with List II and select the correct answer using the codes
given.

Codes:

[UPSC-2002]

182.Plants having thorny leaves and penetrating roots are classified as

(a)xerophytes.

(b)hygrophytes.

(c)mesophytes.

(d)desert plants.

[UPSC-2003]

183.Match List I with List II and select the correct answer using the codes
given.
Codes:

[UPSC-2003]

184.Match List I with List II and select the correct answer using the codes
given.

Codes:
[UPSC-2003]

185.Select the correct sequence of food chain in the aretic waters.

(a)Zooplankton, diatoms, fish, polar bear, seal

(b)Diatoms, zooplankton, fish, seal, polar bear

(c)Fish, diatoms, seal, zooplankton, polar bear

(d)Diatoms, fish, polar bear, seal, zooplankton

[UPSC-2003]

186.Eutrophication of lakes is a consequential effect of the accumulation of

(a)silt load.

(b)sewage.

(c)nitrates and phosphates.

(d)vegetation.

[UPSC-2003]

187.Which of the following type of soil produced by granite and gneiss of


weathering?

(a)Red
(b)Black

(c)Yellow

(d)Laterite

[UPSC-2003]

188.With which one of the following is the term `Maitri' associated?

(a)HYV of rice

(b)SAARC

(c)Indo-Russian Treaty

(d)Antarctica Research Centre

[UPSC-2003]

189.Match List I with List II and select the correct answer using the codes
given.

Codes:
[UPSC-2003]

190.What happens to tropical soils when the rainforest is cleared?

(a)The soil improves to the extent that soluble nutrients are no longer
depleted by the trees.

(b)There is little effect on the soil because of its depth of richness.

(c)Clearing interrupts the critical nutrient cycle and leads to soil leaching.

(d)The soils stabilize and become excellent for grain crops.

[UPSC-2004]

191.Among the following climatic factors, which one has the least effect
upon terrestrial ecosystem'?

(a)Temperature variation

(b)Wind

(c)Conditions of sunlight

(d)Availability of water

[UPSC-2004]
192.Which of the following has/have salt marshes as an ideal place for
its/their growth'?

1.Halophytes

2.Mesophytes

3.Xerophytes

Select the correct answer.

(a)1 only

(b)1 and 2

(c)2 only

(d)1 and 3

[UPSC-2005]

193.Match List I with List II and select the correct answer using the codes
given.
Codes:

[UPSC-2005]

194.Traveling from the Equator towards the pole, what would be the correct
sequence of vegetation types you would encounter?

(a)Tropical rainforest, tundra, savannah

(b)Tropical rainforest, desert scrub, tundra

(c)Desert scrub, tropical rainforest, tundra

(d)Mediterranean scrub, desert scrub, tundra


[UPSC-2005]

195.Consider the following statements:

Majuli, the largest river island in the world, is environmentally degraded


due to:

1.floods.

2.erosion.

3.increasing population pressure.

4.development of transport activities.

Which of the above statements are correct?

(a)1 and 4

(b)1 and 3

(c)1, 2 and 3

(d)2, 3 and 4

[UPSC-2000]

196.Gleization is associated with

(a)areas which have experienced recent glacial deposition and have


deranged drainage pattern.

(b)areas where decomposition proceeds slowly.

(c)areas where organic matter is unrecompensed of partially decomposed.


(d)highly acidic and oxygen-poor soils.

[UPSC-2000]

197.Which one of the following statement is not correct?

(a)Highly weathered tropical and subtropical soils are called oxisols.

(b)Grassland soils rich in organic matter and high in bases are called
molisols.

(c)Soils in waterlogged areas with high organic matter are called


histosols.

(d)Young soils with no horizons are called inceptisols.

[UPSC-2001]

198.The transition zone between two exosystems is called

(a)biome.

(b)biotope.

(c)ecotone.

(d)sere.

[UPSC-2001]

199.Which one of the following statements is not correct?

(a)The energy levels within a food chain are called tropic levels.

(b)In deep oceans, primary production is almost.


(c)The primary consumers are called autographs.

(d)The decomposers are called saprotrophs.

[UPSC-2001]

200.The tallest trees of the world are found in the

(a)equatorial biome.

(b)tropicaldeciduous biome.

(c)temperate rainforest biome.

(d)Mediterranean biome.

[UPSC-2002]

201.Laterite is produced by

(a)hydration and carbonation.

(b)hydrolysis and oxidation.

(c)leaching and oxidation.

(d)hydration and calcification.

[UPSC-2003]

202.The important species of the Mediterranean biome are

(a)mahogany, ebony, rosewood.

(b)cork, olive, citrus trees.


(c)eucalyptus, oak, wattle.

(d)pine, cedar, fir.

[UPSC-2003]

203.Plants of tropical and subtropical regions are called

(a)megatherms.

(b)mesotherms.

(c)microtherms.

(d)heksotherms.

[UPSC-2003]

204.The biome with the shortest growing season is

(a)Savannah.

(b)Taiga

(c)Chaparral.

(d)Tundra.

[UPSC-2004]

205.Consider the following statements:

1.The Taiga biome stretches across large parts of Canada, Europe and
Asia.

2.Taiga is the largest biome in the world.


3.A lot of coniferous trees grow in the Taiga.

Which of the statements given above are correct'?

(a)1 and 2

(b)2 and 3

(c)1 and 3

(d)1, 2 and 3

[UPSC-2004]

206.Which one of the following statements is not correct?

(a)Make-up of the soil according to proportions of sand, silt and clay is


known as soil texture.

(b)The way in which sand, silt clay and humus bond together to form
peds is known as soil structure.

(c)The soil moisture budget is the balance of water in the soil.

(d)A group of soil formed from different parent rocks and having
different horizons and soil profiles is known as soil series.

[UPSC-2005]

207.Which one of the following pairs is correctly matched'?


[UPSC-1996]

208.Assertion (A): The pH of acid rain lies between 9 and 12.

Reason (R): Acid rain occurs due to heavy air pollution.

(a)Both (A) and (R) are true and (R) is the correct explanation of (A).

(b)Both (A) and (R) are true but (R) is not the correct explanation of (A).

(c)(A) is true but (R) is false.

(d)(A) is false but (R) is true.

[UPSC-2000]

209.Which one of the following statements is correct'?

(a)All the abiotic resources are nonrenewable and all the biotic resources
are renewable.

(b)All the abiotic resources are nonrenewable but some of the biotic
resources are non-renewable.

(c)Some of the abiotic resources are renewable but the biotic resources
are renewable.

(d)Some of the abiotic resources are renewable and some of the biotic
resources are non-renewable.

[UPSC-1995]

210.Which one of the following statements is correct'?

(a)While most soft woods are deciduous, most hardwoods are evergreen.
(b)The trees of equatorial evergreen forests are mostly of softwood type.

(c)While most softwoods occur in the tropical areas, the hardwoods


arecharacteristically of temperate region.

(d)While most hardwoods are either deciduous or evergreen, softwoods


usually coniferous in characteristic.

[UPSC-1995]

211.The vegetation which survives on aerial roots is

(a)mangrove.

(b)selves.

(c)caatinga.

(d)taiga.

[UPSC-1995]

212.Which one of the following stands for the symbol pH?

(a)Phosphorus and hydrogen content in soil

(b)Percentage of hydrogen in soil

(c)Proportion of potash and hydrogen in soil

(d)Concentration of hydrogen ions in the soil

[UPSC-1996]

213.Which one of the following is azonal soil'?


(a)Sierozem

(b)Podzol

(c)Lithosol

(d)Solonchak

[UPSC-1996]

214.Which of the following pairs is/arecorrectly matched?

1.Sal: Tropical deciduous

2.Pine: Coniferous

3.Teak: Evergreen

4.Cactus: Desert

Select the correct answer using the codes given.

(a)1 and3

(b)2, 3 and 4

(c)1, 2 and 4

(d)1, 2 3 and 4

[UPSC-1996]

215.The most suitable measure for soil conservation is

(a)afforestation.
(b)irrigation.

(c)crop rotation.

(d)contour farming.

[UPSC-1997]

216.Laterite soils are formed due to

(a)removal of silicates, lime and organic matter.

(b)decayed organic matter.

(c)disintegration of volcanic matter.

(d)deposition of limestone.

[UPSC-1997]

217.Consider the following statements:

Low soil fertility in rainy tropics can be caused by

1.high temperature and heavy rainfall.

2.eluviation and leaching.

3.the existence of an extremely active bacterial life on the forest floor.

Of these statements

(a)1, 2 and 3 are correct.

(b)1 and 2 are correct.


(c)2 and 3 are correct.

(d)1 and 3 are correct.

[UPSC-1998]

218.Assertion (A): Volcanic soils are usually fertile.

Reason (R): The heat of eruption causes fertility of such soils.

(a)Both (A) and (R) are true and (R) is the correct explanation of (A).

(b)Both (A) and (R) are true but (R) is not the correct explanation of (A).

(c)(A) is true but (R) is false.

(d)(A) is false but (R) is true.

[UPSC-1998]

219.Consider the following statements:

1.Mahogany is a native of the equatorial forests.

2.Cactus plants are natives of areas of heavy precipitation.

3.Fir trees are found widely in the Amazon basin.

4.Tea is a plant of cool dry climates.

Of these statements

(a)1 alone is correct.

(b)1, 2 and 3 are correct.


(c)1, 3 and 4 are correct.

(d)2 and 4 are correct.

[UPSC-1998]

220.Social forestry aims at

(a)adopting scientific silviculture practices for healthy growth of plants.

(b)motivating villagers in the generation and management of forests.

(c)bringing wastelands under massive afforestation programme.

(d)adopting measures for the maximum utilization of forest products.

[UPSC-1999]

221.Which one of the following pairs is correctly matched?

(a)Taiga: Rosewood

(b)Tropical deciduous: Maple

(c)Mediterranean: Olive

(d)Selva: Mahogany

[UPSC-1999]

222.A submarine rising more than 1,000 metres above the ocean floor is
known as

(a)sea mount.

(b)abyssal hill.
(c)guyots.

(d)submarine ridge.

223.Which of the following is not a calcareous ooze?

(a)Pteropod ooze

(b)Diatom ooze

(c)Globigerina ooze

(d)None of the above

[UGC D-2011]

224.Which of the following is the most important element of ecosystem?

(a)Energy flow

(b)Ecological succession

(c)Food chain

(d)Food web

[UGC D-2011]

225.Which of the following is considered to be the global carbon sink'?

(a)Antarctic ice sheet

(b)Sahara Desert

(c)Equatorial rainforest
(d)Arctic Ocean

[UGC D-2011]

226.Which of the following is not correct about the desert biomes


vegetation?

(a)Short tape roots

(b)Reflective surface of leaf

(c)Ephemeral plants

(d)Waxy coating

[UGC D-2011]

227.Given below are two statements, one labelled as Assertion (A) and the
other labelled as Reason (R). Select your answer from the codes given.

Assertion (A): Coral reefs grow in shallow and warm water.

Reason (R): Near vertical and direct incidence of solar radiation is


essentially required for the growth of coral reefs.

(a)Both (A) and (R) are true.

(b)Both (A) and (R) are false.

(c)(A) is true, but (R) is false.

(d)(A) is false, but (R) is true.

[UGC D-2010]
228.Which of the following statements hold true for thermocline layer'?

1.They are absent in polar areas.

2.With increasing depth, temperature increases.

3.With increasing depth, temperature decreases exponentially.

4.The temperature ranges between 20°Cand 25°C.

5.This layer is approximately 500m thick.

(a)1,2,4and5

(b)1, 3,4 and 5

(c)1 and 3

(d)3, 5 and 1

229.The daily range of temperature of surface water of the ocean is


approximately

(a)1°C.

(b)5°C.

(c)10°C.

(d)20°C.

230.With respect to temperature, ocean in polar areas are divided into

(a)one layer.
(b)two layers.

(c)three layers.

(d)four layers.

231.Which of the following statements will hold true for the annual range of
temperatures of oceans and seas'?

(a)The bigger the ocean, the higher the range.

(b)Enclosed seas will have higher range.

(c)Both (a) and (b)

(d)Neither (a) nor (b)

232.Which of the following will have the lowest annual range of


temperature?

(a)Baltic Sea

(b)Indian Ocean

(c)Atlantic Ocean

(d)Pacific Ocean

233.Which of the following statements will NOT hold true for the surface
temperature and latitudes?

(a)The temperature of surface water between 40°N and 40°S will be


lower than air temperature.
(b)The temperature between 40°N and respective pole will be lower than
air temperature.

(c)The temperature between 40°S and respective pole will be lower than
air temperature.

(d)None of these

234.Assertion (A): The temperature of surface water decreases from Equator


towards the poles.

Reason (R): The distance between the Sun and the Earth is greater at the
poles.

(a)Both (A) and (R) are true and (R) is the correct explanation of (A).

(b)Both (A) and (R) are true but (R) is not the correct explanation of (A).

(c)(A) is true but (R) is false.

(d)(A) is false but (R) is true.

235.Which of the following statements about the isotherms is true?

1.They are regular in the Northern Hemisphere.

2.They do not follow the latitudes in the Southern Hemisphere.

(a)Only 1

(b)Only 2

(c)Both 1 and 2
(d)Neither 1 nor 2

236.Assertion (A): The isotherms are regular in the Southern Hemisphere.

Reason (R): Land mass does not dominate this hemisphere.

(a)Both (A) and (R) are true and (R) is the correct explanation of (A).

(b)Both (A) and (R) are true but (R) is not the correct explanation of (A).

(c)(A) is true but (R) is false.

(d)(A) is false but (R) is true.

237.Prevailing winds

(a)lower the temperature of the surface water.

(b)increase the temperature of the surface water.

(c)have no effect on the surface temperature of the water.

(d)Either (a) or (b) depending on their direction

238.The ocean currents

(a)increase the temperature of the surface water.

(b)have no effect on the surface temperature of the water.

(c)either increase or decrease the temperature of the surface water


depending on their direction.

(d)None of these
239.The temperature of the eastern coast of Siberia is influenced by

(a)Kuroshio.

(b)Labrador.

(c)Kurile.

(d)Gulf Stream.

240.Which of the following statements is true for euphotic zone'?

(a)The rate of decrease of temperature slows down in this zone.

(b)This zone extends up to 200m below the surface of the ocean.

(c)The solar radiation seldom reach this zone.

(d)All of these.

241.The statement, `the temperature of water at sea surface is lower than the
temperature below'

(a)is false.

(b)holds true for unenclosed seas at high latitudes.

(c)holds true for enclosed seas at higher latitudes.

(d)holds true for enclosed seas at lower latitudes.

242.Which of the following statements are NOT true for the salinity of the
sea water'?

1.Evaporation is controlled by salinity.


2.Salinity has no effect on the density of the water.

3.Salinity affects the plant community.

4.Salinity does not affect the temperature of the water.

5.Ocean currents are not affected by the salinity.

(a)1, 2 and 4

(b)2, 4 and 5

(c)1, 3 and 5

(d)1 and 5

243.Assertion (A): Aman is seldom drowned in the sea water having very
high salinity.

Reason (R): Salinity of the sea water makes it easier to swim long
distances without fatigue.

(a)Both (A) and (R) are true and (R) is the correct explanation of (A).

(b)Both (A) and (R) are true but (R) is not the correct explanation of (A).

(c)(A) is true but (R) is false.

(d)(A) is false but (R) is true.

244.Which of the following statements holds true for evaporation and


salinity of water?

1.Salinity of water affects the evaporation of the water.


2.Evaporation affects the salinity of the water.

(a)Only 1

(b)Only 2

(c)Both 1 and 2

(d)Neither 1 nor 2

245.The salinity of the ocean water will be highest in

(a)equatorial zone.

(b)subtropical high-pressure belt.

(c)polar regions.

(d)temperate region.

246.The salinity of the ocean water will be minimum in

(a)equatorial zone.

(b)subtropical high-pressure belt.

(c)polar region.

(d)temperate region.

247.The most important factor influencing the salinity of ocean in the polar
region is

(a)precipitation.
(b)evaporation.

(c)meltwater.

(d)currents.

248.The most important factor influencing the salinity of oceans in the


equatorial zone is

(a)precipitation.

(b)evaporation.

(c)meltwater.

(d)currents.

249.The salinity of the water in the Indian Ocean is highest in

(a)Bay of Bengal.

(b)Arabian Sea.

(c)Persian Gulf.

(d)Red Sea.

250.Equatorial currents are generated due to the influence of

(a)Coriolis force.

(b)rotation of Earth.

(c)gravitational pull of the Sun.


(d)gravitational pull of both the Sun and the Moon.

251.Coriolis force

(a)does not affect the ocean currents.

(b)is the primary force responsible for generation of currents.

(c)is the force which deflects the path of the ocean currents.

(d)None of these

252.Which of the following statements about ebbs holds true?

(a)Two ebbs are experienced twice at every place on Earth's water surface
in a period of 24 hours.

(b)One ebb is experienced twice at every place on Earth's water surface in


a period of 24 hours.

(c)Two ebbs are experienced once at every place on Earth's water surface
in a period of 24 hours.

(d)One ebb is experienced once at every place on Earth's water surface in


a period of 24 hours.

253.Which of the following statements about tides holds true?

(a)Two tides are experienced twice at every place on Earth's water surface
in a period of 24 hours.

(b)One tide is experienced twice at every place on Earth's water surface in


a period of 24 hours.
(c)Two tides are experienced once at every place on Earth's water surface
in a period of 24 hours.

(d)One tide is experienced once at every place on Earth's water surface in


a period of 24 hours.

254.A person experiences a tide at 3.00 a.m., he will experience the second
tide at

(a)3.00 a.m., next day.

(b)3.00 p.m.

(c)3.26 a.m., next day.

(d)3.26 p.m.

255.Aperson experiences an ebb at 4.30 p.m., he will experience tide at

(a)10.30 p.m.

(b)10.43 p.m.

(c)4.30 a.m.

(d)00.43 p.m.

256.Assertion (A): Each day tide is delayed.

Reason (R): This happens due to the rotation of the Moon.

(a)Both (A) and (R) are true and (R) is the correct explanation of (A).

(b)Both (A) and (R) are true but (R) is not the correct explanation of (A).
(c)(A) is true but (R) is false.

(d)(A) is false but (R) is true.

257.Syzygy happens

(a)once a month.

(b)twice a month.

(c)thrice a month.

(d)None of these

258.The syzygy

(a)has no effect on the tides.

(b)affects the tides due to placement of Sun and Moon in a straight line.

(c)affects the tides due to placement of Sun, Moon and Earth in a straight
line.

(d)affects the tides due to placement of Moon and the Earth in a straight
line.

259.The tides will be

(a)highest at the position of conjunction.

(b)highest at the position of opposition.

(c)same at position of conjunction and opposition.

(d)lowest at the position of conjunction.


260.The height of spring tides is than the normal tides.

(a)10 per cent more

(b)10 per cent less

(c)20 per cent more

(d)20 per cent less

261.Spring tides occur

(a)in the season of spring.

(b)twice in a year.

(c)twice in a month.

(d)once every 3 months.

262.Neap tide is than the normal tide.

(a)10 per cent more

(b)10 per cent less

(c)20 per cent more

(d)20 per cent less

263.Neap tide occurs at the position of

(a)conjunction.

(b)syzygy.
(c)opposition.

(d)quadrature.

264.Tropical tides occur

(a)along the Equator.

(b)along the Tropic of Cancer.

(c)along the Tropic of Capricorn.

(d)Both (b) and (c)

265.The perigee tides occur when

(a)Sun is closest to Earth.

(b)Moon is closest to Earth.

(c)Sun is at maximum distance from Earth.

(d)Moon is at maximum distance from Earth.

266.The apogean tides occur when

(a)Sun is closest to Earth.

(b)Moon is closest to Earth.

(c)Sun is at maximum distance from Earth.

(d)Moon is at maximum distance from Earth.


267.What will happen when spring and perigee tides occur at the same
time'?

(a)The tides will be abnormally high.

(b)The tides will be abnormally low.

(c)The two effects will cancel each other and the tides will be normal.

(d)The tides will be higher than normal tide but lower than spring tide.

268.The terms centrifugal force and centripetal force are associated with

(a)progressive wave theory.

(b)stationary wave theory.

(c)equilibrium theory.

(d)None of these.

269.Which of the following forms the basis of progressive wave theory'?

(a)Earth is a perfect fluid.

(b)There is a lagging of time of tides away from the source.

(c)Tides occur at the same time at different places on same longitude.

(d)All of these.

270.Which of the following statements is/are true?

1.Centrifugal force works towards the centre.


2.Centripetal force works towards the centre.

3.Resultant force is the result of balance between gravitational and


centrifugal forces.

(a)Only 1

(b)Only 2

(c)1 and 3

(d)2 and 3

271.According to the progressive wave theory, tides are generated in

(a)Southern Hemisphere.

(b)Northern Hemisphere.

(c)both (a) and (b) simultaneously.

(d)either (a) or (b) depending on the tide.

272.According to progressive wave theory, derived waves are generated

(a)due to the gravitational pull of the Sun.

(b)due to the gravitational pull of the Moon.

(c)due to obstruction by land masses.

(d)due to difference in salinity of the water.

273.According to the progressive wave theory


(a)tides are responsible for generating waves.

(b)waves are responsible for generating tides.

(c)waves and tides act on each other simultaneously.

(d)waves and tides are directly unrelated.

274.Stationary wave theory was purposed by

(a)G.B. Airy.

(b)R.A. Harris.

(c)William Whewell.

(d)Sir Isaac Newton.

275.Which of the following statements holds true for the ocean currents'?

(a)Surface water moves from poles to the Equator due to the temperature
difference.

(b)Subsurface water moves from poles to the Equator due to temperature


difference.

(c)Subsurface water moves from poles to Equator for balancing the loss
of water due to movement of surface water.

(d)Subsurface water moves from Equator to the poles due to temperature


difference.

276.Equatorial currents flow


1.westwards

2.eastwards.

3.are under the influence of North-East trade winds.

4.are under the influence of South-East trade winds.

5.are under the influence of westerlies.

(a)1 and 5

(b)1, 3 and 4

(c)2 and 5

(d)2, 3 and 4

277.Which of the following statements are true for the currents in Indian
Ocean'?

1.The direction of the currents changes twice a year.

2.The currents flow in one direction throughout the year.

3.The currents are influenced by monsoon winds.

4.The winds in the ocean cause the direction of the currents to change.

5.Gulf Stream causes direction of the currents to remain stable

(a)1, 3 and 4

(b)2, 3 and 5
(c)1 and 4

(d)2 and 3

278.Which of the following statements is/are true?

(a)Brazil current after being obstructed by Brazilian coast splits into Gulf
Stream and North Atlantic Drift.

(b)The North Atlantic Drift is deflected to the right when it crosses over
the Wyville Thomson Ridge.

(c)South equatorial current splits into Brazil current and Gulf Stream.

(d)All of these.

279.Which of the following statements holds true for Sargasso Sea?

(a)It records the lowest density of the Atlantic Ocean.

(b)The water of this sea does not mix with the remaining water of the
North Atlantic Ocean.

(c)The mean annual temperature is around 34oC.

(d)All of these.

280.Which of the following statements holds true for South equatorial


current'?

(a)It is stronger than North equatorial current.

(b)It is a cold current.


(c)The current is bifurcated on reaching the western coast of Mexico.

(d)None of these.

281.Tides are produced by the gravitational pull of

1.Sun.

2.Moon.

(a)Only 1

(b)Only 2

(c)Both 1 and 2

(d)Neither 1 nor 2

282.Tsunamis are caused by

(a)gravitational pull of the Moon.

(b)gravitational pull of the Sun.

(c)activity at the seafloor.

(d)All of these.

283.Which of the following statements are true'?

1.Coral reef is a biologically derived sedimentary rock.

2.Corals require sediment-rich waters for their development.

3.Corals live in a symbiotic relationship with algae.


(a)1 and 2

(b)2 and 3

(c)3 and 1

(d)1, 2 and 3

284.Coral bleaching

(a)is a result of use of bleach for increasing the fish yield.

(b)is a result of dynamite fishing.

(c)is caused due to corals expelling their own algae.

(d)None of these.

285.The coral polyps

(a)can thrive in fresh water as well as sea water.

(b)live on lime.

(c)thrive at depths ranging from 200 to 2,000 feet.

(d)cannot survive in sunlight.

286.Fringing coral reefs

(a)can face open sea.

(b)can be protected by a barrier.

(c)Both (a) and (b)


(d)Neither (a) nor (b)

287.Which of the following are coral reefs are largest and the most
extensive'?

(a)Fringing reef

(b)Barrier reef

(c)Atoll

(d)None of these

288.Green Barrier Reef is located in

(a)southern Florida.

(b)east coast of Australia.

(c)Red Sea.

(d)China Sea.

289.The largest barrier reef in the world is approximately long.

(a)500 metres

(b)500 kilometres

(c)1,000 kilometres

(d)2,000 kilometres

290.Which of the following statement hold true for Atoll?


1.Atoll Island and Island Atoll are one and the same thing.

2.Island Atoll has an island in the central part of lagoon.

3.Atoll Island has an island in the Central part of lagoon.

4.Island Atoll does not have an island in the beginning but later on the
island is formed.

5.Atoll Island does not have an island in the beginning but later on the
island is formed.

(a)1, 2 and 3

(b)1, 4 and 5

(c)3 and 4

(d)2 and 5

291.Subsidence theory of origin of coral reefs was purposed by

(a)Charles Darwin.

(b)Murray.

(c)W.M. Davis.

(d)Daly.

292.Subsidence theory was purposed to explain

(a)the formation of fringing reef.


(b)the confinement of coral polyps to shallow depths but their occurrence
in practice at greater depths.

(c)the difference between Atoll and fringing reefs.

(d)the reason for existence of the coral reefs.

293.Which of the following statements is true about the subsidence theory'?

(a)If the theory is accepted as true then the cliffs should be present along
the costal reefs which are non-stationary.

(b)If the theory is accepted as true then the thickness of coral reefs should
decrease downwards.

(c)If the theory is accepted as true then most of the islands of the Pacific
Ocean must be submerged.

(d)If the theory is true then the fringing reefs and barrier reefs can be
found on either side of the same island.

294.According to the stand still theory of Murray, the fringing reefs

(a)with time are transformed into barrier reefs.

(b)are barrier reefs which with time transformed into fringing reefs.

(c)are Atolls which with time transformed into fringing reefs.

(d)None of these.

295.The Murray's theory of formation of fringe is criticized because

(a)the lagoons formed by Atoll must not have disappeared.


(b)the formation of Atoll contradicts formation of barrier reefs.

(c)the limit of 30 fathoms for deposition and erosion cannot be accepted.

(d)None of these.

296.Which of the followings assumptions were made by Murray while


formulating stand still theory'?

1.Coral reefs cannot be found beyond the depth of 30 fathoms.

2.Islands cannot exist between the coasts and coral reefs.

3.Numerous suitable submarine platforms exist at depth of 180 feet.

(a)1 and 2

(b)2 and 3

(c)3 and 1

(d)1, 2 and 3

297.According to Glacial Control theory of formation of coral reefs,

(a)fringing reefs are converted into barrier reefs.

(b)barrier reefs are converted into Atoll.

(c)fringing reefs are converted into Atoll.

(d)fringing reefs, barrier reefs and Atoll are formed independent of each
other.
298.Which of the following facts contradict the glacial control theory of
reefs formation?

(a)The absence of islands between coasts and coral reefs

(b)The variation between depth of different lagoons

(c)The absence of suitable submarine platforms at the depth of 180 feet

(d)All of these.

299.Which one of the following statements is correct'?

(a)The first Meeting of the Parties (MOP) to the Cartagena Protocol on


Biosafety was held in Philippines in the year 2004.

(b)India is not a signatory to the Biosafety Protocol/Convention on


Biological Diversity.

(c)The Biosafety Protocol deals with genetically modified organisms.

(d)The USA is member of Biosafety Protocol/Convention on Biological


Diversity.

[UPSC-2005]

300.In which one of the following countries did hundreds of people die in
the year 2004 as a result of flooding and mudslides triggered by the
tropical storm Jeanne?

(a)Columbia

(b)Haiti
(c)Sudan

(d)Ghana

[UPSC-2005]

ANSWER KEYS
 
1.Consider the following statements:

1.Ritter saw in all his geographical studies the evidence of God's plan.

2.All the volumes of the Die Erdkunde completed by Ritter covered


Africa and a part of Asia.

3.Like Humboldt, Ritter's great work was largely put together on the basis
of his own observations.

Which of the statements given above is/are correct'?

(a)1 and 2 only

(b)1 and 3 only

(c)1, 2 and 3

(d)Neither 1 nor 2

[UPSC-2006]

2.was an important 18th-century English geologist and proponent of


uniformitarians.

(a)James Hutton

(b)Isaac Newton

(c)James Ussher
(d)Charles Lyell

3.was a highly influential, ancient Greek philosopher noted for his writings
and teachings on natural philosophy and on the workings of Earth.

(a)Pappagapolis

(b)Odysseus

(c)Nero

(d)Aristotle

4., a popular natural philosophy of the 17th and early 18th centuries, was
based on a firm belief in a very short geologic history for Earth.

(a)Catastrophism

(b)Ecospherism

(c)Exoschism

(d)Uniformitarianism

5.`Climatic influences are persistent, often obdurate in their control. Arid


regions permit agriculture only through irrigation. The economic
prosperity of Egypt todaydepends on the distribution of the Nile waters as
in the days of Pharaohs'. This statement was made by

(a)Herodotus.

(b)Huntington.

(c)Ratzel.
(d)Semple.

[UPSC-2002]

6.`Environment presents a range of opportunities and man is free to choose


them.' This basic premise in geography is known as

(a)Determinism.

(b)Neo-determinism.

(c)Possibilism.

(d)Probabilism.

[UPSC-2003]

7.Katab-ul-Hind was written by

(a)Al-Masudi.

(b)Al-Biruni.

(c)Al-Idrisi.

(d)Ibn-Batuta.

[UPSC-1996]

8.`Unit area' approach was first adopted in/by

(a)land use survey in Britain.

(b)land utilization in China.

(c)Tennessee Valley Authority.


(d)Damodar Valley Corporation.

[UPSC-1996]

9.`Who controls the Rimland rules Eurasia, who rules Eurasia rules the
World'. This was said by

(a)Mackinder.

(b)Spykman.

(c)Alfred Mahan.

(d)Whittlesey.

[UPSC-1997]

10.Almagest of Ptolemy is a standard reference of

(a)the precise location of known places.

(b)tables of latitudes and longitudes.

(c)Mp projections and world maps.

(d)the movement of celestial bodies.

[UPSC-2004]

11.An approach of reasoning from particular to general is known as

(a)positivist.

(b)normative.

(c)inductive.
(d)deductive.

[UPSC-2001]

12.An early statement of geography as chronology was provided by

(a)Kant.

(b)Humboldt.

(c)Ritter.

(d)Varenius.

[UPSC-2001]

13.Assertion (A): According to determinists, people living in similar


environment similar social, economic and/or historical development.

Reason (R): Every aspect of a man's life mirrors the influences of the
physical environment of his habitat.

(a)Both(A) and (R) are true and (R) is the correct explanation of (A).

(b)Both (A) and (R) are true but (R) is not the correct explanation of (A).

(c)(A) is true but (R) is false.

(d)(A) is false but (R) is true.

[UPSC-2000]

14.Assertion (A): Despite marked development and large volume of output,


Indian geography is yet to make a significant impact on the international
scene.
Reason (R): Indian geographers accept the themes and teachings of
Western geographers without criticism.

(a)Both (A) and (R) are true and (R) is the correct explanation of (A).

(b)Both (A) and (R) are true but (R) is not the correct explanation of (A).

(c)(A) is true but (R) is false.

(d)(A) is false but (R) is true.

[UPSC-2002]

15.Assertion (A): Friedrich Ratzel has been the greatest single contributor to
the development of the geography of man.

Reason (R): His contribution is highly esteemed by cultural


anthropologists.

(a)Both (A) and (R) are true and (R) is the correct explanation of (A).

(b)Both (A) and (R) are true but (R) is not the correct explanation of (A).

(c)(A) is true but (R) is false.

(d)(A) is false but (R) is true.

[UPSC-1996]

16.Assertion (A): No signification progress was made in the development of


geography during the Dark Ages.

Reason (R): Intellectual discourse during the period was guided by


theocracy.
(a)Both (A) and (R) are true and (R) is the correct explanation of (A).

(b)Both (A) and (R) are true but (R) is not the correct explanation of (A).

(c)(A) is true but (R) is false.

(d)(A) is false but (R) is true.

[UPSC-1999]

17.Assertion (A): The concept of Lebensraum was misused by Karl


Haushofer.

Reason (R): It was misused for the furtherance of German expansion


under Nazi.

(a)Both (A) and (R) are true and (R) is the correct explanation of (A).

(b)Both (A) and (R) are true but (R) is not the correct explanation of (A).

(c)(A) is true but (R) is false.

(d)(A) is false but (R) is true.

[UPSC-2002]

18.Assertion (A): The dichotomy of idiographic and nonathletic was started


by David Harvey.

Reason (R): Idiographic and homothetic approaches are concerned with


the particular and the general laws, respectively.

(a)Both (A) and (R) are true and (R) is the correct explanation of (A).

(b)Both (A) and (R) are true but (R) is not the correct explanation of (A).
(c)(A) is true but (R) is false.

(d)(A) is false but (R) is true.

[UPSC-1999]

19.Assertion (A): The followers of Quantitative Revolution promoted


human geography as spatial science.

Reason (R): They assumed people to be rational and try to optimize their
benefits.

(a)Both (A) and (R) are true and (R) is the correct explanation of (A).

(b)Both (A) and (R) are true but (R) is not the correct explanation of (A).

(c)(A) is true but (R) is false.

(d)(A) is false but (R) is true.

[UPSC-2001]

20.Assertion (A): The one who led the development of the new geography in
France was Paul Vidal de la Blache.

Reason (R): He found his way into geography through his study of
ancient history and classical literature.

(a)Both (A) and (R) are true and (R) is the correct explanation of (A).

(b)Both (A) and (R) are true but (R) is not the correct explanation of (A).

(c)(A) is true but (R) is false.

(d)(A) is false but (R) is true.


[UPSC-1995]

21.Behaviouralism came into existence as reaction against

(a)environmental determinism.

(b)quantification.

(c)possibilism.

(d)regionalism.

[UPSC-2004]

22.Compared to the age of Earth accepted as correct today, how did 17th -
and 18thcentury proponents of catastrophism envision the Earth's age'?

(a)They believed Earth to be much younger than current estimates.

(b)They believed it to be about the same as current estimates, give or take


a few million years.

(c)They believed Earth to be much older than current estimates.

(d)None of the above-they did not really address the age of Earth.

23.Consider the following Arab geographers:

1.Ibn Hauqal

2.Al-Idrisi

3.Al-Biruni

4.Ibn-Batuta
The correct chronological order of these geographers is:

(a)1, 3, 4, 2.

(b)3, 1, 4, 2.

(c)3, 1, 2, 4.

(d)1, 3, 2, 4.

[UPSC-1999]

24.Consider the following statement with respect to Quantitative


Revolution:

1.It focused on areal differentiation.

2.It reduced geography to space geometry.

3.It followed the methodology of spatial science.

4.It helped in the development of methods of spatial structure.

Which of these are correct?

(a)1, 2 and 3

(b)2, 3 and 4

(c)1, 3 and 4

(d)1, 2 and 4

[UPSC-2002]
25.Consider the following statements:

1.Ptolemy presumed that the prime meridian passes through Bermuda


Island.

2.Ptolemy believed that Ceylon marks the southern limit of the world.

Which of the statements given above is/are correct'?

(a)1 only

(b)2 only

(c)Both 1 and 2

(d)Neither 1 nor 2

[UPSC-2009]

26.Consider the following statements:

1.Ellen Semple accepted the Ritzel's ideas about the state as an organism
and introduced them to America.

2.Ellen Semple published her first professional article dealing with the
role of the Appalachians as a barrier in American history.

Which of the statements given above is/ are correct'?

(a)1 only

(b)2 only

(c)Both 1 and 2
(d)Neither 1 nor 2

[UPSC-2006]

27.Consider the following statements:

1.Columbus estimated that Asia must lie very close to Europe on the west
on the basis of the work of Ptolemy.

2. Strabo gave a correct explanation of the floods of the Nile, attributing


them to the heavy rain in Ethiopia.

Which of the statements given above is/are correct'?

(a)1 only

(b)2 only

(c)Both 1 and 2

(d)Neither 1 nor 2

[UPSC-2006]

28.Consider the following statements:

1.Humboldt gave the first scientific description of the relation of altitude,


air temperature, vegetation sand agriculture in tropical mountains.

2.Ritter developed the concept that continental climates are colder in


winter and warner in summer than the climate at places near the oceans
at the same latitude.

Which of the statements given above is/are correct?


(a)1 only

(b)2 only

(c)Both 1 and 2

(d)Neither 1 nor 2

[UPSC-2006]

29.Consider the following statements:

1.Ritter made use of the regional approach to geography rather than the
systematic study of individual feature.

2.La Blache presented an effective refutation of the idea of environmental


determinism.

3.From Ratzel's second volume of Anthropogeography, Huntington


formulated the concept of possibilism.

Which of the statements given above is/are correct'?

(a)1 only

(b)2 only

(c)Both 1 and 2

(d)Neither 1 nor 2

[UPSC-2007]

30.Consider the following statements:


1.Semple was very careful to make the point that the environment
controls human action.

2.Huntington posited climate, the quality of people and culture as a triadic


causation of human progress.

Which of the statements given above is/are correct'?

(a)1 only

(b)2 only

(c)Both 1 and 2

(d)Neither 1 nor 2

[UPSC-2007]

31.Consider the following statements:

1.Nature sets limits and offers possibilities for development.

2.Man' adjustment to nature is the function of traditions and mental


structuring.

Which of the statements given above is/ are correct?

(a)1 only

(b)2 only

(c)Both 1 and 2

(d)Neither 1 nor 2
[UPSC-2008]

32.Consider the following statements:

1.Ritter's scientific geography was based on the concept of unity in


diversity.

2.Ritter's concept regarding the meaning of the observed geographical


patterns on Earth was teleological.

Which of the statements given above is/ are correct'?

(a)1 only

(b)2 only

(c)Both 1 and 2

(d)Neither 1 nor 2

[UPSC-2008]

33.Consider the following statements:

1.Ritter identified races by skin colour.

2.Ritter's concepts regarding the Earth's geographical patterns were


teleological.

Which of the statements given above is/ are correct'?

(a)1 only

(b)2 only
(c)Both 1 and 2

(d)Neither 1 nor 2

[UPSC-2009]

34.Consider the following statements:

1.The genre de vie of Vidal de la Blache refers to the inherited traits that
members of a human group learn, what we may call culture.

2.The genre de vie is a factor in determining which are the various


possibilities offered by nature that a particular human group will select.

Which of the statements given above is/ are correct'?

(a)1 only

(b)2 only

(c)Both 1 and 2

(d)Neither 1 nor 2

[UPSC-2009]

35.Consider the following statements of Kant:

1.Knowledge can be classified either logically or physically.

2.The logical classification collects all individual items according to


similarity of morphology.

3.The physical classification collects individual.


4.History is narrative and geography a description.

Which of these are correct'?

(a)1, 2 and 3

(b)1, 3 and 4

(c)2 and 4

(d)1, 2, 3 and 4

[UPSC-2002]

36.Consider the following statements regarding Kant's view on origin of the


Earth:

1.Kant introduced the Newtonian law of gravitation in his theory.

2.Kant development his theory accepting the principle of conservation of


angular momentum.

3.Though Laplace put forward the nebular hypothesis of origin of the


Earth, Kant put nebular hypothesis.

Which of this statement are correct?

(a)1, 2 and 3

(b)1 and 3

(c)2 and 3

(d)1 and 2
[UPSC-2003]

37.Consider the following statements:

1.Exceptionalism is closely related to nomotheic approach in geography.

2.Idiographic approach is concerned mainly with the unique and the


particular.

3.Radicalism developed in opposition of Marxism.

4.Humanism gives the central and active roe lot human awareness.

Which of these are correct'?

(a)1 and 2

(b)2 and 3

(c)2 and 4

(d)1 and 4

[UPSC-2002]

38.Consider the following statements:

1.Al-Masudi translated Patanjali's book into Arabic.

2.Ibn-Batuta was appointed as the ambassador of the Sultanate of Delhi to


China.

3.Baitul-Hilkam (Academy of Science) at Baghdad was founded by


Caliph Al-Mamoon.
4.The Arabs borrowed the `Geocentric Concept' from Greeks.

Which of the above statements are correct'?

(a)1 and 2

(b)1 and 3

(c)2 and 4

(d)1, 2, 3 and 4

[UPSC-2000]

39.Consider the following statements:

The main objectives of Quantitative Revolution in geography was to

1.make geography an Earth science.

2.make geography the geometry of space.

3.develop precise regional laws.

4.develop universal laws in geography.

Which of the above statements are correct'?

(a)1, 2 and 3

(b)1, 3 and 4

(c)2, 3 and 4

(d)1, 2 and 4
[UPSC-2000]

40.Consider the following statements:

1.Qualitative Revolution means the use of statistical and mathematical


techniques in understanding geographical system.

2.Quantitative methods made geography as a `regional science'.

3.Quantitative methods range from passive.

4.Quantitative methods influenced geography mainly through the


Location Theory.

Which of the above statement are correct'?

(a)1 and 2

(b)1, 2 and 3

(c)1, 2 and 4

(d)1, 2, 3 and 4

[UPSC-2001]

41.Consider the following statements:

1.Radicalism in geography came in the form of the reaction geography as


a `spatial science'.

2.The origin of radical approach can be traced to civil right, poverty and
inequality.

3.The founders of radicalism in geography had a strong Marxist base.


4.Some of the radical geographers termed radicalism as `the
politicaleconomy perspective'.

Which of the above statements are correct'?

(a)1, 2 and 3

(b)1, 2 and 4

(c)1, 3 and 4

(d)1, 2, 3 and 4

[UPSC-2001]

42.Consider the following statements:

1.Spatial analysis is closely associated with positivism.

2.The behaviouralists concentrated on the issues of inequality.

3.Humanistic geography attempts at understanding meanings and values


of life events.

Which of these are correct'?

(a)1 and 2

(b)1 and 3

(c)2 and 3

(d)1, 2 and 3

[UPSC-2002]
43.Consider the following statements:

1.Ocean currents are the slow surface movements of water in the same.

2.Ocean currents assist in maintaining the Earth's heat balance.

3.Ocean currents are set in motion primarily by prevailing winds.

4.Ocean currents are affected by the configuration of the ocean.

Which of these statements are correct'?

(a)1 and 2

(b)2, 3 and 4

(c)1, 3 and 4

(d)1, 2, 3 and 4

[UPSC-2002]

44.Consider the following statements:

1.Ratzel dominated German geography in the second half of the


19thcentury.

2.Ratzel coined the term 'anthropology'.

3.Ratzel was highly influenced by Darwin's theory of evolution of


species.

Which of these are correctly associated with Ratzel'?

(a)1, 2 and 3
(b)1 and 2

(c)2 and 3

(d)1 and 3

[UPSC-2003]

45.Consider the following statements:

Arab geographers contributed to the development of geography by

1.translating and absorbing of learning from Greek, Persian and Sanskrit


sources.

2.preserving important writings.

3.expanding geographical knowledge to Europe and Asia.

4.contributing their original work to Mathematics.

Which of the statements given above are correct'?

(a)1, 3 and 4

(b)1, 2 and 3

(c)2, 3 and 4

(d)1, 2 and 4

[UPSC-2004]

46.Consider the following statements:


1.The environmental determinism dominated the writings of early Arab
geographers.

2.Vidal de la Blache supplemented classical geographical determinism


with `social Darwinism'.

Which of the statements given above is/ are correct?

(a)1 only

(b)2 only

(c)Both 1 and 2

(d)Neither 1 nor 2

[UPSC-2005]

47.Consider the following statements:

1.An approach to human geography that emphasizes questions of


inequality is known as welfare geography.

2.An approach to human geography that gives central and active role to
human awareness is known as behavioural geography.

3.Radical geography developed as a reaction to spatial analysis.

Of these statements:

(a)1 alone is correct.

(b)1 and 2 are correct.

(c)1 and 3 are correct.


(d)2 and 3 are correct.

[UPSC-1999]

48.Consider the following statements:

1.The Roman's main contribution was in the field of historical and


mathematical geography.

2.The concept of Terra-Incognita was propounded by Eratosthenes.

3.The book Outline of Geography was written by Ptolemy.

4.Ptolemy used astronomical principles to prepare.

Which of the above statements are correct?

(a)1, 2 and 3

(b)1, 3 and 4

(c)2, 3 and 4

(d)1, 2 and 4

[UPSC-2000]

49.Consider the following statements:

1.Plaska-Dwipa lies to the south of Kraunca-Dwipa.

2.Salmali-Dwipa lies to the south of Kusa-Dwipa.

3.Pushkara-Dwipa lies to the southeast of Jambu-Dwipa.


4.Kusa-Dwipa lies to the west to jambu-Dwipa.

Which of these are correct?

(a)1, 2 and 3

(b)1, 2 and 4

(c)2, 3 and 4

(d)1, 3 and 4

[UPSC-2002]

50.Consider the following statements:

1.Herodotus declared Earth as flat and discovered that the sun travelled in
an arc form east to west.

2.In the opinion of Ptolemy, Earth is situated in the centre of the universe.

3.The `T'-`O' map was oriented towards the east.

4.The Arab geographers oriented their maps towards the south.

Which of these are correct?

(a)1, 2and 4

(b)2 and 4

(c)3 and 4

(d)1, 2, 3 and 4
[UPSC-2002]

51.Erdkunde written by Ritter is a/an

(a)Earth science concentrating on aspect of the Earth.

(b)regional geography book dealing with the Earth's surface as a mere


aggregate of parts.

(c)regional geography book dealing with areas as

(d)science of Earth in relation to its nature and history of making.

[UPSC-2004]

52.For preparing the world map, Eratosthenes used prime meridians trough
crime one of the following'?

(a)Canary Island

(b)Alexandria

(c)Rome

(d)Syene (Aswan)

[UPSC-2007]

53.Form amongst the following Arab scholars, who is best known for his
extensive travels and written record of geographical importance'?

(a)Ibn-Hauqal

(b)Al-Masudi
(c)Al-Biruni

(d)Ibn-Batuta

[UPSC-2003]

54.Geographical thoughts by Sudipa Adhikari

(a)Varenius

(b)Kant

(c)Humboldt

(d)Ritter

[UPSC-2009]

55.Humboldt synthesized his ideas in `Cosmos' with the specific objective

(a)to define the contents and methods of modern physical geography.

(b)to lay foundation of the Universal Science as a knowledge of the


universe in its totality.

(c)to establish geography as the study of the Earth's surface or its areal
segments.

(d)to explain the concept of dualism between man and the physical world.

[UPSC-2004]

56.Huntington is noted for describing

(a)man as a product of the Earth's surface.


(b)the evolution of landforms.

(c)the effects of climate on human life.

(d)the historical-ecological study of the cultural landscape.

[UPSC-2001]

57.In addition to Anthropogeography, Ratzel made important contribution to

(a)cultural geography.

(b)regional geography.

(c)physical geography.

(d)political geography.

[UPSC-2002]

58.In support of his hypothesis on the origin of the Earth, which one of the
following scholars said, `Give me matter and I will build a world out of
it'?

(a)James Jeans

(b)T.C. Chamberlin

(c)Immanuel Kant

(d)Laplace

[UPSC-2001]
59.Match List I with List II and select the correct answer using the codes
given.

Codes:

[UPSC-1996]

60.Match List I with List II and select the correct answer using the codes
given.
Codes:

[UPSC-1997]

61.Match List I with List II and select the correct answer using the codes
given.

Codes:

[UPSC-1997]
62.Match List I with List II and select the correct answer using the codes
given.

Codes:

[UPSC-1998]

63.Match List I with List II and select the correct answer using the codes
given.

Codes:
[UPSC-1998]

64.Match List I with List II and select the correct answer using the codes
given.

Codes:

[UPSC-1999]

65.Match List I with List II and select the correct answer using the codes
given.
Codes:

[UPSC-2000]

66.Match List I with List II and select the correct answer using the codes
given.

Codes:
[UPSC-2000]

67.Match List I with List II and select the correct answer using the codes
given.

Codes:

[UPSC-2000]

68.Match List I with List II and select the correct answer using the codes
given.
Codes:

[UPSC-2001]

69.Match List I with List II and select the correct answer using the codes
given.
Codes:

[UPSC-2001]

70.Match List I with List II and select the correct answer using the codes
given.
Codes:

[UPSC-2002]

71.Match List I with List II and select the correct answer using the codes
given.
Codes:

[UPSC-2003]

72.Match List I with List II and select the correct answer using the codes
given.
Codes:

[UPSC-2005]

73.Match List I with List II and select the correct answer using the codes
given.

Codes:
[UPSC-2005]

74.Of the following, who enunciated that history should be studied


geographically and all geography historically?

(a)Herodotus

(b)Strabo

(c)Hecataeus

(d)Posidonius

[UPSC-2009]

75.One of the pioneer German geographers of 19th century, who presented


the organic theory of state?

(a)Alexander von Humboldt

(b)Carl Ritter

(c)Friendrich Ratzel

(d)Alfred Hettner

[UPSC-2004]
76.Physical environment offers opportunities for a range of possible
directions of development and depends on human initiatives in particular
direction of progress. This basic premise in geography is known as

(a)determinism.

(b)neo-determinism.

(c)possibilism.

(d)probabilism.

[UPSC-2004]

77.Ratzel coined the term `Anthropogeography' which he used for the

(a)geography of man in terms of individuals and races.

(b)geography of man and his works in relation to the Earth's surface.

(c)organic theory of society and state.

(d)study of harmonious reciprocal relations of man and nature.

[UPSC-2009]

78.Ratzel's second volume of Anthropogeography emphasized on

(a)influence of physical environment of man.

(b)lebensraum.

(c)human migration.

(d)influence of man on natural environment.


[UPSC-2006]

79.Ratzel's work was based on the concept that

(a)there is a dichotomy between the physical and cultural aspects of


geography.

(b)geography is a scientific discipline.

(c)physical environment controlled human activities.

(d)geography is necessarily descriptive.

[UPSC-2001]

80.Regarding von Humboldt, consider the following statements:

1.He assembled materials to show how environments varied with


differences in agricultural practices, reflecting interaction among
altitude, temperature and vegetation.

2.He developed synthetic reasoning leading to inductive generalization.

Which of the statements given above is/ are correct'?

(a)1 only

(b)2 only

(c)Both 1 and 2

(d)Neither 1 nor 2

[UPSC-2008]
81.Select the correct sequence in the chronological order of the following
Greeks who have contributed vastly to the evolution of geographical
thought during the ancient period.

(a)Aristotle, Eratosthenes, Anaximander, Ptolemy

(b)Eratosthenes, Anaximander, Ptolemy, Aristotle

(c)Anaximander, Aristotle, Eratosthenes, Ptolemy

(d)Ptolemy, Anaximander, Aristotle, Eratosthenes

[UPSC-2003]

82.The approach in geography which states that actions are motivated by


cognitive process is the sense of

(a)behaviouralism.

(b)humanism.

(c)positivism.

(d)radicalism.

[UPSC-1998]

83.The approach in the study of geography which claims that geography


should be concerned essentially with the formulation of scientific laws is
known as the

(a)Idiographic Approach.

(b)Nomothetic Approach.
(c)Positivistic Approach.

(d)Radical Approach.

[UPSC-1998]

84.The Arab geographer who authored the book Amusement for Him Who
Desires to Travel around the World also called the book of Roger and who
is famous for his `Sliver Map' was

(a)Al-Idrisi.

(b)Al-Biruni.

(c)Al-Maqdisi.

(d)Al-Istakhari.

[UPSC-2004]

85.The behavioural environment is meant by

(a)reality as it exists in nature.

(b)the decision based on reasoned thought.

(c)man's behaviour as a function of the environment.

(d)reality as is perceived by individuals.

[UPSC-2001]

86.The book The Geography of Puranas was written by

(a)R.N. Kubey.
(b)S.M. Ali.

(c)T. Razvi.

(d)S.P. Chatterjee.

[UPSC-2000]

87.The central idea of various cosmogony theories of Vedic and post-Vedic


period is the creation of a cosmic nucleus was given by

(a)Vishwakarma.

(b)Praj apati.

(c)Brahmand.

(d)Surya.

[UPSC-2004]

88.The dichotomy of general geography and special geographical was


advocated by

(a)Alexander von Humboldt.

(b)Alfred Hettner.

(c)Elisce Reclus.

(d)Varenius.

[UPSC-2000]
89.The doctrine of uniformitarianism, that is `the present is the key to the
past' was advocated by

(a)E.E. Dutton.

(b)G.K. Gibert.

(c)J. Hutton.

(d)J. Playfair.

[UPSC-1999]

90.The first treatise on cartography was written by

(a)Aristotle.

(b)Eratosthenes.

(c)Mercator.

(d)Ptolemy.

[UPSC-1996]

91.The heliocentric concept of the universe was first formulated by

(a)Ptolemy.

(b)Newton.

(c)Copernicus.

(d)Cassini.

[UPSC-1996]
92.The idea for digging a canal across panama Isthmus was put forward by

(a)Ptolemy.

(b)Strabo.

(c)Ratzel.

(d)Humboldt.

[UPSC-2003]

93.The new geography of 1950 was characterized by emphasis on

(a)empirical enquiry.

(b)geographical aspects of social issue.

(c)locational theory and use of quantitative methods and models.

(d)human ecology.

[UPSC-2002]

94.The position in philosophy that defines meaning and knowledge in terms


of their of their function and experience, with reference to adjustment and
the resolution of problematic situation, is termed as

(a)pragmatism.

(b)idealism.

(c)realism.

(d)nominalism.
[UPSC-1997]

95.The principal contribution of Varenius was to synthesize the approaches


of geographical studies as

1.special geography.

2.physical geography.

3.general Geography.

4.human geography.

Select the correct answer using the codes given:

(a)1, 2 and 3

(b)1, and 4

(c)3 and 4

(d)1 and 3

[UPSC-2003]

96.The radical thinking in geography is focused on

(a)attempt to engage the discipline of societal irregularities

(b)only surface manifestations of societal irregularities.

(c)the premise that progress is independent of the material conditions.

(d)the mental images that the decision makers have of the environment.

[UPSC-2003]
97.The statement, `History differs from geography in consideration of time
and space. The former is the report of phenomena that follow one another
and has reference to time. The latter is the report of phenomena besides
each other in space.' is attributed to

(a)Alexander von Humboldt.

(b)Carl Ritter.

(c)Immanuel Kant.

(d)Bernhard Varen (Varenius).

[UPSC-2000]

98.The term `Anthropogeographic' was coined by Ratzel and was used by


him in the context of

(a)geography of man in terms of individual and races.

(b)geography of man and his works in relation to Earth's surface.

(c)organic theory of society and state.

(d)study of harmonious reciprocal relations of man and nature.

[UPSC-2000]

99.The terms longitude and latitude were first used by

(a)Eratosthenes.

(b)Anaximander.

(c)Ptolemy.
(d)Homer.

[UPSC-2005]

100.The theory that events can only be explained as stages in a movement


towards a pre-ordained end is known as

(a)functionalism.

(b)humanism.

(c)phenomenology.

(d)teleology.

[UPSC-1998]

101.Varenius clear distinguished between

(a)physical and human geography.

(b)general and special geography.

(c)quantitative and qualitative geography.

(d)historical and contemporary geography.

[UPSC-2002]

102.What are the basic differences between the disciplines of physical and
historical geology?

(a)Physical geology involves the study of rock strata, fossils and


deposition in relation to plate movements in the geologic past;
historical geology charts how and where the plates were moving in the
past.

(b)Physical geology is the study of fossils and sequences of rock strata;


historical geology is the study of how rocks and minerals were used in
the past.

(c)Historical geology involves the study of rock strata, fossils and


geologic events, utilizing the geologic time scale as a reference;
physical geology includes the study of how rocks form and of how
erosion shapes the land surface.

(d)None of the above-physical geology and historical geology are


essentially the same.

103.Which of the following are three important approaches to humanistic


geography?

1.Idealistic

2.Phenomenal

3.Naturalistic

4.Existential

Select the correct answer using the codes given.

(a)1, 2 and 3

(b)1, 2 and 4

(c)1, 3 and 4
(d)2, 3 and 4

[UPSC-2004]

104.Which of the following methods were employed by Humboldt in his


universal science `Cosmos'?

1.Empirical and inductive

2.Deductive procedure of science

3.Comparative regional geography

4.Graphical representation

Select the correct answer using the codes given.

(a)1, 2 and 3

(b)1, 3 and 4

(c)2, 3 and 4

(d)1, 2 and 4

[UPSC-2004]

105.Which of the following were objectives of the quantitative revolution in


geography'?

1.To get rid of deterministic thinking

2.To make geography more scientific

3.To make geographical study more precise and accurate


4.Formulation of models and theories

Select the correct answer using the codes given.

(a)1, 2 and 3

(b)1, 2 and 4

(c)2, 3 and 4

(d)1, 3 and 4

[UPSC-2004]

106.Which one of the following approaches to geography stressed on


holistic view of economy, society and polity'?

(a)Behaviouralism

(b)Humanism

(c)Positivism

(d)Radicalism

[UPSC-2000]

107.Which one of the following are the correct statements? Varenius `special
geography' was

(a)systematic geography.

(b)regional geography.

(c)an empirical idea proved by incident.


(d)a teleological concept.

[UPSC-2007]

108.Which one of the following concepts is associated with Vidal de la


Blache?

(a)Determinism

(b)Possibilism

(c)Environmentalization

(d)Probabilism

[UPSC-2001]

109.Which one of the following concepts is associated with Vidal de la


Blache?

(a)Landscape chorology

(b)Mathematical geography

(c)Terrestrial whole

(d)Landschaft

[UPSC-2005]

110.Which one of the following forms the subject matter of radical


geography'?

(a)Socioeconomic inequalities of man


(b)Geopolitics in different area

(c)Disparities in industrial development

(d)Agricultural problems

[UPSC-1995]

111.Which one of the following geographers wrote the book Influences of


Geographic Environment?

(a)Humboldt

(b)Ratzel

(c)Ritter

(d)Semple

[UPSC-2009]

112.Which one of the following is a key concept in humanistic geography?

(a)Development and quality of life

(b)Spatial location

(c)Spatial concentration

(d)Geographical distribution

[UPSC-2003]

113.Which one of the following is the correct sequence in the evolution of


geographical ideologies?
(a)Determinism, Quantitative Revolution, Possibilism, Radicalism

(b)Determinism, Possibilism, Quantitative Revolution, Radicalism

(c)Possibilism, Determinism, Radicalism, Quantitative Revolution

(d)Radicalism, Possibilism, Determinism, Quantitative Revolution

[UPSC-2005]

114.Which one of the following pairs is correctly matched?

(a)Ellen Semple: Stop-and-godeterminism

(b)Friedrich Ratzel: Pulsation hypothesis

(c)Vidal de la Blache: Genre de vie

(d)Griffith Taylor: Lebensraum

[UPSC-2004]

115.Which one of the following pairs is not correctly matched?

(a)Hecataeus: Ges-periodos

(b)Anaximander: Gnomon

(c)Eratosthenes: Pangaea

(d)Hipparchus: Astrolabe

[UPSC-2005]

116.Which one of the following scholars of pre-modern period clearly


established the relationship between regional geography and systematic
geography?

(a)Cluverius

(b)Bushing

(c)Buache

(d)Varenius

[UPSC-2005]

117.Which one of the following scholars suggested the Earth's origin from
gases and dust particles?

(a)James Jeans

(b)H. Alfven

(c)F. Hoyle

(d)O. Schmidt

[UPSC-1999]

118.Which one of the following statements applies to the Arab geographers?

(a)They developed various concepts of geography.

(b)Their main contribution to geography was in the field of measurement


based on astronomical observations.

(c)They laid the foundation of modern geography.

(d)They did not place importance to writing travel narratives.


[UPSC-1995]

119.Which one of the following terms is credited to Humboldt?

(a)Uranography

(b)Biogeography

(c)Geomorphology

(d)Climatology

[UPSC-2005]

120.While explaining the relation between nature and man, Blache cited the
instance of the tradition of a regional concept in France known as

(a)compage.

(b)formal region.

(c)functional region.

(d)pays.

[UPSC-2000]

121.Who advocate climate as the main determinant of history, culture,


lifestyle and occupations?

(a)G. Taylor

(b)F. Ratzel

(c)E. Semple
(d)E. Huntington

[UPSC-2004]

122.Who among the following contributed most to the development of


astronomy?

(a)Greeks

(b)Romans

(c)Indians

(d)Arabs

[UPSC-2002]

123.Who among the following is regarded as the founder of Exceptionalism


in geography'?

(a)Blaut

(b)Haggett

(c)Harvey

(d)Hartshorne

[UPSC-1999]

124.Who among the following rejected the teleological idea of final causes
and insisted that explanations must be sought in what is chronologically
antecedent and, in this view, opposed Linnaeus and Leibniz but supported
and amplified the ideas of Hume and Goethe?
(a)Varenius

(b)Kant

(c)Humboldt

(d)Ratzel

[UPSC-2006]

125.Who among the following scholars followed the teleological approach


in the study of the Earth'?

(a)Alexander von Humboldt

(b)Carl Ritter

(c)Immanuel Kant

(d)Bernhard Varenius

[UPSC-2005]

126.Who among the following was the first to distinguish between general
geography and special geography and to show the mutual dependence of
these approaches?

(a)Varenius

(b)Kant

(c)Humboldt

(d)Strabo
[UPSC-2006]

127.Who among the following was the first to propose a classification of


sciences dealing with reality'?

(a)Alfred Hettner

(b)Immanuel Kant

(c)Alexander von Humboldt

(d)Richard Hartshorne

[UPSC-2004]

128.Who amongst the following was the first to state that the Earth was
spherical'?

(a)Aristotle

(b)Copernicus

(c)Ptolemy

(d)Strabo

[UPSC-2001]

129.Which of the following advocated the idea that `the state is an organism
attached to the land"?

(a)Ratzel

(b)Ritter
(c)Kant

(d)Humboldt

[UPSC-2008]

130.Who among the following devoted himself to the study of the modes of
life of Germans living outside Germany'?

(a)Ratzel

(b)Mackinder

(c)Jafferson

(d)Sauer

[UPSC-2008]

131.Which of the following geographers stated that the Earth is an


inseparable organic whole'?

(a)Von Humboldt

(b)Ritter

(c)Richthofen

(d)Ratzel

[UPSC-2008]

132.Who is the author ofAlmagest and Guide to Geography, two great books
of the Roman period'?
(a)Strabo

(b)Plini

(c)Ptolemy

(d)Pomponious

[UPSC-2008]

133.Who named the human dominance theory of `man-environment


relationship' as `Possibilism'?

(a)Lucien Febvre

(b)Lucien Gallois

(c)Vidal de la Blache

(d)Jean Brunhes

[UPSC-2008]

134.Who was the first scientific geographer who ascertained the length of
Equator on sound principles'?

(a)Herodotus

(b)Hecataeus

(c)Anaximander

(d)Eratosthenes

[UPSC-2006]
135.Who related the rise for civilization in the mid-latitudes and lack of
development in the tropics to climatic conditions'?

(a)E. Huntington

(b)E.C. Semple

(c)W.M. Davis

(d)F. Ratzel

[UPSC-2007]

136.Who among the following geographers adopted the `deductive method'


of study in geography'?

(a)Varenius

(b)Cluverius

(c)Kant

(d)Humboldt

[UPSC-2007]

137.`Egypt is the gift of the Nile' was stated by

(a)Aristotle.

(b)Seneca.

(c)Herodotus.

(d)Hypocratis.
[UGC D-2005]

138.Given below are two statements, one labelled as Assertion (A) and the
other labelled as Reason (R). Select your answer from the codes given.

Assertion (A): Most of the radical geographers have a very strong Marxist
base.

Reason (R): Radical geographers have made great inroads into


establishment of the discipline of geography.

(a)Both (A) and (R) are true and (R) is the correct explanation of (A).

(b)Both (A) and (R) are true but (R) is not the correct explanation of (A).

(c)(A) is true but (R) is false.

(d)(A) is false but (R) is true.

[UGC D-2005]

139.Match List I with List II and select the correct answer from the codes
given.

Codes:
[UGC D-2005]

140.Match List I with List II and select the correct answer using the codes
given.

Codes:

[UGC D-2006]

141.The philosophical view advocating that man is responsible for making


his own nature is known as

(a)positivism.
(b)functionalism.

(c)existentialism.

(d)pragmatism.

[UGC D-2006]

142.The correct sequence of the advent of prominent British geographers to


the development of geography is

(a)A.J. Herbertson, S.W.Wooldrige, H.J.Mackinder, L.D.Stamp.

(b)S.W. Wooldrige, A.J.Herbertson, L.D.Stamp, H.J.Mackinder.

(c)H.J. Mackinder, A.J.Herbertson, S.W.Wooldrige, L.D.Stamp.

(d)L.D. Stamp, A.J.Herbertson, S.W. Wooldrige, H.J.Mackinder.

[UGC D-2006]

143.Given below are two statements, one labelled as Assertion (A) and the
other labelled as Reason (R). Select your answer from the codes given.

Assertion (A): Ritter did not state the relationship between history and
geography as clearly as Humboldt.

Reason (R): Immanual Kant understood the relationship between history


and geography much more clearly than Ritter.

(a)Both (A) and (R) are correct and (R) explains (A).

(b)Both (A) and (R) are correct but (R) does not explain (A).

(c)(A) is correct but (R) is false.


(d)(A) is false but (R) is true.

[UGC D-2007]

144.The book Cosmos written by Alexander von Humboldt was based on his
travel to which continent'?

(a)Asia

(b)North America

(c)Europe

(d)South America

[UGC D-2007]

145.Given below are two statements, one labelled as Assertion (A) and the
other labelled as Reason (R). Select your answer from the codes given.

Assertion (A): Humboldt had no immediate followers in academic circle


in Germany, since he never occupied any academic position.

Reason (R): The real followers of Humboldt were scientific travellers.

(a)Both (A) and (R) are correct and (R) explains (A).

(b)Both (A) and (R) are correct but (R) does not explain (A).

(c)(A) is correct but (R) is false.

(d)(A) is false but (R) is correct.

[UGC D-2007]
146.`Nature does not drive a man along a particular road, but it offers a
number of opportunities from among which man is free to select'. This
statement implies

(a)deterministic approach.

(b)possibilistic approach.

(c)both deterministic and possibilistic approach.

(d)probabilistic approach.

[UGC D-2007]

147.Match List I with List II and select the correct answer from the codes
given.

[UGC D-2007]
148.Who among the following suggested the triad of mutual relationship
among `place, work and folk"?

(a)Le Play

(b)Clarence Stein

(c)Le Corbusier

(d)Ebenezer Howard

[UGC D-2007]

149.Match List I with List II and select the correct answer from the codes
given.

Codes:

[UGC D-2008]
150.Who wrote the book Anthropogeographie'?

(a)F. Ratzel

(b)E.C. Semple

(c)Vidal de la Blache

(d)Charles Darwin

[UGC D-2008]

151.Humboldt wrote his famous book Cosmos after he conducted fieldwork


in

(a)Europe.

(b)west Africa.

(c)South America.

(d)North America.

[UGC D-2008]

152.Philosophical approach in geographical writings was first adopted by

(a)Carl Ritter.

(b)Richard Hartshorne.

(c)Bernhard Varenius.

(d)Immanuel Kant.

[UGC D-2008]
153.Who edited the book Geography of the Twentieth Century?

(a)H.J. Mackinder

(b)Vidal de la Blache

(c)T.W. Freeman

(d)Griffith Taylor

[UGC D-2008]

154.Match List I with List II and select the correct answer from the codes
given.

[UGC D-2008]
155.Match List I with List II and select the correct answer from the codes
given.

[UGC D-2008]

156.Which one of the following philosophies was advanced by August


Comte?

(a)Empiricism

(b)Positivism

(c)Marxism

(d)Humanism

[UGC D-2008]
157.Which one of the following scholars was the first to divide the world
landmass into three continents: Europe, Asia and Libya'?

(a)Anaximander

(b)Hecataeus

(c)Herodatus

(d)Eratosthenes

[UGC J-2005]

158.In which of the following books did Ratzel compare the stateto an
organism'?

(a)Anthropogeographie

(b)Geographic Humaine

(c)Political Geography

(d)Both (a) and (c)

[UGC J-2005]

159.The concept of Lebensraum was propounded by

(a)Humboldt.

(b)Ritter.

(c)Ratzel.

(d)Semple.
[UGC J-2005]

160.With whom is the concept of `pragmatic possilulism' associated?

(a)Bowman.

(b)Griffith Taylor.

(c)Humboldt.

(d)Ritter.

[UGC J-2005]

161.Match List I with List II and select correct answer from the codes given.

Codes:
[UGC J-2005]

162.Identify the author who wrote several volumes entitled Geographica.

(a)Eratosthenes

(b)Ptolemy

(c)Strabo

(d)Herodotus

[UGC J-2007]

163.Who among the following Arab geographers spent several years in India
and wrote a book on India?

(a)Ibn-Batuta

(b)Al-Idrisi

(c)Ibn Khaldun

(d)Al-Biruni

[UGC J-2007]

164.Who propounded the organic theory of the state?

(a)Humboldt

(b)Richthofen

(c)Ratzel

(d)Ritter
[UGC J-2007]

165.Name the scholar who used the term `geopolitik' for the first time before
the Second World War.

(a)Mackinder

(b)Kjellan

(c)Houshofer

(d)Spykman

[UGC J-2007]

166.The concept of `cultural hearth' was first used by

(a)Berkeley School of Geography.

(b)French School of Geography.

(c)German School of Geography.

(d)Chicago School of Geography.

[UGC J-2007]

167.Who tried to resolve the problem of dualism in geography by modifying


the `environment' concept with major focus on man?

(a)Vidal de la Blache

(b)H.H. Barrows

(c)E. Huntington
(d)E.C. Semple

[UGC J-2007]

168.Match List I with List II and select the correct answer from the codes
given.

Codes:

[UGC J-2008]

169.Quantitative Revolution in Britain was mainly popularized by

(a)David M.Smith.

(b)T.W. Freeman.

(c)Peter Haggett.
(d)L. Dudley Stamp.

[UGC J-2008]

170.Which one of the following sequences related to the development of


geographical thought is correct'?

(a)Kant, Varenius, Vidal de la Blache, Humboldt

(b)Varenius, Kant,Humboldt,Vidal de la Blache

(c)Vidal de la Blache, Humboldt, Varenius, Kant

(d)Humboldt, Varenius, Kant, Vidal de la Blache

[UGC J-2008]

171.Match List I with List II and select the correct answer from the codes
given.

Codes:
[UGC J-2008]

172.Match List I with List II and select the correct answer using the codes
given.

Codes:

[UGC J-2011]

173.Match List I with List II and select the correct answer from the codes
given.
Codes:

[UGC J-2011]

174.Given below are two statements, one labelled as Assertion (A) and the
other labelled as Reason (R). Select your answer from the codes given.

Assertion (A): The work on geography by Strabo was largely an


encyclopaedic description of the world known to the Greeks.

Reason (R): Strabo's book had laid down a clear foundation for
chorological writing in geography.

(a)Both (A) and (R) are correct and (R) explains (A).

(b)Both (A) and (R) are correct but (R) does not explain (A).
(c)(A) is correct but (R) is false.

(d)(A) is false but (R) is true.

[UGC J-2011]

175.The correct sequence of the advent of the following geographers is

(a)Ritter, Humboldt, Richthofen, Ratzel.

(b)Humboldt, Ritter, Ratzel, Richthofen.

(c)Humboldt, Ritter, Richthofen, Ratzel.

(d)Ritter, Humboldt, Ratzel, Richthofen.

[UGC J-2011]

176.Kant viewed geography as

(a)spatial science.

(b)chorological science.

(c)regional science.

(d)systematic science.

[UGC J-2010]

177.Match List I with List II and select the correct answer from the codes
given.
Codes:

[UGC J-2010]

178.`The special purpose of geography is the comparison of unity in


multiplicity' is the statement given by

(a)Carl Ritter.

(b)Ratzel.

(c)Hartshorne.

(d)Humboldt.

[UGC J-2010]

179.Match List I with List II and select the correct answer from the codes
given.
Codes:

[UGC D-2011]

180.Arrange the following authors in chronological orderusing the codes


given.

1.Strabo

2.Ptolemy

3.Herodotus

4.Erastothenes
(a)Herodotus, Erastothenes, Strabo, Ptolemy

(b)Erastothenes, Strabo, Ptolemy, Herodotus

(c)Herodotus, Ptolemy, Strabo, Erastothenes

(d)Erastothenes, Herodotus, Ptolemy, Strabo

[UGC D-2011]

181.Who was the first geographer to ascertain the length of the Equator'?

(a)Eratosthenes

(b)Herodotus

(c)Anaximander

(d)Thales

[UGC D-2010]

182.Who was the first person to plot the Bay of Bengal?

(a)Strabo

(b)Ptolemy

(c)Al-Masudi

(d)Humboldt

[UGC D-2010]

183.Who is the author of the book Modern Geographical Thought?


(a)David Harvey

(b)R.J. Johnston

(c)Brian J.L.Berry

(d)Richard Peet

[UGC D-2010]

184.Match the following List I with List II and select the correct answer
from the codes given.

Codes:

[UGC D-2010]

185.A Handbook of Commercial Geography is a translated work published


in 1889. Who is the translator'?

(a)Dudley Stamp
(b)George Chisholm

(c)Von Thunen

(d)Karl Marx

[UGC D-2010]

186.French geographer Vidal de la Blache

(a)thought that how people obtained their livelihoods was important for
the geographer.

(b)got his formal geographic training under Paul Knox and Sallie
Marston.

(c)found that industrialization had only a small impact on rural France.

(d)concentrated on studying large geographical units.

187.Human geography is best described as the

(a)study of how human activity is organized in space.

(b)relationship between people and their environment(s).

(c)description of the world's environments in terms of its landforms.

(d)Both (a) and (b)

188.According to Knox and Marston, globalization of the last 25 years is


linked to all of the following except

(a)new international division of labour.


(b)growth and homogenization of consumer markets.

(c)internationalization of finance.

(d)end of neo-colonialism.

189.According to Knox and Marston, the `short 20th century' technology


system was powered around a cluster of technological innovations based
on

(a)nuclear power.

(b)coal and coal-powered steam engines.

(c)solar power.

(d)oil and the internal combustion engine.

190.The founder of historical materialism was

(a)Karl Marx.

(b)Arnold Toynbee.

(c)Charles Darwin.

(d)Walter Lippmann.

191.The 19th-century French approach of possibilism stated that

(a)the environment determines cultural characteristics.

(b)environmental change is not possible.

(c)people should adopt practices that do not harm the environment.


(d)the environment sets limits on what is feasible but does not dictate a
culture's choices.

192.Drawing from social Darwinism, Friedrich Ratzel's model of the state in


which state growth and change were `natural' and inevitable portrayed the
state as a

(a)religion.

(b)biological organism.

(c)soldier.

(d)volcano.

193.Match List I with List II and select the correct answer using the code
given.

Codes:
[UPSC-1995]

194.Which of the following scholars are related to the geocratic viewpoint in


human geography?

1.Humboldt

2.Ritter

3.Ratzel

4.Semple

Select the correct answer using the codes given.

(a)3 and 4

(b)2 and 4

(c)1, 2 and 3

(d)1, 2, 3 and 4

[UPSC-1996]

195.The concept of `stop-and-godeterminism' was put forward by

(a)Griffith Taylor.
(b)Jean Brunhes.

(c)J.E. Spencer.

(d)Vidal de la Blache.

[UPSC-1997]

196.The most salient contribution to the study of `spatial diffusion processes'


was made by

(a)Hagerstrand.

(b)Haggett.

(c)Harshorne.

(d)Hirschcmann.

[UPSC-1997]

197.Match List I with List II and select the correct answer using the codes
given.

Codes:
[UPSC-2002]

198.Consider the following:

1.The Geographical Pivot of History

2.The Normal Cycle of Erosion

3.Regenstein's Laws of Migration

4.Ratzel's Anthropogeographie

Their correct chronological sequence is

(a)2, 3, 1, 4.

(b)3, 1, 2, 4.

(c)4, 2, 3, 1.

(d)3, 4, 2, 1.

[UPSC-2002]

199.Which one of the following geographers has developed the idea of the
`mobility transition'?

(a)L.A. Kosinski
(b)E.G. Ravenstein

(c)E.S. Lee

(d)W. Zelinsky

[UPSC-2004]

200.Who amongst the following founded the French School of Regional


Geography?

(a)Paul Vidal de la Blache

(b)Philippe Pinchemel

(c)Elisse Reclus

(d)Lucien Gallois

[UPSC-2004]

ANSWER KEYS
 
1.About 50 per cent of the world's population is concentrated in between the
latitudes of

(a)5°N and 20°N

(b)20°N and 40°N

(c)40°N and 60°N

(d)20°N and 40°N

[UPSC-1997]

2.According to archaeological evidence, the reason for establishment of


settlements was

(a)economical.

(b)cultural.

(c)fortification.

(d)immigration.

3.According to the United Nation's projections, the Earth's population will


be approximately by 2050.

(a)7 billion

(b)8 billion
(c)9 billion

(d)10 billion

4.An empirical method used to describe the city-size distribution of many


countries and regions is called

(a)urban hierarchy.

(b)rank-size rule.

(c)central place model.

(d)urban location model.

[UPSC-1995]

5.Approximately half of the world's population lives in

(a)east Asia and Europe.

(b)Europe and eastern-north America.

(c)south and Southeast Asia.

(d)east and south Asia.

[UPSC-1995]

6.As per India's National Population Policy, 2000, by which of the following
years is our long-term objective to achieve population stabilization?

(a)2025

(b)2035
(c)2045

(d)2055

[UPSC-2008]

7.Assertion (A): Great Britain and New Zealand are remarkably similar in
terms ofarea, oceanic location and climate, yet Great Britain is densely
populated and an important centre of political and economic power.

Reason (R): New Zealand unlike Great Britain suffers from the
disadvantage of its location in the vast expanse of ocean in the Southern
Hemisphere.

(a)Both A and R are true and R is the correct explanation of A.

(b)Both A and R are true but R is not the correct explanation of A.

(c)A is true but R is false.

(d)A is false but R is true.

[UPSC-2003]

8.Assertion (A): A negligible proportion of the people of the hot desert


regions of the world are engaged in industrial pursuits.

Reason (R): Those regions are very poorly endowed with mineral and/or
energy resources.

(a)Both A and R are true and R is the correct explanation of A.

(b)Both A and R are true but R is not the correct explanation of A.


(c)A is true but R is false.

(d)A is false but R is true.

[UPSC-1995]

9.Assertion (A): A warm moist climate helps a wide variety of plants and
animal forms to flourish yet many of these regions such as Amazon basin
are sparsely populated.

Reason (R): Many of the plant and animal life is hostile to humans and
agriculture.

(a)Both A and R are true and R is the correct explanation of A.

(b)Both A and R are true but R is not the correct explanation of A.

(c)A is true but R is false.

(d)A is false but R is true.

10.Assertion (A): In developed countries, the economically active


population in agriculture sector has declined.

Reason (R): The per capita arable land has also significantly fallen down.

(a)Both A and R are true and R is the correct explanation of A.

(b)Both A and R are true but R is not the correct explanation of A.

(c)A is true but R is false.

(d)A is false but R is true.

[UPSC-2002]
11.Assertion (A): In Kerala, the density of human settlement decreases from
south to north along the coast.

Reason (R): The width of the coastal plain decreases form south to north.

(a)Both A and R are true and R is the correct explanation of A.

(b)Both A and R are true but R is not the correct explanation of A.

(c)A is true but R is false.

(d)A is false but R is true.

[UPSC-1995]

12.Assertion (A): Outmigration results in increase in employment


opportunities and wage rates a loss in investment.

Reason (R): Generally young people migrate and outmigration areas have
fewer numbers of young adults relative to the number of children and
aged people.

(a)Both A and R are true and R is the correct explanation of A.

(b)Both A and R are true but R is not the correct explanation of A.

(c)A is true but R is false.

(d)A is false but R is true.

[UPSC-2002]

13.Assertion (A): The Darwinian theory of natural selection sometimes


referred to as `the survival of the fittest' explains the origin of any species
as a normal process of descent from parent forms.

Reason (R): The contemporary distribution pattern of many organisms is


often

the result of natural migration or dispersal from original centre(s) of


development.

(a)Both A and R are true and R is the correct explanation of A.

(b)Both A and R are true but R is not the correct explanation of A.

(c)A is true but R is false.

(d)A is false but R is true.

[UPSC-1999]

14.Assertion (A): The first major group of migrants from the UK to


northeastern USA were the `Pilgrim Fathers'.

Reason (R): Roman Catholics were allowed to practice the religion fully
in the UK at that time.

(a)Both A and R are true and R is the correct explanation of A.

(b)Both A and R are true but R is not the correct explanation of A.

(c)A is true but R is false.

(d)A is false but R is true.

[UPSC-1998]
15.Assertion (A): Homo sapiens are generally divided into Caucasoid,
Mongoloid, Negroid and Australoid races.

Reason (R): Terminal charges are reduced.

(a)Both A and R are true and R is the correct explanation of A.

(b)Both A and R are true but R is not the correct explanation of A.

(c)A is true but R is false.

(d)A is false but R is true.

[UPSC-1998]

16.Assertion (A): The oldest urban settlements developed around 8200 bc.

Reason (R): Human settlements developed with the domestication of


plants and animals.

(a)Both A and R are true and R is the correct explanation of A.

(b)Both A and R are true but R is not the correct explanation of A.

(c)A is true but R is false.

(d)A is false but R is true.

[UPSC-2002]

17.Assertion (A): The study of sex composition of population assumes


added significance for national planners.

Reason (R): The two sexes play partly complementary roles in the
economy and society.
(a)Both A and R are true and R is the correct explanation of A.

(b)Both A and R are true but R is not the correct explanation of A.

(c)A is true but R is false.

(d)A is false but R is true.

[UPSC-1995]

18.Celtic, Germanic and Ural-Altaic language groups migrated to west


because

(a)the grasslands of Central Asia dried up.

(b)the expanding Chinese Empire disrupted the life of nomadic groups.

(c)they were pushed westwards when the eastern most groups moved
west.

(d)these were forced migrations which resulted from the slave trade.

[UPSC-2003]

19.Christaller postulated his central place theory which does not refer to

(a)K = 7.

(b)K=4.

(c)K = 3.

(d)K = 1.

[UPSC-1995]
20.Consider the following cities:

1.Canton

2.Peking

3.Singapore

4.Tehran

The correct sequence of these cities in ascending order in terms of


average annual temperatures is

(a)1, 3, 2, 4

(b)2, 4, 1, 2

(c)3, 1, 4, 2

(d)4, 2, 3, 1

[UPSC-2002]

21.Consider the following countries:

1.Brazil

2.Indonesia

3.Iran

4.Pakistan

Which one of the following is the correct sequence of the above countries
in the descending order of their population size'?
(a)1, 3, 2, 4

(b)2, 4, 3, 1

(c)2, 1, 4, 3

(d)3, 2, 1, 4

[UPSC-2005]

22.Consider the following countries:

1.Brazil

2.Indonesia

3.Japan

4.Russia

What is the descending order of the size of the following countries


population-wise?

(a)1, 2, 4, 3

(b)2, 3, 1, 4

(c)2, 1, 4, 3

(d)1, 2, 3, 4

[UPSC-2002]

23.Consider the following countries:


1.Belgium

2.Italy

3.Netherlands

4.United Kingdom

Their correct sequence in descending order in terms of population density


is

(a)3, 1, 2, 3

(b)2, 4, 1, 3

(c)3, 1, 4, 2

(d)4, 2, 3, 1

[UPSC-2002]

24.Consider the following countries:

1.Bangladesh

2.Brazil

3.Indonesia

4.Japan

The correct sequence of these counties is descending order of their


population is

(a)1, 2, 3, 4
(b)2, 1, 4, 3

(c)3, 2, 4, 1

(d)4, 3, 2, 1

[UPSC-1995]

25.Consider the following rural settlements patterns:

1.Block pattern

2.Circular pattern

3.Fan pattern

Select the correct combination of areas from amongst the following in


which 1, 2 and 3 are found, respectively

(a)Delta, desert, lakeside

(b)Desert, delta, lakeside

(c)Desert, lakeside, delta

(d)Delta, lakeside, desert

[UPSC-2004]

26.Consider the following situations:

1.Very low birth rate and low death rate

2.Birth rate greater than death rate


3.Low birth and death rates

Demographic transition in a country follows the sequence

(a)3, 2, 4, 1

(b)1,3,2,4

(c)1, 4, 2, 3

(d)4, 3, 2, 1

[UPSC-1995]

27.Consider the following statements about the Central Place Theory:

1.It provides an account of the size and distribution of settlements.

2.it assumes that shopowners and consumers are rational persons.

3.It is based on octagonal settlements.

4.It is based on three-order settlements.

Of these statements

(a)1, 2 and 3 are correct

(b)2, 3 and 4 are correct

(c)1, 2 and 4 are correct

(d)1, 3 and 4 are correct

[UPSC-1999]
28.Consider the following statements in respect of the United Arab
Emirates:

1.United Arab Emirates is surrounded in the northwest by Oman.

2.United Arab Emirates has highest percentage of the foreigners in its


population in the world.

3.Dubai is the most populous of the Emirates.

Which of the statements given above is/ are correct?

(a)1 and 3

(b)2 and 3

(c)2 only

(d)none

[UPSC-2005]

29.Consider the following statements:

1.The shelter of Bedouins is known as Khaimas.

2.The shelter of Kirghiz is called as Kraals.

3.The cattle shade of the Bakarwals of Jammu and Kashmir is known as


kothas.

4.The Innuit's shelters are known as igloos.

Which of these statements are correct'?


(a)1, 2 and 3

(b)2, 3 and 4

(c)1 and 3

(d)1 and 4

[UPSC-2003]

30.Consider the following statements:

1.The present infant mortality rate in India is about 80 per 1,200 live
births.

2.The Tenth Plan envisages a reduction in infant mortality rate to 45 per


1,200 live births.

3.The Tenth Plan envisages a reduction in decadal growth rate of the


population between 2001 and 2011 to 10 per cent.

Which of the statements given above is/ are correct?

(a)1 and 2

(b)2 only

(c)1 and 3

(d)1, 2 and 3

[UPSC-2005]

31.Consider the following statements:


1.The population density of Japan is close to that of India.

2.Crude birth rate of China is much lower than that of India.

3.Surface area of India is about five times that of UK.

Which of the statements given above are correct?

(a)1 and 2

(b)2 and 3

(c)1 and 3

(d)1, 2 and 3

[UPSC-2005]

32.Consider the following statements:

The term Barbary States denotes

1.the region comprising Morocco, Tunisia and the northern part of


Algeria.

2.the original population of the region Barbers.

3.the irrigated crops which account for a large part of the value of all
agricultural exports.

Of these statements

(a)1, 2 and 3 are correct

(b)1 and 2 are correct


(c)2 and 3 are correct

(d)1 and 3 are correct

[UPSC-1995]

33.Consider the following statements:

1.Long, wavy soft hair

2.Narrow to medium broad face

3.Very broad, flat broad to very broad nose

Of these statements

(a)1, 2 and 3 are correct

(b)1 and 2 are correct

(c)2 and 3 are correct

(d)1 and 3 are correct

[UPSC-1995]

34.Consider the following statements:

The problem of overpopulation in most of Asia is due to

1.large birth rate and large death rate.

2.inadequate agroindustrial development.

3.immobility of the people.


4.the continent being the original home of mankind.

Of these statements

(a)1 and 4 are correct

(b)2 alone is correct

(c)2 and 3 are correct

(d)1, 2, 3 and 4 are correct

[UPSC-1995]

35.Consider the following statements:

1.The distribution of population in the southern continents is peripheral.

2.Among the continents, Europe has the highest density of population.

3.The most densely populated region of the USA is its Californian coast.

4.Equatorial region generally has low density of population due to dense


forest.

Of these statements

(a)1, 3 and 4 are correct

(b)1, 2 and 4 are correct

(c)2, 3 and 4 are correct

(d)1 and 2 are correct


[UPSC-1998]

36.Consider the following statements:

1.As per the optimum theory of population, per capita income will
increase with the population growth in under populated areas

2.According to Malthus, population doubles once every 20 years

3.Celibacy, self-control and late marriages are the preventive checks


postulated by Malthus.

4.According to Malthus, war, earthquake and flood are some of the


positive checks on population growth.

Of these statements

(a)1, 3 and 4 correct

(b)1, 2 and 4 are correct

(c)2, 3 and 4 are correct

(d)1, 2 and 3 are correct

[UPSC-1998]

37.Consider the following statements:

According to the Malthusian model

1.population tends to increase arithmetically.

2.positive and preventive checks prevent excessive growth rate of


population.
3.the basis of preventive checks is `moral restraint'.

4.postponement of marriage is to be emphasized.

Of these statements

(a)1 and 2 are correct

(b)1 and 2 are correct

(c)1 and 2 are correct

(d)1 and 2 are correct

[UPSC-1999]

38.Consider the following statements:

Demographic transitions is a general model

1.describing the evolution of levels of fertility and mortality.

2.that has been developed with particular reference to developed


countries.

3.that has been accepted as the universal model of population growth.

4.that suggests for highly stylized phases in the process.

Which of the above statements are correct?

(a)1, 2 and 3

(b)2, 3 and 4
(c)1, 2 and 4

(d)1, 3 and 4

[UPSC-1999]

39.Consider the following statements:

1.Low-income countries of Asia lower level of urbanization but had some


of the largest citizens in the world.

2.Most of the Southeast Asian countries have primate patterns of


urbanization.

3.In the addition of world urban population after 1950, the share of
developed countries is higher than that of the developing countries.

Which of the above statements are correct'?

(a)1 and 2

(b)2 and 3

(c)1 and 3

(d)1, 2 and 3

[UPSC-1999]

40.For India, China, The UK and the USA, which one of the following is the
correct sequence of the median age of their populations'?

(a)China < India < UK < USA

(b)India < China < USA < UK


(c)China < India < USA < UK

(d)India < China < UK < USA

[UPSC-2008]

41.In Central Place theory, K = 7 settlement denotes

(a)market town.

(b)market-cum-administrative town.

(c)administrative town.

(d)transport town.

[UPSC-2005]

42.In Christaller's Central Place theory for organization of settlement, K = 4


denotes

(a)transport principle.

(b)market principle.

(c)administrative principle.

(d)economic principle.

[UPSC-1997]

43.In developing countries, the population to pressure increase in some parts


due immigration. Such parts are characterized by

(a)low cost of living.


(b)more suitable climate.

(c)large employment opportunities.

(d)accessibility.

[UPSC-1995]

44.In Lee's opinion, which of the following factors are responsible for
migration'?

1.Factors operating in the area of origin

2.Factors operating at the destination

3.Factors that act as intervening obstacles

4.Personal factors that are specific to individuals

Select the correct answer using the codes given.

(a)1, 2 and 3

(b)1, 2 and 4

(c)3 and 4

(d)1, 2, 3 and 4

[UPSC-2003]

45.In stage one of the demographic transition, the crude death rate is
relatively high due to

(a)practice of human sacrifice.


(b)periodic epidemics.

(c)practice of cannibalism.

(d)difficulty of getting correct numbers.

46.In stage three of the demographic transition, children are seen as

(a)economic assets.

(b)expenses.

(c)blessing of God.

(d)a necessary evil which cannot be avoided.

47.In the Newling model of urban population density distribution, the term
`density crater' is applied to

(a)central business district.

(b)high-income residential area.

(c)suburb.

(d)zone in transition.

[UPSC-2001]

48.India can be regarded to be in

(a)stage one of the demographic transition.

(b)stage two of the demographic transition.


(c)stage three of the demographic transition.

(d)none of these.

49.Infant mortality and birth tend to vary together, which suggests that

(a)decrease in one leads to a decrease in the other.

(b)decrease in one leads to an increase in the other.

(c)increase in one leads to a decrease in the other.

(d)increase in one leads to an increase in the other.

[UPSC-1995]

50.Internally displaced people are those who

(a)have to flee their countries due to opposition and/or unrest.

(b)have to flee from one region to another in their own country.

(c)who are targets of religious persecutions.

(d)none of these.

51.Life expectancy is the highest in the world in

(a)Canada.

(b)Germany.

(c)Japan.

(d)Norway.
[UPSC-2003]

52.Marxist philosophy of population dynamics states that

(a)population growth is not checked by the society.

(b)economic and social benefits can only be enhanced by an increase in


the labour force.

(c)nature will reduce surplus population by war, diseases, famine and


vice.

(d)man tends to reproduce prolifically.

[UPSC-1995]

53.Masai belongs to

(a)Gobi desert.

(b)Atacama desert.

(c)tropical Africa.

(d)Sahara desert.

[UPSC-1995]

54.Match List I (book) with List II (author) and select the correct answer
using the codes given.
Codes:

[UPSC-2004]

55.Match List I (factors of population problem) with List II (corresponding


examples) and select the correct answer using the codes given.
Codes:

[UPSC-2004]

56.Match List I (settlement pattern) with List II (region) and select the
correct answer using the codes given.
Codes:

[UPSC-2005]

57.Match List I (language) with List II (country) and select the correct
answer using the codes given.
Codes:

[UPSC-1995]

58.Match List I (tribe) with List II (country) and select the correct answer
using the codes given.

Codes:

[UPSC-1995]

59.Match List I (races) with List II (place of habitation) and select the
correct answer using the codes given.
Codes:

[UPSC-1995]

60.Match List I (tribe) with List II (country) and select the correct answer
using the codes given.

Codes:
[UPSC-1995]

61.Match List I (tribe) with List II (region/ country) and select the correct
answer using the codes given.

Codes:

[UPSC-1999]

62.Match List I (country) with List II (indigenous) and select the correct
answer using the codes given.
Codes:

[UPSC-1999]

63.Match List I (tribe) with List II (country) and select the correct answer
using the codes given.

Codes:
[UPSC-2002]

64.Match List I (characteristic features) with List II (country) and select the
correct answer using the code given.

Codes:

[UPSC-2003]

65.Match List I (tribe) with List II (place of habitat) and select the correct
answer using the codes given.
Codes:

[UPSC-2004]

66.Match List I (climate) with List II (house type) and select the correct
answer using the codes given.

Codes:
[UPSC-1995]

67.Match List I (classification of towns) with List II (example of towns) and


select the correct answer using the codes given.

Codes:

[UPSC-1997]

68.Match List I (country) with List II (important river) and select the correct
answer using the code given.
Codes:

[UPSC-1997]

69.Match List I (term/model) with List II (author) and select the correct
answer using the codes given.

Codes:
[UPSC-2001]

70.Match List I (settlements/hamlets) with List II (description) and select the


correct answer using the codes given.

Codes:
[UPSC-2003]

71.Match List I (process of urbanization) with List II (stage of urbanization)


and select the correct answer using the codes given.

Codes:

[UPSC-2005]

72.Match List I (proponent) with List II (criterion for functional


classification) and select the correct answer using the codes given.
Codes:

[UPSC-2005]

73.Maximum human population belongs to the

(a)Australoid.

(b)Negroid.

(c)Mongoloid.

(d)Caucasian.

[UPSC-1995]

74.Most of the people in the world live in lowland areas except those in
(a)central and western Africa.

(b)southern and southeastern Europe.

(c)southern and Southeast Asia.

(d)central America and northwestern South.

[UPSC-2004]

75.Population explosion is associated with

(a)high birth rate and low death rate.

(b)high birth rate and high death rate.

(c)low birth rate and high death rate.

(d)low birth rate and low death rate.

[UPSC-1995]

76.Since 1950, the trend of urbanization in west Africa has led to

(a)migration of people from the coast to inland.

(b)migration of people from the upland to lowland.

(c)concentration of people in the Savannah belt.

(d)growth of shanty towns around the cities.

[UPSC-1999]

77.Southwest Asia is inhabited chiefly by


(a)Caucasians.

(b)Mongoloids.

(c)Negroes.

(d)Red Indians.

[UPSC-1995]

78.The aboriginal population of the Andaman Islands belongs to which one


of the following racial categories'?

(a)Negroids

(b)Negrito

(c)Palaeo-Australodis

(d)Negillos

[UPSC-2002]

79.The agriculture density of population is represented by


[UPSC-1995]

80.The area where several cities grow and merge together into vast urban
areas is called

(a)primate city.

(b)conurbation.

(c)cosmopolitan.

(d)none of these.

81.The basic distinction between urban and rural settlements is the

(a)population size.

(b)population density.

(c)function.

(d)location.

[UPSC-1997]

82.The clustered pattern of a village depends upon

(a)distance between villages.

(b)placing of tenements.

(c)population of a village.
(d)location of settlement.

[UPSC-1996]

83.The correct sequence in the descending order with respect to life


expectancy at birth of the given countries is

(a)China, India, Sri Lanka, Pakistan

(b)China, Sri Lanka, Pakistan, India

(c)Sri Lanka, India, China, Pakistan

(d)Sri Lanka, China, Pakistan, India

[UPSC-1999]

84.The correct sequence in the descending order with respect to present total
population of the given countries is

(a)Columbia, Brazil, USA, Mexico

(b)Mexico, USA, Columbia, Brazil

(c)USA, Brazil, Mexico, Columbia

(d)Brazil, USA, Mexico, Columbia

[UPSC-1995]

85.The doubling period of population is the lowest in

(a)Bangladesh.

(b)Egypt.
(c)Iran.

(d)Pakistan.

[UPSC-1999]

86.The fertility rate of is considered as replacement rate.

(a)1.2

(b)2.1

(c)1.5

(d)2.5

87.The first scholar to formulate the laws of migration was

(a)L.A. Kosinki

(b)E. Lee

(c)E.G. Ravenstein

(d)W. Zelinsky

[UPSC-1999]

88.The high density of population in Nile Valley and Islands of Java is


primarily due to

(a)intensive agriculture.

(b)industrialization.
(c)urbanization.

(d)topographic constraints.

[UPSC-2001]

89.The highest growth rate of population is found in

(a)India.

(b)Malaysia.

(c)Bangladesh.

(d)Sri Lanka.

[UPSC-1995]

90.The international migrations of populations in Eurasia in the late 1940s


after the Second World War were caused by

(a)natural calamities such as floods and famines.

(b)favourable immigration laws.

(c)religious fanaticism.

(d)stabilized international situation.

[UPSC-1995]

91.The Islamic slave system which is considered a part of African Diaspora


lasted for approximately

(a)3 centuries.
(b)7 centuries.

(c)10 centuries.

(d)12 centuries.

92.The largest number of tribal groups live in

(a)Uganda.

(b)Tanzania.

(c)South Africa.

(d)Nigeria.

[UPSC-1995]

93.The largest religious system in the world in both geographic extent and
number of adherents is

(a)Christianity.

(b)Islam.

(c)Buddhism.

(d)Hinduism.

[UPSC-1995]

94.The largest urban agglomeration in China is

(a)Shangha.
(b)Canton.

(c)Beijing.

(d)Wu-Han.

[UPSC-1997]

95.The Mexico city is regarded as geography's great paradox

(a)because it testifies to the power of history and geographical inertia.

(b)it is located in one of the most unfavourable spots on the Earth.

(c)it is supporting population thrice its maximum capacity.

(d)it changes its location every year.

96.The Mobility Transition Model of Migration was propounded by

(a)W.A.V. Clark

(b)E. Lee

(c)E.G. Ravenstein

(d)W. Zelinsky

[UPSC-2001]

97.The Mongol culture hearth was originally situated in

(a)southern China.

(b)northern Manchuria.
(c)eastern China.

(d)Mongolia.

[UPSC-1995]

98.The most densely populated African country is

(a)Sudan.

(b)Egypt.

(c)Libya.

(d)Uganda.

[UPSC-1995]

99.The most widely accepted scheme of cultural realms was given by

(a)Spencer and Thomas.

(b)Brock and Webb.

(c)Haggett and Chorley.

(d)Dicken and Pitts.

[UPSC-1995]

100.The original home of the gypsies was

(a)Egypt.

(b)Russia.
(c)India.

(d)Persia.

[UPSC-1995]

101.The population density of city state of Singapore is approximately

(a)3,500 people per square kilometre.

(b)5,000 people per square kilometre.

(c)6,500 people per square kilometre.

(d)8,000 people per square kilometre.

102.The process of defining a city territory and establishing a government is


called

(a)megapolis.

(b)establishment.

(c)incorporation.

(d)urbanization.

103.The Red Indians or the American Indians belong to the

(a)Mongolian race.

(b)Caucasoid race.

(c)Australoid race.
(d)Negrito race.

[UPSC-2003]

104.The region to which any city provides services and upon which it draws
for its needs is called its

(a)hinterland.

(b)rimland.

(c)heartland.

(d)primate.

105.The rich countries of the world today demonstrate

(a)stage one of the demographic transition.

(b)stage two of the demographic transition.

(c)stage three of the demographic transition.

(d)none of these.

106.The Semitic people belong to the

(a)Caucasoid race.

(b)Mongoloid race.

(c)Negroid race.

(d)Australoid race.
[UPSC-2002]

107.The spread of cultural elements or complexes from one society to


another is called

(a)diffusion.

(b)acculturation.

(c)assimilation.

(d)transmission.

[UPSC-2003]

108.Tribes belonging to which one of the following groups earn their


livelihood through food gathering and hunting?

(a)Bushman, Pigmy and Eskimo

(b)Masai, Kirghiz and Boro

(c)Pigmy, Eskimo and Kirghiz

(d)Boro, Bushman and Masai

[UPSC-2005]

109.Which among the following countries has the largest population?

(a)Indonesia

(b)Japan

(c)Pakistan
(d)Sudan

[UPSC-2003]

110.Which of the following are associated with Saudi Arabia'?

1.High proportion of urban population

2.Extensive agriculture pattern

3.Considerable extent of land under pasture lands.

Select the correct answer using the codes given below:

(a)1, 2 and 3

(b)1 and 2

(c)2 and 3

(d)1 and 3

[UPSC-2004]

111.Which of the following are associated with the origin of dispersed


settlements in the UK and USA?

1.Desire of family privacy

2.Farmer's easy access to fields

3.Law and order in countryside

4.Uneven topography of farm lands


Select the correct answer using the codes given below:

(a)1 and 3

(b)2 and 4

(c)1 and 4

(d)3 and 4

[UPSC-2005]

112.Which of the following are represented in the population pyramid'?

1.Age

2.Gender

3.Birth rate

4.Death rate

5.Growth rate

(a)3, 2 and 5

(b)1, 2 and 5

(c)2, 3 and 4

(d)1 and 2

113.Which of the following are the characteristics of stage two of


demographic transition'?
1.Infant mortality rate is high.

2.Infant mortality rate is low.

3.Crude birth rate is high.

4.Crude birth rate is low.

(a)1 and 3

(b)1 and 4

(c)2 and 3

(d)2 and 4

114.Which of the following are the characteristics of stage two of


demographic transition'?

1.Infant mortality rate is high.

2.Infant mortality rate is low.

3.Crude birth rate is high.

4.Crude birth rate is low.

(a)1 and 3

(b)1 and 4

(c)2 and 3

(d)2 and 4
115.Which of the following characteristics symbolize the oriental cultural
realm'?

1.Industrialization

2.Joint family bond

3.Subsistence agriculture

4.Urbanization

Select the correct answer using the codes given below:

(a)1, 2 and 4

(b)2 and 3

(c)1 and 3

(d)2, 3 and 4

[UPSC-1995]

116.Which of the following pairs are correctly matched?

1.Primary sex ratio: sex at the time to conception

2.Secondary sex ratio: sex at the time of birth

3.Tertiary sex ratio: sex at the time or enumeration

4.Imbalanced sex ratio: sex of workers

Select the correct answer using the codes given below.


(a)1, 2, 3 and 4

(b)1, 3 and 4

(c)2 and 4

(d)1, 2 and 3

[UPSC-1999]

117.Which of the following pairs are not correctly matched?

1.Indonesians - Australoid

2.North Europeans - Caucasoid

3.West Europeans - Mongoloid

4.South Americans - Negroid

(a)1 and2

(b)1, 3 and 4

(c)2 and 3

(d)2 and 4

[UPSC-1995]

118.Which of the following statements best describe the relationship


between population distribution and environmental factors'?

(a)There is no correlation between the environment and the population


distribution.
(b)Cold and dry areas generally have high population density, e.g.,
Norway and Switzerland.

(c)Cold and dry areas generally have low population density.

(d)None of these.

119.Which of the following statements hold true about human settlement?

1.At least 75 per cent of human population lives in the Northern


Hemisphere.

2.At least 75 per cent of human population lives between 20oN and 60oN
latitude.

3.About 90 per cent of the population is concentrated on less than 20 per


cent of the land.

(a)1 and 2

(b)2 and 3

(c)3 and 1

(d)1, 2 and 3

120.Which of the following statements hold true for growth of world's


population?

1.Seventy five per cent of population increase projected by UN for the


year 2050 will be in the countries least able to support a larger
population.

2.During 1995-2000. Europe showed no population growth.


(a)Only 1

(b)Only 2

(c)Both 1 and 2

(d)Neither 1 nor 2

121.Which of the following statements hold true for human settlements?

(a)There is a clear correlation between the density of population and


welfare of the people.

(b)Physiological density cannot tell us about the country's ability to


support its population.

(c)Arithmetic density tells us about the country's ability to support its


population.

(d)None of these.

122.Which one among the following South Asian countries has the highest
population density'?

(a)India

(b)Nepal

(c)Pakistan

(d)Sri Lanka

[UPSC-2009]

123.Which of the following continents has the lowest birth and death rates'?
(a)Europe

(b)Australia

(c)North America

(d)South America

[UPSC-2004]

124.Which of the following does not represent the primitive gathering in the
latitudes?

(a)Yukaghirs

(b)Hahjans

(c)Eskimos

(d)Auca India

[UPSC-2003]

125.Which of the following is the aboriginal tribe of Malaysia'?

(a)Ainu

(b)Fula

(c)Sakai

(d)Onge

[UPSC-1999]
126.Which of the following models is concerned with the principle that `a
central city would generally be surrounded by concentric zones'?

(a)Agricultural regions

(b)Agricultural locations

(c)Concentric zones

(d)Isolated state

[UPSC-1999]

127.Which of the following pairs is correctly matched'?

[UPSC-1998]

128.Which of the following pairs is correctly matched'?

[UPSC-1999]

129.Which of the following pairs is not correctly matched?

(a)Tharu: Uttar Pradesh

(b)Gaddi: Himachal Pradesh


(c)Konyak: Kerala

(d)Toda: Tamil Nadu

[UPSC-1999]

130.Which of the following pairs is not correctly matched?

Theory relation to urban structure: proponent

(a)Concentric Theory: Sinclair

(b)Sector Theory: Homer Hoyt

(c)Central Place Theory: Christaller

(d)Multiple Nuclei Theory: Harris and Ullman

[UPSC-2005]

131.Which of the following pairs is not correctly matched?

Concept: Scholar

(a)Rank-size rule: G.K.Zipf

(b)Law of retail gravitation: W.J.Reilly

(c)Concept of conurbation: Patrick Geddes

(d)Concept of megalopolis: E.W.Burgess

[UPSC-2005]

132.Which of the following represents the correct sequence of descending


order of population size of the four given megacities of the world'?
(a)Tokyo-Mexico City-New YorkMumbai

(b)Mexico City-Tokyo-Mumbai-New York

(c)Mexico City-Tokyo-New YorkMumbai

(d)Tokyo-Mexico City-Mumbai-New York

[UPSC-1999]

133.Which of the following stages indicates the initiation of population


explosion in the demographic transition'?

(a)Stage I

(b)Stage II

(c)Stage III

(d)Stage IV

[UPSC-2003]

134.Which of the following statements is not correct'?

(a)Europeans immigrated into the densely populated tropical and


subtropical coastlands.

(b)The great exodus from Ireland in mid-1900s was mainly due to the
failure of potato crops.

(c)Chinese immigrants have contributed greatly to the economic


development of Malaysia.
(d)The cities of the northeastern USA are now experiencing net migration
losses.

[UPSC-2005]

135.Which of the following states was formed exclusively by the migrants in


the 20th century'?

(a)Maldives

(b)Mauritius

(c)Israel

(d)Myanmar

[UPSC-2003]

136.Which of the following was formulated by George K.Ziph in 1940 to


explain the observed movement of people'?

(a)Principal of least of effort

(b)Intervening opportunities

(c)Gravity model

(d)Mean information field

[UPSC-2004]

137.Which two countries follow China and India in the decreasing order of
their population'?

(a)Brazil and USA


(b)USA and Indonesia

(c)Canada and Malaysia

(d)Russia and Nigeria

[UPSC-2008]

138.Who said: `Similar locations lead to similar mode of life"?

(a)F. Ratzel

(b)Carl Ritter

(c)Alexander von Humboldt

(d)E.C. Sample

[UPSC-2003]

139.Xanthoderons is the term of skin applied to

(a)Caucasoid race.

(b)Negroid race.

(c)Mongoloid race.

(d)Australoid race.

[UPSC-1998]

140.The region to which a city provides services and from which it draws its
needs is its

(a)situation.
(b)hinterland.

(c)central place.

(d)economy.

141.The first cities appeared in

(a)Turkey and Iraq.

(b)Egypt.

(c)the Indus Valley.

(d)China.

142.is the process by which cities bring people and activities together in one
place for greater convenience.

(a)Incorporation

(b)Geomancy

(c)Urbanization

(d)Agglomeration

143.A firm that sells its products primarily to consumers outside a settlement
is a

(a)nonbasic industry.

(b)basic industry.

(c)functional classification.
(d)primate city.

144.When several cities grow and merge together into vast urban areas, they
are called

(a)primate cities.

(b)agglomerations.

(c)regions.

(d)conurbations.

145.Which of the following countries does not contain a primate city'?

(a)France

(b)United States

(c)Thailand

(d)Primate cities no longer exist.

146.By ad 100, Rome had probably reached a population of

(a)3,00,000.

(b)5,00,000.

(c)4,50,000.

(d)2,00,000.

147.Cities rely on their for food and provide them goods and services.
(a)populations

(b)farmers

(c)neighbouring cities

(d)hinterlands

148.Jobs in the basic sector often create or support jobs in the nonbasic
sector through the

(a)multiplier effect.

(b)reduction in government workers.

(c)efforts of employees to save money rather than spend money.

(d)development of primate cities.

149.Which of the following was NOT true of early settlements'?

(a)They contained society's schools, museums, libraries and archives.

(b)They protected the group's land claims and food sources.

(c)They were often surrounded by defensive walls.

(d)They housed women and children while the men searched for food.

150.In central place theory, the minimum demand for a product or service is
called the

(a)threshold.

(b)radius.
(c)hinterland.

(d)hierarchy.

151.The only geometric shape that meets the three requirements of


Christaller's central place theory is a pattern of

(a)polygons.

(b)hexagons.

(c)triangles.

(d)squares.

152.Gateway cities are those that

(a)developed in European countries.

(b)arise where two physical areas meet or at the border between two
cultures.

(c)are surrounded by walls.

(d)were established during the Industrial Revolution.

153.An example of a major city that is located in an unfavourable location


would be

(a)Boston.

(b)New York City.

(c)Louisville.
(d)Mexico City.

154.In an urban hierarchy, there is a relationship between the market area


and the diffusion of goods and services, which is an example of a region.

(a)vernacular

(b)formal

(c)peripheral

(d)functional

155.Itinerant businesses are those that

(a)travel from place to place to sell their goods.

(b)are based on providing convenience items.

(c)agglomerate services to save travel costs.

(d)are only open seasonally.

156.Central place theory

(a)was formulated by Walter Christaller.

(b)asserts that the ideal shape for the market hinterland is oval.

(c)asserts that market hinterlands must be of intermediate shape.

(d)states that cities and towns are surrounded by core areas.

157.Self-built houses on the periphery of cities in developing countries are


known as
(a)slums.

(b)ghettos.

(c)squatter settlements.

(d)refugee camps.

158.In the 17th century, became the first modem urban country, with over
one-half of its population living in cities and towns.

(a)The Netherlands

(b)Greece

(c)France

(d)England

159.Today urbanization is occurring in many places without economic


development, especially in

(a)rich countries.

(b)the United States.

(c)European countries.

(d)poor countries.

160.Governments often invest more in cities than in rural areas because it

(a)increases opportunities.

(b)is more cost-effective.


(c)is politically popular.

(d)improves migration.

161.Backyard shops, street hawkers and providers of simple services on


street corners constitute the

(a)basic economy.

(b)workforce.

(c)tax base.

(d)informal sector.

162.An estimated 4 billion people cannot create wealth or recuperate from


disaster because of the lack of

(a)legal records.

(b)money.

(c)credit.

(d)capital investment.

163.Which of the following is unlikely to be found in the zone of transition


that surrounds the CBD?

(a)Warehouses

(b)Light industry

(c)Upper-class residences
(d)Wholesaling

164.The central business district is characterized by

(a)a high degree of accessibility.

(b)single-family homes.

(c)low buildings.

(d)a random arrangement of different functions.

165.Functions within the CBD are often

(a)housed in one-storey buildings.

(b)exempt from zoning laws.

(c)clustered.

(d)randomly arranged.

166.This urban form model describes how highways disperse central city
activities around the region.

(a)Peripheral model

(b)Concentric zone model

(c)Sector model

(d)Multiple-nuclei model

167.According to the sector model, lowincome housing is located near


(a)industry and its associated transportation corridors.

(b)renovated inner city neighbourhoods.

(c)an outer ring surrounding the city.

(d)nodes near universities and parks.

168.Government restrictions on the use to which various parcels of land may


be put is called

(a)restrictive ordinances.

(b)districting.

(c)suburbanization.

(d)zoning.

169.According to the authors, the most important city planner of the 20th
century was

(a)Le Corbusier.

(b)Howard.

(c)Christaller.

(d)Frank Lloyd Wright.

170.Retail activities which tend to concentrate in the CBD include those


which have

(a)a low range.


(b)services for office workers.

(c)a manufacturing component.

(d)a low threshold.

171.Land values are high in the CBD primarily because of

(a)competition for limited space.

(b)less intensive land use.

(c)construction of skyscrapers.

(d)high threshold and range.

172.The multiple nuclei theory recognized that

(a)urban sprawl has developed in cities.

(b)residential areas are racially segregated.

(c)cities grow.

(d)cities develop around several nodes.

173.Which of the following groups is NOT likely to be attracted to a


gentrifying neighbourhood'?

(a)Empty-nesters

(b)The highly educated

(c)Young professionals
(d)Young family with many children

174.The process by which a city expands its boundaries is called

(a)incorporation.

(b)central place.

(c)annexation.

(d)rank-size rule.

175.The Federal Housing Administration (FHA) programs had the effect of

(a)integrating neighbourhoods racially.

(b)promoting gentrification.

(c)improving urban schools.

(d)promoting housing segregation.

176.When populations spread out from urban centres creating conurbations,


the Census Bureau created the termto describe these areas.

(a)high-density rural areas

(b)major urbanized regions

(c)metropolitan statistical areas

(d)agglomeration

177.The desire to live in suburbs relates to US fondness for


(a)heavy traffic.

(b)row houses and apartments.

(c)lower opportunity for home ownership.

(d)return to nature.

178.When a developer buys land beyond the edge of growth causing the
infrastructure network to be developed in an irregular pattern, it is called

(a)urban sprawl.

(b)leapfrogging.

(c)agglomeration.

(d)suburban growth.

179.The US government has encouraged the use of cars in part by

(a)protecting prime agricultural land.

(b)building interstate highways.

(c)constructing new subways.

(d)charging high gasoline taxes.

180.A measure of the economic impact of the young and old on the more
economically active and productive members of a population is known as
the

(a)youth elderly cohort.


(b)crude death rate.

(c)expectancy ratio.

(d)infant mortality rate.

181.Assume that Israel has a population of 7.5 million and Mexico has a
population of 107 million. Both have a rate of natural increase (RNI) of
1.6. This means that

(a)Israel's population will double more quickly than Mexico's.

(b)Israel's and Mexico's populations will double in the same amount of


time.

(c)Mexico's population will double more quickly than Israel's.

(d)Israel's and Mexico's populations will never double with such a low
RNI.

182.For which of the following populations is total population change equal


to the rate of natural increase'?

(a)World

(b)Los Angeles

(c)Egypt

(d)North America

183.The 20th century's great growth in world population is due to a


phenomenal
(a)rise in birth rates.

(b)global baby boom.

(c)increase in migration rates.

(d)decline in death rates.

184.Demography is the study of population

(a)attitudes.

(b)politics.

(c)characteristics.

(d)distribution.

185.The most widely used tool to assess the size, distribution and
characteristics of a population is

(a)geographic information system.

(b)geodemographic analyses.

(c)demographic transition.

(d)census.

186.Inaccuracies in US censuses

(a)are a big deal as they affect the distribution of professional sports


franchises around the country.
(b)are a big deal as they affect the distribution of federal revenues and
congressional seats.

(c)are the result of government mismanagement.

(d)are no big deal as they are just a count of the population, anyway.

187.A population density map of the world shows that the distribution of
people is most obviously affected by

(a)political and economic factors such as development, types of


government, trade history.

(b)preferences for places with tropical landscapes.

(c)cultural factors such as language, cuisine and religion.

(d)environmental factors such as natural resources, climate, water


availability, topography.

188.The number of people per unit of area (such as people per square
kilometre) is a measure of

(a)health density.

(b)crude density.

(c)natural density.

(d)nutritional density.

189.For countries of similar area and population, the one with the greatest
amount of is most likely to have the highest nutritional density.
(a)desert lands

(b)mountains

(c)farmland

(d)forest land

190.Of the following, which one generally has the highest population
density'?

(a)Peripheral countries

(b)Rural areas

(c)Urban areas

(d)Areas with high birth rates.

191.Age-sex pyramids are graphical representations of

(a)the structure of population at a moment in time.

(b)birth rates and death rates of a country.

(c)the distribution of a population over space.

(d)a hierarchy of core periphery relationships.

192.If an age-sex pyramid is asymmetrical, lopsided from one side to the


other, it means that

(a)the rate of natural increase is declining.

(b)males experienced events differently than females of the same cohorts.


(c)generation gaps are evident.

(d)the population is in the middle of the demographic transition.

193.To a demographer, a group of people that begin something at the same


time is known as

(a)a dependency ratio.

(b)a team.

(c)a transnational migrant.

(d)a cohort.

194.Which of the following best explains a population pyramid (age-sex


pyramid) that looks like an umbrella, with very narrow cohorts at the
bottom that widen broadly near the top'?

(a)The population is a college town.

(b)The population pyramid is upside down.

(c)The population has an open-door migration policy.

(d)The population is a retirement village.

195.As the baby boom ages and the elderly cohorts get larger, the
dependency ratio in the USA will

(a)fluctuate more and more.

(b)rise.
(c)disappear.

(d)drop.

196.The Crude Birth Rate (CBR) is NOT affected by which of the


following'?

(a)Opportunities for women

(b)The availability of birth control

(c)Level of economic development

(d)The CBR is influenced by all of the above.

197.A population in which the total fertility rate is 2 means that

(a)birth rates and death rates are roughly equal.

(b)the population is neither increasing nor decreasing.

(c)the population will double in about 40 years.

(d)both (a) and (b)

198.Crude death rates refer to the annual ratio of

(a)deaths to every 1,000 live births.

(b)deaths from preventable diseases to deaths from natural causes.

(c)deaths to every 1,000 people.

(d)accidental deaths to deaths from natural causes


199.Whether or not a given population grows is a function of

(a)immigration.

(b)emigration.

(c)birth rates.

(d)all of the above

200.The combination of the CBR and the CDR is (the)

(a)infant mortality rate.

(b)rate of natural increase.

(c)net migration.

(d)life expectancy.

201.The rate of natural increase is a useful indicator for examining

(a)access to nutrition and health.

(b)nutritional density.

(c)levels of economic development and growth.

(d)population growth.

202.When calculating the rate of natural increase, one does NOT include

(a)birth rates.

(b)death rates.
(c)migration.

(d)the rate of natural increase uses all of the above.

203.Infant mortality rates vary between countries. However, they do not


generally vary among the peoples and regions inside

(a)core countries.

(b)peripheral countries.

(c)semiperipheral countries.

(d)Infant mortality rates vary between the people and regions of all
countries.

204.In sub-Saharan African countries hit hard by HIV/AIDS, have dropped


most sharply.

(a)migration rates

(b)crude death rates

(c)crude birth rates

(d)life expectancies

205.The demographic transition is a model of population change used to


explain high population growth rates. According to the theory,

(a)birth rates decline, eventually followed by a decline in death rates.

(b)birth rates rise, eventually followed by a rise in death rates.


(c)death rates rise, eventually followed by a rise in birth rates.

(d)death rates decline, eventually followed by a decline in birth rates.

206.According to the demographic transition model, the rate of natural


(RNI) at the beginning (preindustrial phase) is generally than the RNI at
the end (industrial phase).

(a)lower

(b)higher

(c)the same as

(d)cannot generalize as it depends on the starting and ending birth and


death rates.

207.A Texan is the one who moves to Canada from Texas andto Canada.

(a)immigrates/emigrates

(b)emigrates/immigrates

(c)emigrates/emigrates

(d)immigrates/immigrates

208.If 1,500 people migrate into Goodhue, MN and 1,000 people move out,
gross migration is

(a)500.

(b)1,500.
(c)negative 500.

(d)2,500.

209.If 1,500 people migrate into Goodhue, MN and 1,000 people move out,
net migration is

(a)1,500.

(b)1,000.

(c)2,500.

(d)500.

210.Of the following, a typical example of a push factor of migration would


be

(a)abundant job opportunities.

(b)close family ties.

(c)beautiful weather.

(d)social conflict.

211.Hong Kong Chinese that maintain residences and businesses in Hong


Kong and Vancouver, and Turks that work in Germany while maintaining
close connections and property in Turkey, are specifically known as

(a)guest workers.

(b)transnational migrants.
(c)international emigrants.

(d)voluntary emigrants.

212.temporarily migrate to take jobs in other countries and send money


(remittance payments) to their home communities.

(a)Refugees

(b)International forced migrants

(c)Voluntary migrants

(d)Guest workers

213.In the USA, there have been numerous waves of internal voluntary
migration. These include all of the following except

(a)the great northwards migration of African-Americans.

(b)the westwards expansion of the frontier.

(c)the Trail of Tears.

(d)rural-to-urban migration of agricultural workers.

214.Moderate Neo-Malthusians believe that solutions to the global


population issue are best found in

(a)letting diseases thin out human populations.

(b)reducing the size of the periphery's population.

(c)the redistribution of wealth and resources.


(d)technological innovations.

215.Writing on the eve of the Industrial Revolution, Thomas Malthus argued


that population growth

(a)was so low, and food so abundant, that people would become


increasingly obese.

(b)and procreation were fundamental rights of human beings.

(c)presented numerous opportunities for technological innovation.

(d)would exceed the ability of people to feed themselves.

216.People who share Thomas Malthus' perspectives on population growth


are known as

(a)Thomasians.

(b)Neoliberals.

(c)Proto-Malthusians.

(d)Neo-Malthusians.

217.Most international population policies at the end of the 20th century


have emphasized

(a)increase death rates.

(b)reduced birth rates.

(c)increased opportunities for women.


(d)increased health care for children.

218.The diffusion of HIV/AIDS in subSaharan Africa has been most


prominent

(a)along transportation routes.

(b)among children and the elderly.

(c)in jungles and other similarly remote areas.

(d)among homosexuals and drug users.

219.Whites are most likely to marry outside of their racial group in the

(a)Midwest/North.

(b)South.

(c)Prairies and plains.

(d)West.

220.With just over 4 billion people, Asia has nearly two-thirds of the world's
population. From among those below, the world region with the smallest
population is

(a)Europe.

(b)Latin America and the Caribbean.

(c)North America.

(d)Africa.
221.The value and power of geographic information systems come with
linking demographic data to particular

(a)policies.

(b)censuses.

(c)populations.

(d)locations.

222.The 15- to 25-year period of low total fertility rates after the US baby
boom is known as the baby

(a)bear.

(b)boom echo.

(c)bust.

(d)bomb.

223.For which of the following is the US national average the same as that
for minority groups in the US?

(a)Infant mortality rate

(b)Death rate

(c)Birth rate

(d)All of the above


224.The world region with by far the greatest number of internally displaced
persons (IDPs)-nearly half of all 25 million-is

(a)the Middle East.

(b)Africa.

(c)Latin America.

(d)North America.

225.Generally, internally displaced persons (IDPs) suffer more than


international refugees because

(a)the world community treats refugees so well.

(b)foreign governments do not care.

(c)foreign governments do not know about the IDPs.

(d)their own governments are unable or unwilling to help.

226.In which of the following are Sweden, Poland, Yemen and Cambodia
most similar'?

(a)Infant mortality rate

(b)Crude death rate

(c)Total fertility rate

(d)Crude birth rate


227.For over 200 years, a census of the entire US population has occurred
every

(a)2 years.

(b)1 year.

(c)5 years.

(d)10 years.

228.Which continent has the greatest number of inhabitants'?

(a)South America

(b)North America

(c)Africa

(d)Asia

229.The `Trail of Tears' is an example of

(a)internal voluntary migration.

(b)internal forced migration.

(c)asylum seeking.

(d)refugee migration.

230.In general, a geographer would least expect to find population clusters

(a)in areas with fertile soils.


(b)along navigable rivers.

(c)in mountainous interiors.

(d)around seaports.

231.The baby boom refers to people who were born between the years of

(a)1946 and 1964.

(b)1965 and 1990.

(c)1990 and the present.

(d)1928 and 1945.

232.In which of the following countries does one expect the fastest
population growth'?

(a)Denmark

(b)Germany

(c)New Zealand

(d)Guatemala

233.Businesses and marketers use geographic information systems to

(a)coordinate the shipment of goods around the world.

(b)forecast national population growth rates.

(c)calculate exchange rates of currencies around the world.


(c)Age

(d)match demographic data with specific locations.

234.Which of the following types of data is useful in geographic information


systems'?

(a)Size of household

(b)Income level

(d)All of the above

235.For which country would one expect the age-sex pyramid to look most
like an actual pyramid'?

(a)Australia

(b)Japan

(c)France

(d)Bolivia

236.In the 1950s, the outlook for the parents of baby boomers was generally
positive because they

(a)faced an expanding labour market.

(b)were generally able to attain a better standard of living than their


parents.

(c)got educational help from the government, if they were veterans.


(d)all of the above

237.Baby boomers

(a)have a lower standard of living than any previous generation.

(b)have less education than the previous generation.

(c)are more likely to get divorced than any previous generation.

(d)get married younger than any previous generation.

238.Which of the following cohorts is the most economically active and


productive in the USA'?

(a)Old-age cohort

(b)Middle cohort

(c)Youth cohort

(d)All three cohorts are about equally economically productive.

239.The youth cohort

(a)are between the ages of 16 and 30.

(b)have little effect on the dependency ratio.

(c)are not fully active in the labour force.

(d)refers to the net generation.

240.Generation X is (are)
(a)the result of high birth rates.

(b)the name given to the parents of baby boomers.

(c)the children of migrants to the USA.

(d)the members of the baby bust.

241.The crude death rate

(a)typically declines as overall levels of economic development go up.

(b)is typically higher in core countries than in periphery countries.

(c)is not considered a useful measure by most population geographers.

(d)is the sum total of infant mortality rate, youth mortality rate and the
elderly mortality rate.

242.When crude birth rates are higher than crude death rates, the difference
between them is known as the

(a)infant natality rate

(b)natural decrease.

(c)total fertility rate.

(d)natural increase.

243.Infant mortality rates are lowest in which of the following areas'?

(a)Western Europe

(b)Asia
(c)Africa

(d)South America

244.Which of the following groups of states has been affected the least by
the AIDS epidemic?

(a)Southeast Asia

(b)North America

(c)Central Africa

(d)The Middle East

245.In sub-Saharan Africa, AIDS primarily strikes

(a)heterosexuals.

(b)homosexuals.

(c)haemophiliacs.

(d)drug users.

246.Demographic transition theory holds that

(a)advanced industrial states will have high rates of population growth.

(b)there is a maximum population size for every place.

(c)periphery and semiperiphery states cannot become core states.

(d)high levels of economic production will lower birth rates.


247.In which phase(s) of the demographic transition model are birth rate and
death rate about equal?

(a)Preindustrial and industrial

(b)Preindustrial and postindustrial

(c)Transitional

(d)Preindustrial

248.Critics of demographic transition theory argue that

(a)the theory has a Marxist ideological bias.

(b)the theory does not work for European countries.

(c)birth rates and death rates are irrelevant in the 21st century.

(d)the theory is not very relevant for countries on the periphery.

249.Emigration

(a)is the same as gross migration.

(b)does not involve push factors.

(c)refers to a move from a place.

(d)is equal to the difference between voluntary migration and forced


migration.

250.International voluntary migration

(a)is most likely to be from the periphery to the core.


(b)is done typically for religious reasons.

(c)is insignificant now compared to prehistoric times.

(d)is usually coordinated by the United Nations.

251.The first wave of internal voluntary migration within the USA had two
parts, westwards expansion and

(a)the movement of native Americans to reservations.

(b)the movement of Canadians to Florida and Arizona.

(c)the shift of a large part of the population to the sunbelt.

(d)a rural-to-urban shift.

252.In the second wave of internal voluntary migration within the USA,
AfricanAmericans moved from the

(a)rural South to cities.

(b)rural South to New York City and Boston.

(c)eastern cities to rural areas in the West.

(d)frostbelt to the sunbelt.

253.The third wave of internal voluntary migration within the USA

(a)brought people back to the cities through national urban renewal


programs Kennedy and Johnson administrations.

(b)resulted in the passing of federal laws restricting internal migration.


(c)emerged from the expansionist settlement policy of the American
Revolution.

(d)saw many people move to the sunbelt.

254.The demographic centre of the USA

(a)has been shifting westwards for the last 200 years.

(b)is a sociological concept to help us understand the aging of a


population.

(c)is currently along the California and Nevada boundary.

(d)has been New York City since the late 1800s.

255.The best explanation for the third internal voluntary migration wave in
the USA is

(a)the high degree of racism in the USA.

(b)the environmental problems in large cities.

(c)the pursuit of greater economic opportunity.

(d)the romantic longing for a simple country life.

256.The term `Trail of Tears' refers to

(a)the "Long March" of the Chinese communists in the 1930s.

(b)the suffering of Indonesians forced to leave their home islands.


(c)the internment of Japanese Americans in US camps during World War
II.

(d)the forced migration of the Cherokee Nation in the 19th century.

257.Malthus argued that the factor limiting population size was

(a)environmental pollution.

(b)money.

(c)food.

(d)land.

258.Neo-Malthusians think that the greatest danger to the environment is

(a)population growth.

(b)politicians.

(c)computer technology.

(d)industrial agriculture.

259.Most demographers and policy makers think that the best way to control
population is to

(a)increase the power of religious institutions that preach abstinence and


celibacy.

(b)pass laws that limit families to one child.

(c)educate women and give them equal status with men.


(d)let HIV/AIDS, tropical diseases, plagues and epidemics run freely.

260.In general, has the highest crude birth rates.

(a)China and India

(b)Africa

(c)Europe

(d)North America

261.In general, has the highest crude death rates.

(a)North America

(b)Europe

(c)South America

(d)Africa

262.In general, the world region of subSaharan Africa is highest for all of
the following except

(a)rate of natural increase.

(b)infant mortality rates.

(c)life expectancy at birth.

(d)birth rates.

263.Which world region has the lowest rate of natural increase?


(a)Europe

(b)North America

(c)Asia

(d)Latin America

264.Which world region has the highest levels of economic development-


measured in dollars as GNI PPP-nearly double that of the second
highest'?

(a)Asia

(b)Oceania

(c)Europe

(d)North America

265.Since the start of the 21st century, has annually received more global
voluntary migrants than any other country.

(a)China

(b)the United States

(c)Germany

(d)Russia

266.Nearly all of the world's inhabitants live onof Earth's land.

(a)50 per cent


(b)25 per cent

(c)10 per cent

(d)90 per cent

267.Approximately of the world's population lives above the equator (in the
Northern hemisphere).

(a)25 per cent

(b)50 percent

(c)90 per cent

(d)75 percent

268.In a healthy population, the left and right sides of age-sex pyramids are
symmetrical. Of the following, which is most likely to create an
asymmetrical pyramid, i.e., to affect the shape of one side differently than
the other'?

(a)Birth control

(b)War

(c)Baby boom

(d)Baby bust

269.The largest population cohort in US demographic history is

(a)the baby bust.


(b)the baby boom.

(c)the undocumented immigrant cohort.

(d)generation Y.

270.Among these countries, which is the only one expected to increase in


population by 2050'?

(a)Greece

(b)Finland

(c)Switzerland

(d)United States

271.In a population with a TFR of 2, the

(a)doubling time = 0.

(b)CBR = CDR.

(c)life expectancy = 0.

(d)demographic transition is in full swing.

272.Which of the following cannot be true at the same time as all of the
others?

(a)Demographic transition is complete.

(b)TFR = 2

(c)Natural increase = 1
(d)CBR = CDR

273.The country in the USA with the greatest number of internally displaced
persons-around 2 million-is Columbia. has the most in the world,
approximately 6 million.

(a)Sudan

(b)Afghanistan

(c)Bosnia and Herzegovina

(d)Burma (Myanmar)

274.The international forced migration of Palestinians from their homeland


since the creation of the state of Israel in 1948 is a response to

(a)land dispossession.

(b)discrimination.

(c)ongoing violence.

(d)all of the above.

275.All of the following nations have suffered international forced migration


in the 20th century except

(a)Kurds.

(b)Palestinians.

(c)Jews.
(d)All of the above have recently been forced to migrate.

276.Whereas Marx believed the equitable distribution would solve perceived


imbalances between populations and food supply, Malthus argued for

(a)colonialism.

(b)emigration.

(c)technology.

(d)birth control.

277.US suburbanization is associated with all of the following except

(a)loss of prime agricultural land.

(b)high use of automobiles for commuting.

(c)efficient use of space.

(d)taking the tax base out of inner cities.

278.On maps like the Washington DC metro map, the region where most of
the subway lines intersect is most probably the

(a)CBD.

(b)suburbs.

(c)zone of transition.

(d)exurbs.
279.As accessibility and utility decline in a concentric zone model of the
city, land use generally changes from

(a)factories to high-volume retailing.

(b)warehousing to factories.

(c)residential to warehousing.

(d)high-volume retailing to residential.

280.The urban land use model is sometimes referred to as the trade-off


model because people make a trade-off between accessibility and

(a)living space.

(b)jobs.

(c)crime.

(d)public transportation.

281.It is clear that a critical assumption behind the concentric zone model of
a city is that it has an isotropic surface, meaning that the model city

(a)is located on a slope, like a mountain side.

(b)has high buildings in the centre which get lower with distance
outwards.

(c)does not have motorized transportation.

(d)is flat, uniform and barrier free.


282.Which of the following is NOT a specific advantage of congregation to
urban minority groups?

(a)It helps the minority groups merge with other minority groups.

(b)It helps preserve minority group membership and identity.

(c)It provides a power base and a defence in relation to the host society.

(d)It provides mutual support networks for members of the minority


group.

283.Though congregation can be voluntary, segregation is when


congregation is combined with

(a)the fiscal squeeze.

(b)counterurbanization.

(c)discrimination.

(d)immigration.

284.The characteristics by which urban minority groups are typically


defined and congregate typically include any of the following except

(a)language and religion.

(b)sexual orientation or lifestyle.

(c)nationality.

(d)any of the above.


285.In the concentric zone model of cities, the zone of transition typically
begins at the dynamic outer edge of a city's

(a)residential neighbourhoods.

(b)warehouse district.

(c)factory area.

(d)central business district.

286.The classic case study of urban ecology and the example of concentric
zones of dynamic neighbourhoods of different ethnicity and social status
was

(a)London.

(b)Los Angeles.

(c)Detroit.

(d)Chicago.

287.The rippling process of change throughout a city, whereby displaced


social or ethnic groups displace others in residential areas, is known as

(a)smart growth.

(b)invasion and succession.

(c)the cycle of poverty.

(d)congregation and segregation.


288.Until the last quarter of the 20th century, the direction of invasion and
succession was

(a)outwards from the suburban districts.

(b)along the various concentric zones or rings.

(c)outwards from the centre.

(d)inwards from the CBD.

289.The invasion of older, centrally located working-class neighbourhoods


by higher income professionals in search of character and convenience is
called

(a)the fiscal squeeze.

(b)congregation.

(c)gentrification.

(d)smart growth.

290.When increased demand for urban services combines with a shrinking


tax base (where businesses and affluent people move out to the suburbs
and beyond), central cities around the country struggle with

(a)urban sprawl.

(b)the fiscal squeeze.

(c)invasion and succession.

(d)how to fill ecological niches.


291.The fiscal squeeze has made it extremely difficult for central cities to
address a wide variety of urban problems. Which of the following is NOT
directly one of such problems faced by central cities?

(a)Urban sprawl

(b)Neighbourhood decay

(c)Poverty

(d)Infrastructure decay

292.By making loans unattainable to people in certain neighbourhoods, the


practice of compounds the difficulty in achieving inner city
improvements.

(a)microlending

(b)redlining

(c)gentrification

(d)fiscal squeezing

293.Redlining is, most specifically, a form of

(a)fiscal squeezing.

(b)ethnic succession.

(c)housing discrimination.

(d)gender poverty.
294.In the summer of 2001, the President of the USA signed tax cuts
totalling $1.35 trillion. According to a report issued by the American
Society of Civil Engineers in 2005, this was nearly enough to

(a)eliminate smog.

(b)create 20 new cities in the American heartland.

(c)bring the country's infrastructure up to acceptable standards.

(d)establish a rail network in the USA.

295.European cities are different from North American cities largely because
European cities

(a)do not suffer from problems of infrastructure decay and poverty.

(b)have long histories of numerous periods of urban development.

(c)did not grow according to competitive land markets.

(d)have almost no ethnic congregation and segregation.

296.Which of the following is NOT true'? Compared to US cities, European


cities

(a)are more socio economically stable.

(b)are more compact.

(c)have lower skylines.

(d)have lower population densities.


297.The Beaux Arts style of architecture was

(a)the unique style of construction used in New Orleans to withstand


flooding.

(b)the name given by European orientalists for the architecture of the


Islamic city.

(c)used in the urban redevelopment and planning of Paris in the 19th


century.

(d)what has made Quebec City a UNESCO World Heritage Site.

298.The Modern movement of the early 20th century was based on the idea
that cities should be designed and run like

(a)machines.

(b)democracies.

(c)ant colonies.

(d)biological organisms.

299.The most compelling feature of the International Style of urban design


and architecture was that it was quite

(a)artistic.

(b)environmental.

(c)inexpensive.

(d)culturally sensitive.
300.The tradition of urban design that has imposed more uniformity on the
big cities of the world than any other is

(a)the Beaux Arts style.

(b)the Global Village style.

(c)the International Style.

(d)the Modem movement.

301.The best example of where environmental responses and social and


cultural values are integrated into urban form and design is found in the

(a)Islamic city.

(b)North American city.

(c)International Style.

(d)Modern movement.

302.Of the following, which is NOT among the fundamentals of urban


layout and design that stem from the Holy Qur'an and that guide
development of traditional Islamic cities'?

(a)The outer appearance of things

(b)The inner essence of things

(c)Emphasis on personal privacy and virtue

(d)Communal well-being
303.Suqs, kasbahs, jamis and medinas are typically found in

(a)east European cities.

(b)Islamic cities.

(c)cities designed by Le Corbusier.

(d)postmodern cities.

304.The stark contrast of high-rise modern office buildings and luxury


housing with the slums and shantytowns of unintended metropolises is
visual evidence of their

(a)economic development.

(b)isotropism.

(c)dualism.

(d)gentrification.

305.The informal economies of peripheral cities

(a)rarely involved children.

(b)have significantly declined over the last couple of decades.

(c)often benefit consumers in the core.

(d)are insignificant compared to their formal economies.

306.Contributing to the unprecedented population growth that has made


Lagos, Nigeria a shock city are all of the following except
(a)the demographic transition.

(b)the discovery of oil.

(c)replacement of informal economic activities with high-paying, formal


jobs.

(d)political independence.

307.Shantytowns and squatter housing are growing in the unintended


metropolises of the periphery due to

(a)misleading advertising.

(b)environmental conditions that make permanent housing difficult to


build.

(c)poverty and the lack of affordable housing.

(d)unavailability of construction material.

308.Environmental degradation in peripheral cities is largely the result of

(a)the accumulation of human wastes.

(b)inadequate infrastructure to handle sewage.

(c)industrial and automobile air emissions.

(d)all of the above

309.The decentralized commercial and office districts that have emerged on


the fringes of metropolitan areas near such features as freeway
interchanges and airports have been called
(a)boomburbs.

(b)edge cities.

(c)polycentric metropolises.

(d)100-mile cities.

310.Proponents of urban growth and expansion support urban sprawl


because it

(a)discourages segregation.

(b)reduces reliance on the automobile.

(c)enhances open and public spaces.

(d)reflects freedom of markets and individual choice.

311.Smart growth attempts to address the problem of

(a)segregation.

(b)cycle of poverty.

(c)redlining.

(d)urban/suburban sprawl.

312.Globalization and the increasing financial gap between the rich and the
poor have resulted in the segregation of urban spaces into

(a)compartmentalized residential enclaves.

(b)Modern and post modern neighbourhoods.


(c)suburbs and edge cities.

(d)CBDs and edgeless cities

313.The central business district

(a)is usually the headquarters for national government.

(b)contains many factories but few shops or offices.

(c)typically has the tallest buildings.

(d)contains the zone in transition.

314.The zone in transition

(a)is where railway terminals and the intersection of transportation


networks are found.

(b)contains apartment buildings, public housing and old neighbourhoods.

(c)typically contains edge cities.

(d)is outside the area of gentrification.

315.Gentrification typically

(a)displaces original occupants.

(b)leads to the renovation of older buildings.

(c)raises property values.

(d)Gentrification is associated with all of the above.


316.Which of the following is a specific advantage of congregation for a
minority group'?

(a)Minimization of conflict with "outsiders"

(b)Easier means of cultural preservation

(c)Easy establishment of mutual support networks and institutions

(d)All of the above

317.Jewish districts in the cities of the eastern USA are best considered as

(a)colonies.

(b)ghettos.

(c)enclaves.

(d)edge cities.

318.The area of maximum accessibility in an idealized city located on an


isotropic surface is

(a)the CBD.

(b)an edge city.

(c)the zone in transition.

(d)the upper-scale suburbs.

319.According to our understanding of urban accessibility and land use,


which of the following is located farthest from the city centre?
(a)Residences

(b)High-volume retail stores

(c)Warehouses

(d)Factories

320.The cores of older European cities have complex street patterns because

(a)they reflected ancient patterns of ownership and boundary formation.

(b)the patterns were developed before the introduction of automobiles and


trucks.

(c)the streets were the product of slow growth.

(d)of all of the above.

321.Which of the following is generally NOT a characteristic of European


cities?

(a)Relatively low skylines

(b)Plazas and squares

(c)Scars of war

(d)Rapid neighbourhood change

322.Which of the following was a major influence on the shaping of


European cities'?

(a)The presence of defensive walls


(b)Urban ideas brought from Central America

(c)Roman influence on town layout

(d)Careful planning

323.Why do western European cities typically have low skylines compared


to American cities?

(a)Much urban growth occurred before technology made skyscrapers


possible.

(b)Soviet planners preferred thick and massive buildings over tall ones.

(c)The economy was not advanced enough to finance tall skyscrapers.

(d)Buildings were poorly constructed and did not last long.

324.In the larger cities of Britain, France and Germany, roughly per cent of
all housing is public housing.

(a)less than 1

(b)5-20

(c)1-5

(d)20-40

325. was one characteristic of socialist government policy in the cities of


eastern Europe.

(a)Industrial zones in city centres


(b)Large public housing projects in outlying districts

(c)Massive restructuring of city centres

(d)Destruction of all buildings closely related to capitalist activities

326.Which of the following is NOT a common and important element of


cities in the periphery'?

(a)Squatter zones or shantytowns

(b)A central zone with a concentration of commerce, industry and


retailing

(c)A zone devoted to tourism

(d)A zone of elite residential neighbourhoods

327.It is least likely you would find an Islamic city in this region.

(a)Pakistan

(b)Indonesia

(c)southern Africa

(d)the Middle East

328.The values that shape the layout and design of Islamic cities derive from

(a)the Qur'an.

(b)Greek culture.

(c)the various peoples conquered by Muslims in the 8th and 9th centuries.
(d)Islamic fundamentalists.

329.The dominant feature of a traditional Islamic city is

(a)the main marketplace, the suq.

(b)the principal mosque, the Jami.

(c)the Kasbah.

(d)the sultan's palace.

330.The traditional Islamic city typically contains all of the following except

(a)cul-de-sacs.

(b)multiple jamis.

(c)a monastery.

(d)suqs.

331.Deliberate urban design began as early as

(a)the early medieval period.

(b)the Renaissance.

(c)the Baroque period.

(d)the height of the Roman Empire.

332.The Beaux Arts style of urban design

(a)flourished in the 18th century.


(b)had its greatest influence in America.

(c)synthesized earlier styles to fit the Industrial Age.

(d)emerged in London from the ideas of Baron Haussmann.

333.The Modern movement

(a)opposed high-rise buildings.

(b)stressed an unadorned, functional type of architecture.

(c)was inspired by the ideals of 19thcentury Romanticism.

(d)paved the way for the Beaux Arts style.

334.The International Style drew primarily from the tradition of urban


design.

(a)Greek

(b)postmodern

(c)modern

(d)Beaux Arts

335.Urban change in core regions has been influenced the most by

(a)higher levels of education.

(b)postmodern architectural styles.

(c)increased racial tensions.


(d)the economic transformation to a postindustrial economy.

336.The inability of central city areas to generate sufficient tax revenue is


due to all of the following except

(a)decentralization of cities.

(b)inadequate collection mechanisms.

(c)increased demands for infrastructure repairs.

(d)increased demands for social services.

337.Maintenance and repair on housing in poor, inner-city neighbourhoods


is inadequate because

(a)government officials choose not to do it or are unable to do it.

(b)renters cannot afford to do it.

(c)landlords have no incentive to do it.

(d)of all of the above.

338.The cycle of poverty

(a)is of little consequence in core countries.

(b)is found only in the periphery.

(c)transmits poverty from one generation to the next.

(d)results in the middle class dropping down into poverty.

339.is a characteristic of the cycle of poverty.


(a)Unemployment

(b)Psychological stress from crowding

(c)Improper nutrition

(d)All of the above are found in the cycle of poverty.

340.Redlining

(a)is legal.

(b)promotes geographic literacy.

(c)gives government-subsidized loans to low-income families.

(d)targets the underclass.

341.In most peripheral cities, it is true that of the people who would like
full-time work,

(a)a majority have it.

(b)nearly all have it.

(c)the majority is either unemployed or underemployed.

(d)none really want it.

342.Slum removal and demolition

(a)usually results in the quick construction of another slum in another part


of the city.

(b)was banned by the South Korean government in the early 1980s.


(c)could affect up to 1 billion people worldwide if it were applied to all
slums.

(d)is most commonly tried in core countries.

343.In some instances, squatter settlements in peripheral cities can be


considered `slums of hope' because

(a)settlements provide a supportive environment for newly arrived


migrants.

(b)settlements provide affordable shelter for the poor.

(c)governments sometimes provide help to squatters.

(d)all of the above

344.According to the World Bank, approximately what percentage of urban


residents in peripheral cities has access to a satisfactory water source'?

(a)15 percent

(b) per cent

(c)95 per cent

(d)65 per cent

345.The typical African-American neighbourhood in a US city can best be


termed a(n)

(a)ghetto.

(b)colony.
(c)enclave.

(d)congregation.

346.Which of the following is NOT a problem currently being faced by the


North American cities?

(a)Municipal socialism

(b)Poverty

(c)Mortgage foreclosures

(d)Infrastructural decay

347.Which of the following is the newest problem to be faced by the North


American cities'?

(a)Infrastructural decay

(b)Mortgage foreclosures

(c)Municipal socialism

(d)Fiscal squeeze

348.Chateau Regalia, Sydney Coast, Thames Town, Napa Valley and


Yosemite are neighbourhoods in the sprawling suburbs of biggest cities.

(a)U.S.A.'s

(b)Great Britain's

(c)China's
(d)Australia's

349.Smart growth that attempts to manage urban growth and minimize


urban sprawl is most likely to target growth at or around

(a)existing public mass transit stations.

(b)industrial areas.

(c)low population density areas.

(d)the CBD.

350.The megalopolis that extends from Washington, D.C. to Boston is best


exemplified by which of the following models of urban structure'?

(a)Smart growth

(b)Concentric ring

(c)Invasion and succession

(d)Polycentric new metropolis

351.The urban renewal schemes of Frenchman Baron Georges Haussmann


were designed to enhance

(a)efficiency and order.

(b)revolutionary ideas and politics.

(c)history and culture.

(d)traditional neighbourhoods and activities.


352.Edge cities, edgeless cities, micropolitan centres, polycentrism and
metrourban landscapes characterize

(a)modernist urban planning.

(b)megapolitan regions.

(c)the reaction to gentrification.

(d)virtual cities.

353.The quartering of cities into spatially partitioned, compartmentalized


residential enclaves of rich, poor, ethnic, working class is the result of

(a)satellite sprawl.

(b)invasion and succession.

(c)modernism.

(d)globalization.

354.The segregation and congregation that is creating increasingly distinct


urban security zones and protected enclaves of the rich, middle-class
suburbs, working class neighbourhoods, ethnic enclaves and excluded
ghettos characterizes the that is taking place in globalizing cities.

(a)integrating

(b)planning

(c)quartering

(d)clustering
355.Dubai, of the United Arab Emirates, embodies surprising and disturbing
changes in economic, social and cultural lives. Thus, it is a good example
of a

(a)shock city.

(b)polycentric city.

(c)world city.

(d)post-industrial city.

356.Primate cities are found in the world's

(a)core countries.

(b)peripheral countries.

(c)both core and peripheral countries.

(d)South.

357.The urban system of which country is NOT centred around a primate


city?

(a)England

(b)United States

(c)France

(d)Argentina
358.According to statistics compiled by the United Nations, nearly of the
world's population today lives in cities.

(a)50 per cent

(b)75 per cent

(c)25 per cent

(d)100 per cent

359.North America is about 80 per cent urbanized. This mean that 80 per
cent of North American

(a)places are cities.

(b)territory is covered with urban structures like streets and commercial


buildings.

(c)cities contain 1 million or more people.

(d)people live in cities.

360.The very first region of independent urbanism was in

(a)the Andes.

(b)the Middle East.

(c)the Pearl River Delta.

(d)the Baltics.
361.The urbanized economies of a number of a number of early empires
(Mesopotamian, Mayan, Roman) collapsed because of

(a)imperialism.

(b)labour shortages.

(c)mismanagement.

(d)technological innovations.

362.The specific roles of prominent urban settlements of medieval Europe


include all of the following except the role of

(a)centre of education.

(b)agricultural centre.

(c)administrative centre.

(d)defensive stronghold.

363.In contrast to 16th century Spanish colonizers that established military


and administrative centres in the New World, Portuguese colonizers
situated cities to

(a)blend in as well as possible with the local environment and lifestyles.

(b)replicate European lifestyles.

(c)facilitate trade and commerce.

(d)convert locals to Christianity.


364.The European cities growing the fastest by the year 1700 were

(a)Scandinavian cities because of their humane social programs.

(b)Atlantic coast cities through which resources from the colonies came.

(c)southern European cities which retained the Roman trade networks.

(d)Mediterranean cities through which Silk Road products flowed.

365.Which of the following is NOT true about gateway cities?

(a)They tend to be port cities.

(b)They funnelled mineral and agricultural resources from continental


interiors to Europe.

(c)They are control centres providing access into and out of particular
countries or regions.

(d)They have become the urban centres of the world's core.

366.Geographers are interested in urbanism as a

(a)way of life that emerges in urban settings.

(b)form of counterurbanization.

(c)taxonomy by which cities in urban systems are categorized.

(d)philosophy drawing people to urban settlements.

367.Manchester, England, and Chicago, USA, are both good examples of

(a)informal cities.
(b)megacities.

(c)colonial cities.

(d)shock cities.

368.By the late 18th century and the Industrial Revolution, cities of Europe
began to grow with rural to urban migration and

(a)migration from Asia.

(b)falling death rates.

(c)rising birth rates.

(d)importation of slaves.

369.Chicago's explosive population growth in the late 19th century was due
to industrialization and its excellent

(a)natural resources.

(b)political leadership.

(c)educational system.

(d)geographic location.

370.Urbanization in the world's core was stimulated by increased


agricultural productivity that

(a)provided food for the growing urban populations.


(b)benefitted from improved tools and machinery manufactured in
industrial areas.

(c)provided surplus rural labour for urban industrialization.

(d)contributed all of the above.

371.European planning and construction of cities in the colonies was based


on

(a)building up local political and economic capacities.

(b)European concepts and practices.

(c)local cultures and preferences.

(d)local environments and climate.

372.The interdependent relationship between cities and settlements of


different sizes and functions within a given region is known as (an)

(a)urban ecology.

(b)urban form.

(c)urban system.

(d)urbanization.

373.Central place theory explains the relative size and spacing of urban
centres based on

(a)cultural differences.
(b)urban ecology.

(c)government planning.

(d)markets and consumer behaviour.

374.In the context of urban systems, basic tenets of central place theory
include all of the following except

(a)the larger the settlement, the greater the variety of specialized goods
and services.

(b)people will travel farther for more expensive, less frequently


purchased goods.

(c)settlements of similar population sizes are grouped together.

(d)the smallest settlements provide goods and services that meet everyday
needs.

375.The Spanish urban system is dominated by

(a)two cities of national scope (Madrid and Barcelona).

(b)cities built around tourism and leisure activities.

(c)the European Union.

(d)coastal cities (Bilbao, Valencia, Malaga).

376.The two largest cities in the USA are New York City and Los Angeles.
The 2010 Census provides evidence to suggest that cities in the USA tend
to follow the ranksize rule. Thus, the actual populations for New York
City and Los Angeles can be
(a)8 and 1 million.

(b)8 and 3.7 million.

(c)8 and 7.2 million.

(d)8 and 12.3 million.

377.The functional dominance (cultural, economic, political) of a city within


an urban system is known as

(a)urban imperialism.

(b)overurbanization.

(c)centrality.

(d)reurbanization.

378.Today's world cities are distinguished by being centres of all of the


following except

(a)industrial production for global mass consumption.

(b)world's most powerful media organizations.

(c)international NGO and IGO headquarters.

(d)specialized and advanced business services.

379.The definition of urban used by the United Nations

(a)is 50,000 people.

(b)is 1,000 people.


(c)is 2,000 people.

(d)varies by country.

380.According to our text, urban growth in peripheral countries is a


consequence of migration to cities, stimulated, largely, by the onset of

(a)pollution and disease.

(b)structural adjustment programs.

(c)war.

(d)the demographic transition.

381.The doubling time of some of the world periphery's largest, fastest


growing cities is

(a)10-20 years.

(b)50-70 years.

(c)100-150 years.

(d)6-12 months.

382.Gecekondus (Turkey), favelas (Brazil), gourbevilles (North Africa) and


bustees (India) all refer to urban

(a)open spaces.

(b)transportation networks.

(c)high-rent neighbourhoods.
(d)squatter settlements.

383.is when cities grow faster than their ability to provide jobs, housing and
other basic needs.

(a)Deindustrialization

(b)Megacities

(c)Overurbanization

(d)Decentralization

384.Squatter settlements refer to residential housing

(a)in massive, multi-storey, government-supported apartment blocks.

(b)on land neither owned nor rented by its occupants.

(c)that emerges with reurbanization.

(d)serving as transitional housing for recent immigrants to the city.

385.Megacities are determined by all of the following except their

(a)status as world city.

(b)size.

(c)primacy.

(d)centrality.

386.In the 1970s and 1980s, the USA entered a period of, during which time
there was a net loss of population from big cities to smaller towns and
rural areas.

(a)overurbanization

(b)urbanism

(c)reurbanization

(d)counterurbanization

387.By 2025, 7 of the 10 biggest cities in the world will be in

(a)Africa.

(b)Europe.

(c)North America.

(d)Asia.

388.Given that urban populations are growing at twice the general


population growth rate, it is likely that

(a)urban areas have adequate housing.

(b)urban areas have low unemployment.

(c)people live better lives in urban areas than in rural areas.

(d)there is a net movement of people from rural areas to urban areas.

389.Many of the world's megacities

(a)are gateway cities.


(b)are more populated than about half of the countries in the United
Nations.

(c)hold half of the world's population.

(d)are declining in population.

390.Urban settlements

(a)facilitate decision making by public policymakers and private


institutions.

(b)generate innovation and the development of new knowledge.

(c)increase the range of lifestyle choices of inhabitants.

(d)do all of the above.

391.Urban geographers are quite interested in

(a)relationships between urban areas and surrounding territory.

(b)how the identities of urban areas evolve.

(c)land use within urban areas.

(d)all of the above.

392.(An) is an interdependent group of urban settlements in a given region.

(a)urban ecology

(b)urban form

(c)urban system
(d)urbanism

393.The physical structure and organization of cities is termed

(a)urban ecology.

(b)urban system.

(c)urban form.

(d)urbanism.

394.The feudal system in early medieval Europe was characterized by

(a)strong central governments that ruled large areas.

(b)estates that were relatively selfsufficient.

(c)highly developed and widely distributed urban areas.

(d)a general lack of warfare.

395.By 1400, long-distance trading in Europe

(a)centred on port cities.

(b)dealt primarily with energy resources.

(c)was dominated by Paris.

(d)was minimal compared to trade with China and Spanish colonies.

396.Which of the following changes occurred between the 15th and 17th
centuries that eventually led to the growth of urban areas in Europe and
its colonies'?
(a)An increase in merchant capitalism

(b)The Protestant Reformation

(c)The Scientific Revolution

(d)All of the above

397.Spanish colonial towns were established primarily to

(a)serve as administrative and military centres.

(b)spread Catholicism to the natives.

(c)provide education to the natives.

(d)exploit nearby natural resources.

398.During the Renaissance in Europe, these urban areas grew the fastest.

(a)Cities in Russia

(b)Cities in the interior of France and the German states

(c)Port cities

(d)Towns where French was the dominant language

399.Most gateway cities were

(a)in existence for at least a millennium.

(b)ports.

(c)controlled by native elites.


(d)located primarily in northern Germany and Scandinavia.

400.The growth of cities in the 19th century was most closely linked with
the

(a)increased importance of industrialization.

(b)decreased output of agriculture in rural areas.

(c)increased amount of social services available in cities.

(d)increased power of states.

401.Which of these factors was a significant cause of 19th-century


urbanization in Europe'?

(a)Higher wages in urban areas

(b)Drops in death rates

(c)Increase in labour supply

(d)All of the above

402.This city was the shock city of European industrialization in the 19th
century.

(a)Venice, Italy

(b)London, England

(c)Manchester, England

(d)Berlin, Germany
403.The most important reason for Chicago's emergence and large size is

(a)military.

(b)industrial.

(c)political.

(d)religious.

404.Which region has the largest percentage of its population living in urban
regions?

(a)Europe

(b)Africa

(c)Asia

(d)North America

405.Rapid urbanization is greatest in which of the following areas?

(a)China

(b)Western Europe

(c)South America

(d)Africa

406.For decades since the middle of the 20th century, Chinese stayed in rural
areas primarily because

(a)they needed to stay there to take care of their elders.


(b)the government forced them to stay.

(c)they were obligated to stay under a centuries-old feudal system.

(d)of religious reasons.

407.Reasons for urban growth in peripheral countries include

(a)desertification in rural areas.

(b)deforestation in rural areas.

(c)population shifts caused by war.

(d)all of the above.

408.Central place theory holds that the size and spacing of urban areas is a
function of

(a)historical development of religious centres.

(b)the specific types of economic activity in the overall area studied.

(c)where people go to purchase goods and services.

(d)government planning.

409.People would most likely travel the farthest to

(a)get the car's engine oil changed.

(b)see a professional sports event.

(c)get a haircut.
(d)buy bread and milk.

410.Which of the following goods and services is most likely to be found in


the largest settlement'?

(a)Beauty salon

(b)Restaurant

(c)Hospital

(d)Drug store

411.Which of the following American cities is NOT a world city'?

(a)Seattle

(b)Los Angeles

(c)New York City

(d)Chicago

412.Following the rank-size rule, if the population of the largest city in a


country or region is 1 million, the fourth-largest city should contain about
how many people'?

(a)5,00,000

(b)4,00,000

(c)2,50,000

(d)1,00,000
413.Primacy

(a)occurs only in core countries.

(b)occurs only in peripheral countries.

(c)occurs in both core and peripheral countries.

(d)occurs only in semiperipheral countries.

414.World cities

(a)did not exist until the 20th century.

(b)typically do not change in influence over time.

(c)were initially intimately involved with imperialism and colonialism.

(d)are none of the above.

415.The most important functions of world cities have to do with

(a)international politics and diplomacy.

(b)economic functions.

(c)the influential media they host.

(d)dissemination of global culture.

416.Which of the following is NOT a top-tier world city'?

(a)New York

(b)London
(c)Tokyo

(d)Los Angeles

417.Increased agricultural productivity in the 19th century

(a)provided the food to feed the swelling urban population.

(b)was fuelled in part by tools and machinery provided by urban


factories.

(c)freed up rural labour for urban factories.

(d)all of the above

418.Counterurbanization

(a)has been successful at shrinking the size of the world's megacities.

(b)results in a much slower increase in population growth for cities.

(c)is due in part to agglomeration economies.

(d)is due, in part, to the increased accessibility of smaller towns and rural
areas.

419.Urban growth in peripheral countries

(a)usually is far in advance of industrial development in urban areas.

(b)can lead to overurbanization.

(c)often occurs at a rapid rate.

(d)involves all of the above.


420.By ad 1000, a city-based world empire had NOT yet occurred in

(a)North America.

(b)the Middle East.

(c)China.

(d)Europe.

421.Of the following regions, which is the least urbanized?

(a)Latin America

(b)North America

(c)Europe

(d)Africa

422.In 2025, it is projected that the largest city will be in

(a)Africa.

(b)Asia.

(c)North America.

(d)Europe.

423.The Pearl River Delta is an extended metropolitan region of nearly 50


million people, encouraged by the Chinese government as an engine of
economic growth and built around the already established economic
success of
(a)Hong Kong.

(b)Tokyo.

(c)Beijing.

(d)Shanghai.

424.The economic activities not a part of the official record and that take
place outside official systems of regulation or remuneration make up the
sector of the economy and tend to be an important part of urban economic
life around the world.

(a)underground

(b)market

(c)informal

(d)black market

425.In the first decade of the 21st century, cities in which region experienced
decreased global connectivity'?

(a)South Asia

(b)Eastern Europe

(c)The United States

(d)China

426.Callampas, favelas and villas miserias in South America are the


consequences of
(a)counterurbanization.

(b)overurbanization.

(c)decentralization.

(d)reurbanization.

427.Which of the following is NOT among the reasons why world cities are
typically targets of terrorism?

(a)The political and social reverberations are greatest and have the
potential to spread the most fear.

(b)The impact value - targeting urban infrastructure has the potential to be


much more destructive.

(c)The symbolic value - prestige is associated with penetrating sites of


power.

(d)The perpetrators and their groups are not from urban areas.

428.Cities like Rio de Janeiro, Accra, Buenos Aires, Kolkata and other
colonial cities emerged and grew due to

(a)war.

(b)trade.

(c)industrialization.

(d)the rank-size rule.


429.gave us a new kind of city, and many of them. These cities were based
on assembling and processing raw materials and producing and
distributing manufactured goods.

(a)Central Place Theory

(b)The Industrial Revolution

(c)Counterurbanization

(d)The Green Revolution

430.Population geography is

(a)the number of pets per person in the USA.

(b)the distribution of mankind across our planet.

(c)the tracking of deer within an ecosystem.

(d)the analysis of the migratory patterns of the monarch butterfly.

431.The study of individual populations in terms of specific group


characteristics is

(a)ecology.

(b)geography.

(c)demography.

(d)ekistics.
432.A world map drawn so that the size of each country is the reflection of
its population rather than its size is called

(a)a Mercator projection.

(b)a conic projection.

(c)a standard land area map.

(d)a cartogram.

433.The agricultural crop that produces the highest number of calories per
acre is

(a)potato.

(b)corn.

(c)rice.

(d)wheat.

434.Population densities tend to be higher

(a)in cold climates.

(b)in areas of fertile soil.

(c)near the sea.

(d)on steep terrain.

435.The difference between population geography and demography is that


population geography is
(a)the study of the distribution of mankind across Earth.

(b)the study of individual populations in terms of specific group


characteristics.

(c)the study of internal population dynamics.

(d)totally concerned with human migration.

436.Which continent has more than one area with major population
concentration'?

(a)North America

(b)Europe

(c)Asia

(d)Africa

437.The West African country of Togo and the European country of France
have approximately the same arithmetic density. From this, we can
conclude that the two countries have the same

(a)standard of living.

(b)number of people per area of land.

(c)level of output per farmer.

(d)pressure placed by people on the land to produce food.

438.`Arithmetic Density' is determined by


(a)the number of people per unit of farmland.

(b)the number of people per unit habitable land.

(c)the number of people per unit area.

(d)the number of people in urban areas divided by the number of people


in rural areas.

439.`Physiological density' is a term applied to human populations that


means

(a)the number of people per unit of area.

(b)a region is overpopulated.

(c)the strength of the local football team.

(d)the density of population per unit of arable land.

440.Most of the world's population is concentrated in areas that are


designated climates.

(a)A

(b)B

(c)C

(d)D

441.Human beings cluster in which of these regions'?

(a)Cold lands
(b)Hilly lands

(c)Flat lands

(d)Dry lands

442.The increase in world population is now

(a)slowing down.

(b)remaining about the same.

(c)accelerating.

(c)cannot tell.

443.The region that is projected to have the highest population increase for
2014 is

(a)Europe.

(b)Sub-Saharan Africa.

(c)Latin America.

(c)North America.

444.The most popular form of birth control in poor countries is

(a)birth control pills.

(b)abortion.

(c)intrauterine devices.
(d)sterilization.

445.Throughout the world, fertility rates are in urban areas than they are in
rural areas.

(a)insignificantly different

(b)higher

(c)about equal

(d)lower

446.Of all communicable diseases, the one that causes the most deaths
worldwide currently is

(a)tuberculosis.

(b)respiratory infection(s).

(c)malaria.

(d)cholera.

447.Rates of obesity in poor countries have been increasing because

(a)of poor quality food.

(b)cheap cooking oil has become more widely available, adding fat to the
diet.

(c)people have stopped working.

(d)sports are unpopular.


448.Country X has a crude birth rate of 42 and a crude death rate of 17,
while Country Y has a crude birth rate of 20 and a crude death rate of 5.
Which country has a lower natural increase rate'?

(a)Country Y

(b)Country X

(c)The rate is the same in both countries.

(d)The rate cannot be computed.

449.Of the following four countries, the lowest natural increase rate is found
in

(a)Uganda.

(b)Mexico.

(c)Indonesia.

(d)Denmark.

450.The region with the highest growth rate is

(a)Asia.

(b)Europe.

(c)North America.

(d)sub-Saharan Africa.
451.In the first stage of the demographic transition, crude birth rate is and
crude death rate is

(a)low; high

(b)low; low

(c)high; low

(d)high; high

452.Thomas Malthus

(a)is known as a cornucopian.

(b)predicted a grim view of the future due to overpopulation.

(c)advocated a high birth rate to offset the high death rate of his day.

(d)argued for a high birth rate in order to have a large work force.

453.Under natural conditions, the sex ratio at birth is boys born for every
100 girls.

(a)110

(b)120

(c)105

(d)107

454.The estimated sex ratio at birth for 2014 is boys born for every 100
girls.
(a)107

(b)105

(c)120

(d)110

455.The problems associated with rapidly aging populations today are

(a)more serious than the massive young population of the developing


world.

(b)of no consequence to the future of social security programs being paid


by the young.

(c)equal throughout the world.

(d)primarily restricted to the rich countries of the world.

456.Aging populations will present tremendous challenges for poor


countries because

(a)retirees from rich countries may migrate to poor countries.

(b)there are fewer elderly in poor countries.

(c)of the increase in young people in the population.

(d)today the elderly are cared for by their families.

457.The reporting of higher number of males to females in some countries


may be because
(a)females engage in riskier professions, increasing the risk of death at
earlier ages.

(b)some countries do not count all of their women in the census.

(c)males comprise a larger portion of the workforce.

(d)males have a longer life expectancy.

458.In rich countries, such as the USA, higher percentages of national


budgets are spent on senior citizen centres than on

(a)military.

(b)elementary schools.

(c)social security.

(d)health care.

459.Human movements are a result of

(a)forced relocation.

(b)the environment.

(c)natural disasters.

(d)push and pull factors.

460.The largest migration field is from

(a)Mexico to the United States.

(b)Bangladesh to India.
(c)Russia to the Ukraine.

(d)Germany to Poland.

461.Brain drain occurs when

(a)agricultural experts leave cities for the countryside.

(b)schools are closed by civil unrest.

(c)universities become unaffordably expensive.

(d)highly educated citizens leave a country to move to another country.

462.Replacement migration is used to

(a)replace younger people with older people.

(b)admit people who require fewer medical treatments.

(c)replace uneducated people with those with college degrees.

(d)admit newcomers with useful skills.

463.Barriers to migration include all EXCEPT

(a)economic barriers.

(b)motivational barriers.

(c)physical barriers.

(d)cultural barriers.
464.Which of the following types of migration does not take place within a
country's borders?

(a)Internal

(b)Interregional

(c)Intraregional

(d)International

465.International labour migration tends to originate in

(a)rich countries with weak economies.

(b)rich countries with strong economies.

(c)poor countries with weak economies.

(d)poor countries with strong economies.

466.Amenity migration is driven by two types of migrants, namely

(a)young families and job hunters.

(b)illegal and undocumented workers.

(c)tourists and business owners.

(d)the wealthy and the retired.

467.One of the laws of migration is

(a)involuntary migrations are easier than voluntary migrations.


(b)most of the migrations are to distant locations rather than local areas.

(c)the more distant the destination, the fewer the benefits and the higher
the cost.

(d)sometimes you do not have a choice.

468.Even though many Europeans emigrated during its demographic


transition, many European countries now

(a)encourage new immigration, especially from their former colonies.

(b)are open-minded and welcome Muslims.

(c)have developed easier methods for existing immigrants to achieve


citizenship.

(d)discourage immigration, even from their former colonies.

469.Almost all Asian countries today have net emigration, that is they

(a)are restrictive about the type of people who can enter their countries.

(b)have more people immigrate to their country than those who leave.

(c)send more people out of their country than they receive from other
countries.

(d)are restrictive about the type of people they allow to emigrate out of
their countries.

470.Which of the following is a push factor for international migration?

(a)Internal conflicts
(b)Greater freedom

(c)Higher wages abroad

(d)Greater security

[UGC D-2005]

471.What type of central tendency measure would you use to depict the
growth of population in a country?

(a)Arithmetic mean

(b)Harmonic mean

(c)Geometric mean

(d)Median

[UGC D-2005]

472.Migration of people from early periods has had a beneficial impact. The
importance is

(a)diffusion of ideas.

(b)spread of language.

(c)spread of scientific and technological innovations.

(d)All the above.

[UGC D-2006]

473.In rural areas, small towns commonly grow up for providing


(a)market facilities.

(b)educational facilities.

(c)health facilities.

(d)administrative facilities.

[UGC D-2006]

474.The most important criterion for classification of Indian urban centres is

(a)site.

(b)shape.

(c)population.

(d)function.

[UGC D-2006]

475.In the rank-size rule, the population of lower rank city is proportionate
to

(a)population of smallest city.

(b)population of largest city.

(c)population of middle order city.

(d)None of the above.

[UGC D-2006]
476.An increase in urban population takes place at a rate much faster than
rural populations because

(a)urban population has a higher birth rate.

(b)rural population has a higher death rate.

(c)a large number of people migrate to urban centres.

(d)urban centres offer better scope for employment.

[UGC D-2007]

477.The most important criterion for the classification of towns is

(a)population.

(b)site.

(c)shape.

(d)functions.

[UGC D-2007]

478.A high and gradually declining fertility of over 30 per thousand and a
sharply reduced mortality rate of over 15 per thousand is characteristic of
demographic transition of

(a)first stage.

(b)second stage.

(c)third stage.
(d)last stage.

[UGC D-2007]

479.When the population density of an area is obtained through division of


the total population by the total agricultural area, it is termed

(a)arithmetic density.

(b)agricultural density.

(c)economic density.

(d)physiological density.

[UGC D-2007]

480.Given below are two statements, one is labelled as Assertion (A) and the
other labelled as Reason (R). Select your answer from the codes given
below:

Assertion (A): The levels of urbanization in the advanced countries


follow an elongated `S' shaped curve.

Reason (R): The levels of urbanization in India follow almost a straight


line.

(a)Both (A) and (R) are correct and (R) explains (A).

(b)Both (A) and (R) are correct but (R) does not explain (A).

(c)(A) is correct but (R) is false.

(d)(A) is false but (R) is correct.


[UGC D-2007]

481.Which of the following is the most suitable term to define a


metropolitan region'?

(a)Formal

(b)Functional

(c)Economic

(d)Generic

[UGC D-2007]

482.If the population of the largest city of a region is 8 lakhs, what will be
the population of fourth ranked city according to Ziff's rank-size rule?

(a)6 lakhs

(b)4 lakhs

(c)2 lakhs

(d)0.5 lakhs

[UGC D-2008]

483.Pattern of population distribution is determined by

(a)availability of minerals.

(b)agricultural practices.

(c)cultural practices.
(d)physiographic characteristics.

[UGC D-2008]

484.Match List I (tribe) with List II (state) and select the correct answer
from the codes given.

Codes:

[UGC D-2008]

485.Given below are two statements, one labelled as Assertion (A) and the
other labelled as Reason (R). Select your answer from the codes given.

Assertion (A): The spatial analysis of regions advocated by Haggett


considers them as closed systems.

Reason (R): The concepts that spatial analysis uses are: focality,
localization, interconnection and discontinuity.

(a)Both (A) and (R) are correct and (R) explains (A).
(b)Both (A) and (R) are correct but (R) is not the correct explanation of
(A).

(c)(A) is correct but (R) is false.

(d)(A) is false but (R) is correct.

[UGC D-2009]

486.Theories of spatial organization draw mainly from

[UGC D-2009]

(a)positivism.

(b)structuralism.

(c)functionalism.

(d)behaviouralism.

487.Natural population growth is a function of

(a)births.

(b)deaths.

(c)fertility and mortality.

(d)migration.

[UGC D-2009]

488.Migration in India, as per 2001 Census, is maximum in which of the


following streams'?
(a)Rural to urban

(b)Urban to rural

(c)Urban to urban

(d)Rural to rural

[UGC D-2009]

489.The sequence of events in the Demographic Transition Theory is

1.high birth rate and high death rate

2.high birth rate and low death rate

3.low birth rate and low death rate

4.low birth rate and high death rate

Codes:

[UGC D-2009]

490.The spatial distribution pattern of rural settlements can best be observed


from

(a)wall maps.

(b)cadastral maps.
(c)geological maps.

(d)topographical maps.

[UGC D-2009]

491.Match List I (principle) with List II (contributor) and select the correct
answer from the codes given.

Codes:

[UGC D-200]

492.In rural areas, small towns develop mainly for providing

(a)market facilities.
(b)educational facilities.

(c)health facilities.

(d)administrative facilities.

[UGC J-2005]

493.Given below are two statements, one labelled as Assertion (A) and the
other labelled as Reason (R). Select your answer from the codes given.

Assertion (A): Urban fringe is the surrounding area of large cities which
have strong socioeconomic link with the city but are not a part of the city.

Reason (R): Rapid growth of cities leads to urban fringe development.

(a)Both (A) and (R) are correct and (R) explains (A)

(b)Both (A) and (R) are correct and (R) does not explain (A)

(c)(A) is correct but (R) is wrong

(d)(A) is wrong but (R) is correct.

[UGC J-2005]

494.In villages, the type of house in which a man lives depends on

(a)cost of construction.

(b)climate.

(c)locally available materials.

(d)technology available.
[UGC J-2005]

495.Rural to urban migration is the characteristic feature of

(a)developed countries.

(b)developing countries.

(c)socialist countries.

(d)capitalist countries.

[UGC J-2005]

496.An increase in population at the rate of 2.1 per cent per annum will
account for its getting doubled in

(a)37 years.

(b)29 years.

(c)33 years.

(d)28 years.

[UGC J-2005]

497.Which of the following indicators is NOT relevant in the context of


social well-being?

(a)Provision of housing

(b)Improved accessibility to health facilities

(c)Provision of educational facilities


(d)Entry of multinational companies in developing countries

[UGC J-2007]

498.The theory of `demographic transition' was developed by

(a)Warren Thompson.

(b)C. P.Blacker.

(c)Frank Notestein.

(d)A.J. Coale and E.M.Hoover.

[UGC J-2007]

499.Which of the following is the correct sequence in terms of density of


population'?

(a)Africa, Asia, USA, Europe

(b)USA, Asia, Africa, Europe

(c)Europe, Asia, USA, Africa

(d)Asia, Europe, Africa, USA

[UGC J-2007]

500.The following are the four land use zones assumed to exist around the
central town in an isolated state according to the Von Thunen's model

1.firewood and lumber

2.intensive crop farming (grazing and root crops)


3.crop farming (with fallow and pasture)

4.market gardening and fresh milk

Select the correct sequence of the land use zones from centre to outwards
using the codes given.

(a)(1), (2), (3) and (4)

(b)(4), (2), (3) and (1)

(c)(4), (1), (2) and (3)

(d)(3), (2), (4) and (1)

[UGC J-2008]

501.Settlement pattern in a region is affected in India by

(a)social infrastructure.

(b)physical linkages.

(c)population growth rate.

(d)agricultural practices.

[UGC J-2008]

502.A city region is marked with

(a)underdevelopment.

(b)backwardness.
(c)availability of mineral resources.

(d)concentrated development.

[UGC J-2008]

503.Decadal population growth rate in India during 1991-2001 was

(a)20.11 per cent.

(b)26.20 per cent.

(c)23.60 per cent.

(d)21.34 per cent.

[UGC J-2008]

504.The centrality of a place is determined on the basis of its

(a)modes of transport.

(b)administrative status.

(c)functional strength.

(d)number of educational institutions.

[UGC J-2008]

505.Population migration streams are determined by

(a)political decision.

(b)push and pull factor.


(c)historical events.

(d)religious factor.

[UGC J-2008]

506.Hoyt propounded the

(a)sector model of city structure.

(b)rank-size rule.

(c)concept of primate city.

(d)multi-nuclei model of city structure.

[UGC J-2008]

507.Demographic transition model is based on data from the demographic


experience of

(a)northwestern Europe.

(b)USA.

(c)southeast Asia.

(d)South America.

[UGC J-2008]

508.Given below are two statements, one labelled as Assertion (A) and the
other labelled as Reason (R), select your answer from the codes given.
Assertion (A): The new industrial towns are numerically dominated by
the male adult population.

Reason (R): Migration is age - and sexselective.

(a)Both (A) and (R) are correct and (R) is the correct explanation of (A).

(b)Both (A) and (R) are correct but (R) is not the correct explanation of
(A).

(c)(A) is correct, but (R) is false.

(d)(A) is false, but (R) is correct.

[UGC J-2008]

509.Which of the population theory/model postulates a necessary causal link


between modernization on the one hand and fertility and mortality
reduction on the other'?

(a)Malthusian model

(b)Communist-socialist theory

(c)Demographic transition model

(d)Neo-Malthusian model

[UGC J-2011]

510.The regularity between the sizes of cities and their rank was first noted
by

(a)Jefferson
(b)Zipf

(c)Christaller

(d)Auerbach

[UGC J-2011]

511.According to Burgess, which of the following sequences of the


concentric zones of cities (from the centre to the outer limit) is correct'?

(a)CBD, Zone of transition, Residential zone, Zone of worker's home,


Commuters zone

(b)Residential zone, CBD, Zone of transition, Commuters zone, Zone of


worker's home

(c)Residential zone, Zone of transition, CBD, Zone of Worker's home,


Commuters zone

(d)CBD, Zone of transition, Zone of worker's home, Residential zone,


Commuters zone

[UGC J-2011]

512.Match List I (theory) with List II (propounder) and select the correct
answer from the codes given.
Codes:

[UGC J-2011]

513.Match List I (author) with List II (theory) and select the correct answer
from the codes given.
Codes:

[UGC J-2010]

514.City-size distribution is best depicted by

(a)urban hierarchy.

(b)central place theory.

(c)rank-size rule.

(d)Losch's hexagonal model.

[UGC J-2010]

515.According to UN projections, which one of the following regions will


contribute maximum to the global urban population by 2030?

(a)South Asia

(b)East Asia

(c)Southeast Asia

(d)Latin America

[UGC J-2010]
516.Given below are two statements, one is labelled as Assertion (A) and the
other is labelled as Reason (R). Select your answers from the codes given
below:

Assertion (A): Social landscape is a region in which one or several groups


live and share a common set of ideas about their immediate environment.

Reason (R): It is a region where human needs, identity, security and


stimulation are satisfied.

(a)Both (A) and (R) are correct and (R) is the correct explanation of (A).

(b)Both (A) and (R) are correct, but (R) is not the correct explanation of
(A).

(c)(A) is correct, but (R) is false.

(d)(A) is false, but (R) is correct.

[UGC J-2010]

517.Given below are two statements, one is labelled as Assertion (A) and the
other is labelled as Reason (R). Select your answers from the codes given
below:

Assertion (A): Logistic population growth is allowed to develop in an


optimal environment of unlimited size.

Reason (R): The growth follows an exponential curve.

(a)Both (A) and (R) are correct and (R) is the correct explanation of (A).

(b)Both (A) and (R) are correct, but (R) is not the correct explanation of
(A).
(c)(A) is correct, but (R) is false.

(d)(A) is false, but (R) is correct.

[UGC J-2010]

518.Given below are two statements, one labelled as Assertion (A) and the
other labelled as Reason (R). Select your answer from the codes given
below:

Assertion (A): Malthus stated that population has a tendency to increase


in geometric ratio, while food production tends to increase in an
arithmetic ratio.

Reason (R): Malthus was optimistic that man could control his own
growth in numbers.

(a)Both (A) and (R) are correct and (R) is the correct explanation of (A).

(b)Both (A) and (R) are correct, but (R) is not the correct explanation of
(A).

(c)(A) is correct, but (R) is false.

(d)(A) is false, but (R) is correct.

[UGC D-2011]

519.Given below are two statements, one labelled as Assertion (A) and the
other labelled as Reason (R). Select your answer from the codes given
below:

Assertion (A): Population concentration is largely found in regions like


river basins and coastal plains.
Reason (R): Concentration of population is mainly due to the availability
of basic resources like soil, minerals and water.

(a)Both (A) and (R) are true and (R) is the correct explanation.

(b)Both (A) and (R) are true, but (R) is not the correct explanation.

(c)(A) is true, but (R) is false.

(d)(A) is false, but (R) is true.

[UGC D-2011]

520.The concept of the `Radiant City' was propounded by

(a)Le Corbusier

(b)Ernst May

(c)Soria Y Mata

(d)Clarence Perry

[UGC D-2011]

521.The sector theory of city morphology

(a)incorporates the importance of transportation systems into the


evolution of the social structure of the city.

(b)is particularly concerned with the concept of ecology.

(c)postulates that multiple centres or nuclei are responsible for urban


growth.
(d)is concerned with directional bias.

[UGC D-2011]

522.Harris and Ullman are best known for their contributions to

(a)political geography.

(b)urban geography.

(c)economic geography.

(d)agricultural geography.

[UGC D-2011]

523.`Principle of least effort' related to human migration was propounded by

(a)Stouffer

(b)Revenstein

(c)Weber

(d)Jiff

[UGC D-2010]

524.Arrange the following countries in accordance with their population


size:

(a)Pakistan, Bangladesh, Sri Lanka, Nepal

(b)Pakistan, Sri Lanka, Nepal, Bangladesh

(c)Bangladesh, Pakistan, Nepal, Sri Lanka


(d)Sri Lanka, Bangladesh, Nepal, Pakistan

[UGC D-2010]

525.Among the following South Asian countries, which one is the most
urbanized'?

(a)Nepal

(b)Bangladesh

(c)Maldives

(d)Sri Lanka

[UGC D-2010]

526.Given below are two statements, one labelled as Assertion (A) and the
other labelled as Reason (R). Select your answer from the codes given.

Assertion (A): At present, urban natural increase is not an important


contributor to high rates of urban growth as the rural urban migration.

Reason (R): Because of lower mortality, natural rate of increase is higher


in the modern era than in the historical past.

(a)Both (A) and (R) are true and (R) is the correct explanation of (A).

(b)Both (A) and (R) are true, but (R) is not the correct explanation of (A).

(c)(A) is true, but (R) is false.

(d)(A) is false, but (R) is true.

[UGC D-2010]
527.Other than Kerala, which of the following states has already achieved
the replacement level of fertility'?

(a)Assam

(b)Gujarat

(c)Karnataka

(d)Tamil Nadu

[UPSC-2006]

528.In which of the following countries is the urban population as per cent
of its total population the highest?

(a)Australia

(b)Japan

(c)New Zealand

(d)USA

[UPSC-2006]

529.Assertion (A): The percentage of population whose income is less than


1 US dollar/day is less in Pakistan than in India.

Reason (R): The adult literacy rate (percentage) of age 15 years and
above (with literacy) is more in Pakistan than in India.

(a)Both (A) and (R) are true and (R) is the correct explanation of (A).

(b)Both (A) and (R) are true, but (R) is not the correct explanation of (A).
(c)(A) is true, but (R) is false.

(d)(A) is false, but (R) is true.

[UPSC-2006]

530.As per the World Development Indicators of World Bank, in which


South Asian country among the following is the average annual growth
rate of population during 2002-15 expected to be the highest'?

(a)Bangladesh

(b)India

(c)Nepal

(d)Pakistan

[UPSC-2006]

531.Which one of the following patterns of settlement is most likely to


develop along the convergence of several transport routes'?

(a)Circular

(b)Linear

(c)Rectangular

(d)Star-shaped

[UPSC-2006]

532.Consider the following statements:


1.Khasi society is patrilineal.

2.Pastoralism is the main occupation of the Tharus.

Which of the statements given above is/ are correct?

(a)1 only

(b)2 only

(c)Both 1 and 2

(d)Neither 1 nor 2

[UPSC-2006]

533.Consider the following statements:

1.Abujhmaria

2.Gaddi

For which of the above tribes is the economic activity mainly pastoral
herding'?

(a)1 and 2 only

(b)2 and 3 only

(c)1 and 3 only

(d)1, 2 and 3

[UPSC-2006]
534.Who propounded the concentric circle theory of city morphology'?

(a)Burgess

(b)Hoyt

(c)Ulman

(d)Mumford

[UPSC-2006]

535.Consider the following statements

1.Radicalism in geography is critical in geography as a spatial science.

2.Radicalism is aimed at a general restructuring of the capitalist society.

3.Radicalism in geography relied heavily on empiricist methods.

Which of the statements given above is/ are correct'?

(a)1 only

(b)1 and 2 only

(c)2 and 3 only

(d)1, 2 and 3

[UPSC-2006]

536.Consider the following statements:


1.Humanistic geography depended on the growing need of social
relevance of the subject.

2.Humanistic geography developed due to reaction to excessive


quantification and model building in geography.

3.Humanistic geography drew much from the radical trend in geography.

Which of the statements given above is/ are correct'?

(a)1 only

(b)2 only

(c)Both 1 and 2

(d)Neither 1 nor 2

[UPSC-2006]

537.Consider the following statements: Behaviourism refers to:

1.spatial behaviour of people.

2.spatial perception.

3.spatial experience and imagination.

4.mental maps.

Which of the statements given above is/ are correct'?

(a)1 and 2

(b)1 and 3
(c)2 and 3

(d)2 and 4

[UPSC-2006]

538.Who among the following propounded the `law of primate city"?

(a)Carl Sauer

(b)Isaiah Bowman

(c)Mark Jefferson

(d)Walter Christaller

[UPSC-2007]

539.With reference to migration, who among the following formulated the


`Gravity Model' which demonstrates the relationship between population
size, distance and migration?

(a)Zipf

(b)Miller

(c)Malthus

(d)Zelinsky

[UPSC-2009]

540.Kampung is a form of rural settlement found in

(a)China
(b)Malaysia

(c)Philippines

(d)Cambodia

[UPSC-2009]

ANSWER KEYS
 
1.accounts for the majority of the coal burned annually in the USA.

(a)Coke manufacture for the steel industry

(b)Generation of electricity

(c)Production of carbon black, synthetic diamonds and graphite

(d)Home and industrial space heating

2.accounts for the majority of the coal burned annually in the USA.

(a)Coke manufacture for the steel industry

(b)Generation of electricity

(c)Production of carbon black, synthetic diamonds and graphite

(d)Home and industrial space heating

3.are both calcium-bearing minerals.

(a)Calcite and gypsum

(b)Bauxite and kaolin

(c)Halite and sylvite

(d)Quartz and plagioclase

4.are important fertilizer minerals with their correct elements.


(a)Apatite and sylvite

(b)Halite and calcite

(c)Cassiterite and albite

(d)Sylvite and gypsum

5. are used as abrasives.

(a)Talc and graphite

(b)Calcite and gypsum

(c)Apatite and galena

(d)Diamond and garnet

6.coal is typically found only in association with tightly folded strata.

(a)Bituminous

(b)Peat

(c)Lignite

(d)Anthracite

7.has the largest current production and reserves of crude oil.

(a)Texas

(b)The North Sea field

(c)Offshore Japan
(d)Central and eastern Saudi Arabia

8.has the largest current production and reserves of crude oil.

(a)Texas

(b)The North Sea field

(c)Offshore Japan

(d)Central and eastern Saudi Arabia

9. is a non-renewable resource.

(a)Water

(b)Solar power

(c)Petroleum

(d)Timber

10. is a non-renewable resource.

(a)Water

(b)Solar power

(c)Petroleum

(d)Timber

11.is an essential component of plasters and plasterboard.

(a)Talc
(b)Gypsum

(c)Garnet

(d)Apatite

12.f is the main fuel used in nuclear ission reactors to produce electricity.

(a)Uranium-239

(b)Thorium-238

(c)Thorium-232

(d)Uranium-235

13. is the mineral source of phosphorous in phosphate fertilizers.

(a)Phosphorite

(b)Kaolinite

(c)Garnet

(d)Apatite

14. is too soft to be used as an abrasive for steel, glasses and most silicate
rocks.

(a)Diamond

(b)Garnet

(c)Corundum
(d)Calcite

15. is too soft to be used as an abrasive for steel, glasses and most silicate
rocks.

(a)Diamond

(b)Garnet

(c)Corundum

(d)Calcite

16.is used in pencils and as a solid lubricant.

(a)Graphite

(b)Galena

(c)Bornite

(d)Gypsum

17.supplies the largest percentage of energy consumed annually in the USA.

(a)Coal

(b)Hydroelectricity

(c)Uranium

(d)Petroleum

18.supplies the largest percentage of energy consumed annually in the USA.


(a)Coal

(b)Hydroelectricity

(c)Uranium

(d)Petroleum

19., a rock-forming silicate mineral used as an insulator in electrical


equipment, occurs as large crystals with quartz and feldspars in
pegmatites.

(a)Hornblende

(b)Muscovite

(c)Olivine

(d)Graphite

20.` Estancias' are

(a)vast cattle farms in Argentina.

(b)smaller estates in Brazil.

(c)plantation farms in Malaysia.

(d)commercial grain farms in western USA.

[UPSC-2000]

21.`Transhumance' refers to

(a)migration of animals in ranches.


(b)human migration in search of job.

(c)seasonal movement of people and their herds form valley to mountain


and vice versa.

(d)migration of nomads.

[UPSC-2000]

22.A major part of freshwater fish is caught in

(a)eastern and southern Asia.

(b)western Africa.

(c)South Africa.

(d)western Europe.

[UPSC-1996]

23.A majority of the population of Newfoundland is engaged in

(a)lumbering.

(b)fishing.

(c)hunting.

(d)dairying.

[UPSC-1995]

24.According to Central Place Theory, which one of the following


statements most aptly describes the distribution area of a commodity
around a central place'?

(a)The area which supports the threshold population for the commodity

(b)The area in which the central place supplies more of the commodity
than does any other central place.

(c)The area bounded by the line joining the points of maximum distance
up to which the commodity is supplied by the central place

(d)The area bounded by the line joining the points of maximum distance
from which people come to the central place to buy the commodity.

[UPSC-1997]

25.Assertion (A): Agriculture in monsoon Asia is characterized by the


predominance of food crop cultivation.

Reason (R): Farming in monsoon Asia is mainly meant for subsistence


purpose.

(a)Both (A) and (R) are true and (R) is the correct explanation of (A).

(b)Both (A) and (R) are true but (R) is not the correct explanation of (A).

(c)(A) is true but (R) is false.

(d)(A) is false but (R) is true.

[UPSC-2003]

26.Assertion (A): Aluminium industry is located near the source of


power.
Reason (R): Industries are usually located close to the source of raw
materials.

(a)Both (A) and (R) are true and (R) is the correct explanation of (A).

(b)Both (A) and (R) are true but (R) is not the correct explanation of (A).

(c)(A) is true but (R) is false.

(d)(A) is false but (R) is true.

[UPSC-1995]

27.Assertion (A): Butter, wool and frozen meat are the most important
exports of New Zealand.

Reason (R): New Zealand has a cool moist climate and vast grasslands to
support livestock.

(a)Both (A) and (R) are true and (R) is the correct explanation of (A).

(b)Both (A) and (R) are true but (R) is not the correct explanation of (A).

(c)(A) is true but (R) is false.

(d)(A) is false but (R) is true.

[UPSC-2002]

28.Assertion (A): Chile continues to be an important producer copper in the


world.

Reason (R): Chile is endowed with the world's largest deposit of porphyry
copper.
(a)Both (A) and (R) are true and (R) is the correct explanation of (A).

(b)Both (A) and (R) are true but (R) is not the correct explanation of (A).

(c)(A) is true but (R) is false.

(d)(A) is false but (R) is true.

[UPSC-1999]

29.Assertion (A): Denmark supplies dairy products and bacon to


neighbouring markets in northern Europe.

Reason (R): Denmark produces highquality grain and grass and its
neighbours are industrialized.

(a)Both (A) and (R) are true and (R) is the correct explanation of (A).

(b)Both (A) and (R) are true but (R) is not the correct explanation of (A).

(c)(A) is true but (R) is false.

(d)(A) is false but (R) is true.

[UPSC-2002]

30.Assertion (A): In areas of intensive subsistence agriculture, animal


husbandry is a meagre sideline of crop farming.

Reason (R): Throughout the region, the farmer is handicapped in raising


cattle due to the lack of scientific methods of breeding.

(a)Both (A) and (R) are true and (R) is the correct explanation of (A).

(b)Both (A) and (R) are true but (R) is not the correct explanation of (A).
(c)(A) is true but (R) is false.

(d)(A) is false but (R) is true.

[UPSC-2000]

31.Assertion (A): In Australia, cattle rearing is done more for meat than for
milk.

Reason (R):Australians are traditionally.

(a)Both (A) and (R) are true and (R) is the correct explanation of (A).

(b)Both (A) and (R) are true but (R) is not the correct explanation of (A).

(c)(A) is true but (R) is true.

(d)(A) is false but (R) is true.

[UPSC-2000]

32.Assertion (A): In Kazakhstan, transhumance is practiced.

Reason (R): There is a seasonal shifting of snowline in the mountains.

(a)Both (A) and (R) are true and (R) is the correct explanation of (A).

(b)Both (A) and (R) are true but (R) is not the correct explanation of (A).

(c)(A) is true but (R) is false.

(d)(A) is false but (R) is true.

[UPSC-1998]
33.Assertion (A): Increase in agricultural production in China has been due
to higher per hectare yields.

Reason (R): China accomplished socialist transformation in all branches


of economy, including agriculture.

(a)Both (A) and (R) are true and (R) is the correct explanation of (A).

(b)Both (A) and (R) are true but (R) is not the correct explanation of (A).

(c)(A) is true but (R) is false.

(d)(A) is false but (R) is true.

[UPSC-2002]

34.Assertion (A): Insect-resistant transgenic cotton has been produced by


inserting Bt gene.

Reason (R): The Bt gene is derived from a bacterium.

(a)Both (A) and (R) are true and (R) is the correct explanation of (A).

(b)Both (A) and (R) are true but (R) is not a correct explanation of (A).

(c)(A) is true but (R) is true.

(d)(A) is false but (R) is true.

[UPSC-1999]

35.Assertion (A): Sri Lanka produces more tea in central and eastern
regions.
Reason (R): The seasonally occurring heavy rainfall is responsible for
much higher production.

(a)Both (A) and (R) are true and (R) is the correct explanation of (A).

(b)Both (A) and (R) are true but (R) is not the correct explanation of (A).

(c)(A) is true but (R) is false.

(d)(A) is false but (R) is true.

[UPSC-2002]

36.Assertion (A): The concept of plantation agriculture has changed in


recent years.

Reason (R): Change in the size of landholding led to the introduction of


small and medium farmers in the market.

(a)Both (A) and (R) are true and (R) is the correct explanation of (A).

(b)Both (A) and (R) are true but (R) is not the correct explanation of (A).

(c)(A) is true but (R) is false.

(d)(A) is false but (R) is true.

[UPSC-2001]

37.Assertion (A): The North Atlantic Ocean route is the world's most
important ocean route.

Reason (R): The North Atlantic Ocean route brings together the
developing and developed nations.
(a)Both (A) and (R) are true and (R) is the correct explanation of (A).

(b)Both A and (R) are true but (R) is not the correct explanation of (A).

(c)(A) is true but (R) is false.

(d)(A) is false but (R) is true.

[UPSC-2003]

38.Assertion (A): New England region in the USA continues to be an


important area for cotton textiles.

Reason (R): Market and geographical inertia are the main reasons for
cotton textiles here.

(a)Both (A) and (R) are true and (R) is the correct explanation of (A).

(b)Both (A) and (R) are true but (R) is not the correct explanation of (A).

(c)(A) is true but (R) is false.

(d)(A) is false but (R) is true.

[UPSC-2000]

39.Augusta and Atlanta, the two important cotton textile centres, are located
in

(a)South Carolina.

(b)Massachusetts.

(c)North Carolina.
(d)Georgia.

[UPSC-1996]

40.Bauxite is an ore of

(a)iron

(b)gold

(c)tin

(d)aluminium

41.Cap rock and reservoir strata refer to accumulations of

(a)oil shale

(b)bedded rock salt

(c)sedimentary iron ore

(d)petroleum

42.Chemical decomposition of produces acidic soil waters that can cause


secondary enrichment in copper and in other ore deposits.

(a)quartz

(b)bauxite

(c)calcite

(d)pyrite
43.Coal and petroleum are considered to be fossil fuels because

(a)their energy content was derived from ancient sunlight

(b)coal beds and petroleum reservoir rocks contain abundant fossils

(c)carbon dioxide, released when they burn, contributes to the greenhouse


effect

(d)their oxygen and nitrogen contents were derived from the ancient
atmosphere

44.Comparatively speaking, the USA uses energy fuels and minerals

(a)about equally as efficient as other technologically advanced countries

(b)less efficiently than other technologically advanced countries

(c)so efficiently that only domestic coal, petroleum and minerals are
utilized

(d)very efficiently, so consumption per person is below than in less


industrialized countries like Brazil and Mexico

45.Comparatively speaking, the USA uses energy fuels and minerals

(a)about equally as efficient as other technologically advanced countries

(b)less efficiently than other technologically advanced countries

(c)so efficiently that only domestic coal, petroleum and minerals are
utilized
(d)very efficiently, so consumption per person is below than in less
industrialized countries like Brazil and Mexico

46.Consider the following activities:

1.Fishing and forestry

2.Pastoral herding

3.Milk and meat diet

Which of these are common among the Kurds and Kirghiz?

(a)1 and 2

(b)2 and 3

(c)1 and 3

(d)1, 2 and 3

[UPSC-2002]

47.Consider the following Southeast Asian countries which produce 90 per


cent of the world's rubber.

1.Indonesia

2.Malaysia

3.Sri Lanka

4.Vietnam
The correct sequence of these rubber producers in decreasing order of
importance is

(a)1, 2, 3, 4.

(b)2, 1, 3, 4.

(c)1, 2, 4, 3.

(d)2, 1, 4, 3.

[UPSC-1997]

48.Consider the following statements:

1.Germany and USA are the two largest producers of windpower.

2.USA has a nuclear energy generation capacity of around 1,00,00. MW.

3.USA has more nuclear reactors than any other country in the world.

Which of the statements given above are correct'?

(a)1 and 2

(b)2 and 3

(c)1 and 3

(d)1, 2 and 3

[UPSC-2004]

49.Consider the following statements regarding equatorial forests:


1.They consist mostly of hardwood trees.

2.In very dense areas, undergrowth is absent.

3.There is segregation of the same species of trees.

4.Lumbering is well developed in these forests.

Of these statements

(a)1, 2 and 4 are correct

(b)2, 3 and 4 are correct

(c)1 and 2 are correct

(d)1 and 3 are correct

[UPSC-1998]

50.Consider the following statements:

1.Iron and steel industries developed around lake Michigan in USA due
to the availability of good quality coal.

2.Pittsburgh-Youngstown area is the leading producer of steel in USA.

3.The Lorraine area in France is significant for steel production.

4.The south Manchurian region accounts for nearly 60 per cent of China's
pig iron production.

Which of the above statements are correct'?

(a)1, 3 and 4
(b)1, 2 and 4

(c)2, 3 and 4

(d)2 and 3

[UPSC-2000]

51.Consider the following statements:

1.Cane sugar is preferred to beet sugar.

2.Beet sugar is insufficient to meet the local demand.

3.There is a low demand and surplus production of cane sugar in


producing countries.

4.Cane sugar producing countries were colonies of importing countries.

5.Production cost of cane is low.

Which of the above reasons account for the fact that cane sugar has a
major share in the international sugar trade?

(a)1, 2 and 3

(b)2, 3 and 4

(c)1, 4 and 5

(d)2, 3 and 5

[UPSC-2000]

52.Consider the following statements:


1.Nomadism is a high degree of spatial mobility as a basis for a particular
way of life.

2.Nomadism is confined to the semiarid regions of the world.

3.Nomadism is presumed to be a synonymous with movements of herds


and livestock rearing.

4.Hunter gatherers who typically have no domesticated livestock are also


defined as nomads.

Which of these are correct?

(a)1, 2 and 3

(b)2, 3 and 4

(c)1, 3 and 4

(d)1, 2 and 4

[UPSC-2002]

53.Consider the following statements:

Tropical crops which are in great demand in Anglo-America and Europe


are produced in great variety and volume in Latin America and Africa
because of

1.availability of cultivable land and suitability of soil conditions.

2.combination of climatic and topographical conditions.

3.lack of mineral resources for trading purpose.


4.non-existence of agro-based industries.

Which of these are correct?

(a)1 and2

(b)2 and 3

(c)1, 3 and 4

(d)2, 3 and 4

[UPSC-2002]

54.Consider the following statements:

Decline of world fish haul is due to

1.availability of cheap alternatives.

2.changing food habit.

3.indiscriminate fishing.

4.over fishing.

5.water pollution.

Of these statements

(a)1, 2 and 3 are correct.

(b)2, 3 and 4 are correct.

(c)1, 4 and 5 are correct.


(d)3, 4 and 5 are correct.

[UPSC-1995]

55.Consider the following statements:

1.Rice is predominantly cultivated in the tropical areas.

2.USA is one of the exporters of rice.

3.Thailand is an important importer of rice.

4.In Italy, rice is cultivated in the Po valley.

Of these statements

(a)1, 2, 3 and 4 are correct.

(b)1, 2 and 4 are correct.

(c)1 and 3 are correct.

(d)2 and 4 are correct.

[UPSC-1998]

56.Consider the following statements:

1.Most of the fishing grounds occur in areas where the continental shelf is
wide.

2.Fishing activity is well developed in warm tropical waters.

3.Mixing of warm and cold currents brings plant nutrients for fish.
4.Inland fisheries are more significant than other types in India.

Of these statements

(a)1 and 4 are correct.

(b)1 and 3 are correct.

(c)2, 3 and 4 are correct.

(d)1, 2 and 3 are correct.

[UPSC-1998]

57.Copper and copperzinc sulphide deposits formed around ancient seafloor,


hot spring vents would be most likely to be found in

(a)an exotic, oedipus complex

(b)an accretionary wedge complex

(c)a far travelled, cratonic complex

(d)an ophiolite complex

58.Detrital grains of are not likely to be concentrated in placer deposits.

(a)native gold

(b)diamond

(c)gypsum

(d)corundum
59.Fishing in inland waters is most important in

(a)Russia.

(b)Anglo-America.

(c)monsoon Asia.

(d)central Africa.

[UPSC-1997]

60.Fishing, hunting and food gathering activities are carried out by

(a)Semangs of Malaysia.

(b)Masais of east Africa.

(c)Bedouins of Arabia.

(d)Kirghiz of central Asia.

[UPSC-1995]

61.Graphite deposits typically form geologically

(a)by decomposition of humus and soil gases during intense, tropical


weathering

(b)by regional metamorphism of organic-rich, black shales

(c)as deposits around submarine, hot spring vents

(d)as hydrothermal vein deposits in limestone around a granitic batholith


62.Horticulture is most widespread in the

(a)Mediterranean regions.

(b)subtropical regions.

(c)temperate regions.

(d)equatorial regions.

[UPSC-1997]

63.Identify the correct descending order of production of wood pulp of the


following countries:

1.USA

2.Sweden

3.Japan

4.Canada

Select the correct answer using the codes given.

(a)1, 3, 2, 4

(b)2, 4, 1, 3

(c)1, 4, 2, 3

(d)2, 3, 1, 4

[UPSC-2004]
64.In the world, viticulture, horticulture, sericulture are well developed in
the

(a)monsoon regions.

(b)tropical highlands.

(c)Mediterranean regions.

(d)marine west coast areas.

[UPSC-1998]

65.For which one of the following crops is international trade low in the
context of total produce?

(a)Rice

(b)Coffee

(c)Rubber

(d)Wheat

[UPSC-1995]

66.In which one of the following mountain ranges of South Africa are the
world's greatest goldfields located'?

(a)Roggeveldberg

(b)Groot Swartberg

(c)Witwatersrand
(d)Drakensberg

[UPSC-2003]

67.In which one of the following natural regions is subsistence farming the
main occupation?

(a)Mediterranean type

(b)Steppe type

(c)China type

(d)Monsoon type

[UPSC-1996]

68.Le Have bank is a fishing bank in

(a)North Pacific coast.

(b)South Pacific coast.

(c)North Atlantic coast.

(d)South Atlantic coast.

[UPSC-1999]

69.Main bunting ground for blue whales is located in

(a)Arctic region.

(b)Antarctic region.

(c)West Pacific Ocean.


(d)South Atlantic Ocean.

[UPSC-1997]

70.Match List I with List II and select the correct answer from the codes
given.

Codes:

[UPSC-2002]

71.Match List I with List II and select the correct answer from the codes
given.
Codes:

[UPSC-2000]

72.Match List I with List II and select the correct answer from the codes
given.
Codes:

[UPSC-1998]

73.Match List I with List II and select the correct answer from the codes
given.

Codes:

[UPSC-2004]

74.Match List I with List II and select the correct answer from the codes
given.
Codes:

[UPSC-1999]

75.Match List I with List II and select the correct answer from the codes
given.

Codes:
[UPSC-2005]

76.Match List I with List II and select the correct answer from the codes
given.

Codes:

[UPSC-1998]

77.Match List I with List II and select the correct answer from the codes
given.
Codes:

[UPSC-1999]

78.Match List I with List II and select the correct answer from the codes
given.
Codes:

[UPSC-1997]

79.Match List I with List II and select the correct answer from the codes
given.

Codes:
[UPSC-1995]

80.Match List I with List II and select the correct answer from the codes
given.

Codes:

[UPSC-1995]

81.Match List I with List II and select the correct answer from the codes
given.
Codes:

[UPSC-1995]

82.Match List I with List II and select the correct answer from the codes
given.

Codes:
[UPSC-1995]

83.Match List I with List II and select the correct answer from the codes
given.

Codes:

[UPSC-1995]

84.Match List I with List II and select the correct answer from the codes
given.
Codes:

[UPSC-1996]

85.Match List I with List II and select the correct answer from the codes
given.

Codes:
[UPSC-1996]

86.Match List I with List II and select the correct answer from the codes
given.

Codes:

[UPSC-1996]

87.Match List I with List II and select the correct answer from the codes
given.
Codes:

[UPSC-1997]

88.Match List I with List II and select the correct answer from the codes
given.

Codes:
[UPSC-1997]

89.Match List I with List II and select the correct answer from the codes
given.

Codes:

[UPSC-1997]

90.Match List I with List II and select the correct answer from the codes
given.
Codes:

[UPSC-1997]

91.Match List I with List II and select the correct answer from the codes
given.

Codes:
[UPSC-1997]

92.Match List I with List II and select the correct answer from the codes
given.

Codes:

[UPSC-1997]

93.Match List I with List II and select the correct answer from the codes
given.
Codes:

[UPSC-1998]

94.Match List I with List II and select the correct answer from the codes
given.

Codes:
[UPSC-1998]

95.Match List I with List II and select the correct answer from the codes
given.

Codes:

[UPSC-1999]

96.Match List I with List II and select the correct answer from the codes
given.
Codes:

[UPSC-1999]

97.Match List I with List II and select the correct answer from the codes
given.

Codes:
[UPSC-2000]

98.Match List I with List II and select the correct answer from the codes
given.

Codes:

[UPSC-2000]

99.Match List I with List II and select the correct answer from the codes
given.
Codes:

[UPSC-2000]

100.Match List I with List II and select the correct answer from the codes
given.

Codes:
[UPSC-2001]

101.Match List I with List II and select the correct answer from the codes
given.

Codes:

[UPSC-2002]

102.Match List I with List II and select the correct answer from the codes
given.
Codes:

[UPSC-2002]

103.Match List I with List II and select the correct answer from the codes
given.

Codes:
[UPSC-2002]

104.Match List I with List II and select the correct answer from the codes
given.

Codes:

[UPSC-2003]

105.Match List I with List II and select the correct answer from the codes
given.
Codes:

[UPSC-2003]

106.Match List I with List II and select the correct answer from the codes
given.

Codes:
[UPSC-2003]

107.Match List I with List II and select the correct answer from the codes
given.

Codes:

[UPSC-2003]

108.Match List I with List II and select the correct answer from the codes
given.
Codes:

[UPSC-2005]

109.Maximum amount of herring fish is obtained from

(a)the North Sea around the Netherlands.

(b)the coastal areas from Alaska to California.

(c)the Sea of Japan.

(d)the Gulf of Mexico.

[UPSC-2004]

110.Mekong Ganga Co-operation Project is

(a)an irrigation project involving India and Myanmar.

(b)a joint tourism initiative of some Asian countries.


(c)a hydroelectric power project involving India.

(d)a defence and security agreement of India with its eastern neighbours.

[UPSC-2001]

111.Most of the Japanese industries are located on the

(a)Hokkaido coast.

(b)Honshu coast.

(c)Shikoku coast.

(d)Kyushu coast.

[UPSC-2004]

112.Nitrate deposits are found in the northern part of

(a)Ecuador.

(b)Chile.

(c)Brazil.

(d)Columbia.

[UPSC-1997]

113.Non-commercial pastoral nomadism is a way of life in the

(a)Veld of South Africa.

(b)Steppes of Mongolia.
(c)Prairies of Canada.

(d)Australian Downs.

[UPSC-1999]

114.Of the following fossil fuels, is most likely to have the lowest sulphur
content and smallest volume of solid ash residue.

(a)bituminous coal

(b)natural gas

(c)heavy, asphaltic, crude oil

(d)lignite

115.On global perspective, the pattern of energy consumption (in descending


order of proportion) is

(a)coal, oil, natural gas, hydel power.

(b)oil, coal, hydel power, natural gas.

(c)oil, coal, natural gas, hydel power.

(d)coal, oil, hydel power, natural gas.

[UPSC-1998]

116.Ore deposits of form by prolonged, intense, tropical weathering of


specific kinds of bedrock.

(a)silver
(b)mercury

(c)magnesium

(d)aluminium

117.Assertion (A): Interactions among inherent factors in economic


activities affect the texture of spatial organization.

Reason (R): There are regional disparities in the spatial organizations of


economy.

(a)Both (A) and (R) are true and (R) is the correct explanation of (A).

(b)Both (A) and (R) are true but (R) is not the correct explanation of (A).

(c)(A) is true but (R) is false.

(d)(A) is false but (R) is true.

[UPSC-1997]

118.Rich coal deposits of China are found in

(a)Shansi and Shensi.

(b)Shantung.

(c)Hunan.

(d)Kwangtung and Kwangsi.

[UPSC-1997]
119.Sandstone is a much more common reservoir rock for petroleum than
shale because

(a)shales, especially black shales, are much richer in primary organic


matter than sandstones

(b)sandstone is more abundant than shale

(c)shale is more porous so the oil tends to leak out over time

(d)sandstones are more permeable than shales so subsurface flows of


fluids tend to be directed through sandstone strata rather than through
shales

120.Shaba province in the Democratic Republic of the Congo is known for


which of the following reserves?

(a)Bauxite

(b)Iron ore

(c)Tungsten

(d)Copper

[UPSC-2004]

121.Synthetic rubber is likely to replace natural rubber for domestic and


industrial purposes in future because

(a)natural rubber is grown in the tropical counties but is utilized in the


rich and industrialized temperate countries.
(b)natural rubber can hardly meet the growing demand of various
industries.

(c)the raw materials for synthetic rubber are available in various parts of
the world.

(d)the demand for rubber in the world is gradually on the increase.

[UPSC-1997]

122.Tarsands containing large molecular hydrocarbons like crude oil are


found in limited quantities in several parts of the world but the most
important known occurrence is in

(a)USA.

(b)Canada.

(c)Australia.

(d)Russia.

[UPSC-2004]

123.The largest producer of cloves in the world is

(a)Comoros.

(b)Mauritius.

(c)Seychelles.

(d)Zanzibar.

[UPSC-1998]
124.The formation, rich in oil shale strata, was deposited in early tertiary
lakes that once covered parts of Colorado, Utah and Wyoming.

(a)White Prairie

(b)Green River

(c)Red Lake

(d)Blue Mountain

125.The formation, rich in oil shale strata, was deposited in early tertiary
lakes that once covered parts of Colorado, Utah and Wyoming.

(a)White Prairie

(b)Green River

(c)Red Lake

(d)Blue Mountain

126.The American cotton textile industry moved from New England to the
Southern States because

(a)the latter have a more humid climate.

(b)New England diversified its manufacturing industries.

(c)the population growth in the south provided a major market.

(d)the south offered lower cost of production.

[UPSC-2002]
127.The Appalachian mountains are economically important because they
provide

(a)a storehouse of timber and fuel wood.

(b)natural passes, easily negotiable by modern means of transport.

(c)mineral wealth forming the basics of key industries.

(d)health resorts attracting tourists both in summer and in winter.

[UPSC-1995]

128.The Canadian Pacific Railway runs between

(a)Edmonton and Vancouver.

(b)Montreal and Vancouver.

(c)Ottawa and Prince Rupert.

(d)Halifax and Vancouver.

[UPSC-2003]

129.The classical model of industrial location theory in which least cost


approach is of primary consideration is recognized as the industrial
location theory of

(a)Weber.

(b)Smith.

(c)Isard.
(d)Fetter.

[UPSC-1997]

130.The correct sequence of the types of farming which a surveyor will


come across from Omaha to Chicago will be

(a)livestock, dairy, grain.

(b)livestock, grain, dairy.

(c)grain, mixed farming, dairy.

(d)grain, livestock, dairy.

[UPSC-1996]

131.The correct sequence of Von Thune's optional land use rings is

(a)cereal farming, woodlands marketgardening, grazing.

(b)marketgardening, woodlands, cereal farming, grazing.

(c)marketgardening, grazing, cereal farming, woodlands.

(d)grazing, marketgardening, cereal farming, woodlands.

[UPSC-1998]

132.The country well-known for its smallprecision high cost industrial


goods is

(a)Poland.

(b)Switzerland.
(c)France.

(d)Germany.

[UPSC-1996]

133.The emerging trading blocs in the world, such as NAFTA, ASEAN and
the like, are expected to

(a)act as constrictions in free trade across the world.

(b)promote free trade on the lines laid down by the WTO.

(c)permit transfer of technology between member centuries.

(d)promote trade in agricultural commodities between countries of the


North and the South.

[UPSC-1996]

134.The extensive coffee plantations in Brazil are called

(a)fazendas.

(b)trasil.

(c)corrals.

(d)estancias.

[UPSC-1997]

135.The first large-scale commercial, geothermal, electrical power-


generation facility in the USA was brought into production at
(a)Yellowstone National Park in 1925

(b)Hot Springs, Arkansas in 1970

(c)The Geysers, northern California in 1960

(d)Oak Ridge, Tennessee in 1945

136.The first large-scale commercial, geothermal, electrical power


generation facilityin the USA was brought into production at

(a)Yellowstone National Park in 1925

(b)Hot Springs, Arkansas in 1970

(c)The Geysers, northern California in 1960

(d)Oak Ridge, Tennessee in 1945

137.The Geysers and Larderello are facilities in California and Italy,


respectively.

(a)diamond mining

(b)copper mining

(c)geothermal power

(d)hydroelectric

138.The Geysers and Larderello are facilities in California and Italy,


respectively.

(a)diamond mining
(b)copper mining

(c)geothermal power

(d)hydroelectric

139.The highest hydropower potential in the African continent is in

(a)north-western Africa.

(b)north-eastern Africa.

(c)equatorial Africa.

(d)southern Africa.

[UPSC-2003]

140.The hunting and gathering economies can support only

(a)1 person per sq. km.

(b)3 persons per sq. km.

(c)5 persons per sq. km.

(d)7 persons per sq. km.

[UPSC-2001]

141.The important iron ore mining region situated in Siberian Russia is

(a)Krivoy Rog.

(b)Kursk.
(c)Magnitogorsk.

(d)Kuznetsk.

[UPSC-1997]

142.The Japanese are fish and rice eaters as they

(a)are Asians.

(b)are Buddhists.

(c)cultivate rice and catch fish.

(d)import fish and rice.

[UPSC-1995]

143.The largest coal resources are found at

(a)Irkutsk region.

(b)Omsk-Tomsk region.

(c)Kuznetsk Basin.

(d)Tashkent Basin.

[UPSC-1996]

144.The largest oilfield of Iraq is

(a)Al-Faw.

(b)Half Sulaiman.
(c)Kirkuk.

(d)Khor-al-Amaya.

[UPSC-1997]

145.The largest producer of fruits in the Anglo-American region is

(a)New England region.

(b)Great Lakes region.

(c)California region.

(d)Application region.

[UPSC-1995]

146.The largest producer of paper in the world is

(a)Canada.

(b)Japan.

(c)Sweden.

(d)USA.

[UPSC-1997]

147.The most important pelagic fish found in the North Sea adjacent to the
UK is

(a)cod.

(b)halibut.
(c)herring.

(d)mackerel.

[UPSC-1999]

148.The most important reason for dwindling forest resources today is

(a)soil erosion.

(b)forest fire.

(c)floods.

(d)over felling.

[UPSC-1995]

149.The most important type of forest in terms of economic value and easy
exploitation is

(a)temperate deciduous forest.

(b)coniferous forest.

(c)tropical monsoon forest.

(d)equatorial forest.

[UPSC-1998]

150.The percapita hydel power output is the highest in

(a)Japan.

(b)Norway.
(c)Sweden.

(d)USA

[UPSC-1999]

151.The plantation farming regions of the world differ from subsistence


agricultural regions because they

(a)have organized agriculture.

(b)are carried on in entirely different environmental conditions.

(c)do not occupy large portions of the Earth's surface.

(d)provide livelihood for people distributed over wide area.

[UPSC-1997]

152.The shift of aluminium industry from the Dnieper region to Siberia was
made possible because of

(a)exploration of new bauxite sources.

(b)development of hydroelectricity.

(c)new favourable market location.

(d)development of efficient transportation.

[UPSC-2001]

153.The shortest route from Los Angeles to Moscow is over the

(a)Atlantic Ocean.
(b)Pacific Ocean.

(c)North Pole.

(d)South Pole.

[UPSC-2003]

154.The south-western part of California is known for

(a)subsistence agriculture.

(b)forestry.

(c)temperate grain farming.

(d)Mediterranean agriculture.

[UPSC-2003]

155.The USA, with about 6 per cent of the world's population, uses about of
the world's total annual energy production.

(a)30 per cent

(b)15 per cent

(c)9 per cent

(d)3 per cent

156.The world's largest plant for generating electricity from tides is

(a)in the Rance River estuary, Brittany, along the Atlantic coast of France
(b)along the mouth of the Amazon River in Brazil

(c)in the Netherlands along the Rhine River delta

(d)along the coast of California, just north of San Francisco

157.The world's largest plant for generating electricity from tides is

(a)in the Rance River estuary, Brittany, along the Atlantic coast of France

(b)along the mouth of the Amazon River in Brazil

(c)in the Netherlands along the Rhine River delta

(d)along the coast of California, just north of San Francisco

158.The world's largest commercially developed tar sand deposit is

(a)in northern Brunei, Island of Borneo (b) in north-western Colorado,


USA

(c)in southern Iraq, near Basra

(d)near Lake Athabasca in central Canada

159.The world's largest commercially developed tar sand deposit is

(a)in northern Brunei, Island of Borneo

(b)in north-western Colorado, USA

(c)in southern Iraq, near Basra

(d)near Lake Athabasca in central Canada


160.The yield of cotton per hectare is the highest in

(a)CIS.

(b)USA.

(c)Egypt.

(d)India.

[UPSC-1996]

161.Vein - or fissure-filling deposits of lead and zinc minerals would be


more common in

(a)rhyolitic lava flowing interbedded with pyroclastics

(b)rocks formed in a deep, high-grade regional metamorphic zone

(c)impactites and shattered rock of a meteorite impact zone

(d)rocks of a contact metamorphic zone formed around a shallow, granite


pluton

162.Weber introduced a famous locational theory for localization of


manufacturing industries known as

(a)hexagon.

(b)circle.

(c)triangle.

(d)rectangle.
[UPSC-2003]

163.Weber's model of industrial location has been usefully applied to


interpret the changing location of the British.

(a)Iron and steel industry

(b)Cotton textile industry

(c)Ship building industry

(d)Heavy engineering industry

[UPSC-1996]

164.Which among the following has the world's largest reserves of


uranium'?

(a)Australia

(b)Canada

(c)Russian Federation

(d)USA

[UPSC-2009]

165.Which country among the following has the highest proportion of its
electricity generation by nuclear plants'?

(a)USA

(b)UK
(c)France

(d)Germany

[UPSC-2005]

166.Which of the following pairs are correctly matched'?

1.Nomadic herding: Afro-Asian and lands

2.Livestock ranching: Amazon Basin

3.Mixed farming: Eastern USA

Select the correct answer using the codes given.

(a)1, 2 and 3

(b)1 and 3

(c)1 and 2

(d)2 and 3

[UPSC-1999]

167.Which of the following pairs are correctly matched'?

1.Ekibartuz: Coal

2.Kursk: Copper

3.Tyumen: Oil and natural gas

4.Zaporozhye: Aluminium
Select the correct answer using the codes given.

(a)1, 2, 3 and 4

(b)1, 3 and 4

(c)2 and 4

(d)1, 2 and 3

[UPSC-1999]

168.Which of the following statements are correct'?

1.Lignite and low-grade bituminous coal were formed during the


quaternary period.

2.Dolomite is used as refractory material and flux in iron and steel


industry.

3.Chhota Nagpur plateau is the most extensive, rich and diversified


mineral belt.

Select the correct answer using the codes given.

(a)1, 2 and 3

(b)1 and 2

(c)2 and 3

(d)1 and 3

[UPSC-1999]
169.Which of the following statements regarding cotton cultivation in China
are correct'?

1.A growing season of about 200 days free from frost.

2.Intermittent rainfall during harvesting period benefits higher yields.

3.Rainfall of 500 to 1,500 mm during growing season is a common


feature in the east.

4.Mechanized farming and heavy irrigation are common features in the


east.

Select the correct answer using the codes given.

(a)3 and 4

(b)2, 3 and 4

(c)1 and 2

(d)1 and 3

[UPSC-1996]

170.Which of the following techniques is useful for predicting the wheat


yield?

(a)Regression

(b)Chi-square

(c)Simulation

(d)Correlation
[UPSC-2004]

171.Which one amongst the following has the largest livestock population in
the world'?

(a)Brazil

(b)China

(c)India

(d)USA

[UPSC-2008]

172.Which one of the following agricultural practices is eco-friendly?

(a)Organic farming

(b)Shifting cultivation

(c)Cultivation of high yielding varieties

(d)Growing plants in glasshouses

[UPSC-1999]

173.Which one of the following areas is dominant in hunting and gathering?

(a)Hot desert

(b)Cold desert

(c)Tropical forest

(d)Temperate grassland
[UPSC-2004]

174.Which one of the following best describes how talc deposits might
originate geologically?

(a)Dehydration of gypsum and limestone during contact metamorphism

(b)Reaction of olivine and pyroxene with hydrothermal fluids during


regional metamorphism

(c)High-pressure metamorphism associated with impact of an asteroid or


large meteorite

(d)Reaction of hydrothermal fluids with quartz-rich sandstones during


contact metamorphism

175.Which one of the following countries has the highest percentage of


developed water power to the available potential?

(a)Canada

(b)France

(c)Japan

(d)Norway

[UPSC-1998]

176.Which one of the following countries is the largest producer of


electricity by wind power in the world'?

(a)Germany
(b)Australia

(c)Norway

(d)Switzerland

[UPSC-2005]

177.Which one of the following countries is the largest producer of fuel


wood in the world?

(a)Indonesia

(b)Russia

(c)India

(d)China

[UPSC-1997]

178.Which one of the following countries is the largest producer of marine


fish?

(a)China

(b)Russia

(c)Chile

(d)Japan

[UPSC-2003]
179.Which one of the following countries is the leading producer of
uranium?

(a)USA

(b)Canada

(c)Germany

(d)Zambia

[UPSC-2006]

180.Which one of the following countries occupies the first place in the
`Global Competitive Report' of World Economic Forum?

(a)USA

(b)Singapore

(c)Hong Kong

(d)France

[UPSC-2000]

181.Which one of the following geographical conditions makes hunting and


fishing the principal means of livelihood in the Tundra region?

(a)Sparse population

(b)Infertile dissected terrain

(c)Scarcity of vegetation
(d)Harsh climate

[UPSC-2000]

182.Which one of the following has influenced most the location of


industries in the world?

(a)Water

(b)Petroleum

(c)Coal

(d)Natural gas

[UPSC-1997]

183.Which one of the following is a false statement concerning the Bingham


Canyon mine?

(a)The ore contains, on average, more than 3 per cent by weight of


copper.

(b)Most of the mineralized rock is part of an igneous rock pluton.

(c)It is located in Utah near Salt Lake City.

(d)It is the largest open-pit copper mine in North America.

184.Which one of the following is a false statement concerning the Bingham


Canyon mine?

(a)The ore contains, on average, more than 3 per cent by weight of


copper.
(b)Most of the mineralized rock is part of an igneous rock pluton.

(c)It is located in Utah near Salt Lake City.

(d)It is the largest open-pit copper mine in North America.

185.Which one of the following is an important crop of the Barak valley?

(a)Jute

(b)Tea

(c)Sugarcane

(d)Cotton

[UPSC-1996]

186.Which one of the following is cultivated by transplanting seedlings?

(a)Maize

(b)Sorghum

(c)Onion

(d)Soya bean

[UPSC-1997]

187.Which one of the following is the most important basis of the


Whittlesey's classification of agriculture of the world?

(a)Functioning forms of agriculture


(b)Distribution of climatic elements

(c)Forms of livestock grazing activities

(d)Regional distribution of the principal crops and animals of commercial


significance

[UPSC-2003]

188.Which one of the following known as the `Coffee Port' of the world?

(a)Sao Paulo

(b)Santos

(c)Rio de Janeiro

(d)Buenos Aires

[UPSC-2000]

189.Which one of the following mining areas is significant for a different


mineral from that of the other three areas?

(a)Kuznetsk

(b)Fushun

(c)Karaganda

(d)Kirkuk

[UPSC-1998]

190.Which one of the following pairs is not correctly matched?


Type of agriculture: Urban structure

(a)Viticulture: France

(b)Sericulture: Poland

(c)Horticulture: Netherlands

(d)Pisciculture: Japan

[UPSC-2005]

191.Which one of the following pairs of forest resources are animal


products?

(a)Fibres and flosses

(b)Gums and resins

(c)Lac and wax

(d)Drugs and spices

[UPSC-2002]

192.Which one of the following port cities in Venezuela has been developed
as an oil port'?

(a)Caracas

(b)Maracaibo

(c)Maracay

(d)Carupano
[UPSC-1999]

193.Which one of the following regions is practicing most intensive


subsistence farming?

(a)Pampas Region

(b)Murray-Darling Basin

(c)California Valley

(d)Monsoon Asia

[UPSC-2003]

194.Which one of the following sectors of the economy is state-owned and


its service sector is a public enterprise?

(a)Irrigation

(b)Agriculture

(c)Forests

(d)Plantation

[UPSC-1995]

195.Which one of the following sets of condition is necessary for a good


cultivation of wheat?

(a)Moderate temperature and moderate rainfall

(b)High temperature and heavy rainfall


(c)High temperature and moderate rainfall

(d)Low temperature and low rainfall

[UPSC-1996]

196.Which one of the following types of coal contains a higher percentage


of carbon than the rest'?

(a)Bituminous coal

(b)Lignite

(c)Peat

(d)Anthracite

[UPSC-1999]

197.Who among the following divided the world into major agricultural
regions?

(a)Stamp

(b)Whittlesey

(c)Barker

(d)Weaver

[UPSC-1999]

198.World's leading producer of tobacco is

(a)China.
(b)USA.

(c)India.

(d)Russia.

[UPSC-1995]

199.The important fishing areas called Grand Banks and Georges Bank are
located in the waters of which of the following?

(a)North Pacific

(b)South Pacific

(c)South-west Atlantic

(d)North-west Atlantic

[UPSC-2008]

200.In which one of the following regions is the maximum amount of


world's phosphate produced?

(a)Northern Africa

(b)Northern Europe

(c)North America

(d)Northern Australia

[UPSC-2008]

201.Consider the following:


1.Central Africa

2.Borneo and Papua New Guinea

3.Amazon Basin

In which of the above areas is the primitive agriculture such as shifting


cultivation or rotational bush-fallow type of cultivation found?

(a)1 only

(b)1 and 2

(c)2 and 3

(d)1, 2 and 3

[UPSC-2008]

202.Consider the following statements:

1.Costa Rica is a leading exporter of cocoa.

2.Malaysia is a leading exporter of rubber.

Which of the statements given above is/ are correct?

(a)1 only

(b)2 only

(c)Both 1 and 2

(d)Neither 1 nor 2
[UPSC-2008]

203.Among the following countries, which one is a major producer of


wolfram (tungsten ore)?

(a)Australia

(b)China

(c)Indonesia

(d)Russia

[UPSC-2008]

204.What is the formula for calculating the cephalic index?

[UPSC-2008]

205.Among the following countries, which one is the most densely


populated?

(a)Belgium
(b)France

(c)Germany

(d)The Netherlands

[UPSC-2008]

206.Consider the following statements:

Nomadictribes: Region

1.San: Kalahari desert

2.Samoyed: Siberia

3.Wodaabe: North-eastern Asia

4.Yakut: Lena basin

Which of the statements given above is/ are correctly matched'?

(a)1 and 2

(b)2 and 4

(c)1, 2 and 4

(d)1, 3 and 4

[UPSC-2008]

207.In which one of the following countries do the ethnic communities


called Karen, Kachin and Chin live'?
(a)Australia

(b)Indonesia

(c)Myanmar

(d)Sri Lanka

[UPSC-2008]

208.Match List I with List II and select the correct answer from the codes
given.

Codes:

[UPSC-2008]

209.Among the following countries, which one has the higher total fertility
rate?
(a)India

(b)Pakistan

(c)Nepal

(d)Bangladesh

[UPSC-2008]

210.Among the following South Asian counties, in which one country is the
maternal mortality Ratio (per 1,00,000 live births) lowest?

(a)Bangladesh

(b)India

(c)Nepal

(d)Pakistan

[UPSC-2008]

211.Raven stein and Zelinsky are well known for their work in the context
of which one of the following models/theories?

(a)Locations theories

(b)Nearest neighbour analysis

(c)Simulation models

(d)Migration

[UPSC-2008]
212.With reference to the Central Place Theory, consider the following
statements:

1.Christaller dealt only with retailing functions in his theory.

2.Losch attempted to incorporate manufacturing into his model.

Which of the statements given above is/ are correct?

(a)1 only

(b)2 only

(c)Both 1 and 2

(d)Neither 1 nor 2

[UPSC-2008]

213.In which of the following countries is the per capita arable land highest?

(a)Germany

(b)Norway

(c)Sweden

(d)United Kingdom

[UPSC-2006]

214.Among the following Asian countries, whose share in the world exports
is the highest?

(a)India
(b)Malaysia

(c)Singapore

(d)Thailand

[UPSC-2006]

215.Consider the following statements:

1.Australia has the world's largest artesian basin.

2.Australia produces nearly one-third of the wool production of the


world.

3.Broken Hill has good reserves of diamond deposits.

Which of the statements given above is/ are correct'?

(a)1 and 2only

(b)2 and 3 only

(c)land 3 only

(d)1, 2 and 3

[UPSC-2006]

216.Where are mulberry trees grown mainly for silk production in China?

(a)Hwang Ho basin

(b)Si Kiang valley


(c)Manchurian plain

(d)Yangtze delta

[UPSC-2006]

217.Match List I with List II and select the correct answer from the codes
given.

Codes:

[UPSC-2006]

218.What does Caribou provide?

(a)Meat and skin

(b)Meat, milk and non-shrinkable wool


(c)Meat and fur

(d)Meat and blubber for fuel

[UPSC-2006]

219.Which one of the following deserts is famous for its deposits of


nitrates'?

(a)Atacama

(b)Kalahari

(c)Gobi

(d)Sahara

[UPSC-2007]

220.Consider the following pairs:

Produce: Region

1.Coal: Donabas

2.Iron ore :Magnitogorsk

3.Mica: Mama

Which of the statements given above is/ are correctly matched'?

(a)1 only

(b)2only
(c)2 and 3

(d)1, 2 and 3

[UPSC-2009]

221.The shifting cultivation that is leading to soil erosion in Brazil is known


as

(a)Milpa.

(b)Conuco.

(c)Tavy.

(d)Roca.

[UPSC-2009]

222.Which one of the following regions is the world's best cod fishing
ground'?

(a)North-east Atlantic

(b)South-east pacific

(c)West Central Pacific

(d)Indian Ocean

[UPSC-2009]

223.The Dogger Bank, the main fishing area of Europe, lies in

(a)Baltic Sea.
(b)English Channel.

(c)North Sea.

(d)Norwegian Sea.

[UPSC-2009]

224.Match List I with List II and select the correct answer from the codes
given.

Codes:

[UPSC-2009]

225.Factory farming is the characteristic of

(a)north-western North America.


(b)north-western Europe.

(c)eastern Europe.

(d)southeast Asia.

[UPSC-2009]

226.In which one of the following countries is the iron mining centre called
Skuthorpe located'?

(a)France

(b)Germany

(c)Sweden

(d)United Kingdom

[UPSC-2009]

227.Which the three countries were members of European Free Trade


Association (EFTA) when it was founded in 1960?

(a)Italy, Belgium, Poland

(b)Hungary, Finland, Spain

(c)Austria, Denmark, Norway

(d)The Netherlands, France, Romania

[UPSC-2009]

228.Agriculture probably originated in


(a)the Middle East.

(b)Nigeria.

(c)Ethiopia.

(d)South America.

229.Which of the following is NOT a major centre of plant and/or animal


domestication'?

(a)The Middle East

(b)Meso-America

(c)Southeast Asia

(d)Eastern North America

230.Maize (corn) was domesticated in

(a)Mexico.

(b)Australia.

(c)Africa.

(d)Asia.

231.The Green Revolution

(a)allowed farmers in the Midwestern United States to feed most of the


world.

(b)is an attempt to preserve traditional seed varieties.


(c)allowed civilizations to develop in the ancient Middle East.

(d)concentrated on higher yielding varieties of crops.

232.Genetically modified (GM) plants are produced by

(a)using large quantities of fertilizers.

(b)applying pesticides.

(c)mass production in a factory.

(d)joining the DNA of two organisms to produce an organism with


recombined DNA.

233.Which of the following is a characteristic of commercial agriculture'?

(a)Technology is low.

(b)A great deal of human labour goes into farming.

(c)The farmer depends on his/her own production for most food.

(d)Most of the production is sold to companies that process the food and
sell it to consumers.

234.Quinoa and amaranth are

(a)traditional South American grains, dating back to the Inca, that are
now being grown in new places and enjoying renewed popularity.

(b)newly developed kinds of fertilizer.

(c)sources of illegal drugs.


(d)African names for millet and sorghum.

235.The Cartagena Protocol (1992) is an international agreement dealing


with

(a)trade in genetically modified (GM) food products.

(b)pollution from fertilizer runoff.

(c)the illegal drug trade.

(d)global warming from methane produced by cattle.

236.Since Thomas Malthus' theory was first published, all of the following
have happened except

(a)disasters such as wars and epidemics have killed tens of millions of


people.

(b)many food crops have been transplanted to new areas where they have
thrived.

(c)vast areas of the planet that were scarcely utilized during Malthus'
lifetime have been opened to productive agriculture.

(d)mass starvation has occurred throughout the world, placing a strong


control on population growth.

237.Which of the following does NOT improve local food supplies'?

(a)Transportation

(b)Spread of disease
(c)Trade

(d)Storage

238.Which is NOT one of the world's most basic foods'?

(a)Potatoes

(b)Cattle

(c)Wheat

(d)Rice

239.Earth's non-agricultural land is usually characterized by one or more of


the following except

(a)too wet.

(b)too hot.

(c)too dry.

(d)too cold.

240.Which is a form of subsistence agriculture?

(a)Mixed farming with livestock

(b)Shifting cultivation

(c)Prairie cereal farming

(d)Plantation farming
241.Nomadic herding is least likely to occur in

(a)the Middle East.

(b)North Africa.

(c)central America.

(d)central Asia.

242.Which of the following is an animal NOT characteristically raised by


nomadic herders'?

(a)Sheep

(b)Pigs

(c)Horses

(d)Cattle

243.Shifting cultivation

(a)makes extensive use of machinery.

(b)is characterized by monoculture.

(c)is most commonly found in high latitude regions.

(d)can support low population densities.

244.Shifting cultivation is being replaced by all of the following except

(a)commercial logging.
(b)industry.

(c)cattle ranching.

(d)cultivation of cash crops.

245.Wetrice cultivation needs to take place on flat land because

(a)major highways are the main means to get the crop to market.

(b)the fields are flooded for major part of the year.

(c)hillside fields would erode.

(d)large-scale machinery is used.

246.About what proportion of the world's population depends wholly or in


part on wet rice'?

(a)Two-thirds

(b)Half

(c)One-quarter

(d)One-third

247.In areas where climate makes growing wet rice difficult, the major grain
is

(a)corn.

(b)wheat.

(c)oats.
(d)sorghum.

248.Of the following crops, which has both the shortest growing season and
the ability to grow farther north than the others'?

(a)Barley

(b)Maize

(c)Wheat

(d)Rice

249.Which of the following is least likely to be produced in Mediterranean


agriculture?

(a)Butter

(b)Grapes

(c)Fruits

(d)Olives

250.Which of the following is not a typical practice in growing rice in Asia?

(a)Flooding the ploughed land with rain water

(b)Harvesting with a plough drawn by oxen

(c)Transplanting seedlings grown in a nursery

(d)Flattening the land

251.The first domesticated animals were probably


(a)cattle.

(b)dogs.

(c)horses.

(d)sheep.

252.The number of domesticated livestock in 2011 was over, more than


times the human population.

(a)3 billion; 0.5

(b)10 billion; 1.5

(c)27 billion; 3

(d)15 billion; 2

253.Over one-third of the world's total grain harvest is fed to

(a)developed countries.

(b)poor countries.

(c)livestock.

(d)people.

254.Of all the domesticated animals, the isf the most efficient transormer of
grain into edible meat.

(a)turkey

(b)pig
(c)sheep

(d)chicken

255.Livestock, especially cattle, contribute to through the production of

(a)overpopulation; meat

(b)greenhouse gasses; methane

(c)religious significance; milk

(d)environmental devastation; milk

256.In general, the farther a dairy farm is from a large urban area, the lower
the percentage of output devoted to fresh milk. This occurs primarily
because

(a)transport costs are greater farther from the urban area.

(b)milk is more perishable than butter.

(c)the quality of soil is lower near an urban area.

(d)land costs are lower farther from the urban area.

257.Worldwide, about two-thirds of aquatic foods are captured in the wild;


the rest are the result of

(a)artisanal fishing.

(b)aquaculture.

(c)marine capture production.


(d)freshwater capture production.

258.Traditional fishing is most likely to occur in

(a)North America.

(b)Oceania.

(c)South America.

(d)Europe.

259.Traditional fishing provides low incomes, but these activities employ


aboutof the world's fishermen.

(a)60

(b)80

(c)90

(d)30

260.Almost one-fourth of the global fish catch now goes into meal and oil,
mostly to in wealthy countries.

(a)feed livestock and pets

(b)feed school children

(c)feed people

(d)provide vitamin supplements

261.`Depleting the Common' is a process where


(a)international funding for fishing operations declines.

(b)the most desired fish are over harvested, requiring fishermen to gather
less desirable fish, which are worth less money.

(c)over fishing by several countries creates a rapid depletion of the


fisheries.

(d)local fishermen are pushed out of their fishing grounds by commercial


fishermen.

262.Marine animals rely on coastal wetlands, mangrove swamps and rivers


for spawning grounds,

(a)but human intervention has provided artificial spawning grounds.

(b)but these only constitute a small percentage of marine animals.

(c)but in recent years, these animals have found other spawning grounds.

(d)but these are being destroyed by pollution and overdevelopment.

263.Depletion of atmospheric ozone has increased ultraviolet radiation


which is responsible for a decrease in plankton.

(a)10 per cent

(b)20 per cent

(c)30 per cent

(d)40 per cent

264.About 90 per cent of the world's marine fish harvest takes place within
(a)coastal countries.

(b)the international fishing waters.

(c)the exclusive economic zone.

(d)12 miles of shore.

265.The process of herding and domesticating aquatic animals is known as

(a)processing.

(b)aquaculture.

(c)agriculture.

(d)artisanal fishing.

266.Which of the following is not a cause for hunger?

(a)Nutrient surplus

(b)Vitamin deficiency

(c)Calorie deficiency

(d)Protein deficiency

267.A person is considered malnourished when he/she consumes

(a)too few nutrients.

(b)the ideal number of nutrients.

(c)too few or too many nutrients.


(d)too many nutrients.

268.The world today produces pounds of food per person per day.

(a)4.3

(b)5.6

(c)1.2

(d)2.7

269.Ejidos means

(a)a form of land tenure in which a peasant community collectively owns


a piece of land.

(b)Mexican cowboys.

(c)the slums where migrants to Mexican cities live.

(d)land in Mexico that can be used for collateral in obtaining a loan for
agriculture.

270.Food supply problems are often a result of problems with

(a)economics or politics.

(b)climate.

(c)technology.

(d)geography.
271.Many governments enact policies that affect what farmers can earn and
keep them at the subsistence level. These policies include all of the
following EXCEPT

(a)reduce the cost to purchase equipment.

(b)price controls.

(c)export and import taxes.

(d)exchange rate manipulation.

272.The redistribution of large land holdings such as plantations to poor


would-be farmers is known as

(a)land segmentation.

(b)land rededication.

(c)land reform.

(d)land redistribution.

273.Poor countries do not have the opportunity to sell food products to rich
countries because

(a)poor countries would not know how to use the foreign exchange they
received in such an enterprise.

(b)the rich countries have erected tariff walls to close their markets of
food imports from poor countries.

(c)the types of food grown in poor countries are alien to the tastes found
in rich countries.
(d)the quality of their produce does not meet the standards set by rich
countries.

274.Sustainable agriculture is food production that can be continued


indefinitely and that

(a)supports subsistence agricultural initiatives.

(b)increases food production profits.

(c)limits or even reverses environmental degradation.

(d)creates demand for cash crops.

275.In comparison to the trend in the USA, average farm size in many areas
where intensive subsistence is practiced (Turkey, Bangladesh, China) is

(a)fluctuating in response to global crop prices.

(b)remaining constant.

(c)increasing.

(d)decreasing.

276.Which of the following is a plantation crop'?

(a)Jatropha

(b)Flowers

(c)Coffee

(d)All of the above can be plantation crops.


277.Commercial agricultural systems in the core are especially efficient in
terms of food production per unit of

(a)area.

(b)fuel energy.

(c)labour.

(d)water.

278.Plantations - from coffee, cocoa and tea to rubber, palm oil and bananas
- typically practice

(a)multiculture.

(b)silviculture.

(c)polyculture.

(d)monoculture.

279.Which of the following is NOT one of the major changes occurring


globally in agriculture?

(a)Increase in the use of machinery

(b)Increase in the use of chemicals

(c)Increase in the number of people employed in agriculture

(d)Increased integration into wider economic and industrial systems

280.An agrarian culture is one (in) which


(a)benefited from the Green Revolution.

(b)agricultural production is predominant in daily life.

(c)has gone through first, second and third agricultural revolutions.

(d)agricultural production results in plentiful food exports.

281.Subsistence agriculture is characterized by agriculturalists who

(a)reject globalization and monoculture.

(b)practice transhumance.

(c)consume what they produce.

(d)farm on swiddens.

282.Shifting cultivation is characterized by the practice of rotating to


maintain soil fertility.

(a)ownership

(b)crops

(c)fields

(d)chemicals

283.The benefits of intertillage include all except

(a)spreading out food production over the farming season.

(b)soil moisture retention and maximization.


(c)increased ability to replace people with equipment like tractors.

(d)reduced risk of crop failure.

284.Characteristic of intensive subsistence agriculture is the great amount of


required for attaining high yields.

(a)labour

(b)fertilizers

(c)land

(d)Both (a) and (b)

285.Compared to large, industrial farms, traditional intensive subsistence


farms tend to

(a)rely more on sophisticated forms of mechanization.

(b)be a more sustainable form of production.

(c)have low yields.

(d)lack agricultural and environmental understanding.

286.Pastoral nomads

(a)wander around harsh environments hoping to find water and forage for
their animals.

(b)depend on animals rather than crops for their livelihood.

(c)are found only in the Middle East and North Africa.


(d)are increasing in number as the nomadic lifestyle appeals to people
tired of the city.

287.The dramatic increase in yields associated with the Second Agricultural


Revolution coincided with

(a)the large influx of migrants from peripheral to core countries.

(b)the arrival of crops (potatoes, tomatoes, peppers, chocolate) from the


New World.

(c)the introduction of hybrid seeds and GMOs (genetically modified


organisms).

(d)a commercial market for food created by the growing urban labour
force.

288.In contrast to the first two agricultural revolutions, the third one
originated in the

(a)core.

(b)periphery.

(c)Old World.

(d)New World.

289.Which of the following is NOT a characteristic of the Third Agricultural


Revolution?

(a)Turned agricultural outputs into secondary sector inputs

(b)Ended with the arrival of the Green Revolution


(c)Occurred in the 20th century and continues through today

(d)Mechanized inputs - replacement of human farm labour with machines

290.The industrialization of agriculture has made the farm

(a)the centre of the service sector of the economy.

(b)the centre of agricultural production.

(c)the cutting edge of new technology.

(d)a component in the agro-commodity production system.

291.The industrialization of agriculture involves all of the following except

(a)the globalization of the food chain.

(b)the replacement of chemically oriented agriculture with organic


production.

(c)technological changes in inputs that affect the biological outputs.

(d)the replacement of agricultural products with manufactured (synthetic/


artificial) products.

292.The Green Revolution innovations in agriculture based on a package of


inputs including all of the following except

(a)chemical pesticides.

(b)high yielding seed varieties.

(c)synthetic fertilizers.
(d)natural fertilizers (compost, green manure, animal manure).

293.The primary worldwide benefit of the Green Revolution has been an


increase in

(a)land reform.

(b)food production.

(c)conflicts between the primary and secondary sectors.

(d)environmental benefits.

294.For the most part, at least in its first decades, the Green Revolution
targeted for improvements.

(a)agricultural production in Africa

(b)non-traditional agricultural exports (NTAEs) like fruit, flowers and


vegetables

(c)grains like rice, corn and wheat

(d)export luxury crops like coffee, tea, and cocoa

295.In the context of agribusiness and the globalization of agriculture, the


food chain is a way of understanding complex connections between

(a)monoculture and polyculture.

(b)agricultural producers and food consumers.

(c)governments and transnational food corporations.


(d)foods in the major food groups.

296.A food regime is the specific set of links that exists among food
production, consumption, capital investment and accumulation
opportunities. Since the 1960s, global agriculture has changed from a
wheat and livestock food regime to a

(a)fish regime.

(b)poultry regime.

(c)fast food regime.

(d)fresh fruits and vegetables regime.

297.The great thing about biotechnology is that it

(a)has improved environments through reduced use of agricultural


chemicals.

(b)has saved the small farm.

(c)reduced hunger in the world's periphery.

(d)has done none of (a)-(c) above.

298.An organism that has had its DNA modified in a laboratory rather than
through cross-pollination or other forms of evolution is known as a

(a)DDT.

(b)GMO.

(c)CSA.
(d)HBO.

299.Because the health and environmental impacts of GMOS are not well-
understood,

(a)the USA allows GMOs to be used only in research.

(b)they are not widely used and were just another passing fad.

(c)Europe severely limits GMO food imports.

(d)farmers in the USA refuse to use them.

300.As a whole, the core cause of global under nutrition is

(a)food security.

(b)famine.

(c)the green revolution.

(d)poverty.

301.For a person, household or country to be food secure means that

(a)the person, household or country is self-reliant.

(b)enough food is available.

(c)the food supply is safe and diseaseand bacteria-free.

(d)access to enough food is assured.

302.In metropolitan areas around the world, is/are critical in supporting food
security for around 800 million.
(a)NTAEs

(b)CSAs

(c)U.S. food aid

(d)urban agriculture

303.The biorevolution is expected to lead to more

(a)widely accessible seed varieties.

(b)environmental protection.

(c)agricultural jobs.

(d)monoculture.

304.From the following, the most characteristic product of the Blue


Revolution is

(a)berries.

(b)poultry.

(c)dairy products.

(d)fish.

305.Agriculture is a(n)

(a)art.

(b)business.
(c)science.

(d)all of these

306.Which of the following is in proper historical sequence'?

(a)Hunting and gathering, subsistence agriculture, commercial agriculture

(b)Commercial agriculture, subsistence agriculture, hunting and gathering

(c)Hunting and gathering, commercial agriculture, subsistence agriculture

(d)Subsistence agriculture, commercial agriculture, hunting and gathering

307.Which of the following has dominated food-generating practices in the


20th century'?

(a)Swidden agriculture

(b)Subsistence agriculture

(c)Commercial agriculture

(d)Pastoralism

308.Subsistence agriculture

(a)is designed to produce a surplus for sale.

(b)produces food for the direct consumption of the growers and their
families.

(c)does not include the raising of livestock.

(d)has grown in importance over the course of the 20thcentury.


309.Shifting cultivation is most commonly practiced in

(a)core countries.

(b)southern Africa and Australia.

(c)Siberia.

(d)tropical and subtropical regions.

310.Subsistence agriculture is most common in

(a)the Middle East.

(b)Europe.

(c)Australia.

(d)Asia.

311.Shifting cultivation

(a)requires more expenditure of energy than modem farming methods.

(b)works best with low population densities.

(c)has changed technology and practice dramatically over the last 3


centuries.

(d)is experiencing a revival among organic agriculturalists.

312.In shifting cultivation, fields are generally worked for about

(a)2 years.
(b)1 year.

(c)10-15 years.

(d)5-7 years.

313.Practitioners of shifting cultivation usually begin the process of clearing


a new field when

(a)they are certain the last frost is past.

(b)the soil of the old field is depleted of nutrients.

(c)the government tells them to do so.

(d)seasonal and annual rainfall patterns shift.

314.Intensive subsistence agriculture

(a)involves more human labour than swidden agriculture.

(b)is most commonly practiced in areas with little rainfall.

(c)is used primarily in parts of the world with small populations.

(d)usually achieves only low productivity.

315.dominate subsistence agriculture.

(a)Potatoes, yams and other tubers

(b)Lettuce and other leafy vegetables

(c)Rice and other grains


(d)Various varieties of squash

316.Pastoralism

(a)has largely replaced intensive subsistence agriculture in Southeast


Asia.

(b)is increasing in importance in most parts of the world.

(c)requires elaborate fencing networks and patterns to manage herds.

(d)is most commonly practiced in drier climates unsuitable for


subsistence agriculture.

317.Which agricultural revolution is characterized by the emergence of a


significant surplus of production that could be sold and the beginnings of
the industrialization of agriculture'?

(a)First

(b)Second

(c)Third

(d)Fourth

318.In which agricultural revolution was the family farm considered `ideal'
for making a living and living a good life'?

(a)First

(b)Second

(c)Third
(d)It was considered ideal in all three revolutions.

319.The height of the second agricultural revolution coincided with

(a)the Renaissance.

(b)the Industrial Revolution.

(c)the use of non-organic fertilizers and synthetic herbicides and


pesticides.

(d)the Reformation.

320.In the middle of the l8thcentury in Europe, peasant agriculture was


characterized by

(a)the first use of the tractor.

(b)communal farming practices borrowed from Russia.

(c)crop rotation that increased yields and soil fertility.

(d)an ever-tightening grip by feudal landlords on the peasants.

321.When did the third agricultural revolution occur'?

(a)About 2,000 years ago

(b)About 1,000 years ago

(c)In the 18th century

(d)In the 20th century


322.Which of the following does NOT characterize one of the three phases
of the third agricultural revolution'?

(a)Chemical farming

(b)Food manufacturing

(c)Mechanization

(d)Significant surplus production

323.Which phase of the third agricultural revolution affected agricultural


outputs rather than inputs'?

(a)Mechanization

(b)Surplus production

(c)Food manufacturing

(d)Chemical farming

324.Which of the following is NOT one of the three important developments


in the industrialization of agriculture'?

(a)Increased focus on the family farm as the centrepiece of agricultural


production

(b)The introduction of agrochemicals, biotechnologies, fertilizers and


hybrid seeds

(c)The development of industrial substitutes for agricultural products

(d)Changes in rural labour activities


325.Which of the following has NOT had an impact on commercialized
agriculture?

(a)Fluctuations in the value of international currency

(b)Changes in the price of oil

(c)Inflation

(d)All of the above have affected and continue to affect commercialized


agriculture.

326.The Green Revolution

(a)has been most successful in Africa.

(b)has been an unqualified success.

(c)began with the peace and environmental movement of the 1960s.

(d)None of the above is true.

327.The initial goal of Green Revolution scientists was to develop

(a)higher yielding seeds.

(b)more effective fertilizers.

(c)training manuals in local languages of the periphery.

(d)more efficient irrigation systems.

328.Which region or country has benefited the least from the Green
Revolution?
(a)Africa

(b)Mexico

(c)East Asia

(d)India

329.During recent years, Green Revolution scientists have

(a)entered the political arena to seek more hands-on control of agriculture


in developing countries.

(b)produced seeds for a widening variety of crops.

(c)engaged in little activity since their funding has dwindled to nothing.

(d)focused attention on developing colourful and snazzy looking strains


of wheat and corn.

330.Agricultural subsidies

(a)provide family farmers the opportunity earn living wages from their
farms.

(b)distort the true market value of the agricultural products being


subsidized.

(c)are no longer used by the U.S. government.

(d)are often achieved without spending money.

331.From 19th-century Europe to the 21stcentury Tunisia, the high cost of


food has often lead toriots.
(a)coffee

(b)bread

(c)cheese

(d)meat

332.In core economies, which of the following dominates the food


production process?

(a)Transnational corporations

(b)Government officials

(c)Consumers

(d)Agricultural workers

333.Which of the following is NOT one of the five central and connected
sectors of the food supply chain but, rather, a mediating influence?

(a)The physical environment

(b)Food consumption

(c)Food distribution

(d)Agricultural inputs

334.Rachel Carson's Silent Spring

(a)focused on the negative effects of chemical fertilizers.

(b)led to the banning of DDT and other harmful chemicals in the USA.
(c)did major damage to the environmental movement in the USA.

(d)led to international rules prohibiting the use of harmful pesticides in


developing countries.

335.In a dry climate with cold winters, this grain would least likely be
cultivated. (a) Rice

(b)Wheat

(c)Oats

(d)Barley

336.Transhumance is least common in

(a)Greece.

(b)North Africa.

(c)Turkey.

(d)Denmark.

337.Synthetic fertilizer use is least common in

(a)Africa.

(b)the USA.

(c)Asia.

(d)Europe.

338.The `postmodern diet' is centred on


(a)free-range chicken and hormone-free beef.

(b)homeopathic herbs.

(c)fresh fruits and vegetables.

(d)wheat and livestock.

339.This grain was NOT a major focus of the Green Revolution:.

(a)Millet

(b)Maize

(c)Rice

(d)Wheat

340.Plantations, the Green Revolution and the agricultural production of


biofuels all promote

(a)local foods.

(b)food security.

(c)intensive subsistence.

(d)monoculture.

341.Which of the following is not a factor behind the last decade's rise in
global food prices'?

(a)Market speculation

(b)Government purchases of commodities for food aid


(c)Rising price of energy

(d)Rising biofuel production

342.The more one eats from the local food shed, the more one is likely to be
a(n)

(a)carnivore.

(b)locavore.

(c)vegetarian.

(d)herbivore.

343.Richard and Maurice McDonald

(a)mastered the concept of franchising.

(b)popularized the hamburger withcheese.

(c)invented the assembly-line style of industrial food preparation.

(d)invented a new way to eat cheaplythe Frenchfries.

344.Alternative food movement that directly connects farmers with


consumers originated in Japan and is spreading throughout the core

(a)WTO.

(b)NTAE.

(c)BMI.

(d)CSA.
345.Though they are on the margins of the global agro-food system, organic
farming practices are on the rise in the core. In growing crops and raising
livestock organically, which of the following are permitted?

(a)Composted cow manure

(b)Commercial fertilizers

(c)Synthetic pesticides

(d)Growth hormones

346.In contrast to conventional agro-food systems, alternative food networks

(a)have cheap food as the primary goal.

(b)emphasize monoculture.

(c)encourage the use of the latest chemical fertilizers and genetically


modified seeds.

(d)attempt to remove the uncertainties and risks of nature from food


production.

347.International organizations and nongovernmental organizations argue


that the key to food security and reduced risk from famines and chronic
hunger can be attained only through attention to

(a)alternative food movements.

(b)food sovereignty.

(c)increased production.
(d)GMOs and improved technologies.

348.In the USA,

(a)all foods with genetically engineered ingredients are required to be


clearly labelled.

(b)only genetically engineered food stuffs from Europe and Japan are
allowed to be imported.

(c)GMOs are widely permitted because they have not been proven to be
unsafe.

(d)GMOs are required to undergo rigorous and systematic testing.

349.According to our text, rainforest destruction, biodiversity, topsoil


conservation, water quality, water quantity and global climate are
associated with

(a)bipharming.

(b)industrial beef production.

(c)urban agriculture.

(d)pastoral nomadism.

350.Around the world, runoff from heavy use of synthetic fertilizers has
created

(a)biochemical farming zones.

(b)coastal dead zones.


(c)harvestable deposits of reusable fertilizers.

(d)new deltas of arable lands.

351.Which one of the following is a renewable resource?

(a)Oil

(b)Land

(c)Coal

(d)Iron

[UGC D-2005]

352.Tea plantations have been developed in the

(a)equatorial regions.

(b)tropical deciduous regions.

(c)tropical grassland regions.

(d)subtropical grassland regions.

[UGC D-2005]

353.Sal and teak are commonly found in

(a)equatorial forests.

(b)monsoon forests.

(c)mangrove forests.
(d)savannah.

[UGC D-2005]

354.The major exporter of dates in the world is

(a)Iran.

(b)Iraq.

(c)Turkey.

(d)Afghanistan.

[UGC D-2005]

355.Which one of the following is a footloose industry'?

(a)Cotton textile

(b)Sugar

(c)Cement

(d)Handloom

[UGC D-2006]

356.Which one of the following is not associated with a transport system?

(a)Von Thunen Theory

(b)Distance Decay Function

(c)Principle of Least Effort


(d)Central Place Theory

[UGC D-2006]

357.North Atlantic Oceanic route is the world's most important route


because

(a)there is Labrador coldwater current.

(b)there are developed nations on both sides.

(c)there is USA on the west coast.

(d)it is a safe navigation route.

[UGC D-2007]

358.Which one of the following is not related to plantation agriculture?

(a)Tea cultivation

(b)Coffee cultivation

(c)Rubber latex production

(d)Soya bean cultivation

[UGC D-2007]

359.Given below are two statements, one labelled as Assertion (A) and the
other labelled as Reason (R). Select your answer from the codes given.

Assertion (A): Weber's theory hinges on transportation costs but ignores


the fact that freight rates may not always increase in direct proportion to
the distance.
Reason (R): The freight rates, however, may not be similar for raw
material and finished goods.

(a)Both (A) and (R) are correct and (R) explains (A).

(b)Both (A) and (R) are correct but (R) does not explain (A).

(c)(A) is correct but (R) is wrong.

(d)(A) is wrong but (R) is correct.

[UGC D-2007]

360.The best-suited soil for cotton cultivation is

(a)laterite soil.

(b)regur soil.

(c)sandy soil.

(d)loamy soil.

[UGC D-2007]

361.Given below are two statements, one labelled as Assertion (A) and the
other labelled as Reason (R). Select your answer from the codes given.

Assertion (A): Virgin soils newly cleared off forests account for highest
yield of coffee.

Reason (R): Coffee is a soil-exhaustive crop.

(a)Both (A) and (R) are correct and (R) explains (A).
(b)Both (A) and (R) are correct but (R) does not explain (A).

(c)(A) is right but (R) is wrong.

(d)(A) is wrong but (R) is right.

[UGC D-2007]

362.Which one of the following is an example of footloose industry'?

(a)Paper industry

(b)Manufacturing of electronic components

(c)Cement industry

(d)Aluminium industry

[UGC D-2008]

363.Which sector of the economy is least necessary for sustainable


economic development'?

(a)Primary

(b)Secondary

(c)Tertiary

(d)Quaternary

[UGC D-2008]

364.Which one of the following is considered a non-renewable resource'?

(a)Water
(b)Animal

(c)Coal

(d)Forest

[UGC D-2008]

365.Who evolved the method of measurement of agricultural efficiency'?

(a)S.S. Bhatia

(b)R.L. Singh

(c)S.P. Chatterjee

(d)Enayat Ahmad

[UGC D-2008]

366.Which one of the following is an ideal combination for the


establishment of heavy industries'?

(a)Coal, iron, water

(b)Coal, electricity, market

(c)Coal, electricity, bank

(d)Coal, cotton, Bank

[UGC D-2008]

367.How many agricultural systems in the world were recognized by


Whittlesey?
(a)Five

(b)Six

(c)Nine

(d)Thirteen

[UGC D-2009]

368.Match List I with List II and select the correct answer from the codes
given.

Codes:

[UGC D-2009]

369.Match List I with List II and select the correct answer from the codes
given.
Codes:

[UGC D-2009]

370.Given below are two statements, one labelled as Assertion (A) and the
other labelledas Reason (R). Select your answer from the codes given.

Assertion (A): In Von Thunen's model of agricultural zones around a city,


each zone specializes in the production of those agricultural products to
which it is best suited.

Reason (R): Von Thunen did not conceive the advantages of combined
operation of specialised agricultural enterprises and the livestock.

(a)Both (A) and (R) are correct and (R) explains (A).

(b)Both (A) and (R) are correct but (R) does not explain (A).

(c)(A) is correct but (R) is wrong.

(d)(A) is wrong but (R) is right.


[UGC J-2005]

371.Weber's material index (MI) refers to the ratio between

(a)the cost of raw materials and cost of product.

(b)transport cost and cost of labour.

(c)cost of energy and cost of labour.

(d)the weight of raw material and weight of product.

[UGC J-2005]

372.Match List I with List II and select the correct answer from the codes
given.

Codes:

[UGC J-2005]
373.In analysis of transportation network, which of the following indices
shows the highest centrality in the lowest value'?

(a)Beta index

(b)Alpha index

(c)Pie index

(d)Diameter

[UGC J-2005]

374.In recent times, the highest importer of tea is

(a)UK.

(b)China.

(c)Pakistan.

(d)USA.

[UGC J-2005]

375.Coffee as a plantation crop is likely to be adversely affected by many


causes; of which the most important one is

(a)decline in international prices.

(b)attack by pests and diseases.

(c)tariff barriers for trade.

(d)development of substitutes.
[UGC J-2007]

376.Taaffe's transportation model has a number of development phases. The


number of phases are

(a)four.

(b)two.

(c)six.

(d)eight.

[UGC J-2007]

377.Mechanization of agriculture is best suited for countries with

(a)high population density.

(b)low population density.

(c)vast agricultural land.

(d)high standard of living.

[UGC J-2007]

378.Which one of the following crops enters international trade in large


quantities'?

(a)Rice

(b)Wheat

(c)Corn
(d)Barley

[UGC J-2007]

379.The book entitled Socio-Economic Models was edited by

(a)Spate and Learmonth.

(b)Herbert and Smith.

(c)Johnston and Herbert.

(d)Chorley and Haggett.

[UGC J-2007]

380.The leading green tea producing country in the world is

(a)China.

(b)Sri Lanka.

(c)Kenya.

(d)India.

[UGC J-2007]

381.Which one of the following transport models is suitable for rural


development in India?

(a)Truck, Railway, Shipment, Bullock cart

(b)Bullock cart, Truck, Railway, Ships

(c)Railway, Ship, Air transport, Bullock cart


(d)Bullock cart, Railway, Air transport, Ship

[UGC J-2008]

382.Who among the following was first to formulate the method of crop
combination?

(a)Nelson

(b)Doi

(c)Rafiullah

(d)Weaver

[UGC J-2008]

383.Given below are two statements, one labelled as Assertion (A) and the
other labelled as Reason (R). Select your answer from the codes given.

Assertion (A): Japan is a developed country.

Reason (R): It is necessarily critical to have natural resources within the


country for its development.

(a)Both (A) and (R) are true and (R) is the correct explanation of (A).

(b)Both (A) and (R) are true but (R) is not the correct explanation of (A).

(c)(A) is true and (R) is false.

(d)(R) is true but (A) is false.

[UGC J-2008]
384.Name the theory of industrialization proposed by Weber.

(a)Transport cost theory

(b)Resource-based theory

(c)Footloose industries theory

(d)Labour-based theory

[UGC J-2008]

385.Modern concept of agricultural diversification is related to

(a)large number of crops.

(b)value-added crops.

(c)monoculture crops.

(d)ecologically most suitable crops.

[UGC J-2008]

386.In which year did Von Thunen develop the model of agricultural land
use zonation?

(a)1817

(b)1826

(c)1850

(d)1847

[UGC J-2008]
387.Which one of the following crop is used for large-scale supply and
demand in international market'?

(a)Millets

(b)Rice

(c)Wheat

(d)Gram

[UGC J-2008]

388.Given below are two statements, one labelled as Assertion (A) and other
labelled as Reason (R). Select your answer from the codes given.

Assertion (A): Japan is a developed nation.

Reason (R): An ample supply and diversity of natural resources is a major


advantage to a country for its economic growth.

(a)Both (A) and (R) are true and (R) is the correct explanation of (A).

(b)Both (A) and (R) are true and (R) is not the correct explanation of (A).

(c)(A) is true, but (R) is false.

(d)(A) is false, but (R) is true.

[UGC J-2011]

389.Given below are two statements, one labelled as Assertion (A) and the
other labelled as Reason (R). Select your answer from the codes given.
Assertion (A): A footloose can be defined as an industry with material
index = 1.

Reason (R): A footloose industry can be located at any location without


any special economic advantage or disadvantage.

(a)Both (A) and (R) are true and (R) is correct explanation of (A).

(b)Both (A) and (R) are true and (R) is not the correct explanation of (A).

(c)(A) is true, but (R) is false.

(d)(A) is false, but (R) is true.

[UGC J-2011]

390.The transport model which suggests a sequential expansion of the


transport network was propounded by

(a)Alan Gilbert.

(b)Taaffe, Morrill and Gould.

(c)de Souza and Porter.

(d)Edward Soja.

[UGC J-2011]

391.Match List I with List II and select the correct answers from the codes
given.
Codes:

[UGC J-2011]

392.Which one of the following agricultural systems is described as Child of


Industrial Revolution'?

(a)Intensive subsistence tillage

(b)Livestock ranching

(c)Subsistence tillage

(d)Collective farming

[UGC J-2010]

393.Which one of them is a footloose industry'?


(a)Iron and steel industry

(b)Automobile industry

(c)Cement industry

(d)Cotton textile industry

[UGC J-2010]

394.For the establishment of cotton textile industry, which of the following


factors is more ideal'?

(a)Nearness to market

(b)Nearness to raw material

(c)Nearness to water resource

(d)Transport network

[UGC D-2011]

395.Who among the following was the first to develop the method of crop
combinations?

(a)Weber

(b)Wegener

(c)Weaver

(d)Wittfogel

[UGC D-2011]
396.Which sector of economy is the most important for economic
development?

(a)Primary

(b)Secondary

(c)Tertiary

(d)None of these

[UGC D-2011]

397.Given below are two statements, one labelled as Assertion (A) and the
other labelled as Reason (R). Select your answer from the codes given.

Assertion (A): Agricultural productivity refers to the sum total of land


productivity, labour productivity and capital productivity.

Reason (R): All the three components are factors of production.

(a)Both (A) and (R) are true and (R) is the correct explanation of (A).

(b)Both (A) and (R) are true and (R) is not the correct explanation of (A).

(c)(A) is true, but (R) is false.

(d)(A) is false, but (R) is true.

[UGC D-2011]

398.Which one of the following is a renewable resource?

(a)Coal
(b)Wind energy

(c)Iron ore

(d)Mica

[UGC D-2010]

399.`All resources are renewable on some time scale..... what matters for the
sustainability of future supplies is the relative rates of replenishment and
use.....' In view of the above statement, identify the correct order of the
resource continuum in descending order of renewability.

(a)Air, tidal power, bauxite, forest

(b)Air, bauxite, forest, tidal power

(c)Tidal power, air, bauxite, forest

(d)Forest, air, tidal power, bauxite

[UGC D-2010]

400.Match List I with List II and select the correct answer from the codes
given.
Codes:

[UGC D-2010]

ANSWER KEYS
 
1.Missionizing religions

(a)actively seek converts.

(b)do not seek converts.

(c)are identified with particular ethnic or tribal groups.

(d)are place based.

2.To human geographers, which of the following statements is NOT true


about culture'?

(a)It is based on our genetic inheritance.

(b)It is lived through the materials and symbolic practices of everyday


live.

(c)It can be transformed by forces internal and external to particular


groups.

(d)It is a shared set of meanings.

3.The fastest growing religion in the USA is

(a)Christianity.

(b)Hinduism.
(c)Islam.

(d)Animism.

4. is/are the heart and soul of the globalized Hip-Hop Nation.

(a)African-Americans

(b)Gangs

(c)The Bronx, New York

(d)Music

5.was an American pioneer in exploring and understanding the many ways


that humans transform the surface of the Earth to create cultural
landscapes.

(a)The Dalai Lama

(b)Notorious B.I.G.

(c)Carl Sauer

(d)Paul Vidal de la Blache

6.Human-built features over a land surface - such as small dairy farms over
the Wisconsin countryside or vineyards over California's Napa Valley -
are examples of

(a)cultural media.

(b)cultural agents.
(c)natural agents.

(d)cultural landscapes.

7.Carl Sauer, H.C.Darby and Paul Vidal de la Blache placed at the heart and
centre of their studies of humanenvironment interactions.

(a)language and religion

(b)natural landscapes

(c)cultural landscapes

(d)climatological factors

8.Early French geographers interested in cultural landscapes examined how


France's various genres de vie emerged from the constraints and
possibilities posed by local

(a)cultures.

(b)political and economic systems.

(c)cultural environments.

(d)physical environments.

9.Pork avoidance among Muslims, eating fish on Fridays among Roman


Catholics, bar mitzvahs among Jews and eating black-eyed peas on New
Year's Day in the USA South are all

(a)dietary restrictions.

(b)cultural traits.
(c)coming-of-age ceremonies.

(d)rites of passage.

10.Coming of age ceremonies - marking the transition from childhood into


adulthood - are examples of

(a)rites of passage.

(b)folk culture.

(c)cultural transitions.

(d)sexual identity.

11.A cultural region is an area where certain cultural practices, beliefs and
values

(a)are shared by the majority.

(b)share the same origins.

(c)are linked by the same religion.

(d)are linked by an official language or lingua franca.

12.A cultural system of traits, territorial affiliation and shared history

(a)does not allow for internal variation within the system.

(b)helps shape a group's collective identity.

(c)means that all group members speak the same language.

(d)means that all group members practices the same religion.


13.Diasporas have contributed especially to the spread of around the world.

(a)religion

(b)dialects

(c)disease

(d)trade

14.The world's four major world religions

(a)diffused from the New World following the Columbian Exchange.

(b)originated from Old World agricultural and urban hearth areas.

(c)began in the region of modern-day Israel/Palestine/Arabian Peninsula.

(d)emerged from the world's core regions.

15.Which of the following does NOT share religious and regional roots with
the others?

(a)Islam

(b)Hinduism

(c)Buddhism

(d)Sikhism

16.Despite having a long history and followers around the world, Judaism
does not have many followers. In large part, this is because Judaism
(a)recognizes Saturday as the holy day, which conflicts with Saturday as a
day of work, play and college football.

(b)like Feng Shui is just a passing fad.

(c)is an off-shoot of the much more popular Sikhism.

(d)does not seek converts.

17.In our postcolonial era, the flow of religious influence is increasing from

(a)core to core.

(b)periphery to periphery.

(c)periphery to core.

(d)core to periphery.

18.Before the Colombian Exchange and contact with the Old World, the
people of the USA practiced

(a)liberation theology.

(b)orthodox Christianity.

(c)Shamanism and animism.

(d)Shiism and Sunnism.

19.Candomble, Umbanda, Voudou and Santeria are religions of Latin


America that merge indigenous beliefs of Native Americans with

(a)Roman Catholicism and traditional African beliefs brought by slaves.


(b)Roman Catholicism brought by colonizers.

(c)Mormonism and Roman Catholicism

(d)Mormonism brought by missionaries.

20.The Dalai Lama travels the world supporting the people of Tibet and
promoting

(a)Catholicism.

(b)Hinduism.

(c)Islam.

(d)Buddhism.

21.Before attaining enlightenment and becoming the Buddha, Prince


Gautama was a(n)

(a)Hindu.

(b)animist.

(c)Muslim.

(d)Christian.

22.Swahili, Arabic, English, French and Hausa are all spoken by many
people across Africa. What they have in common is that they are all

(a)Indo-European languages.

(b)spoken by the bushmen of southern Africa.


(c)languages of trade and business in Africa.

(d)languages indigenous to Africa.

23.To be in the same family, languages must share the same

(a)origins.

(b)grammar.

(c)dialect.

(d)alphabet (or script).

24.The English spoken in Minnesota and the English spoken in Australia can
best be characterized as two different

(a)languages.

(b)dialects.

(c)language families.

(d)language groups.

25.The use of language distinctive of a particular group is known as a(n)

(a)colloquialism.

(b)diaspora.

(c)dialect.

(d)idiom.
26.The idea of an emerging, homogenous global culture is based on the
perception of a pervasive

(a)materialism.

(b)cultural nationalism.

(c)hybridization.

(d)Americanization.

27.Currently, around the world, there are roughly living languages.

(a)5

(b)5,000

(c)50

(d)500

28.The greatest concentration of UNESCO World Heritage sites is in

(a)Africa.

(b)Asia.

(c)Europe.

(d)North America.

29.Kinship is a form of social organization in which bonds between people


are based on

(a)adoption.
(b)marriage.

(c)blood relations.

(d)all of the above.

30.The effort to protect regional and national cultures from Americanization


and the homogenizing effects of globalization is known as

(a)traditional nationalism.

(b)religious fundamentalism.

(c)cultural nationalism.

(d)anti-Americanism.

31.The hajj, fasting during Ramadan and giving alms (charitable donations)
are

(a)among the five prayers of Islam.

(b)cultural traits of Arabs.

(c)cultural traits of Islamic countries.

(d)among the primary obligations of Muslims.

32.The most widespread and effective anticolonial, anti-imperialist and


generally anticore movement to challenge the modernizing and
secularizing forces of globalization has been

(a)postcolonialism.
(b)jihad.

(c)cultural nationalism.

(d)Islamism.

33.Islamism attempts to protect the purity and centrality of Islamic percepts


through

(a)secular democracy.

(b)hajj.

(c)the Five Pillars.

(d)a return to a universal Islamic state.

34.Muslim is to Islam as

(a)Hindu is to Hindi.

(b)Christian is to Christianity.

(c)English is to Latin.

(d)Turkish is to Arabic.

35.Which of the following spread around the world with the purpose of
making converts and saving souls (as well as for the purposes of political
control)?

(a)Christianity

(b) Islam
(c)Judaism

(d)Both (a) and (b)

36.Whereas race is a problematic way of differentiating human beings based


on physical characteristics such as skin colour, creates differences out of
cultural traits and complexes.

(a)racism

(b)ethnicity

(c)cultural nationalism

(d)class

37.A human geographer studying gender issues would most likely study

(a)pink spending.

(b)prostitution in and around red-light districts.

(c)the international sextrade.

(d)men's compared to women's salaries.

38.The British historical geographer H.C. Darby was best known for

(a)reconstructing past landscapes from the Domesday Book.

(b)writing the Domesday Book documenting England's dialects.

(c)starting the first geography department in England (at Oxford).

(d)interpreting the Old English dialect spoken by William, the Conqueror.


39.Hip-hop music is the coming together of all of the following except

(a)place.

(b)space.

(c)solace.

(d)race.

40.Which of the following has NOT been used to create social identities
and/or power differences among groups?

(a)Gender

(b)Ethnicity

(c)Racialization

(d)All of the above have been the basis for identity and power
differences.

41.The `shared set of meanings' of culture generally refers to

(a)ideas about gender roles.

(b)family values.

(c)religious practices.

(d)all of the above.

42.Cultural geography

(a)is no longer relevant because the world has become homogeneous.


(b)explores the interactions of culture with space, place and landscape.

(c)was denounced for its racist and upper-class bias by Carl Sauer.

(d)is concerned only with material culture.

43.According to Carl Sauer, a cultural landscape

(a)results from the action of a cultural group within and on a landscape.

(b)need not require a natural landscape.

(c)exists prior to human habitation of the given landscape.

(d)is an old-fashioned concept with limited use by contemporary


geographers.

44. According to Sauer's model, is (are) important in the creation of cultural


landscapes.

(a)cultural practices

(b)natural landscapes

(c)time

(d)All of the above are important in the process of creating cultural


landscapes.

45.Confirmation of 12-year-old boys and girls in the Roman Catholic


Church is best described as a

(a)bar mitzvah or bat mitzvah.


(b)cultural trait.

(c)rite of passage.

(d)cultural complex.

46.Diaspora

(a)refers solely to the emigration of people from the region of Palestine.

(b)occurred frequently during biblical times but have been rare since.

(c)has been the most common reason for conversion to Christianity.

(d)occurs when people of a homogeneous group are dispersed from their


traditional home region.

47.On world maps of religion, Central and South America are represented as
Roman Catholic and

(a)Muslim.

(b)Eastern Orthodox.

(c)animist.

(d)Buddhist.

48.Outside Israel, Judaism is concentrated in

(a)Europe and North America.

(b)South America and the Caribbean.

(c)West Africa.
(d)West Asia.

49.Buddhism originated in

(a)India.

(b)Thailand.

(c)Japan.

(d)Jordan.

50.Prior to European contact and colonization, the people of the US


practiced

(a)Buddhism and Taoism.

(b)Sikhism.

(c)Animism and Shamanism.

(d)Mormonism.

51.Which of the following terms is most likely to encompass the largest


number of languages?

(a)Language dialect

(b)Language group

(c)Language family

(d)Language branch

52.Spanish and French


(a)are mutually intelligible, i.e., if you speak one, you can understand the
other.

(b)are related to Hindi, Farsi, Greek and other Indo-European languages.

(c)emerged from Arabic.

(d)used to be spoken across all of Europe.

53.About 50 per cent of the global population speaks languages belonging to


the language family.

(a)Sino-Tibetan

(b)Indo-European

(c)Afro-Asiatic

(d)Caucasian

54.Which of the following countries or regions is least likely to engage in


cultural nationalism?

(a)USA

(b)Saudi Arabia

(c)Quebec

(d)France

55.Cultural nationalism
(a)has been on the wane over the last two decades as the world has turned
more towards democracy.

(b)is exemplified by France's efforts to protect its language from


'Americanization'.

(c)occurs when governments actively try to export their culture to other


countries with the specific aim of changing the cultures of those
countries.

(d)is practiced primarily in the Islamic world.

56.The Qur'an

(a)does not recognize Jesus as a prophet.

(b)is considered by Muslims to be the word of God, delivered through


Muhammad.

(c)is used by the Shi'a, but not by Sunnis.

(d)was written by Muhammad.

57.The number of television sets per capita is highest in

(a)South America.

(b)Southeast Asia.

(c)Africa.

(d)North America.

58.Islamism
(a)advocates the integration of Western values into Islamic society.

(b)opposes all forms of cultural nationalism.

(c)is concerned about individualism and the corrupting influence of the


core.

(d)attempts to maintain a strict Islamic fundamentalism.

59.The cultural hearth of Islam is in

(a)China, East Asia.

(b)Indonesia and Malaysia, Southeast Asia.

(c)the Balkans, in Europe.

(d)Saudi Arabia, in the Middle East.

60.Which of the following countries is NOT more than 50 per cent Muslim?

(a)Iran

(b)Iraq

(c)Egypt

(d)India

61.For the geographer, ethnicity is about a particular group

(a)and how it uses space to indicate its place in the world.

(b)with perceived or actual commonality in language or religion.


(c)that has a socially created system of rules governing who is and is not
a member of the group.

(d)All of the above.

62.Gender is something people

(a)consume.

(b)are.

(c)inhabit.

(d)do.

63.From a biological standpoint, race does not exist. Practically, however,


the term has come to refer to

(a)the places of certain ethnic groups, such as the Chinese in a


`Chinatown'.

(b)differences between people as determined by intelligence tests.

(c)differences in cultures and practice.

(d)differences between people based on skin colour and other physical


characteristics.

64.Power differences between men and women in a society are determined


primarily by

(a)physical geography.

(b)society and culture.


(c)biological differences.

(d)performance in standardized testing.

65.This is NOT a major forefather of hip-hop.

(a)Nat King Cole

(b)Bob Marley

(c)Huey Newton

(d)James Brown

66.In the USA, hip-hop is least associated with

(a)Oakland.

(b)Washington, D.C.

(c)Los Angeles.

(d)New York City.

67.The leadership of hip-hop is predominantly

(a)Latino and male.

(b)black and male.

(c)black and female.

(d)white and male.

68.Hinduism originated in modem-day


(a)Israel.

(b)China.

(c)Iran.

(d)India.

69.Of the following religions, which is oldest?

(a)Buddhism

(b)Islam

(c)Sikhism

(d)Hinduism

70.Protestantism is least prevalent in

(a)the USA.

(b)South Africa.

(c)northern Europe.

(d)South America.

71.This religion does NOT have its origins in the Middle East.

(a)Judaism

(b)Islam

(c)Sikhism
(d)Christianity

72.Of the following religions, which was founded most recently'?

(a)Hinduism

(b)Daoism

(c)Christianity

(d)Islam

73.In the USA, the rise of televangelism led to

(a)out-migration from the Bible Belt.

(b)social justice awareness.

(c)a rise in Christian fundamentalism.

(d)increased interest in Eastern religions.

74.In what country is Shi'ism the official state religion'?

(a)Egypt

(b)Bangladesh

(c)Malaysia

(d)Iran

75.Muslims do NOT make up a majority of this country.

(a)Egypt
(b)Algeria

(c)Sudan

(d)South Africa

76.On world maps of religion, the USA is represented almost uniformly as

(a)Protestant.

(b)secularist.

(c)Buddhist.

(d)Orthodox.

77.Though the USA runs a trade deficit of many hundreds of billions of


dollars per year, the U.S. industry actually runs a trade surplus of around
$25 billion. It is the USA's leading source of foreign income.

(a)automobile

(b)clothing

(c)entertainment

(d)soft drink

78.The majority ethnic groups in China are the

(a)Hmong.

(b)Han.

(c)Dong.
(d)Miao.

79.Veiling across the Islamic world refers to

(a)keeping women away from public places.

(b)dressing plainly.

(c)covering a woman's face.

(d)dressing modestly.

80.The Uyghurs of China are protesting for

(a)freedom from Han Chinese assimilation.

(b)freedom of religious expression.

(c)freedom of cultural expression.

(d)all of the above.

81.The name Islam comes from an Arabic word that means `submission' or
'surrender'. This refers, specifically, to the submission of

(a)nature to humankind.

(b)women to men.

(c)humans to God.

(d)non-Muslims to Muslims.

82.Hausa, Amhara, Lingala and TswanaSotho are


(a)some of Africa's most dominant indigenous languages.

(b)Africa's newest fusion languages.

(c)Africa's most recently extinct languages.

(d)are South Africa's official languages.

83.Xinjiang province in western China is the traditional homeland of the

(a)Han Chinese.

(b)Buddhist Koreans.

(c)Hindu Dravidians.

(d)Turkic and Islamic Uyghurs.

84.Called the All Blacks, powerful and admired national rugby team dance a
Maori haka in front of their opponents before every match in a traditional
display of intimidation and fearlessness.

(a)New Zealand's

(b)South Africa's

(c)Australia's

(d)England's

85.Pork avoidance is a cultural trait of many culture groups. For Muslims,


meat that is permissible to eat is called

(a)sunna.
(b)halal.

(c)kosher.

(d)Quranic.

86.Pork avoidance is a cultural trait of

(a) Hindus.

(b)Jews.

(c)Muslims.

(d)all of the above groups.

87.Of the following, which was the first religion to emerge'?

(a)Sikhism

(b)Hinduism

(c)Christianity

(d)Islam

88.Of the major religions that emerged from the Indo-Gangetic plain, which
spread the furthest (to China, Korea, Japan and Southeast Asia)'?

(a)Mormonism

(b)Hinduism

(c)Buddhism
(d)Sikhism

89.Jews, Christians, Muslims, Baha'i and Druze all have a common


connection in God and

(a)the Dalai Lama.

(b)the Prophet Abraham.

(c)the Holy See.

(d)the Prophet Muhammad.

90.Some argue that films produced by the Nollywood film industry are more
popular than Hollywood. This is certainly the case, at least, to

(a)Africans and the African diaspora.

(b)Norwegians and other northern people.

(c)the worldwide LBGT community.

(d)Christian fundamentalists.

91.Culture is

(a)an attribute found only among folks of less developed countries.

(b)derived only from contact with other societies.

(c)learned behaviour.

(d)genetically derived.
92.The process where aspects of culture move from their area of innovation
to other areas is called

(a)acculturation.

(b)diffusion.

(c)cultural diversity.

(d)dispersion.

93.Humankind has adapted itself to varying conditions through the evolution


of

(a)culture and technology.

(b)environmental determinism.

(c)avoiding racial interbreeding.

(d)living in temperate environments.

94.What was Varro's sequence of cultural stages?

(a)Hunter/gatherers, pastoral nomads, settled agriculture

(b)Peasants, merchants, kings

(c)Warriors, statesmen, clergymen

(d)Savage, barbarian, civilized

95.Environmental determinism

(a)explains the economic and social conditions in Russia at this time.


(b)states that culture groups utilize the natural environment as they see fit.

(c)is the simplistic belief that human cultures can be explained as a result
of the physical environment.

(d)is none of these.

96.When considering that in the past human societies developed in greater


isolation from one another than today, each of the following statements is
true except

(a)the extraordinary diversity of cultures testifies to human ingenuity.

(b)some aspects of cultures that have developed in different physical


environments are startlingly similar.

(c)each group evolved its own distinct culture.

(d)one can easily assume a direct causeand-effect relationship between a


physical environment and an aspect of culture.

97.All the following statements about the concept of diffusion and about
cultural phenomena are true except

(a)Independent invention of different kinds of innovations, such as the


use of the zero as the basis of a numerical place system, has occurred in
human history.

(b)Diffusionist arguments have been used to minimize or underestimate


the ingenuity of various people.

(c)Sometimes the same phenomenon does occur spontaneously and


independently at two or more places.
(d)Diffusion explains the distribution of all cultural phenomena.

98.Concerning folk and popular culture, which of the following statements is


correct?

(a)Folk culture is mass produced.

(b)Folk cultures tend to be spread over large areas and involve rapid
change.

(c)Items of popular culture are individually crafted.

(d)In popular culture, tastes must be relatively homogeneous.

99.A subculture group

(a)is always defined by an ethnic or racial heritage.

(b)exits only in the USA.

(c)shares a part of the mainstream culture and also a bundle of culture


traits of its own.

(d)stems from being marginalized or discriminated against.

100.Ethnocentrism is the term given to the act of

(a)judging other cultures by the standards and practices of one's own


culture.

(b)one culture claiming political rights over other groups.

(c)equating spatial identity with cultural identity.


(d)thinking your nation should dominate other nations.

101.Biological characteristics are used to divide people into ; however


science has shown there is less than 1% genetic difference among
humans.

(a)races

(b)religions

(c)subcultures

(d)cultures

102.Gender roles

(a)have been determined for centuries and cause little debate.

(b)vary across cultures and even within complex societies.

(c)cannot be differentiated spatially.

(d)are common and as a result are very similar in every culture.

103.The subfield of geography that studies our perceptions of the world and
how it influences our behaviour is known as

(a)mental geography.

(b)human geography.

(c)cultural geography.

(d)behavioural geography.
104.Humans make decisions on how they will act in the real world based on

(a)the real world as it is.

(b)incomplete understanding of the world.

(c)how they perceive the world.

(d)predestination.

105.The study of proxemics deals with how different cultures

(a)see relationships.

(b)assess costs.

(c)determine time.

(d)use space.

106.Group identities can help determine which groups of people have a


connection with; however, it can also produce incorrect assumptions
better known as

(a)stereotyping.

(b)over identifying.

(c)individual identities.

(d)heritage building.

107.The belief that onehaso inherent superiority ver another has played a
major role in defining human relations over the past 500 years and is
known as

(a)race; racism

(b)culture; culturalism

(c)nation; nationalism

(d)sex; sexism

108.The entire region throughout which a cultural trait prevails is called a

(a)culture area.

(b)cultural landscape.

(c)built environment.

(d)hearth area.

109.The subfield of geography that studies the geography of the past and
how geographic distributions have changed through time is called

(a)human geography.

(b)cultural geography.

(c)the history of geography.

(d)historical geography.

110.Styles of architecture often represent local adaptations to all of the


following except

(a)severe weather patterns.


(b)climatic conditions.

(c)resource availability.

(d)cultural preferences.

111.Public monuments and statuary often reflect important people and


events within a culture and

(a)are often formed from inexpensive materials.

(b)are permanent symbols of those cultural components.

(c)may change as the culture changes.

(d)are typically only found in Europe.

112.Which of the following is true of settlement patterns?

(a)People seldom choose to live near other people.

(b)Isolated farmsteads are numerous in China.

(c)Compact villages are the norm in North America.

(d)Often compact villages are surrounded by farmland.

113.The force that keeps patterns and activities stable on the landscape is
called

(a)development.

(b)persistence.

(c)cultural landscape.
(d)inertia.

114.Historical consciousness is

(a)defined as a people's sense of their own history.

(b)unrelated to the way in which people view their own culture.

(c)the same as cognitive behavioralism.

(d)irrelevant to the understanding of any alien culture.

115.The term `Latin' America grew out of the fact that the languages spoken
in these countries grew out of Latin. Which languages were used?

(a)Spanish and English

(b)Spanish and Portuguese

(c)Dutch and German

(d)French and English

116.Which of the following is NOT a reason for clustering of settlements'?

(a)Environmental reasons, such as access to water

(b)To stay in close contact with cattle and livestock

(c)Provision of services such as education and healthcare

(d)Security against bandits or invaders

117.Voyages of exploration and conquest by connected the world.


(a)the Japanese

(b)the Ottoman Turks

(c)the Chinese

(d)Europeans

118.The substitution of one set of cultural traditions for another, either by


force or by degrading those who fail to acculturate and by rewarding
those who do, is called

(a)cultural imperialism.

(b)conquest.

(c)cultural diffusion.

(d)enlightenment.

119.Europeans took bananas from their point of origin in and replanted them
in South America.

(a)Southeast Asia

(b)Africa

(c)Europe

(d)Hawaii

120.Increasing the amount of trade to a culture decreases cultural isolation


and increases
(a)dependency on the local environment.

(b)economic self-sufficiency.

(c)economic possibilities.

(d)cultural dependency.

121.In countries with limited natural resources, such as Switzerland, the


impact of trade

(a)can provide goods and services causing an increase in wealth in the


country.

(b)may cause an increase in war within the country.

(c)will reduce the economic viability of the country.

(d)can destroy the native culture.

122.takes place more rapidly today because of a decrease in cost and time to
move ideas, objects and people.

(a)Diffusion

(b)Military response

(c)Trade

(d)Transportation

123.In modem-world trade,

(a)all trade is regulated by the United Nations.


(b)virtually all people have moved from self-sufficiency and cultural
isolation to trade and cultural exchange.

(c)all parts of the world have eagerly, voluntarily participated in world


trade.

(d)some places are still completely isolated and dependent on local


resources.

124.The major reason European countries engaged in building large empires


was to

(a)import cultures from other areas into Europe.

(b)work cooperatively with indigenous peoples to learn new agricultural


techniques.

(c)expand their country's borders.

(d)protect their commercial investments in foreign lands and to control


the markets.

125.The increasingly rapid movement of information and goods around the


globe is known as

(a)transportation enhancement.

(b)transportation efficiency.

(c)diffusion.

(d)time-space compression.

126.Cultural preservation is the effort to


(a)document, repopularize and rejuvenate traditional cultures.

(b)support music and art programs in schools.

(c)invest in museum and religious institutions.

(d)preserve the existing culture for future generations.

127.Diffusion was originally a person-toperson activity and was relatively


placebased. The invention of the allowed information to travel far past its
place of origin.

(a)printing press

(b)computer

(c)Internet

(d)all of the above

128.Technology was developed within the 1800s that allowed information to


move between places faster than ever before.

These inventions included the

(a)postal system and the telegraph.

(b)printing press and the postal system.

(c)telegraph and the telephone.

(d)telephone and the postal system.

129.Mass media takes advantage of to reach as many people as possible.


(a)contiguous diffusion

(b)the Internet

(c)hierarchical diffusion

(d)relocation diffusion

130.The annihilation of space with can convey information almost


instantaneously around the globe.

(a)air transportation systems

(b)cell phones

(c)automobiles

(d)electronic networks

131.A virtual reality is a new `place' that has been created

(a)among a group of users.

(b)mentally, through the use of computers.

(c)within areas of the government.

(d)within video games.

132.Imperialism begins with

(a)colonization.

(b)treaty negotiations.
(c)exploration.

(d)settlement building.

133.Prior to Belgian colonialism in Rwanda, the Hutus and Tutsis lived


together quite peacefully.According to our text, conflicts between the
groups emerged after the Belgians introduced the concept of

(a)slavery.

(b)socio political difference.

(c)money.

(d)cash crops.

134.Contributing to the conflicts and tensions in Africa is the colonial legacy


of borders that

(a)separated people from traditional resources.

(b)divided nations that had been united.

(c)united nations that had been separate.

(d)All of the above.

135.On 30 October 1995, in a vote related to sovereignty, the people of


chose not to separate from

(a)Kurdistan/Turkey.

(b)Chiapas/Mexico.
(c)the Basque Region/Spain.

(d)Quebec/Canada.

136.Which is the most inclusive organization (has most members) and has as
a primary goal the maintenance of international peace and security?

(a)United Nations

(b)OPEC

(c)NAFTA

(d)Organization of Economic Cooperation and Development

137.Which two countries are NOT members of the European Union?

(a)Spain and France

(b)Norway and Switzerland

(c)Germany and Denmark

(d)Portugal and Austria

138.A federal state (or federation) is the one in which.

(a)local units of government work to adopt uniform rules and regulations.

(b)government power and control is centralized.

(c)government power is disbursed to local units.

(d)countries unite towards a common economic, or sometimes military,


goal.
139.Nationalists that want an independent state of Quebec are properly
described as

(a)provincialists.

(b)irredentists.

(c)statists

(d)secessionists.

140The Zapatista rebellion in Chiapas (Mexico), which coincided with the


beginning of NAFTA, was over the issue of

(a)indigenous rights.

(b)a peasant way of life.

(c)exploitation.

(d)All of the above.

141Ratzel's belief that geopolitics stems from the interaction ofis still
popular today.

(a)space and place

(b)power and territory

(c)men and women

(d)the rich and the poor


142.The Berlin Wall and the Mexican border are good examples of
boundaries.

(a)unitary

(b)monetary

(c)exclusionary

(d)illusionary

143.Among the following, the best example of an inclusionary boundary is


the boundary between

(a)Israel and the West Bank.

(b)the USA and Mexico

(c)North and South Korea.

(d)Wisconsin and Minnesota.

144.Frontier regions, where territoriality is underdeveloped, are known for


their.

(a)untapped resources.

(b)marginality.

(c)rapid growth.

(d)belonging.
145.Only currently exists as a frontier region where territoriality is
undeveloped.

(a)Antarctica

(b)Northern Canada

(c)sub-Saharan Africa

(d)Siberia

146.Which of the following can best be considered a modem frontier?

(a)Sub-Saharan Africa

(b)The Andean highlands

(c)Antarctica

(d)The Yukon

147.Territories like Canada, Michigan and Washtenaw County that are


delimited by formal boundaries are known as spaces or regions.

(a)de facto

(b)de rigeur

(c)de limito

(d)dejure

148.As used in our text, the term state is closest in meaning to.

(a)territory.
(b)colony.

(c)nation.

(d)country.

149.In the context of the course and text, is the best example of a state.

(a)Quebec

(b)Kurdistan

(c)Egypt

(d)Minnesota

150.Which of the following pairs best represents nationstates'?

(a)The Basque Region and Kurdistan

(b)India and Russia

(c)Japan and Iceland

(d)USA and Canada

151.A cultural group with a territorial identity is known as a

(a)province.

(b)state.

(c)race.

(d)nation.
152.Of the following, the USA is best characterized as a

(a)monarchy.

(b)supranational organization.

(c)confederation.

(d)republic.

153.To be considered a nation, a group of people must

(a)share a sense of nationalism.

(b)share cultural elements like religion, language, history, politics.

(c)have their own country.

(d)be sovereign and self-determined.

154.The process of allocating electoral seats to geographical areas is known


as

(a)irredentism.

(b)secessionism.

(c)separatism.

(d)reapportionment

155.In the transition of the USSR to the CIS, Russia went from being the
core of a unitary state to being the most powerful member of a

(a)zionist state.
(b)confederation.

(c)supranational organization.

(d)nationstate.

156.Which of the following have been tried by 20th-century states to


eliminate or minimize the potential for fragmenting or breaking apart'?

(a)Destruction of ethnic minorities

(b)Increased autonomy to ethnic minorities

(c)Consolidation or centralization of power

(d)All of the above have been used.

157.From the following, the best example of a supranational organization is


(are)

(a)the European Union.

(b)the United Nations.

(c)the Commonwealth of Independent States.

(d)the Canadian provinces.

158.According to Ratzel's theory of the state, imperialism emerges from the


need for territorial expansion tied to

(a)irredentism.

(b)population growth.
(c)a new world order.

(d)centrifugal forces.

159.Despite independence of India, Rwanda and Algeria, the continuing


effects of colonialism are still seen in.

(a)violent conflicts involving territory and power.

(b)the large secondary (manufacturing) sector of the economy.

(c)their incorporation into the world system core.

(d)English as the official language or lingua franca.

160.The North/South Divide is used in our text to distinguish.

(a)(former) communist from noncommunist states

(b)North African states from subSaharan states.

(c)anti-slavery from pro-slavery states.

(d)former colonizers from formerly colonized states.

161.Contract farming - whereby production in the periphery is for


consumption in the core - is one example of.

(a)the end of colonialism.

(b)self-determination.

(c)regionalism.

(d)neocolonialism.
162.The (re)acquisition of sovereignty and territorial integrity following
colonization is known as

(a)decolonization.

(b)protocolonialism.

(c)quasicolonialism.

(d)neocolonialism.

163.Of the following, the most prominent theory supporting U.S.


involvement in the Vietnam War was

(a)the domino theory.

(b)heartland theory.

(c)anti-terrorism.

(d)zionism.

164.According to Halford Mackinder's Heartland Theory, the `geographical


pivot' from which world conquest could be controlled includes the area
of.

(a)Southeast Asia, including Indonesia.

(b)Eurasia, including Iran, Afghanistan and Central Asia.

(c)Europe from Italy to Holland.

(d)the Arabian peninsula.


165.The Heartland Theory held that

(a)sea power was the dominant form of military advantage.

(b)Britain should give its colonies independence.

(c)North America was the best place from which to launch a campaign
for world domination.

(d)None of these

166.Halford Mackinder, the developer of the Heartland Theory, thought that


this country was most likely to make a bid for world domination.

(a)Britain

(b)The USA

(c)Russia

(d)France

167.The East/West divide refers to the divide between

(a)the Christian and Islamic spheres of belief.

(b)the New and Old Worlds.

(c)the core and the periphery.

(d)Indo-European language speakers and Sino-Tibetan/Afro-Asiatic


language speakers.
(d)none of the above - the East/West divide disappeared with the end of
the Cold War.

168.The fall of the Berlin Wall symbolizes the

(a)beginning of the new world order.

(b)triumph of capitalism over communism.

(c)end of the Cold War.

(d)all of the above.

169.The new world order is characterized by all of the following except.

(a)reduction in world terrorism.

(b)U.S. hegemony.

(c)regional instabilities.

(d)transnational corporate growth.

170.OPEC has member countries from all of the following regions except.

(a)Middle East.

(b)sub-Saharan Africa.

(c)Europe.

(d)Southeast Asia.

171.The recent experience of the Chechens and Kurds is similar in that both.
(a)have struggled unsuccessfully for independence.

(b)are the newest members of the European Union.

(c)are oil-rich members of the Commonwealth of Independent States.

(d)recently received independence after a long and bloody struggle.

172.With globalization and the increasing importance of trade-facilitating


organizations, states are

(a)disappearing.

(b)becoming sites of transnational flows and connections.

(c)becoming irrelevant.

(d)growing as containers of political and economic power.

173.The territorially based sense of identity in groups like the Basques,


Kurds and Quebec's French-speaking population is referred to as.

(a)nationalism.

(b)territorialism.

(c)multinationalism.

(d)regionalism.

174.is the strong devotion to local interests and customs and has been used
to explain the Civil War as the Union's effort to make sure that the strong
attachment to the institution of slavery would not take priority over the
unity of the whole.
(a)Nationalism

(b)Sectionalism

(c)Multinationalism

(d)Regionalism

175.Whereas reapportionment adjusts the amount of political representation


given to a certain geographical area, redistricting adjusts.

(a)country, state and/or county boundaries.

(b)federal lands.

(c)population size.

(d)the geographical area.

176.Antarctica has been colonized by

(a)the United Nations.

(b)countries of the periphery.

(c)the core countries.

(d)a handful of countries from both the North and the South.

177.According to a map in your book, claim the largest portions of


Antarctica.

(a)Australia and Norway

(b)Chile and Argentina


(c)New Zealand and France

(d)the USA and Russia

178.Of the six sovereign states that emerged from the ruins of Yugoslavia,
all but one are a mix of nationalities. Only come(s) close to being a
nationstate.

(a)Croatia

(b)Bosnia and Herzegovina

(c)Serbia and Montenegro

(d)Slovenia

179.Political geography

(a)has generally ignored the peopleland tradition.

(b)places little importance on globalization.

(c)has its roots in Hinduism.

(d)was influenced by environmental determinism in the late 19th to early


20th centuries.

180.For political geographers, is at the centre of most inquiry.

(a)the state

(b)natural rights

(c)the nation
(d)sovereignty

181.The true nationstate

(a)gives sovereignty to each nation within its borders.

(b)often contains two or more sizable nations within it.

(c)is best exemplified by the USA.

(d)is relatively uncommon.

182.Lenin's solution to the `national problem' was

(a)the granting of independence to areas with less than 25% ethnic


Russian population.

(b)a federalist system of government.

(c)multiparty democracy and an independent judiciary.

(d)brutal suppression of nationalism.

183.In which form of government is significant power given to smaller units


of government within the state?

(a)Unitary state

(b)confederate state

(c)Democratic state

(d)Federal state

184.Under Mikhail Gorbachev, the USSR


(a)expanded its territory farther into eastern Europe.

(b)changed from a confederation to a unitary state.

(c)experienced no nationalist uprisings.

(d)introduced perestroika (economic and government restructuring).

185.The relatively peaceful dismantling of the USSR.

(a)left many of the Soviet Republics without independence.

(b)was facilitated in part by the fact that the USSR had a federal structure.

(c)occurred in the mid-1980s.

(d)would not have occurred without the brilliant statesmanship of


President Bush.

186.Which of the following is NOT one of the states that formed from the
former Soviet Union after its breakup?

(a)Russia

(b)Kazakhstan

(c)Estonia

(d)Chechnya

187.This region leads in the efforts to bring Soviet rule to an end.

(a)The Balkans.

(b)The Caucasus.
(c)Central Asia.

(d)The Baltics.

188.The state.

(a)is a set of institutions.

(b)is active, not static.

(c)controls territory.

(d)All of the above are true for the state.

189.The British presence in India.

(a)did not formally end until just after World War II.

(b)started with the interjection of the British military around 1700.

(c)had little lasting effect on Indian society.

(d)was welcomed by the indigenous ruling elites.

190.The postcolonial history of India has included

(a)regional conflicts.

(b)partition.

(c)ethnic conflicts.

(d)all of the above.

191.The main cause of civil strife in Rwanda stems from


(a)interference by South Africa.

(b)antidemocratic values learned from the Germans.

(c)the legacy of superpower competition left over from the Cold War.

(d)the Belgian colonial practice of favouring Tutsis.

192.The League of Nations

(a)established the colonial mandate system.

(b)was formed in the middle of the 19thcentury.

(c)never allowed the Soviet Union to join.

(d)had no significant successes.

193.Neocolonialism

(a)peaked near the end of the 19thcentury.

(b)has in general been anticapitalist.

(c)allows core states to continue exploiting periphery states.

(d)is still relevant in Africa, but not in other parts of the Southern
Hemisphere.

194.Antarctica

(a)has a small but thriving native population.

(b)has been carved like a territorial pie into claims by various states.
(c)is the coldest place in the Northern Hemisphere.

(d)is owned by the United Nations.

195.Fidel Castro

(a)still receives large amounts of foreign aid from Russia.

(b)spent more years in power than any four American presidents put
together.

(c)was quickly removed from power by the USA.

(d)was initially hostile to the USA and the West.

196.The domino theory

(a)was applied to Greece and Turkey, but not to Berlin.

(b)was confined to the Eurasian landmass.

(c)was rejected by President Eisenhower, a former general.

(d)was the rationale behind stopping the spread of communism.

197.Which of the following countries is NOT a member of the United


Nations Security Council?

(a)Japan

(b)Britain

(c)China

(d)France
198.Which of the following organizations is an international military
alliance'?

(a)North American Free Trade Agreement

(b)Organization of Petroleum Exporting Countries

(c)European Union

(d)North Atlantic Treaty Organization

199.The process of determining the boundaries of districts for the purpose of


electing political officials is called

(a)gerrymandering.

(b)confabulation.

(c)redistricting.

(d)reapportionment.

200.This was NOT an independent country in 1989.

(a)Austria

(b)Poland

(c)Slovakia

(d)Hungary

201.The Alps form Italy's boundary with all of the following countries,
except.
(a)France.

(b)Greece.

(c)Austria.

(d)Switzerland.

202.From 1992 to 2006, this and Serbia was still a part of the former
Yugoslavia.

(a)Slovenia

(b)Macedonia

(c)Bosnia

(d)Montenegro

203.This part of the former Yugoslavia saw a horrific civil war between
Serbs and ethnic Albanians.

(a)Kosovo

(b)Slovenian

(c)Croatia

(d)Macedonia

204.Which country most extensively colonized South America'?

(a)Portugal

(b)Spain
(c)Britain

(d)USA

205.Hundreds of thousands of Palestinian refugees were created when they


were driven from their homes.

(a)with the creation and territorial expansion of Israel.

(b)by Islamist organizations like Hamas and Islamic Jihad.

(c)when the British created Palestine and Transjordan.

(d)when the Ottoman Empire came into power.

206.The West Bank and Gaza Strip are called the `Occupied Territories'
because they are

(a)parts of Israel occupied by Palestinians.

(b)parts of Israel under attack by Palestinians.

(c)remnants of Palestine occupied by Israel.

(d)controlled by the British as part of the British mandate.

207.Israel's security fence, designed to protect its citizens from terrorist


attacks,

(a)reaches deep into Palestinian territory and separates them from their
livelihoods.

(b)was authorized by the UN General Assembly as a critical first step in


the peace process.
(c)follows the territorial boundary between Israel and the West Bank.

(d)now enables the orderly flow of goods, services and labour between
the West Bank and Israel.

208.All of the following were European states in the 1980s. Which


continues as a European state even today'?

(a)Yugoslavia

(b)Hungary

(c)Czechoslovakia

(d)East Germany

209.In their push to secede from only Russia has (illegally) recognized the
independence of South Ossetia and Abkhazia.

(a)Moldova

(b)Ukraine

(c)Serbia

(d)Georgia

210.In the shifting map of Europe and regional allegiances, closer political-
economic ties between Europe and have heightened tensions between
Russia and EU/NATO countries.

(a)Georgia and Ukraine

(b)Slovenia and Slovakia


(c)Armenia and Azerbaijan

(d)Serbia and Montenegro

211.From its origins around modem-day Moscow in 1462, the territory of


the Russian land empire and Soviet Union had expanded to fill in the
entire area between all of the following bodies of water except.

(a)the Mediterranean Sea.

(b)the Arctic Ocean.

(c)the Black Sea.

(d)the Baltic Sea.

212.In practice, the USSR operated as a

(a)republic.

(b)unitary state.

(c)monarchy.

(d)federation.

213.Popular European revolutions of the late 18th to mid-19th centuries


transformed numerous monarchies into

(a)nations.

(b)supranational organizations.

(c)confederations.
(d)republics.

214.For his work on characterizing the state based on such factors as


climate, terrain and the relationship between population and territory, is
often considered the first political geographer.

(a)Friedrich Ratzel

(b)Louis Althussser

(c)Montesquieu

(d)Aristotle

215.In the USA, one of the most prominent expressions of sectionalism


today is the political tension between.

(a)the Sunbelt and the Snowbelt.

(b)documented and undocumented immigrants.

(c)cities and suburbs.

(d)Eastern and Western states.

216.In what was known as the Great Game, the Russian and British empires
each struggled unsuccessfully for control of . They did, though, along the
way manage to establish its modern-day boundaries.

(a)Persia

(b)Tibet

(c)Nepal
(d)Afghanistan

217.In the 1980s, the US, Saudi Arabia and Pakistan provided weapons and
training to enable Islamic freedom fighters known as theto drive the
Soviets out of Afghanistan.

(a)Islamic Jihad

(b)Taliban

(c)Michigan Militia

(d)Muj ahideen

218.Before it split off from India in 1947 to become a part of Pakistan, it


was East Bengal. In 1971, it split off from Pakistan to become the
independent state of

(a)Brunei.

(b)Bhutan.

(c)Bermuda.

(d)Bangladesh.

219.With a few small exceptions, the countries of the European Union share
all of the following except

(a)a European Union Constitution.

(b)an EU court system.

(c)a common currency.


(d)an EU parliament.

220.The way any language is spoken and written according to formal rules
of diction and grammar is called a(n)

(a)language family.

(b)standard language.

(c)lingua franca.

(d)official language.

221.A creole language is

(a)a pidgin that has survived long enough to become a native language.

(b)a language spoken in Louisiana.

(c)a language that does not have grammar.

(d)a language used to describe food.

222.The predominant world lingua franca is

(a)Russian.

(b)Spanish.

(c)French.

(d)English.

223.The language with the most native speakers in the world is


(a)French.

(b)English.

(c)Hindi.

(d)Mandarin Chinese.

224.The language of the Qur'anis

(a)Farsi.

(b)Arabic.

(c)Hebrew.

(d)Urdu.

225.Which of the following is true concerning lingua francas?

(a)They are all a mixture of different languages.

(b)They are usually spoken by the elite only.

(c)They are the official dialect of their respective languages.

(d)Swahili is a lingua franca of east Africa.

226.Lines that are used to show the presence of linguistic features on maps
are known as

(a)dialect maps.

(b)topographic lines.
(c)language charts.

(d)isoglosses.

227.Languages that are related by descent from a common protolanguage


make up a

(a)language group.

(b)family of dialects.

(c)language family.

(d)language region.

228.The family of languages known as the Indo-European family contains


languages spoken by approximately of the world population.

(a)70 per cent

(b)30 per cent

(c)50 per cent

(d)25 per cent

229.The hearth area for Indo-European languages was probably located in

(a)North Africa.

(b)the Italian peninsula.

(c)north-western Europe.

(d)Turkey.
230.A system for writing symbols that represent sounds of a language is
known as the

(a) orthography.

(b)cuneiform.

(c)ideograms.

(d)pictograms.

231.Which of the following languages is NOT part of the Uralic language


family?

(a)Gaelic

(b)Estonian

(c)Hungarian

(d)Finnish

232.The languages that do not use written alphabets are used in

(a)Greece and Rome.

(b)China and Japan.

(c)religious documents.

(d)aboriginal cultures.

233.The following language has become a lingua franca of approximately


335 million people as a first language, 375 million as a second language
and 600 million as a foreign language.

(a)Chinese

(b)French

(c)Spanish

(d)English

234.The global spread of English is primarily the result of

(a)American media influences.

(b)colonization by the British Empire.

(c)Australian currency production.

(d)British involvement in World War II.

235.Philological nationalism is the idea that have given birth to nations.

(a)official languages

(b)standards languages

(c)mother tongues

(d)dialects

236.Official policies and social attitudes may target as potentially disloyal or


dangerous to the rest of the nation.

(a)minority politicians
(b)minority ethnic groups

(c)minority language speakers

(d)minority races

237.In polyglot states,

(a)the language that is native to the region is the most important for
economic advancement.

(b)all spoken languages have equal economic and social standing.

(c)legal equality is granted to more than one language.

(d)There are no polyglot states, so the issue is mute.

238.American English evolved from influences of European settlers as well


as

(a)Mexican languages.

(b)native American languages.

(c)Canadian languages.

(d)Chinese languages.

239.The official language of Nigeria is

(a)Hausa.

(b)Yoruba.

(c)English.
(d)Ibo.

240.In the context of religion, the strictest adherence to traditional beliefs is


called

(a)devotion.

(b)secularism.

(c)piety.

(d)fundamentalism.

241.The first great monotheistic religion was

(a)Shinto.

(b)Buddhism.

(c)Catholicism.

(d)Judaism.

242.Zionism is the belief that the Jews should have a homeland of their own
in

(a)France.

(b)Palestine.

(c)Germany.

(d)the USA.

243.Christianity emerged as a separate faith from


(a)Hinduism.

(b)Judaism.

(c)Islam.

(d)Buddhism.

244.The Christian church in western Europe was split in the 15th and 16th
centuries as a result of the

(a)efforts to convert people in Asia.

(b)immigration of the people to the USA.

(c)protestant reformation.

(d)incorporation of Jewish beliefs into Christianity.

245.The largest single Christian denomination in the world today is

(a)Baptist.

(b)Lutheran.

(c)Anglican.

(d)Roman Catholicism.

246.Religions that focus on appropriate behaviour are classified as

(a)orthopraxy.

(b)orthodoxy.
(c)fundamentalist.

(d)secular.

247.A lifestyle or policy that purposely ignores or excludes religious


considerations is known as

(a)orthopraxy.

(b)orthodoxy.

(c)fundamentalist.

(d)secular.

248.Religions that focus on a set of philosophical or theological arguments


are classified as

(a)orthopraxy.

(b)orthodoxy.

(c)fundamentalist.

(d)secular.

249.Universalizing religions are those that

(a)are based on science rather than faith.

(b)people are born into as part of a cultural group.

(c)involve the worship of multiple gods.

(d)spread through proselytization.


250.Many governments have designated that is the official religion of the
country.

(a)an organized government agency

(b)an ethnic religion

(c)an established church

(d)an universalizing church

251.The Five Pillars of Islam include all of the following except

(a)five daily prayers.

(b)recognition of the month of Ramadan as a time to fast.

(c)polygamy.

(d)generous giving of alms.

252.Muslims believe that Muhammad was the last of God's prophets, who
also included all the following except

(a)Abraham.

(b)Guru Nanak.

(c)Moses.

(d)Jesus.

253.Shinto is the ancient religion native to

(a)India.
(b)Vietnam.

(c)Indonesia.

(d)Japan.

254.Religions provide guidance when people are confronted by rapid


change, even in

(a)local schools.

(b)many governmental agencies.

(c)secular modern societies.

(d)large scientific research firms.

255.Religions that combine Christianity with traditional practices are called

(a)sacerdotalism.

(b)zionism.

(c)syncretic religions.

(d)liberation theology.

256.The leading Muslim countries in the world today in terms of numbers


are

(a)Indonesia and Pakistan.

(b)Bangladesh and Afghanistan.

(c)India and Nepal.


(d)Iraq and Iran.

257.Which of the following European countries is predominantly Muslim?

(a)Albania

(b)Romania

(c)Bulgaria

(d)Poland

258.Martin Luther translated the Bible into

(a)French.

(b)Spanish.

(c)English.

(d)German.

259.Recent immigrants to the USA are more likely to be

(a)Catholic.

(b)Protestant.

(c)Jewish.

(d)Buddhist.

260.Jews and Muslims both refuse to eat due to religious considerations.

(a)chicken
(b)turkey

(c)beef

(d)pork

261.Religions differ in the attitudes that their teachings advocate towards


women and

(a)approve the ordination of women.

(b)increase availability of contraception.

(c)national laws often reflect those teachings.

(d)women are granted more freedoms as a result.

262.The word artefact literally means

(a)what was lost.

(b)a thing made by skill.

(c)a thing belonging to ancient people.

(d)a thing influencing a culture.

263.In cultural geography, the term 'evolutionism' denotes

(a)Darwin's theory of origin of men.

(b)that the most important source of cultural changes are embedded in the
culture itself.
(c)that the most important source of cultural change is the natural
environment of an alien culture.

(d)none of these.

264.Which of the following statements is/are true regarding cultural


diffusion'?

(a)It can be freely chosen by the target population.

(b)It can be forced on the target population.

(c)It is the process by which different aspects of a culture spread out from
the places they originated and are adopted by other people.

(a)1 and 2

(b)2 and 3

(c)3 and 1

(d)1, 2 and 3

265.The statement that `ethnicity is largely a matter of self-identification' is

(a)true.

(b)false.

(c)partially true.

(d)cannot be determined.

266.The cultural regions are defined on the basis of


(a)any aspect of culture.

(b)race.

(c)religion.

(d)economy.

267.Why is delineation of cultural regions important?

(a)Because it predisposes us to stereotype the people within each region


as homogeneous.

(b)Because our pattern predisposes us to stereotype the people within


each region as homogeneous.

(c)It helps in stereotyping people within each region.

(d)All of these.

268.Which of the following statements is true regarding the problem of


defining great culture regions'?

1.A region need not display certain homogeneity.

2.Homogeneity of the group might only exist in the eyes of an outsider.

(a)Only 1

(b)Only 2

(c)Both 1 and 2

(d)Neither 1 nor 2
269.Which of the following questions must be asked of any pattern of
cultural regions'?

1.Has the Cartographer drawn the region correctly according to the


chosen criteria'?

2.Are the criteria chosen meaningful?

3.Do the insider and outsider views of the culture match'?

(a)1 and 2

(b)2 and 3

(c)3 and 1

(d)1, 2 and 3

270.Which of the following statements holds true for styles of architecture


as visual clues to cultural realms?

(a)They are mostly adapted to local climatic conditions.

(b)It might diffuse beyond the limits of where its building material can be
found.

(c)It is independent of the cultural preferences of the people.

(d)All of these.

271.Which of the following statements holds true about languages?

(a)Language is the most important force that defines a cultural area.


(b)Any group of people who communicate exclusively with one another
will soon develop `their own language'.

(c)The distribution of any language does not illustrate the pattern of


dispersal of original speakers.

(d)None of these.

272.The pidgin language

(a)is normally English.

(b)is the one which follows a set of formal rules.

(c)is a dialect of Dutch.

(d)is used among people who do not share a common language.

273.Which of the following pidgin languages has survived long enough to


become a mother tongue?

(a)Celtic

(b)Norwegian

(c)Creole

(d)Seychellois

274.Which of the following statements holds true for isoglosses'?

(a)They can be defined as thestudy of different dialects across space.

(b)They frequently parallel physical landscape features.


(c)Lingua franca is normally used for defining isoglosses.

(d)They study different dialects across time.

275.The theory of common original language was developed by

(a)Walter Smith Jr.

(b)Jacob Grimm.

(c)Sir Ahmed Filani.

(d)Sir William Jones.

276.Which of the following statements holds true for proto-Indo-European


language'?

1.Archaeologists agree that it diffused quickly carried by warriors.

2.Archaeologists agree that is diffused slowly carried by farmers.

3.It tells us how the proto-IndoEuropean society lived.

(a)Only 1

(b)1 and 3

(c)2 and 3

(d)Only 3

277.Which of the following can be regarded as the main characteristic of


polyglot state?

(a)It recognizes the right to develop own languages.


(b)It grants legal equality to two or more religions.

(c)It grants legal equality to two or more languages.

(d)It recognizes the right of having more than one wife.

278.Which of the following statements holds true for orthopraxy and


orthodoxy'?

(a)Both are system of religions.

(b)One deals with appropriate behaviour other deals with philosophical


argument.

(c)Both are sects of Christianity.

(d)Both are sects of Islam.

279.In Islamic countries,

(a)there is a clear distinction between the state and the religion.

(b)state and the religion are the one and the same thing.

(c)minorities cannot be officially recognized.

(d)fundamentalism acts as the basis of state.

280.Which of the following statements holds true for the relationship


between religion and consumption pattern'?

(a)Religion does not influence consumption pattern at all.

(b)Religion can substantially influence certain consumption pattern.


(c)Religion's effect on consumption pattern cannot be measured.

(d)Religion's effect on consumption pattern is too minute to be studied.

281.In which of the following ways does religion affect the economies of the
world'?

1.In certain countries, interest on borrowed money is discouraged by the


religion.

2.In certain countries, the religious rites and practices can influence the
labour markets.

(a)Only 1

(b)Only 2

(c)Both 1 and 2

(d)Neither 1 nor 2

282.According to the Mackinder, Russia was at disadvantage because

(a)it lacked land power.

(b)it was made up on many ethnic groups.

(c)it had many borders.

(d)it had Communist Government.

283.According to Mackinder, which of the following countries should be


outside the pivot zone'?
(a)Russia

(b)England

(c)Both (a) and (b)

(d)Neither (a) nor (b)

284.According to Mackinder, Britain became the new pivot because

1.its industrial revolution gave it economic power.

2.its naval strength gave it sea power.

(a)Only 1

(b)Only 2

(c)Both 1 and 2

(d)Neither 1 nor 2

285.Bantus tribe is found in

(a)southern Africa.

(b)eastern Africa.

(c)Africa, North of Sahara.

(d)western Africa.

[UGC D-2005]

286.Kinghiz nomads are found in


(a)Sahara Desert region.

(b)Congobasin.

(c)central Asia.

(d)Saudi Arabia.

[UGC D-2005]

287.Mongoloid race is predominantly found in

(a)South Africa.

(b)east Asia.

(c)western Europe.

(d)east Africa.

[UGC D-2005]

288.Tribal people practice shifting cultivation primarily because

(a)soil fertility gets reduced after a few years.

(b)the forest re-establish itself in a few years.

(c)people prefer migratory life.

(d)land is left fallow to regain its fertility.

[UGC D-2005]

289.Who is known as the Father of Political Geography'?


(a)Rudolf Kjellen

(b)K. Haushofer

(c)F. Ratzel

(d)R. Hartshorme

[UGC D-2006]

290.Who gave the concept of Rimland Theory in political geography'?

(a)N.J. Spykman

(b)E. Warner

(c)H.W. Weigert

(d)A.T. Mahan

[UGC D-2006]

291.Mongoloid race is predominantly found in

(a)South Africa.

(b)east Asia.

(c)western Europe.

(d)east Africa.

[UGC D-2006]

292.Which one of the following is related to the tribal economy'?


(a)Footloose industries

(b)Settled cultivation

(c)Shifting cultivation

(d)Industrial economy

[UGC D-2006]

293.Racial traits of man are studied in

(a)palaeontology.

(b)hydrology.

(c)sociology.

(d)anthropology.

[UGC D-2006]

294.Who formulated the Heartland Theory?

(a)H.J. Mackinder

(b)R. Hartshone

(c)Peter Hagget

(d)Herbertson

[UGC D-2006]

295.Cultural landscape shows


(a)human impact on environment.

(b)dominance of nature.

(c)impact of ecological factors.

(d)association of fauna and flora.

[UGC D-2007]

296.Which one of the following districts is dominated by tribal population'?

(a)Kanyakumari

(b)Bastar

(c)Rohtak

(d)Karnal

[UGC D-2007]

297.The primitive people who live in Kalahari desert are known as

(a)Pigmies.

(b)Bantus.

(c)Bushman.

(d)Todas.

[UGC D-2007]

298.Which one of the following has no caste in the scheduled category?


(a)Nagaland

(b)Manipur

(c)Dadra and Nagar Haveli

(d)Pondicherry

[UGC D-2007]

299.Which of the following groups of religions originated in the Middle


East'?

(a)Judaism, Christianity, Confucianism

(b)Judaism, Islam, Christianity

(c)Judaism, Islam, Taosim

(d)Islam, Christianity, Confucianism

[UGC D-2008]

300.Which one of the following is most spoken Indo-Austric language in


India'?

(a)Khasi

(b)Santhali

(c)Munda

(d)Kharia

[UGC D-2009]
301.Match List I with List II and select the correct answer from the codes
given.

Codes:

[UGC D-2009]

302.Which of the following is not a social indicator of regional


development?

(a)Number of primary schools in the region

(b)Number of primary health centres in the region

(c)Number of medical professionals per thousand population

(d)Number of tertiary workers in the population


[UGC D-2009]

303.Which one of the following is a South Indian caste'?

(a)Madiga

(b)Gujar

(c)Khatik

(d)Dom

[UGC J-2005]

304.Given below are two statements one labelled as Assertion (A) and the
other labelled as Reason (R). Select your answer from the codes given.

Assertion (A): Social diversity is the most effective manifestation of


Indian identity.

Reason (R): There are well-defined tribal identities based on ethnic


origin.

(a)Both (A) and (R) are correct and (R) explains (A).

(b)Both (A) and (R) are correct but (R) does not explain (A).

(c)(A) is correct but (R) is false.

(d)(A) is false but (R) is correct.

[UGC J-2005]

305.Which one of the following states has the highest concentration index of
Urdu speaking population (1991)?
(a)Maharashtra

(b)Bihar

(c)Madhya Pradesh

(d)Uttar Pradesh

[UGC J-2005]

306.The theory of diffusion of innovation was propounded by

(a)Christaller.

(b)Hagerstrand.

(c)Hagget.

(d)Chorley.

[UGC J-2005]

307.Gonds have their major concentration in

(a)Koraput region.

(b)Aravalli region.

(c)Chhota Nagpur region.

(d)Bastar region.

[UGC J-2007]

308.Name the propounder of Rimland theory.


(a)Mackinder.

(b)Spykman.

(c)Monkhouse.

(d)Hartshorne.

[UGC J-2008]

309.`Gonds' concentration is largely located in

(a)Indian Thar desert.

(b)Baghel Khand region.

(c)Bastar region.

(d)Bundelkh and region.

[UGC J-2008]

310.The Zones and Strata Theory pertaining to the origin and diffusion of
human races was propounded by

(a)Ripley.

(b)Taylor.

(c)Huntington.

(d)Baker.

[UGC J-2011]
311.Boundaries that are allocated and delimited before significant settlement
in an area are called

(a)antecedent boundaries.

(b)subsequent boundaries.

(c)superimposed boundaries.

(d)relic boundaries.

[UGC J-2011]

312.Given below are two statements one labelled as Assertion (A) and the
other labelled as Reason (R). Select your answer from the codes given.

Assertion (A): Mackinder's Theory of Heartland put forward a key


formula `Who rules east Europe commands the Heartland; who rules the
Heartland commands the world island; who rules the world island
commands the world'.

Reason (R): Mackinder's formula offered Poland the strategy for


territorial expansion and international supremacy.

(a)Both (A) and (R) are correct and (R) is the correct explanation of (A).

(b)Both (A) and (R) are correct but (R) is not the correct explanation of
(A).

(c)(A) is correct, but (R) is false.

(d)(A) is false, but (R) is correct.

[UGC J-2011]
313.Given below are two statements, one is labelled as Assertion (A) and the
other labelled as Reason (R). Select your answer from the codes given.

Assertion (A): Denmark is a nation state in true sense of the term.

Reason (R): A nationstate has a territorial base but does not have
necessarily a social or a cultural base.

(a)Both (A) and (R) are true and (R) is the correct explanation of (A).

(b)Both (A) and (R) are true, but (R) does not explain (A).

(c)(A) is true, but (R) is false.

(d)(A) is false, but (R) is true.

[UGC J-2011]

314.Match List I with List II and select the correct answer from the codes
given.

Codes:
[UGC J-2011]

315.Carl Sauer is best identified for his classic work related to

(a)cultural landscape.

(b)economic landscape.

(c)social landscape.

(d)physical landscape.

[UGC J-2010]

316.Which of the following set of languages is spoken in descending order'?

(a)English, Chinese, French, Russian

(b)Chinese, French, English, Russian

(c)Russian, English, French, Chinese

(d)Chinese, English, French, Russian

[UGC J-2010]

317.Given below are two statements, one labelled as Assertion (A) and the
other labelled as Reason (R). Select your answer from the codes given.
Assertion (A): Environmental determinism in geography mainly dealt
with racial distribution and the climate.

Reason (R): Environmental determinism was a philosophy of the


coloniser.

(a)Both (A) and (R) are correct and (R) is the correct explanation of (A).

(b)Both (A) and (R) are correct, but (R) is not the correct explanation of
(A).

(c)(A) is correct, but (R) is false.

(d)(A) is false, but (R) is correct.

[UGC D-2011]

318.Given below are two statements, one labelled as Assertion (A) and the
other labelled as Reason (R). Select your answer from the codes given.

Assertion (A): The locational-climatic factors put Mackinder's Heartland


at a relative disadvantage as compared to certain other larger and well-
endowed areas of the world.

Reason (R): Heartland was a region of permanent difficulties because of


its interior location and resultant extremes of climate.

(a)Both (A) and (R) are correct and (R) is the correct explanation of (A).

(b)Both (A) and (R) are correct but (R) is not the correct explanation of
(A).

(c)(A) is correct, but (R) is false.


(d)(A) is false, but (R) is correct.

[UGC D-2011]

319.Given below are two statements, one labelled as Assertion (A) and the
other labelled as Reason (R). Select your answer from the codes given.

Assertion (A): The Christian population is relatively large in the tribal hill
areas of North-Eastern India.

Reason (R): Many tribals converted to Christianity during the British rule.

(a)Both (A) and (R) are correct and (R) is the correct explanation of (A).

(b)Both (A) and (R) are correct, but (R) is not the correct explanation of
(A).

(c)(A) is correct, but (R) is false.

(d)(A) is false, but (R) is correct.

[UGC D-2011]

320.The boundary between India and Pakistan is an example of

(a)subsequent boundary.

(b)superimposed boundary.

(c)relict boundary.

(d)antecedent boundary.

[UGC D-2011]
321.Arrange the following religions in chronological order of their
emergence.

(a)Judaism, Christianity, Islam, Sikhism

(b)Sikhism, Christianity, Judaism, Islam

(c)Islam, Sikhism, Judaism, Christianity

(d)Judaism, Islam, Christianity, Sikhism

[UGC D-2011]

322.Who among the following is not associated with political geography'?

(a)F. Ratzel

(b)I. Bowman

(c)R.J. Johnston

(d)H.J. Mackinder

[UGC D-2010]

323.Match List I with List II and select the correct answer from the codes
given.
Codes:

[UGC D-2010]

324.Given below are two statements, one labelled as Assertion (A) and the
other labelled as Reason (R). Select your answer from the codes given.

Assertion (A): In modem times, artefacts are changing faster than the
mentefacts of culture.

Reason (R): Economic liberalization and modernization forces tend to


bring critical economy and cultural change.

(a)Both (A) and (R) are correct, but (R) is not the correct explanation of
(A).
(b)Both (A) and (R) are correct and (R) is the correct explanation of (A).

(c)(A) is correct, but (R) is false.

(d)(A) is false, but (R) is true.

[UGC D-2010]

325.Match List I with List II and select the correct answer using the codes
given.

Codes:

[UPSC-2006]

326.Match List I with List II and select the correct answer using the codes
given.
Codes:

[UPSC-2006]

327.Which one of the following ethnic groups DOES NOT belong to


Mongoloids?

(a)The Yakuts

(b)The Samoyeds

(c)The Red Indians of North America

(d)The Bantus

[UPSC-2006]

328.Regarding the Zone Strata Theory of Taylor, which of the following


statements is NOT correct'?
(a)Central Asia has been the cradle land of human races.

(b)Oldest human races are farthest away from the cradle land.

(c)Recent human races are nearest to the cradle land.

(d)Caucasoid are the latest human races.

[UPSC-2006]

329.Which one of the following is the speech area of `Austric Linguistic


Family"?

(a)Assam, Nagaland and Manipur

(b)Bihar, Orissa and West Bengal

(c)Tamil Nadu, Karnataka and Kerala

(d)Punjab, Himachal Pradesh and Rajasthan

[UPSC-2006]

330.Where does the ethnic group called Dayak live'?

(a)Bali

(b)Borneo

(c)Tenggara Sunda Islands

(d)West Timor

[UPSC-2006]
331.Match List I with List II and select the correct answer using the codes
given.

Codes:

[UPSC-2009]

332.Which one of the following is the country's administrative capital/new


federal administrative centre of Malaysia'?

(a)Kota Bharu

(b)Kaula Terengganu

(c)Putrajaya

(d)Taiping

[UPSC-2009]

333.Claims to the historical Macedonian territory have been a bone of


contention between which of the following countries'?
(a)Portugal and Spain

(b)Bulgaria and Greece

(c)Romania and Bulgaria

(d)Portugal and Greece

[UPSC-2006]

334.Which one of the following countries is not a member of the


Commonwealth of Independent States (CIS)?

(a)Armenia

(b)Belarus

(c)Estonia

(d)Georgia

[UPSC-2006]

335."........ They are people of yellow complexion, oblique eyes, high cheek
bones, sparse hair and medium height." The reference here is to

(a)Nordic Aryans.

(b)Austrics.

(c)Negroids.

(d)Mongoloids.

[UPSC-1997]
336.The tribal population in Andaman and Nicobar islands belongs to the

(a)Australoid race.

(b)Caucasoid race.

(c)Mongoloid race.

(d)Negroid race.

[UPSC-1997]

337.Which one of the following languages belongs to the Austric group'?

(a)Marathi

(b)Ladakhi

(c)Khasi

(d)Tamil

[UPSC-1998]

338.A person of mixed European and Indian blood in Latin America is


called a

(a)Mulatto.

(b)Mestozo.

(c)Maiji.

(d)Mau Mau.

[UPSC-1999]
339.The language spoken by the largest number of people of the world is

(a)Hindi.

(b)English.

(c)Mandarin.

(d)Spanish.

[UPSC-1999]

340.Which of the following lakes forms as international boundary between


Tanzania and Uganda'?

(a)Chained

(b)Malawi

(c)Victoria

(d)Zambanzi

[UPSC-2000]

341.Match List I with List II and select the correct answer using the codes
given.
Codes:

[UPSC-2002]

342.Berber speaking community, frequently in the news, lives in

(a)Afghanistan.

(b)Algeria.

(c)Argentina.

(d)Australia.

[UPSC-2002]

343.Consider the following statements:

1.In Macedonia, ethnic Albanians are a minority.

2.In Kosovo, Serbians are a majority.

Which of these statements is/are correct'?

(a)Only 1

(b)Only 2

(c)Both 1 and 2
(d)Neither 1 nor 2

[UPSC-2003]

344.The Basque people who are frequently in the news for their separatist
activity live in

(a)France.

(b)Italy.

(c)Portugal.

(d)Spain.

[UPSC-2003]

345.Which of the following statements is correct'?

(a)East Timor was a British colony for 4 centuries.

(b)The European colonial rulers handed over East Timor to Indonesia in


1975.

(c)The United Nations took over East Timor in 1999 to prepare it for
independence.

(d)East Timor finally declared its independence in December 2001.

[UPSC-2003]

346.Which of the following languages has the largest number of speakers in


the world?

(a)Bengali
(b)French

(c)Japanese

(d)Portuguese

[UPSC-2003]

347.In which one of the following countries, did an ethnic violence between
the communities of Hema and Lendu result in the death of hundreds of
people?

(a)Democratic Republic of Congo

(b)Indonesia

(c)Nigeria

(d)Zambia

[UPSC-2004]

348.Consider the following international languages:

1.Arabic

2.French

3.Spanish

The correct sequence of the language given above in the decreasing order
of the number of their speakers is

(a)3, 1, 2.
(b)1, 3, 2.

(c)3, 2, 1.

(d)1, 2, 3.

[UPSC-2004]

349.In which one of the following countries is Tamil a major language'?

(a)Myanmar

(b)Indonesia

(c)Mauritius

(d)Singapore

[UPSC-2005]

350.For which one of the following countries is Spanish not an official


language'?

(a)Chile

(b)Colombia

(c)Republic of Congo

(d)Cuba

[UPSC-2005]

ANSWER KEYS
 
1.The concept of sequent occupance was advocated by

(a)H.H. Barrows.

(b)C.D. Harris.

(c)D.S. Whittlesey.

(d)Richard Hartshorne.

[UGC D-2008]

2.Regional planning aims at

(a)development of agriculture.

(b)balanced development.

(c)development of heavy industries.

(d)development of small-scale industries.

[UGC D-2008]

3.The concept of countermagnet is associated with

(a)Western Ghats plan.

(b)South-East resource region.

(c)Rayalseema plan.
(d)National Capital Region Plan.

[UGC D-2008]

4.Regional diversity is essentially caused by

(a)per capita income.

(b)natural resource base.

(c)industrial development.

(d)levels of urbanization.

[UGC D-2009]

5.Given below are two statements, one labelled as Assertion (A) and the
other is labelled as Reason (R). Select your answer from the codes given.

Assertion (A): The growth pole strategy failed to bring balanced regional
development in India.

Reason (R): Diffusion of development under conditions of general


regional backwardness is weak.

(a)Both (A) and (R) are correct and (R) explains (A).

(b)Both (A) and (R) are correct but (R) does not explain (A).

(c)(A) is correct but (R) is wrong.

(d)(A) is wrong but (R) is correct.

[UGC D-2009]
6.Patrick Geddes applied which of the following perspectives in regional
planning?

(a)Technological

(b)Socioeconomic

(c)Environmental

(d)Historical-ecological

[UGC D-2009]

7.Concern with regional disparities in development was explicitly


mentioned in which Five Year Plan of India'?

(a)Second

(b)Fourth

(c)Seventh

(d)Tenth

[UGC D-2009]

8.Which of the following is a direct indicator of regional development?

(a)Hospital beds per 1,000 population

(b)Length of roads per 1,000 square kilometre

(c)Percapita consumption of steel

(d)Per capita income


[UGC J-2005]

9.Which one of the following methods is used for identifying regions with
variations in levels of development?

(a)Boundary girdles

(b)Lorenze curve

(c)Principal component analysis

(d)Nearest neighbour analysis

[UGC J-2005]

10.Which one of the five year plans for economic development of India
elaborated thepolicy on minimizing regional disparities in development?

(a)Fourth Five Year Plan

(b)Sixth Five Year Plan

(c)Third Five Year Plan

(d)Fifth Five Year Plan

[UGC J-2005]

11.Which of the following is not a planned city?

(a)Chandigarh

(b)Bhubaneswar

(c)Jaipur
(d)Lucknow

[UGC J-2005]

12.The following states were carved out after 1960. Arrange them in
ascending chronological order of their creation.

1.Haryana

2.Sikkim

3.Nagaland

4.Meghalaya

(a)1,2, 3, 4

(b)2, 3, 4, 1

(c)2, 4, 1, 3

(d)3, 1, 4, 2

[UGC J-2005]

13.In the delineation of physiographic regions of India, the terms macro,


meso and micro were introduced by

(a)R.L. Singh.

(b)M.B. Pithawala.

(c)O.H.K. Spate.

(d)S.P. Chatterjee.
[UGC J-2005]

14.Which one of the following indicators is a direct measure of regional


imbalance in development?

(a)Percentage of cultivated area to total geographical area

(b)Percapita consumer expenditure

(c)Accessibility to roads

(d)Literacy

[UGC J-2007]

15.River basin is a

(a)microregion.

(b)macroregion.

(c)hilly region.

(d)backward region.

[UGC J-2008]

16.Which one of the following is a nodal region?

(a)Industrial region

(b)Tribal region

(c)Metropolitan region
(d)Central district in a state

[UGC J-2008]

17.Forward linkages facilitate

(a)diversification.

(b)unbalanced development.

(c)excessive pressure on resource.

(d)social development.

[UGC J-2008]

18.A functional region is delineated on the basis of

(a)politico-administrative boundaries.

(b)isolines.

(c)physical division.

(d)fields of interactions.

[UGC J-2011]

19.Who developed a methodology for the study of human communities


through focused attention on the `Place-WorkFolk' progression?

(a)T.H. Huxley

(b)Patrick Geddes
(c)H.R. Mill

(d)Vidal de la Blache

[UGC J-2011]

20.The correct sequence of planners in the Anglo-American tradition is

(a)Unwin, Perry, Howard, Geddes.

(b)Geddes, Howard, Perry, Unwin.

(c)Howard, Geddes, Unwin, Perry.

(d)Perry, Geddes, Unwin, Howard.

[UGC J-2011]

21.Regions delineated on the basis of relationship are called

(a)natural regions.

(b)nodal regions.

(c)planning regions.

(d)developing regions.

[UGC J-2010]

22.Improving human welfare without causing damage to the environment is


called

(a)regional development.
(b)regional planning.

(c)sustainable development.

(d)resource analysis.

[UGC J-2010]

23.Territorial production complex is associated with

(a)Indian model of development.

(b)Brazilian model of development.

(c)Korean model of development.

(d)Russian model of development.

[UGC D-2011]

24.This region has a core area and a periphery.

(a)Formal region

(b)Perceptual region

(c)Functional region

(d)Vernacular region

[UGC D-2011]

25.Who laid the philosophical basis of regional planning'?

(a)Benton MacKaye
(b)Fredric le Play

(c)F.S. Chapin

(d)Carol Aronovinci

[UGC D-2011]

26.In India, the state is used as a national planning unit because

(a)it is an administrative unit.

(b)it is a homogeneous unit.

(c)of political considerations.

(d)of uniform physical features.

[UGC D-2010]

27.The control or direction of resource development is called

(a)resource utilization.

(b)resource allocation.

(c)resource management.

(d)resource availability.

[UGC D-2010]

28.The hierarchical (also known as cascade) diffusion pattern by which


HIV/AIDS spread across the world is characterized by moving across the
world's
(a)lowest income countries.

(b)island countries.

(c)most popular tourist areas.

(d)major urban areas.

29.Regional geography is

(a)the theoretical side of cartography.

(b)the part of human geography that emphasizes landscapes.

(c)a subcategory of physical geography.

(d)combination of elements from both human and physical geography.

30.Which of the following is most likely to be found around the outer parts
of

aculture region as modelled by geographer Donald Meinig?

(a)Symbolic landscapes

(b)Core

(c)Cultural hearth

(d)Sphere

31.are interdependent.

(a)Regions
(b)People

(c)Spaces

(d)All of the above

32.Of the following, the best example of a functional region is

(a)the Corn Belt.

(b)Chicago.

(c)the West Coast.

(d)North America.

33.In an example of global interdependence among places and regions,


which of the following is NOT an explanation for rising food prices
across the globe'?

(a)Increased price of energy

(b)Increased consumption of meat

(c)Booming economies of China and India

(d)Increased interest in local foods in the USA

34.Understanding the first law of geography, that `Everything is related to


everything else, but near things are more related than are distant things',
is to recognize the concept of

(a)time-space convergence.
(b)situation.

(c)friction of distance.

(d)regionalism.

35.Which of the following regions is also known as perceptual region?

(a)Vernacular region

(b)Nodal region

(c)Formal region

(d)Functional region

36.Look at the following characteristics:

1.A spatial unit with focus on some kind of activity

2.The activity is most intense at the centre

3.The activity is less intense at the borders

These are the characteristics of

(a)formal region.

(b)nodal region.

(c)perceptual region.

(d)none of these.

37.Look at the following characteristics:


1.This region exists in minds of many people.

2.It may play an important role in cultural identity.

3.It does not have clear-cut or official borders.

These are the characteristics of

(a)uniform region.

(b)functional region.

(c)vernacular region.

(d)none of these.

38.The Gross Domestic Product (GDP) of a region or country includes the


total value (in a given year) of all of the following except which?

(a)Services produced in the country

(b)Machinery produced in the country

(c)Foodstuffs produced in the country

(d)Profits from abroad (overseas)

39.In the context of economic development over the last 30 years, regional
inequality across the globe

(a)is not evident in core countries.

(b)has declined with increased globalization.


(c)has not worsened in countries like the USA and Scandinavia.

(d)has increased with increasing globalization.

40.A country's ecological footprint is calculated from all of the following


except which one?

(a)Size of the population

(b)The affluence of the populationcalculated in terms of how much they


consume

(c)The technologies being usedmeasured in terms of the intensity of


resource use

(d)The education of the population

ANSWER KEYS

 
1.`Sal' trees are concentrated in

(a)Tamil Nadu.

(b)Assam.

(c)Bihar.

(d)Madhya Pradesh.

[UPSC-1996]

2.A mean monthly temperature of about 27°C, relatively high humidity and
a rainfall of 175-200 cm during spring and summer are the physical
requirements during the period of its vegetative growth. Which one of the
following crops is referred to in the question'?

(a)Rubber

(b)Jute

(c)Sugarcane

(d)Tea

[UPSC-1995]

3.Indian has the largest population of the Asian X.Today, there are just about
20,000 to 25,00. X in their natural habitat spreading across the evergreen
forests, dry thorn forest, swamps and grasslands. Their prime habitats are
however the moist deciduous forests. The X population in Indian ranges
from northwest India where they are found in the forest divisions of
Dehradun, Bijnor and national districts of UP to the Western Ghats in the
states of Karnataka and Kerala and in Tamil Nadu. In central India, their
population is distributed in southern Bihar and Orissa. In the east, they are
seen in north Bengal, Assam and a few other states. The animal X
referred to in the question is

(a)Lion

(b)Elephant

(c)Tiger

(d)One-homed rhinoceros

[UPSC-1999]

4.You might see a few curious Danes around, but that is because this used to
be a Danish outpost. This quaint town with itsfort and a beautiful church,
the New Jerusalem, empty streets and deserted beachfront is a quaint
gem. The place referred to in lies on the

(a)Tamil Nadu coast.

(b)Kerala coast.

(c)Karnataka coast.

(d)Goa coast.

[UPSC-1996]

5.A person over flying India saw the natural vegetation below in the
sequential order of tropical evergreen forest, savannah, dry deciduous and
deciduous forests. His flight was from

(a)Calcutta to Delhi.

(b)Bombay to Bhubaneswar.

(c)Trivandrum to Delhi.

(d)Delhi to Madurai.

[UPSC-1995]

6.Among the following cities, which one is nearest to the Tropic of Cancer'?

(a)Delhi

(b)Kolkata

(c)Jodhpur

(d)Nagpur

[UPSC-2003]

7.Among the following cities, which one has the highest altitude above
mean sea level'?

(a)Bangalore

(b)Delhi

(c)Jodhpur

(d)Nagpur

[UPSC-2003]
8.Among the following who are the Agaria community'?

(a)A traditional today community of Andhra Pradesh

(b)A traditional fishing community of Maharashtra

(c)A traditional silk-weaving community of Karnataka

(d)A traditional salt pan workers community of Gujarat

[UPSC-2009]

9.Among which one of the following sets of social/religious groups is the


extent of poverty the highest, as per Government statistics for the 1990s?

(a)Muslims in Kerala, Gujarat and Andhra Pradesh

(b)Tribals in Bihar, Orissa, MP and Maharashtra

(c)Scheduled castesin Punjab, western UP, northern Rajasthan and Tamil


Nadu

(d)Christians in Gujarat, Maharashtra and Assam

[UPSC-1999]

10.Amongst the following Indian states which one has the minimum total
forest cover?

(a)Sikkim

(b)Goa

(c)Haryana
(d)Kerala

[UPSC-2004]

11.Assertion (A): Assam has the largest area under canal irrigation in India.

Reason (R): The Assam valley is flat and the Brahmaputra carries
enormous volume of water.

(a)Both (A) and (R) are true and (R) is the correct explanation of (A).

(b)Both (A) and (R) are true but (R) is not a correct explanation of (A).

(c)(A) is true but (R) is false.

(d)(A) is false but (R)is true.

[UPSC-1997]

12.Assertion (A): Chemical industries have reached a high level of


development in the Hooghly-Calcutta belt.

Reason (R): Cheap industrial labour force is readily available in this belt.

(a)Both (A) and (R) are true and (R) is the correct explanation of (A).

(b)Both (A) and (R) are true but (R) is not a correct explanation of (A).

(c)(A) is true but (R) is false.

(d)(A) is false but (R) is true.

[UPSC-1996]
13.Assertion (A): In India, fishing occupies a very important place in its
economy.

Reason(R): The production of fish in India has shown appreciable


increase after Independence.

(a)Both (A) and (R) are true and (R) is the correct explanation of (A).

(b)Both (A) and (R) are true but (R) is not a correct explanation of (A).

(c)(A) is true but (R) is false.

(d)(A) is false but (R) is true.

[UPSC-1996]

14.Assertion (A): In India, per hectare production of sugarcane is the highest


in the world.

Reason (R): India has about one-third of the world's sugarcane area.

(a)Both (A) and (R) are true and (R) is the correct explanation of (A).

(b)Both (A) and (R) are true but (R) is not a correct explanation of (A).

(c)(A) is true but (R) is false.

(d)(A) is false but (R) is true.

[UPSC-1996]

15.Assertion (A): Indian agriculture is no longer the gamble of the


monsoons.

Reason (R): There is a recent expansion of dry farming in India.


(a)Both (A) and (R) are true and (R) is the correct explanation of (A).

(b)Both (A) and (R) are true but (R) is not a correct explanation of (A).

(c)(A) is true but (R) is false.

(d)(A) is false but (R) is true.

[UPSC-1997]

16.Assertion (A): Indira Gandhi Canal irrigates large areas of west


Rajasthan.

Reason (R): It is fed by the Ravi River in Rajasthan.

(a)Both (A) and (R) are true and (R) is the correct explanation of (A).

(b)Both (A) and (R) are true but (R) is not a correct explanation of (A).

(c)(A) is true but (R) is false.

(d)(A) is false but (R) is true.

[UPSC-1998]

17.Assertion (A): Irrigation is an essential input to revitalize Indian


agriculture.

Reason (R): Two thirds of the farmlands of India are located in semi-arid
regions.

(a)Both (A) and (R) are true and (R) is the correct explanation of (A).

(b)Both (A) and (R) are true but (R) is not a correct explanation of (A).
(c)(A) is true but (R) is false.

(d)(A) is false but (R) is true.

[UPSC-1995]

18.Assertion (A): Major industrial regions in India developed in the


immediate hinterlands of the ports of Kolkata, Mumbai and Chennai.

Reason (R): The ports provide access to the raw materials available in the
hinterlands of these ports as well as to the world market.

(a)Both (A) and (R) are true and (R) is the correct explanation of (A).

(b)Both (A) and (R) are true but (R) is not a correct explanation of (A).

(c)(A) is true but (R) is false.

(d)(A) is false but (R) is true.

[UPSC-2003]

19.Assertion (A): Mawsynram is the wettest place in India.

Reason (R): It receives heavy rainfall throughout the year.

(a)Both (A) and (R) are true and (R) is the correct explanation of (A).

(b)Both (A) and (R) are true but (R) is not a correct explanation of (A).

(c)(A) is true but (R) is false.

(d)(A) is false but (R) is true.

[UPSC-1998]
20.Assertion (A): Most cyclones hit the eastern coast of India than the
western coast.

Reason (R): The eastern coast of India lies in the path of the north-east
trade wind.

(a)Both (A) and (R) are true and (R) is the correct explanation of (A).

(b)Both (A) and (R) are true but (R) is not a correct explanation of (A).

(c)(A) is true but (R) is false.

(d)(A) is false but (R) is true.

[UPSC-1997]

21.Assertion (A): Offshore fishing in Malabar coast is more developed than


that in the Coromandel coast.

Reason (R): Malabar coast has better facilities to export sea fish to
African countries.

(a)Both (A) and (R) are true and (R) is the correct explanation of (A).

(b)Both (A) and (R) are true but (R) is not a correct explanation of (A).

(c)(A) is true but (R) is false.

(d)(A) is false but (R) is true.

[UPSC-1995]

22.Assertion (A): Peninsular India is a region free from tectonic


disturbances.
Reason (R): The configuration and drainage of Peninsular India are
influenced by

(a)Both (A) and (R) are true and (R) is the correct explanation of (A).

(b)Both (A) and (R) are true but (R) is not a correct explanation of (A).

(c)(A) is true but (R) is false.

(d)(A) is false but (R) is true.

[UPSC-2002]

23.Assertion (A): Production of natural gas in India has increased in the


recent years.

Reason (R): India has a high reserve of natural gas.

(a)Both (A) and (R) are true and (R) is the correct explanation of (A).

(b)Both (A) and (R) are true but (R) is not a correct explanation of (A).

(c)(A) is true but (R) is false.

(d)(A) is false but (R) is true.

[UPSC-1997]

24.Assertion (A): Rihand power plant is located where rainfall in limited and
seasonal.

Reason (R): Hydel power plants are found on the hills with heavy rainfall.

(a)Both (A) and (R) are true and (R) is the correct explanation of (A).
(b)Both (A) and (R) are true but (R) is not a correct explanation of (A).

(c)(A) is true but (R) is false.

(d)(A) is false but (R) is true.

[UPSC-1995]

25.Assertion (A): River Kalindi is an eastflowing river in the southern part


of India.

Reason (R): The Deccan Plateau is higher along its western edge and
gently slopes towards the Bay of Bengal in the east.

(a)Both (A) and (R) are true and (R) is the correct explanation of (A).

(b)Both (A) and (R) are true but (R) is not a correct explanation of (A).

(c)(A) is true but (R) is false.

(d)(A) is false but (R) is true.

[UPSC-2007]

26.Assertion (A): Tank irrigation is mostly practiced in Peninsular India


including Maharashtra and Gujarat regions.

Reason (R): Many streams in Deccan Plateau region experience torrential


rainy season but dry up when the rains cease.

(a)Both (A) and (R) are true and (R) is the correct explanation of (A).

(b)Both (A) and (R) are true but (R) is not a correct explanation of (A).

(c)(A) is true but (R) is false.


(d)(A) is false but (R) is true.

[UPSC-1999]

27.Assertion (A): The desert soils of India are poor in mineral nutrients and
hence infertile.

Reason (R): The dearth of rainfall and humus does not allow good soil
formation.

(a)Both (A) and (R) are true and (R) is the correct explanation of (A).

(b)Both (A) and (R) are true but (R) is not a correct explanation of (A).

(c)(A) is true but (R) is false.

(d)(A) is false but (R) is true.

[UPSC-1996]

28.Assertion (A): The Himalayan meadows are suitable for transhumance.

Reason (R): In these areas, grasses grow in abundance during summer.

(a)Both (A) and (R) are true and (R) is the correct explanation of (A).

(b)Both (A) and (R) are true but (R) is not a correct explanation of (A).

(c)(A) is true but (R) is false.

(d)(A) is false but (R) is true.

[UPSC-1998]
29.Assertion (A): The hydel power plants in Maharashtra are operated by the
rivers coming down from the Western Ghats.

Reason (R): Most of these rivers are rain-fed.

(a)Both (A) and (R) are true and (R) is the correct explanation of (A).

(b)Both (A) and (R) are true but (R) is not a correct explanation of (A).

(c)(A) is true but (R) is false.

(d)(A) is false but (R) is true.

[UPSC-1998]

30.Assertion (A): The once prosperous Hooghly-side Industrial Complex is


now showing signs of decay.

Reason (R): The decline is mainly because of the silting of Hooghly


channel.

(a)Both (A) and (R) are true and (R) is the correct explanation of (A).

(b)Both (A) and (R) are true but (R) is not a correct explanation of (A).

(c)(A) is true but (R) is false.

(d)(A) is false but (R) is true.

[UPSC-1998]

31.Assertion (A): The percentage of net sown area in the total area of
Andhra Pradesh is less as compared to that of West Bengal.

Reason (R): The soil of most of Andhra Pradesh is laterite.


(a)Both (A) and (R) are true and (R) is the correct explanation of (A).

(b)Both (A) and (R) are true but (R) is not a correct explanation of (A).

(c)(A) is true but (R) is false.

(d)(A) is false but (R) is true.

[UPSC-2006]

32.Assertion (A): The tarai soils in India are rich in nitrogen and organic
matter but are deficient in phosphate.

Reason (R): These soils are generally covered by tall grass and forests.

(a)Both (A) and (R) are true and (R) is the correct explanation of (A).

(b)Both (A) and (R) are true but (R) is not a correct explanation of (A).

(c)(A) is true but (R) is false.

(d)(A) is false but (R) is true.

[UPSC-2003]

36.Assertion (A): Ahmedabad is a major centre of sick cotton textile


industry.

Reason (R): Cotton required by the industry is grown on large scale in the
black soil around it.

(a)Both (A) and (R) are true and (R) is the correct explanation of (A).

(b)Both (A) and (R) are true but (R) is not a correct explanation of (A).
(c)(A) is true but (R) is false.

(d)(A) is false but (R) is true.

[UPSC-1999]

33.Assertion (A): The Western disturbance causes winter rain in north-west


India.

Reason (R): The south-west monsoon starts retreating from north-west


India during winter.

(a)Both (A) and (R) are true and (R) is the correct explanation of (A).

(b)Both (A) and (R) are true but (R) is not a correct explanation of (A).

(c)(A) is true but (R) is false.

(d)(A) is false but (R) is true.

[UPSC-1997]

37.Assertion (A): Bangalore receives much higher average annual rainfall


than Mangalore.

Reason (R): Bangalore has the benefit of receiving rainfall both from
south-west and north-east monsoons.

(a)Both (A) and (R) are true and (R) is the correct explanation of (A).

(b)Both (A) and (R) are true but (R) is not a correct explanation of (A).

(c)(A) is true but (R) is false.

(d)(A) is false but (R) is true.


[UPSC-2004]

34.Assertion (A): Tropical seas in India are rich in fish species but there are
few important fishing grounds around the country.

Reason (R): The quantity of fish of a particular species is lacking and


hence not commercially exploitable.

(a)Both (A) and (R) are true and (R) is the correct explanation of (A).

(b)Both (A) and (R) are true but (R) is not a correct explanation of (A).

(c)(A) is true but (R) is false.

(d)(A) is false but (R) is true.

[UPSC-2001]

38.Assertion (A): Ganga Plain is the most densely populated part of India.

Reason (R): Ganga is the most harnessed river of India.

(a)Both (A) and (R) are true and (R) is the correct explanation of (A).

(b)Both (A) and (R) are true but (R) is not a correct explanation of (A).

(c)(A) is true but (R) is false.

(d)(A) is false but (R) is true.

[UPSC-2000]

35.Assertion (A): According to statistics, more female children are born


each year than male children in India.
Reason (R): In India, the death rate of a male child is higher than that of
the female child.

(a)Both (A) and (R) are true and (R) is the correct explanation of (A).

(b)Both (A) and (R) are true but (R) is not a correct explanation of (A).

(c)(A) is true but (R) is false.

(d)(A) is false but (R) is true.

[UPSC-1999]

39.Assertion (A): In winters, the northern half of India is warmer than areas
at similar latitudes outside the country by 3°C to 8°C.

Reason (R): The presence of the Himalayas with their east and west
extent is the cause.

(a)Both (A) and (R) are true and (R) is the correct explanation of (A).

(b)Both (A) and (R) are true but (R) is not a correct explanation of (A).

(c)(A) is true but (R) is false.

(d)(A) is false but (R) is true.

[UPSC-2002]

40.Assertion (A): India does not sport natural rubber.

Reason (R): About 97% of India's demand for natural rubber is met from
domestic production.

(a)Both (A) and (R) are true and (R) is the correct explanation of (A).
(b)Both (A) and (R) are true but (R) is not a correct explanation of (A).

(c)(A) is true but (R) is false.

(d)(A) is false but (R) is true.

[UPSC-2004]

41.Assertion (A): The eastern coast of India produces more rice than the
western coast.

Reason (R): The eastern coast receives more rainfall than the western
coast.

(a)Both (A) and (R) are true and (R) is the correct explanation of (A).

(b)Both (A) and (R) are true but (R) is not a correct explanation of (A).

(c)(A) is true but (R) is false.

(d)(A) is false but (R) is true.

[UPSC-2003]

42.Assertion (A): The frequency of floods in North Indian plains has


increased during the last couple of decades.

Reason (R): There has been a reduction in the depth of river valleys due
to deposition of silt.

(a)Both (A) and (R) are true and (R) is the correct explanation of (A).

(b)Both (A) and (R) are true but (R) is not a correct explanation of (A).

(c)(A) is true but (R) is false.


(d)(A) is false but (R) is true.

[UPSC-2000]

43.Assertion (A): West-flowing rivers of Peninsular India have no deltas.

Reason (R): These rivers do not carry any alluvial sediments.

(a)Both (A) and (R) are true and (R) is the correct explanation of (A).

(b)Both (A) and (R) are true but (R) is not a correct explanation of (A).

(c)(A) is true but (R) is false.

(d)(A) is false but (R) is true.

[UPSC-2004]

44.At which one of the following places do two important rivers of India
originate; while one of them flows towards north and merges with another
important river flowing towards Bay of Bengal and the other one flows
towards Arabian Sea?

(a)Amarkantak

(b)Badrinath

(c)Mahabaleshwar

(d)Nasik

[UPSC-2009]

45.Badland topography is characteristic of the


(a)Gulf of Kutch.

(b)Sunderban delta.

(c)Konkan coast.

(d)Chambal valley.

[UPSC-1998]

46.Bhabar belt is found in

(a)Chhota Nagpur plateau.

(b)Himalayan piedmont zone.

(c)Western Ghats.

(d)coastal Orissa.

[UPSC-2002]

47.Canal irrigation is most important in the northern plains of India because

(a)the soil is porous.

(b)the level of underground water is high.

(c)the sources of canals are perennial rivers.

(d)the region is densely populated.

[UPSC-2004]

48.Chapchar Kut is a festival celebrated in the state of


(a)Arunachal Pradesh.

(b)Assam.

(c)Mizoram.

(d)Sikkim.

[UPSC-2002]

49.Consider the following statements:

1.Petronet LNG Ltd. is setting up another LNG terminal at Mangalore.

2.The Head Office of the Dredging Corporation of India is at


Visakhapatnam.

3. The Narwapahar mine is operated by the Uranium Corporation of India


Limited.

Which of the statement given above are correct?

(a)1, 2 and 3

(b)1 and 2 only

(c)2 and 3 only

(d)1 and 3 only

[UPSC-2006]

50.Consider the following about the soils of India:

1.High temperature during most of the year retards application of humus.


2.Soils which have humus require regular application of chemical
fertilizers to remove nitrogen deficiency.

3.Titanium salts in basalt impart black colour to the soils of the Indian
plateau.

4.Red soils are confined to the peripheral areas of the Indian plateau.

Which of the statements given above are correct'?

(a)1 and 2

(b)1, 3 and 4

(c)2 and 3

(d)2 and 4

[UPSC-2004]

51.Consider the following export items:

1.Textiles

2.Agricultural and allied products

3.Chemical and related products

4.Gems and jewellery

The correct sequence in descending order of these items in terms of value


of exports from India as per 1997-98 figures is

(a)1, 4, 3, 2.
(b)1, 4, 2, 3.

(c)4, 1, 2, 3.

(d)4, 1, 3, 2.

[UPSC-2000]

52.Consider the following measures:

1.Creation of mass awareness on the need for conservation of petroleum

2.Promotion of measures to curb wasteful practices

3.Reducing the number of vehicles and engines using petrol

4.Improving the oil use efficiency of equipment, devices and vehicles

5.Research and development for improving oil use efficiency.

The measures adopted by the Petroleum Conservation Research


Association of Indian to conserve petroleum include

(a)1, 2 and 3.

(b)2, 3, 4 and 5.

(c)1, 3, 4 and 5.

(d)1,2,4and5.

[UPSC-1996]

53.Consider the following pairs:


Which of the pairs given above is/are correctly matched'?

(a)1, 2 and 3

(b)1 and 2

(c)2 and 3

(d)3 only

[UPSC-2008]

54.Consider the following pairs of places and types of natural hazards faced
by them:

1.Latur: Riverine flooding

2.Nashri: Landslide

3.Kandla: Cyclonic storm

Which of the above pairs are correctly matched'?

(a)1 and 2

(b)2 and 3

(c)1 and 3

(d)1, 2 and 3
[UPSC-2000]

55.Consider the following peaks:

1.Dhaulgiri

2.Kanchenjunga

3.Nanda Devi

4.Kamet Peak

The correct sequence of the peaks from west to east will be

(a)3, 1, 4, 2.

(b)3, 4, 1, 2.

(c)1, 3, 2, 4.

(d)4, 3, 1, 2.

[UPSC-1995]

56.Consider the following places:

1.Ghaziabad

2.Siliguri

3.Pune

4.Rourkela

Place(s) better suited for the location of cement industry would include
(a)1 and 4.

(b)3 and 4.

(c)3 alone.

(d)4 alone.

[UPSC-1997]

57.Consider the following principal crops of India:

1.Pulses

2.Fibre crops

3.Oil seeds

4.Plantation crops

The correct sequence in descending order principal group of crops with


respect to the share in the total cropped area is

(a)1, 2, 3, 4.

(b)1, 3, 4, 2.

(c)3, 1, 2, 4.

(d)3, 1, 4, 2.

[UPSC-1998]

58.Consider the following programmes:


1.Afforestation and development of wastelands

2.Reforestation and replantation in existing forests

3.Encouraging the wood substitutes and supplying other types of fuel

4.Promotion of wide use of insecticides and pesticides to restrict the loss


of forest area from degradation caused by pests and insects

The National Forest Policy of 1988 includes

(a)1, 2, 3 and 4.

(b)2 and 4.

(c)1, 3 and 4.

(d)1, 2 and 3.

[UPSC-1996]

59.Consider the following regions:

1. Gulf of Khambhat

2.Gulf of Kutch

3.Sunderbans

The correct sequence of these regions in the descending order of their


tidal energy potential is

(a)1, 2, 3.

(b)3, 2, 1.
(c)3, 1, 2.

(d)1, 3, 2.

[UPSC-2003]

60.Consider the following relief features:

1.Zaskar Range

2.Dhauladhar Range

3.Ladakh Range

4.Karakoram Range

The correct northward sequence of these relief features is

(a)2, 1, 3, 4.

(b)2, 3, 4, 1.

(c)4, 3, 2, 1.

(d)4, 2, 1, 3.

[UPSC-2002]

61.Consider the following rivers:

1.Kishenganga

2.Ganga

3.Wainganga
4.Penganga

The correct sequence of these rivers when arranged in the north to south
direction is

(a)1, 2, 3, 4.

(b)2, 1, 3, 4.

(c)2, 1, 4, 3.

(d)1,2,4,3.

[UPSC-1996]

62.Consider the following sites/monuments:

1.Champaner-Pavagadh Archaeological Park

2.Chhatrapati Shivaji Railway Station, Mumbai

3.Mamallapuram

4.Sun Temple (Konark Temple)

Which of the above are included in the World Heritage List of UNESCO'?

(a)1, 2 and 3

(b)1, 3 and 4

(c)2 and 4

(d)1, 2, 3 and 4
[UPSC-2005]

63.Consider the following statements about the megacities of India:

1.Population of each megacity is more than 5 million.

2.All the megacities are important sea ports.

3.Megacities are either national or state capitals.

Which of these statements are correct?

(a)1, 2 and 3

(b)1 and 2

(c)2 and 3

(d)1 and 3

[UPSC-2000]

64.Consider the following statements associated with the Deccan Traps:

1.From the Cretaceous to the Eocene, intense volcanic activity took place
in Peninsular India.

2.The volcanic lava spread out in horizontal sheets, filling the


irregularities of the pre-existing topography.

3.The trap country is characterized by conical hills and extensive flat


surfaces.

Of these statements
(a)1 and 2 are correct.

(b)1, 2 and 3 are correct.

(c)2 and 3 are correct.

(d)1 and 3 are correct.

[UPSC-1996]

65.Consider the following statements concerned with the bajra cultivation in


India:

1.Bajra occupies about 11% of area under food grains.

2.Rajasthan is the leading producer of bajra in India.

3.India exports a large quantity of bajra.

Of these statements

(a)1 and 2 are correct.

(b)1, 2, and 3 are correct.

(c)2 and 3 are correct.

(d)1 and 3 are correct.

[UPSC-1996]

66.Consider the following statements concerning the Indian Railways:

1.The Headquarters of the NorthWestern Railway are located at Jodhpur.


2.Indrail pass - a travel-as-you-please ticket has been created especially
for freedom fighters and sportspersons who have represented India in
any games/sport.

3.Fairy Queen is a train using the world's oldest working engine and the
Indian Railways conducts a journey of wildlife and heritage sites on it.

Which of the statements give above is/ are correct'?

(a)2 only

(b)3 only

(c)1 and 2

(d)None

[UPSC-2005]

67.Consider the following statements with respect to India's advanced


satellite INSAT-4A:

1.INSAT-4A was launched in December 2005 from New Mexico.

2.The European Commercial Launch Service Provider Arianespace was


associated with the launch of INSAT-4A.

3.Tata-DTH television broadcasting service is from INSAT-4A.

Which of the statements given above are correct'?

(a)1, 2 and 3

(b)1 and 2 only


(c)2 and 3 only

(d)1 and 3 only

[UPSC-2006]

68.Consider the following statements regarding environment issues of India:

1.Gulf of Mannar is one of the biosphere reserves.

2.The Ganga Action Plan Phase II has been merged with the National
River Conservation Plan.

3.The National Museum of Natural History at New Delhi imparts


nonformal education in environment and conservation.

4.Environmental Information System (ENVIS) acts as a decentralized


information network for environment information.

Which of these statements are correct'?

(a)1, 2 and 4

(b)1, 2, 3 and 4

(c)2 and 3

(d)1, 3 and 4

[UPSC-2001]

69.Consider the following statements regarding power sector in India:

1.The installed capacity of power generation is around 95,00. MW.


2.Nuclear plants contribute nearly 15% of total power generation.

3.Hydroelectricity plants contribute nearly 40% of total power generation.

4.Thermal plants at present account for nearly 80% of total power


generation.

Which of the statements is/are correct?

(a)1 only

(b)2 and 3

(c)3 and 4

(d)1 and 4

[UPSC-2001]

70.Consider the following statements:

1.Maharashtra has the highest acreage under jowar in India.

2.Gujarat is the largest producer of groundnut in India.

3.Rajasthan has the largest area of cultivable wastelands in India.

4.Andhra Pradesh has the highest per hectare yield of maize in India.

Which of these statements are correct?

(a)1 and 4

(b)2 and 3
(c)1 and 3

(d)2 and 4

[UPSC-2000]

71.Consider the following statements:

1.NTPC is the largest power utility in India.

2.ONGC accounts for half of the LPG production of India.

3.Indian Oil Corporation operators all the refineries in India.

4.The Indian Ordinance Factory is the largest departmentally run


industrial undertaking in the country.

Which of these statements is/are correct'?

(a)1 only

(b)2 and 3

(c)2, 3 and 4

(d)1 and 4

[UPSC-2002]

72. Consider the following statements:

1.India launched its first full-fledged meteorological satellite (METSAT)


in September 2002.
2.For the first time, the space vehicle PSLV-C-4 carried a payload of more
than 1,000 kg into a geosynchronous orbit.

Which of these statement is/are correct

(a)Only 1

(b)Only 2

(c)Both 1 and 2

(d)Neither 1 nor 2

[UPSC-2003]

73.Consider the following statements:

1.India is the original home of the cotton plant.

2.India is the first country in the world to develop hybrid cotton variety
leading to increased production.

Which of these statements is/are correct?

(a)Only 1

(b)Only 2

(c)Both 1 and 2

(d)Neither 1 nor 2

[UPSC-2003]

74.Consider the following statements:


1.Longitude of Jabalpur's location is between those of Indore and Bhopal.

2.Latitude of Aurangabad's location is between those of Vadodara and


Pune.

3.Bangalore is situated more southward than Chennai.

Which of these statements is/are correct'?

(a)1 and 3

(b)Only 2

(c)2 and 3

(d)1, 2 and 3

[UPSC-2003]

75.Consider the following statements:

1.India ranks first in the world in fruit production.

2.India ranks second in the world in the export of tobacco.

Which of these statements is/are correct?

(a)Only 1

(b)Only 2

(c)Both 1 and 2

(d)Neither 1 nor 2
[UPSC-2003]

76.Consider the following statements:

1.National Thermal Power Corporation has diversified into hydropower


sector.

2.Power Grid Corporation of India has diversified into telecom sector.

Which of the statements given above is/ are correct'?

(a)1 only

(b)2 only

(c)Both 1 and 2

(d)Neither 1 nor 2

[UPSC-2004]

77.Consider the following statements:

1.Damodar Valley Corporation is the first multipurpose river valley


project of independent India.

2.Damodar Valley Corporation includes thermal and gas power stations.

Which of the statements given above is/ are correct?

(a)1 only

(b)2 only

(c)Both 1 and 2
(d)Neither 1 nor 2

[UPSC-2004]

78.Consider the following statements: Among the Indian states

1.Andhra Pradesh has the longest coastline.

2.Gujarat has the highest number of airports.

Which of the statement given above is/ are correct'?

(a)1 only

(b)2 only

(c)Both 1 and 2

(d)Neither 1 nor 2

[UPSC-2004]

79.Consider the following statements:

1.Indira Gandhi Centre for Atomic Research uses fast reactor technology.

2.Atomic Minerals Directorate for Research and Exploration is engaged


in heavy water production.

3.Indian Rare Earths Limited is engaged in the manufacture of zircon for


India's Nuclear Programme beside other rare Earth products.

Which of the statements given above are correct'?

(a)1, 2 and 3
(b)1 and 2

(c)1 and 3

(d)2 and 3

[UPSC-2004]

80.Consider the following statements:

1.The forest cover in India constitutes around 20% of its geographical


area. Out of the total forest cover, dense forest constitutes around 40%.

2.The National Forestry Action Programme aims at bringing one third of


the area of India under tree/forest cover.

Which of the statements given above is/ are correct'?

(a)1 only

(b)2 only

(c)Both 1 and 2

(d)Neither 1 nor 2

[UPSC-2005]

81.Consider the following statements:

1.India is the second country in the world to adopt a National Family


Planning Programme.

2.The National Population Policy of India 2000 seeks to achieve


replacement level of fertility by 2010 with a population of 111 crores.
3.Kerala is the first state in India to achieve replacement level of fertility.

Which of the statements given above is/ are correct'?

(a)1 only

(b)1 and 2

(c)2 and 3

(d)1, 2 and 3

[UPSC-2005]

82.Consider the following statements:

1.Silent Valley National Park is in the Nallamalai range.

2.Pathrakkadavu Hydroelectric Project is proposed to be built near the


Silent Valley National Park.

3.The Kunthi River originates in Silent Valley's rainforests.

Which of the statements given above is/ are correct'?

(a)1 and 3

(b)2 only

(c)2 and 3

(d)1, 2 and 3

[UPSC-2005]
83.Consider the following statements:

1.Assam shares a border with Bhutan and Bangladesh.

2.West Bengal shares a border with Bhutan and Nepal.

3.Mizoram shares a border with Bangladesh and Myanmar.

Which of the statement gives above are correct'?

(a)1, 2 and 3

(b)1 and 2 only

(c)2 and 3 only

(d)1 and 3 only

[UPSC-2006]

84.Consider the following statements:

1.Sikkim has the minimum area among the 28 Indian states (Delhi and
Pondicherry not included).

2.Chandigarh has the highest literacy rate among Pondicherry, National


Capital Territory of Delhi and the Union Territories.

3.Maharashtra has the highest population after Uttar Pradesh among the
28 Indian states (Delhi and Pondicherry not included).

Which of the statements given above is/ are correct'?

(a)1 and2
(b)2 and 3

(c)1 only

(d)3 only

[UPSC-2006]

85.Consider the following statements:

1.Apellate Tribunal for Electricity has been established by each state


government in India.

2.One of the components of the Accelerated Power Development and


Reforms Program (APDRP) is up gradation of subtransmission and
distribution system for electricity in India.

Which of the statement given is/are correct?

(a)1 only

(b)2 only

(c)Both 1 and 2

(d)Neither 1 nor 2

[UPSC-2006]

86.Consider the following statements:

1.Balaghat is known for its diamond mines.

2.Majhgawan is known for its manganese deposits.


Which of the statement given above is/ are correct'?

(a)1 only

(b)2 only

(c)Both 1 and 2

(d)Neither 1 nor 2

[UPSC-2007]

87.Consider the following statements:

1.Chikmagalur is well-known for sugar production.

2.Mandya is well-known as a coffeeproducing region.

Which of the statements given above is/ are correct'?

(a)1 only

(b)2 only

(c)Both 1 and 2

(d)Neither 1 nor 2

[UPSC-2008]

88.Consider the following statements:

1.Saltwater crocodile is found in the Andaman and Nicobar Islands.

2.Shrew and tapir are found in the Western Ghats of the Malabar region.
Which of the statements given above is/ are correct'?

(a)1 only

(b)2 only

(c)Both 1 and 2

(d)Neither 1 nor 2

[UPSC-2008]

89.Consider the following statements:

1.India does not have any deposits of thorium.

2.Kerala's monazite sands contain uranium.

Which of the statements given above is/ are correct'?

(a)1 only

(b)2 only

(c)Both 1 and 2

(d)Neither 1 nor 2

[UPSC-2009]

90.Consider the following statements:

1.There are no east-flowing rivers in Kerala.

2.There is no west-flowing river in Madhya Pradesh.


Which of the statements given above is/ are correct'?

(a)1 only

(b)2 only

(c)Both 1 and 2

(d)Neither 1 nor 2

[UPSC-2009]

91.Consider the following statements:

The belt of coniferous forest in Nepal, Sikkim and Assam Himalayas is at


a higher altitude than those in Kashmir Himalayas because Sikkim and
Assam Himalayas

1.are at lower altitudes.

2.get lesser snowfall at lower heights.

3.have higher temperatures at lower heights.

4.have lower altitudes.

5.have more rainfall at lower heights.

Which of these are correct?

(a)1, 2 and 3

(b)2, 3 and 4

(c)1, 4 and 5
(d)1, 2, 3 and 5

[UPSC-2002]

92.Consider the following statements:

1.Rice and tobacco are the major crops of Andhra Pradesh.

2.Rice andjowar are the major crops of Orissa.

4.Rice and coffee are the major crops of Tamil Nadu.

Which of these are correct'?

(a)1, 2 and 3

(b)1, 2 and 4

(c)1, 3 and 4

(d)2, 3 and 4

[UPSC-2002]

93.Consider the following statements:

1.The coastal area between the Eastern Ghats and the Bay of Bengal is
narrower than the coastal area between the Western Ghats and the
Arabian Sea.

2.The Indus rises near Mansarovar in Tibet and finally falls in the
Arabian Sea near Karachi.

Which of the statements given above is/ are correct'?


(a)1 only

(b)2 only

(c)Both 1 and 2

(d)Neither 1 nor 2

[UPSC-2005]

94.Consider the following statements:

1.National Highway 7 is the longest among India's National Highways.

2.Nagpur, Hyderabad, Vijaywada, Chennai and Kanyakumari fall on


National Highway 7.

3.Gwalior falls on National Highways2 and 3.

Which of the statements given above is/ are correct'?

(a)1 only

(b)1 and 2

(c)2 and 3

(d)1, 2 and 3

[UPSC-2005]

95.Consider the following statements:

The black cotton soil of India


1.is mainly distributed over the river valleys of the lava region of the
Deccan.

2.has a great capacity of retaining moisture.

3.is extremely fertile at surface.

4.occupies about 10% of the total area of the country.

Of these statements

(a)1, 2and 3 are correct.

(b)1, 2 and 4 are correct.

(c)1 and 2 are correct.

(d)3 and 4 are correct.

[UPSC-1995]

96.Consider the following statements:

The south-west monsoon originates in India primarily due to

1.low pressure in the Punjab plain.

2.high pressure in areas south of 8°N latitude.

3.equatorial low being filled up by descending currents.

4.the Himalayas.

Of these statements
(a)1 and 4 are correct.

(b)1 and 2 are correct.

(c)1, 2 and 3 are correct.

(d)3 and 4 are correct.

[UPSC-1996]

97.Consider the following:

1.Food control

2.Irrigation

3.Soil conservation

4.Power generation

5.Industrial development

The primary objectives of Damodar Valley Project include

(a)2, 3 and 4.

(b)1, 2 and 4.

(c)2, 4 and 5.

(d)1, 4 and 5.

[UPSC-1996]

98.Consider the following statements:


In India, the problem of soil erosion is associated with

1.excessive rainfall.

2.excessive cultivation.

3.overgrazing.

4.excessive irrigation

(a)1 and 2 are correct.

(b)1, 2 and 3 are correct.

(c)2, 3 and 4 are correct.

(d)2 and 4 are correct.

[UPSC-1997]

99.Consider the following statements:

The type of natural vegetation in India varies due to variation in

1.the amount of rainfall.

2.soil types.

3.mean annual temperature.

4.altitude,

Of these statements

(a)1 and 2 are correct.


(b)1, 2 and 3 are correct.

(c)2, 3 and 4 are correct.

(d)1, 3 and 4 are correct.

[UPSC-1997]

100.Consider the following statements:

1.In India, red panda is naturally found in the western Himalayas only.

2.In India, slow loris lives in the dense forests of the North East.

Which of the statement gives above is/ are correct?

(a)1 only

(b)2 only

(c)Both 1 and 2

(d)Neither 1 nor 2

[UPSC-2007]

101.Consider the following states:

1.Maharashtra

2.Gujarat

3.West Bengal

4.Tamil Nadu
As per the 1991 census, the correct sequence in descending order of these
states in terms of the number of workers employed in industries is

(a)1, 3, 2, 4.

(b)1, 2, 4, 3.

(c)3, 1, 2, 4.

(d)3, 2, 1, 4.

[UPSC-1996]

102.Consider the following states:

1.Goa

2.Andhra Pradesh

3.Orissa

4.West Bengal

The correct sequence of these states in ascending order in terms of


mangrove vegetation coverage is

(a)2, 1, 3, 4.

(b)1, 2, 3, 4.

(c)2, 1, 4, 3.

(d)1, 2, 4, 3.

[UPSC-1999]
103.Consider the following states:

1.Karnataka

2.Orissa

3.Madhya Pradesh

4.Bihar

The correct sequence of these states in ascending order in terms of


production of iron ore is

(a)1, 3, 2, 4.

(b)1, 3, 4, 2.

(c)3, 1, 4, 2.

(d)3, 1, 2, 4.

[UPSC-1999]

104.Consider the following:

1.Mahadeo Hills

2.Sahyadri Parvat

3.Satpura Range

What is the correct sequence of the above from the north to the south?

(a)1, 2, 3
(b)2, 1, 3

(c)1, 3, 2

(d)2, 3, 1

[UPSC-2004]

105.Consider the hydroelectric power generation in the following states:

1.Maharashtra

2.Tamil Nadu

3.Uttar Pradesh

4.Kerala

The correct sequence of installed hydroelectric power capacity in


decreasing order will be

(a)1, 2, 4, 3.

(b)2, 1, 3, 4.

(c)3, 2, 1, 4.

(d)3, 4, 2, 1.

[UPSC-1995]

106.Consider the share of forest area in the following states:

1.Punjab
2.Haryana

3.Gujarat

4.Rajasthan

The correct sequence of the descending order of forested areas in these


states is

(a)3, 4, 2, 1.

(b)4, 3, 2, 1.

(c)3, 4, 1, 2.

(d)4, 3, 1, 2.

[UPSC-1997]

107.Copper is produced in

(a)Rajasthan and Bihar.

(b)Uttar Pradesh and Rajasthan.

(c)Bihar, Uttar Pradesh and Rajasthan.

(d)Orissa, Rajasthan, Bihar and Uttar Pradesh.

[UPSC-1996]

108.Doon valley is able to grow rice because

(a)it has warm summers and snow melt waters for irrigation.
(b)people in the valley are rice eaters.

(c)other crops cannot be grown.

(d)there is a huge export demand.

[UPSC-1995]

109.During winter, north-western part of India gets rainfall from

(a)thunderstorms.

(b)retreating monsoon.

(c)western disturbances.

(d)tropical cyclones.

[UPSC-2000]

110.Earthquakes along the Himalayas and the foothills are best explained in
terms of

(a)plate tectonics.

(b)young arc to the Himalayas.

(c)fragile and largely unconsolidated rocks.

(d)divergence of plates.

[UPSC-2002]

111.Following four cities are located almost on WE Longitude:

1.Indore
2.Solapur

3.Ludhiana

4.Jaipur

Which one of the following is the correct sequential order of the above
cities from north to south if one travel almost parallel along that
longitude?

(a)4, 1, 3, 2.

(b)3, 4, 1, 2.

(c)2, 3, 1, 4.

(d)3, 4, 2, 1.

[UPSC-2005]

112.In which of the following places are deposits of wolfram found'?

(a)Chamdigiri and Amarkantak

(b)Chendipathar and Degana

(c)Bellary and Radhanagari

(d)Guntur and Hazaribagh

[UPSC-2009]

113.For which one of the following is Satara well known'?

(a)Thermal power plant


(b)Wind energy plant

(c)Hydroelectric plant

(d)Nuclear power plant

[UPSC-2005]

114.From north to south, which one of the following is the correct sequence
of the given rivers in India'?

(a)Shyok, Spiti, Zaskar, Sutlej

(b)Shyok, Zaskar, Spiti, Sutlej

(c)Zaskar, Shyok, Sutlej, Spiti

(d)Zaskar, Sutlej, Shyok, Spiti

[UPSC-2006]

115.Gandhi Sagar Dam is a part of which one of the following'?

(a)Chambal Project

(b)Kosi Project

(c)Damodar Valley Project

(d)Bhakra Nangal Project

[UPSC-2005]

116.Ganga is navigable between

(a)Hardwar and Kanpur.


(b)Allahabad and Varanasi.

(c)Patna and Calcutta.

(d)Allahabad and Haldia.

[UPSC-2001]

117.HINDALCO, an aluminium factory located at Renukut, owes its site


basically to

(a)proximity of raw materials.

(b)abundant supply of power.

(c)efficient transport network.

(d)proximity to the market.

[UPSC-2002]

118.If it is 10.00 a.m. IST, then what would be the local time at Shillong on
92°E longitude?

(a)9.38 a.m.

(b)10.38 a.m.

(c)10.22 a.m.

(d)9.22 a.m.

[UPSC-1999]
119.In eastern Himalayas, the subtropical broad-leaved forests are
commonly found between the altitudes ranging from

(a)500 to 1,000 metres.

(b)750 to 1,000 metres.

(c)1,000 to 2,000 metres.

(d)2,000 to 2,500 metres.

[UPSC-2001]

120.In India, how many states share the coastline'?

(a)7

(b)8

(c)9

(d)10

[UPSC-2008]

121.In India, one of the following states has the largest inland saline
wetland?

(a)Gujarat

(b)Haryana

(c)Madhya Pradesh

(d)Rajasthan
[UPSC-2009]

122.In India, the ports are categorized as major and non-major ports. Which
one of the following is a non-major port'?

(a)Kochi (Cochin)

(b)Dahej

(c)Paradip

(d)New Mangalore

[UPSC-2009]

123.In India, the tropical deciduous forests form the natural cover in nearly
all the places where the annual rainfall is between

(a)201 and 250 cm.

(b)251 and 300 cm.

(c)70 and 100 cm.

(d)101 and 200 cm.

[UPSC-1999]

124.In India, which of the following crops is the largest consumer of


irrigation waters'?

(a)Rice

(b)Wheat
(c)Pulses

(d)Sugarcane

[UPSC-2003]

125.In terms of different sources of irrigation (viz. canals, wells, tanks and
others), which one of the following states strikes a balance between all'?

(a)Uttar Pradesh

(b)Andhra Pradesh

(c)Tamil Nadu

(d)Karnataka

[UPSC-1998]

126.In terms of total energy generation in India, the correct sequence in


descending order of the concerned zones is

(a)Southern, Western, Eastern, Northern.

(b)Western, Southern, Northern, Eastern.

(c)Western, Northern, Southern, Eastern.

(d)Southern, Eastern, Western, Northern.

[UPSC-1998]

127.In terms of value, India is the largest exporter of

(a)textiles.
(b)gems and jewellery.

(c)engineering goods.

(d)agricultural products.

[UPSC-1996]

128.In the Indian context, the term 'denotified tribes' refers to

(a)tribes which are aboriginals.

(b)nomadic tribes.

(c)tribes practicing shifting cultivation.

(d)tribes which were earlier classified as criminal tribes.

[UPSC-1996]

129.In the process of industrial development of India, the investment


enterprise in the public sector was the highest during the

(a)First Five Year Plan.

(b)Third Five Year Plan.

(c)Fifth Five Year Plan.

(d)Seventh Five Year Plan.

[UPSC-1997]

130.In the vicinity of Mumbai, a number of specialized towns have been


developed. Match List I with List II and select the correct answer using
the codes given.

Codes:

[UPSC-1998]

131.In which of the following states is the marine production of fish less
than the inland production?

(a)Tamil Nadu

(b)West Bengal

(c)Andhra Pradesh

(d)Kerala

[UPSC-2003]
132.In which one of the following cities is the Global Automotive Research
Centre being set up?

(a)Chennai

(b)Hyderabad

(c)Pune

(d)Gurgaon

[UPSC-2007]

133.In which one of the following districts have large reserves of diamond-
bearing kimberlite been discovered in the recent past?

(a)Hoshangabad

(b)Raipur

(c)Sambalpur

(d)Qarangal

[UPSC-2007]

134.In which one of the following oceans is Diamantine Trench situated?

(a)Pacific Ocean

(b)Atlantic Ocean

(c)Indian Ocean

(d)Arctic Ocean
[UPSC-2006]

135.In which one of the following places is the Shompen tribe found?

(a)Nilgiri Hills

(b)Nicobar Islands

(c)Spiti Valley

(d)Lakshadweep Islands

[UPSC-2009]

136.In which one of the following statesare Namchik-Namphuk coalfields


located?

(a)Arunachal Pradesh

(b)Meghalaya

(c)Manipur

(d)Mizoram

[UPSC-2008]

137.In which one of the following states has India's largest private sector
seaport been commissioned recently?

(a)Andhra Pradesh

(b)Karnataka

(c)Kerala
(d)Tamil Nadu

[UPSC-2009]

138.In which one of the following Union territories, do the people of the
Onge tribe live?

(a)Andaman and Nicobar Islands

(b)Dadra and Nagar Haveli

(c)Daman and Diu

(d)Lakshadweep

[UPSC-2004]

139.In which sate is the Guru Shikhar Peak located?

(a)Rajasthan

(b)Gujarat

(c)Madhya Pradesh

(d)Maharashtra

[UPSC-2007]

140.In which state is the Buddhist site Tabo Monastery located?

(a)Arunachal Pradesh

(b)Himachal Pradesh

(c)Sikkim
(d)Uttarakhand

[UPSC-2009]

141.In which state is the Rajiv Gandhi National Institute Youth


Development located?

(a)Tamil Nadu

(b)Karnataka

(c)Himachal Pradesh

(d)Uttaranchal

[UPSC-2006]

142.India has the higher productive potential of fisheries in her

(a)inland water bodies.

(b)shallow continental shelf.

(c)deep sea areas.

(d)brackish water lagoons.

[UPSC-1997]

143.India's Eastern Ghats and Western Goths meet at the

(a)Cardamom Hills.

(b)Annamalai Hills.

(c)Nilgiri Hills.
(d)Palani Hills.

[UPSC-2000]

144.India's population growth during the 20th century can be classified into
four distinct phases. Match List I with List II and select the correct
answer using the codes given.

Codes:

[UPSC-2002]

145.ISRO successfully conducted a rocket test using cryogenic engines in


the year 2007. Where is the test stand used for the purpose located'?

(a)Balasore

(b)Thiruvananthapuram
(c)Mahendragiri

(d)Karwar

[UPSC-2008]

146.Karewas are terraces of glacial origin found in

(a)Teesta valley.

(b)Ravi valley.

(c)Jhelum valley.

(d)Alaknanda valley.

[UPSC-1997]

147.Kathiawar Peninsula is an example of

(a)emerged shoreline.

(b)submerged shoreline.

(c)Ria shoreline.

(d)Dalmatian shoreline.

[UPSC-2001]

148.Koppen's Amw type of climate prevails over

(a)the interior peninsula of India.

(b)the Coromandel coast of India.


(c)Western coast of India, south of Goa.

(d)Eastern coast of India, south of Calcutta.

[UPSC-1996]

149.Lake Sambhar is nearest to which one of the following cities of


Rajasthan?

(a)Bharatpur

(b)Jaipur

(c)Jodhpur

(d)Udaipur

[UPSC-2004]

150.Local supply of coal is not available to

(a)TISCO, Jamshedpur.

(b)VSL, Bhadravati.

(c)HSL, Durgapur.

(d)HSL, Bhilai.

[UPSC-1996]

151.Location of sugar industry in India is shifting from north to south


because of

(a)cheap labour.
(b)expanding regional market.

(c)cheap and abundant supply of power.

(d)high yield and high sugar content in sugarcane.

[UPSC-2000]

152.Major share of the total production of marine fish is landed along the

(a)Kerala and Maharashtra coasts.

(b)Maharashtra and Karnataka coasts.

(c)Karnataka and West Bengal coasts.

(d)West Bengal and Maharashtra coasts.

[UPSC-2000]

153.Mangrove vegetation in India is most extensively found in

(a)Malabar Coast.

(b)The Sunderbans.

(c)The Rann of Kutch.

(d)coastal Orissa.

[UPSC-2005]

154.Match List I with List II and select the correct answer using the codes
given.
Codes:

[UPSC-2001]

155.Match List I with List II and select the correct answer using the codes
given.

Codes:
[UPSC-1998]

156.Match List I with List II and select the correct answer using the codes
given.

Codes:

[UPSC-1999]

157.Match List I with List II and select the correct answer using the codes
given.
Codes:

[UPSC-1997]

158.Match List I with List II and select the correct answer using the codes
given.
Codes:

[UPSC-2004]

159.Match List I with List II and select the correct answer using the codes
given.

Codes:

[UPSC-2004]

160.Match List I with List II and select the correct answer using the codes
given.
Codes:

[UPSC-2004]

161.Match List I with List II and select the correct answer using the codes
given.
Codes:

[UPSC-2004]

162.Match List I with List II and select the correct answer using the codes
given.
Codes:

[UPSC-2004]

163.Match List I with List II and select the correct answer using the codes
given.

Codes:
[UPSC-2004]

164.Match List I with List II and select the correct answer using the codes
given.

Codes:

[UPSC-1996]

165.Match List I with List II and select the correct answer using the codes
given.
Codes:

[UPSC-2000]

166.Match List I with List II and select the correct answer using the codes
given.
Codes:

[UPSC-2004]

167.Match List I with List II and select the correct answer using the codes
given.

Codes:

[UPSC-2004]

168.Match List I with List II and select the correct answer using the codes
given.
Codes:

[UPSC-2008]

169.Match List I with List II and select the correct answer using the codes
given.

Codes:
[UPSC-2007]

170.Match List I with List II and select the correct answer using the codes
given.

Codes:

[UPSC-1999]

171.Match List I with List II and select the correct answer using the codes
given.
Codes:

[UPSC-1999]

172.Match List I with List II and select the correct answer using the codes
given.
Codes:

[UPSC-2000]

173.Match List I with List II and select the correct answer using the codes
given.
Codes:

[UPSC-2003]

174.Match List I with List II and select the correct answer using the codes
given.

Codes:
[UPSC-1997]

175.Match List I with List II and select the correct answer using the codes
given.

Codes:

[UPSC-1997]

176.Match List I with List II and select the correct answer using the codes
given.
Codes:

[UPSC-1997]

177.Match List I with List II and select the correct answer using the codes
given.

Codes:
[UPSC-1997]

178.Match List I with List II and select the correct answer using the codes
given.

Codes:

[UPSC-1999]

179.Match List I with List II and select the correct answer using the codes
given.
Codes:

[UPSC-1995]

180.Match List I with List II and select the correct answer using the codes
given.

Codes:

[UPSC-1996]
181.Match List I with List II and select the correct answer using the codes
given.

Codes:

[UPSC-1997]

182.Match List I with List II and select the correct answer using the codes
given.

Codes:
[UPSC-1997]

183.Match List I with List II and select the correct answer using the codes
given.

Codes:

[UPSC-1997]

184.Match List I with List II and select the correct answer using the codes
given.
Codes:

[UPSC-1997]

185.Match List I with List II and select the correct answer using the codes
given.
Codes:

[UPSC-1998]

186.Match List I with List II and select the correct answer using the codes
given.

Codes:

[UPSC-2000]

187.Match List I with List II and select the correct answer using the codes
given.
Codes:

[UPSC-2001]

188.Match List I with List II and select the correct answer using the codes
given.

Codes:
[UPSC-2001]

189.Match List I with List II and select the correct answer using the codes
given.

Codes:

[UPSC-2003]

190.Match List I with List II and select the correct answer using the codes
given.
Codes:

[UPSC-2003]

191.Match List I with List II and select the correct answer using the codes
given.

Codes:
[UPSC-2003]

192.Match List I with List II and select the correct answer using the codes
given.

Codes:

[UPSC-2004]

193.Match List I with List II and select the correct answer using the codes
given.
Codes:

[UPSC-1995]

194.Match List I with List II and select the correct answer using the codes
given.

Codes:
[UPSC-1995]

195.Match List I with List II and select the correct answer using the codes
given.

Codes:

[UPSC-1996]

196.Match List I with List II and select the correct answer using the codes
given.
Codes:

[UPSC-1996]

197.Match List I with List II and select the correct answer using the codes
given.

Codes:
[UPSC-1996]

198.Match List I with List II and select the correct answer using the codes
given.

Codes:

[UPSC-1997]

199.Match List I with List II and select the correct answer using the codes
given.

Codes:
[UPSC-1997]

200.Match List I with List II and select the correct answer using the codes
given.

Codes:

[UPSC-1997]

201.Match List I with List II and select the correct answer using the codes
given.
Codes:

[UPSC-1998]

202.Match List I with List II and select the correct answer using the codes
given.

Codes:
[UPSC-1998]

203.Match List I with List II and select correct answer by using the codes
given.

Codes:

[UPSC-1998]

204.Match List I with List II and select the correct answer using the codes
given.
Codes:

[UPSC-1998]

205.Match List I with List II and select the correct answer using the codes
given.

Codes:
[UPSC-1998]

206.Match List I with List II and select the correct answer using the codes
given.

Codes:

[UPSC-1999]

207.Match List I with List II and select the correct answer using the codes
given.
Codes:

[UPSC-1999]

208.Match List I with List II and select the correct answer using the codes
given.

Codes:
[UPSC-1999]

209.Match List I with List II and select the correct answer using the codes
given.

Codes:

[UPSC-1999]

210.Match List I with List II and select the correct answer using the codes
given.

Codes:
[UPSC-2000]

211.Match List I with List II and select the correct answer using the codes
given.

Codes:

[UPSC-2000]

212.Match List I with List II and select the correct answer using the codes
given.
Codes:

[UPSC-2001]

213.Match List I with List II and select the correct answer using the codes
given.

Codes:
[UPSC-2001]

214.Match List I with List II and select the correct answer using the codes
given.

Codes:
[UPSC-2001]

215.Match List I with List II and select the correct answer using the codes
given.

Codes:

[UPSC-2002]

216.Match List I with List II and select the correct answer using the codes
given.
Codes:

[UPSC-2003]

217.Match List I with List II and select the correct answer using the codes
given.

Codes:
[UPSC-2003]

218.Match List I with List II and select the correct answer using the codes
given.

Codes:

[UPSC-2003]

219.Match List I with List II and select the correct answer using the codes
given.
Codes:

[UPSC-2005]

220.Match List I with List II and select the correct answer using the codes
given.
Codes:

[UPSC-2005]

221.Match List I with List II and select the correct answer using the codes
given.

Codes:
[UPSC-1998]

222.Match List I with List II and select the correct answer using the codes
given.

Codes:

[UPSC-1998]

223.Match List I with List II and select the correct answer using the codes
given.
Codes:

[UPSC-2006]

224.Match List I with List II and select the correct answer using the codes
given.

Codes:
[UPSC-2006]

225.Match List I with List II and select the correct answer using the codes
given.

Codes:

[UPSC-2006]

226.Match List I with List II and select the correct answer using the codes
given.
Codes:

[UPSC-2006]

227.Modern industrial development of India is largely due to

(a)rationalization of raw materials.

(b)availability of skilled manpower.

(c)liberalization of government policies.

(d)increase in market demand.

[UPSC-1995]

228.`Monoculture of commercially viable trees is destroying the unique


natural profile of thoughtless exploitation of timber, deforesting vast
tracts for palm cultivation, destruction of mangroves, illegal logging by
tribals and poaching only compound the problem. Freshwater pockets are
fast drying up due to deforestation and distribution of mangroves'. The
place referred to in this quotation is

(a)Sunderbans.

(b)Kerala coast.

(c)Orissa coast.

(d)Andaman and Nicobar Islands.

[UPSC-1995]

229.Nagaland mountains are becoming increasingly barren mountains,


mainly due to

(a)insurgency.

(b)shifting cultivation.

(c)urbanization.

(d)rapid population growth.

[UPSC-2000]

230.Nanda Devi peak forms a part of

(a)Assam Himalayas.

(b)Kumaon Himalayas.

(c)Nepal Himalayas.
(d)Punjab Himalayas.

[UPSC-2003]

231.Nuclear power plant is associated with

(a)Madras.

(b)Kota.

(c)Ganganagar.

(d)Mathura.

[UPSC-1995]

232.On which one of the following rivers is the Tehri Hydropower Complex
located'?

(a)Alaknanda

(b)Bhagirathi

(c)Dhauliganga

(d)Mandakini

[UPSC-2008]

233.Out of the four southern states Andhra Pradesh, Karnataka, Kerala and
Tamil Nadu, which shares boundaries with the maximum number of
Indian states?

(a)Andhra Pradesh only


(b)Karnataka only

(c)Andhra Pradesh and Karnataka

(d)Tamil Nadu and Kerala

[UPSC-2007]

234.Per capita availability of forest land is higher in

(a)Madhya Pradesh.

(b)Assam.

(c)Jammu and Kashmir.

(d)Sikkim.

[UPSC-1996]

235.Projects and Development India Limited is mainly engaged in design


engineering procurement and supervision of construction/ commissioning
in which area'?

(a)Pharmaceutical plants

(b)Petroleum refineries and sugar factories

(c)Ship-building yards

(d)Fertilizer and allied chemical plants

[UPSC-2006]

236.Rajiv Gandhi National Institute is being established in which state?


(a)Karnataka

(b)Maharashtra

(c)Kerala

(d)Orissa

[UPSC-2008]

237.Recently, Uttar Pradesh and Madhya Pradesh governments signed a


Memorandum of Understanding of the linking of two rivers as a link
project. Which are these two rivers'?

(a)Betwa and Chambal

(b)Betwa and Ken

(c)Chambal and Son

(d)Ken and Narmada

[UPSC-2006]

238.Shillong plateau is also known as `Meghalaya' meaning `abode of


clouds'. It was so named by

(a)O.H.K. Spate.

(b)S.P. Chatterjee.

(c)D.N. Wadia.

(d)R.L. Singh.
[UPSC-1998]

239.Structurally, the Great Himalaya is

(a)an autochthonous zone.

(b)a para-autochtonous zone.

(c)an allochthonous zone.

(d)a suture zone.

[UPSC-2004]

240.Talcher and Ennore and noted for

(a)atomic power plants.

(b)hydel power plants.

(c)geothermal power plants.

(d)thermal power plants.

[UPSC-1999]

241.The biggest ship-building yard of India is

(a)Garden Reach Workshop, Calcutta.

(b)Hindustan Shipyard, Vishakhapatnam.

(c)Mazagaon Dock, Bombay.

(d)Cochin Shipyard, Kochi.


[UPSC-1995]

242.The Alamatti is on the river

(a)Godavari.

(b)Kaveri.

(c)Krishna.

(d)Mahanadi.

[UPSC-1997]

243.The Amarkantak Hill is the source of two rivers flowing in two different
directions (west and east). They are

(a)Narmada and Tapti.

(b)Narmada and Mahanadi.

(c)Tapti and Betwa.

(d)Tapti and Son.

[UPSC-1998]

244.The approximate age of the Aravalli range is

(a)370 million years.

(b)470 million years.

(c)570 million years.

(d)670 million years.


[UPSC-2001]

245.The area between Chhota Nagpur plateau and Shillong plateau is

(a)agarben.

(b)formed by erosion of Ganga.

(c)a downfaulted region.

(d)a downwarped region in which the Ganga has deposited sediments.

[UPSC-2004]

246.The area spread of fishing ground of the continental shelf along the
peninsula coast of India is over

(a)2 lakh km2.

(b)3 lakh km2.

(c)4 lakh km2.

(d)5 lakh km2.

[UPSC-2001]

247.The average annual temperature of a meteorological station is 26°C, its


average annual rainfall is 63 cm and the annual range temperature is 9°C.
The station in question is

(a)Allahabad.

(b)Chennai.
(c)Cherrapunji.

(d)Kolkata.

[UPSC-2002]

248.The boundary of the Siwalik Belt with the Indo-Gangetictrough is


known as

(a)main boundary thrust.

(b)main boundary fault.

(c)great boundary fault.

(d)scarp fault.

[UPSC-2004]

249.The canal-carrying capacity of Farrakka is

(a)75,000 cusecs.

(b)70,000 cusecs.

(c)40,000 cusecs.

(d)35,000 cusecs.

[UPSC-1997]

250.The correct sequence (in descending order) of the given industries in


terms of consumption of coal in India is

(a)iron and steel, cement, textile, thermal power.


(b)thermal power, textile, iron and steel, cement.

(c)iron and steel, thermal power, cement, textile.

(d)thermal power, iron and steel, cement, textile.

[UPSC-2001]

251.The correct sequence in decreasing order in terms of total production of


the given non-ferrous metals in India is

(a)lead zinc, copper, aluminium.

(b)zinc, lead, aluminium, copper.

(c)aluminium, zinc, copper, lead.

(d)aluminium, copper, zinc, lead.

[UPSC-1998]

252.The correct sequence in decreasing order of the four sugarcane-


producing statesin India is

(a)Maharashtra, UP, Tamil Nadu, Andhra Pradesh.

(b)UP, Maharashtra, Tamil Nadu, Andhra Pradesh.

(c)Maharashtra, UP, Andhra Pradesh, Tamil Nadu.

(d)Up, Maharashtra, Andhra Pradesh, Tamil Nadu.

[UPSC-2000]
253.The correct sequence in descending order of the given soils with respect
to area coverage of India is

(a)alluvial, black, red, laterite.

(b)alluvial, red, black, laterite.

(c)alluvial, red laterite, black.

(d)red, alluvial, black, laterite.

[UPSC-1996]

254.The correct sequence in descending order of the given states in coal


mining is

(a)West Bengal, Bihar, Madhya Pradesh.

(b)Bihar, Madhya Pradesh, West Bengal.

(c)Bihar, West Bengal, Madhya Pradesh.

(d)Madhya Pradesh, Bihar, West Bengal.

[UPSC-1996]

255.The correct sequence in the ascending order of the given cities in terms
of altitude above mean sea level is

(a)Marmagao, Mumbai, Kolkata, Chennai.

(b)Mumbai, Kolkata, Chennai, Marmagao.

(c)Chennai, Marmagao, Mumbai, Kolkata.


(d)Kolkata, Mumbai, Chennai, Marmagao.

[UPSC-2001]

256.The correct sequence of the eastward flowing rivers of the peninsular


India from north to south is

(a)Subarnarekha, Mahanadi, Godavari, Krishna, Pennar, Kaveri and


Vagai.

(b)Subarnarekha, Mahanadi, Krishna, Godavari, Kaveriand Vagai.

(c)Mahanadi, Subarnarekha, Godavari, Krishna Kaveri, Pennar and


Vagai.

(d)Mahanadi, Subarnarekha, Krishna, Godavari, Kaveri, Vagai and


Pennar.

[UPSC-2002]

257.The correct sequence of the given states in descending order with


respect of population of bajra is

(a)Rajasthan, Gujarat, Maharashtra, Uttar Pradesh.

(b)Rajasthan, Gujarat, Uttar Pradesh, Maharashtra.

(c)Gujarat, Rajasthan, Maharashtra, Uttar Pradesh.

(d)Gujarat, Rajasthan, Uttar Pradesh, Maharashtra.

[UPSC-1999]

258.The discovery of oak flora in 1966 added a new chapter to the history of
Indian Sericulture. Which one of the following states is the leading
producer of Oak tasar silks?

(a)Assam

(b)Bihar

(c)Manipur

(d)Orissa

[UPSC-1998]

259.Consider the following statements:

1.The Cochin shipyard was constructed with Japanese assistance.

2.The Integral Coach Factory was set up with German collaboration

Which of the statements given above is/ are correct'?

(a)1 only

(b)2 only

(c)Both 1 and 2

(d)Neither 1 nor 2

[UPSC-2009]

260.The Dul Hasti Power Station is based on which one of the following
rivers'?

(a)Beas
(b)Chenab

(c)Ravi

(d)Sutlej

[UPSC-2009]

261.The first `high tech' port of India is

(a)Quilon.

(b)Nagapathinam.

(c)Paradeep.

(d)Nava Sheva.

[UPSC-2001]

262.The first Indian state to have its Human Development Report prepared
and released by Amartya Kumar Sen in Delhi is

(a)West Bengal.

(b)Kerala.

(c)Madhya Pradesh.

(d)Andhra Pradesh.

[UPSC-2000]

263.The first marine sanctuary in India, whining its bounds coral reefs,
mollusc, dolphins, tortoises and various kinds of sea birds, has been
established in

(a)Sundarbans.

(b)Chilka Lake.

(c)Gulf of Kutch.

(d)Lakshadweep.

[UPSC-1999]

264.The following are the important events in coal mining in India:

1.First production of coal at Raniganj.

2.The establishment of Coal India Limited.

3.Nationalization of coal mines

4.Establishment of Neyveli Lignite Corporation

Their correct chronological sequence is:

(a)1, 2, 3, 4

(b)2, 1, 3, 4

(c)1, 2, 4, 3

(d)2, 1,4,3

[UPSC-1996]

265.The following table contains temperature and rainfall date of a city.


The data is associated with

(a)Ahmedabad.

(b)Allahabad.

(c)Nagpur.

(d)New Delhi.

[UPSC-2001]

266.Permission of The Genetic Engineering Approval Committee is required


for cultivation of any genetically modified crop such as Btcotton in India.
This is under the Union Ministry of
(a)Agriculture.

(b)Environment and Forest.

(c)Commerce and Industry.

(d)Rural Development.

[UPSC-2003]

267.The greatest potential for the generation of tidal power in India is


available in the

(a)Malabar coast.

(b)Konkan coast.

(c)Gujarat coast.

(d)Coromandel coast.

[UPSC-1996]

268.The group of states which has forest coverage of more than 75% of the
total geographic area is

(a)Assam, Meghalaya, Nagaland.

(b)Assam, Arunachal Pradesh, Nagaland.

(c)Arunachal Pradesh, Manipur, Nagaland.

(d)Arunachal Pradesh, Madhya Pradesh, Nagaland.

[UPSC-1997]
269.The Himalayan front fault is located between

(a)Siwalik and Lesser Himalaya.

(b)Great and Trans-Himalayas.

(c)Lesser and Great Himalayas.

(d)Siwalik and piedmont zone.

[UPSC-2004]

270.The Himalayan region is poor in mineral resources because

(a)it is made of crystalline rocks.

(b)the displacement of rock strata has disturbed the arrangement of rocks


and made it complex.

(c)the climatic conditions are not suitable for exploitation of minerals.

(d)the terrain makes exploitation of minerals difficult and very costly due
to transport difficulties.

[UPSC-2004]

271.The irregularity in the amount of rainfall in different parts of the north


Indian plains is mainly due to

(a)irregular intensity of low pressure in the north-western parts of India.

(b)variation in the location of the axis of low-pressure trough.

(c)difference in frequency of cyclones.


(d)variation in the amount of moisture.

[UPSC-2005]

272.The January isotherm taken as a basis for dividing India into tropical
and subtropical zones is

(a)21°C.

(b)180°C.

(c)12°C.

(d)15°C.

[UPSC-1997]

273.The Kathiawar Peninsula is an example of

(a)emerged shorelines.

(b)submerged shorelines.

(c)Ria shorelines.

(d)Dalmatian shorelines.

[UPSC-2003]

274.The largest area under wasteland in India is in

(a)Uttar Pradesh.

(b)Madhya Pradesh.

(c)Rajasthan.
(d)Gujarat.

[UPSC-1996]

275.The largest number of Buddhists are found in

(a)Bihar.

(b)Karnataka.

(c)Maharashtra.

(d)Uttar Pradesh.

[UPSC-2001]

276.The largest portion of India's wasteland occurs in

(a)upland without scrub.

(b)water-logged and marshy lands.

(c)degraded pastures and grazing land.

(d)wastelands created by shifting cultivation.

[UPSC-1997]

277.The location of iron and steel mills in India is determined in the order of
availability factor of

(a)iron ore, coking coal, limestone and manganese.

(b)coking coal, iron ore, limestone and manganese.

(c)iron ore, manganese, limestone and coking coal.


(d)coking coal, manganese, iron ore and limestone.

[UPSC-1995]

278.The main ore from which manganese is extracted in India is

(a)siderite.

(b)chalcopyrite.

(c)pyrolusite.

(d)covellite.

[UPSC-1995]

279.The major forest product in India is

(a)timber.

(b)fuel food.

(c)cane products.

(d)resins.

[UPSC-1996]

280.The Meghalaya Plateaus are largely formed of

(a)cretaceous lava.

(b)Gondwana rocks.

(c)Dharwarian quartzies.
(d)tertiary sedimentary rocks.

[UPSC-2003]

281.The minimum land area recommended for forest cover to maintain


proper ecological balance in India is

(a)25 per cent.

(b)33 per cent.

(c)43 per cent.

(d)53 per cent.

[UPSC-1999]

282.The most important cement-producing states in India are

1Andhra Pradesh.

2Rajasthan.

3Madhya Pradesh.

4Tamil Nadu.

Select the correct sequence in order of decreasing importance in the


production of cement.

(a)1, 2, 3, 4

(b)3, 1, 4, 2

(c)2, 3, 4, 1
(d)4, 2, 1, 3

[UPSC-1995]

283.The most plausible explanation of the location of the Thar desert in


western India is

(a)the obstruction caused by Aravalli to the rain-bearing wind that


proceeds to the Ganga Valley.

(b)the evaporation of moisture by heat.

(c)the absence of mountains to the north of Rajasthan to cause geographic


rainfall in it.

(d)that the moisture carried by the south-west monsoon is driven away by


the dry upper air current.

[UPSC-1996]

284.The National Highway 5 joins

(a)Delhi with Mumbai.

(b)Varanasi with Bangalore.

(c)Kolkata with Mumbai.

(d)Chennai with Kolkata.

[UPSC-2002]

285.The Palghat Gap serves inland communication from

(a)Madurai to Thiruvananthapuram.
(b)Chennai to Kochi.

(c)Pune to Mumbai.

(d)Bangalore to Mangalore.

[UPSC-2000]

286.The Palghat Gap serves inland communication from

(a)Panaji to Kochi.

(b)Kochi to Chennai.

(c)Mangalore to Mumbai.

(d)Madurai to Thiruvananthapuram.

[UPSC-2003]

287.The power plant of Manikaran based on geothermal energy is in the


state of

(a)Arunachal Pradesh.

(b)Himachal Pradesh.

(c)Jammu and Kashmir.

(d)Uttaranchal.

[UPSC-2001]

288.The present forest area of India, according to satellite data, is

(a)increasing.
(b)decreasing.

(c)static.

(d)Open forest areas is decreasing but closed forest areas is increasing.

[UPSC-1997]

289.The Ravva offshore block, with great potential for oil, is located in

(a)Krishna-Godavari basin.

(b)Kaveribasin.

(c)Mahanadi basin.

(d)Palar-Pennar basin.

[UPSC-1999]

290.The refining capacity of crude oil is highest in the

(a)Haldia oil refinery.

(b)Mathura oil refinery.

(c)Gujarat oil refinery.

(d)Vishakhapatnam oil refinery.

[UPSC-1997]

291.The sea coast of which one of the following states has become famous
as nesting place for the giant Olive Ridley turtles of South America?

(a)Goa
(b)Gujarat

(c)Orissa

(d)Tamil Nadu

[UPSC-2002]

292.The state which has the largest number of seats reserved for the
Scheduled Tribes in the Lok Sabha is

(a)Bihar.

(b)Gujarat.

(c)Uttar Pradesh.

(d)Madhya Pradesh.

[UPSC-2000]

293.The state which leads the others in marine fishing is

(a)Kerala.

(b)Tamil Nadu.

(c)West Bengal.

(d)Andhra Pradesh.

[UPSC-1996]

294.The state which records the highest productivity of fish (per unit areas)
in India is
(a)Kerala.

(b)Tamil Nadu.

(c)Orissa.

(d)West Bengal.

[UPSC-1995]

295.The temperature of Thiruvananthapuram is lower than that of Mumbai


in May and higher than that of Mumbai in January because

(a)Thiruvananthapuram faces cold current and Mumbai is faced with


warm current.

(b)Thiruvananthapuram has higher rainfall in summer and it is nearer to


the Equator.

(c)Thiruvananthapuram is on the windward side and Mumbai is on the


leeward side.

(d)Thiruvananthapuram is thickly vegetated while Mumbai is not.

[UPSC-2004]

296.The term `Regur' refers to

(a)deltaic alluvial soils.

(b)laterite soils.

(c)black cotton soils.

(d)red and yellow soils.


[UPSC-1995]

297.The thermal power plant of Bokaro is located in

(a)Bihar.

(b)Chhattisgarh.

(c)Jharkhand.

(d)Orissa.

[UPSC-2003]

298.The transhumance is still an important component of the socioeconomic


life of the people in regions of

(a)Andaman and Nicobar Islands.

(b)Meghalaya.

(c)Jammu and Kashmir.

(d)Himachal Pradesh.

[UPSC-2001]

299.The upper course of Damodar River occupies a/an

(a)rift valley.

(b)synclinal valley.

(c)eroded valley.

(d)depositional valley.
[UPSC-2001]

300.The western syntaxial bend of the Himalayas is near

(a)Zaskar Range.

(b)Pir Panjal Range.

(c)Nanga Parbat.

(d)Siwalik Hills.

[UPSC-2003]

301.The zonal soil type of peninsular India belongs to

(a)red soils.

(b)yellow soils.

(c)black soils.

(d)older alluvium.

[UPSC-1996]

302.There is now a tendency to set up more sugar factories in South India


because of

1.longer crushing season

2.higher productivity of sugarcane

3.scarcity of other cash crops

4.efficient transport system


Which of the above answers are correct?

(a)1, 2 and 3

(b)1, 3 and 4

(c)1, 2 and 4

(d)2, 3 and 4

[UPSC-2001]

303.Transchart is the

(a)chartering wing of the Minister of Surface Transport.

(b)container service established by Indian Railways.

(c)training institute of maritime studies and research.

(d)passenger insurance scheme of Indian Railways.

[UPSC-1999]

304.Under Thornthwaite's classification of climate, almost the entire state of


Orissa comes under

(a)dry subhumid.

(b)moist subhumid.

(c)semi-arid.

(d)humid,

[UPSC-1996]
305.Unlike other parts of the Indian coast, fishing industry has not
developed along the Saurashtra coast because

(a)there are few indentations suitable for fishing.

(b)of overwhelming dependence on agriculture and animal husbandry.

(c)the sea water is relatively more saline.

(d)of industrial developed leading to widespread pollution of coastal area.

[UPSC-1998]

306.What is Gomia in Jharkhand known for?

(a)Coalfields

(b)Manganese mining

(c)Fertilizer plant

(d)Explosive factory

[UPSC-2005]

307.What is the correct sequence of the following Indian states in


descending order oftheir length of surface roads per 100 km2 of their
areas'?

1.Haryana

2.Maharashtra

3.Punjab
4.Tamil Nadu

Select the correct answer using the codes given.

(a)4, 3, 2, 1

(b)4, 3, 1, 2

(c)3, 4, 1, 2

(d)3, 4, 2, 1

[UPSC-1998]

308.Arrange the rivers in the correct sequence of descending order of their


lengths'?

(a)Godavari, Mahanadi, Narmada, Tapi

(b)Godavari, Narmada, Mahanadi, Tapi

(c)Narmada, Godavari, Tapi, Mahanadi

(d)Narmada, Tapi, Godavari, Mahanadi

[UPSC-2003]

309.Where are Shevaroy Hills located'?

(a)Andhra Pradesh

(b)Karnataka

(c)Kerala
(d)Tamil Nadu

[UPSC-2007]

310.Where are Tapovan and Vishnugarh Hydroelectric Projects located?

(a)Madhya Pradesh

(b)Uttar Pradesh

(c)Uttarakhand

(d)Rajasthan

[UPSC-2008]

311.Where do the Sahariya tribals live'?

(a)Andhra Pradesh

(b)Assam

(c)Rajasthan

(d)Orissa

[UPSC-2005]

312.Where is the Central Water and Power Research Station located?

(a)Khadakwasla

(b)Sileru

(c)Jamnagar
(d)Srisailam

[UPSC-2006]

313.Which of the following are known for paper manufacturing industry'?

1.Yamuna Nagar

2.Guwahati

3.Shahabad

4.Balarpur

Choose the correct answer using the codes given.

(a)1, 2 and 3

(b)1, 2 and 4

(c)1, 3 and 4

(d)2, 3 and 4

[UPSC-1997]

314.Which of the following coasts generally remains dry during the months
of July and August'?

(a)Coromandel Coast

(b)Konkan Coast

(c)Malabar Coast
(d)Northern Circars

[UPSC-2003]

315.Which of the following correctly states the locations of `cold desert' of


India'?

(a)To the north-east of Karakoram Range

(b)To the west of Pir-Panjal Range

(c)To the south of Siwalik Range

(d)To the west of Aravalli Range

[UPSC-2003]

316.Which of the following hills are found where the Eastern Ghats and the
Western Ghats meet'?

(a)Annamalai Hills

(b)Cardamom Hills

(c)Nilgiri Hills

(d)Shevaroy Hills

[UPSC-2008]

317.Which of the following is the largest coal field in India?

(a)Jharia

(b)Raniganj
(c)Bokaro

(d)Girdih

[UPSC-2003]

318.Which of the following minerals are found in a natural way in


Chhattisgarh?

1.Bauxite

2.Dolomite

3.Iron ore

4.Tin

Select the correct answer using the codes given.

(a)1, 2 and 3

(b)1 and 3

(c)2 and 4

(d)1, 2, 3 and 4

[UPSC-2008]

319.Which of the following pairs are correctly matched'?

1.Idukki: Thermal power project

2.Sabarigiri: Hydroelectric project


3.Ghatprabha: Irrigation project

4.Ramganga: Multipurpose project

Select the correct answer using the codes given.

(a)2, 3 and 4

(b)1, 2, 3 and 4

(c)3 and 4

(d)1 and 2

[UPSC-1996]

320.Which of the following pairs are correctly matched'?

Waterfalls: River

1.Kapildhara Falls: Godavari

2.Jog Falls: Sharavati

3.Sivasamudram Falls: Kaveri

Select the correct answer using the codes given.

(a)1 and 2

(b)2 and 3

(c)1 and 3

(d)1, 2 and 3
[UPSC-2008]

321.Which of the following pairs are correctly matched?

Select the correct answer using the codes given.

(a)1, 2 and 3

(b)1 and 2

(c)2 and 3

(d)1 and 3

[UPSC-2005]

322.Which of the following pairs are correctly matched?

1.Okha: Salt

2.Bastar: Mica

3.Jodhpur: Gypsum

4.Nasik: Chromite

Select the correct answer using the codes given.

(a)2, 3 and 4
(b)1, 3 and 4

(c)1, 2 and 4

(d)1, 2 and 3

[UPSC-1996]

323.Which of the following pairs are correctly matched'?

1.Winter rain in north-west India Western disturbance

2.Summer rain in Malabar coast Retreating monsoon

3.Summer rain in Bengal basin: Northwesterly

Select the correct answer using the codes given.

(a)1, 2, 3 and 4

(b)1, 3 and 4

(c)3 and 4

(d)1 and 2

[UPSC-1998]

324.Which of the following pairs are correctly matched'?

Irrigation project: State

1.Damanganga: Gujarat

2.Girna: Maharashtra
3.Pamba: Kerala

(a)1 and 2

(b)2 and 3

(c)1 and 3

(d)1, 2 and 3

[UPSC-2008]

325.Which of the following pairs in respect of current power generation in


India is/ are correctly matched?

1.Installed electricity generation capacity: 1,20,000 MW

2.Electricity generation: 660 billion kWh

(a)lonly

(b)2 only

(c)Both 1 and 2

(d)Neither 1 nor 2

[UPSC-2008]

326.Which of the following pairs is/are correctly matched?

1.Korba coal mine: Orissa

2.Khetri copper mine: Rajasthan


3.Kodarma mica mines: Madhya Pradesh

(a)1, 2 and 3

(b)2 and 3

(c)2 alone

(d)1 and 3

[UPSC-1998]

327.Which of the following pairs of states and their important crops are
correctly matched?

1.Kerala: Tapioca

2.Maharashtra: Cotton

3.West Bengal: Jute

4.Gujarat: Groundnut

(a)1, 2 and 3

(b)1, 2 and 4

(c)1, 3 and 4

(d)2, 3 and 4

[UPSC-1996]

328.Which of the following pairs of thermal power stations and their


location are correctly matched?
1.Satpura: Maharashtra

2.Neyveli: Tamil Nadu

3.Kottagudem: Andhra Pradesh

4.Kota: Rajasthan

Select the correct answer using the codes given.

(a)2, 3 and 4

(b)1 and 4

(c)1, 2 and 3

(d)1, 2, 3 and 4

[UPSC-1999]

329.Which of the following ports is/are located on the Arabian Sea coast of
India?

1.New Mangalore

2.Kandla

3.Tuticorin

4.Marmagao

Select the correct answer using the codes given.

(a)1 only
(b)1 and 3

(c)1, 2, 3 and 4

(d)1, 2 and 4

[UPSC-2003]

330.Which of the following statements are correct?

1.The Peninsular India has not undergone marine submergence since


Archean times.

2.The Aravalli mountains were one of the oldest geosynclines of the


world.

3.Dharwar system is rich in minerals.

4.The peninsular block of India is regarded as an earthquake-free zone.

Select the correct answer using the codes given.

(a)1, 2, 3 and 4

(b)2 and 4

(c)1,2and3

(d)1, 2 and 4

[UPSC-1999]

331.Which of the following statements are related to the favourable


conditions for the growth of tobacco?
1.It requires moderately high temperature.

2.It cannot be grown 1,000 m above sea level.

3.It requires a heavy average annual rainfall.

4.Red soil is suitable for its growing.

Select the correct answer using the codes given.

(a)3 and 4

(b)2 and 4

(c)1 and 4

(d)1 and 3

[UPSC-1995]

332.Which of the following statements regarding laterite soils of India are


correct?

1.Laterites are generally red in colour.

2.Laterites are rich in nitrogen and potash.

3.These are well developed in Rajasthan and UP.

4.Tapioca and cashewnuts grow well on this soil.

Select the correct answer using the codes given.

a)1,2and3
(b)2, 3 and 4

(c)1 and 4

(d)2 and 3

[UPSC-1997]

333.Which of the following states border Uttar Pradesh'?

1.Punjab

2.Rajasthan

3.Chhattisgarh

4.Jharkhand

Select the correct answer using the codes given.

(a)1, 2, 3 and 4

(b)2, 3 and 4

(c)1 and 4

(d)1 and 3

[UPSC-2005]

334.Which of the following three rivers of Peninsular India have


Amarkantak region as their source?

(a)Chambal, Betwa, Luni


(b)Godavari, Krishna, Kaveri

(c)Son, Mahanadi, Narmada

(d)Narmada, Krishna, Wainganga

[UPSC-2003]

335.Which of the following towns are not associated with silk weaving
industry in India?

1.Murshidabad

2.Ludhiana

3.Vadodara

4.Ambala

Select the correct answer using the codes given.

(a)2, 3 and 4

(b)1, 2 and 4

(c)1, 2 and 3

(d)1, 3 and 4

[UPSC-1997]

336.Which of the followings species of trees is confined to 2,000 to 2,500


metres of altitude in the Himalayas?

(a)Khair (Acacia, Catechu)


(b)Teak

(c)Deodar

(d)Rosewood

[UPSC-1995]

337.Which one among the following has the largest shipyard in India'?

(a)Kolkata

(b)Kochi

(c)Mumbai

(d)Visakhapatnam

[UPSC-2003]

338.Which one among the following has the maximum number of national
parks?

(a)Andaman and Nicobar Islands

(b)Arunachal Pradesh

(c)Assam

(d)Meghalaya

[UPSC-2008]

339.Which one among the following major Indian city is most eastwards
located?
(a)Hyderabad

(b)Bhopal

(c)Lucknow

(d)Bengaluru (Bangalore)

[UPSC-2007]

340.Which one among the following states is smallest in area?

(a)Andhra Pradesh

(b)Gujarat

(c)Karnataka

(d)Tamil Nadu

[UPSC-2003]

341.Which one among the following states of India has the lowest density of
population?

(a)Himachal Pradesh

(b)Meghalaya

(c)Arunachal Pradesh

(d)Sikkim

[UPSC-2007]

342.Which one of the following is not a tributary of the Barak River?


(a)Jiri

(b)Rukni

(c)Khaleswari

(d)Pagladiya

[UPSC-2005]

343.Which one of the following agencies is working for the development of


shrimp culture for the farmers?

(a)Fish farmers' Development Agency

(b)Technology Mission on Dairy Development

(c)Fishing Survey of India

(d)Brackish Water Fish Farmers' Development Agency

[UPSC-2004]

344.Which one of the following agricultural products is the largest earner of


foreign exchange for India?

(a)Sugarcane

(b)Jute

(c)Tea

(d)Tobacco

[UPSC-1996]
345.Which one of the following areas of India produces largest amount of
cotton?

(a)North-western India and genetic West Bengal

(b)North-western and western India

(c)Western and southern India

(d)Plains of northern India

[UPSC-1996]

346.Which one of the following characteristics is not relevant of Ganga


River'?

(a)It is a braided river with numerous channels.

(b)It has multiple intertwined sandbars.

(c)It has extensive gully erosion.

(d)Deposits enormous sediments annually into the Bay of Bengal.

[UPSC-2000]

347.Which one of the following countries has replaced Italy as the major
importer of bauxite from India'?

(a)Canada

(b)Greece

(c)Ukraine
(d)United Arab Emirates

[UPSC-1998]

348.Which one of the following dams is not meant for irrigation'?

(a)Bhavani Sagar

(b)Sivasamudram

(c)Krishnaraja Sagar

(d)Bhakra Nangal

[UPSC-1996]

349.Which one of the following dances involves solo performance?

(a)Bharatanatyam

(b)Kuchipudi

(c)Mohiniattam

(d)Odissi

[UPSC-1997]

350.Which one of the following eastflowing rivers of India has rift valley
due to downwarping?

(a)Damodar

(b)Mahanadi

(c)Son
(d)Yamuna

[UPSC-1998]

351.Which one of the following is also known as Top Slip'?

(a)Simlipal National Park

(b)Periyar Wildlife Sanctuary

(c)Manjira Wildlife Sanctuary

(d)Indira Gandhi Wildlife Sanctuary and National Park

[UPSC-2007]

352.Which one of the following is associated with Indian agriculture'?

(a)LC.A.R

(b)I. C.C.R

(c)LC.M.R

(d)I.C.W.A

[UPSC-1997]

353.Which one of the following is at the lowest height above the mean sea
love'?

(a)Delhi

(b)Jodhpur

(c)Kota
(d)Nagpur

[UPSC-1999]

354.Which one of the following is correct sequence of the stratigraphy in


India from oldest to the youngest'?

(a)Deccan lava, Gondwana sediment, Dharwarian sediment, tertiary


folding

(b)Gondwana sediment, Deccan lava, tertiary folding, Dharwarian


sediment

(c)Dharwarian sediment, tertiary folding, Deccan lava, Gondwana


sediment

(d)Dharwarian sediment, Gondwana sediment, Deccan Lava, tertiary


folding

[UPSC-2005]

355.Which one of the following is located in the Bastar region?

(a)Bandhavgarh National Park

(b)Dandeli Sanctuary

(c)Rajaji National Park

(d)Indravati National Park

[UPSC-2007]

356.Which one of the following is most abundant in Kerala'?


(a)Thorium

(b)Uranium

(c)Mica

(d)Monazite

[UPSC-2002]

357.Which one of the following is not a biosphere reserve'?

(a)Agasthyamali

(b)Nallamalai

(c)Nilgiri

(d)Panchmarhi

[UPSC-2005]

358.Which one of the following is not a causative factor with respect to poor
coverage of forest areas in Jammu and Kashmir?

(a)Low amount of rainfall

(b)Large areas under cultivation

(c)Steep barren slopes

(d)Snow-covered peaks

[UPSC-1997]

359.Which one of the following is not a lagoon?


(a)Ashtamudi lake

(b)Chilka lake

(c)Periyar lake

(d)Pulicat lake

[UPSC-2002]

360.Which one of the following is the correct sequence of the given hills
starting from the north going towards the south?

(a)Nallamalai Hills, Nilgiri Hills, Javadi Hills, Annamalai Hills

(b)Annamalai Hills, Javadi Hills, Nilgiri Hills, Nallamalai Hills

(c)Nallamalai Hills, Javadi Hills, Nilgiri Hills, Annamalai Hills

(d)Annamalai Hills, Nilgiri Hills, Javadi Hills, Nallamalai Hills

[UPSC-2005]

361.Which one of the following is the correct sequence of the given Indian
cities in the decreasing order of their normal annual rainfall?

(a)Kochi, Kolkata, Delhi, Patna

(b)Kolkata, Kochi, Patna, Delhi

(c)Kochi, Kolkata, Patna, Delhi

(d)Kolkata, Kochi, Delhi, Patna

[UPSC-2005]
362.Which one of the following is the correct statement?

The Alpine vegetation in the eastern Himalayas is found up to greater


altitudes than the western Himalayas because

(a)the eastern Himalayas are higher than the western Himalayas.

(b)the eastern Himalayas are higher than the Equator atsea coast than the
western Himalayas.

(c)the eastern Himalayas receive more rainfall than the western


Himalayas.

(d)the eastern Himalayas soils are more fertile than those of the western
Himalayas.

[UPSC-2005]

363.Which one of the following is the junction point of the Eastern Ghats
and Western Ghats?

(a)Javadi Hills

(b)Annamalai Hills

(c)Nilgiri Hills

(d)Shevaroy Hills

[UPSC-2004]

364.Which one of the following legislations does not deal with the
protection of environment?
(a)The Water (Cess) Act, 1977

(b)The Forest (Conservation) Act, 1980

(c)The Public Liability Insurance Act, 1991

(d)The Port Laws Amendment Act, 1997

[UPSC-1999]

365.Which one of the following mountain chains has two dissimilar types of
vegetation on its two slopes?

(a)Aravalli

(b)Vindhyas

(c)Eastern Ghats

(d)Western Ghats

[UPSC-1996]

366.Which one of the following mountain ranges is spread over only one
state in India?

(a)Aravalli

(b)Satpura

(c)Ajanta

(d)Sahyadri

[UPSC-1995]
367.Which one of the following National Highways passes through
Maharashtra, Chhattisgarh and Orissa?

(a)NH 4

(b)NH 5

(c)NH 6

(d)NH 7

[UPSC-2007]

368.Which one of the following ores is abundant in the region of Balaghat-


BhandaraNagpur?

(a)Iron

(b)Manganese

(c)Mica

(d)Bauxite

[UPSC-2001]

369.Which one of the following pairs is correctly matched'?

[UPSC-1997]
370.Which one of the following pairs is not correctly matched'?

[UPSC-1998]

371.Which one of the following pairs is not correctly matched?

[UPSC-2003]

372.Which one of the following pairs is not correctly matched'?

[UPSC-2006]
373.Which one of the following pairs is NOT correctly matched?

[UPSC-2006]

374.Which one of the following pairs is not correctly matched?

[UPSC-2004]

375.Which one of the following pairs is not correctly matched'?


[UPSC-2005]

376.Which one of the following pairs of cities has recently been connected
through a six-lane express way?

[UPSC-1998]

377.Which one of the following pairs of folk dance forms and states is
correctly matched?

[UPSC-1999]
378.Which one of the following pairs of NOT correctly matched'?

[UPSC-1995]

379.Which one of the following pairs of primitive tribes and place of their
inhabitation is NOT correctly matched?

[UPSC-2000]

380.Which one of the following pairs of states and tribes is not correctly
matched?

[UPSC-1999]

381.Which one of the following places is located at the confluence of


Alaknanda and Bhagirathi?

(a)Vishnuprayag

(b)Devaprayag
(c)Rudraprayag

(d)Karnaprayag

[UPSC-1999]

382.Which one of the following ports in Gujarat is famous for ship breaking
and ship repairing'?

(a)Porbandar

(b)Patan

(c)Pipavav

(d)Mandvi

[UPSC-2000]

383.Which one of the following ports of India handles the highest tonnage
of import cargo?

(a)Calcutta

(b)Kandla

(c)Mumbai

(d)Visakhapatnam

[UPSC-2000]

384.Which one of the following power plants is not a thermal plant'?

(a)Faridabad
(b)Dhuvaram

(c)Idukki

(d)Loktak

[UPSC-1998]

385.Which one of the following problems is not encountered in Indira


Gandhi Canal command area?

(a)Increase in soil salinity

(b)Progressive water logging

(c)Aeolian sitting of the canal

(d)Decline of water supply

[UPSC-2000]

386.Which one of the following problems is not encountered in Indira


Gandhi Canal command area?

(a)Increase in soil salinity

(b)Progressive water logging

(c)Aeolian silting of the canal

(d)Decline in water supply

[UPSC-1995]
387.Which one of the following regions of India has the highest percentage
of its area under forest cover?

(a)Andaman and Nicobar Islands

(b)Arunachal Pradesh

(c)Mizoram

(d)Jammu and Kashmir

[UPSC-2005]

388.Which one of the following regions of India has the potentiality of


harnessing tidal power?

(a)Gulf of Khambhat

(b)Gulf of Mannar

(c)Palk Strait

(d)Kerala Coast

[UPSC-1999]

389.Which one of the following rivers does not originate in India'?

(a)Beas

(b)Chenab

(c)Ravi

(d)Sutlej
[UPSC-2009]

390.Which one of the following rivers does not represent antecedent


drainage?

(a)Chenab

(b)Sutlej

(c)Ravi

(d)Subansiri

[UPSC-1999]

391.Which one of the following rivers originates at Amarkantak?

(a)Damodar

(b)Mahanadi

(c)Narmada

(d)Tapti

[UPSC-2007]

392.Which one of the following sets of commodities is exported to India by


and and semi-arid countries in the Middle East?

(a)Raw wool and carpets

(b)Fruits and palm oil

(c)Precious stones and pearls


(d)Perfume and coffee

[UPSC-1996]

393.Which one of the following sets of commodities is imported by India


from south-west Asian countries?

(a)Raw wool and carpets

(b)Dates and olives

(c)Precious stones and pearls

(d)Perfumes and coffee

[UPSC-1998]

394.Which one of the following sets of states benefits the most from the
Konkan Railway?

(a)Goa, Karnataka, Maharashtra, Kerala

(b)Madhya Pradesh, Maharashtra, Tamil Nadu, Kerala

(c)Tamil Nadu, Kerala, Goa Maharashtra

(d)Gujarat, Maharashtra, Goa, Tamil Nadu

[UPSC-1998]

395.Which one of the following species has the largest area coverage in
India?

(a)Chir
(b)Deodar

(c)Sal

(d)Teak

[UPSC-1999]

396.Which one of the following statement is correct'?

(a)Alliance Air is a whollyowned subsidiary of Indian Airlines.

(b)The Airports Authority of India manages seven of the country's


international airports.

(c)The Airports Authority of India is the regulatory organization for


enforcing civil air regulations in India.

(d)It is the function of Directorate General of Civil Aviation to plan and


construct runway and terminal buildings and to provide air safety
services.

[UPSC-2003]

397.Which one of the following statement is not correct?

(a)Rourkela Steel Plant, the first integrated steel plant in the Public Sector
of India, was set up with the Soviet Union collaboration.

(b)Salem Steel Plant is a premier producer of stainless steel in India.

(c)Maharashtra Elektrosmelt Ltd. is a subsidiary of the Steel Authority of


India Ltd.
(d)Visakhapatnam Steel plant is a unit of the Rashtriya Ispat Nigam Ltd.

[UPSC-2005]

398.Which one of the following statements is not correct?

(a)Mahanadi River rises in Chhattisgarh.

(b)Godavari River rises in Maharashtra.

(c)Cauvery River rises in Andhra Pradesh.

(d)Tapti River rises in Madhya Pradesh.

[UPSC-2006]

399.Which one of the following statements about Koyali is correct?

(a)It is a hydroelectric plant in UP.

(b)It is a historic, cultural tourist centre in Rajasthan.

(c)It is an oil refinery located in Gujarat.

(d)It is a site of steel plant under construction in MR.

[UPSC-2000]

400.Which one of the following statements explains that mining is `an


increasing coast' economic activity?

(a)Mine workers enhance their wages.

(b)Best and most easily available mineral deposits are mined first.

(c)Mining technique becomes expensive.


(d)Mining royalties increase.

[UPSC-2001]

401.Which one of the following statements is correct?

(a)Goa attained full statehood in 1987.

(b)Diu is an island in the Gulf of Khambhat.

(c)Daman and Diu were separated from Goa by the 56th Amendment of
the Constitution of India.

(d)Dadra and Nagar Haveli was under French colonial rule till 1954.

[UPSC-2000]

402.Which one of the following statements is correct?

(a)The mountains of the Peninsular India are about 40 million years old.

(b)The Eastern Ghats extend as a continuous range for a distance of about


1,600 km.

(c)The Western Ghats consist of a series of discontinuous hill with no


structural continuity.

(d)The Siwalik Ranges have hogback appearance.

[UPSC-2004]

403.Which one of the following statements is correct'?

(a)Diamonds occur in the sediments of Dudapah series.


(b)Natural gas occurs in the Gondwana beds.

(c)Mica occurs in abundance at Kondarma

(d)Dharwars are famous for petroleum.

[UPSC-1997]

404.Which one of the following statements is correct about Deccan


Odysseys?

(a)It is a book on Chhatrapati Shivaji.

(b)It is a warship recently acquired by the Indian Navy.

(c)It is a recently started air service between Mumbai and Colombo.

(d)It is a luxury train which travels though Maharashtra and includes Goa
in its journey.

[UPSC-2004]

405.Which one of the following statements is NOT correct'?

(a)India is the second largest producer of nitrogenous fertilizers in the


world.

(b)India is the ninth largest steelproducing country in the world.

(c)India is the second largest producer of silk in the world.

(d)India ranks third in the world in coal production.

[UPSC-2003]
406.Which one of the following statements is not correct'?

(a)The largest Buddhist monastery in India is in Assam.

(b)The language Konyak is spoken in Nagaland.

(c) The largest river island in the world is in Assam.

(d)Sikkim is the least-populated state of the Indian Union.

[UPSC-2004]

407.Which one of the following statements is not correct?

(a)There is no definition of scheduled tribesin the Constitutions of India.

(b)North-East India accounts for a little over half of the country's tribal
population.

(c)The people known as Todas live in the Nilgiri areas.

(d)Lotha is a language spoken in Nagaland.

[UPSC-2005]

408.Which one of the following statements is not correct?

(a)The Western Ghats are relatively lower in their northern region.

(b)The Anamudi is the highest peak in the Western Ghats.

(c)Tapi River lies to the south of Satpura.

(d)The Narmada and the Tapti River Valleysare said to be old rift valleys.

[UPSC-2005]
409.Which one of the following statements is not correct?

(a)Maximum number of cotton yarn and textile mills are located in


Gujarat.

(b)Carpet industry is chiefly in the state of Uttar Pradesh.

(c)Cotton textile industry provides jobs to the maximum number of


people.

(d)The first cotton mill was set up at Fort Gloster in Calcutta.

[UPSC-1996]

410.Which one of the following statements is not true of the Konkan


Railways'?

(a)The total length is about 760 km.

(b)Itruns through the states of Karnataka, Goa, Maharashtra and Kerala.

(c)It is the only rail route that cuts across the Western Ghats.

(d)The Konkan Railway Construction Company which came into being


raised money through Public Issue.

[UPSC-1999]

411.Which one of the following statements is not true'?

(a)Ghaggar's water is utilized in the Indira Gandhi Canal.

(b)Narmada rises from Amarkantak region.

(c)Nizam Sagar is situated on the Manjra River.


(d)Penganga is a tributary of tee Godavari.

[UPSC-2000]

412.Which one of the following states has the longest coastline'?

(a)Maharashtra

(b)Tamil Nadu

(c)Andhra Pradesh

(d)Karnataka

[UPSC-1999]

413.Which one of the following states has a higher percentage of rank


irrigation'?

(a)Tamil Nadu

(b)West Bengal

(c)Kerala

(d)Karnataka

[UPSC-2005]

414.Which one of the following states is the largest producer of thermal


power in India'?

(a)Bihar

(b)Tamil Nadu
(c)Maharashtra

(d)Chhattisgarh

[UPSC-2002]

415.Which was the capital of Andhra when it was made a separate state in
the year 1953'?

(a)Guntur

(b)Kurnool

(c)Nellore

(d)Warangal

[UPSC-2008]

416.With reference to India, which one of the following statements is not


correct'?

(a)About onethird of the area of the country records more than 750
millimetres of annual rainfall.

(b)The dominant source of irrigation in the country is wells.

(c)Alluvial soil is the predominant type of soil in northern plains of the


country.

(d)The mountain areas account for about 30 percent of the surface area of
the country.

[UPSC-2002]
417.With reference to Indian agriculture, which one of the following
statements is correct'?

(a)About 90 per cent of the areas under pulses in India is rain fed.

(b)The share of pulses in the gross cropped are at the national level has
doubled in the last 2decades.

(c)India accounts for about 15 per cent of the total area under rice in the
world.

(d)Rice occupies about 34 per cent of the gross cropped area of India.

[UPSC-2002]

418.With reference to Indian satellite and their launchers, consider the


following statements:

1.All the INSAT series of satellites had their launch abroad.

2.PSLVs were used to launch IRS series of satellites.

3.India used the indigenously built cryogenic engines for the first time for
powering the third stages of GSLV.

4.GSAT, launched in year 2001, has payloads to demonstrate digital


broadcast and internet services.

Which of these statements are correct?

(a)1, 2, 3 and 4

(b)2, 3 and 4
(c)1, 2 and 4

(d)1 and 3

[UPSC-2002]

419.With reference to Indian transport systems, consider the following


statements:

1.Indian railway system is the largest in the world.

2.National highways cater to 45 per cent of the total transport demand.

3.Among the states, Kerala has the highest density of surface road.

4.National Highway 7 is the longest in the country.

Which of these statements are correct'?

(a)1 and 2

(b)1 and 3

(c)2 and 3

(d)2 and 4

[UPSC-2002]

420.With which one of the following is BRIT (Government of India)


engaged?

(a)Railway wagons

(b)Information technology
(c)Isotope technology

(d)Road transport

[UPSC-2008]

421.With which one of the following mountain tribes did the British first
come into contact with after the Grant of Diwani in the year 1765?

(a)Garos

(b)Khasis

(c)Kukis

(d)Tiperahs

[UPSC-2002]

422.With which one of the following rivers is the Omkareshwar Project


associated?

(a)Chambal

(b)Narmada

(c)Tapi

(d)Bhima

[UPSC-2008]

423.Xeorphytic vegetation is a characteristic feature of

(a)Chhota Nagpur plateau.


(b)Khasi hills.

(c)Eastern Ghats.

(d)Kutch.

[UPSC-2004]

424.Indo-Gangetic plain is fertile because it is covered by

(a)saline layer.

(b)volcanic debris.

(c)metamorphic rocks.

(d)transported alluvium.

[UGC D-2005]

425.Which state has the highest length of irrigation canal system'?

(a)UP

(b)Raj asthan

(c)Punjab

(d)Haryana

[UGC D-2005]

426.Which one of the following coal fields has been identified to have a
high potential of commercial exploitations of coalbed methane gas'?

(a)Jharia
(b)Kanhan Valley

(c)Talcher

(d)Godavari Valley

[UGC D-2005]

427.Where is the National Remote Sensing Agency (NRSA) located'?

[UGC D-2005]

(a)Delhi

(b)Kolkata

(c)Chennai

(d)Hyderabad

428.Khasis are a major mongoloid tribe of

(a)Manipur.

(b)Assam.

(c)Meghalaya.

(d)Orissa.

[UGC D-2005]

429.Which one of the North-East states has the lowest concentration of tribal
population'?

(a)Meghalaya
(b)Mizoram

(c)Assam

(d)Arunachal Pradesh

[UGC D-2005]

430.Arrange the following in the correct sequence of India's non-


conventional energy sources in respect of their potentiality:

1.Solar

2.Biofuel

3.Wind

4.Ocean thermal

(a)1, 3,4, 2

(b)1, 2, 3, 4

(c)1, 4, 2, 3

(d)1, 3, 2, 4

[UGC D-2005]

431.Jharia coal fields arelocated in

(a)Bihar.

(b)Jharkhand.
(c)Orissa.

(d)Chhattisgarh.

[UGC D-2006]

432.When was Project Tiger, one of the most intensive conservation efforts,
launched in India?

(a)1963

(b)1967

(c)1973

(d)1977

[UGC D-2006]

433.Which state is the largest producer of wheat in India?

(a)Haryana

(b)Punjab

(c)Uttar Pradesh

(d)Bihar

[UGC D-2006]

434.The petro-dollar economy has highly influenced one of the following


states more than others.

(a)Tamil Nadu
(b)Kerala

(c)Punjab

(d)Gujarat

[UGC D-2006]

435.Population distribution in India at state level is best represented by

(a)dot method.

(b)pie diagrams.

(c)choropleth.

(d)isopleth.

[UGC D-2006]

436.The Indian Railway Network is ranked in the world.

(a)first

(b)second

(c)third

(d)fourth

[UGC D-2006]

437.Which of the following is associated with the cotton textile industry?

(a)Pune
(b)Varanasi

(c)Ahmedabad

(d)Chandigarh

[UGC D-2006]

438.Kargil is located on the banks of

(a)Indus River.

(b)Jhelum River.

(c)Suru River.

(d)Chenab River.

[UGC D-2006]

439.Imphal is the capital of

(a)Manipur.

(b)Nagaland.

(c)Meghalaya.

(d)ArunachalPradesh.

[UGC D-2006]

440.McMohan Line is an international boundary between

(a)India and Bangladesh.


(b)India and China.

(c)India and Pakistan.

(d)France and Germany.

[UGC D-2006]

441.In which of the following states did the Chipko Movement start?

(a)Madhya Pradesh

(b)Jharkhand

(c)Uttaranchal

(d)Rajasthan

[UGC D-2007]

442.Which of the following states has the maximum number of common


borders with other Indian states?

(a)West Bengal

(b)Madhya Pradesh

(c)Chhattisgarh

(d)Karnataka

[UGC D-2007]

443.Which river of the following is older than the Himalayas'?

(a)Sutlej
(b)Ganga

(c)Beas

(d)Ravi

[UGC D-2007]

444.Which one of the following states experienced heavy snowfall during


February 2005?

(a)Assam

(b)J & K

(c)Mizoram

(d)Sikkim

[UGC D-2007]

445.The areas lying to the east of the Western Ghats covering Karnataka,
interior Tamil Nadu, western Andhra Pradesh and central Maharashtra are
known as

(a)sub-tropical steppe region.

(b)tropical steppe region.

(c)tropical savannah.

(d)tropical rainy climate.

[UGC D-2008]
446.Which state of India is the largest producer of soyabean?

(a)Rajasthan

(b)Gujarat

(c)Uttar Pradesh

(d)Madhya Pradesh

[UGC D-2008]

447.Which state of India is the largest producer of wheat'?

(a)Punjab

(b)Haryana

(c)Uttar Pradesh

(d)Madhya Pradesh

[UGC D-2008]

448.Which state of India has the highest productivity of wheat per hectare?

(a)Bihar

(b)Uttar Pradesh

(c) Punjab

(d)Haryana

[UGC D-2008]
449.Which one of the following places of India was worst affected by
Tsunami in 2004?

(a)Nagapattinam

(b)Kanyakumari

(c)Vishakhapatnam

(d)Chennai

[UGC D-2008]

450.Arrange the following river systems of India in sequence from north to


south.

(a)Godavari, Mahanadi, Kaveri, Krishna

(b)Mahanadi, Godavari, Krishna, Kaveri

(c)Krishna, Kaveri, Godavari, Mahanadi

(d)Kaveri, Godavari, Mahanadi, Krishna

[UGC D-2008]

451.Which city of Gujarat is associated with salt production'?

(a)Surat

(b)Kandla

(c)Vadodara

(d)Gandhidham
[UGC D-2009]

452.Match List I with List II and select the correct answer using the codes
given.

Codes:

[UGC D-2009]

453.Which of the following cities is situated on the mouth of river Tapi


(Tapti)?

(a)Ankleshwar

(b)Vadodara

(c)Ahmedabad

(d)Surat

[UGC D-2009]

454.Which one of the following is in the rain shadow of the Western Ghats?
(a)Mahabaleshwar

(b)Baramati

(c)Panchgani

(d)Ratnagiri

[UGC D-2009]

455.Arrange the following states in descending order of their geographical


area.

1.Maharashtra

2.Karnataka

3.Rajasthan

4.Tamil Nadu

(a)3,1,4,2

(b)1,3,4,2

(c)4,3,1,2

(d)3, 1 2, 4

[UGC D-2009]

456.The Census of India recognizes how many size classes of towns?

(a)Five
(b)Three

(c)Six

(d)Four

[UGC J-2005]

457.Which one of the following has contributed to low level of economic


development in India'?

(a)Slow decline in share of agriculture to Gross Domestic Product

(b)Rapid increase in share of employment in secondary sector

(c)Rapid increase in share of employment in tertiary sector

(d)Outmigration of skilled population to developed countries

[UGC J-2005]

458.Narora nuclear power station is located in

(a)Maharashtra.

(b)TamilNadu.

(c)Uttar Pradesh.

(d)Rajasthan.

[UGC J-2005]

459.Which of the following statements about Green Revolution in India is


NOT true'?
(a)It was characterized by the use of high-yielding variety of seeds and
intensive irrigation and application of chemical fertilizers.

(b)It started with increased wheat production in Punjab, Haryana and


western UP.

(c)It has resulted in increase in disparity in rural income.

(d)It has caused substantial increase in the production of millets.

[UGC J-2007]

460.Mangrove forests are mainly found along the sea faces of the delta of
which one of the following rivers?

(a)Mahanadi

(b)Krishna

(c)Kaveri

(d)Ganga

[UGC J-2007]

461.Arunachal Pradesh was formerly known as

(a)NWEP.

(b)NEFA.

(c)Buffer zone.

(d)Frontier.
[UGC J-2007]

462.Winter rains in the North Indian Plains are caused by depressions


coming from the

(a)Arabian Sea.

(b)Bay of Bengal.

(c)Mediterranean Sea.

(d) Pacific Ocean.

[UGC J-2007]

463.Match List I with List II and select the correct answer using the codes
given.

Codes:

[UGC J-2007]

464.The leading saffron-producing state of India is


(a)Himachal Pradesh.

(b)Arunachal Pradesh.

(c)Jammu and Kashmir.

(d)Nagaland.

[UGC J-2007]

465.Which of the following pairs does NOT match correctly'?

[UGC J-2007]

466.The predominant soil type developed on the Archaean and Cuddapah


geological systems in India is

(a)black soil.

(b)alluvial soil.

(c)red soil.

(d)intrazonal soil.

[UGC J-2007]

467.Which of the following pairs does NOT match correctly'?


[UGC J-2007]

468.Match List I with List II and select the correct answer using the codes
given.

Codes:

[UGC J-2008]

469.Match List I with List II and select the correct answer using the codes
given.
Codes:

[UGC J-2008]

470.The origin of monsoon in the Indian subcontinent is largely linked with

(a)El Nino and ENSO events.

(b)localized weather perturbations.

(c)shape of the Indian subcontinent.

(d)sea surface temperature variation in the Indian Ocean.

[UGC J-2008]

471Which one of the following sequences of states is in the decreasing order


of sex ratio'?

(a)Andhra Pradesh, Tamil Nadu, Chhattisgarh, Kerala

(b)Tamil Nadu, Kerala, Andhra Pradesh, Chhattisgarh


(c)Kerala, Chhattisgarh, Tamil Nadu, Andhra Pradesh

(d)Chhattisgarh, Andhra Pradesh, Kerala, Tamil Nadu

[UGC J-2008]

472.The Western Ghats in Maharashtra is known as

(a)Nilgiris.

(b)Cardamom Hills.

(c)Annamalai.

(d)Sahyadris.

[UGC J-2008]

473.Which of the following states is the largest producer of wheat?

(a)Uttar Pradesh

(b)Haryana

(c)Punj ab

(d)Rajasthan

[UGC J-2011]

474.Electricity production in India through coal is

(a)60 per cent.

(b)30 per cent.


(c)90 per cent.

(d)10 per cent.

[UGC J-2011]

475.Which of the following statement are correct?

1.The soils of the Indian plateau are transported ones.

2.Rivers have cut alluvial terraces in the Himalayas.

3.Red soils occur in Tamil Nadu, south Karnataka plateau and Kerala.

4.Petroleum is the major source of commercial energy in India.

Select the answer from the codes given.

(a)1 and 3

(b)2, 3and4

(c)2 and 3

(d)2 and 4

[UGC J-2011]

476.Match List I with List II and select the correct answer using the codes
given.
[UGC J-2011]

Codes:

477.Find out the correct sequence of the following passes from west to east.

(a)Lipu Lekh, Nathu La, Shipki La, Zojila

(b)Zojila, Shipki La, Nathu La, Lipu Lekh

(c)Zojila, Shipki La, Lipu Lekh, Nathu La

(d)Shipki La, Zojila, Nathu La, Lipu Lekh

[UGC J-2011]

478.Which of the following is the correct sequence in terms of east-flowing


rivers from north to south in India?

(a)Mahanadi, Godavari, Krishna, Pennar, Kaveri


(b)Mahanadi, Krishna, Godavari, Pennar, Kaveri

(c)Mahanadi, Godavari, Krishna, Kaveri, Pennar

(d)Mahanadi, Krishna, Godavari, Pennar, Kaveri

[UGC J-2011]

479.After economic liberalization started in India in 1991, the per capita


state incomes in the country have

(a)converged.

(b)remained unchanged.

(c)diverged.

(d)spread.

[UGC J-2011]

480.Given below are two statements, one labelled as Assertion (A) and the
other labelled as Reason (R). Select your answer from the codes given.

Assertion (A): Dandakaranya was the first large interstate effort to


reclaim land for the settlement of refugees.

Reason (R): Dandakaranya region lies partly in Bihar and partly in West
Bengal.

(a)Both (A) and (R) are correct and (R) explains (A).

(b)Both (A) and (R) are correct, but (R) does not explain (A).

(c)(A) is correct, but (R) is false.


(d)(A) is false, but (R) is true.

[UGC J-2011]

481.Which of the following ports has an outer harbour for export of iron
ore?

(a)Kolkata

(b)Mumbai

(c)Vishakhapatnam

(d)Cochin

[UGC J-2010]

482.In which of the following states uranium is embedded in the igneous


and metamorphic rocks?

(a)Tamil Nadu

(b)Maharashtra

(c)Rajasthan

(d)Kerala

[UGC J-2010]

483.Which is the largest physiographic division of India?

(a)Great Northern Plains

(b)Great Indian Plateau


(c)Greater Himalayas

(d)The Coastal Plains

[UGC J-2010]

484.Which of the following earthquakes was severely devastating and


disastrous that occurred in India?

(a)Kangra earthquake of 1905

(b)Bihar earthquake of 1934

(c)Gujarat earthquake of 2001

(d)Koyna earthquake of 1967

[UGC J-2010]

485.Match List I with List II and select the correct answer using the codes
given.

Codes:
[UGC J-2010]

486.Economic reforms in India are leading to

1.concentration of economic activities.

2.widening gaps between the rich and poor people.

3.higher growth rate of metropolitan areas.

4.low cost access to urban amenities and services.

(a)1 and 2 are correct.

(b)1, 2and 3are correct.

(c)2and 4are correct.

(d)1, 3and 4are correct.

[UGC J-2010]

487.Given below are two statements, one labelled as Assertion (A) and the
other labelled as Reason (R). Select your answer from the codes given.

Assertion (A): Many jute mills in India were closed down after
independence.
Reason (R): There is acute shortage of raw materials and also there is
shortfall in foreign demand for jute goods.

(a)Both (A) and (R) are true and (R) is correct explanation of (A).

(b)Both (A) and (R) are true, but (R) is not the correct explanation of (A).

(c)(A) is true, but (R) is false.

(d)(A) is false, but (R) is true.

[UGC D-2011]

488.Which of the following scheduled tribe has a dispersed core'?

(a)AO

(b)Garo

(c)Santhal

(d)Bhil

[UGC D-2011]

489.Match List I with List II and select the correct answer using the codes
given.

Codes:
[UGC D-2011]

490.What is the correct sequence of area under soil cover in India in


descending order'?

(a)Alluvial soils, black soils, red soils, laterite soils

(b)Alluvial soils, red soils, black soils, laterite soils

(c)Red soils, laterite soils, black soils, alluvial soils

(d)Red soils, black soils, laterite soils, alluvial soils

[UGC D-2011]

491.The office of Registrar General and Census Commissioner of India is


responsible for

1.conducting decennial census and publication of its related data.

2.publication of annual landuse data.

3.publication of annual health and education data.

4.publication of annual vital statistics based on sample registration


system.

(a)land 2are correct.


(b)1 and 4are correct.

(c)land 3are correct.

(d)All are correct.

[UGC D-2010]

492.Which of the following steel plants is port-based'?

(a)Bhilai

(b)Durgapur

(c)Vishakhapatnam

(d)Rourkela

[UGC D-2010]

493.Which of the following is not an Agroclimatic region of India'?

(a)East coast plains and hills

(b)The Eastern plateau and hills

(c)The Western Ghats

(d)The Trans-Ganga plains

[UGC D-2010]

494.Which one of the following is wrongly matched'?


[UGC D-2010]

495.Given below are two statements, one labelled as Assertion (A) and the
other labelled as Reason (R). Select your answer from the codes given.

Assertion (A): The Northern plains and Eastern coastal areas possess a
dense network of railways in India.

Reason (R): These regions have welldeveloped industries.

(a)Both (A) and (R) are true and (R) is the correct explanation of (A).

(b)Both (A) and (R) are true, but (R) is not the correct explanation of (A).

(c)(A) is true, but (R) is false.

(d)(A) is false, but (R) is true.

[UGC D-2010]

496.Match List I with List II and select the correct answer using the codes
given.
Codes:

[UGC D-2010]

497.Consider the following:

1.Jammu hills

2.Mikir hills

3.Zaskar range

On which of the above is annual rainfall generally above 100 cm?

(a)1 and 2

(b)2 only

(c)1 and 3

(d)1, 2 and 3

[UPSC-2008]

498.Consider the following statements:

1.Bajra (peal millet) is essentially a rabi crop in India.


2.Though maize is cultivated as a rabi crop in some areas, it is also grown
as kharif crop.

Which of the statements given above is/ are correct'?

(a)1 only

(b)2 only

(c)Both 1 and 2

(d)Neither 1 nor 2

[UPSC-2008]

499.In which one of the following is the Himalayan pass Shipki La located?

(a)Chandra Valley

(b)Hunza Valley

(c)Nubra Valley

(d)Sutlej Valley

[UPSC-2008]

500.Consider the following:


Which of the statements given above is/ are correctly matched'?

(a)1 only

(b)2 and 2

(c)2 and 3

(d)1, 2 and 3

[UPSC-2008]

501.Consider the following statements:

1.Most of the iron ores found in India are of the limonite type.

2.The magnetite type of iron ore occurs in the Dharwarand Cuddapah


rock systems of the Peninsular India.

3.In India, sphalerite and galena occur in the pre-Cambrian rocks.

Which of the statements given above is/ are correct'?

(a)1 and 3
(b)2 only

(c)2 and 3

(d)1, 2 and 3

[UPSC-2008]

502.Match List I with List II and select the correct answer using the codes
given.

Codes:

[UPSC-2008]

503.Consider the following statements:

1.Calcareous alluvial soils are found in the north-eastern districts of Uttar


Pradesh.
2.Calcareous sierozemic soils are found in the coastal districts of Orissa
and north Andhra Pradesh.

Which of the statements given above is/ are correct'?

(a)1 only

(b)2 only

(c)Both 1 and 2

(d)Neither 1 nor 2

[UPSC-2008]

504.In which of the following districts is the highest point of Eastern Ghats
located'?

(a)Ganjam

(b)Visakhapatnam

(c)Kurnool

(d)North Arcot

[UPSC-2008]

505.Which one of the following statement is not correct?

(a)The Karnaphuli River originates in Meghalaya and flows into


Bangladesh.

(b)The Sabarmati River originates in Rajasthan and flows into the Gulf of
Khambhat.
(c)The Ghaghara River originates in Tiber and enters India through
Nepal.

(d)The Jhelum River originates in Kashmir and enters Pakistan through


Pir Panjal Range.

[UPSC-2008]

506.Consider the following:

Which of the pair/pairs given above is/ are correct'?

(a)1 only

(b)2 and 2

(c)3 only

(d)1, 2 and 3

[UPSC-2008]

507.Consider the following statements:

1.Gondwana coal is a laminated bituminous coal.

2.Gondwana coal is completely free from moisture, sulphur and


phosphorus.
Which of the statements given above is/ are correct'?

(a)1 only

(b)2 only

(c)Both 1 and 2

(d)Neither 1 nor 2

[UPSC-2008]

508.Which one of the following national parks has a climate that varies from
tropical to subtropical, temperate and arctic'?

(a)Khangchendzonga National Park

(b)Nandadevi National Park

(c)Neora National Park

(d)Namdapha National Park

[UPSC-2008]

509.Match List I with List II and select the correct answer using the codes
given.
Codes:

[UPSC-2008]

510.Among the following states, which one has the largest forest area'?

(a)Gujarat

(b)Karnataka

(c)Orissa

(d)Tamil Nadu

[UPSC-2008]

511.Among the following states, which one has the largest net irrigated
area'?
(a)Andhra Pradesh

(b)Maharashtra

(c)Gujarat

(d)Tamil Nadu

[UPSC-2008]

512.Consider the following statements:

1.Nokrek Biosphere Reserve contains tropical wet evergreen forest type


of vegetation.

2.The river Pindar flows through Nandadevi Biosphere Reserve.

Which of the statements given above is/ are correct'?

(a)1 only

(b)2 only

(c)Both 1 and 2

(d)Neither 1 nor 2

[UPSC-2008]

513.Which one of the following pairs of cities hasthe largest annual average
rainfall difference as compared to the other three pairs'?

(a)Jabalpur and Nagpur

(b)Mumbai and Pune


(c)Kolkata and Bhubaneswar

(d)Guwahati and Shillong

[UPSC-2008]

514.In Thornthwaite's scheme of classification of climate, what does `E'


indicate in the context of India'?

(a)Semi-arid climate

(b)Humid climate

(c)Arid climate

(d)Dry subhumid climate

[UPSC-2008]

515.Consider the following statements:

1.In India, natural rubber is produced in southern India and Andaman and
Nicobar Islands only.

2.Among the coffee-growing states of India, the lowest average yield


perhectare of plucked coffee is in Kerala.

Which of the statements given above is/ are correct'?

(a)1 only

(b)2 only

(c)Both 1 and 2
(d)Neither 1 nor 2

[UPSC-2008]

516.Among the following states of India, which one has the oldest rock
formations in the country?

(a)Assam

(b)Bihar

(c)Karnataka

(d)Uttar Pradesh

[UPSC-2008]

517.Match List I with List II and select the correct answer using the codes
given.

Codes:
[UPSC-2008]

518.Which one of the following rivers has the largest catchment area'?

(a)Kaveri

(b)Krishna

(c)Mahanadi

(d)Narmada

[UPSC-2008]

519.Consider the following statements:

1. Cherrapunji is located on the south side of Garo hills.

2.Cherrapunji is about 1,700 m above the mean sea level.

Which of the statements given above is/ are correct?

(a)1 only

(b)2 only

(c)Both 1 and 2

(d)Neither 1 nor 2

[UPSC-2008]

520.Which of the following pairs is not correctly matched?

Irrigation project: District


(a)Bargi project: Jabalpur

(b)Kadana project: Panchmahals

(c)Pochampad project: Guntoor

(d)Malaprabha: Belgaum

[UPSC-2008]

521.On which one of the following rivers are Tikarpara and Naraj dams
constructed under a multipurpose project?

(a)Damodar

(b)Kosi

(c)Mahanadi

(d)Sutlej

[UPSC-2008]

522.Consider the following statements:

1.As per Census 2001, the population growth rate of Tamil Nadu during
1991-2001 was the lowest in India.

2.As per Census 2001, the population growth rate of Nagaland during
1991-2001 was the highest in India.

Which of the statements given above is/ are correct'?

(a)1 only
(b)2 only

(c)Both 1 and 2

(d)Neither 1 nor 2

[UPSC-2008]

523.Consider the following:

1.Andaman and Nicobar Islands forests

2.Nilambur tract of Kerala

3.Low outcrop of Garo and Khasi hills

4.Mikir hills up to 1,000 meters

Which of the above areas have tropical wet evergreen forests'?

(a)1 and2

(b)2 and3

(c)1, 2 and 4

(d)1, 2, 3 and 4

[UPSC-2008]

524.With reference to Indian forest, consider the following pairs:

1.Tropical moist deciduous forest Sandalwood (Santalum album)

2.Tropical dry deciduous forest: Sal (Shorea rubusta)


3.Tropical thorn Forest: Shisham (Dalbergia sissoo)

Which of the statements given above is/ are correctly matched'?

(a)1 only

(b)1 and 2

(c)2 and 3

(d)1, 2 and 3

[UPSC-2008]

525.With reference to north-east monsoon (winter monsoon) in India,


consider the following statements:

1.Unlike the summer monsoon, there is no easterly jet stream over the
Peninsula during the winter monsoon.

2.More than two-thirds of annual rainfall in Coimbatore, Salem and


Nilgiri districts occurs from October to December.

Which of the statements given above is/ are correct?

(a)1 only

(b)2 only

(c)Both 1 and 2

(d)Neither 1 nor 2

[UPSC-2008]
526. Which one of the following groups of states are the major producers of
sunflower'?

(a)Tamil Nadu, Haryana and Rajasthan

(b)Kerala, Orissa and Uttar Pradesh

(c)Madhya Pradesh, Punjab and Chhattisgarh

(d)Karnataka, Maharashtra and Andhra Pradesh

[UPSC-2008]

527.As per Koppen's classification of climate, in which one of the following


do the regions of central Maharashtra and Telangana fall?

(a)Tropical wet and dry climate (monsoon savannah)

(b)Tropical wet and dry climate with winter rain (monsoon savannah with
winter rain)

(c)Tropical semi-arid steppe

(d)Humid subtropical with dry winter

[UPSC-2008]

528.In which one of the following countries is Barail Range located'?

(a)Afghanistan

(b)Sri Lanka

(c)Pakistan
(d)India

[UPSC-2008]

529.Between which one of the following pairs of islands is the Duncan


Passage located'?

(a)South Andaman and Little Andaman

(b)Little Andaman and Car Nicobar Island

(c)Car Nicobar and Little Nicobar Island

(d)Little Nicobar Island and Great Nicobar Island

[UPSC-2008]

530.Which one of the following states in Indiahas broadest continental


shelf?

(a)Andhra Pradesh

(b) Gujarat

(c)Karnataka

(d)Tamil Nadu

[UPSC-2008]

531.Which among the following is/are the major factors responsible for the
monsoon type of climate in India?

1.Location
2.Thermal contrast

3.Upper air circulation

4.Intertropical convergence zone

Select the correct answer using the codes given.

(a)1 only

(b)2, 3 and 4 only

(c)1, 3 and 4 only

(d)1, 2, 3 and 4

[UPSC-2006]

532.Match List I with List II and select the correct answer using the codes
given.
Codes:

[UPSC-2006]

533.The following have been proposed as the climatic regions of India by


Koppen:

1.AMw

2.CWg

3.As

4.E

Whichone of the following is the correct sequence of the climatic region


given above as they occur from South to North India?

(a) 3, 1, 2, 4

(b) 2, 4, 3, 1

(c) 3, 4, 1, 2

(d) 2, 1, 3,4

[UPSC-2006]

534.Consider the following statements: River: Merges with


1.Parvati: Beas

2.Chandra: Sutlej

3.Pindar: Alaknanda

4.Tons: Bhagirathi

Which of the statements given above is/ are correct'?

(a)1 and 2 only

(b)3 only

(c)1 and 3 only

(d)1, 2, 3 and 4

[UPSC-2006]

535.When a person traverses from Mangalore (west) to Chennai (east),


which one of the following sequences (from west to east) of forest types
does he observe'?

(a)Tropical evergreen, tropical wet deciduous, tropical semi-evergreen,


tropical dry

(b)Tropical wet deciduous, Tropical evergreen, tropical semievergreen,


tropical dry

(c)Tropical semi-evergreen, tropical evergreen, tropical wet deciduous,


tropical dry
(d)Tropical evergreen, tropical wet deciduous, tropical dry, tropical semi-
evergreen

[UPSC-2006]

536.Consider the following statements:

1.The light, sandy red and yellow soils in Balaghat, Durg and Raipur
districts are of alluvial origin.

2.In Tamil Nadu, the red soils occupy over two-thirds of its total area.

3.The red soils of Burdwan, Birbhum and Bankura are derived and
transported form the Chhota Nagpur plateau.

Which of the statements given above is/ are correct?

(a)1 and 2 only

(b)2 only

(c)land 3 only

(d)1, 2 and 3

[UPSC-2006]

537.Assertion (A): The Himalayan rivers are older than the mountain ranges
they cross.

Reason (R): The Himalayan rivers cut their channels deep and incisive.

(a)Both (A) and (R) are true and (R) is the correct explanation of (A).

(b)Both (A) and (R) are true but (R) is not a correct explanation of (A).
(c)(A) is true but (R) is false.

(d)(A) is false but (R) is true.

[UPSC-2006]

538.Among the following states of India, which one has the largest forested
area?

(a)Andhra Pradesh

(b) Assam

(c)Gujarat

(d)Tamil Nadu

[UPSC-2006]

539.Consider the following statements:

1.Structurally, the Meghalaya Plateau is an extended part of the Deccan


Plateau.

2.The Valley of Kashmir was formed in a syclinorium.

Which of the statements given above is/ are correct?

(a)1 only

(b)2 only

(c)Both 1 and 2

(d)Neither 1 nor 2
[UPSC-2006]

540.`Syntaxial bend' is a feature of which one of the following?

(a)Himalayas

(b)Aravallis

(c)Satpuras

(d)Vindhyas

Economic India

[UPSC-2006]

541.Consider the following statements:

1.In India, tertiary deposits of coal occur in Assam, Arunachal Pradesh,


Meghalaya and Nagaland.

2.In India, lignite does not occur anywhere except in Tamil Nadu and
Rajasthan.

3.In India, metallurgical coal is restricted to Jharkhand and Andhra


Pradesh only.

Which of the statements given above is/ are correct?

(a)1 only

(b)2 and 3only

(c)1 and 3 only


(d)1, 2 and 3

[UPSC-2006]

542.If one traverses from west to east in India, which one of the following is
the correct sequence of coal field deposits (from west to east) that he will
observe?

(a)Karanpura, Singrauli, Bokaro, Raniganj

(b)Singrauli, Karanpura, Bokaro, Raniganj

(c)Karanpura, Singrauli, Raniganj, Bokaro

(d)Singrauli, Karanpura, Raniganj, Bokaro

[UPSC-2006]

543.Which one of the following pairs is NOT correctly matched'?

Hydel power plant: State

(a)Kundah: Karnataka

(b)Rihand: Uttar Pradesh

(c)Sileru: Andhra Pradesh

(d)Rangeet: Sikkim

[UPSC-2006]

544.Match List I with List II and select the correct answer using the codes
given.
Codes:

[UPSC-2006]

545.Match List I with List II and select the correct answer using the codes
given.

Codes:
[UPSC-2006]

546.Among the following states, which one is the largest producer of


rubber'?

(a)Assam

(b)Meghalaya

(c)Tripura

(d)Sikkim

[UPSC-2006]

547.In the USA, where is the Silicon Valley located'?

(a)Northern end of Salton Sea

(b)Northern end of San Francisco bay

(c)Southern end of Strait of Georgia

(d)Southern end of San Francisco bay

[UPSC-2006]

548.Assertion (A): In India, the net sown area increased by about 19% from
the year 1950-51 to 1999-2000.

Reason (R): During the same period, there had been a concurrent decrease
in the area under forest.

(a)Both (A) and (R) are true and (R) is the correct explanation of (A).
(b)Both (A) and (R) are true but (R) is not a correct explanation of (A).

(c)(A) is true but (R) is false.

(d)(A) is false but (R) is true.

[UPSC-2006]

549.Consider the following statements:

1.Most of the coal reserves in China occur in the southern parts bordering
Vietnam, Laos and Myanmar.

2.The largest share of coal of USA is mined in the western half of the
country.

Which of the statements given above is/ are correct'?

(a)1 only

(b) 2 only

(c)Both 1 and 2

(d)Neither 1 nor 2

[UPSC-2006]

550.Which one of the following statements is NOT correct'?

(a)Vindhyan system mostly consists of sandstones, limestones and shale.

(b)Gneisses and schist of Achaeans are the oldest rocks in India.

(c)Rocks of Gondwana system are of fluvial or lacustrine origin.


(d)Deccan trap formations were due to volcanic eruptions during
Oligocene.

[UPSC-2006]

551.For the purpose of identification of backward areas in India, which one


of the following is NOT a measure of identification'?

(a)Percentage of scheduled castes and scheduled tribes to the total


population of the areas

(b)Ratio of population to the cultivable waste land

(c)Ratio of urban to rural population

(d)Percentage of working force engaged in agriculture

[UPSC-2006]

552.Which of the following explains the `Late Expending Phase' of


Demographic Theory?

(a)High birth rate and high death rate

(b)High birth rate and declining death rate

(c)Declining birth rate and declining death rate

(d)Low birth rate and low death rate

[UPSC-2006]

553.Consider the following statements:


1.Except in some districts of Kerala, there is no district in other states
where females outnumber males.

2.The sex ratio in Orissa and Manipur is less than that of the national
average.

Which of the statements given above is/ are correct?

(a)1 only

(b)2 only

(c)Both 1 and 2

(d)Neither 1 nor 2

[UPSC-2007]

554.Kurnool-Cuddapah canalis taken off from which one of the following


rivers?

(a)Kaveri

(b)Penneru

(c)Palar

(d)Tungabhadra

[UPSC-2007]

555.Match List I with List II and select the correct answer using the codes
given.
Codes:

[UPSC-2007]

556.Consider the following statements:

1.In the wet temperate forests at above 1,500 m,the rhododendrons are
commonly found.

2.Tropical dry evergreen forests are mostly found in Andhra Pradesh,


Chhattisgarh and Orissa.

Which of the statements given above is/ are correct?

(a)1 only

(b)2 only

(c)Both 1 and 2

(d)Neither 1 nor 2
[UPSC-2007]

557.Match List I with List II and select the correct answer using the codes
given.

Codes:

[UPSC-2007]

558.Taking into account the amount of rainfall occurring from June to


September, which one of the following is the correct order of cities in
terms of rainfall?

(a)Kolkata >Ahmedabad > Allahabad


(b)Kolkata > Allahabad > Ahmedabad

(c)Allahabad > Kolkata >Ahmedabad

(d)Ahmedabad > Kolkata > Allahabad

[UPSC-2007]

559. Which type of climates prevails in the long corridor (Leeward side) of
the Western Ghats and the Nilgiri Hills?

(a)Tropical wet and dry climate

(b)Tropical wet and dry with winter rain

(c)Tropical semi-arid steppe

(d)Subtropical monsoon rainforest

[UPSC-2007]

560.Assertion (A): For the onset of wet summer monsoons of Indian, the
southeast trade winds over Indian Ocean cross the Equator and recurve
into a south westerly flow.

Reason (R):The north-easterly surface winds of Siberian High blowing


towards south Asia cause the deflection of these winds.

(a)Both (A) and (R) are true and (R) is the correct explanation of (A).

(b)Both (A) and (R) are true but (R) is not a correct explanation of (A).

(c)(A) is true but (R) is false.

(d)(A) is false but (R) is true.


[UPSC-2007]

561.Assertion (A): In Gondwana coal, the carbon percentage is rarely above


30.

Reason (R): The Gondwana coal is laminated bituminous coal,


characterized by high volatility and ash contents.

(a)Both (A) and (R) are true and (R) is the correct explanation of (A).

(b)Both (A) and (R) are true but (R) is not a correct explanation of (A).

(c)(A) is true but (R) is false.

(d)(A) is false but (R) is true.

[UPSC-2007]

562.Assertion (A): Black cotton soils are ideal for dry farming.

Reason (R): Black cotton soils are finegrained.

(a)Both (A) and (R) are true and (R) is the correct explanation of (A).

(b)Both (A) and (R) are true but (R) is not a correct explanation of (A).

(c)(A) is true but (R) is false.

(d)(A) is false but (R) is true.

[UPSC-2007]

563.Consider the following statements:


1.As compared to wheat, barley can withstand more porous and alkaline
soils.

2.The millet ragi (Eleusine coracana) is cultivated only in loamy soils.

Which of the statements given above is/ are correct?

(a)1 only

(b)2 only

(c)Both 1 and 2

(d)Neither 1 nor 2

[UPSC-2007]

564.Which one of the following is the correct sequence of the Union


Territories in terms of population density from the lowest to the highest
value?

(a)Chandigarh, Lakshadweep, Delhi, Pondicherry (Puducherry)

(b)Lakshadweep, Pondicherry (Puducherry), Chandigarh, Delhi

(c)Pondicherry (Puducherry), Lakshadweep, Chandigarh, Delhi

(d)Lakshadweep, Pondicherry (Puducherry), Delhi, Chandigarh

[UPSC-2007]

565.Consider the following statements:

1.Pearl millet (Pennisetum typhoides) can be cultivated under hothumid


conditions only.
2.Pigeon pea (Cajanus cajan) can be cultivated both under hot humid and
hot dry conditions.

Which of the statements given above is/ are correct'?

(a)1 only

(b)2 only

(c)Both 1 and 2

(d)Neither 1 nor 2

[UPSC-2007]

566.Which one of the following pairs is not correctly matched'?

Tributary: River

(a)Kinnarasani:Bhagirathi

(b)Amaravati:Krishna

(c)Malaprabha:Krishna

(d)Pranhita :Godavari

[UPSC-2007]

567.Match List I with List II and select the correct answer using the codes
given.
Codes:

[UPSC-2007]

568.Match List I with List II and select the correct answer using the codes
given.

Codes:
[UPSC-2007]

569.Which system of rocks in India produces manganese?

(a)Gondwana

(b)Dharwar

(c)Vindhya

(d)Tertiary

[UPSC-2007]

570.The Damodar-Subarnarekha system is an example of which one of the


following?

(a)Superimposed drainage pattern

(b)Dendritic drainage pattern

(c)Radial drainage pattern

(d)Rectangular drainage pattern

[UPSC-2007]

571.Which one of the following pairs is associated with the shortest distance
between mainland of India and Sri Lanka?
(a)Jaffna:Vedaranniyam

(b)Talaimannar:Dhanushkodi

(c)Colombo: Kanyakumari

(d)Talaimannar: Tuticorin

[UPSC-2007]

572.Match List I with List II and select the correct answer using the codes
given.

Codes:

[UPSC-2007]

573.Match List I with List II and select the correct answer using the codes
given.
Codes:

[UPSC-2007]

574.Among the following regions of India, which one receives


comparatively lessamount of annual rainfall?

(a)An 80 km wide coastal belt from Nellore to Point Calimere

(b)The middle and lower Assam valley (c) North-eastern Rajasthan

(d)The coastal plains of Gujarat, south of Narmada

[UPSC-2009]

575.As per Koppen's classification of climate, which one of the following is


the suitable description for north-east India including north Bengal'?

(a)Tropical monsoon rainforest (Amw)

(b)Subtropical monsoon rainforest (Aw)

(c)Humid subtropical with dry wind (Cwg)


(d)Tropical wet and dry climate (Moho Savannah, Aw)

[UPSC-2009]

576.Consider the following statements

1.In India, groundnut is principally grown as rain-fed crop.

2.In India, sesamum is grown as kharif crop in northern states. But in


southern states, it is generally grown during the rabi season.

Which of the statements given above is/ are correct'?

(a)1 only

(b)2 only

(c)Both 1 and 2

(d)Neither 1 nor 2

[UPSC-2009]

577.Match List I with List II and select the correct answer using the codes
given.
Codes:

[UPSC-2009]

578.Consider the following pairs:

Mineral: District with large/significant/ commercially important deposits

Which of the pairs given above are correctly matched'?

(a)1 and 2

(b)1, 2and 3
(c)1, 2and 4

(d)2, 3and 4

[UPSC-2009]

579.The Bhakra Nangal Multipurpose Project is a joint venture of

(a)Punjab and Haryana only.

(b) Punjab, Haryana and Rajasthan only.

(c)Punjab, Haryana, Rajasthan and Himachal Pradesh.

(d)Punjab and Himachal Pradesh.

[UPSC-2009]

580.Consider the following statements:

1.Maharashtra has more net irrigated area as compared to Madhya


Pradesh.

2.Punjab has more areas under canal irrigation as compared to Haryana.

Which of the statements given above is/ are correct'?

(a)1 only

(b)2 only

(c)Both 1 and 2

(d)Neither 1 nor 2

[UPSC-2009]
581.Match List I with List II and select the correct answer using the codes
given.

Codes:

[UPSC-2009]

582.Consider the following statements:

1.The continental shelves of Lakshadweep haveoriginated due to coral


reefs.

2.The continental shelves of the western coast are due to faulting and
submergence.

Which of the statements given above is/ are correct?

(a)1 only
(b)2 only

(c)Both 1 and 2

(d)Neither 1 nor 2

[UPSC-2009]

583.Match List I with List II and select the correct answer using the codes
given.

Codes:

[UPSC-2009]

584.Consider the following:

1.Agra
2.Indore

3.Nasik

4.Vadodara

Which of the above are million-plus cities as per Census 2001?

(a)1 and 4

(b)1, 2 and 3

(c)2, 3 and 4

(d)1, 2, 3 and 4

[UPSC-2009]

585.Consider the following statements:

1.The sex ratio in India as per Census 2001 is lower as compared to that
of Census 1951.

2.From 1991 to 2001, the sex ratio in India has increased.

Which of the statements given above is/ are correct'?

(a)1 only

(b)2 only

(c)Both 1 and 2

(d)Neither 1 nor 2
[UPSC-2009]

586.Consider the following:

Centre: Place

Which of the statements given above is/ are correctly matched'?

(a)1 only

(b)2 only

(c)Both 1 and 2

(d)Neither 1 nor 2

[UPSC-2009]

587.Match List I with List II and select the correct answer using the codes
given.
Codes:

[UPSC-2009]

588.In the Government of India, under which ministry is the National River
Conservation Directorate'?

(a)Ministry of Agriculture

(b)Ministry of Earth Sciences

(c)Ministry of Environment and Forests


(d)Ministry of Water Resources

[UPSC-2009]

589.In the context of the flow of south westerly monsoons into India,
consider the following statements:

1.The Arabian Sea current merges with the Bay of Bengal current over
Punjab and adjoining Himalayas.

2.Breaks or spells of dry weather are not the characteristic features of


south-west monsoon

Which of the statements given above is/ are correct'?

(a)1 only

(b)2 only

(c)Both 1 and 2

(d)Neither 1 nor 2

[UPSC-2009]

590.With reference to the availability of minerals in India, consider the


following statements:

1.The highest concentration of manganese isfound in the Dharwar system


of rocks.

2.The chromite mainly occurs in ultrabasic igneous rocks.

Which of the statements given above is/ are correct?


(a)1 only

(b)2 only

(c)Both 1 and 2

(d)Neither 1 nor 2

[UPSC-2009]

591.With reference to the occurrence of minerals in India, consider the


following statements:

1.Sliver is a mineral of primary origin occurring in acid igneous rocks.

2.Bauxite is a secondary mineral mainly associated with lateritic rocks


formed during tertiary period.

Which of the statements given above is/ are correct'?

(a)1 only

(b)2 only

(c)Both 1 and 2

(d)Neither 1 nor 2

[UPSC-2009]

592.In which of the following states is the percentage of forest area to total
geographical area the highest?

(a)Arunachal Pradesh
(b)Himachal Pradesh

(c)Sikkim

(d)Uttarakhand

[UPSC-2009]

593.Consider the following pairs:

Tree: Location of its natural habitat

1.Rosewood: Slopes of Western Ghats of Karnataka and Kerala

2.Spruce: Western Himalayas at elevations from 2,100 to 3,600 m

3.Walnut:Lower elevation on Eastern

Which of the pairs given above is/are correctly matched'?

(a)1 only

(b)1 and 2

(c)2 and 3

(d)1, 2 and 3

[UPSC-2009]

594.Consider the following statements:

1.The Buckingham Canal runs through the coast from Guntur district to
south Arcot district.
2.For Indira Gandhi Canal, additional waters of Beas and Yamuna are
diverted and the canal terminates in Barmer district.

Which of the statements given above is/ are correct?

(a)1 only

(b)2 only

(c)Both 1 and 2

(d)Neither 1 nor 2

[UPSC-2009]

595.Consider the following statements:

1.The Kangra valley is a strike valley extending form the foot of the
Dhauladhar Range to the south of the Beas River.

2.The Kulu valley is a transverse valley in the upper course of Ravi.

Which of the statements given above is/ are correct?

(a)1 only

(b)2 only

(c)Both 1 and 2

(d)Neither 1 nor 2

[UPSC-2009]
596.Match List I with List II and select the correct answer using the codes
given.

Codes:

[UPSC-2009]

597.Consider the following:

1.Banas

2.Ghaggar

3.Machhu

4.Rupen

Which of the above neither flow into the sea nor merge with any major
river that flows into the sea but are lost in the desert?
(a)1 and 2

(b)2, 3 and 4

(c)1, 3 and 4

(d)1, 2, 3 and 4

[UPSC-2009]

598.Match List I with List II and select the correct answer using the codes
given.

Codes:

[UPSC-2009]

ANSWER KEYS
 
1.A helpful way of understanding geographic scale is to think of it as

(a)the partitioning of space within which different processes are played


out.

(b)how human impacts to the environment are measured.

(c)the delicate balance between nature and society in the protection of the
environment.

(d)the balance between physical and human geography.

2.A map with a scale of 1 cm to 30 km is enlarged nine times in size. The


R.F. of the new map will be

(a)1:10,00,000.

(b)1:90,00,000.

(c)1:1,00,000.

(d)1:000,000.

[UPSC-1998]

3.A point in mathematical space corresponds to in behavioural space.

(a)a landmark

(b)distribution
(c)a pathway

(d)a territory

4.A straight road across a series of hills and valleys appears as a marked
zigzag road near the margin of a vertical aerial photograph because

(a)there is some tilt in the photograph.

(b)geometric fidelity is poor in the margins in the photograph.

(c)there is variable movement of images due to camera (or aircraft) being


in motion during camera exposure.

(d)there is variable parallax of position in the flight directions due to


variable heights of points

[UPSC-2002]

5.A survey which is specifically concerned with landed property is known as

(a)cadastral survey.

(b)geodetic survey.

(c)thematic survey.

(d)triangulation survey.

[UPSC-1998]

6.A(n) is the ratio between linear distance on a map and linear distance on
the Earth's surface.

(a)map projection
(b)map scale

(c)economy of scale

(d)isoline

7.Absolute space is

(a)behavioural space.

(b)experiential/cultural space.

(c)mathematical space.

(d)socioeconomic space.

8.An S-curve showing the growth in the percentage of adopters is useful in


helping to explain

(a)the spatial diffusion of an innovation.

(b)the impact of sense of place on outsiders.

(c)the relationship between relative and absolute distance.

(d)connectivity in topological space.

9.As contour lines on a topographic map get closer and closer,

(a)the slope of the land gets steeper and steeper.

(b)the distortion of the projection gets greater and greater.

(c)the scale of the map gets larger and larger.


(d)the projection of the distortion gets greater and greater.

10.Assertion (A): Both Remote Sensing and GIS are involved in the analysis
of phenomena of spatial significance.

Reason (R): Both Remote Sensing and GIS are involved in the analysis of
phenomena of spatial significance.

(a)Both(A) and (R) are true and (R) is the correct explanation of (A).

(b)Both (A) and (R) are true but (R) is not the correct explanation of (A).

(c)(A) is true but (R) is false.

(d)(A) is false but (R) is true.

[UPSC-2005]

11.Assertion (A): If a surveyor is to select for surveying one instrument out


of Dumpy level and Theodolite, he will select Theodolite.

Reason (R): Theodolite can perform both horizontal and vertical


measurements.

(a)Both (A) and (R) are true and (R) is the correct explanation of (A).

(b)Both (A) and (R) are true but (R) is not the correct explanation of (A).

(c)(A) is true but (R) is false.

(d)(A) is false but (R) is true.

[UPSC-1996]
12.Assertion (A): In comparative scales, two plain scales of different units
with the same RF are made one over the other.

Reason (R): The length of both the scales may not necessarily be the
same.

(a)Both (A) and (R) are true and (R) is the correct explanation of (A).

(b)Both (A) and (R) are true but (R) is not the correct explanation of (A).

(c)(A) is true but (R) is false.

(d)(A) is false but (R) is true.

[UPSC-2002]

13.Assertion (A): Oceans are always shown in blue in coloured maps.

Reason (R): It indicates the natural colour of the oceans.

(a)Both (A) and (R) are true and (R) is the correct explanation of (A).

(b)Both (A) and (R) are true but (R) is not the correct explanation of (A).

(c)(A) is true but (R) is false.

(d)(A) is false but (R) is true.

[UPSC-1997]

14.Assertion (A): On the Interrupted Sinusoidal Projection, shapes of


continents are much improved as compared to those of the Sinusoidal
Projection.
Reason (R): The Interrupted Sinusoidal Projection employs several
straight line meridians, one for each continent.

(a)Both (A) and (R) are true and (R) is the correct explanation of (A).

(b)Both (A) and (R) are true but (R) is not the correct explanation of (A).

(c)(A) is true but (R) is false.

(d)(A) is false but (R) is true.

[UPSC-1997]

15.Assertion (A): Plano-convex lens is used in pocket stereoscope.

Reason (R): Photographs must overlap while viewing a three-dimensional


image.

(a)Both (A) and (R) are true and (R) is the correct explanation of (A).

(b)Both (A) and (R) are true but (R) is not the correct explanation of (A).

(c)(A) is true but (R) is false.

(d)(A) is false but (R) is true.

[UPSC-2001]

16.Assertion (A): Mercator projection is used by navigators.

Reason (R): Photographs must overlap while viewing a three-dimensional


image.

(a)Both (A) and (R) are true and (R) is the correct explanation of (A).
(b)Both (A) and (R) are true but (R) is not the correct explanation of (A).

(c)(A) is true but (R) is false.

(d)(A) is false but (R) is true.

[UPSC-2001]

17.Assertion (A): On small-scale conformal projections, large areas may be


imperfectly represented with respect to shape.

Reason (R): There is no angular deformation at any point of a conformal


projection.

(a)Both (A) and (R) are true and (R) is the correct explanation of (A).

(b)Both (A) and (R) are true but (R) is not the correct explanation of (A).

(c)(A) is true but (R) is false.

(d)(A) is false but (R) is true.

[UPSC-2005]

18.Assertion (A): Vernier scale enables to estimate fraction of a division in


both linear and angular measurements with greater accuracy.

Reason (R): It consists of a moving scale which a slides along a primacy


scale.

(a)Both (A) and (R) are true and (R) is the correct explanation of (A).

(b)Both (A) and (R) are true but (R) is not the correct explanation of (A).

(c)(A) is true but (R) is false.


(d)(A) is false but (R) is true.

[UPSC-2003]

19.At the heart of geographic research is

(a)collection of facts.

(b)learning place names.

(c)analysis of data.

(d)getting published in National Geographic.

20.Because of the difficulty of projecting a round, 3-dimensional Earth on a


2-dimensional piece of paper, maps of the world typically vary according
to

(a)shapes of the land masses.

(b)relative altitude of the land masses.

(c)relative sizes of the land masses.

(d)shapes and relative sizes of land masses.

21.Choropleth maps represent data with

(a)arrows of varying lengths.

(b)dots.

(c)tonal shadings.

(d)special symbols.
22.Cognitive images (mental maps) can be based on

(a)people's imaginations.

(b)visual representations of actual locations.

(c)direct experiences.

(d)all of the above

23.Consider the following statements about Geographical Information


System (GIS):

1.It handles attribute data on a technical basis.

2.It can draw maps and other graphics with great ease and speed.

3.It is a slow and tedious task involving creation of GIS database.

4.It works only in combination with remote sensing data.

5.It can analyse date for specified task and thus help in decision making.

Which of these are correct?

(a)1, 2 and 4

(b)2, 3 and 4

(c)3, 4 and 5

(d)1, 2, 3 and 5

[UPSC-2002]
24.Consider the following statements associated with representation of scale
by representative fraction (RF):

1.In RF, the units of the numerator and the denominator are taken as the
same.

2.The units are never mentioned when the scale is expressed in the form
of RF.

3.The denominator is always 10.

4.The numerator should always be 1.

Which of these are correct?

(a)1, 2 and 4

(b)1 and 4

(c)2 and 4

(d)3 and 4

[UPSC-2002]

25.Consider the following statements regarding conical projection with two


standard parallels:

1.The parallels are arcs of concentric circles and are equispaced.

2.The meridian are straight lines radiating from the common centre as
radii of concentric curves/

3.The scale is true along two standard parallels only.


4.The pole is represented by point.

Of these statements

(a)1, 2 and 3 are correct.

(b)2 and 4 are correct.

(c)1 and 2 are correct.

(d)1, 2, 3 and 4 are correct.

[UPSC-1996]

26.Consider the following statements:

Colour-composite imagery based on satellite data is mostly seen in


unnatural colour scheme because

1.blue wavelengths of solar radiation are strongly scattered.

2.red wavelengths of solar radiation are markedly absorbed.

3.it contains an invisible wavelength data which must also be assigned a


specific colour filter.

Which of these is/are correct'?

(a)Only 1

(b)1 and 2

(c)Only 3

(d)1, 2 and 3
[UPSC-2002]

27.Consider the following statements:

An aerial photograph will be considered to be truly vertical

1.if the sky does not appear in the scene.

2.if the camera axis is held perpendicular to horizon.

3.if the area covered in the image is relatively small.

Which of the statements given above are correct'?

(a)1 and 2

(b)1 and 3

(c)Only 2

(d)Only 3

[UPSC-2005]

28.Consider the following statements:

A climograph is drawn with the help of the monthly data of

1.rainfall.

2.relative humidity.

3.wet bulb temperature.

Of these statements
(a)1, 2 and 3 are correct

(b)1 and 2 are correct

(c)2 and 3 are correct

(d)1 and 3 are correct

[UPSC-1997]

29.Contour lines

(a)are required to make sense of a threedimensional map.

(b)were common on maps made before the 19th century but are rarely
used now.

(c)cannot be used to make an isoline map.

(d)connect points of equal elevation.

30.Each l:million series (India and adjacent countries) topographical sheet


contains sixteen 1:2,50,000 sheets and each 1:2,50,000 sheets contain
sixteen 1:50, 000 sheets. How many 1:25,000 sheets are contained in each
1:50,000 sheet?

(a)2

(b)4

(c)8

(d)16

[UPSC-2003]
31.For drawing the map of the world, Ptolemy apparently used a modified
form of

(a)conical projection.

(b)azimuthal equidistant projection.

(c)Mercator's projection.

(d)cylindrical equal area projection.

[UPSC-1995]

32.For each person who makes the 250-mile trip from Saint Louis to Kansas
City, which of the following varies the least from person to person'?

(a)Relative distance

(b)Cognitive distance

(c)Absolute distance

(d)All of the above will vary significantly from person to person.

33.For the following three types of maps, identify the correct descending
order based on the scale starting form the largest map scale.

(a)Atlas map, topographical map, cadastral map

(b)Cadastral map, atlas map, topographical map

(c)Cadastral map, topographical map, Atlas map

(d)Topographical map, atlas map, cadastral map


[UPSC-2004]

34.Global Positioning Systems (GPS) rely upon satellites orbiting the Earth
to help us accurately identify the location of a place on earth.

(a)nominal

(b)absolute

(c)relative

(d)cognitive

35.Hawaii's tropical location, sandy beaches, volcanoes, volcanic soils and


plentiful moisture play an important role in its economic activities. In
terms of its relative location, these characteristics refer to Hawaii's

(a)place.

(b)distance.

(c)site.

(d)space.

36.If LANDSAT data for an area is received today, then the data from
adjacent area to its west will be available

(a)in the next orbit of the satellite same day.

(b)next day at same local time.

(c)after certain number of days at same local time.

(d)after a variable controllable time.


[UPSC-2002]

37.If the map drawn to a scale of 1cm to 30 m is to be enlarged three times,


then representative fraction of the enlarged map will be

(a)1:10,00,000.

(b)1:30,00,000.

(c)1:60,00,000.

(d)1:90,00,000.

[UPSC-1995]

38.In contrast to the suburbs on the far outskirts of a city, the location of a
neighbourhood next to a downtown university has high to the students.

(a)ubiquity

(b)time-space convergence

(c)intersubj ectivity

(d)utility

39.In the grid of coordinates that cover the Earth, lines of intersect at the
North and South poles.

(a)prime meridians

(b)latitude

(c)longitude
(d)parallel

40.In which of the following projections, scale is correct along 45° latitude?

(a)Equal area cylindrical projection

(b)Gall's projection

(c)Mercator's projection

(d)Mollweide's projection

[UPSC-2002]

41.Match List I with List II and select correct answer by using the codes
given.

Codes:
[UPSC-2005]

42.Match List I with List II and select correct answer by using the codes
given.

Codes:

[UPSC-2002]
43.Match List I with List II and select correct answer by using the codes
given.

Codes:

[UPSC-2005]

44.Match List I with List II and select correct answer by using the codes
given.
Codes:

[UPSC-2005]

45.Match List I with List II and select correct answer by using the codes
given.
Codes:

[UPSC-1996]

46.Match List I with List II and select correct answer by using the codes
given.
Codes:

[UPSC-1997]

47.Match List I with List II and select correct answer by using the codes
given.
Codes:

[UPSC-1997]

48.Match List I with List II and select correct answer by using the codes
given.

Codes:
[UPSC-1998]

49.Match List I with List II and select correct answer by using the codes
given.

Codes:

[UPSC-1998]

50.Match List I with List II and select correct answer by using the codes
given.
Codes:

[UPSC-2000]

51.Match List I with List II and select correct answer by using the codes
given.
Codes:

[UPSC-2000]

52.Match List I with List lI and select correct answer by using the codes
given.

Codes:

[UPSC-2000]
53.Match List I with List II and select correct answer by using the codes
given.

Codes:

[UPSC-2002]

54.Match List I with List II and select correct answer by using the codes
given.
Codes:

[UPSC-2003]

55.Of the following tools of the geographer, which is most commonly used?

(a)Lab experiments

(b)Computer simulations

(c)Surveys

(d)Maps

56.Of the following which would probably be the most useful source of
data/ information for evaluating the loss of farmland to urban growth
around the edges of Chicago over a 20-year period'?

(a)Global positioning systems

(b)Spatial diffusion
(c)Geodemographic research

(d)Remote sensing

57.On a map drawn on gnomonic projection, the great circle course between
two points is shown by

(a)a sine curve.

(b)a straight line.

(c)a parabola.

(d)an arc of a circle.

[UPSC-1997]

58.Remote sensing is one of today's geographer's numerous tools.


Fundamental to remote sensing are

(a)computerized library archives.

(b)binoculars.

(c)mental maps.

(d)aerial photographs and satellite imagery.

59.Rhumb line is a

(a)line on the Earth's surface which cuts all meridians at the same angle.

(b)technical name for the international date line.


(c)line connecting the Greenwich meridian with the international date
line.

(d)line which give correct distance on Mercator's projection.

[UPSC-1995]

60.Select the correct sequence of the SOI topographic map index numbers
which would lie to the North, East and West of the sheet bearing number
481/2.

(a)48 1/1, 48 E/14, 48 1/6, 48 1/3

(b)48 1/6, 48 1/1, 48 E/14, 48 1/3

(c)48 1/1, 48 1/6, 48 1/3, 48 1/14

(d)48 E/14, 48 1/3, 48 1/1, 48 1/6

[UPSC-2005]

61.Sense of place is experienced

(a)only by outsiders.

(b)only by insiders.

(c)by both insiders and outsiders.

(d)by animals, not humans.

62.Some types of permanent adjustments of a theodolite during survey are


given below:

1.Adjustment of the line of collimation


2.Adjustment of the vertical index frame

3.Adjustment of the plate levels

4.Adjustment of the horizontal axis

The correct sequence of these adjustment is:

(a)1, 3, 4, 2

(b)3, 1, 2, 4

(c)1, 3, 2, 4

(d)3, 1, 4, 2

[UPSC-1998]

63.Spatial data is stored in the computer by using

(a)digitizer and scanner.

(b)plotter digitizer and keyboard.

(c)keyboard and plotter.

(d)scanner, digitizer and keyboard.

[UPSC-2000]

64.The map projection deliberately emphasizes the true areas of countries.

(a)Mollweide

(b)Peters
(c)Notsram

(d)Mercator

65.The map projection presents the true shapes of landmasses but distorts
their relative sizes.

(a)Robinson

(b)Mollweide

(c)Peters

(d)Mercator

66.The beauty of equidistant maps is that they

(a)display areas on the Earth's surface in their true proportions.

(b)objectively represent the world.

(c)accurately represent distance in all directions.

(d)are aesthetically pleasing.

67.The best cartographic method for representing the distribution of rural


population in a region of greatly varying size of villages is

(a)choropleth method.

(b)isopleth method.

(c)multiple dot method.

(d)proportional squares method.


[UPSC-1997]

68.The correct sequence of the given parts of electromagnetic spectrum in


terms of their wavelength is

(a)ultraviolet, visible, gamma, infrared, radio

(b)gamma, ultraviolet, visible, infrared, radio

(c)gamma, radio, infrared, ultraviolet, visible

(d)visible, ultraviolet, gamma, infrared, radio

[UPSC-2001]

69.The distribution of dairy farms across the USA would most effectively
and precisely be represented using a

(a)isoline map.

(b)cartogram.

(c)dot map.

(d)choropleth map.

70.The form lines are

(a)the interpolated contours.

(b)the extrapolation of contours.

(c)significant contours.

(d)generalized contours.
[UPSC-2001]

71.The model developed by Hagerstrand in relation to spatial diffusion is

(a)Loschina model.

(b)Monte Carlo model.

(c)Lowvy-type model.

(d)multiple-nuclei model.

[UPSC-1996]

72.The most appropriate diagram for showing the actual distribution of


rainfall over a substantial long period is the

(a)wheel diagram.

(b)dispersion diagram.

(c)scatter diagram.

(d)bar diagram.

[UPSC-1996]

73.The Prime Meridian

(a)passes through Greenwich, England.

(b)was established by the Greeks.

(c)is essentially a latitude line.

(d)forms an angle of 0° with the Earth's equator.


74.The Prime Meridian

(a)goes through Paris and New York.

(b)always crosses the Equator at an angle of less than 90°.

(c)runs through both poles.

(d)is the main reference point for determining latitude.

75.The ratio of the `map lengths' of the Equator and the meridian `is pie' in
the

(a)Mercator's projection.

(b)sinusoidal projection.

(c)cylindrical equalareaprojection.

(d)simple conical projection with one standard parallel.

[UPSC-1998]

76.The scale of the one-inch Indian toposheet converted into metric system
is now given as

(a)1:5,000.

(b)1:25,000.

(c)1:50,000.

(d)1:50,000.

[UPSC-2000]
77.The spatial variations in the wind velocity in a weather map can be best
explained by a reference to the distribution of

(a)barometric gradient.

(b)cyclones and depressions.

(c)fronts and air masses.

(d)high and lows.

[UPSC-1997]

78.The Sten-de-Geer's diagram is used to represent

(a)sex ratio.

(b)industrial population.

(c)urban population.

(d)rural and urban population.

[UPSC-2001]

79.The success of Geographic Information Systems is primarily due to the


use of this technology.

(a)Global satellites

(b)The world wide web

(c)Computers

(d)Shipping
80.The utility of a specific place to live is generally measured in terms of

(a)quality of life.

(b)housing costs.

(c)access to things like work, school and leisure.

(d)All of the above.

81.This is NOT one of the fundamental concepts of spatial analysis.

(a)Spatial interaction

(b)Distance

(c)Location

(d)Value

82.Though all of the following distort the shape of land masses and area,
which of the following maps or map types does so most blatantly and
intentionally'?

(a)Isoline

(b)Cartograms

(c)Peters

(d)Polyconic

83.Time-space convergence

(a)has been hampered by the computer revolution.


(b)has severely hindered the ability of companies to take advantage of
economies of scale.

(c)increases as the friction of distance decreases.

(d)All of the above.

84.Topological space is best represented by a

(a)map of the trails and features in a state park.

(b)topographic map.

(c)weather map.

(d)metro (subway) system map.

85.Two places situated on the same isogonics' line is a world map will have
the same

(a)ground pollution.

(b)cloud cover.

(c)seismic activity.

(d)magnetic declination.

[UPSC-1998]

86.What is shading in the form of lines showing a hill or lope on a map


termed as'?

(a)Spot height
(b)Contour

(c)Hachure

(d)Farm lines

[UPSC-2005]

87.What is the Representative Fraction (R.F.) if the given scale is 2:1 mile'?

(a)1:31,680

(b)1:62,360

(c)1:1,26,720

(d)1:63,360

[UPSC-2005]

88.When point symbols are inserted over non-quantitative shaded maps


showing spatial distribution the map is

(a)chorochromatic.

(b)choroschematic.

(c)choropleth.

(d)chorographic compage.

[UPSC-1998]

89.Which of the following cannot appropriately and effectively be used in


the creation of thematic maps?
(a)Proportional symbols

(b)Located charts

(c)Isolines

(d)All of the above can be used appropriately and effectively

90.Which of the following depict relief?

1.Contour lines

2.Isochrones

3.Hachures

4.Isobars

Select the correct answer suing the codes given:

(a)1 and 4

(b)1 and 2

(c)1 and 3

(d)2 and 4

[UPSC-2003]

91.Which of the following geographic phenomena is most effectively


understood through spatial analysis?

(a)The distance people are willing to travel for groceries


(b)Regionalization and the classification of a continent into sub-regions

(c)Place-making and the production of cultural landscapes

(d)The strong feelings people have toward their ethnic home region

92.Which of the following is a tool or technique of the contemporary


geographer?

(a)GPS

(b)Cartography

(c)Remote sensing

(d)All of the above are used by contemporary geographers.

93.Which of the following is NOT one of the basic concepts of spatial


interaction'?

(a)Diffusion

(b)Transferability

(c)Intervening opportunity

(d)Concentration

94.Which of the following properties are associated with the conical map
projection?

1.The distance between the meridians

2.The standard parallel is correct to the scale


3.The distance between the parallels decreases towards the pole

4.The central meridian is a straight line

Select the correct answer using the codes given.

(a)1 and 4

(b)2 and 3

(c)1, 2 and 3

(d)1, 2 and 4

[UPSC-2005]

95.Which one of following statements about maps is not correct'?

(a)A map is a graphic representation of the milieu.

(b)A map is a tangible product.

(c)A map at small scale shows a small area on a single map sheet.

(d)A map is a mental and manual creation of man.

[UPSC-2004]

96.Which one of the following cartographic techniques is suitable for


measuring spatial association among different attributes of the regional
economy'?

(a)Dot method

(b)Pie diagram
(c)Bar diagram

(d)Choropleth method

[UPSC-2005]

97.Which one of the following is not a conventional map projection?

(a)Interrupted sinusoidal projection

(b)Mollweide's projection

(c)Sinusoidal projection

(d)Gall's stereographic projection

[UPSC-1996]

98.Which one of the following is not common between cylindrical equal


area and Mollweide's map projection?

(a)Parallels are closer towards the poles.

(b)Parallels are straight lines.

(c)Meridians are curves.

(d)Area is shown correctly.

[UPSC-1995]

99.Which one of the following is not a GIS software?

(a)ARCVIEW

(b)OSIRIS
(c)ILWIS

(d)IMAGE ANALYST

[UPSC-2005]

100.Which one of the following is the most suitable projection for showing
TransSiberian Railway?

(a)Cylindrical equal area projection

(b)Mercator's projection

(c)Conical with one standard parallel

(d)Polyconic projection

[UPSC-1998]

101.Which one of the following map projections looks like a photograph of


the Earth grid taken form a considerable distance/ height?

(a)Gnomonic

(b)Orthomorphic

(c)Stereographic

(d)Orthographic

[UPSC-1998]

102.Which one of the following pairs is not correctly matched?


[UPSC-2001]

103.Which one of the following statements is correct?

When contours are placed very close to each other in a river valley, they
represent a

(a)waterfall.

(b)rapid.

(c)convex slope.

(d)concave slope.

[UPSC-2005]

104.Which one of the following statements is not correct?

(a)LANDSAT is a geostationary satellite.

(b)IRS is a sun-synchronous satellite.

(c)INSAT is a geostationary satellite.

(d)SPOT is a sun-synchronous satellite.


[UPSC-2003]

105.Which one of the following statements is not correct?

(a)IKONOS satellite image has a 1-m resolution.

(b)IRS satellite image has a 5.8-m resolution.

(c)INSAT satellite image has a 10-m resolution.

(d)SPOT satellite image has a 10-m r esolution.

[UPSC-2003]

106.Which type of map projection is best for representing cultural,


demographic and economic data?

(a)Mollweide projection

(b)Robinson projection

(c)Mercator projection

(d)Dymaxion projection

107.Which one of the following is a largescale map?

(a)1:2,50,000

(b)1:10,00,000

(c)1:25,000

(d)1:50,000

[UGC D-2007]
108.Isochrone map shows lines connecting

(a)equal travel time.

(b)equal transport costs.

(c)equal travel distance.

(d)areas with similar accessibility.

[UGC D-2007]

109.Dasymetric map is an improved version of choropleth map as it


effectively shows

(a)the homogeneity and rapid change.

(b)the differences and uniqueness.

(c)the gradual areal changes.

(d)the random distribution.

[UGC D-2007]

110.Which one of the following is a smallscale map?

(a)Cadastral map

(b)SOI Topographical map

(c)World Atlas map

(d)Tamil Nadu State map

[UGC D-2008]
111.In remote sensing, there are four different types of resolution characters.
Identify the correct order in which they are used.

(a)Radiometric, spatial, spectral, temporal

(b)Spatial, spectral, radiometric. temporal

(c)Temporal, spectral, radiometric, spatial

(d)Spectral, temporal, spatial, radiometric

[UGC D-2008]

112.Road accessibility can best be represented by

(a)bar diagram.

(b)pie diagram.

(c)graded symbol map.

(d)flow line diagram.

[UGC D-2008]

113.The projection in which loxodromes are shown as straight lines is

(a)gnomonic projection.

(b)Mercator's projection.

(c)Gall's stereographic projection.

(d)Cylindrical equal area projection.

[UGC D-2009]
114.A system which consists of data acquisition, data processing and data
analysis is called

(a)Digital Image.

(b)Geographic Information System.

(c)Remote Sensing System.

(d)Global Positioning System.

[UGC D-2009]

115.The method which shows by the use of light and shade is called

(a)hill shading.

(b)dpot heights.

(c)hachure.

(d)benchmarks.

[UGC D-2009]

116.Match List I and List II and select the correct answer from the codes
given.
Codes:

[UGC J-2005]

117.Topographical maps are produced in India by

(a)NATMO.

(b)Survey of India.

(c)NRSA.

(d)DRDO.

[UGC J-2007]

118.Map showing the depths of oceans and seas is termed as

(a)ethnographic map.
(b)orographic map.

(c)bathymetric map.

(d)geological map.

[UGC J-2008]

119.Which one of the following is the largest scale of map'?

(a)1:5,00,000

(b)1:1,00,000

(c)1:50,000

(d)1:10,000

[UGC J-2008]

120.National Remote Sensing Agency (NRSA) is located at

(a)Dehradun.

(b)Hyderabad.

(c)Bangalore.

(d)Thumba.

[UGC J-2008]

121.Vector and raster data are used in

(a)Global Positioning System.


(b)Remote Sensing System.

(c)Geographical Information System.

(d)All of these.

[UGC J-2011]

122.Match List I with List II and select the correct answer from the codes
given.

Codes:

[UGC J-2011]

123.According to nearest-neighbour index, what would be the maximum


value for a complete dispersed settlement pattern?

(a)0.00
(b)1.55

(c)2.15

(d)2.88

[UGC J-2010]

124.Which of the following is a geosynchronous satellite?

(a)LANDSAT

(b)METEOSAT

(c)IRS

(d)KITSAT

[UGC J-2010]

125.Match List I with List II and select the correct answer from the codes
given.
Codes:

[UGC J-2010]

126.Which of the following cartographic method is generally used to show


population density?

(a)Contours

(b)Choropleth

(c)Dot method

(d)Isopleth

[UGC D-2011]

127.Lorenz curve measures

(a)curvature.

(b)inequality.

(c)similarity.

(d)diversity.

[UGC D-2011]
128.Arrange the following maps in order in which they are prepared with
small to large scale. Use the codes given.

1.World map

2.Toposheet

3.Cadastral map

4.State map

(a)2, 3, 1, 4

(b)1, 2, 4, 3

(c)1, 4, 2, 3

(d)3, 4, 2, 1

[UGC D-2011]

129.Match List I with List II and select the correct answer from the codes
given.

Codes:
[UGC D-2011]

130.In remote sensing systems which sensors measure naturally available


energy'?

(a)Active sensors

(b)Passive sensors

(c)Synthetic aperture radar

(d)Microwave remote sensing

[UGC D-2010]

131.In which of the following diagrams is the relationship of relative


humidity and temperature depicted'?

(a)Hythergraph

(b)Climograph

(c)Ergograph

(d)Band graph

[UGC D-2010]
132.Variable class interval for mapping will prove more meaningful when
the variable is

(a)normally distributed.

(b)highly skewed.

(c)highly consistent.

(d)measured on ordinal scale.

[UGC D-2010]

133.Match List I with List II and select the correct answer from the codes
given.

Codes:

[UGC D-2010]
134.Consider the following statements:

1.In truly vertical photograph, the nadir coincides with the photo's
principal point.

2.Straight lines connecting opposite fiducial marks intersect at the


principal point.

Which of the statements given above is/ are correct'?

(a)1 only

(b)2 only

(c)Both 1 and 2

(d)Neither 1 nor 2

[UPSC-2008]

135.With reference to Mollweide projection, consider the following


statements:

1.Mollweide projection is elliptical in shape.

2.The parallel are straight lines.

Which of the statements given above is/ are correct'?

(a)1 only

(b)2 only

(c)Both 1 and 2
(d)Neither 1 nor 2

[UPSC-2008]

136.In spherical diagram, the radii of spheres are proportionate to which one
of the following'?

(a)Square root of the quantities

(b)Cube root of the quantities

(c)Percentage of the quantities

(d)Actual quantities

[UPSC-2008]

137.Match List I with List II and select the correct answer using the codes
given.
Codes:

[UPSC-2008]

138.If the tilt angle is less than 3°, the air photograph is known as

(a)oblique photograph.

(b)vertical photograph.

(c)orthophotograph.

(d)trimetrogon photograph.

[UPSC-2008]

139.What is a line drawn through a series of points which have been equally
elevated (or depressed) by relative movements of land and sea termed as'?

(a)Isobase

(b)Isorhythm

(c)Isogram

(d)Isonoetic line

[UPSC-2008]
140.Assertion (A): In aerial and satellite remote sensing and photography,
the spectral bands of green and near-infrared radiation are used.

Reason (R): The vegetation reflects green and near-infrared radiation


which appears on the air photographs and imagery as green and red tones.

(a)Both (A) and (R) are individually true and (R) is the correct
explanation of (A).

(b)Both (A) and (R) are individually true, but (R) is not the correct
explanation of (A).

(c)(A) is true, but (R) is false.

(d)(A) is false but (R) is true.

[UPSC-2008]

141.In the context of India, in the 1:50,000 scale toposheet, how many
metres are represented by 1 cm?

(a)1,000 m

(b)500 m

(c)250 m

(d)100 m

[UPSC-2008]

142.Consider the following statements:

1.In Mercator projection, all the rhumbs appear as straight lines.


2.Mercator's projection enlarges areas at a rapidly decreasing rate towards
the higher latitudes.

Which of the statements given above is/ are correct'?

(a)1 only

(b)2 only

(c)Both 1 and 2

(d)Neither 1 nor 2

[UPSC-2008]

143.Consider the following statements:

1.In Albers' equal-area projection, distortion zones are arranged parallel


to the standard lines.

2.Albers' equal-area projection is good for a middle-latitude area of


greater east-west than north-south extent.

Which of the statements given above is/ are correct'?

(a)1 only

(b)2 only

(c)Both 1 and 2

(d)Neither 1 nor 2

[UPSC-2008]
144.With reference to the satellites launched by India, which one of the
following is the correct chronological order'?

(a)Bhaskara II, IRS-lA, Cartosat, INSAT-1B

(b)Bhaskara II, INSAT-1B, IRS-lA, Cartosat

(c)IRS-lA, INSAT-1B, Bhaskara II, Dartosat

(d)INSAT-1B, Bhaskara II, Cartosat, IRS-IA

[UPSC-2008]

145.Which one of the following statements is correct?

(a)Digital Raster Graphics (DRG) is a scanned image of a USGS


topographic map.

(b)DRG is a hard copy of a resource map of India.

(c)DRG is a survey map of NATMO.

(d)DRG is a cadastral map.

[UPSC-2008]

146.Which one of the following sequences of ocean trenches is correct in


terms of the ascending order of their depths'?

(a)Aleutian Trench, Mindanao Trench, New Hebrides Trench

(b)New Hebrides Trench, Mindanao Trench, Aleutian Trench

(c)Aleutian Trench, New Hebrides Trench, Mindanao Trench


(d)New Hebrides Trench, Aleutian Trench, Mindanao Trench

[UPSC-2008]

147.Geographical Information Systems (GIS) are designed to manipulated


spatial objects. Which one of the following is NOT a spatial object?

(a)An attribute data cell

(b)A polygon

(c)A point

(d)A line

[UPSC-2006]

148.Under which of the following conditions would you NOT consider a


given pair of aerial photographs as a stereo pair'?

1.No overlap

2.25% overlap

3.60% overlap

4.100% overlap

Select the correct answer using the codes given below:

(a)1 and 2

(b)1 and 3

(c)1 and 4
(d)2 and 4

[UPSC-2006]

149.Which one of the following pairs is NOT correctly matched?

Software: Organization

(a)Arc Info: ESRI

(b)GRASS: DST

(c)MGE: Intergraph Corporation

(d)IDRISI: Clark University

[UPSC-2006]

150.If the focal length (f) of a camera lens is 20 cm and the height of the
plane (h) is 1,200metres above ground, what will be the scale of the
airphotographs?

(a)1:500

(b)1:5,000

(c)1:1,000

(d)1:50,000

[UPSC-2006]

151.Which one of the following is more useful to a sailor?

(a)Mercator projection
(b)Gnomonic projection

(c)Sinusoidal projection

(d)Mollweide projection

[UPSC-2006]

152.Which one of the following projection is suitable for large maps of


small area fitting on all sides'?

(a)Sinusoidal projection

(b)Modified polyclone projection

(c)Conical projection with one standard parallel

(d)Sinusoidal projection

[UPSC-2006]

153.Which one of the following is a special case of Bonne's projection?

(a)Zenithal polar equidistant projection

(b)Cylindrical equidistant projection

(c)Conical equidistant projection

(d)Sinusoidal projection

[UPSC-2006]

154.The number of a topographic map is 47A/16/NW. Its scale must be

(a)1:2,50,000
(b)1:1,00,000

(c)1:50,000

(d)1:25,000

[UPSC-2006]

155.Consider the following statements:

1.It operates at an altitude of 950 km.

2.Its sensors to collect both multispectral and panchromatic images


concurrently.

Which of the statements given above is/ are correct?

(a)1 only

(b)2 only

(c)Both 1 and 2

(d)Neither 1 nor 2

[UPSC-2006]

156.On an aerial photograph, symbol denotes which one of the following'?

(a)Collimating marks

(b)Principal points

(c)Plumb points
(d)Isocentre

UPSC-2006]

157.With reference to contours use for mapping the terrain, consider the
following statements:

1.Contours are horizontal to the dip of the land and perpendicular to the
direction in which the surface water runs.

2.The steeper the slope, the farther the contour.

3.Contours bend upstream while crossing a river.

4.Contours do not show up minor relief features.

Which of the statements given above is/ are correct'?

(a)1, 2 and 3

(b)1, 3 and 4

(c)2, 3 and 4

(d)1, 2 and 4

[UPSC-2006]

158.In which one of the following projections is the Equator twice the length
of the central meridian'?

(a)Gnomonic projection

(b)Mercator projection
(c)Mollweideprojection

(d)Zenithal orthographic

[UPSC-2007]

159.Diagonal scales are generally used for which of following'?

(a)Wall maps

(b)Cadastral maps

(c)Small-scale maps

(d)Medium-scale maps

[UPSC-2007]

160.A star diagram is used to represent which of the following'?

(a)Duration of winds only

(b)Flows between central places and dependent places

(c)Wind direction and duration

(d)Direction of winds only

[UPSC-2007]

161.Construction of a comfort diagram requires which of the following


data?

(a)Monthly temperature and rainfall

(b)Monthly temperature and relative humidity


(c)Monthly relative humidity and rainfall

(d)Monthly temperature, rainfall and relative humidity

[UPSC-2007]

162.Which one of the following statements is not correct'?

(a)A cylindrical projection with two standard parallels is particularly


useful for representing the low-latitude zone straddling the Equator
between the pair of standard parallels.

(b)Ina Mercator projection, the spacing of parallels decreases towards the


poles.

(c)A straight line on planar projection is the shortest route between two
points on the Earth's surface.

(d)On a conic projection, meridians are shown as straight lines that


converge towards the poles.

[UPSC-2007]

163.What is the thermal band region of Landsat Thematic Mapper Sensor'?

(a)0.63-0.69 mm

(b)0.76-0.90 mm

(c)2.08-2.35 mm

(d)10.40-12.50 mm

[UPSC-2007]
164.Which one of the following colours is used to show the cultivated areas
in Survey of India's topographical maps of 1:50,000 scale'?

(a)Light green

(b)Brown

(c)Yellow

(d)Dark green

[UPSC-2007]

165.In remote sensing, the strong 'chlorophyll absorption band' lies in the
wavelength bands of which one of the following ranges'?

(a)0.45-0.67 mm

(b)0.30-0.38 mm

(c)0.12-0.28 mm

(d)0.72-1.30 mm

[UPSC-2007]

166.The concept of `vanishing point'is used in the context of

1.aerial photographs.

2.satellite images.

3.block diagrams.

4.projections.
(a)Both A and R are individually true R is the correct explanation of A.

(b)Both A and R are individually true but R is not the correct explanation
of A.

(c)A is true but R is false.

(d)A is false but R is true.

[UPSC-2007]

167.Consider the following statements:

1.The location of objects in geographic space and the associated attributes


are handled by GIS.

2.Spatial objects are represented by graphical elements of points, lines


and polygons.

Which of the statements given above is/ are correct?

(a)1 only

(b)2 only

(c)Both 1 and 2

(d)Neither 1 nor 2

[UPSC-2009]

168. For obtaining greater accuracy in the fraction of a division in both


linear and angular measurements, which scale is used?

(a)Linear
(b)Vernier

(c)Plane

(d)Graphic

[UPSC-2009]

169.Which one of the following RF represents 1 cm map distance to 5 km of


ground distance?

(a)1:500

(b)1:5,000

(c)1:50,000

(d)1:5,00,000

[UPSC-2009]

170.A form of diagram in which values are plotted as radii from a point of
origin is known as

(a)star diagram.

(b)columnar diagram.

(c)block pile diagram.

(d)dispersion diagram.

[UPSC-2009]

171.With reference to the dot method, consider the following statements:


1.Dot method is the most convenient method for illustrating the
distribution of absolute figures.

2.The size of the dots depends upon the frequency of the element to be
represented.

3.In it, each dot has an identical value.

4.It is used very effectively when plotting values in detail on a large scale.

Which of the statements given above is/ are correct'?

(a)1, 3 and 4

(b)2, 3 and 4

(c)1 and 2

(d)1, 2 and 3

[UPSC-2009]

172.Match List I with List II and select the correct answer using the codes
given.
Codes:
[UPSC-2009]

173.A graph showing wavelength and reflectance amount is known as

(a)spectral reflectance curve.

(b)spectral signature.

(c)spectral band.

(d)spectral region.

[UPSC-2009]

174.For representing the data relating to coast control, quality control and
assessment of progress, which of the following is/are the correct use of
the diagram'?

(a)Cycle diagram

(b)Cycle or diagram

(c)Chorographic compass maps

(d)Both chorochromatic and chorographic compass maps

[UPSC-2009]

175.A vector-based data structure is made storing terrain information in


digital terrainmodelling in which each sample point has x, y co-ordinates
and a height, or z value of the points are connected by edges to form
networks on nonoverlapping triangles collectively represent the terrain
surface. This network is known as

(a)TIN.
(b)DTM.

(c)TQM.

(d)TIGER.

[UPSC-2009]

ANSWER KEYS

 
1.Which of the following is not true of statistics?

(a)Statistics is used to draw conclusions using data.

(b)Statistics can be used to organize and analyse information.

(c)Statistics involves collecting and summarizing data.

(d)Statistics is used to answer questions with 100% certainty.

2.A mobile phone company has decided that an assembly line is operating
satisfactorily if less than 0.05% of the phones produced per day are
defective. To check the quality of a day's production, the company
decides to randomly sample 10 phones from a day's production to test for
defects. Define the population of interest to the company.

(a)The 0.05% of the phones that is defective

(b)All the phones produced during the day in question

(c)The 10 responses: defective or not defective

(d)The 10 phones sampled and tested

3.Which branch of statistics deals with the organization and summarization


of collected information'?

(a)Inferential statistics

(b)Survey design
(c)Computational statistics

(d)Descriptive statistics

4.Classify `telephone area codes' into following random variables.

(a)Quantitative continuous data

(b)Qualitative data

(c)Quantitative discrete data

(d)Experimental data

5.Thirty-five Math teachers, 29 Science teachers and 26 Social Studies


teachers are randomly selected from 585 Math teachers, 279 Science
teachers and 393 Social Studies teachers at the University. What sampling
technique is used?

(a)Simple random

(b)Convenience

(c)Stratified

(d)Cluster

6.Every 50th visitor entering the Indira Gandhi International Airport is


planned to be checked for extra security screening. What sampling
technique is used by the Airport Security Department'?

(a)Convenience

(b)Stratified
(c)Cluster

(d)Systematic

7.At a state-level athletics camp, six teams are randomly selected and all of
the athletes from each team are considered for trials. What sampling
technique is used?

(a)Cluster

(b)Simple random

(c)Stratified

(d)Convenience

8.An editor randomly selects five fiction and five business stories for the
upcoming issue of her magazine. What sampling technique is used'?

(a)Simple random

(b)Systematic

(c)Stratified

(d)Convenience

9.A ticket inspector checks all of the passengers on five randomly selected
buses. What sampling technique is used?

(a)Simple random

(b)Convenience
(c)Systematic

(d)Cluster

10.A student carries out a survey and approaches everyone in a residential


society to determine who owns a white car. What sampling technique is
used'?

(a)Convenience

(b)Stratified

(c)Cluster

(d)Systematic

11.A sports teacher in a school assigns a number to each student and then
uses a computer to randomly generate 10 numbers. The teacher creates a
team of these students corresponding to these numbers. What sampling
technique was used'?

(a)Convenience

(b)Simple random

(c)Cluster

(d)Systematic

12.Based on 500 responses from 15,000 questionnaires sent to all its


members, a major political party estimated that the annual salary of its
members was 7 2,00,000 per annum. What sampling technique was used?

(a)Cluster
(b)Simple random

(c)Stratified

(d)Systematic

13.In a recent mobile survey carried out by an NGO, participants were asked
to send SMS for answer `yes' or `no' to the question: "Are you in favour
of paying tolltax for better roads?" 3,540 responded `yes' while 1,239
responded `no'. There was a one-rupee charge for the SMS the respondent
had to pay. What sampling technique was used'?

(a)Cluster

(b)Systematic

(c)Convenience

(d)Stratified

14.A sample consists of every ninth student from a class of 100students.


What sampling technique was used'?

(a)Convenience

(b)Cluster

(c)Stratified

(d)Systematic

15.A market researcher randomly selects 200 professors under 48 years of


age and 500professors over 48 years of age teaching in a university. What
sampling technique was used'?
(a)Stratified

(b)Systematic

(c)Cluster

(d)Convenience

16.To complete work on time, the quality inspector at an assembly line


inspected the first 40 articles produced during his shift. What sampling
technique was used'?

(a)Convenience

(b)Systematic

(c)Stratified

(d)Simple random

(e)Cluster

17.There are 30 different numbers printed on 30 red-coloured balls. These


balls are placed in a bag, and three numbers are picked from the bag.
What sampling technique was used?

(a)Systematic

(b)Cluster

(c)Stratified

(d)Simple random
18.An education survey randomly selects 10 of the state's schools and
interviews all of the teachers at each school. What sampling technique
was used?

(a)Convenience

(b)Cluster

(c)Simple random

(d)Systematic

19.What is the difference between a bar chart and a histogram'?

(a)The bars on a bar chart do not touch, while the bars of a histogram do
touch.

(b)The bars in a bar chart are all the same width, while the bars of a
histogram may be of various widths.

(c)The bars in a bar chart may be of various widths, while the bars of a
histogram are all the same width.

(d)There is no difference between these two graphical displays.

20.Describe the shape of the distribution.

(a)Skewed to the right


(b)Bell-shaped

(c)Uniform

(d)Skewed to the left

21.Describe the shape of the distribution.

(a)Bell-shaped

(b)Skewed to the right

(c)Uniform

(d)Skewed to the left

22.The heights of 10 female students (in inches) in a college math class are
listed below. Find the mean.

(a)70.0 inches

(b)67.7 inches

(c)71.1 inches

(d)65.5 inches
23.The number of students enrolled in a Physics class for the last 10
semesters are listed below. Find the median number of students.

(a)67 students

(b)68 students

(c)70 students

(d)66 students

24.The commuting times of 10 employees (in minutes) are listed below.


Find the mode score.

(a)66 minutes

(b)65 minutes

(c)68 minutes

(d)67 minutes

25.Ten last utility bills (in `000 rupees) for a residential society are listed
below. Find the mode amount.

(a)76,000
(b)72,000

(c)67,000

(d)73,000

26.A numerical summary of a population is a

(a)statistic.

(b)qualitative response.

(c)parameter.

(d)variable.

27.The median of a data set for a variable is the data value that

(a)None of these.

(b)lies in the middle of the data when the data is arranged in ascending
order.

(c)is the average, i.e., the sum of all the data values of the variable
divided by the number of observations in the data set.

(d)appears the most often.

28.The of a variable is computed by determining the sum of all the values of


the variable in the data set and dividing this sum by the number of
observations in the data set.

(a)mode
(b)median

(c)geometric mean

(d)arithmetic mean

29.Which measure of central tendency may not exist for all numeric data
sets'?

(a)Mode

(b)Midrange

(c)Mean

(d)Median

30.Which measure of central tendency may have more than one value in a
numeric data set'?

(a)Mean

(b)Midrange

(c)Mode

(d)Median

31.Which measure of central tendency is not resistant to extreme values in a


numeric data set?

(a)Median

(b)Mode
(c)Mean

(d)Parameters

32.Describe the shape of the histogram. The data setconsists of picking 3


lottery results for 10 consecutive weeks.

(a)Skewed to the right

(b)Symmetric

(c)Skewed to the left

(d)Uniform

33.Describe the shape of the histogram. The data set consists of age of 20
household stereo systems randomly selected from a neighbourhood.

(a)Skewed to the right

(b)Uniform

(c)Symmetric

(d)Skewed to the left

34.Describe the shape of the histogram. The data set consists of a round-trip
commuting times (in minutes) of 20 randomly selected employees
(a)Symmetric

(b)Skewed to the right

(c)Uniform

(d)Skewed to the left

35.The distribution of salaries of professional basketball players is skewed


to the right. Which measure of central tendency would be the best
measure to determine the location of the centre of the distribution'?

(a)Mode

(b)Mean

(c)Frequency

(d)Median

36.For the distribution drawn here, identify the mean, median and mode.

(a)A = mode, B = mean, C = median


(b)A = mode, B = median, C = mean

(c)A = median, B = mode, C = mean

(d)A = mean, B = mode, C = median

37.In distributions that are skewed to the right, what is the relationship of the
mean, median and mode'?

(a)Median > mean > mode

(b)Mode > mean > median

(c)Mean > median > mode

(d)Mode > median > mean

38.In distributions that are skewed to the left, what is the relationship of the
mean, median and mode'?

(a)Mode > mean > median

(b)Mode < mean < median

(c)Mean > median > mode

(d)Mode > median > mean

39.The annual profits of five large corporations in a certain area are given
below. Which measure of central tendency should be used'?

(a)Midrange
(b)Mean

(c)Mode

(d)Median

40.If the graph of a distribution of data shows that the graph is skewed to the
right, then

(a)mean < median.

(b)mean z median.

(c)no conclusion about the relative position of the mean and the median can
be made.

(d)mean > median.

41.If the graph of a distribution of data shows that the graph is skewed to the
left, then

(a)mean median.

(b)mean > median.

(c)No conclusion about the relative position of the mean and the median
can be made.

(d)median > mean.

42.If the graph of a distribution of data shows that the graph is symmetric,
then

(a)mean is a better measure of central tendency.


(b)median is a better measure of central tendency.

(c)midrange is a better measure of central tendency.

(d)mode is a better measure of central tendency.

43.Which measure of central tendency is more representative of the typical


observation if the graph of the data is skewed to the right?

(a)Median

(b)Mode

(c)Mean

(d)Midrange

44.Which measure of central tendency is more representative of the typical


observation if the graph of the data is skewed to the left'?

(a)Mode

(b)Mean

(c)Median

(d)Midrange

45.The April precipitation amounts (in inches) for 10 cities are listed below.
Find the range of the data.

(a)2.45 inches
(b)2.8 inches

(c)1.4 inches

(d)3.2 inches

46.Which is not a measure of dispersion?

(a)Range

(b)Variance

(c)Standard deviation

(d)Mean

47. The is the easiest measure of dispersion to calculate.

(a)standard deviation

(b)variance

(c)range

(d)mean absolute deviation

48.A residual is the difference between

(a)the observed value of x and the predicted value of y.

(b)the observed value of y and the predicted value of y.

(c)the observed value of y and the predicted value of x.

(d)the observed value of x and the predicted value of x.


49.The least-squares regression line

(a)minimizes the sum of the residuals squared.

(b)maximizes the mean difference between the residuals squared.

(c)maximizes the sum of the residuals squared.

(d)minimizes the mean difference between the residuals squared.

50.The coefficient of correlation between x and y is r = 0.59. Calculate the


coefficient of determination R2(round R2 to the nearest hundredth).

(a)0.59

(b)0.35

(c)0.65

(d)0.41

51.The measures the percentage of total variation in the response variable


that is explained by the least-squares regression line.

(a)sum of the residuals squared

(b)slope of the regression line

(c)coefficient of linear correlation

(d)coefficient of determination

52.If the coefficient of determination is close to 1, then

(a)the least-squares regression line equation has no explanatory value.


(b)the sum of the square residuals is large compared to the total variation.

(c)the linear correlation coefficient is close to zero.

(d)the least-squares regression line equation explains most of the


variation in the response variable.

53.The coefficient of determination is the off the linear correlation


coeficient.

(a)opposite

(b)square

(c)reciprocal

(d)square root

54.An influential observation is an observation that significantly affects the


value of the

(a)the mean of the response variable.

(b)the median of the response variable.

(c)the median of the predictor variable.

(d)the slope of the least-squares regression line.

55.What effect will an influential observation have upon the graph of the
leastsquares regression line?

(a)It will have no effect.


(b)It will lower the value of the correlation coefficient to make further
analysis meaningless.

(c)It will pull the graph towards the observation.

(d)It will push the graph away from the observation.

56.A contingency table relates

(a)only continuous random variables.

(b)a particular response with order in which that response should be


applied.

(c)the difference in the means of two random variables.

(d)two categories of data.

57.To eliminate the effects of either the row or the column variables in a
contingency table, a distribution is created.

(a)Student's

(b)normalized

(c)marginal

(d)x2

58.The peak shopping time at a home improvement store is between


11.00a.m. and 9.00 p.m. on Saturday mornings. Store Manager at the
Huge Bazzar Shopping Complex randomly selected 80 customers last
Saturday morning and decided to observe their shopping habits. He
recorded the number of items that each of the customers purchased as
well as the total time the customers spent in the store. Identify the types of
variables recorded by the home improvement store.

(a)Number of items - discrete; total time - discrete

(b)Number of items - continuous; total time - continuous

(c)Number of items - continuous; total time - discrete

(d)Number of items - discrete; total time - continuous

59.A random variable is

(a)the variable for which an algebraic equation is solved.

(b)a numerical measure of the outcome of a probability experiment.

(c)generated by a random number table.

(d)a qualitative attribute of a population.

60.The sum of the probabilities of a discrete probability distribution must be

(a)greater than one.

(b)equal to one.

(c)less than or equal to zero.

(d)between zero and one.

61.The random variable x represents the number of girls in a family of three


children. Assuming that boys and girls are equally likely, find the mean
and standard deviation for the random variable x.
(a)Mean: 2.25; standard deviation: 0.76

(b)Mean: 1.50; standard deviation: 0.76

(c)Mean: 1.50; standard deviation: 0.87

(d)Mean: 2.25; standard deviation: 0.87

62.Which of the below is not a requirement for binomial experiment?

(a)For each trial, there are two mutually exclusive outcomes.

(b)The trials are mutually exclusive.

(c)The probability of success is fixed for each trial of the experiment.

(d)The experiment is performed a fixed number of times.

63.If p is the probability of success of a binomial experiment, then the


probability of failure is

(a)n/x.

(b)- p.

(c)I - p.

(d)x/n.

64.A quiz consists of 920 true or false questions. If the student guesses on
each question, what is the mean number of correct answers'?

(a)

(b)460
(c)920

(d)184

65.A quiz consists of 780 true or false questions. If the student guesses on
each question, what is the standard deviation of the number of correct
answers'?

(a)

(b)I

(c)2

(d)More than 2

66.Given that a random variable x, the number of successes, follows a


Poisson process, the probability of 2 or more successes in any sufficiently
small subinterval is

(a)zero.

(b)none of these

(c)any number between zero and one.

(d)one.

67.Given that a random variable x, the number of successes, follows a


Poisson process, then the probability of success for any two intervals of
the same size

(a)is the same.


(b)are reciprocals.

(c)none of these

(d)are complementary.

68.If a hypothesis test of the linear relation between the explanatory and the
response variable is of the type where HO: 01 = 0, H1: (31 > 0, then we
are testing the claim that

(a)no linear relationship exists.

(b)the slope of the least-squares regression model is positive.

(c)the slope of the least-squares regression model is negative.

(d)a relationship exists without regard to the sign of the slope.

69.The difference between the observed and predicted value of the response
variable is a

(a)variance.

(b)residual.

(c)standard error of the estimate.

(d)test statistic.

70.In linear regression, what is the unbiased estimator of a called'?

(a)Standard deviation

(b)Sample standard deviation


(c)Standard error of the estimate

(d)Variance

71.One of the requirements for conducting inference on the least-squares


regression model is that the

(a)mean of the explanatory variable remains constant while the standard


deviation changes at a constant rate.

(b)mean of the response variable changes at a constant rate while the


standard deviation remains constant.

(c)mean of the response variable remains constant while the standard


deviation changes at a constant rate.

(d)mean of the explanatory variable changes at a constant rate while the


standard deviation remains constant.

72.How does a confidence interval differ from a prediction interval'?

(a)Confidence intervals are constructed about the predicted values of y,


while prediction intervals a constructed about a particular value of x.

(b)Confidence intervals are used to measure the accuracy of a single


individual's predicted value, while a prediction interval is used to
measure the accuracy of the mean response of all the individuals in the
population.

(c)Confidence intervals are constructed about the predicted values of x,


while prediction intervals a constructed about a particular value of y.
(d)Confidence intervals are used to measure the accuracy of the mean
response of all the individuals in the population, while a prediction
interval is used to measure the accuracy of a single individual's
predicted value.

73.By saying that we reject the null hypothesis in a hypothesis test of the
coefficients of a multiple linear regression, we are implying that

(a)the relationship is not linear.

(b)non-parametric methods must be used on this problem.

(c)at least one coefficient is different from zero.

(d)all coefficients are different from zero.

74.As the number of the degrees of freedom increases, the x2 distribution

(a)does not change shape as the degrees of freedom change.

(b)becomes less symmetric.

(c)becomes more symmetric.

(d)becomes exponential.

75. Atestp is an inferential rocedure used to determine whether a frequency


distribution follows a defined distribution.

(a)normality

(b)xz

(c)F
(d)goodness-of-fit

76.In a x2 test of independence, the null hypothesis is that

(a)the random variables are dependent.

(b)there is not an association.

(c)there is an association.

(d)each element of each set has the same probability of occurrence.

77.How, in general, is the requirement of equal population variances to


allow the use of analysis of variance method verified?

(a)The largest sample standard deviation is equal to the sum of the other
standard deviations.

(b)The largest sample standard deviation must be less than or equal to


twice the smallest sample standard deviation.

(c)All three sample standard deviations must be equal.

(d)The largest sample standard deviation is less than or equal to the


variance of the smallest sample.

78.Given the length of a human's arm, x, and the length of human's middle
finger, y, would you expect a positive correlation, a negative correlation
or no correlation'?

(a)Negative correlation

(b)No correlation
(c)Positive correlation

(d)Could be anything

79.Given the supply of a commodity, x, and the price of a commodity, y,


would you expect a positive correlation, a negative correlation or no
correlation'?

(a)Negative correlation

(b)No correlation

(c)Positive correlation

(d)Could be anything

80.Given the size of a human's brain, x, and their score on an IQ test, y,


would you expect a positive correlation, a negative correlation or no
correlation'?

(a)Positive correlation

(b)Negative correlation

(c)No correlation

(d)Could be anything

81.In a chi-square test of homogeneity of proportions, we test the claims that

(a)across a single sample, the proportion of individuals with the same


characteristic is the same as the population.
(b)the proportion of individuals with a given characteristic doesn't change
over time.

(c)the proportion of a population having a given characteristic is based on


the homogeneity of the population.

(d)different populations have the same proportions of individuals with the


same characteristics.

82.Identify the major objective of using regression model in a data set.

(a)Causability between dependent and independent variables

(b)Association between dependent and independent variables

(c)Skewness in the data

(d)Dispersion in the data

[UGC D-2006]

83.Match List I with List II and select the correct answer using the codes
given.
Codes:

[UGC D-2006]

84.Which one of the following is correct?

(a)Arithmetic mean is a numerical value.


(b)Arithmetic mean is not affected by the variability in the data set.

(c)Arithmetic mean is always a positive number.

(d)Arithmetic mean is not useful for any further statistical analysis of the
data.

[UGC D-2007]

85.Which of the following is the appropriate method for study of


dispersion'?

(a)Mean

(b)Median

(c)Mode

(d)Standard deviation

[UGC D-2007]

86.Which one of the following is a measure of dispersion'?

(a)Median

(b)Geometric mean

(c)Standard deviation

(d)Arithmetic mean

[UGC D-2008]

87.Who was the originator of the regression analysis?


(a)Clark and Evans

(b)Karl Pearson

(c)Spearman

(d)Francis Galton

[UGC D-2008]

88.Circle and sector diagram is also known as

(a)ring diagram.

(b)pile diagram.

(c)pie diagram.

(d)pictorial diagram.

[UGC D-2009]

89.Match List I with List II and select the correct answer by using the codes
given.

Codes:
[UGC D-2009]

90.Given below are two statements, one labelled as Assertion (A) and the
other labelled as Reason (R) Select your answer from the codes given.

Assertion (A): Standard deviation is a measure of dispersion of values in


a data set.

Reason (R): Value tends to move away from the central value in the
dataset.

(a)Both (A) and (R) are true and (R) is the correct explanation of (A).

(b)Both (A) and (R) are true but (R) is not the correct explanation of (A).

(c)(A) is true but (R is false.

(d)(A) is false but (R) is true.

[UGC J-2005]

91.Significance of regression coefficient in bivariate analysis is studied by


using the

(a)t - test.

(b)F - test.

(c)standard error of estimate.


(d)standard deviation.

[UGC J-2005]

92.Representation of the Universe is best done by which of the following?

(a)Random sampling

(b)Systematic random sampling

(c)Simple random sampling

(d)Stratified random sampling

[UGC J-2005]

93.Which one of the following is the commonly used measure of central


tendency?

(a)Mean

(b)Median

(c)Mode

(d)Harmonic mean

[UGC J-2007]

94.Which one of the following is centrographic measure'?

(a)Standard distance

(b)Median

(c)Arithmetic mean
(d)Geometric mean

[UGC J-2008]

95.Consider the following statements, which of these statements are


correct'?

1.Median is positional average in nature.

2.Rn. value of 2.15 indicates dispersed pattern.

3.Standard deviation will alter by change of origin.

4.Gini coefficient is a measure of inequality in distribution.

(a)1, 2 and 4

(b)1, 2, 3 and 4

(c)1, 3 and 4

(d)1 and 4

[UGC J-2011]

96.Sector diagram is also known as

(a)pictorial diagram.

(b)ring diagram.

(c)block pile diagram.

(d)pie diagram.

[UGC J-2011]
97.Relative shift of mean centre of population distribution in any region over
a period of time can be examined applying

(a)nearest neighbour analysis.

(b)harmonic mean.

(c)mean deviation from mean.

(d)weighted arithmetic mean.

[UGC J-2010]

98.Temperature measured on Kelvin scale is an example of

(a)comparative scale.

(b)Vernier scale.

(c)interval scale.

(d)ratio scale.

[UGC J-2010]

99.The suitable method to measure the change in spatial concentration of


scheduled tribes population in India during the last three decades is

(a)coefficient of variation.

(b)Gini coefficient.

(c)index of diversification.

(d)index of dissimilarity.
[UGC J-2010]

100.Harmonic mean as a method of central tendency is suitable for data


pertaining to

(a)index numbers.

(b)growth rates.

(c)wind speed.

(d)population density.

[UGC D-2010]

ANSWER KEYS

 
1.The areas marked `X' on the given map are the plateaus of the type known
as

(a)coastal.

(b)continental.

(c)intermundane.

(d)piedmont.

[UPSC-1995]

2.The given diagram indicating the curve of W, Penck's cycle of erosion,


shows which of the following features?
1.Only upliftment in stage A.

2.Rate of upliftment of stage A is lower than that of B.

3.Upliftment ins more than erosion in the stage B.

4.Rate of erosion in stage E is higher than that of D.

Select the correct answer using the codes given.

(a)1, 2 and 3

(b)3 and 4

(c)2 and 4

(d)1, 2 and 4

[UPSC-1997]

3.The given diagram represents


1.young

2.old

3.oldest

(a)monoclinal fold.

(b)overfold.

(c)recumbent fold.

(d)symmetrical fold.

[UPSC-1998]

4.In the given map, `A' refers to


(a)Alexandria.

(b)Port Said.

(c)Suez.

(d)Cairo.

[UPSC-1998]

5.The shaded landform region in the given Anglo-American map is

(a)Rocky mountains.

(b)interior highlands.
(c)great plains.

(d)interior plains.

[UPSC-1998]

6.Which one of the following patterns of drainage does the given diagram
represent'?

(a)Annular

(b)Dendritic

(c)Radial

(d)Trellis

[UPSC-1999]

7.In the given map of Antarctic, Dakshin Gangotri has been labelled as
(a)1.

(b)2.

(c)3.

(d)4.

[UPSC-1999]

8.The following features produced by volcanic activities are shown in the


diagram:
(a)Batholith

(b)Dyke

(c)Crater

(d)Sill

[UPSC-2000]

9.In the given diagram, the portion of the river marked as `X' is called

(a)braided river.

(b)distributary.
(c)incised meander.

(d)yazoo.

[UPSC-2001]

10.The given lock diagram shows

(a)soil.

(b)debris flow.

(c)rock slide.

(d)slump.

[UPSC-2001]

11.The diagram given below represents a


(a)normal fault.

(b)reverse fault.

(c)hinge fault.

(d)transform fault.

[UPSC-2002]

12.Consider the following drainage patterns:


A.Rectangular

B.Radial

C.Annular

D.Trellis

Codes:

[UPSC-2002]

13.In the diagram below, `A' represents

(a)arete.

(b)bergschrunds.

(c)cirque.
(d)Roches moutonnees.

[UPSC-2002]

14.Consider the following straits:

(a)Bosporus

(b)Hormuz

(c)Dardanelles

(d)Bab-el-Mandeb

Identify the straits marked as 1, 2, 3 and 4 on the given map and select the
correct answer using the codes given.

Codes:
[UPSC-2002]

15.The given in the diagram is an example of

(a)normal fault.

(b)over-thrust fault.

(c)reverse fault.

(d)transcurrent fault.
[UPSC-2003]

16.In the given diagram, the point marked as `X' in B part indicates

(a)flood plain.

(b)natural levee.

(c)meander scar.

(d)wind gap.

[UPSC-2003]

17.The given diagram represent a/an


(a)asymmetrical fold.

(b)monoclonal fold.

(c)mover fold.

(d)recumbent fold.

[UPSC-2004]

18.In the diagram given below, which of the numbers represent bluffs?

Select the correct answer using the codes given.

(a)1 and 3

(b)3 and 4

(c)1 and 5

(d)2 and 3

[UPSC-2004]
Which type of fold is formed at A in the figure given above?

(a)Isoclinals fold

(b)Plunge fold

(c)Open fold

(d)Recumbent fold

[UPSC-2005]

Which one of the following faults has been shown in the figure above?

(a)Reverse fault

(b)Normal fault

(c)Strike-slip fault
(d)Step fault

[UPSC-2005]

21.The type of forests found in the shaded region of the map is

(a)thorn and scrub forests.

(b)monsoon forests.

(c)tropical rainforests.

(d)deciduous forests.

[UPSC-1995]

22.Which one of the following types of forests grow in the shaded portion of
the given rough outline map of India?
(a)Mixed deciduous

(b)Tropical savannah

(c)Semi-arid scrublands

(d)Mangrove forests

[UPSC-1997]

23.Identify the islands marked 1, 2, 3 and 4 in the given map and select the
correct answer by using the codes given.
A.Car Nicobar

B.Little Andaman

C.Great Nicobar

D.Little Nicobar

Codes:

[UPSC-1998]

24.The dotted line in the map shows

(a)average annual temperature divide.

(b)annual isohyets of 200-400.

(c)al-India water divide.


(d)contour of 300 metres.

[UPSC-1997]

25.Match types of soil distribution shown in the rough outline map of India
with the region labelled 1, 2, 3and 4 and select the correct answer using
the codes given.

A.Red and yellow

B.Grey and brown

C.Mixed red and black

D.Red loamy

Codes:
[UPSC-1998]

26.The shaded area in the given rough outline map represents which one of
the following types of climatic regions (after Koppen)?

(a)Am W

(b)BS hw

(c)BS

(d)Cwg

[UPSC-1998]

27.The shaded areas in the rough outline map of India shows


(a)summer rainfall of 80-120 cm.

(b)summer humidity of 80-90 per cent.

(c)January pressure ranging between 1,016 and 1,018 mb.

(d)winter rainfall of 5-10 cm.

[UPSC-1998]

28.The climatic graph given below indicates rainfall and temperature


conditions of which of the following cities?
Here, 1 represents Average Mean Temp (Maximum) and 2 represents
Average Mean Temp (Minimum).

(a)Chandigarh

(b)Delhi

(c)Jaipur

(d)Lucknow

[UPSC-1998]

29.In the given map of Deccan region of India, rivers Mani ira, Godavari,
Bhima and Penganga have been labelled, respectively, as
(a)2, 3, 4 and 1.

(b)2, 3, 1 and 4.

(c)3, 2, 1 and 4.

(d)3, 2, 4 and 1.

[UPSC-1999]

30.The climate graph (showing mean monthly distribution of temperature


and rainfall) in the figure depicts conditions of
(a)Bangalore.

(b)Nagpur.

(c)Chennai.

(d)Jodhpur.

[UPSC-1999]

31.Match the areas labelled as 1, 2, 3 and 4 in the given rough map of India
with the following characteristics:

Codes:

[UPSC-1999]

32.The given diagram show the river regime of


(a)Kaveri.

(b)Godavari.

(c)Ganga.

(d)Sutlej.

[UPSC-2000]

33.Match the monthly distribution of temperature indicated as 1, 2, 3 and 4


in the given diagram with the following towns:
A.Shillong

B.Delhi

C.Jodhpur

D.Thiruvananthapuram

Codes:

[UPSC-2000]

34.Match the four types of soil areas marked as 1, 2, 3 and 4 in the given
map with the following soil types:
A.Red sandy soil

B.Mixed red and black soil

C.Red loamy soil

D.Red and yellow soil

Select the correct answer using the codes given.

Codes:

[UPSC-2000]

35.In the given diagram, spatialspread of a river basin of India has been
shown. Identify the river.
(a)Kaveri

(b)Brahmini

(c)Godavari

(d)Mahanadi

[UPSC-2001]

36.The given diagram shows the temperature regime (°C) of

(a)Kolkata.

(b)Cherrapunji.

(c)Kochi.
(d)Bhopal.

[UPSC-2001]

37.In the following map of India, the shaded area records mean July
temperature between

(a)20°C and 22.5°C.

(b)25°C and 27.5°C.

(c)27.5°C and 29°C.

(d)29°C and31°C.

[UPSC-2002]

38.What is represented by the shaded portion in the map given'?


(a)Black soil

(b)Desert soil

(c)Grey and brown soil

(d)Red and yellow soil

[UPSC-2005]

39.The given map with shaded portion shows

(a)area affected by gully erosion.

(b)distribution of sugarcane.
(c)area of deciduous forest.

(d)rice-growing area.

[UPSC-1995]

40.The location `X' marked in the map shown below is that of a hydel power
station. This is

(a)Koyna on Krishna river.

(b)Sharavati on Sharavati river.

(c)Tungabhadra on Tungabhadra river.

(d)NagarjunaSagar on Krishna river.

[UPSC-2004]

41.The correct sequence of four railway stations (1, 2, 3, 4) shown in the


given map between Delhi and Mumbai is
(a)Jhansi, Itarsi, Jalgaon, Manmad.

(b)Itarsi, Manmad, Jhansi, Jalgaon.

(c)Itarsi, Jhansi, Manmad, Jalgaon.

(d)Jhansi, Itarsi, Manmad, Jalgaon.

[UPSC-2001]

42.In the given map, the shaded area represents a region of important forest
resource of our country.
Name of the forest is

(a)Tropical wet evergreen.

(b)Tropical wet deciduous.

(c)Subtropical wet forest.

(d)Tropical dry deciduous.

[UPSC-2001]

43.Match List I with List II and select correct answer by using the codes
given.
A.Tur

B.Maize

C.Potato

D.Sugarcane

Codes:

[UPSC-2001]

44.In the given map, the petrochemical concentration, known as the Golden
corridor industrial area, is located at the place marked
(a)1.

(b)2.

(c)3.

(d)4.

[UPSC-2000]

45.The site of which one of the following hydel power stations is marked X
in the given map?

(a)Koyna on Koyna river.


(b)Sharavation Sharavati river.

(c)Srisailam on Krishna river.

(d)Nagarjuna Sagar on Krishna river.

[UPSC-2000]

46.Consider the map given below:

The regions marked 1, 2, 3 and 4 are, respectively, known for their


cultivation of

(a)sugarcane, ragi, cotton and gram.

(b)sugarcane, gram, cotton and ragi.

(c)cotton, ragi, gram and sugarcane.

(d)cotton, ragi, gram and sugarcane.

[UPSC-1998]
47.The portions marked in the given rough outline map of India indicate the
distribution of

(a)coal fields.

(b)hydel power plants.

(c)oil fields.

(d)iron ore deposits.

[UPSC-1997]

48.The shaded portion in the given rough outline map of India represents

(a)areas affected by drought.


(b)areas under sugarcane cultivation.

(c)densely urbanized areas.

(d)areas under jute cultivation.

[UPSC-1997]

49.The shaded region in the given map relates to

(a)iron ore-producing regions.

(b)mineral-bearing regions.

(c)regions deficient in non-metallic minerals.

(d)regions of mica mining.

[UPSC-1995]

50.The given map with shaded region shows


(a)sugarcane-growing areas.

(b)cotton-growing areas.

(c)areas having high density of population.

(d)area of high industrial development.

[UPSC-1995]

51.The mean monthly temperature and mean monthly rainfall plotted in the
above diagram pertain to
(a)London.

(b)New York.

(c)Rome.

(d)Vancouver.

[UPSC-2002]

52.In the diagram below, wind shown is an example of


(a)anabatic wind.

(b)adiabatic wind.

(c)katabatic wind.

(d)stroph.

[UPSC-2002]

53.Which one of the following illustrations correctly represents an average


temperate cyclone in Northern hemisphere (where CF stands for cold
front, WF sands for warm front and WS stands for warm sector)?
[UPSC-2002]

54.The shaded area in the given world map represents the natural
regioncharacterized by
(a)warm summer continental climate.

(b)cool summer continental climate.

(c)southern Tundra climate.

(d)marine west coast climate.

[UPSC-2000]

55.In the given map, the highest annual range of temperature is recorded in
the area labelled as

(a)3.
(b)1.

(c)4.

(d)2.

[UPSC-2000]

56.In the sketch of a tropical cyclone shown in the given figure, the area
marked `X' is characterized by

(a)rain.

(b)clear sky.

(c)cloud.

(d)dirty sky.

[UPSC-1997]

57.In the rough outline map given below, the shaded portion marked X
represents region of.
(a)tropical grassland climate.

(b)subtropical semi-arid climate.

(c)subtropical grassland climate.

(d)humid subtropical climate.

[UPSC-1996]

58.Which of the following pairs are correctly matched?

Select the correct answer using the codes given.

(a)1 and 3
(b)2, 3 and 4

(c)1 and 4

(d)1, 2, 3 and 4

X in the figure given is a river. In digital database terms, it is an/a

(a)node and denotes a linear feature.

(b)arc and denotes a linear feature.

(c)polygon and denotes an area feature.

(d)arc and denotes a point feature.

60.In the diagram shown below, points A and B are located on the ground
surface 7.5 km apart. These two points appear on a vertical aerial
photograph as `a' and `b' separated by a photographic distance of 100 mm.
The scale of the photograph is
(a)1:75,00,000.

(b)1:7,50,000.

(c)1:75,000.

(d)1:750.

61.Match List I with List II and select correct answer by using the codes
given.
Codes:

62.Match List I with List II and select correct answer by using the codes
given.
Codes:
63.Match List I with List II and select correct answer by using the codes
given.

Codes:
64.In the diagram given below, the areas marked `X' and `Y', respectively,
represent which of the following soil types'?

(a)Sand and clay

(b)Loam and clay

(c)Clay and sandy loam

(d)Silt and sand

[UPSC-2004]
65.Which one of the following statements is not correct from the diagram of
the given food web'?

(a)Mouse is a secondary consumer.

(b)Shrew is a tertiary consumer.

(c)Grasshopper is a primary consumer.

(d)Owl is both a secondary and a quaternary consumer.

[UPSC-2001]

66.The shaded regions in the given world map represents

(a)temperate grasslands.
(b)temperate deciduous forests.

(c)tropical grasslands.

(d)mixed deciduous forests.

[UPSC-1999]

67.Consider the following industrial regions of Japan:

A.Kinki region

B.Kitakyushu region

C.Kanto region

D.Nagoya region

Match the above region with points marked 1 to 6 in the map. Select
correct answers using the codes given.

Codes:
[UPSC-2001]

68.In the Multiple Nuclei Model of Harris Ullman, the shaded area marked
A represents

(a)heavy manufacturing area.

(b)residential suburb.

(c)outlying business district.

(d)Industrial suburb.

[UPSC-2000]

69.Consider the following crop regions of Sri Lanka:


A.Coconut region

B.Corn and millet region

C.Rubber region

D.Tea region

Identify the location of the respective regions on the map using the codes
given.

Codes:

[UPSC-2001]

70.Places marked as 1, 2, 3 and 4 in the given map are noted for the
following industries:
A.Iron and steel

B.Dairy products

C.Textile

D.Wine

Match the industry with the notation on the map and select the correct
answer using the codes given.

Codes:

[UPSC-1999]

71.The shaded area in the given map represents the distribution of


(a)wheat and sugarcane.

(b)wheat and tobacco.

(c)maize and tobacco.

(d)maize and sugarcane.

[UPSC-1999]

72.In the rough outline map given below, the shaded portion marked X
represents region of

(a)tropical grassland climate.


(b)subtropical semi-arid climate.

(c)subtropical grassland climate.

(d)humid subtropical climate.

[UPSC-1996]

73.Transportation cost-distance relationship is shown in the following


figures. If a personwants to travel from A to B, then which one of the
following relationships will be beneficial to him'?
[UPSC-1996]

74.On the basis of the given map, the best place for locating a big iron and
steel plant would be the site marked as
(a)A.

(b)B.

(c)C.

(d)D.

[UPSC-1996]

75.The area marked X in the given diagram representing concentric zone


theory of urban land use is called

(a)Central Business District.


(b)Low Class Residential.

(c)Medium Class Residential.

(d)High Class Residential.

[UPSC-1995]

76.In the given outline map of the world, the shaded areas indicate the
cultivation of

(a)barley.

(b)oats.

(c)maize.

(d)wheat.

[UPSC-1998]

77.Match List I with List II and select the correct answer using the code
given.
Codes:

[UPSC-2003]

78.The circumference of geography given in the diagram below was


propounded by
(a)E.A. Ackerman.

(b)V.A. Anuchin.

(c)J.N.L. Baker.

(d)N.M. Fennerman.

[UPSC-2002]

79.Consider the following cities of Greek and Roman periods:

A.Carthage

B.Byzantium

C.Sardis

D.Loari

Identify cities marked as 1, 2, 3 and 4 on the given map and select the
correct answer using the codes given.

Codes:
[UPSC-2002]

80.The model of environmental perception and behaviour shown in the


given block diagram was propounded by

(a)Bounding.

(b)Down.

(c)Kirk.

(d)Porteous.

[UPSC-2000]

81.The kind of relationship that exists between the elements in the given
system is
(a)complex.

(b)feedback.

(c)parallel.

(d)simple.

[UPSC-2001]

82.According to ancient Indian geographers, 1, 2, 3 and 4 marked on the


modern map show, respectively

(a)Kraunca, Kusa, Salmali, Plaska, Dwipas.

(b)Plaska, Kusa, Kraunca, Salmali, Dwipas.


(c)Plaska, Kusa, Salmali, Kraunca Dwipas.

(d)Salmali, Plaska, Kusa, Kraunca, Dwipas.

[UPSC-2000]

83.The given outline map of the world was prepared by

(a)Hecataeus.

(b)Heordotus.

(c)Ptolemy.

(d)Strabo.

[UPSC-2000]

84.Who among the following developed the techniques shown in the figure
for the measurement of the length of Equator'?
(a)Anaximander

(b)Eratosthenes

(c)Hipparchus

(d)Ptolemy

[UPSC-1999]

85.In the diagram given below, which out of 1, 2, 3 and 4 are cold currents'?

(a)1 and 3

(b)2 and 4

(c)2 and 3
(d)1 and2

86.In the diagram of marine biological zones, `A' represents

(a)epipelagic zone.

(b)abysopelagic zone.

(c)meropelagic zone.

(d)bathypelagic zone.

87.Which reef type is shown by `X' marked on the diagram'?

(a)Fringing reef

(b)Barrier reef
(c)Patch reef

(d)Atoll

88.The area marked `X' in the given diagram represents

(a)Kurile Trench.

(b)Mindano Trench.

(c)Marina's Trench.

(d)Tonga-Kermadec Trench.

89.Consider the following labels in the given diagram of a vertical air


photograph:
1.Negative plane

2.Nadir point

3.Camera axis

4.Ground plane

Which of the above labels are correct?

(a)1, 2 and 3

(b)2, 3 and 4

(c)1, 3 and 4

(d)1, 2 and 4

90.The given graph of spatial point random sampling is:


(a)simple.

(b)nested.

(c)stratified.

(d)systematic.

91.Match List I with List II and select correct answer by using the codes
given.
Codes:

92.Consider the figures X and Y Which one of the following statements is


correct?
(a)Figure X is small scale and the distance between A and B is 40 metres.

(b)Figure Y is large scale and the distance between C and D is km.

(c)Figure X is large scale and Figure Y is small scale, the distances


between A and B and C and D are 40 and 20 km, respectively.

(d)Figure X is large scale and Figure Y is small scale. The ground


distance between A and B and C and D is equal.

93.The given map was prepared by

(a)Arabs.

(b)Greeks.

(c)Phoenicians.
(d)Romans.

94.A comparison of polar zenithal projections drawn on the same scale is


shown in the figure. Here, X and Y are, respectively

(a)gnomonic and orthographic projections.

(b)orthographic and gnomonic projections.

(c)gnomonic and zenithal stereographic projections.

(d)orthographic and zenithal stereographic projections.

95.The three profiles shown in diagram X, Y and Z represent, respectively


(a)projected, composite and superimposed profiles.

(b)superimposed, projected and composite profiles.

(c)composite, superimposed and projected profiles.

(d)superimposed, composite and projected profiles.


In the given map, four cities, namely, (A) Adelaide, (B) Brisbane, (C)
Melbourne and (D) Sydney are marked with numbers. Match the city with
the correct number and select the correct answer using the codes given.

Codes:

[UPSC-2008]
In the map given above, four cities of Germany, namely, (A) Berlin, (B)
Cologne, (C) Frankfurt and (D) Munich are marked with numbers. Match
the city with the correct number and select the correct answer using the
codes given.

Codes:

[UPSC-2008]
In the map given above, four lakes, namely, (A) Lake Turkana (Lake
Rudolf), (B) Lake Nyasa, (C) Lake Tanganyika and (D) Lake Vitoria are
marked with numbers. Match the lake with the correct number and select
the correct answer using the codes given.

Codes:

[UPSC-2008]
In the given map, four important towns of South Africa namely, (A)
Durban, (B) Cape Town, (C) East London and (D) Port Elizabeth are
marked with numbers. Match the city with the correct number and select
the correct answer using the codes given.

Codes:

[UPSC-2008]
In the map given, which one of the following types of climates based on
Koppen's system is indicated by the shaded portion?

(a)AW

(b)Caf/Cfa

(c)Csa

(d)Daf

[UPSC-2008]
In the map given, four cities, namely, (A) Birmingham, (B) Bristol, (C)
Liverpool and (D) Manchester are marked with numbers. Using the code
given below, match them correctly and select the correct answer.

Codes:

[UPSC-2007]
In the map given, four cities of USA are marked with numbers (1), (2),
(3) and (4). Consider the map and match List I with List II and select the
correct answer using the code given.

Codes:
[UPSC-2007]

Consider the map of India given. The shaded area in the above outline
map of India represents which one of the following climatic types as
propose by C.W. Thornthwaite?

(a)CA' w

(b)DB'd

(c)EA' d

(d)CB'w

[UPSC-2007]
In the given map, which one of the following types of soils is indicated by
the marked area'?

(a)Black soil

(b)Grey and brown soil

(c)Laterite soil

(d)Red and yellow soil

[UPSC-2006]
In the map given here, four Emirates Cities,namely, (A) Sharjah, (B)
Dubai, (C) Abu Dhabi and (D) Dohaare marked with numbers. Using the
codes given below, match them correctly and select the correct answer.

Codes:

[UPSC-2006]

In the given map, four areas of sea/gulf bordering Australia, namely, (A)
Arafura Sea, (B) Coral Sea, (C) Gulf of Carpentaria and (D) Timor Sea
are marked with numbers. Match them and select the correct answer using
the codes given.

Codes:
[UPSC-2006]

107.In the given map, four cities viz:

ABordeaux

BLyon

CMarseille and

DParis

are marked with numbers. Match them and select the correct answer using
the codes given below.
[UPSC-2006]

In the given map, four republics of Russian Federation are marked as 1, 2,


3 and 4. Which one of them is Chechnya?

(a)1

(b)2

(c)3

(d)4

[UPSC-2006]

109.Given below is a map of some countries which were parts of the


erstwhile Soviet Union, with water bodies shown by shaded areas.
The countries marked 1, 2, 3, 4 and 5 are, respectively,

(a)Tajikistan, Turkmenia, Uzbekistan, Kirgizia, Kazakhstan.

(b)Turkmenia, Kirgizia, Tajikistan, Uzbekistan, Kazakhstan.

(c)Kazakhstan, Uzbekistan, Tajikistan, Kirgizia, Turkmenia.

(d)Kazakhstan, Turkmenia, Uzbekistan, Kirgizia, Tajikistan. [UPSC-


1995]

110.The graph given below shows the mean average monthly temperatures
(in °C) and mean monthly rainfall (in cms) of place.

This graph is indicative of which one of the climatic zones of the world'?
(a)Wet and dry tropical

(b)Rainy tropical

(c)Semi-arid tropical

(d)Temperate marine

[UPSC-1995]

111.Consider the map given below:

The division along India's coastal region indicates

(a)coastal pollution zones.

(b)salinity density isopleths.

(c)the areas up to which sovereignty extends.

(d)underwater relief contours.

[UPSC-1995]
112.In the map given, three out of the four places marked indicate places
where a thermal power station is located.

The place where there is no thermal power station is marked:

(a)1.

(b)2.

(c)3.

(d)4.

[UPSC-1995]

113.In the map given, four areas are differently shaded, three of which
indicate cereal crops production areas and one indicates a non-cereal
cropproduction area.As per the index given, the non-cereal crop
production area is
(a)1.

(b)2.

(c)3.

(d)4.

[UPSC-1995]

114.Of the four places marked 1, 2, 3 and 4 on the map, the one indicating a
tidal port is
(a)1.

(b)2.

(c)3.

(d)4.

[UPSC-1995]

115.Consider the map givenindicating the places frequently figuring in the


news.
Which one of them is Chechnya? Choose the correct answer.

(a)1

(b)2

(c)3

(d)4

[UPSC-1995]

116.Examine the map of Jammu and Kashmir given. The mountains ranges
marked 1, 2, 3 and 4 are, respectively,

(a)Ladakh, Zanskar, Karakoram and Pir Panjal.

(b)Karakoram, Ladakh, Zanskar and Pir Panjal.

(c)Karakoram, Zanskar, Pir Panjal and Ladakh.

(d)Ladakh, Pir Panjal, Karakoram and Zanskar.

[UPSC-1995]
117.Consider the map given.

The dotted (broken) line in the map is the

(a)Durand line.

(b)McMahon line.

(c)line of demarcation between Indian and Pakistan suggested by the


Boundary Commission (1947).

(d)route followed by the young husband expenditure.

[UPSC-1996]

118.The rough outline map shows a portion of the Middle East. The
countries labelled A, B, C and D are, respectively,
(a)Syria, Iraq, Jordan and Saudi Arabia.

(b)Syria, Iraq, Saudi Arabia, and Jordan.

(c)Iraq, Syria, Saudi Arabia and Jordan.

(d)Iraq, Syria, Jordan and Saudi Arabia.

[UPSC-1996]

119.A rough outline map of northern Sri Lanka is shown in the figure. Jaffna
is located at the point marked

(a)A.
(b)B.

(c)C.

(d)D.

[UPSC-1996]

120.Consider the map given.

The river shown on the map is

(a)Irrawaddy.

(b)Mekong.

(c)Chao Phraya.

(d)Salween.

[UPSC-1997]

121.Consider the map given.


The predominant languages spoken in the areas marked A, B, C and D
are, respectively,

(a)Brajbhasha, Bhojpuri, Maithili and Awadhi.

(b)Awadhi, Maithili, Bhojpuri and Brajbhasha.

(c)Brajbhasha, Awadhi, Bhojpuri and Maithili.

(d)Maithili, Bhojpuri, Brajbhasha and Awadhi.

[UPSC-1997]

122.Consider the map given.


The places marked A, B, C and D in the map are, respectively,

(a)Rift valley region, Chhattisgarh plain, rain shadow region and Chhota
Nagpur.

(b)Chhattisgarh plain, Chhota Nagpur plateau, rift valley region and rain
shadow region.

(c)Rift valley region, Chhattisgarh plain, Chhota Nagpur plateau and rain
shadow region.

(d)Chhattisgarh plain,rain shadow region, Chhota Nagpur plateau and rift


valley region.

[UPSC-1997]

123.Consider the map given.


The places marked A, B, C and D in the map are, respectively, known for
the cultivation of

(a)groundnut, ragi, tobacco and sugarcane.

(b)groundnut, sugarcane, ragi and tobacco.

(c)ragi, sugarcane, groundnut and tobacco.

(d)ragi, groundnut, sugarcane and tobacco.

[UPSC-1997]

124.Consider the geographical details given in the following figure:


The point marked by A in the figure indicates the countries in

(a)North America.

(b)South America.

(c)Europe.

(d)Asia.

[UPSC-1997]

125.In the map shown, rivers labelled as 1, 2, 3 and 4 are, respectively,

(a)Kosi, Gomati, Ghaghara and Gandak.

(b)Kosi, Ganga, Gomati and Ghaghara.

(c)Gandak, Ganga, Gomati and Ghaghara.


(d)Teesta, Gomati, Ghaghara and Kosi.

[UPSC-1997]

126.Consider the map given.

Of the four shaded areas in the map, the one which is characterized by hot
dry summers, mild and moist winters and seasonal reversal of winds is
the area labelled

(a)1.

(b)2.

(c)3.

(d)4.

[UPSC-1997]

127.Forest areas have been labelled as 1, 2, 3 and 4 in the rough outline map
of India.
Among these, those which were threatened in 1997 by a serious epidemic
include

(a)teak forests of 3 and 4.

(b)oak forests of 1 and sal forests of 2.

(c)sal forests of 3.

(d)sandalwood forests of 4.

[UPSC-1998]

128.Commercial production of mineral oil has started recently in which one


of the areasof India labelled 1, 2, 3 and 4 in the rough map given.
(a)1

(b)2

(c)3

(d)4

[UPSC-1998]

129.The rough outline map given shows centres of cement industry labelled
1, 2, 3 and 4. Match these centres with the following sets:
(a)Katni

(b)Tirunelveli

(c)Sikka

(d)Churk

Select the correct answer using the codes given.

Codes:

[UPSC-1998]

130. The four railway junctions shown by numerals 1, 2, 3 and 4 on the


rough outline map of Gujarat are, respectively
(a)Palanpur, Mahesana, Ahmedabad and Vadodara.

(b)Mahesana, Surendranagar, Rajkot and Junagarh.

(c)Palanpur, Kanda, Bhuj and Okla.

(d)Ahmedabad, Vadodara, Bhavnagar and Broach.

[UPSC-1998]

131.Which one of the following is the correct sequence of the states


(labelled 1, 2, 3 and 4) of India shown on the map in descending order in
terms of their available ground water resources for irrigation?
(a)3, 4, 1, 2

(b)3, 4, 2, 1

(c)4, 3, 1, 2

(d)4, 3, 2, 1

[UPSC-1998]

132.Among the Indian states shown and labelled 1, 2, 3 and 4 in the rough
outline map given, the correct sequence ofdescending order of percent of
scheduled tribe population to their total population is
(a)1, 3, 2, 4.

(b)3, 1, 2, 4.

(c)3, 1, 4, 2.

(d)1, 3, 4, 2.

[UPSC-1998]

133.Match the following research institutes A, B, C and D with their


respective location labelled as 1 to 6 in the given rough outline map.
(a)Central Drug Research Institute

(b)National Atlas and Thematic Mapping Organization

(c)National Institute of Ocean Technology

(d)Temperate Forest Research Centre

Codes:

[UPSC-1998]

134. Match rivers labelled A, B, C and D the given map with their names
given in the list and select the correct answer using the codes given.
Codes:

[UPSC-1998]

135.Match the cities labelled as A, B, C and D in the given map with the
names of cities and select the correct answer using the codes given.

Name of cities

1.Darwin

2.Kuala

3.Lagos

4.Nairobi

5.Singapore
Codes:

[UPSC-1999]

136.Which one of the countries labelled as 1, 2, 3 and 4 on the given may


was granted membershipof G-15 in its VII Summit at Kuala Lumpur?

(a)1
(b)2

(c)3

(d)4

[UPSC-1999]

137.The physical regions marked as 1, 2, 3 and 4 on the given map are,


respectively

(a)Andes, Brazilian Shield, Guyana Highlands and Amazon Basin.

(b)Andes, Guyana Highlands, Brazilian Shield and Amazon Basin.

(c)Amazon Basin, Guyana Highlands, Brazilian Shield and Andes.

(d)Guyana Highlands, Brazilian Shield, Andes and Amazon Basin.

[UPSC-1999]

138.The given figure shows a portion of southern India. The proposed site
(Koodankulam) for the construction of two 1000 MW nuclear power
plants has been labelled in the map as
(a)A.

(b)B.

(c)C.

(d)D.

[UPSC-1999]

139.In which one of the following areas in the given map was there a recent
discovery of copper deposits by the Atomic Minerals Division of
Department Atomic Energy?
(a)1

(b)2

(c)3

(d)4

[UPSC-1999]

140.Match the areas shown as A, B, C and D on the given map showing with
the largest religious minorities. Select the correct answer using the codes
given.

Largest religious minorities

1.Buddhist

2.Christians

3.Jains
4.Muslims

5.Sikhs

Codes:

[UPSC-1999]

141.In the rough outline map of a part of Jammu and Kashmir shown in the
figure, places marked A, B, C and D represent, respectively

(a)Anantnag, Baramula, Srinagar and Kargil.

(b)Baramula, Srinagar, Kargil and Anantnag.

(c)Baramula, Srinagar, Anantnag and Kargil.

(d)Srinagar, Baramula, Kargil and Anantnag.


[UPSC-1999]

142.In the given map, which one of the following pairs of ocean currents is
shown?

(a)Benguela and Falkland

(b)Canary and Humboldt

(c)Agulhas and Guinea

(d)Benguela and Guinea

[UPSC-1999]

143.At which one of the following positions shown in the diagram will the
height of the ocean tide be maximum?
(a)M 1

(b)M2

(c)M3

(d)M4

[UPSC-1999]

144.Which one of the areas marked as A, B, C and D in the given figure of


the cyclone witnesses heavy torrential short-duration rainfall
accompanied by thunderstorms?

(a)A
(b)B

(c)C

(d)D

[UPSC-1999]

145.Match the cities labelled as 1, 2, 3, 4 and 5 in the given map with the
name of the institutes located in these and select the correct answer using
the codes given.

Name of institutes

A.Central Marine Fisheries Research Institute

B.Central Sheep Breeding Farm

C.National Dairy Research Institute

D.National Institute of Agricultural Extension Management


Codes:

[UPSC-2000]

146.Match the location of ports labelled as A, B, C and D in the given map


with the names of those ports and select the correct answer using the
codes given.

Names of the port

1.Kakinada

2.Karwar

3.Mangalore
4.Tuticorin

5.Veraval

Codes:

[UPSC-2000]

147.Match the drainage basins a labelled as A, B, C and D with the names


listed below and select the correct answer using the codes given.

Name of drainage basins

1.Ganga-Brahmaputra

2.Indus

3.Parana

4.Zambezi
Codes:

[UPSC-2000]

148.Match the international events listed below with their respective places
labelled as 1, 2, 3, 4 and 5 in the given map and select the correct answer
using the codes given.

International Events
A.Venue of Commonwealth Conference held in 1999

B.Venue of World Trade Organization meeting held in 1999

C.Place of Israel-Syria peace talks held in January 2000

D.Place of military action by Russian troops in January 2000

Codes:

[UPSC-2000]

149.At which one of the cities labelled as A, B, C and D on the even map of
Europe was the historic treaty between NATO and Warsaw pact countries
signed in 1998?

(a)A

(b)B
(c)C

(d)D

[UPSC-2000]

150.The given map shows location of airports labelled as 1, 2, 3, 4 and 5.


What is the correct sequence of the airports in which the hijacked Indian
Airlines plane IC-814 landed after its initial take off from Kathmandu in
December 1999?

(a)3, 1, 2, 4

(b)2, 4, 1, 3

(c)5, 4, 2, 3

(d)5, 1, 3, 2

[UPSC-2000]
In the above map, the black marks show the distribution of

(a)asbestos.

(b)gypsum.

(c)limestone.

(d)mica.

[UPSC-2001]
In the shaded area of the above map, the mean temperature for the month
July varies between

(a)22.5°C and 25.0°C

(b)25.0°C and 27.5°C.

(c)27.5°C and 30.0°C.

(d)30.0°C and 32.5°C.

[UPSC-2001]
The above map is the Union Territory of

(a)Chandigarh.

(b)Daman and Diu.

(c)Dadra and Nagar Haveli.

(d)Pondicherry.

[UPSC-2001]

154.The given map shows four towns of central Asian region marked as 1, 2,
3 and 4. Identify these from the following and select the correct answer
using the codes given.

Towns

A.Bishkek

B.Ashkhabad

C.Tashkent

D.Dushanbe
Codes:

[UPSC-2001]

155.The annual agricultural production of a product for the period 1991-92


to 1998-99 is shown in the figure below. Which one of the following is
the product in question?

(a)Pulses

(b)Wheat

(c)Oilseeds
(d)Rice

[UPSC-2001]

156.In the map given below, four islands of Indian Ocean region listed
below are marked as 1, 2, 3 and 4. Match them and select the correct
answer from the codes given.

A.Seychelles

B.Chagos

C.Mauritius

D.Socotra

Codes:
[UPSC-2002]

ANSWER KEYS

 
CHAPTER 1: GEOMORPHOLOGY

A stream is confined within a definite channel incised into the surface of the
land, with a distinctive pattern and cross section, which varies from source to
mouth and changes its form as the drainage system develops. In order to
introduce some definitiveness, a distinctive, if somewhat obvious,
terminology has been developed, mainly by civil engineers. The depth and
width of the water measured from bank to bank (the margin of the channel)
are not always easy to specify, since unless the stream has been regularized
for flood control, navigation or power production, the cross section is rarely
a distinct rectangle; a mean depth is commonly used. Their relationship is
expressed in ratio form, an expression derived from depth: width, as 1:50
where the width is 50m (150 ft) and the mean depth is 1 m (3 ft). Sometimes
the banks are clear-cut and well-defined, at other places, uneven beds of
sand and shingle may slope gradually from the water's edge, which itself
fluctuates. The term wetted perimeter denotes the length from bank to bank
of the section in contact with the stream, and the crosssectional area is, as its
name would imply, the area of a cross section of the water at any particular
point. Another relationship, important because it is a guide to the amount of
friction between the water and the channel, and therefore an indication of
energy loss, is the hydraulic radius, the cross-sectional area divided by the
wetted perimeter. A low hydraulic radius, where a shallow stream occupies a
very wide channel, is the least efficient in energy terms.

The base flow of a river is that portion of its volume which is contributed
by groundwater inflow; this fluctuates during the year with the general water
table but will not occupy the entire channel. When surface or overland flow
increases after rain or snowmelt, the depth will of course increase. When it
just occupies the entire channel, it is at bank full stage, beyond which it may
rise to the flood (or over barzk) stage, when it overspills the banks.

Read the above passage carefully and choose the correct answers to the
following questions.

1.Under what circumstances, will the shape of a river channel will


geometric?

(a)When the river takes a new course during a flood

(b)When the river rejuvenates

(c)When the river water is used for economic activities

(d)When the river is at a bankfull stage

[UGC D-2005]

2.Depth: width ratio of a river channel determines

(a)shape of a channel.

(b)form of a channel.

(c)structure of a channel.

(d)all the above.

[UGC D-2005]

3.Wetted perimeter of a channel can be measured from

(a)cross-sectional area and width of a channel.


(b)hydraulic radius and discharge.

(c)discharge and velocity.

(d)cross-sectional area and hydraulic radius.

[UGC D-2005]

4.How does the base flow change'?

(a)When the river is at bank full stage

(b)When surface flow increases

(c)When flood occurs

(d)When water table changes

[UGC D-2005]

5.A channel flow is efficient when

(a)the river is with shallow depth.

(b)the discharge is moderate.

(c)the velocity is high.

(d)both the velocity and discharge are high.

[UGC D-2005]

Wegener's drift hypothesis also provided an alternate explanation for the


formation of mountains (orogenesis). The theory being discussed during his
time was the 'Contraction theory' which suggested that the planet was once a
molten ball, and in the process of cooling, the surface cracked and folded up
on itself. The big problem with this idea was that all mountain ranges should
be approximately the same age, and this was known not to be true.
Wegener's explanation was that as the continents moved, the leading edge of
the continent would encounter resistance and thus compress and fold
upwards forming mountains near the leading edges of the drifting continents.
The Sierra Nevada mountains on the Pacific coast of North America and the
Andes on the coast of South America were cited. Wegener also suggested
that India drifted northwards into the Asian continent, thus forming the
Himalayas.

Wegener eventually proposed a mechanism for continental drift that


focused on his assertion that the rotation of the Earth created a centrifugal
force towards the Equator. He believed that Pangaea originated near the
South Pole and that the centrifugal force of the planet caused the
protocontinent to break apart and the resultant continents to drift towards the
Equator. He called this the `polefleeing force'. This idea was quickly rejected
by the scientific community primarily because the actual forces generated by
the rotation of the Earth were calculated to be insufficient to move
continents. Wegener also tried to explain the westward drift of the USA by
invoking the gravitational forces of the Sun and the Moon, this idea was also
quickly rejected. Wegener's inability to provide an adequate explanation of
the forces responsible for continental drift and the prevailing belief that the
Earth was solid and immovable resulted in the scientific dismissal of his
theories.

6.According to the author, Pangaea is

(a)a continent from which all the other continents were formed.
(b)a type of continent which showed the properties shown by the
continents today.

(c)one of the continents today.

(d)a continent covered with ice.

7.The contraction theory assumed that

(a)Earth was fluid but immovable.

(b)Earth was solid and immovable.

(c)Earth was solid but movable.

(d)Earth was fluid and movable.

8.The idea proposed by Wegener was rejected because

(a)it was absurd.

(b)it was changing the status quo.

(c)the evidence was lacking.

(d)he was unable to explain it properly.

9.Which of the following statements holds true about the idea proposed by
Wegener'?

(a)Both the reasons supplied as evidence were true.

(b)The first reason is right, but second is wrong.

(c)First reason is wrong, but second is right.


(d)Both the reasons supplied as evidence were wrong.

The exploration and scientific study of caves is speleology. Although


professional physical and biological scientists carry on investigation,
amateur cavers or `spelunkers' have made many important discoveries. As an
example, in the early 1940s, George Colglazier, a farmer southwest of
Bedford, Indiana, awoke to find his farm pond at the bottom of a deep
collapsed sinkhole. This sinkhole is now the entrance to an extensive cave
system that includes a subterranean navigable stream. Cave habitats are
unique. They are nearly closed, selfcontained ecosystems with simple food
chains and great stability. In total darkness, bacteria synthesize inorganic
elements and produce organic compounds that sustain many types of cave
life, including algae, small invertebrates, amphibians, and fish. In a cave
discovered in 1986, near Movile in south-eastern Romania, cave-adapted
invertebrates were discovered after millions of year of sunless isolation.
Thirty-one of these organisms were previously unknown. Without sunlight,
the ecosystem in Movile is sustained on sulphur-metabolizing bacteria that
synthesize organic matter using energy from oxidation processes. These
chemosynthetic bacteria feed other bacteria and fungi that in turn support
cave animals. The sulphur bacteria produce sulphuric acid compounds that
may prove to be important in the chemical weathering of some caves.

The mystery, intrigue and excitement of cave exploration lie in the variety
of dark passageways, enormous chambers that narrow to tiny crawl spaces,
strange formations and underwater worlds that can be accessed only by cave
diving. Private property owners and amateur adventurers discovered many of
the major caves, a fact that keeps this popular science/sport very much alive.

10.Why are different cave habitats unique'?

(a)Each habitat develops as an open ecosystem.


(b)Each habitat develops in the absence of light.

(c)Each habitat develops independent of each other.

(d)None of these.

11.The writer states that, `thirty one of.........unknown'. This is because

(a)nobody tried to search for these organisms.

(b)they depended on oxidation process for energy.

(c)they are involved in total darkness.

(d)in all probability, these organisms do not exist anywhere else.

Imagine almost a third of Earth's land surface buried beneath ice sheet and
glaciers-most of Canada, the northern Midwest, England and northern
Europe, and many mountain ranges, beneath thousands of meters of ice!
This is how it was at the height of the Pleistocene Epoch of the late
Cenozoic Era. In addition, periglacial regions along the margins of the ice
during the last ice age covered about twice their present areal extent.

The Pleistocene is thought to have begun about 1.65 million years ago and
is one of the

more prolonged cold periods in Earth's history. It features not just one glacial
advance and retreat, but at least 18 expansions of ice over Europe and North
America, each obliterating and confusing the evidence from the one before.
Apparently, glaciations can take about 90,000 years, whereas deglaciation is
rapid, requiring less than about 10,000 years to melt away the accumulation.
The term ice age is applied to any extended period of cold (not a single
brief cold spell). An ice age is a period of generally cold climate that
includes one or more glacial, interrupted by brief warm spells, known as
interglacial. Each glacial and interglacial is given a name that is usually
based on the location where evidence of the episode is prominent, e.g.,
Wisconsinan glacial.

Modern research techniques to understand past climates include the


examination of ancient ratios of oxygen isotopes, depths of coral growth in
the tropics, analysis of ocean and lake sediments worldwide and analysis of
the latest ice cores from Greenland and Antarctica, including the Dome C ice
core that plunges the record back to 7,40,000 years (as of 2004). These
techniques have opened the way for a new chronology and understanding of
past climates and for putting perspective on the present warming trends. The
record is clear and correlates across all proxy methods: present levels of
(CO2) and methane (CH4) are the highest in the past 4,40,000 years. Earth
systems have no experience in dealing with levels this high during this time
span, and these concentrations are increasing.

Glaciologists currently recognize the Illinoisans glacial and Wisconsian


glacial periods, with the Sangamon interglacial between them. These events
span the 3,00,000year period prior to our present Holocene Epoch. The
Illinoisans glacial actually consisted of two glacial occurring during Marine
(oxygen) Isotope Stages (MIS) 6 and 8, as did the Wisconsinan (MIS 2 and
4), which are dated at 10,000 to 35,000 years ago. The oxygen isotope
glacial/interglacial stages are numbered back to MIS 23 at approximately
9,00,000 years ago. To overcome local bias in core records, investigators
correlate oxygen isotope data with other indicators worldwide.

12.During interglacial period, the


(a)glacial advances.

(b)glacial retreats.

(c)ice age ends.

(d)ice age starts.

13.The ice core data includes

(a)ratio of oxygen isotopes.

(b)measuring depths of coral growth.

(c)analysis of ocean sediments.

(d)all of these.

14.MIS number is

(a)used to date the glacial stage.

(b)used to date the interglacial stage.

(c)Both (a) and (b)

(d)Neither (a) nor (b)

15.The Pleistocene period consisted of

(a)less than 18 glacial events.

(b)more than 18 glacial events.

(c)more than 25 glacial events.


(d)more than 13 glacial events.

Important controls on weathering rates are climatic elements precipitation,


temperature and freeze-thaw cycles. There is a relation among climate
(annual precipitation and temperature), physical weathering and chemical
weathering processes. In general, physical weathering dominates in drier,
cooler climates, whereas chemical weathering dominates in wetter, warmer
climates. Extreme dryness reduces weathering rates, as is experienced in
desert climates (low-and mid-latitude hot desert). In the hot, wet, tropical
and equatorial rainforest climates (tropical rainforest), most rocks weather
rapidly, and the weathering extends deep below the surface. Also related to
climate and significant weathering is the position of the water table and
water movement on and within rock structures and the subsurface
environment.

Another control over weathering rates is the geographic orientation of a


slope, whether it faces north, south, east or west. Orientation controls the
slope's exposure to Sun, wind and

precipitation. Slopes facing away from the Sun's rays tend to be cooler,
moister and more vegetated than slopes in direct sunlight. This effect of
orientation is especially noticeable in the middle and higher latitudes.

Vegetation is also a factor in weathering. Although vegetative cover can


protect rock by shielding it from raindrop impact and providing roots to
stabilize soil, it also produces organic acids from the partial decay of organic
matter; these acids contribute to chemical weathering. Plant roots can enter
crevices and break up a rock, exerting enough pressure to drive rock
segments apart, thereby exposing greater surface area to other weathering
processes. You may have observed how tree roots can heave the sections of a
sidewalk or driveway sufficiently to raise and crack the concrete.
The scale at which we analyse weathering processes is important.
Research at micro scale levels reveals greater complexity in the relation of
climate and weathering. At the small scale of actual reaction sites on the
rocks surface, both physical and chemical weathering processes can occur
across varied climate types. Hygroscopic water (a molecule-thin water layer
on soil particles) and capillary water (soil water) activate chemical
weathering processes, even in the driest landscape.

16.In which of the following places will the maximum weathering take
place?

(a)Tropical rainforest

(b)Low-latitude cold desert

(c)Low-latitude hot desert

(d)Mid-latitude hot desert

17.Consider the following statements:

1.Vegetation protects rocks from weathering.

2.Vegetation contributes towards weathering of rock.

(a)Only 1

(b)Only 2

(c)Both 1 and 2

(d)Neither 1 nor 2

18.Geographical orientation of the slope affectsthe least.


(a)lower latitudes

(b)middle latitudes

(c)higher latitudes

(d)Cannot be determined.

19.Why is the scale of analysing weathering important?

1.For deciding on the variables to be kept constant

2.The factor which does not affect at macro level might make a difference
at micro level.

3.The factor which does not affect at micro level might make a difference
at the macro level.

(a)1 and 2

(b)Only 2

(c)Only 3

(d)2 and 3

CHAPTER 2: CLIMATOLOGY

Information consists of new facts, data, ideas and routine communication.


Like people, goods and services, information flows from place to place,
moving from places of production to demand areas. When we think of
demand areas, we should not confuse places of need with places of effective
demand. In the flows of people, movements among big and vigorous centres
are much larger than migrations among the depopulated backwaters of
depressed areas. In the shipment of goods, the volume of trade among the
developed countries far exceeds their trade contacts with the underdeveloped
world. Similarly, information flows primarily among the vigorous idea
centres in a system. Islands of lethargy are bypassed because they are
literally out of the system.

Rich nations trade primarily with each other. Migrations often occur
between clusters of mankind. Those who send the most messages get most
of the return contacts. A tendency exists for flows to run from places of
abundance to areas of effective demand according to the effective pulls
exerted at each destination. As flows continue, a tendency towards
equilibrium conditions seems common. In the physical world, things move
through space so that potential can equalize. Water tries to move under the
force of gravity to the lowest point, and if free to move to sea level, it will do
so.

Similarly, air will move from a high-pressure area to a low-pressure area.


High pressure as a causative agent cannot move air very far in one direction.
We know this from common experience when we try to blow air across a
room with a fan or try to blow out a match at a distance. More air will be
moved by putting the fan in one window blowing outward and opening
another window across the room. Instead of emphasizing push factors, it
makes more sense to think of air being pulled in a certain direction by a zone
of low pressure, in the same way that water is pulled from a mountain top.
Similarly, in human affairs, a pull exerts a directed force with respect to
excess supply somewhere else. Therefore, to explain why something moves
from A to B, it helps to emphasize the pull at B rather than the push at A.The
push at A often is unspecified directionally, whereas a pull at B is significant
not only for A, but for any other potential supply source as well.
20.Read the above passage carefully and choose the correct answers to the
following questions.

What causes flow of information'?

(a)Demand

(b)Use

(c)Quality

(d)Volume

[UGC D-2006]

21.What factors primarily influence exchange of goods among rich nations'?

(a)Abundance of production

(b)Effective demand

(c)Level of economic development

(d)Generation of information

[UGC D-2006]

22.Generation of information depends on

(a)nature of ideas.

(b)strength of ideacentres in a system.

(c)duration of ideas.
(d)effectiveness of ideas.

[UGC D-2006]

23.The flow of information from place of origin to place of destination is


mainly due to

(a)repulsion at the place of origin.

(b)attraction of the place of destination.

(c)distance between place of origin and place destination.

(d)abundance at the place of origin.

[UGC D-2006]

24.Which of the following is the most suitable title for the above passage?

(a)Flow of services

(b)Flow of commodity

(c)Migration of people

(d)Flow of information

[UGC D-2006]

You already know that air pollution in cities contributes to the heat island by
inhibiting the loss of long-wave radiation at night and that pollutants may
have a `cloud seeding' effect that increases precipitation in and downwards
of cities. These influences, however, are not the only ways in which
pollutants influence urban climate.
The blanket of particulates over most large cities significantly reduces the
amount of solar radiation reaching the surface. In some cities, the overall
reduction in the receipt of solar energy is 15 percent or more, whereas short
wavelength ultraviolet is decreased by up to 30 percent. This weakening of
incoming solar energy is variable. During air pollution episodes, the
decrease is much greater than for periods when air quality is good.
Furthermore, particulates are most effective in reducing solar radiation near
the ground when the Sun angle is low. This occurs because the length of the
path through the polluted air increases as the Sun angle drops. Thus, for a
given quantity of particulate matter, solar energy will be reduced by the
largest percentage at high-latitude cities and during the winter.

When compared to surrounding rural areas, the relative humidity in cities


is generally from 2 to 8 percent lower. One reason is that cities are hotter. A
second reason is that

less water vapour is supplied to city air by evaporation from the surface.
Evaporation is reduced in cities because rain water rapidly ruins off,
frequently into subsurface storm sewers. Although relative humidity tends to
be lower in cities, the occurrences of clouds and fogs are greater.

25.According to the passage, the cities are hotter because

(a)of Earth's radiation.

(b)the total solar radiation received is reduced.

(c)relative humidity is less.

(d)All of these.

26.According to the passage, the pollution will reduce the incoming solar
radiation most at
(a)lower latitudes.

(b)higher latitudes.

(c)higher heights.

(d)spring time.

27.Hotter temperature causes relative humidity to drop because

(a)less water is evaporated.

(b)more clouds are formed.

(c)the saturation point increases.

(d)the saturation point falls.

28.The last statement of the passage `although.......... are greater' is

(a)Logical because generally higher relative humidity causes more


occurrence of clouds and fogs.

(b)paradoxical because generally lower relative humidity causes less


clouds and fogs.

(c)paradoxical because generally higher relative humidity causes less


occurrence of clouds and fogs.

(d)logical because generally lower relative humidity causes higher


occurrence of clouds and fog.

A common misconception relating to meteorology is the notion that air with


a high relative humidity must have greater watervapour content than air with
a lower relative humidity. Frequently, this is not the case. To of greenhouse
gases are the basic problems. Those are interrelated: both are caused by
human-produced gases released into atmosphere. They differ, however, in
that there is a general scientific and political consensus subject to
controversy.

illustrate, let us compare a typical January day at International Falls,


Minnesota, to one on the desert near Phoenix, Arizona. On this hypothetical
day, the temperature in International Falls is a cold - 10°C (14°F) and the
relative humidity is 100 percent. The saturated - 10°C (14°F) air has a water
vapour content (mixing ratio) of 2 g/kg. In contrast, the desert air at Phoenix
on this January day is a warm 25°C (77°F); air has a saturation mixing ratio
of 20 g/kg. Therefore, with a relative humidity of 20 percent, the air at
Phoenix has a water vapour content of 4 g/kg (20 g x 20 percent).
Consequently, the `dry' air at Phoenix actually contains twice the water
vapour as the `wet' air at International Falls.

This should make clear why places that are very cold are also very dry.
The low water vapour content of frigid air (even when saturated) helps to
explain why many Arctic areas receive only meagre amounts of precipitation
and are sometimes referred to as `polar deserts'. This also helps us
understand why people frequently experience dry skin and chapped lips
during the winter months. The water vapour content of cold air is low, even
when compared to some hot, and regions

29.According to the writer's argument, the air blowing through polar deserts
will have

(a)higher water vapour content.

(b)lower relative humidity.


(c)higher relative humidity.

(d)None of these

30.The higher water vapour content at lower relative humidity happens due
to

(a)difference in temperature.

(b)difference in saturation point.

(c)difference in speed of air.

(d)All of these

Two potential global environmental problems may pose threats to the health
and well-being of humans. Abnormally high incidences of ultraviolet
radiation on the Earth's surface due to depletion of stratospheric ozone, and
climate changes due to artificial introduction

Stratospheric ozone absorbs UV radiation and shields the Earth's surface


from large amount of UV radiations reaching the Earth's surface. Increased
UV radiation exposure can result in increases in skin cancer and cataracts
among humans, the inhibition of photosynthetic processes in certain plants,
the disruption of insect activity and decrease in productivity of ocean
ecosystem. Disruptions of the ozone layer would also cause the stratosphere
to cool, potentially affecting climate patterns of Earth.

Many trace atmospheric gases such as carbon dioxide, methane, water


vapour and man-made CFCs are relatively transparent to sunlight but opaque
to infrared radiation, this results in the heating of the Earth's surface and
lower atmosphere-the greenhouse effect. Without a natural greenhouse
effect, the surface of Earth would be a frozen mass, but too much
greenhouse effect could also cause abnormal global warming.

31.What are CFC's? What are their uses?

1.Chlorofluorocarbon

2.Chlorine fluorocarbon

3.Coolants in refrigeration systems

4.Coolant in air conditioners

5.Solvents to clean electronic components

6.Blowing agents in the production of plastic foams

(a)1, 3, 4, 5 and 6

(b)2, 3, 4, 5 and 6

(c)1, 3 and 4

(d)2, 3 and 4

[UGC D-2009]

32.How do CFCs damage the ozone layer?

(a)Chlorine atom reacts with ozone to create chlorine monoxide and


oxygen.

(b)Chlorine monoxide reacts with oxygen atom to free chlorine atom and
form oxygen.
(c)Both (a) and (b)

(d)Neither (a) nor (b)

[UGC D-2009]

33.What are the impacts of ozone depletion'?

(a)Skin cancer

(b)Cataracts

(c)Inhabitation of photosynthesis

(d)All of these

[UGC D-2009]

34.What are greenhouse gases'?

(a)Gases which trap solar radiation.

(b)Gases which trap Earth's radiation.

(c)Gases which are harmful in the Earth's atmosphere.

(d)Gases which cause depletion of ozone layer.

[UGC D-2009]

35.What are the sources of greenhouse gases?

(a)Natural processes

(b)Man-made chemicals
(c)Pollution

(d)All of these

[UGC D-2009]

Serendipity is defined by Nobel laureate Irving Langmuir as the art of


profiting from unexpected occurrence. In other words, if you are observing
something and the entirely unexpected happens, and if you see in this
accident a new and meaningful discovery, then you have experienced
serendipity. Most non-scientists, some scientists and, alas, many teachers are
not aware that many of the great discoveries in science were serendipitous.

An excellent example of serendipity in science occurred when Tor


Bergeron, the great Swedish meteorologist, discovered the importance of ice
crystals in the initiation of precipitation in super-cooled clouds. Bergeron's
discovery occurred when he spent several weeks at a health resort at an
altitude of 430 metres (1,400 feet) on a hill near Oslo. During his stay,
Bergeron noted that this hill was often `fogged in' by a layer of super-cooled
clouds. As he walked along a narrow road in the fir forest along the hillside,
he noticed that the `fog' did not enter the `road tunnel' at temperatures below
- 5°C.

Bergeron immediately concluded that at temperatures below about - 5°C,


the branches

of the firs acted as freezing nuclei upon which some of the super-cooled
droplets crystallized. Once the ice crystals developed, they grew rapidly at
the expense of the remaining water droplets. The result was the growth of ice
crystals (rime) on the branches of the firs accompanied.
From this experience, Bergeron realized that if ice crystals somehow were
to appear in the midst of a cloud of super-cooled droplets, they would grow
rapidly as water molecules diffused towards them from the evaporating
cloud droplets. This rapid growth forms snow crystals that, depending on the
air temperature beneath the cloud, fall to the ground as snow or rain.
Bergeron had thus discovered in one way that minuscule cloud droplets can
grow large enough to fall as precipitation.

Serendipity influences the entire realm of science. Can we conclude that


anyone who makes observations will necessarily make a major discovery?
Not at all. A perceptive and inquiring mind is required, a mind that has been
searching for order in a labyrinth of facts. As Langmuir said, the unexpected
occurrence is not enough: you must know how to profit from it. Louis
Pasteur observed that `in the field of observation, chance favours only the
prepared mind'. The discoverer of vitamin C, Nobel laureate Albert Szent-
Gyorgyi, remarked that discoveries are made by those who `see what
everybody else has seen, and think what nobody else has thought'.
Serendipity is at the heart of science itself.

36.The fog did not enter the road tunnel at temperature below - 5° C because

(a)the fog was converted into rime.

(b)super-cooled droplets dissolved into air.

(c)the rapid growth of ice crystals caused the relative humidity of the
wind to increase.

(d)All of these

37.The great discoveries in science were serendipitous because


(a)they were not a result of hard work.

(b)the people making these discoveries were not trying to make these
discoveries.

(c)people making the discoveries were lazy.

(d)they were made without scientific enquiry.

38.The serendipity is heart of science itself because

(a)it allows people to think what nobody has thought.

(b)it is a counterintuitive thought process.

(c)many great discoveries of science have been serendipitous.

(d)it allows people to see what others are not able to see.

39.Super-cooled clouds are

(a)clouds in which the temperature of water is low but not below the
freezing point.

(b)clouds in which water has changed into ice crystals.

(c)clouds in which water vapours have frozen without becoming liquid.

(d)clouds in which the water does not freeze even when temperature falls
below the freezing point.

In 1960, Edward Lorenz, an MIT scientist, rocked the scientific world with
the statement that a butterfly flapping its wings in Brazil might produce a
tornado in Texas. He used this strange example to suggest that the
interaction of orderly and deterministic systems may produce chaotic and
unpredictable results.

For example, ice has a rigid internal structure, forced by bonding between
water molecules. This structure dictates that all ice crystal are six sided, yet
no two ice crystals, despite this similarity, are identical. Beneath this chaos
of design exists an order dictated by physical principles.

A major shift in our real-world view occurred with the advent of Chaos
Theory, a revolution in Science that considers the non-linear and
unpredictable behaviour of

operational systems. This theory suggests that the scientific method must
consider the coexistence of disorder and order, randomness and pattern, and
symmetry and chaos in natural systems-the science of complexity and
complex systems.

Consider the weather, mathematical models and numerical equations,


temperature and pressure patterns. Yet weather systems are sensitive to very
small fluctuations in any of those ingredients. Therefore, it is difficult to
exactly predict how a weather system will develop, what track it will follow
or how severe it might be. Two similar chaotic weather systems might
produce a similar result, although it is not possible to say exactly what the
output will be. This understanding of the role of chaos is helping scientists to
improve forecasting of weather phenomena. In a dynamic weather system,
chaos is the rule, just as it is in chemical and biological systems.

Chaos theory is useful in studying all of Earth's physical environments.


For example a river flows in branched channels over the floodplain in a
pattern to conserve energy. The channels constantly shift in a randomness
that is irregular and difficult to predict.
This may seem strange, but the nature of order we observe is the result of
chaos and the almost infinite interaction among physical elements! Chaos
theory is a new dimension of the scientific method and physical geography.

40.Edward Lorenz shocked the scientific world

(a)because he proved something that was considered to be impossible.

(b)because he showed that orderly systems can produce chaotic results.

(c)because he showed the relationship between variables which were not


connected.

(d)None of these

41.In the example of weather system, chaos theory is applicable because

(a)we cannot predict the consequence of minute changes in the


underlying variables.

(b)everything else remaining the same, a minute change will never


produce the same result twice.

(c)the system does not follow cause and effect relationship.

(d)None of these

42.According to chaos theory,

1.order and chaos exist side by side.

2.pattern and randomness exist side by side.

(a)Only 1
(b)Only 2

(c)Both 1 and 2

(d)Neither 1 nor 2

43.The fact that no two ice crystals are the same shows that

(a)the nature cannot be converted into variables and studied.

(b)the chaos is an exception rather than a rule.

(c)the chaos theory is a new dimension of the scientific method.

(d)the complexity of the natural systems can make predictions almost


impossible.

Mid-latitude cyclones are low-pressure systems with diameters often


exceeding 1,000 kilometres (600 miles) that travel from West to East across
the planet. Lasting from a few days to more than a week, these weather
systems have a counter clockwise circulation pattern with a flow inwards
towards their centres. Most mild-latitude cyclones have a cold front and a
warm front extending from the central area of low pressure. Surface
convergence and upwards flow initiate cloud development that frequently
produces precipitation.

As early as the 1800s, it was known that cyclones were the bearers of
precipitation and severe weather. Thus, the barometer was established as the
main tool in `forecasting' dayto-day weather changes. However, this early
method of weather prediction largely ignored the role of air-mass
interactions in the formation of these weather systems. Consequently,
it was not possible to determine the conditions under which cyclone
development was favourable.

The first encompassing model to consider the development and


intensification of a mid-latitude cyclone was constructed by a group of
Norwegian scientists during World War I.The Norwegians were then cut off
from weather reports. Especially those from the Atlantic. To counter this
deficiency, a closely spaced network of weather stations was established
throughout the country. Using this network, several Norwegian-trained
meteorologists made great advances in broadening our understanding of the
weather and in particular our understanding of the middle latitude cyclone.
Included in this group were Vihelm Bjerknes (pronounced Bee-YURKness),
his son Jacob Bjerknes, Jacob's fellow student Halvor Solberg and Swedish
Meteorologist Tor Bergeron. In 1921, the work of these scientists resulted in
a publication outlining a compelling model of how mid-latitude cyclones
progress through stages of birth, growth and decay. These insights, which
marked a turning point in atmospheric science, became known as the polar
front theory-also referred to as the Norwegian cyclone model. Even without
the benefits of upperair charts, these skilled meteorologists presented a
working model that remains remarkably accurate to this day.

In the Norwegian cyclone model, middle latitude cyclones develop in


conjunction with the polar front. Recall that the polar front separates cold
polar air from warm subtropical air. During cool months, the polar front is
generally well-defined and forms a nearly continuous band around Earth that
can be recognized on upperair charts. At the surface, this frontal zone is
often broken into distinct segments. These frontal segments are separated by
regions of more gradual temperature change. It is along frontal zones where
cold, Equator wards moving air collides with warm, pole wards moving air
that most middle latitude cyclones form.
44.What does the passage polar front theory explain'?

(a)The difference between development of cold and warm fronts

(b)How cyclones help in the formation of polar fronts

(c)The development of middle latitude cyclones

(d)None of these

45.Polar front theory was important in weather forecasting because

(a)the theory was formed without accounting for upper air movements.

(b)it was made by Norwegians.

(c)it explained the formation of polar fronts.

(d)it explained the role of air-mass interaction in weather formation.

46.Polar front

(a)can be recognized on upperair charts in summer.

(b)is nearly a continuous band around Earth during winter.

(c)cannot be seen on upper air charts.

(d)is not generally well-defined.

47.Which of the following statements holds true in the light of the passage'?

(a)Cyclonic developments are always bringer of bad weather.

(b)Polar front does not separate cold and warm air.


(c)Cyclonic developments can be favourable from the point of view of
weather.

(d)None of these

Let us examine how permafrost behaves. The active layer is the zone of
seasonally frozen ground that exists between the subsurface permafrost layer
and the ground surface.

The active layer is subjected to consistent daily and seasonal freeze-thaw


cycles. This cyclic melting of the active layer affects as little as 10-cm (4
inches) depth in the North (Ellesmere Island, 78°N), up to 2 m (6.6 ft) in the
southern margins (55°N) of the periglacial region and 15 m (50 ft) in the

Alpine permafrost of the Colorado Rockies (40°N).

The depth and thickness of the active layer and permafrost zone change
slowly in response to climatic change. Higher temperatures degrade (reduce)
permafrost and increase the thickness of the active layer; lower temperatures
gradually aggrade (increase) permafrost depth and reduce active layer
thickness. Although somewhat sluggish in response, the active layer is a
dynamic open system driven by energy gains and losses in the subsurface
environment. As you might expect, most permafrost exists in disequilibrium
with environmental conditions and therefore actively adjusts to inconstant
climatic conditions.

The present trend in concert with global warming is for a deepening of the
active layer at high latitudes. Across Canada and Siberia, this warming is
causing a large release of carbon from peat-rich thawed ground that, in turn,
influences the global greenhouse-a real-time positive feedback mechanism.
The thaw in the active layer in the Alaskan tundra has shortened the number
of days that oil exploration equipment can venture out from 200 to only 100
days a year. The weight of the trucks requires a more frozen solid surface to
operate.

A talik is unfrozen ground that may occur above, below or within a body
of discontinuous permafrost or beneath a water body in the continuous
region. Taliks occur beneath deep lakes and may extend to bedrock and
noncryotic soil beneath large, deep lakes. Taliks form connections between
the active layer and groundwater, whereas in continuous permafrost,
groundwater is essentially cut off from surface water. In this way, permafrost
disrupts aquifers and taliks, leadings to water supply problems.

48.Higher temperatures

1.reduce permafrost depth.

2.increase the active layer.

3.increase permafrost depth.

4.reduce the active layer.

(a)1 and 2

(b)1 and 4

(c)2 and 3

(d)3 and 4

49.Active layer is a

(a)dynamic closed system.

(b)static closed system.


(c)dynamic open system.

(d)static open system.

50.Global warming is causing

1.large quantities of carbon to be released.

2.permafrost layer to reduce.

3.permafrost layer to increase.

(a)Only 1

(b)1 and 2

(c)1 and 3

(d)Only 2

CHAPTER 3: OCEANOGRAPHY AND BIOGEOGRAPHY

Several lines of evidence support the theory. DNA analysis finds that the
earliest sponges likely first emerged at least 700 million years ago, when the
oceans contained little oxygen. Between 700 and 600 million years ago, the
oceans gradually became more oxygenated, meaning more enriched with
oxygen. Fossils of animals dating to 650 million years ago have been found.

Then there is the way that sponges feed. These multicellular organisms
consist of pores and channels that allow nutrient-containing water to
circulate through them. As sponges feed, they filter out tiny particles of
organic matter from the water. The particles millions of years ago would
have included dead microbial matter, which rots and consumes oxygen as it
does so. Sponges therefore helped clean water of this material. Without all of
the rotting going on, the water would have experienced increased oxygen
levels, the researchers suggest.

More oxygen in the water then set the stage for even more complex life
forms to emerge, such as the first predatory animals with guts that started to
eat one another, marking the beginning of a modem marine ecosystem, with
the type of food webs we are familiar with today.

Source:http://news.discovery.com/earth/ oceans/sponges-likely-paved-the-
way-for-alllife-on-earth-140309.htm

51.According to the writer, the oxygenation of deep oceans happened


because of

(a)the increased oxygen available.

(b)the decreased consumption of available carbon dioxide.

(c)the increased available carbon dioxide.

(d)the decreased consumption of available oxygen.

52.The evidence of DNA analysis is important in this passage because

(a)it proves the writer's hypothesis.

(b)it lends supports to the writer's hypothesis.

(c)it discourages counter argument.

(d)it gives evidence to support the counter argument.


Corals are mostly sessile, colonial polyps, although solitary species exist.
Coral polyps have tentacles, a mouth, a gastro vascular cavity and are
connected to one another by a common tissue called coenosarc. The outer,
cellular layer of a polyp's tentacles is highly loaded with nematocysts; cells
which can fire stinging barbs filled with neurotoxins. This allows corals and
anemones to paralyze prey, ranging from small plankton to even small fish,
depending on the size of the polyp. The polyp gut is a simple sac, and many
coral species actually have guts which are connected together, allowing them
to share nutrients. The gut is also the location where its gonads are located.
Along the gut mesenteries, ovaries and testes will produce oocytes and
spermatocytes, which are released during specific times of the year.

Many coral species build skeletons, which provide a refuge against


predators. They also help newly settled coral polyps to attach themselves
onto rocky substrate. These skeletons, which are secreted by the underside of
the polyp skin, eventually created the colourful reefs which we know today.
The basal plate of a polyp is where its skeleton starts, which is made from
calcium carbonate (or aragonite). The molecules which make up the skeleton
are secreted by the calicoblastic layer or epithelium; the ectoderm or skin
lining the lower part of the polyp. This process consumes significant
amounts of energy, and the rate at which it occurs is quite slow. Some stony
corals may grow about 5 mm (0.2 inches) each month, while others such as
deep water corals may grow much slower. This so-called process of
calcification is very energy-demanding; this energy is provided by algae
residing in coral tissue. A group of algae from the genus ,Symbiodinium has
formed a partnership with corals; these are called the zooxanthellae. They
produce sugars by using the Sun's energy, just like higher plants do. We call
this process photosynthesis, and it provides up to 95% of the energy corals
need.
Source: http://www.coralscience.org/main/ articles/biochemistr-2/how-reefs-
growy

53.Which of the following statements holds true in the light of the passage'?

(a)Small plankton and small fishes provide the basic food to coral polyp.

(b)Oocytes provide basic food for the coral polyps

(c)Zooxanthellae provide the basic food for the coral polyp.

(d)Symbiodinium provide the basic food for the coral polyp.

54.Growth rate of 5mm per month is

(a)normal.

(b)minimum.

(c)average.

(d)maximum.

55.The skeletons are produced by the coral polyp for

(a)reproduction purpose.

(b)making feeding easier.

(c)the algae to grow.

(d)defending themselves.

56.The photosynthesis can be defined as a


(a)process by which coral polyps obtain food.

(b)process by which sunlight is converted into sugar.

(c)process by which corals are formed.

(d)All of these

A quick glance at a world precipitation map shows that most tropical rain
falls in the Northern Hemisphere. The Palmyra Atoll, at 6°N, gets 175 inches
of rain a year, while an equal distance on the opposite side of the equator
gets only 45 inches.

Scientists long believed that this was a quirk of the Earth's geometry that
the ocean basins tilting diagonally while the planet spins pushed tropical rain
bands north of the Equator. But a new study shows that the pattern arises
from ocean currents originating from the poles, thousands of miles away.

The findings explain a fundamental feature of the planet's climate and


show that icy waters affect seasonal rains that are crucial for growing crops
in such places as Africa's Sahel region and southern India.

In general, hotter places are wetter because hot air rises and moisture
precipitates out. It rains more in the Northern Hemisphere because it is
warmer. The question is: What makes the Northern Hemisphere warmer? It
is the ocean's circulation.

Frierson and his co-authors first used detailed measurements from


NASA's Clouds and Earth's Radiant Energy System, or CERES, satellites to
show that sunlight actually provides more heat to the Southern Hemisphere
and so, by atmospheric radiation alone, the Southern Hemisphere should be
the soggier one.
After using other observations to calculate the ocean heat transport, the
authors next used computer models to show the key role of the huge
conveyor belt current that sinks near Greenland, travels along the ocean
bottom to Antarctica, and then rises and flows north along the surface.
Eliminating this current flips the tropical rain bands to the south.

The reason is that as the water moved north over many decades, it
gradually heated up, carrying some 400 trillion (that's four with 14 zeroes
after it) Watts of power across the Equator.

57.The above passage shows

(a)how important high latitudes are for other parts of the world.

(b)how important middle latitudes are for other parts of the world.

(c)how important low latitudes are for other parts of the world.

(d)None of these

58.The research presented in the passage assumes that

(a)Northern Hemisphere is warmer than the Southern Hemisphere.

(b)the temperature difference is responsible for the difference in


precipitation.

(c)the huge conveyor belt current travels to Antarctica.

(d)that the water moving towards North takes many years.

59.According to the passage, more solar radiation is received in

(a)Southern Hemisphere.
(b)Northern Hemisphere.

(c)higher latitudes.

(d)lower latitudes.

60.Which of the following statement may be true if the claims made in the
passage are true, but will not be true if the slanting ocean basins are
responsible for the asymmetry in tropical rainfalls?

(a)A likelihood of pole reversal increases

(b)The temperature balance cannot change in the future.

(c)The pattern of the solar radiation can change in the future.

(d)Northern Hemisphere can become colder than Southern Hemisphere in


the future.

61.The scientists accept slanting ocean basins as the reason for the
asymmetry in tropical rainfall. The reasons presented in the passage

(a)support the slanting ocean basins theory.

(b)are more complicated than provided by the slanting ocean basins


theory.

(c)are more simpler than provided by the slanting ocean basins theory.

(d)are same as those provided by the slanting ocean basins theory.

Several distinctive forms of biogeography may be recognized in the


contemporary literature. Prominent among them is a focus on distribution
patterns, often of plant taxa (phytogeography), and their evolution over
geological time. Few would argue that, ecologically, the rainforests of Africa
and South America are functionally similar, but the taxonomic identities of
the two regions are markedly distinct so that taxa in similar habitats are
different because they have different histories. The sub-discipline of
vicariance biogeography arose out of the recognition that history, by way of
continental drift and other large-scale geological and climatic changes, has
played the dominant role in determining taxonomic distribution patterns at
the global scale. Cladistic biogeography goes one stage further in
recognizing that the precise evolutionary relationships between taxa are a
direct function of the particular sequence of events in their respective
geographical areas of distribution, i. e., the relationships between
geographical areas can be reconstructed from knowledge of the taxa
occurring there.

Ecological ideas, in particular the development of the ecosystem concept,


have proved central, especially in the contemporary geographers (as opposed
to biologists) approach to biogeography. This spawned the consideration of
major vegetation formations, or biomes, which could be seen as an
expression of the environmental relationships at the global scale. Another
expression of ecological ideas, in this case the branch of population ecology,
arose out of Mac Arthur and Wilsons's (1967) ideas on island biogeography,
which viewed species complements on islands, microcosms of larger scale
ecosystems, as the product of an equilibrium between colonizing species and
those becoming extinct. The potential for this idea to help resolve the
growing conservation crisis was soon realized and, although direct
application of the equilibrium approach has proved fruitless, it encouraged
critical thinking as to the mechanisms underlying accelerated human-
induced extinction. Indeed, biogeography exhibits a strong degree
of involvement in that area of science dealing more broadly with human-
environment relationships. Biogeographies, particularly those with training
as geographers, have also made significant contributions in the field of
quaternary palaeoecology, where a dynamic approach to the rather more
static large-scale biome studies has been fostered.

Read the above passage carefully and answer the following questions.

62.Which of the following factors are responsible for taxonomic patterns of


biomass?

1.Geological factors

2.Climate

(a)Only 1

(b)Only 2

(c)Both 1 and 2

(d)Neither 1 nor 2

[UGC D-2005]

63.What did Mac Arthur and Wilson tried to explain in the book Island
Biogeography'?

(a)The ecology of the island ecosystem

(b)The equilibrium between old and new species

(c)The relationship between extinction of species and human colonization


(d)The relationship between species becoming extinct and nature

[UGC D-2005]

64.How does biogeography assist in solving contemporary environmental


problems'?

(a)It helps understand the environmental systems in the form of


relationships.

(b)It helps to understand human-environment relationship.

(c)Both (a) and (b)

(d)Neither (a) nor (b)

[UGC D-2005]

65.How is continental drift related to biogeography?

(a)Continental drift gave birth to the study of biogeography.

(b)Continental drift affected the distribution of plant life.

(c)Continental drift caused dinosaurs to become extinct.

(d)Continental drift caused North and South America to separate from


Africa and Europe.

[UGC D-2005]

The impact of people on the physical environment in the 20th century has
been greater than in the previous 3 billion years. Modernization and
industrialization, especially the development of nuclear energy, have
provided the means of drastically changing the biosphere. Unfortunately,
political institutions have not developed or evolved at the same pace as our
ability to degrade the environment; in essence we have outmoded political
institutions trying to deal with environmental problems.

Environmental degradation is an international phenomenon. Many


internationally famous structures, like the Colosseum in Rome, the
Parthenon in Greece and the Lincoln Memorial in Washington, are falling
apart because of deteriorating environmental conditions. These structures are
slowly being eaten away by acid pollution in the air or shaken to pieces by
the rumble of heavy trucks and vehicular traffic. Even more fragile than
these human-made structures are the Earth's natural system. Throughout the
world, species of animals are becoming extinct, oceans are being polluted,
and the air we breathe is becoming contaminated. The pollutants,
particularly those in the air and water, do not respect international
boundaries; it will take an international effort to solve the problem.

A strange dichotomy exists between the developing and the developed


nations in this regard. Most of the developing nations do not want any form
of international control on pollution: they feel it will interfere with their
economic and industrial development. Pollution control costs money and,
with a lack of capital anyway, the developing nations do not want to invest
their scare capital in pollution abatement. On the other hand, the developed
nations like the USA and the Soviet Union support some form of
international regulation of pollution activities.

Policies to deal with international pollution can be set at two general


levels, external and internal. External policies are related to areas outside
national boundaries; whether on the regional or global scale, pollution
control will involve international cooperation and regulation. Internal
policies refer to the regulation of local problems which have international
implications, e.g., policies related to overpopulation or depletion of natural
resources. Although there is general agreement that environmental
management is a task for all nations, and a new framework of international
environmental law should be devised, there is not agreement past these
general principles. When it comes to the specific rules and the institutions to
enforce them, most states and international bodies are floundering.

66.What particular problem does the above passage refer to'?

(a)Inability of political institutions to control pollution

(b)The extent of pollution

(c)The pollution and its consequences at international level

(d)The international pollution control

[UGC D-2008]

67.Mention the factors largely responsible for the changes in biosphere.

(a)Nuclear waste

(b)Air and water pollution

(c)Lack of funds with developing nations

(d)All of these

[UGC D-2008]

68.How has the environmental problem become an international


phenomenon?

(a)Many famous structures are being affected.


(b)Pollution is generated in one region and effects are seen in the other
region.

(c)All the countries are affected with the problem.

(d)Developed countries are encouraging formation of international


policies.

[UGC D-2008]

69.Why does dichotomy exist between developed and developing countries


in reducing emission'?

(a)Developed nations want to control pollution, developing nations want


to generate pollution

(b)Developing nations want to develop their industrial base, developed


nations want them to remain undeveloped

(c)Developed nations are doing their best to control pollution, whereas


developing nations are refusing to enact pollution control regulations.

(d)Developed nations want developing nations to invest in pollution


control, whereas developing nations are not interested and/or they lack
funds.

[UGC D-2008]

Alfred Wallace (1823-1913), the first scholar of zoogeography, used the


stark contrast in animal species among islands in present-day Indonesia to
delimit a boundary between the oriental and Australian realms. He thought
that deep water in the straits had completely prevented specie crossover.
However, he later recognized that a boundary line actually is a wide
transition zone, where one region grades into the other. Boundaries between
natural systems are `zones of shared traits'. Thus, despite distinctions among
individual realms and our discussion of biomes, it is best to think of their
borders as transition zones of mixed identity and composition, rather than as
rigidly defined boundaries.

A boundary transition zone between adjoining ecosystem regions is an


ecotone. Because ecotones are defined by different physical factors, they
vary in width-think of a transition zone instead of a line. Climatic ecotones
usually are more gradual than physical ecotones, where differences in soil or
topography sometimes from abrupt boundaries. An ecotone between prairies
and northern forests may occupy many kilometres of land. The ecotone is an
area of tension, as similar species of plants and animals compete for the
resource base.

Scientists have identified these boundary and edge areas as generating


biodiversity (a species richness) as adaptation and natural selection favour
the population with reproductive success. Organisms may compete, yet they
may act to facilitate other organisms in a positive interaction in whatever
strategies affect rates of ecosystem productivity, such as pollinating insects.
A process of convolution is possible where interacting populations in the
tension zone can lead to genetic changes in each population
(microevolution). Edges and ecotones are dynamic areas in the evolution of
ecosystems and communities.

70.Initially, Alfred Wallace

(a)considered transition zone to be a boundary line.

(b)considered a boundary line to be a transition zone.

(c)considered ecotones to be transition zones.


(d)considered boundary lines to be ecotones.

71.Ecotone is an area of tension because

(a)two different ecosystems are merging to form a third independent


ecosystem.

(b)two independent ecosystems are trying to define their boundaries.

(c)two different ecosystems are competing for the same resources.

(d)two interdependent ecosystems are trying to separate.

72.Biodiversity will be

(a)highest within an ecosystem.

(b)in the ecotone.

(c)in the area where two ecotones merge.

(d)on the border/edge of the ecotones.

73.Generally which of these will have the maximum width?

1.Physical ecotones

2.Climatic ecotones

(a)Only 1

(b)Only 2

(c)Both 1 and 2 will have the same width.


(d)Neither 1 nor 2

Ecological succession occurs when newer communities (usually more


complex) replace older communities of plants and animals (usually simpler)-
a change of species composition. Each successive community of species
modifies the physical environment in a manner suitable for a later
community of species. Changes apparently move towards a more mature
condition, although disturbances are common and constantly disrupt the
sequence.

Traditionally, it was assumed that plants and animals formed a predictable


climax community-a stable, self-sustaining and functioning community with
balanced birth, growth and death-but scientists have abandoned this notion.

Contemporary conservation biology, biogeography and ecology assume


nature to be in constant adaptation and no equilibrium. Rather than thinking
of an ecosystem as a uniform set of communities, think of ecosystems as a
patchwork mosaic of habitats-each striving to achieve an optimal range and
low environmental stress. This is the study of patch dynamics, or disturbed
portions of habitats. Within an ecosystem, individual patches may arise only
to fail later. The smaller the patch, the faster the species loss occurs.
Succession is the interaction among patches. Most ecosystems are in
actuality made up of patches of former landscapes. Biodiversity in some
respects is the result of such patch dynamics.

Given the complexity of natural ecosystems, it is obvious that real


succession involves much more than a series of predictable stages ending
with a specific monoclimax community. Instead, there may be several
stages, or a polyclimax condition, with adjoining ecosystems, or patches
within ecosystems, each at different stages in the same environment. Mature
communities are properly thought of as being in dynamic equilibrium. Or at
times, these communities may be out of phase and in non-equilibrium with
the immediate physical environment because of the usual lag time in their
adjustment.

Succession often requires an initiating disturbance. Examples included


windstorms, severe flooding, a volcanic eruption, a devastating wildfire or
an agricultural practice such as prolonged overgrazing. When existing
organisms are disturbed or removed, new communities can emerge. At such
times of non-equilibrium transition, the interrelationships among species
producing elements of chance and species having an adaptive edge will
succeed in the competitive struggle for light, water nutrients, space, time,
reproduction and survival. Thus, the succession of plant and animal
communities is an intricate process, both in space and in time, and forced by
both internal and external variables.

Land and water experience different forms of succession. We first look at


terrestrial succession. This is characterized by competition for sunlight, and
then at aquatic succession, characterized by progressive changes in nutrient
levels.

74.Traditionally

(a)plants and animals were considered to be in equilibrium.

(b)plants and animals were considered to be in disequilibrium.

(c)plants and animals were considered to be in constant adaption.

(d)None of these

75.Which of the communities will be more complex?

(a)Older communities
(b)Newer communities

(c)Both older and newer communities will be at the similar level.

(d)Cannot be determined

76.Mature communities exist in the state of

(a)disequilibrium.

(b)static equilibrium.

(c)dynamic equilibrium.

(d)None of these

77.The statement that, `non-equilibrium makes it possible for a transition to


happen from one community to another' is

(a)true.

(b)false.

(c)partly true.

(d)Cannot be determined

CHAPTER 4: GEOGRAPHIC THOUGHT

Read the following paragraph and answer the following questions:

[UGC D-2008]

The 17' century witnessed the beginning of the scientific revolution that
led to the development of more useful ways of generalizing, explaining and
communicating. The effort to provide more exact descriptions of specific
things was replaced by the effort to formulate general theory in relation to
which specific things could be made significant. In the formulation and
testing of theory and in communicating the findings, a step of major
importance was the independent development of calculus by Newton and
Leibniz. The use of mathematical procedures made the process of reasoning
more precise and provided a universal language for the communication of
the results. Most of the present fields of science had their roots in the 18'
century, during which time the acceptable methods of study were being
formulated and reliable procedures for verifying hypotheses were being
established.

By the end of the 18th century, the ideas expressed by Kant had become
generally accepted. As bodies of theory developed and proved useful, special
fields of study appeared. These new fields of study became what Kant called
the logical division of knowledge-in contrast to the physical classification of
knowledge in terms of time and space.

The last great figure who could claim universal scholarship was
Humboldt. No student of the Earth before or since has enjoyed such acclaim
by his contemporaries.

The world of scholarship underwent a basic change during the 19hand 20'
centuries. Not only have the fields of learning-the academic disciplines-been
greatly elaborated but also the total number of scholars has reached
unprecedented size. Moreover, the number of recorded facts about the Earth
has increased astronomically. Humboldt could once master a very large part
of the available knowledge concerning the Earth, but this is no longer
possible.

78.The central theme of the passage is concerned with


(a)history of scientific revolution.

(b)scientific research done by Newton and Leibniz.

(c)procedure of verifying hypotheses.

(d)changing methodological and conceptual perspective in geographic


thought.

[UGC D-2008]

79.Large number of various branches of science had their roots in

(a)17th century.

(b)16 thcentury.

(c)18 thcentury.

(d)19 thcentury.

[UGC D-2008]

80.Why, in 18th century, were the concepts and methods of Kant


acceptable'?

(a)Because Kant was a physical geographer.

(b)He provided method of classification of knowledge.

(c)He separated geographical research from historical research.

(d)Development of theory and its verification was possible.

[UGC D-2008]
81.Who was the last of the great 'polymaths'?

(a)Humboldt

(b)Leibniz

(c)Kant

(d)Newton

[UGC D-2008]

The main problem of theory-development in geography is not a failure to


formalize theory. It is, rather, a weak understanding of the role of theory in
explanation and a failure to make verbal statements in ways which are
'explanatory' in some logically consistent way. This is not to say that these
verbal statements are inherently uninteresting. Indeed they are not. Consider
the following statement by Sauer (1963, 359):

The whole task of human geography, therefore, is nothing less than


comparative study of areally localized cultures. But culture

is the learned and conventionalized activity of a group that occupies an area.


A culture trait or complex originates at a certain time in a particular locality.
It gains acceptance-i.e., is learned by a group-and is communicated
outwards, or diffuses, until it encounters sufficient resistance, as from
unsuitable physical conditions, from alternative traits or from disparity of
culture level.

This statement is in part a `directive' for geographic study and in part an


explanation schema for understanding a really localized cultures, it is in part
a description of what happens. But all three notions are intertwined and it is
not clear whether Sauer is proposing a theory or whether he is merely
describing an average process. Yet the statement is surely a stimulating one
and one that manages to sum up intuitively in very short space much of the
activity of the human geographer. We could well pay more attention to
statements of this type and attempt to show how they may function in an
explanatory way.

82.Through this statement, Sauer is proposing a

(a)theory.

(b)process.

(c)directive.

(d)not very clear.

[UGC D-2009]

83.The task of human geography is

(a)intuitive.

(b)intertwined.

(c)stimulating.

(d)all the above.

[UGC D-2009]

84.Culture is

(a)a conventionalized activity.

(b)a directive for the society.


(c)an explanatory way of life.

(d)a stimulation for future generation.

[UGC D-2009]

85.Human geography is the study of

(a)humanbeings in society.

(b)human adaptation to nature.

(c)areal differentiation in culture.

(d)areal differentiation in human resources.

[UGC D-2009]

86.What is the focal theme of the passage given above?

(a)Culture and areal differentiations in cultures

(b)Human geography

(c)Laws and theories in geography

(d)The role of geography in scientific disciplines

[UGC D-2009]

Richthofen's answer to the question: What is geography? As soon to be


acclaimed as the pioneer statement on the scope and method of modem
geography-one that, in the words of Hartshorne `set the direction of
geographic thought for the future'. According to Richthofen, it was the
distinctive purpose of geography to focus attention on the diverse
phenomena that occur in interrelation on the face of the Earth. He
emphasized that in order to reach useful and reliable conclusions,
geographical study of any part of the Earth's surface must start with a careful
description of its physical features and from there the student should move
on to examine the interrelationships of other features of the Earth's surface to
the physical geographic framework described at the outset. He underlined
that the highest goal of geography was the exploration of the relationship of
man to the physical Earth and to the biotic features associated with it.

87.Richthofen's ultimate goal of geography was to explore

(a)the development of regional geography.

(b)areal differentiation.

(c)man-environment relationship.

(d)the development of physical geography.

[UGC J-2011]

88.Hartshorne's opinion that Richthofen's views on geography was `thought


for the future' was due to

(a)its scientific content.

(b)its idea of interrelationship of phenomena.

(c)its emphasis on method of study.

(d)its emphasis on purpose of geography.

[UGC J-2011]
89.Richthofen's method may be categorized in modem terminology as

(a)ideographic.

(b)chorographic.

(c)chorological.

(d)nomothetic.

[UGC J-2011]

90.Richthofen's aim in redefining geography was to develop geography as

(a)human geography.

(b)physical geography.

(c)science.

(d)humanities.

[UGC J-2011]

Read the following paragraph and answer the following questions:

The chorological concept of geography that focused on the study of the


Earth's surface in terms of variations between areas (later named areal
differentiation) was originally developed by Richthofen (1883) and
subsequently advanced by Hettner in Germany. It was popularized and
raised to the level of the dominant concept of geography in the
Englishspeaking world through Hartshorne's monumental Nature of
Geography (1939). A strong tradition of regional geography had already
developed in France under the leadership of Paul Vidal de la Blache. Indeed
regional geography reigned supreme in the world of geographical
scholarship until the early 1950s, after which the regional paradigm for
comprehending the Earth as the world of man began, starting with Schaefer
(1953), to be attacked as one that propagated `exceptionalism' and
`ideography' in the discipline and thereby led the geographer away from the
fundamental goal of science, i.e., the search for general principles of
behaviour. A few years earlier, Kimble (1951) had condemned the concept
of region as an 18r' century concept. He maintained that `it is links in the
landscape... rather than the breaks that impress the scientific mind'. Bunge
(1966) was convinced that the regionalists error lay in their `nation of the
uniqueness of location', owing to which they emphasized exceptions rather
than similarities between places and locations. Coupled with this was a
general impression that the French tradition of viewing the region as an
ecological unit for the analysis of human groups was no longer relevant to
the study of places and regions in the post-Industrial Revolution societies
like those in Europe. All this contributed to a widespread retreat from
regional geography. This meant the `eclipse of a traditional component of
geography which was until recently considered a sine quanon of the
discipline'.

91.The central theme of the passage is concerned with

(a)contributions made by Richard Hartshorne.

(b)research done by Ferdinand von Richthofen and Alfred Hettner.

(c)contributions made by Fred K. Schaefer.

(d)history of regional concept.

[UGC J-2010]
92.Who developed the concept of areal differentiation in geography?

(a)Richard Hartshorne

(b)Paul Vidal de la Blache

(c)Alfred Hettner

(d)Ferdinand von Richthofen

[UGC J-2010]

93.Who stated that `the region is an 18th century concept and it links the
land slope.. .rather the breaks'?

(a)Fred K.Schaefer

(b)Richard Hartshorne

(c)G.H.T. Kimble

(d)Paul Vidal de la Blache

[UGC J-2010]

94.`Pays concept' is associated with

(a)German school.

(b)French school.

(c)American school.

(d)Russian school.

[UGC J-2010]
In the 1980s, areal differentiation made a comeback as a central perspective
for human geography. The revival is neither directly connected to older
debates such as those between Hartshorne and his cities nor monolithic.

There are at least three specific intellectual positions in the revival, none of
which uses the same concepts or vocabulary as the others. The first stream is
Humanistic Geography. Its focus is on the social construction of space and
on `place' as the setting for human action. This has given rise to an interest
in the relationship between specific geographical context and social life. The
second source of revival has come from the analysis of the geography of
capitalism by geographers working within the Marxist perspective. These
geographers explain uneven development of economic activities and social
well-being as an outcome of political and economic relationships of
dominance and dependence which exist in society.

The third source of influence comes from attempts to create contextual


theory in social science The contextual approach consider places and regions
important in the production of society. This approach attempts to show (a)
how time and space (place, region) create the contexts in which human
actions can be explained and (b) how place or region mediate between
`Human Agency' and social structure. Human agency connotes the
capabilities, intentions and actions of `Human Agents'. Human agents can be
individual or other than individuals (companies, corporations, states, etc.).

95.What is the main feature of the new debate on areal differentiation


initiated in the 1980s?

1.It is not connected to older debates.

2.It is monolithic in nature.

3.It has three specific intellectual positions.


(a)1 and 2

(b)2 and 3

(c)1 and 3

(d)1, 2 and 3

[UGC J-2009]

96.What is the main theoretical thrust in Humanistic Geography?

(a)The relationship between specific geographical context and social life

(b)The social construction of space and on `place' as the setting for


human action

(c)The use of human agents

(d)None of these

[UGC J-2009]

97.How have the Marxist geographers explained spatial variations in


economic activities'?

(a)As a result of political differences

(b)As a result of economic differences

(c)As the result of relationship between different classes

(d)All of these

[UGC J-2009]
98.What is the role of time and space in explaining human actions in
contextual theory?

(a)It provides a context for explaining human actions.

(b)It helps in studying and understanding production of human society.

(c)Both (a) and (b)

(d)Neither (a) nor (b)

[UGC J-2009]

99.What do you understand by `Human Agency'?

(a)Why human agents are not what they do

(b)How human agents do what they do

(c)Both (a) and (b)

(d)Neither (a) nor (b)

[UGC J-2009]

The movement that led to the quantitative revolution in Geography was


begun by physicists and mathematicians. It first transformed the physical
and then the biological sciences and, by the 1950s, was strongly represented
in most of the social sciences. The movement towards quantification was
part of the general spread and growth of modes of scientific analysis into a
world formerly dominated by a concern with the exceptional and the unique.
As Burton noted geography has long been a `following' rather than a
`leading' discipline, in that the main currents of thought adopted at various
stages in geography have had their origins in other fields. Thus, the
mechanistic approach in the 19"'-century science found expression in a
deterministic cause and effect approach to the study of man-environment
relationships. There was a similar mechanistic flavour in much of the recent
work of quantifiers. It was as if geography was re-emerging after having
lapsed into ideography which had followed the retreat from environmental
determinism, but although quantification in geography had been mechanistic
in orientation, the new techniques being used were, in line with the
prevailing trend in contemporary science, probabilistic.

100.Why geography could not be a `leading science'?

(a)The main current of thoughts in geography originated in other


disciplines.

(b)The quantitative revolution was begun by physicist and


mathematicians rather than geographers.

(c)Geography followed mechanistic approach.

(d)Burton regarded it as such.

[UGC J-2011]

101.Why did quantitative geography in its later developments get freed from
a deterministic method?

(a)Because there was lapse into ideology.

(b)Because cause and effect approach was initiated.

(c)Because it was mechanistic in nature.

(d)Because quantitative revolution affected it.


[UGC J-2011]

102.Why a mechanistic flavour could be discerned in application of


quantitative methods in its early development'?

(a)Because it was following the trend in the other disciplines.

(b)Because geography was a following rather than a leading discipline.

(c)Because they were fed up with ideology approach.

(d)None of these

[UGC J-2011]

103.Did quantitative revolution in some respect represent moving away from


ideographic approach of the earlier decades in American geography'?

(a)Yes

(b)No

(c)Partially

(d)Cannot be determined

[UGC J-2011]

104.Why did quantitative revolution in its early phases affect more the areas
related to physical geography than those related to human geography?

(a)Because biological and physical sciences needed to be transformed


first.

(b)Because the revolution was started by physicists and mathematicians.


(c)Because geography was not a `leading' discipline.

(d)All of these

[UGC J-2011]

CHAPTER 5: POPULATION AND SETTLEMENT GEOGRAPHY

Yeung and Lo (1996) and Tasaka (1998) have shown that the dynamics of
urbanization in east Asia and Southeast Asia started to change rapidly in
thel980s. Tasaka (1998) characterizes urbanization in developing countries
as `urban involution', describing the characteristics of urbanization in
developing countries by three terms: `concentrated urbanization'
(urbanization occurs while the rural population level is higher than that in
developed countries); `primate cities' (over concentration of economic,
political and cultural functions in primary cities) and `over-
urbanization'(inflow of population beyond a level the city can productively
absorb). Among the factors associated with over-urbanization are high levels
of unemployment and employment insecurity; largescale inadequacies of
infrastructure such as mass transportation systems, water supply and
sewerage systems, and waste treatment facilities; environmental pollution;
severe stress on urban governance; and over all poverty levels that hamper
the formation of financial mechanisms that could facilitate urban
environmental infrastructure improvements (Kidokoro, 1998). It is important
to bear in mind that besides the mechanisms for urban growth described
here,

there are other driving forces for the urbanization process in Asia, and they
vary in different countries. Urbanization in Asia is not a uniform process.
The changes in the mechanisms of urban growth described above can be
seen as imparting a huge impact on urban environmental issues. As the pace
of urbanization accelerates, environmental problems may worsen due to
inadequate urban environmental infrastructure. On the other hand, the
promotion of economic growth could speed up the solutions to
environmental problems.

105.The tone of the above passage can best be described as

(a)analytical and scientific.

(b)factual but descriptive.

(c)behavioural.

(d)tentative and inconclusive.

[UGC D-2010]

106.The author is primarily concerned with

(a)criticizing the urbanization process.

(b)the process of urbanization and over-urbanization in the developing


countries.

(c)factors of suburbanization in the developing countries.

(d)urban infrastructural growth in Asia.

[UGC D-2010]

107.Environmental problems according to the author can be improved with

(a)social inclusion.

(b)economic growth.
(c)political will.

(d)strong legislation.

[UGC D-2010]

108.`Urban Involution' in the developing countries is not characterized by

(a)rural population level is higher than that in developed countries.

(b)a uniform process of rural urban change.

(c)over concentration of economic, political and cultural functions in


primate cities.

(d)inflow of population beyond a level the city can productively absorb.

[UGC D-2010]

It is observed that the rates of urbanization have slowed down in the


Western, industrialized world. Indeed, in some Western countries, an
equilibrium position of low overall population growth and very little
continuing urban growth has been reached. Some major cities, like London,
show little overall changes in population and even absolute declines in the
central areas.

Can we then talk about some limits to the eventual size of cities? We
know that a major factor in urban growth is the centripetal force of
agglomeration economics. But these advantages do not accrue indefinitely as
size increases. Lower production costs may be outweighed by increased
transport costs as urban areas grow to such a size that raw materials must be
imported and finished goods exported over greater distances. Cities may
become increasingly congested, internal transport costs rise and public
health dangers from infectious diseases or antisocial behaviour increase.
These factors contribute to the centrifugal forces that tend to halt
concentration and to scatter population. As the cost of overcoming spatial
separation has fallen, so the relative strength of centripetal forces has grown.
Water pipelines, super tankers and refrigerated ships are symbols of a
continuing transport revolution which enables centralized population
agglomerations to draw on widely distributed resources; equally important
are the sewage lines and disposal services which allow a city to dispose of its
massive daily burden of wastes. It is difficult to measures the exact balance
of cost of cities, but the probable form of the cost curves as the size of the
cities increases suggests that there is a threshold over which further increases
in size are uneconomic and growth will slow down. At any one time, one
particular bottleneck may be critical. For the 18'-century European city, the
water supply and infectious diseases were significant barriers to growth; for
today's cities, crime and finance may be higher on the agenda.

109.What are the recent urbanization trends in the industrial world?

(a)Decline in total area of cities

(b)Lower overall population growth rate

(c)Decline in growth of cities

(d)All of these

[UGC D-2006]

110.How do centripetal and centrifugal forces determine the size of a city?

(a)Centrifugal forces encourage scattering and centripetal forces


encourage growth.
(b)Centripetal forces encourage growth, while centrifugal forces
encourage scattering.

(c)Both centripetal and centrifugal forces encourage growth.

(d)Both centripetal and centrifugal force encourage scattering.

[UGC D-2006]

111.The role of urban infrastructure is that

(a)it encourages urbanization.

(b)it discourages urbanization.

(c)it encourages urbanizations with its presence it discourages


urbanization with its absence.

(d)it does not affect urbanization.

[UGC D-2006]

112.What are barriers to spatial growth of cities'?

(a)Limited access to amenities

(b)Limited financial resources

(c)Both (a) and (b)

(d)Neither (a) nor (b)

[UGC D-2006]

113.Which costs of urban development inhibit the growth of cities?


(a)Transport cost

(b)Social cost

(c)Health costs

(d)All of these

[UGC D-2006]

With the Industrial Revolution, which first began in England and then spread
to other parts of Europe, the process of modem urbanization truly began.
Several economic opportunities became available in industrial centres,
leading to massive rural-urban migration, which resulted in a dramatic
redistribution of population. The main factor contributing to the urbanization
process during and after the Industrial Revolution was economic in nature.
As employment and economic opportunities offered inducements to people
to migrate from the rural to the urban areas, changes occurred, not only in
their residence but also in their occupations. In addition to the attraction
provided by employment opportunities, other factors affecting socio cultural
conditions were also instrumental in influencing rural-urban migration.
Educational infrastructure facilities and amenities are also superior in urban
rather than in rural areas, making the former more attractive for residence
than the later. Even in the developed countries such as the USSR and Japan
where industrialization set in later than in Europe, planned economic and
social growth has been responsible for certain developments which have
relevance in the context of the distribution of population in rural and urban
areas. The planned industrial economy of Japan has led to a great deal of
rural-urban migration motivated by economic factors and has resulted in a
highly urbanized modern society. In the USSR, despite the efforts of the
government to encourage people to settle in sparsely populated territories,
migration from rural areas to the cities has been heavy because of the ample
employment opportunities available in the latter. In the USSR, efforts are
made by the government to arrest the growth of the largest cities rather than
to discourage them from migrating from rural to urban areas.

114.What caused modem urbanization?

(a)Increase in economic opportunities

(b)Employment opportunities

(c)Industrial revolution

(d)All of these

[UGC J-2006]

115.Beside economic opportunities, what other factors caused rural urban


migration'?

1.Increase in employment opportunities

3.Better amenities

2.Better educational facilities

4.Government incentives

5.Beautiful residence

(a)1, 2 and 4

(b)2, 3, 4 and 5

(c)2 and 3
(d)1, 4 and 5

[UGC J-2006]

116.What are the factors which continue to influence rural-urban migration?

(a)Employment

(b)Infrastructure

(c)Government

(d)All of these

[UGC J-2006]

117.In planned economies like former USSR, what factors affected the rural-
urban distribution of population'?

(a)Economic growth

(b)Social growth

(c)Both (a) and (b)

(d)Neither (a) nor (b)

[UGC J-2006]

118.Did the efforts of certain countries to discourage rural-urban migration


succeed?

(a)Yes

(b)No
(c)Partly

(d)Cannot be determined

[UGC J-2006]

A town cannot exist simply by servicing its own population. In order to


flourish and expand, a proportion of its goods and services must be sold
beyond its own limits. It follows from this that a proportion of any urban
labour force is directly concerned with the production of goods and services
for `export'. They are termed basic or city-forming workers, since their
efforts bring money into the town, thus enabling the purchase of raw
materials, foodstuffs and other goods which the town cannot supply for
itself. The remaining workers are referred to go non-basic or city-servicing
workers, since their role is to serve the needs of the urban population itself.
Although essential to the functioning of the town, the non-basic workers
depend upon the revenue brought into the town by those in the basic sector.
This interpretation of urban occupation structure is known as the basic/non-
basic concept.

119.How does a city flourish'?

(a)By servicing its own population

(b)By trading outside its border

(c)By trading within its border

(d)All of these

[UGC D-2011]

120.How does the city's economy expand?


(a)By efforts of the basic workers

(b)By the efforts of the non-basic workers

(c)By purchase of raw material and foodstuff

(d)By sale of goods and services

[UGC D-2011]

121.What constitutes the occupational structure of the city?

(a)Basic workers

(b)Non-basic workers

(c)Both (a) and (b)

(d)Neither (a) nor (b)

[UGC D-2011]

122.What do the non-basic workers do?

(a)Serve the population of the city

(b)Support the basic workers by servicing them

(c)Support the non-basic worker population

(d)Generate revenue for the expansion of the economy

[UGC D-2011]

123.Who are the city-forming workers'?


(a)Workers engaged in producing goods for export

(b)Workers engaged in producing services for export

(c)Both (a) and (b)

(d)Neither (a) nor (b)

[UGC D-2011]

CHAPTER 6: ECONOMIC GEOGRAPHY

One hazard of the Green Revolution lies in the requirement that the genetic
strains bred for high yields must be used to the exclusion of a variety of
native strains. Should the high-yield strain prove vulnerable to an epidemic
plant disease, the entire crop of a whole nation could be wiped out in one
season. Current research in crop breeding is now stressing the devel opment
of more individualized varieties that respond better to local conditions.
These varieties are less subject to epidemic diseases and can still produce
improved yields with less fertilizer input. A second hazard lies in the need to
increase the size of fields by merging many small plots into large ones to
allow mechanized agriculture to work most efficiently. In doing so, a variety
of food crops are no longer grown, and dependence for survival comes to
rest on a single crop. Under traditional practices, the Asiatic farmer planted
several food crops to ensure that if some failed, others would yield enough
food to prevent starvation. The small farmer is therefore reluctant to move to
more efficient, single-crop agriculture.

It seems obvious now that future increases in yields will be achieved only
by modifying the techniques of the Green Revolution to become less
dependent on industrial midlatitude technology. New crop strains and
methods of cultivation and fertilization will have to be more compatible with
ancient agricultural systems and local culture patterns.

Soil scientists have pointed out that large areas of vertisols remain to be
placed under cultivation. The vertisols are rich in nutrient bases but will
require machine cultivation to overcome difficult tillage. Primitive ploughs
and hoes cannot work in these soils. Most of the arable land of the savannah
environment in Southeast Asia has already been intensively developed by
existing standards. To hope for major expansion of agriculture into poor
oxisols and to semi desert zones, marginal to the tropical deserts is, at best,
unrealistic.

124.What is the focal theme of the passage given above?

(a)Need for emphasis on traditional agriculture

(b)Need for emphasis on the use of industrial mid-latitude technology

(c)Need for modification in the techniques of Green Revolution

(d)Need for development of unproductive lands

[UGC J-2008]

125.One of the hazards of the Green Revolution is

(a)monoculture.

(b)multicrop agriculture.

(c)use of Machines.

(d)small size of fields.


[UGC J-2008]

126.Current research in agriculture is being oriented to

(a)the development of new fertilizers and pesticides.

(b)development of the varieties of seeds suitable for local conditions.

(c)development of areas of oxisols.

(d)development of new machines.

[UGC J-2008]

127.Traditional Asiatic cultivation emphasizes on

(a)mechanized agriculture.

(b)use of fertilizers in large quantity.

(c)monoculture.

(d)multicrop agriculture.

[UGC J-2008]

One of the most basic relationships of transportation involves how much


space can be overcome within a given amount of time. The faster the mode,
the larger the distance that can be overcome within the same amount of time.
Transportation, notably improvements in transport systems, changes the
relationship between time and space. When this relationship involves easier,
faster and cheaper access between places, this result is defined as a space-
time convergence because the amount of space that can be overcome for a
similar amount of time increases significantly. Significant regional and
continental gains were achieved during the 18'and 19 'centuries with the
establishment of national and continental railway systems as well as with the
growth of maritime shipping, a process which continued into the 20' century
with air and road transport systems. The outcome has been significant
differences in space-time relationships, mainly between developed and
developing countries, reflecting differences in the efficiency of transport
systems.

At the international level, globalization processes have been supported by


improvements in transport technology. The result of more than 200 years of
technological improvements has been a space-time collapse of global
proportions in addition to the regional and continental processes previously
mentioned. This enabled the extended exploitation of the advantages of the
global market, notably in terms of resources and labour. Significant
reductions in transport and communication costs occurred concomitantly.
There is thus a relationship between the rates of a space-time collapse on the
integration of a region in global trade.

128.In this passage, the word `space' means

(a)length.

(b)distance.

(c)breadth.

(d)area.

129.The passage is talking about

(a)time travel.

(b)geography of transportation.
(c)space travel.

(d)wormholes.

130.Space-time collapse means

(a)ability to send goods from one place to another at less cost.

(b)reduction in communication cost.

(c)exploitation of global markets.

(d)ability to travel a required distance in less time.

131.The statement `significant difference in space-time relationship' means

(a)there is a vast difference in transport systems of developed and


developing countries.

(b)there is a vast difference in transport systems of 19th and 20th


centuries.

(c)Both (a) and (b)

(d)Neither (a) nor (b)

The market town may be viewed as the city port of a small open economy.
Once goods are gathered in the city port, they may be sold within the
country or exported to the rest of the world. In this case, the set of crops is
primarily determined by the goods' prices that prevail on the international
marketplace. Consequently, if the prices of the exported goods are
sufficiently high, the goods consumed by the local population only may be
no longer be available because they generate too low surpluses. In other
words, trade seems to be harmful to the local population, since the
inhabitants must pay higher prices to import the agricultural products they
need. This argument must be qualified; however, the market distribution of
crops generates the highest social surplus. Hence, the population is
potentially betteroff under trade because the surplus net of the costs of the
imported goods is higher than the surplus made when the imported goods are
locally produced. The key issue is the structure of property rights. If a land is
collectively owned, people are better-off. On the contrary, if the land rent is
captured by absentee landlords or by an unproductive elite, people are
worse-off. Therefore, trade per se is not harmful to the local population.
What matters for the well-being of the local population is the ownership
structure of land.

132. Social surplus means

(a)the difference between net imports and net exports.

(b)the difference between the cost of importing needed agriculture goods


and the income from exporting agricultural goods.

(c)the amount available to the society for spending.

(d)the total profit from the exports.

133.According to the passage

(a)absentee landlords are harmful for the economy.

(b)unproductive elite siphon-off the funds needed by the poor members of


the society.

(c)collective ownership will ensure that additional burden of imports is


not felt by the society.
(d)collective ownership will stop people from becoming wealthy.

134.Which of the following statements if true undermines the argument


presented by the author'?

(a)Elites will save all the money available to them.

(b)Absentee landlords will spend all the money on importing goods and
services.

(c)Unproductive elite will spend all the money in the local economy.

(d)Absentee landlords live outside the society.

135.The argument presented in the passage ignores

(a)economy of scales.

(b)transportation cost.

(c)labour costs.

(d)All of the above

Road transport projects, as most other development interventions, often


carry human costs and necessarily bypass or exclude some individuals or
groups of people, thereby making them bear the burden of `social good'.
These involve displacement and involuntary resettlement of people, with its
potentially impoverishing impacts. This has become an unintended but
inescapable aspect of most development interventions, like dams, industrial
units, thermal power plants and even road projects.
Displacements in the road transport projects are unique in many ways. A
World Bank (2004) publication asserts that upgradation and improvement of
highways often does not require large land acquisition at a location due to
their narrow linear alignment. Rather, such projects require smaller patches
of land over a long distance all along its corridor. Although this aspect
reduces the adverse impact of displacement by displacing fewer people per
unit area, it adds more complexity to spread the thin effect over a large
stretch that involves hundreds of settlements. The report further notes that
land acquired in linear projects frequently consists of a strip along property
frontages. Acquiring these strips may not require relocation of occupants or
users and typically may not negate the economic viability of landholdings.
Linear road projects can usually be rerouted to avoid heavy concentrations
of population and to avoid large-scale resettlement. If relocation

is needed, though, people can usually be resettled in the same area-


sometimes, especially in rural zones, even on the same plot of land.
Nonetheless, even small-scale land acquisition can result in significant
hardships for some displace people. In rural areas, linear projects may bisect
existing holdings or isolate communities from their productive resources and
employment centres; in urban areas, they may dislodge many people without
secure title. Although linear projects usually have less severe impacts than
other projects, they still pose considerable challenges in resettlement
planning

136.Suggest a suitable title for the above passage.

(a)Road transport projects and its effects

(b)Development projects and resettlement

(c)Road project and resettlement


(d)Planning for road projects

[UGC D-2010]

137.What are the unique features of displacement in road sector projects?

(a)It involves more number of resettlements.

(b)It involves smaller patches of land.

(c)In general, they do not negate economic viability of landholdings.

(d)All of these

[UGC D-2010]

138.What are the human costs of development projects'?

(a)The people involved in the construction work

(b)Displacement of people from their land

(c)Forcing people to give up their livelihood

(d)All of these

[UGC D-2010]

139.Which of the following resettlement issues are faced in the road


projects?

1.Reallocation of the people in the same area

2.Isolation of communities from employment centres

3.Dislodgement of people without proper title


(a)1 and 2

(b)2 and 3

(c)3 and 1

(d)1, 2 and 3

[UGC D-2010]

140.Which of the following can be regarded as the typical land acquisition


problems in road projects?

1.Presence of heavy concentration of population

2.Dealing with hundreds of individual property owners

3.Forcing people to give up commercial property

(a)1 and 2

(b)2 and 3

(c)3 and 1

(d)1, 2 and 3

[UGC D-2010]

CHAPTER 7: POLITICAL, SOCIAL AND CULTURAL GEOGRAPHY

Read the following paragraph and answer the following questions.

In the USA, many people consider economic growth a necessity, whereas


some still support an attitude of `growth at any cost', increasing numbers are
becoming aware of the environmental costs of sustained economic growth.
During the 1970s, public dissatisfaction with growth mania appeared in a
variety of ways. Oregonians, e.g., were saying that visitors were welcome
but new residents were not. In Colorado, bumper stickers decried the
Californication of Colorado. In California, voters passed proposition 20,
which placed tight control on all coastal zone developments. Communities in
many places became more selective about the types of industries which they
were willing to accept; clean industries were considered desirable but dirty
industries were not.

141.In the USA, many people consider economic growth a necessity,


because of

(a)additional employment potential.

(b)additional tax resources for the government.

(c)potential for higher GDP growth.

(d)potential for higher personal incomes.

[UGC D-2011]

142.Many Americans in recent times are becoming environmentally


conscious because of

(a)spread of environmental awareness.

(b)environment-friendly technologies adding jobs in the economy.

(c)the understanding of trade-offs between economic growth and


environmental costs.
(d)economic growth is desirable and not a necessity.

[UGC D-2011]

143.Growth mania refers to

(a)aversion to growth.

(b)growth only attitude.

(c)growth of employment.

(d)growth with environmental concerns.

[UGC D-2011]

144.Many American States approve of tourists but not immigrants because

(a)the tourists create employment.

(b)the immigrants are non-Englishspeaking.

(c)the immigrants take away jobs of the locals.

(d)the tourists

[UGC D-2011]

Geographers have introduced the concept of territorial social indicators to


identify and analyse socio spatial variations in quality of life at different
geographic scales, ranging from global to local. Most of the research using
territorial social indicators has employed objective measures derived either
from primary field surveys or from analysis of secondary, normally census-
based, data sets. Collectively, this line of research has contributed valuable
insights in to such questions as the extent and distribution of substandard
housing and the differential incidence of deprivation within the city.

The objective perspective has been paralleled, however, by the


development of work using subjective social indicators and an approach
focused on the concept of urban liveability. In contrast to the objective
definition of urban environmental quality, urban liveability is a relative
rather than absolute term whose precise meaning depends on the place, time
and purpose of the assessment and on the value system of the assessor. This
view contends that quality is not an attribute inherent in the environment but
is a behaviour-related function of the interaction of environmental
characteristics and person characteristics. It is axiomatic that in order to
obtain a proper under standing of urban environmental quality, it is
necessary to employ both objective and subjective evaluations. In other
words, we must consider both the city on the ground and the city in the
mind.

145.What is the shortcoming of the using objective measure of social


indicators'?

(a)It assumes that social well-being can be defined in relative terms.

(b)It has failed as a tool for measuring social well-being.

(c)It assumes that the social well-being can be defined in absolute terms.

(d)None of these.

146.Urban liveability considers quality

(a)to be present in the environment.

(b)to be relationship between human beings and the environment.


(c)to be both inherent and behaviour dependent.

(d)None of these

Territoriality is firmly integrated in the mainstream literature on federalism


and federation. Indeed, for many of the major contributors to contemporary
federal theory, it is the key component on which they base their
understanding of modem federation. However, the way that it has been
conceptualized by the earlier scholars of federalism was somewhat limited
and oversimplified. In this section, I want, briefly, to explore this
understanding from the particular standpoint of the conceptual relationship
between federalism and federation. For the moment, I will interpret these
two concepts strictly, since they provide us with a useful benchmark or
yardstick from which we can later introduce modifications to furnish a more
flexible understanding.

Because it is much more familiar, let us begin by defining what we mean


by federation. A federation is a state; it is a federal state. This means that it is
a particular kind of state, one that is founded upon a basic contractual
constitutional agreement or bargain designed primarily to recognize, protect,
promote and sustain diversity. The appeal of the federal idea-and its
manifestation in federation-lies in its infinite institutional and structural
capacity both to accommodate and to reconcile different forms of unity with
different forms of diversity. Federations exist because they formally
acknowledge, via constitutional entrenchment, the sorts of identities and
diversities that constitute that sense of difference so essential to a living
breathing pluralist social and political order. To qualify as a federation, then
it is essential that the state constitute all of the paraphernalia required of a
liberal democratic state, including a written constitution, an independent
judiciary, a sustainable division of powers and competences between at least
two levels of government that is constitutionally entrenched, agreed
procedures for constitutional amendment in which the constituent units can
actively participate, and a range of attributes designed to guarantee the
legitimacy, autonomy and integrity of the constituent units of the federation.
The presumption is that only core liberal democratic values and principles
can be the source of the compromise between liberty and autonomy that
ultimately drives federal unions.

147.Federation is

(a)a union of federal state.

(b)a state having some of the federal characteristics.

(c)a federal state.

(d)a state which is not central.

148.A federal state recognizes

(a)diversity.

(b)similarity.

(c)chaos.

(d)both (a) and (b).

149.Federal state must have at least governments.

(a)one

(b)two

(c)three
(d)four

150.In the passage, the writer is primarily

(a)presenting his point of view.

(b)presenting classical conceptualization of a federation.

(c)opposing the classical view of federation.

(d)All of these

The point to be noted in this conceptual relationship is that federalism


informs federation andvice versa. Federalism is multidimensional: political,
social, economic, philosophical, historical and socio psychological and
manifests itself in a variety of what we might call 'constellations of social
cleavages having political salience' in a state. Different federations and
federal political systems have different combinations of these cleavage
patterns expressed by groups and communities collectively that seek
recognition, accommodation and representation. Once formed, federation
itself-its rules and procedures-acts on federalism and guides political
behaviour in ways that assist towards trust, legitimacy and identity changes
through adaptation. Looked at in this way, it is easy to see how territoriality
fits into this complex interrelationship. This is because federations are ipso
facto composed of clearly demarcated constituent units that are territorial
units. The German lander, Swiss cantons and Canadian provinces are all
territorially bounded communities that are the building blocks of each
federation and represent the political organization of space in the occupation
of territory. And in many, though not all, of these cases, territoriality-the
sense of place-plays a significant role in their self-definition. However, it is
certainly not the only source of such definition. Territorial identity cannot be
construed in isolation from other social cleavages that interact with it to
forge a distinct identity, a strong sense of self. The question arises therefore
of precisely what does each constituent unit in a particular federation mean'?
What is it composed of and how does it see itself as an institutionalized
expression of collective identity?

In one of the standard commentaries on the comparative study of federal


political systems, published in 1970, No Duchacek confirmed that his survey
was about `the territorial distribution of political authority' and that its
central concept was `the territorial community or the territorial interest
group'an aggregate of individuals and groups who shared `not only common
experiences, values, fears and purposes but also an awareness of the
territorial dimensions of their collective interests and actions'. For Duchacek,
the study of federations was limited to `territorial communities', and federal
constitutions expressed what he called `the core creed of democracy,
pluralism, in territorial terms'.

Similarly, Ramesh Dikshit emphasized the importance of spatial


organization in his own contribution to this particular debate in the 1970s.
His interpretation was that of the political geographer and it came as no
surprise, therefore, when he claimed the study of federalism to be `the most
geographically expressive of all forms of government'; it was `the complex
interaction of spatial differences and similarities' that made possible `the
geographical approach to federalism'. Because the study of federations was
based largely upon polities characterized by 'interactions and interrelations
of a complex set of regionally grouped diversities (represented by and
articulated through the constituent states of the nation), a geographical
approach... with its emphasis on areal integrations and regional interactions'
was deemed particularly helpful in this task.

151.The term `cleavage patterns' used in the passage means


(a)unity of diverse opinion.

(b)unity of political ideas.

(c)a division of unified political ideas.

(d)diversity of common opinions.

152.According to the author

(a)territorial identity cannot be isolated from the self-awareness of


territorial identity.

(b)for the purpose of federal understanding, territorial identity alone


cannot be considered without considering other unifying factors.

(c)territorial identity is the sole factor responsible for the federal


structure.

(d)geographical approach to federation cannot be considered without


taking territorial identity as the sole governing factor.

153.According to No Duchacek

(a)territorial awareness gives birth to shared common variables.

(b)shared variables give birth to territorial awareness.

(c)shared factors must be unified with territorial awareness of collective


interest and values.

(d)None of these

154.Ramesh Dikshit considered federalism to be


(a)interactions and interrelations of regionally grouped diversities.

(b)interactions and interrelations of regionally grouped unities.

(c)a building block of geographical polity.

(d)a basis of regionalism.

CHAPTER 8: REGIONAL PLANNING

Read the following paragraph and answer the following questions.

In order to formulate a regional development policy in national planning


with special reference to the federating states, an inventory of the spatial and
locational patterns of the key sectors of the national economy is needed. On
the basis of the analysis of the inventory, the country may be divided into
five or six macroregions comprising groups of federating states. Contiguity,
inter-and intrastate relationship and the resource potential for developing
distinct regional economies could be the considerations in regional
delineation. These would be the regions for formulating a regional
development strategy in National Planning. For this purpose, an attempt
should then be made to analyse the existing spatial and locational
components of the national development plans and to indicate alternative
locations and areas within each of the macro-region to which the additional
targets of production could be allocated. This analysis involves a study of
the spatial dimension of the individual sector as well as regional and inter-
regional analysis. The acceptance of any one inter-regional sectoral plan and
the corresponding locational framework would be subject to political
considerations too. It is in this context that the regional development policy
has to be formulated in the national plan. The impact of such a development
policy on the goals of economic and social development should further be
spelt out on the basis of federating states which are the parts of the macro-
regions. For a lack of the inventory on locational and spatial aspects and
inadequate regional economic analysis of the development plans, the
national plan leans heavily on the aggregated sectoral plans of states and
interstate comparison of selected sectors of development.

The spatial and the locational framework arrived at in the manner


mentioned above would serve to pinpoint areas of growth and corresponding
locations that take care of the key sectors of the national plan. Political
factors that enter into actual policy formulation would then have an objective
basis for reconciliation. More intensive analysis is needed at the state level
for integrating the growth areas and growth points evolved out of the
regional development policy consideration into the plans of different sub-
regions within the states.

155.The central theme of the passage is concerned with

(a)the importance of regional development policy.

(b)the need for inventory of the spatial and locational patterns.

(c)the goals of economic and social development.

(d)the sectoral plans of states.

[UGC J-2007]

156.The crux of the problem for regional development planning is

(a)leaning heavily on the sectoral plans of state.

(b)political factors entering into actual policy formulation.

(c)integration of growth areas and growth points.


(d)lack of inventory of the spatial and locational aspects.

[UGC J-2007]

157.Dividing the country into macro-regions comprising groups of


federating states could be done on the basis of

(a)the inter- and intrastate plans.

(b)the analysis of inventory.

(c)the individual sector analysis.

(d)the inter-regional analysis.

[UGC J-2007]

158.Political considerations are very much essential in

(a)national planning.

(b)regional development policy.

(c)inter-sectoral development.

(d)All the above

[UGC J-2007]

The life and well-being of individual and society depend on the existence
and permanence of natural resources that provide sustenance and
environment, as well as on the coordinated gradual use of these resources.
By making use of different possibilities and capabilities and by introducing
different combinations of natural materials and processes, we produce
`artificially' food, goods and components of our environment; we modify
natural conditions. However these techniques are not enough to establish
habitability. Mostly man has to modify himself and his society as well in
order to meet the demands of his environment and develop a modus vivendi
adapted to natural conditions of space and time, so that they may provide
him permanently with life and well-being. It is correct, therefore, to say that
regional planning deals with the revelation of natural factors on the one hand
and the revelation of ways and needs of man on the other, but this without
any dogmatic prejudices as to what is the determining and the determined
factor and without any preliminary intention to force one on the other. The
proper target of regional planning has to be defined specially for each
geographical case; generally expressed, it is just the creation of a balance in
respect of environment in order to facilitate the basic processes of human
life. For that purpose, a plan may indicate in one case the modification of
nature to fit human needs, and in another case, the modification of social
relations and human conditions to fit environmental needs. With that
unattached approach, leading in different cases to different ways of planning
and development, the essential difference of regional planning for
habitability from all other sorts of contemporary physical, economic and
political planning is marked.

159.How do we modify natural conditions'?

(a)By introducing artificial conditions

(b)By using natural resources for producing new resources

(c)By destroying the natural conditions

(d) None of these

[UGC D-2007]
160.What factors are necessary for building habitability?

(a)Modification of natural conditions

(b)Modification of self and society

(c)Both (a) and (b)

(d)Neither (a) nor (b)

[UGC D-2007]

161.What is the appropriate target of regional planning?

(a)Revelation of natural factors

(b)Establishing an equilibrium between environment and human needs

(c)Conquering of environment for satisfying human needs

(d)Adjustment of human needs for satisfying the environment

[UGC D-2007]

162.Why `technics' are not enough to establish habitability'?

(a)Modification of environment

(b)Modification of human needs

(c)Both (a) and (b)

(d)Neither (a) nor (b)

[UGC D-2007]
163.What is the essential difference between regional planning for
habitability and other types of planning'?

(a)Former deals with human needs, the latter deals with environment
needs.

(b)Former deals with environment needs, the latter deals with human
needs.

(c)Former does not have environmental needs as a sole basis, whereas the
latter concentrates on environmental needs only.

(d)Former does not have fulfilment of human needs as a sole basis,


whereas the latter concentrates on the fulfilment of human needs.

During the height of quantitative era of human geographic studies in the


1960s and 1970s, regional development was theorized as a non-uniform
process of cumulative causation, sustained by technological advancements.
Development tends to originate in small groups of cores, whose peripheries
fall behind, and is in many ways dependent on the cores. Classical theory
states that the economic expansion of one region means at least a
proportional decrease in another. These backwash effects are essentially
linked with mechanisms of migration, employment, finance and commerce.

Migration assembles educated populations of the most productive working


age in regions of expanding economic growth. Ravenstein's laws of
migration still apply because migration flows are increasingly concentrated
towards centres. In countries with one internationally sized centre, such as
Finland, this commonly means the capital area and its surroundings.
Outward migration causes problems in peripheral regions, including
distortion of the age structure and decrease in the tax-paying workforce.
Similarly, finance and global capital flows are directed towards the most
potential and profitable investments, which are abundant in regions of
expanding growth. These regions of expanding growth offer favourable
preconditions for the development and concentration of commerce and
services.

Countering forces exist that hinder the sources of congestion. These


include increase in the cost of living and accommodation and the heightened
crime rate in cities. The widespread effects of regional development
arguably trickle down from the periphery towards the surrounding regions,
leading to increase in cumulative causation outside the cores. The radial
dissemination of innovation outward from the initial core, the
neighbourhood effect, is accompanied by the rise of secondary cores,
identified as the hierarchy effect, which means that the innovation
simultaneously cascades downwards to neighbouring regions and to higher
locations such as rural centres. Since the effects of cumulative causation on
neighbouring regions are both positive and negative, there is no clear
consensus whether the widespread effects surpass the backwash effects. The
outcomes are dependent on numerous factors, including national regional
policy, developmental stage and position in global markets.

164.Which of the following backwash is faced by periphery regions due to


migration'?

(a)Increase in tax base

(b)Inward remittance

(c)Outward remittance

(d)Brain drain
165.Why does outward migration cause distortion of age structure in
periphery regions'?

(a)The supporting population leaves, causing the dependent population to


be left behind.

(b)The dependent population leaves, causing the supporting population to


be left behind.

(c)Increase in crime rate causes more deaths.

(d)None of these

166.In simple terms, the hierarchy effect is caused by

(a)backwash effects.

(b)the right national regional policy.

(c) tickling down of regional development.

(d)None of these

167.The overcrowding of the cities it arrested due to

1.rising crime rate.

2.rising cost of living.

(a)Only 1

(b)Only 2

(c)Both 1 and 2
(d)Neither 1 nor 2

168.According to classical theory

1.growth of one region can happen without loss in the other region.

2.the development of the core and peripheries is simultaneous.

(a)Only 1

(b)Only 2

(c)Both 1 and 2

(d)Neither 1 nor 2

CHAPTER 9: GEOGRAPHY OF INDIA

There exists voluminous literature dealing with the issue of regional


disparity. Most studies have targeted state as unit for measuring disparity and
have sought to gauge the impact of development policy on relative
development of the states. The findings of such studies have been used by
government agencies to frame policies to promote balanced regional
development. These policies have met with limited success and not only
disparity has increased but has started showing its ugliest face. One
significant factor causing limited success to efforts of balanced
developments has been the neglect of variations within states and exclusive
reliance on information relating to disparity at the state level. For a huge
country like India where some of the states are bigger than many nations, it
is very important to look at the disparity at the disaggregate level. This is on
account of a number of reasons. First, the regional development policy is
framed by the government by treating the state as a homogenous unit, which
it is not. Measurement of disparity at the district level would help frame
area-specific plans and policies in a better manner and adopt policies
suitable to tackle different regions within a state. Second, a study of disparity
at the disaggregated level is essential for ascertaining the level of
development in agriculture, industry, infrastructure, per capita net state
domestic product (NSDP), level of literacy among men and women, health
cover and other sectors across all the districts of a particular state and also
for analysing the respective roles of physical/natural factors vis-a-vis man-
made factors in causing (or aggravating) inter-regional economic
inequalities. Third, the widening gulf between advanced and backward
regions within a state leaves those living in backward regions disgruntled
and dissatisfied creates an aversion towards the civic processes and raises
doubt about the viability and usefulness of the political system. This has a
destabilizing impact on district economy and polity. Fourth, growth of
developed pockets within a particular state promotes concentration of
economic units in the region, results in increased internal migration, causes
environmental backlash, etc. Fifth, experience shows that cross-country
comparisons of stage and pace of development are impossible and, following
the same logic, interstate interregional comparisons are also very difficult
when the regions differ significantly in terms of size, principal features,
governance, etc. A study at the disaggregated level is required for
identification of the factors that are instrumental in the controlling regional
disparity and developing policy mix to promote the same.

169.Why is it important to look at disparity at the disaggregate level'?

(a)Indian states have disparity within themselves.

(b)Destabilizing impact of district economy and polity

(c)Interstate inter-regional comparisons are easier than intrastate


intraregional comparisons.
(d)All of these

170.According to the author

(a)micro level planning has failed.

(b)intrastate disparity also needs to be considered for balanced growth.

(c)regional development plans at the national level can succeed.

(d)None of these

171.The formation of area-specific plans and polices for different regions


within the state are needed because

1.plans and policies for balanced regional development at national level


are failing.

2.some of the states in India are larger than some countries.

3.inter-regional inequalities within the states are increasing.

(a)1 and 2

(b)2 and 3

(c)3 and 1

(d)1, 2 and 3

172.District economy and polity are destabilized

(a)due to unviable political system.

(b)due to environmental backlash.


(c)due to dissatisfied backward region.

(d)due to useless political system.

In 2010, the Government of India announced an explicit focus on cities as


loci of economic activity. In January 2012, the Institute for Financial
Management and Research (IMFR) in India released a statement identifying
the Indian economy's dependence on large, rapidly expanding urban centres
as drivers of economic growth and productivity. However, it is not clear
whether agglomeration economies, the benefits arising from the geographic
clustering of people, firms or activities, outweigh the negative congestion
effects that arise in highly dense areas. Agglomeration increases the
proximity of people and resources, leading to decreased transportation costs
for goods, people and ideas; on the other hand, congestion drives up real
estate prices, creates competition for labour and other resources and causes
pollution.

While cities are hubs of economic activity, differences in density across


states in India vary widely, and it is not clear whether the benefits of
agglomeration economies hold true for levels of density that are much higher
than those in the USA. Uttar Pradesh, a state in north India, has over 200
million inhabitants alone, meaning it would be the fifthlargest country ahead
of Brazil according to 2011 estimates. In India, where levels and variation of
spatial disparity are far higher than in most developed economies, are
agglomeration forces more powerful or are congestion effects more
overwhelming?

Economies of agglomeration are an application of the larger study of


economic geography, defined by Economist Paul Krugman as `the location
of factors of production in space'. Contemporary agglomeration studies
emphasize the roles of positive externalities that appear as a result of
increased interaction in dense areas. That is, the physical proximity of
output, people and firms in urban areas creates an environment that
facilitates economies of scale, access to resources, technological
advancement, efficient labour markets and flows of ideas and information,
thereby enhancing labour productivity. If the cost or number of labour hours
required for transporting products from one stage of production to the next
increases with distance, creating the technology to conduct all stages of
production in one geographical area leads to increasing returns to scale.
Thus, the output per worker increases as the density increases. The
concentration of economic activity also increases the number and variety of
complementary goods and services available. Moreover, denser areas
enhance knowledge dissemination in industries where information exchange
is difficult, rapidly changing, or of particular importance, for instance in
high-skilled industries.

173.According to IMFR, the economic growth in India is

(a)region-driven.

(b)rural-driven.

(c)city-driven.

(d)state-driven.

174.According to the passage, the agglomeration economy's benefits

1.hold true for countries having density equivalent to USA density.

2.hold true for countries having density greater than USA density.

(a)Only 1
(b)Only 2

(c)Both 1 and 2

(d)Neither 1 nor 2

175.Which of the following are benefits of agglomeration?

(a)Concentration of tax payers makes tax collection easy.

(b)Knowledge flow increases.

(c)Spatial disparity falls.

(d)Real estate prices rise.

176.Increase in density

(a)increases labour productivity.

(b)increase crimes.

(c)creates competition for resources.

(d)increased spatial disparity.

177.The writer is trying to find out

(a)the benefits of agglomeration.

(b)the cost of agglomeration.

(c)whether agglomeration is beneficial or not for India.

(d)what does agglomeration mean.


CHAPTER 10: CARTOGRAPHY

It is worth noting that our eyes have evolved to be sensitive to the


wavelengths of solar radiation having the greatest intensities (visible
radiation). If early humans had been solely dependent upon hunting
nocturnal mammals (warm-blooded), evolutionary changes might have
resulted in `eyes' sensitive to infrared radiation (heat), just like those of some
reptiles. Despite our inability to naturally see infrared energy, we have
developed artificial detectors (including infrared film) capable of extending
our vision into the infrared part of the electromagnetic spectrum.

One type of infrared imaging employs satellites to detect infrared


radiation emitted by Earth's surface and atmosphere. From these images, the
temperatures of objects emitting radiation, such as clouds, large bodies of
water, and various landforms, can be established. Recall that the
wavelengths of radiation emitted by an object are temperaturedependent.
When more of the energy is emitted in shorter wavelengths, it translates to
objects at higher temperatures, whereas longer wavelength indicates surfaces
at lower temperatures.

One technique uses infrared images to determine which clouds are the
most probable precipitation producers. The high tops of towering
thunderstorms are colder that the tops of other clouds.

Unlike photos produced by exposing film to visible light, infrared images


can be made at any time of the day or night. The military uses infrared
imaging in their `night vision' technology to detect such things as troop
movements made under the cover of darkness.

178.Human eyes cannot see heat because

(a)heat is not light.


(b)infrared radiation is of longer wavelength than visible spectrum.

(c)infrared radiation is of shorter wavelength than visible spectrum.

(d)None of these

179.The high tops of towering thunderstorms are colder because

(a)of high altitude.

(b)of low altitude.

(c)of the presence of ice crystals.

(d)Of the absence of ice crystals.

The choropleth mapping technique, which uses `ranges' or `graduated


colour', is a type of thematic mapping that focuses usually on a single theme
with data summarized by statistical or administrative areas. The name of this
technique is derived from the Greek words choros (meaning space) and pleth
(meaning value).

Although almost all types of analyses using geographically related data


are exploratory, the term exploratory data analysis (EDA) has a very specific
meaning. EDA is a simple and relatively new approach (-25 years old)
developed in statistics. It involves testing whether the distribution of data is
uniform or normal, and if there are any extreme values or outliers. EDA also
incorporates visualizing and analysing the patterns in distribution of data
using such graphical tools as histograms, box plots or the quantile-quantile
(Q-Q) plots. Other typical EDA tools include using factoring variables,
stem-and-leaf plots, spread versus level plots or testing homogeneity of
variances.
EDA should precede any other type of analysis. A histogram helps
visualize the statistical distribution of a given variable, whereas a map can
help understand its geographical distribution. These two tools (histogram
and map) are related. Selecting the most appropriate mapping method
depends on the histogram.

There are two statistical distributions that are most suitable for two
mapping methods. The normal distribution is the most appropriate for using
the standard deviation classification, whereas the uniform distribution is the
most suitable for using the even class width classification. The uniform
distribution is very rare in a real world. A variable having perfectly uniform
distribution is not very interesting from the analytical point of view. Such a
variable would have the same value in every record, the histogram would
have all bars even and a choropleth map would have all polygons shaded
with the same colour. The normal distribution is one of the most common in
a real world.

Source: http://www.directionsmag.com/ articles/ choropleth-mapping-with-


exploratorydata-analysis/123579

180.Uniform distribution is boring because

(a)it is not normal.

(b)it is not variable.

(c)there is almost nothing to analyse.

(d)All of these

181.EDA is

(a)a tool.
(b)a process.

(c)a technique.

(d)All of these

182.Even class width classification can be used with

(a)uniform distribution.

(b)normal distribution.

(c)both (a) and (b).

(d)neither (a) nor (b).

183.EDA can

(a)identify extreme values.

(b)only determine the distribution of the sample.

(c)not be used for uniform distribution without difficulty.

(d)All of these

184.Choice of mapping method depends on

(a)histogram used.

(b)the type of distribution.

(c)both (a) and (b).

(d)neither (a) nor (b).


By the 101 century, techniques and instruments for precise observation and
measurement of physical quantities were well-developed-the beginnings of
the husbandry of visualization. The 171 century saw great new growth in
theory and the dawn of practice-the rise of analytic geometry, theories of
errors of measurement, the birth of probability theory and the beginnings of
demographic statistics and `political arithmetic'. Over the 18' and 19'
centuries, the numbers pertaining to peoplesocial, moral, medical and
economic statistics-began to be gathered in large and periodic series;
moreover, the usefulness of these bodies of data for planning, for
governmental response and as a subject worth of study in its own right began
to be recognized.

This birth of statistical thinking was also accompanied by a rise in visual


thinking: diagrams were used to illustrate mathematical proofs and
functions; nomograms were developed to aid calculations; various graphic
forms were invented to make the properties of empirical numbers-their
trends, tendencies and distributions-more easily communicated or accessible
to visual inspection. As well, the close relation of the numbers of the state
(the origin of the word `statistics') and its geography gave rise to the visual
representation of such data on maps, now called `thematic cartography'.

Maps, diagrams and graphs have always been (and continue to be) hard to
produce, still harder to publish. Initially, they were hand

drawn piece-by-piece. Later, they were etched on a copperplate and


manually coloured. Still later, lithography and photo-etching, and most
recently, computer software was used, but graphic makers have always had
to struggle with the limitations of available technologyand still do today.
Some noteworthy places in the history of visualization must therefore be
reserved for those who contributed to the technology.
Most recently, advances in statistical computation and graphic display
have provided tools for visualization of data unthinkable only a half century
ago. Similarly, advances in human-computer interaction have created
completely new paradigms for exploring graphical information in a dynamic
way, with flexible user control.

While most of the recent contributions relate to the visual display of


statistical data, there hasalso been considerable interplay with advances in
information visualization more generally, particularly for the display of large
networks, hierarchies, databases, text and so forth, where problems of very
large-scale data present continuing challenges.

185.The title of the passage can be

(a)Development of Cartography.

(b)Development of Statistics.

(c)Development of Printing Technology.

(d)Problems Faced in Visualization.

186.Which of the following statement is true in light of the passage'?

(a)Visual representation was born out of the development of statistical


thinking.

(b)Statistics and cartography developed independently.

(c)Cartography helped in rise of statistical thinking.

(d)None of these
187.The statement, `they... struggle with the technology' means that

(a)the gap between the technology and the designer's idea is reducing.

(b)the development in the ideas was faster than the development in the
technology.

(c)the designers did not have the technology to convert their ideas into
realties.

(d)the gap between the technology and the designer's idea is increasing.

188.Why was the need to develop visual tools felt?

(a)To help in communication.

(b)To help in visual inspection.

(c)Both (a) and (b)

(d)Neither (a) nor (b)

189.According to the author

(a)advances in the technology have come to its zenith.

(b)birth of visualization took place in the 16th century.

(c)problem of visualizing very largescale data still continues.

(d)None of these

The next step is to learn how to work with the data so that it is in the correct
form for the type of mapping method you are using.
To correct the problems caused by mapping counts using the choropleth
method, you can convert the data to the correct type so it can be shown by
lightness within areas. This is often necessary for data represented as points,
lines or rasters and with other mapping methods such as proportional circle.

Do this by normalizing or standardizing the data. These two terms, often


used interchangeably, are slightly different. Normalizing the data means
scaling all numerical values to a range from zero to one, while
standardization transforms the data so that is has zero mean and unit
variance. Both techniques have drawbacks. If the dataset has outliers,
normalizing will scale the normal data to a very small interval. When using
standardization, the assumption is that the data has been generated with a
certain mean and standard deviation, although this may not be the case.

In mapping, cartographers often use the term `derived data' to refer to data
that has been transformed through normalization or standardization so it can
be compared

in a meaningful way. Transformations commonly used in mapping include


ratios or rates, proportions, percentages and densities.

It is important to differentiate between spatially intensive and spatially


extensive measures. Density is a spatially extensive measure. A proportion,
generated by dividing the number of items in a unit by the total number of
items, is spatially extensive because the number per unit has been divided by
a constant (the total number of things). For derived values such as
proportions, percentages and rates, the resulting numbers can only be true
for the entire unit, not parts of it. For units of intrinsic importance (e.g.,
counties), mapping the proportion of the value allocated to each unit should
not be mapped using the choropleth method. In such cases, it may be best to
use graduated symbols.
Source: http://www.esri.com/esri-news/ arcuser/winter-2013/ understanding-
statistical-datafor-mapping-purposes

190.Spatially intensive measures

(a)are only true when they represent the unit as a whole.

(b)are not derived from spatially extensive data.

(c)do not depend on the size of the unit.

(d)None of these

191.For using choropleth method, the data can be corrected by

(a)normalizing.

(b)standardizing.

(c)both (a) and (b).

(d)neither (a) nor (b).

192.Spatially extensive measures

(a)are only true when they represent the unit as a whole.

(b)are not derived from spatially extensive data.

(c)do not depend on the size of the unit.

(d)cannot account for outliers.

193.`Mapping the... choropleth method' because


(a)the values are spatially intensive.

(b)the values are spatially extensive.

(c)the result will not be colourful.

(d)the data is not under uniform distribution.

194.Which of the following data will be derived data'?

(a)The data transformed by taking mean

(b)The data transformed by standardization

(c)The data transformed by taking ratio

(d)All of these

The second research paradigm that MacEachren outlined is visual cognition,


where existing knowledge in the mind of the map user is employed to
interpret visual scenes through knowledge schemata that act as an interface
between what is seen by the eye and what is understood in the brain. There
are three kinds of knowledge schemata that most adult humans possess. The
first are propositional schemata, which can be seen in geographical terms as
declarative knowledge, knowledge about geographical objects, attributes of
those objects and attributes of places. The second are image schemata, which
represent the organization of configural knowledge about space, knowledge
of spatial relationships among entities in space. The third are event
schemata, procedural knowledge of the sequence of steps needed to get from
one place to another. They are all applicable to the understanding of maps,
but image schemata and event schemata are the most important in
relationship to creating maps that work well.
Image schemata are the most important of the three in the understanding
of the majority of map types. Based on current psychological research,
humans do not actually see `pictures in the head'. But evidence suggests that
we understand and store the meaningful parts of a visual scene using the
same geometrical, symbolic and minimal vocabulary found in maps.
Embodied image schemata are some the most fundamental schemata that
people possess. They are schemata that come directly from human
experience with the environment. All of the embodied image schemata are
believed

to be, more or less, the result of pre-attentive mental processing. That is,
they elicit mental images before one can bring existing knowledge to bear in
understanding a visual scene. For cartography, the categories defined in
maps are typically understood through container schemata. Hierarchical
structure in maps is understood through part whole and up-down schemata.
Foreground (or figure)-background in maps is understood through front-back
embodied image schemata, while linear quantity scales on map legends are
understood in terms of up-down and linear order schemata.

195.Which of the following schemata is least important for the map maker'?

(a)Image schemata

(b)Provisional schemata

(c)Event schemata

(d)Cannot be determined from the passage

196.The visual cognition paradigm is

(a)based on psychological research.


(b)better than the old paradigms.

(c)failed to account for paradigm paralysis.

(d)All of these

197.The importance of the image schemata is that

1.the maps will be more effective if the same schemata is used in creation
and interpretation of the maps.

2.the maps will be counter-intuitive if the schema used for the creation of
the map is different from the one used for interpretation of the same.

(a)Only 1

(b)Only 2

(c)Both 1 and 2

(d)Neither 1 nor 2

198.The term pre-attentive mental processing means that

(a)a person is not even aware of the process.

(b)a person is not actively making the process happen.

(c)the process is taking place in subconscious mind.

(d)All of these

Both proponents and critics have represented GIS as an arm of spatial


analysis in the service of positivist geography. Three closely associated
attributes of GIS practices are highlighted in this representation: that they are
quantitative, empiricist and positivist. It is seemingly selfevident that GIS is
quantitative and empiricist, given its computational roots in Boolean
mathematics and its use for manipulating empirical spatial databases.
Mathematical precision and empirical accuracy are valued aspects of GIS,
even though its capabilities for mathematical and statistical analysis remain
limited by comparison with those of quantitative geographic practices more
generally. Attached to this image is that of GIS as a tool of positivist
geographic science. As John Pickles (1995) has documented, proponents of
GIS have stressed its parallels to positivist spatial analysis and its links to
hypothesis testing, deductive logic and value neutrality. Critical social
theorists agree, representing GIS as a positivist Trojan horsesmuggling
geographicus quantifactus back in under the cover of darkness, just when
Helen seemed to have been saved from the positivist siege. The
representational limits of GIS itself also have been a common target of
postpositivist critiques: its preoccupation with Newtonian concepts of space,
its Cartesian `view from nowhere' approach to mapping and the presumption
that the world can be accurately captured using Boolean algorithms
combined with translation rules linking abstract variables to real-world
objects (an artificial intelligence perspective). These arguments parallel
postpositivist critiques of quantitative geographic practices more generally.

From a postpositivist perspective, the dangers of positivist GIS as an


epistemological program for geography are associated with an equally
dangerous disregard for the social and ethical implications of GIS: its use for
surveillance; its association with the military; its inaccessibility to
disadvantaged communities and its ability to shape forms of life and the
places where these are lived. Notwithstanding a considerable rapprochement
between certain GIS practitioners and social theoretic critics,
many GIS researchers have neither engaged with critiques of GIS as a
positivist practice, nor taken much interest in the social consequences of GIS
practices. Such attitudes are, in this view, consistent with positivist
conceptions of the role of science; other epistemologies are a distraction
from positivism's foolproof methods for seeking the truth, and furthermore
the practice of science should not be sullied by political considerations.

199.GIS is considered to be a positivist tool because

(a)it considers the value of the variables to be neutral.

(b)it considers variables to be spatially independent.

(c)it is focused of creation of artificial intelligence.

(d)All of these

200.Why is GIS considered to be a Trojan horse?

(a)It fools people into believing what it is not.

(b)It relies on empirical accuracy.

(c)It is an epistemological program.

(d)It is seen as an attempt to insert positivist perspective into


postpositivist world.

201.The social and ethical implications of GIS are considered by

(a)positivists.

(b)postpostivists.
(c)empiricists.

(d)All of these

202.Which of the following will consider science and political consideration


to be separate'?

(a)Positivists

(b)Postpostivists

(c)Both (a) and (b)

(d)Neither (a) nor (b)

203.Postpostivists oppose

1.quantitative geography.

2.qualitative geography.

3.empirical approach.

(a)1 and 2

(b)2 and 3

(c)3 and 1

(d)1, 2 and 3

CHAPTER 11: STATISTICAL METHODS

Models are validated by comparing observed data with what is expected. If


the model is a good representation of reality, there will be a close match
between the two. If observations and expectations are far apart, we need to
`go back to the drawing board' and come up with a new hypothesis. It might
be the case, e.g., the pattern can simply be due to the fact that the population
itself is clustered.

Though models are often used to learn about particular situations, more
often one also wishes to learn about the underlying process that led to it. We
would like to be able to generalize from one study to statements about other
situations. One reason for studying the spatial pattern of cancer cases is to
determine whether there is a relationship between cancer rates and the
distance to specific power plants; amore general objective is to learn about
the relationship between cancer rates and the distance to any power plant.
One way of making such generalizations is to accumulate a lot of evidence.
If we were to repeat our analysis in many locations throughout a country,
and if our findings were similar in all cases, we would have uncovered an
empirical generalization. In a strict sense, laws are sometimes defined as
universal statements of unrestricted range. In our example, our
generalization would not have unrestricted range, and we might want, e.g., to
confine our generalization or empirical law to power plants and cancer cases
in the country of interest.

Einstein called theories `free creations of the human mind'. In our context,
we may think of theories as collections of generalizations or laws. The
whole collection is greater than the sum of its parts in the sense that it gives
greater insight than that produced by the generalizations or laws alone. If,
e.g., we generate other empirical laws that relate cancer rates to other
factors, such as diet, we begin tobuild a theory of the spatial variation in
cancer rates.

204.In which of the following cases do we need to `go back to the drawing
board' and come up with a new hypothesis?
(a)When there is difference between the purposed hypothesis and data
observed.

(b)Where data observed confirms the purposed hypothesis.

(c)Where the hypothesis is confusing.

(d)Where the data observed is questionable.

205.Empirical laws

(a)must have unrestricted range.

(b)are based on lot of observation.

(c)can be made without a lot of observation.

(d)must have restricted range.

206.Theories can be defined as

(a)collection of related empirical laws.

(b)collection of unrelated empirical laws.

(c)either (a) or (b).

(d)both (a) and (b) together.

207.An empirical law in the strict sense should have

(a)restricted range.

(b)unrestricted range.
(c)related data.

(d)None of these

There are at least two ways to go about answering the question of whether
the coin is a fair one. One is to ask what would happen if the coin was fair
and to simulate a series of experiments. That is, if we could repeatedly flip a
known fair coin 10 times, each time recording the number of heads, we
would learn just how unusual a total of eight heads actually was. If eight
heads comes up quite frequently with the fair coin, we will judge our
original coin to be fair. On the other hand, if eight heads is an extremely rare
event for a fair coin, we will conclude that our original coin is not fair.

To pursue this idea, suppose you arrange to carry out such an experiment
100 times. For example one might have 100 students in a large class; each
flip a coin that is known to be fair ten times. Upon pooling together the
results, suppose you see in the result that eight heads occurred 8% of the
time. We still need a guideline to tell us whether our observed outcome of
eight heads should lead us to the conclusion that the coin is (or is not) fair.
The usual guideline is to ask how likely a result equal to or more extreme
than the observed one is, if our initial, baseline hypothesis that we possess a
fair coin (called the null hypothesis) is true. A common practice is to accept
the null hypothesis if the likelihood of a result at least as extreme as the one
we observed is more than 5 per cent. Hence, we would accept the null
hypothesis of a fair coin if our experiment showed that eight or more heads
was not uncommon and in fact tended to occur more than 5 per cent of the
time.

Alternatively, we wish to reject the null hypothesis that our original coin
is a fair one if the results of our experiment indicate that eight or more heads
out of 10 is an uncommon event for fair coins. If fair coins give rise to eight
or more heads less than 5 per cent of the time, we decide to reject the null
hypothesis and conclude that our coin is not fair.

In the example above, if eight or more heads occurred 12 times out of 100,
when afair coin was flipped 10 times. The fact that events as extreme, or
more extreme than the one we observed, will happen 12 per cent of the time
with a fair coin leads us to accept the inference that our original coin is a fair
one. Had we observed nine heads with our original coin, we would have
judged it to be unfair, since events that are rare or more rare than this
(namely where the number of heads is equal to 9 or 10) occurred only four
times in the 100 trials of a fair coin. Note, too, that our observed result does
not prove that the coin is unbiased.

208.Why can we not prove that the coin is unbiased?

(a)Because the result only gives likelihood of it being biased or unbiased.

(b)Because the experiment itself is flawed.

(c)Because the result is in form of spatial extremism.

(d)None of these.

209.The range of 5 percent for null hypothesis is

(a)a universally acceptable standard.

(b)an unqualified assumption.

(c)a qualified assumption limited to the example given.

(d)a qualified assumption limited to all the examples of unfair coins.


210.The writer is mainly discussing

1.descriptive statistical analysis.

2.inferential statistical analysis.

3.setting of null hypothesis.

(a)Only 1

(b)Only 2

(c)1 and 2

(d)2 and 3

211.Which of the following statements is true in the light of the passage?

(a)The null hypothesis need not be tested against a given guideline.

(b)The whole example is faulty as the result does not prove anything.

(c)Rarity of the event cannot be measured.

(d)None of these

Study areas are bounded, and it is important to recognize that events just
outside the study area can affect those inside of it. If we are investigating the
market areas of shopping malls in a county, it would be a mistake to neglect
the influence of a large mall located just outside the county boundary. One
solution is to create a buffer zone around the area of study to include features
that affect analysis within the primary area of interest.
Both the size and shape of areas can affect measurement and
interpretation. There are a lot of migrants leaving Rhode Island each year,
but this is partially due to the state's small size-almost any move will be a
move out of the state! Similarly, Tennessee experiences more out-migration
than other states with the same land area, in part, because of its narrow
rectangular shape. This is because individuals in Tennessee live, on average,
closer to the border than do individuals in other states with the same area. A
move of given length in some random direction is therefore more likely to
take the Tennessean outside of the state.

Spatial autocorrelation refers to the fact that the value of a variable at one
point in space is related to the value of that same variable in a nearby
location. The travel behaviour of residents in a household is likely to be
related to the travel behaviour of residents in nearby households because
both households have similar accessibility to other locations. Hence,
observations of the two households are not likely to be independent, despite
the requirement of statistical independence for standard statistical analysis.
Spatial autocorrelation (or spatial dependence) can therefore have serious
effects on statistical analyses and hence lead to misinterpretation.

212.The author uses the example of Rhode Island to explain

(a)wrong interpretation based on faulty measurement.

(b)wrong interpretation based on correct measurement.

(c)right interpretation based on faulty measurement.

(d)right interpretation based on correct measurement.

213.The Tennessee example shows


(a)result of faulty measurements.

(b)result of faulty interpretations.

(c)danger of interpretation outside the given context.

(d)danger of making measurements outside the given context.

214.The problem faced in Rhode Island resulted

(a)due to spatial dependence.

(b)due to spatial auto correction.

(c)due to interdependence of the sample.

(d)None of these.

215.Spatial dependence results in

(a)faulty measurement.

(b)faulty interpretation.

(c)auto correction of error.

(d)None of these.

216.The bounded problem can be corrected if

(a)we remember that the factors outside the study area can influence the
study area.

(b)we restrict the interpretation to the context.


(c)we make sure that our measurements are unbounded.

(d)All of these.

One of the basic questions geographers face is whether geographic data


exhibits spatial patterns. This is important both in its own right (where, e.g.,
we may wonder whether crime locations are more geographically clustered
than they were in the past), and in addressing the fundamental problem of
spatial dependence in geographic data when carrying out statistical tests.
With respect to the latter, inferential statistical tests almost always assume
that data observations are independent; this, however, is often not the case
when data are collected at geographic locations. Instead data are often
spatially dependent-the value of a variable at one location is likely to be
similar to the value of the variable at a nearby location. This characteristic of
spatial data is often referred to as Tobler's First Law of Geography.

217.For finding out if the data exhibits spatial pattern, we can use

(a)standard deviation.

(b)mean.

(c)regression.

(d)mode.

218.The assumption of inferential statistical tests that the data observations


at a geographic location is independent is

(a)true.

(b)false
(c)partially true.

(d)Cannot be determined from the passage.

219.Tobler's first law of geography states that

(a)The assumptions made by inferential statistics are unreliable.

(b)The data observed at one geographic location will be similar to the


data observed at nearby geographical location.

(c)The data is spatially extensive.

(d)All of these.

ANSWER KEYS
 
1.`Port of Portugal' is a variety of

(a)tobacco.

(b)fish.

(c)fruit.

(d)wine.

[UPSC-1993]

2.A telegram was sent from Greenwich at 12 noon. The time for
transmission was 12 minutes. It reached a town at 6.00p.m.

(a)97°B

(b)81 °E

(c)8.7°W

(d)91°W

[UPSC-1994]

3.The fuel driving the weather is

(a)convection.
(b)inertia of motion.

(c)gravity.

(d)solar energy.

4.In 2004, China surpassed as the world's leading exporter of technology and
telecommunications goods.

(a)India

(b)the United States

(c)Korea

(d)Japan

5.Foreign direct investment means

(a)that foreigners invest directly in domestic industry.

(b)investment in enterprises that are actually operated by a foreign owner.

(c)that a government invests in foreign governments.

(d)that direct investment in industry is a foreign ideal.

6.Which of the following factors plays a relatively small role in the amount
of solar energy received at a particular place on the Earth?

(a)The season of year

(b)Distance of the Earth from the Sun

(c)The length of day


(d)The angle of the Sun

7.Among the following environmental pollutants, which one leads to the


occurrence of acid rain'?

(a)Carbon dioxide

(b)Hydrogen peroxide

(c)Carbon monoxide

(d)Sulphur dioxide

[UPSC-2006]

8.Among the following south Asian countries, which one has the highest
level of urbanization?

(a)Sri Lanka

(b)Bangladesh

(c)Pakistan

(d)Nepal

[UPSC-2008]

9.Among the following, which one has the largest number of land-bordering
neighbouring countries?

(a)Brazil

(b)Russia
(c)Germany

(d)Zambia

[UPSC-2007]

10.Among the following, which one has the largest number of neighbouring
countries sharing the land border'?

(a)Sudan

(b)Brazil

(c)China

(d)Zaire

[UPSC-2009]

11.Animal rearing on a commercial scale has developed well in

(a)monsoon regions.

(b)Savannah region.

(c)Sahel region.

(d)Prairie and Steppe region.

[UGC D-2005]

12.Arakan and Pegu Yomas are located in

(a)Myanmar.

(b)Nepal.
(c)Bhutan.

(d)China.

[UPSC-2009]

13.Assertion (A): Thunderstorm activity in India is characteristic of the pre-


and post-monsoon periods.

Reason (R): These are of the periods of transition between the seasonal
circulations.

(a)Both (A) and (R) are true and (R) is the correct explanation of (A).

(b)Both (A) and (R) are true but (R) is not the correct explanation of (A).

(c)(A) is true but (R) is false.

(d)(A) is false but (R) is true.

[UPSC-1993]

14.Assertion (A): Karst topography is characterized by the formation of


stalactites and stalagmites.

Reason (R): Solution is a dominant process in the development of


landforms in Karst region.

(a)Both (A) and (R) are true and (R) is the correct explanation of (A).

(b)Both (A) and (R) are true but (R) is not the correct explanation of (A).

(c)(A) is true but (R) is false.

(d)(A) is false but (R) is true.


[UPSC-1994]

15.Assertion (A): About 80% of Canadian manufacturing is concentrated in


South Ontario and St. Lawrence lowland regions.

Reason (R): There are abundant deposits of coal in the South Ontario and
St. Lawrence lowland region.

(a)Both (A) and (R) are true and (R) is the correct explanation of (A).

(b)Both (A) and (R) are true but (R) is not the correct explanation of (A).

(c)(A) is true but (R) is false.

(d)(A) is false but (R) is true.

[UPSC-1993]

16.Assertion (A): Cotton cultivation in Egypt is largely concentrated in the


upper Nile Valley.

Reason (R): Cotton cultivation requires fertile clayey soil, abundant


sunshine, frost-free weather and cheap labour.

(a)Both (A) and (R) are true and (R) is the correct explanation of (A).

(b)Both (A) and (R) are true but (R) is not the correct explanation of (A).

(c)(A) is true but (R) is false.

(d)(A) is false but (R) is true.

[UPSC-1993]
17.Assertion (A): The Great Dividing Range of Australia is an example of a
block mountain.

Reason (R): It was uplifted by the Alpian Earth movements.

(a)Both (A) and (R) are true and (R) is the correct explanation of (A).

(b)Both (A) and (R) are true but (R) is not the correct explanation of (A).

(c)(A) is true but (R) is false.

(d)(A) is false but (R) is true.

[UPSC-1993]

18.Assertion (A): The Mediterranean regions of the Northern Hemisphere


specialize in horticulture.

Reason (R): The climatic conditions in the Mediterranean regions are


favourable.

(a)Both (A) and (R) are true and (R) is the correct explanation of (A).

(b)Both (A) and (R) are true but (R) is not the correct explanation of (A).

(c)(A) is true but (R) is false.

(d)(A) is false but (R) is true.

[UPSC-1993]

19.Consider the following cities:

1.Antwerp
2.Maasticht

3.Rotterdam

Which of the above cities has/have port(s) connected to North Sea?

(a)1 only

(b)2 and 3

(c)1 and 3

(d)1, 2 and 3

[UPSC-2008]

20.Consider the following factors:

1.Convergence of cool and warm ocean currents

2.Shallow depth of ocean waters

3.Nearness of fishing posts

4.Abundance of drifting planktons

Those responsible for the location of the great fishing groups would
include

(a)1 alone.

(b)1 and 2.

(c)2, 3 and 4.
(d)1, 2 and 4.

[UPSC-1994]

21.Consider the following pairs:

Which of the statements given above is/ are correct'?

(a)1 and 2

(b)3 and 4

(c)1, 2 and 4

(d)1, 2, 3 and 4

[UPSC-2008]

22.Consider the following rivers:

1.Don

2.Syr Darya

3.Volga

4.Ural

Which of the above rivers flow into the landlocked Caspian Sea?

(a)1 and 2
(b)2 and 3

(c)3 and 4

(d)1 and 4

[UPSC-2008]

23.Consider the following statements:

1.China spans five time ones but runs on a single national time using the
standard meridian of Beijing.

2.Russia uses eight standard time zones.

Which of the statements given above is/ are correct'?

(a)1 only

(b)2 only

(c)Both 1 and 2

(d)Neither 1 nor 2

[UPSC-2006]

24.Consider the following statements:

1.San Francisco

2.Adelaide

3.Cape Town
4.Sydney

Which of the statements given above is/ are correct'?

(a)l and 3

(b)2 and 4

(c)1, 3 and 4

(d)1, 2 and 3

[UPSC-2006]

25.Consider the following statements:

1.Board evergreen leaves

2.Shining, waxy leaf surface

Which of the above is/are the salient characteristics of nature vegetation


of Selves?

(a)1 and 2 only

(b)2 only

(c)1 and 3 only

(d)1,2and3

[UPSC-2006]

26.Consider the following statements:


1.Montreal Protocol in the year 1987 called for the complete cut in the
use of chlorofluorocarbons (CFCs) by the year 2000.

2.India was a signatory to the Montreal Protocol in the year 1987.

Which of the statements given above is/ are correct'?

(a)1 only

(b)2 only

(c)Both 1 and 2

(d)Neither 1 nor 2

[UPSC-2006]

27.Consider the following statements:

1.The boundary of a National Park is circumscribed by legislation


whereas that of a Biosphere Reserve is not circumscribed by
legislation.

2.In a national park, particular attention is paid to the conservation of


genepool, whereas it is not so in a wildlife sanctuary.

Which of the statements given above is/ are correct'?

(a)1 only

(b)2 only

(c)Both 1 and 2

(d)Neither 1 nor 2
[UPSC-2006]

28.Consider the following statements:

1.Mauna Loa is an active shield volcano.

2.Cinder Cones are common in the areas of East Africa.

3.The Ngorongoro Crater in Tanzania is a caldera.

Which of the statements given above is/ are correct'?

(a)1 and 2 only

(b)2 and 3 only

(c)1 and 3 only

(d)1, 2 and 3

[UPSC-2007]

29.During which of the following year was the National Water Policy
formulated in India'?

(a)2002

(b)1987

(c)2007

(d)1985

[UGC D-2010]

30.Find out the correct bottom-up hierarchy of growth foci in India.


(a)Growth centre, growth point, growth pole, service centre

(b)Growth pole, growth point, growth centre, service centre

(c)Service centre, growth point, growth centre, growth pole

(d)Growth pole, growth centre, growth point, service centre

[UGC D-2011]

31.From the viewpoint of land forms, which of the following pairs of


continents are mirror images of one another'?

(a)Africa-Australia

(b)Africa-South America

(c)Asia-Africa

(d)Australia-South America

[UPSC-1993]

32.The study of landforms and the processes that create them is called

(a)cartography.

(b)geomorphology.

(c)demography.

(d)seismography.

33.Earth's rocks, soils and surface landforms together comprise the


(a)hydrosphere.

(b)biosphere.

(c)troposphere.

(d)lithosphere.

34.Distortion is least severe on

(a)globes.

(b)large-scale maps.

(c)small-scale maps.

(d)all maps.

35.The original purpose of the Mercator projection was

(a)to befuddle introductory physical geography students.

(b)for ocean navigation.

(c)to make the first map of the world.

(d)to produce an accurate equal area map.

36.A culture hearth is

(a)a region from which innovative ideas originate.

(b)the material traits of a particular culture.

(c)the evolution of a landscape.


(d)a region in which people of different origins live.

37.Which is an example of contagious diffusion?

(a)The sharing of music preferences among a group of friends

(b)The construction of Buddhist temples in Los Angeles

(c)English colonists bringing their language to America

(d)The introduction of tulip farms in Michigan by Dutch immigrants

38.A city and its metropolitan area form an example of a

(a)climatic region.

(b)uniform region.

(c)functional region.

(d)nodal region.

39.The study of the distribution of objects across Earth's surface and the
processes by which human and environmental phenomena move from one
place to another is

(a)area analysis.

(b)biodiversity.

(c)spatial analysis.

(d)community succession.

40.Longitude and latitude are measured using the unit terms of


(a)degrees, minutes.

(b)degrees, hours, minutes, seconds.

(c) degrees, minutes, seconds.

(d)miles, feet, inches.

41.The meridian of 0° longitude is also called the

(a)Equator.

(b)Prime Meridian.

(c)International Date Line.

(d)90° meridian.

42.The value of total outputs of goods and services in a year in a country is


its

(a)productivity.

(b)level of development.

(c)gross domestic product.

(d)value added.

43.Gross domestic product

(a)is a very good indicator of a state's economic level.

(b)is the total value of all goods and services produced in a country.
(c)is 95 percent of the goods produced in a country divided by the
population.

(d)includes foreign investment.

44.Compared to relatively developed countries, developing countries have a


higher percentage of workers in which sector of the economy?

(a)Tertiary

(b)Quaternary

(c)Secondary

(d)Primary

45.About percent of Canada remains in territorial status.

(a)60

(b)25

(c)40

(d)75

(a)Kurds.

46.An example of a nationality divided among more than one state would be
the

(b)Japanese.

(c)Belgians.
(d)United States.

47.Given below are two statements, one labelled as Assertion (A) and the
other labelled as Reason (R). Select correct answer from the codes given.

Assertion (A): Biomass energy sources are renewable resources lying in


the critical limits.

Reason (R): Biomass can be replenished over a time period.

(a)Both (A) and (R) are true and (R) is the correct explanation of (A).

(b)Both (A) and (R) are true, but (R) is not the complete explanation of
(A).

(c)(A) is true, but (R) is false.

(d)(A) is false, but (R) is true.

[UGC D-2010]

48.An example that comes closest to being a true nation-state would be

(a)Germany.

(b)Iceland.

(c)China.

(d)the United States.

Human geography has used absolute, relative and relational concepts of


space. In the absolute conception, space is considered a distinct physical,
empirical and real entity. In the relative conception, space is a relation
between events and thus related to time and process. In the relational
conception, space is not viewed in term of its geometry, but in terms of how
spatial relations are carved out through human practice, that is through
spatial organization.

49.Morphological studies adopt which concept of space?

(a)Absolute

(b)Relational

(c)Relative

(d)Both (a) and (c)

[UGC D-2007]

50.Spatial organization is primarily based on

(a)spatial geometry.

(b)spatial process.

(c)spatial relations.

(d)spatial-social dialectic.

[UGC D-2007]

1.Given below are two statements, one labelled as Assertion (A) and the
other labelled as Reason (R). Select your answer from the codes given.
Assertion (A): Between the two World Wars, the whole process of
suburban growth and decentralization started to speed up.

Reason (R): The forces behind these are partly economic due to great
depressions, partly social due to rise of middle class and partly
technological due to rapid development in transport system.

(a)Both (A) and (R) are correct and (R) explains (A).

(b)Both (A) and (R) are correct, but (R) does not explain (A).

(c)(A) is correct, but (R) is wrong.

(d)(A) is wrong, but (R) is correct.

[UGC D-2011]

2.The axis of rotation of the Earth is inclined how many degrees away from
being perpendicular to the Sun's rays?

(a)23.5°

(b)450

(c)66.5°

(d)90-

3.At the Tropic of Cancer, the Sun is directly overhead at noon on the

(a)summer solstice.

(b)winter solstice.

(c)autumnal equinox.
(d)vernal equinox.

4.Given below are two statements, one labelled as Assertion (A) and the
other labelled as Reason (R). Select your answer from the codes given.

Assertion (A): Great Lakes region of USA is well-developed with


industries.

Reason (R): This region was the region of earlier settlement in USA by
migrants from Britain.

(a)Both (A) and (R) are true and (R) is the correct explanation of (A).

(b)Both (A) and (R) are true, but (R) is not the correct explanation of (A).

(c)(A) is true, but (R) is false.

(d)(A) is false, but (R) is true.

[UGC D-2010]

5.If the highest point in locality is 1,500 metres and the lowest is 1,000
metres, 500 metres represents the

(a)height above base level.

(b)gradient.

(c)elevation.

(d)relative relief.

[UPSC-1993]
6.In which one of the following situations in South-East Asia are the largest
concentrations of peasant population found?

(a)On lowland forest areas cleared for plantation agriculture

(b)On highland areas where the climate is cooler and healthier

(c)On lowland areas where grain farming is carried on

(d)In the mining areas

[UPSC-1993]

7.Intensive subsistence farming is characteristic of areas of

(a)high density of population and high technology.

(b)low density of population and high technology.

(c)low density of population and low technology.

(d)high density of population and low technology.

[UPSC-1994]

8.Iron is obtained mostly from

(a)haematite ores.

(b)magnetite ores.

(c)siderite ores.

(d)iron pyrites.

[UPSC-1994]
9.Lahaul and Spiti district in Himachal Pradesh has the highest

(a)agricultural density of population.

(b)arithmetic density of population.

(c)physiological density of population.

(d)rural density of population.

[UPSC-1994]

10.Lines joining places of same total transportation costs in Weber's theory


are termed

(a)isotims.

(b)isogonic lines.

(c)osodapanes.

(d)isochrones.

[UPSC-1993]

11.Match List I with List II and select the correct answer from the codes
given.
Codes:

[UGC J-2011]

12.Match List I with List II and select the correct answer from the codes
given.
Codes:

[UGC D-2011]

13.Match List I with List II and select the correct answer using the codes
given.

Codes:

[UGC J-2011]
14.Match List I with List II and select the correct answers from the codes
given.

Codes:

[UGC J-2011]

15.Match List I with List II and select the correct answer using the code
given.

Codes:
[UPSC-2008]

16.Match List I with List II and select the correct answer using the code
given.

Codes:

[UPSC-2007]
17.Match List I with List II and select the correct answer using the code
given.

Codes:

[UPSC-2007]

18.Match List I with List II and select the correct answer using the codes
given.
Codes:

[UPSC-1994]

19.Match List I with List II and select the correct answer using the codes
given.

Codes:

[UPSC-1994]

20.Match List I with List II and select the correct answer using the codes
given.
Codes:

[UPSC-1994]

21.Match List I with List II and select the correct answer using the codes
given.

Codes:
[UPSC-1994]

22.Match List I with List II and select the correct answer using the codes
given.

Codes:

[UPSC-1994]

23.Match List I with List II and select the correct answer using the codes
given.
Codes:

[UPSC-1993]

24.Match List I with List II and select the correct answer using the codes
given.
Codes:

[UPSC-1993]

25.Match List I with List II and select the correct answer using the codes
given.

Codes:

[UPSC-1993]

26.Match List I with List II and select the correct answer using the codes
given.
Codes:

[UPSC-1993]

27.Match List I with List II and select the correct answer using the codes
given.
Codes:

[UPSC-1993]

28.Match List I with List II and select the correct answer using the codes
given.
Codes:

[UPSC-1993]

29.Match List I with List II and select the correct answer using the codes
given.

Codes:

[UPSC-1993]

30.Endogenic processes
(a)result from wind.

(b)include running water.

(c)create volcanoes.

(d)are external.

31.Exogenic processes

(a)create volcanoes.

(b)include running water.

(c)are internal.

(d)result from earthquakes.

32.The rock in the interior of Earth's surface that carries slow-moving


convective currents is known as the

(a)mantle.

(b)shield.

(c)crust.

(d)core.

33.Volcanoes

(a)might erupt runny lava that cools to form basalt.

(b)issue sial.
(c)are randomly distributed.

(d)are absent from the Eastern Hemisphere.

34.In the South-eastern USA, which climate is dominant'?

(a)BS

(b)Dwc

(c)Cfa

(d)AW

35.Marine westcoast climates are NOT

(a)mild climates with a small annual temperature range.

(b)commonly found in low latitudes.

(c)plentifully moist year round.

(d)found in places like Kodiak, Alaska.

36.Which of the following climates would you expect to find in coastal


southern California?

(a)Mediterranean

(b)Humid continental

(c)Desert

(d)Humid tropical
37.Which gas, contributed in part by burning fossil fuels, is used by
scientists as an indicator of future global warming'?

(a)Hydrogen

(b)Nitrogen

(c)Oxygen

(d)CO2

38.The statement '1:24,000' is an example of

(a)a map.

(b)a bar scale.

(c)a fractional scale.

(d)a graphic scale.

39.The highest numerical measurement when calculating latitude is

(a)90°

(b)180-

(c)360°

(d)45-

40.Tertiary sector employment is increasing in poor countries where

(a)the tertiary sector is well integrated into the world economy.


(b)most people are subsistence farmers.

(c)there is an abundance of raw materials.

(d)most people are hunter gatherers.

41.In terms of the stage of its economic sectoral evolution, the USA is

(a)rapacious.

(b)post-industrial.

(c)industrial.

(d)extractive.

42.A society in which a majority of the workers are engaged in the tertiary
sector is said to be a(n) society.

(a)pre-industrial

(b)communist

(c)post-industrial

(d)industrial

43.The result of China's policy of moving industry inland between 1949 and
1976 was

(a)in land areas became rich, while the coast stagnated.

(b)the capital was moved inland.

(c)exports increased enormously.


(d)exports plummeted.

44.Enormous enterprises that own and coordinate production and marketing


facilities in several countries are called

(a)global corporations.

(b)transnational corporations.

(c)capitalist industries.

(d)offshore profit shifting.

45.Critical isodapane refers to

(a)the point of the least transport cost.

(b)the point of the maximum transport cost.

(c)the point of the minimum raw material cost.

(d)the point of the maximum raw material cost.

[UGC D-2011]

Perhaps Thomas S.Kuhn is right to claim that science is not a well-regulated


activity where each generation automatically builds upon the results
achieved by earlier workers, but a process of varying tension in which
tranquil periods characterized by a steady accretion of knowledge are
separated by crises which can lead to upheaval within subject disciplines and
breaks in continuity. While it is possible to determine objectively whether an
explanatory framework is satisfactory and reasonable within a specific
scientific tradition, we must select what Kuhn calls paradigms (models or
exemplars) for scientific activity.
However, Kuhn's model fits the development of geographical science only
superficially. As we have followed the development of the subject, we have
seen how new paradigms have, to some extent, included ideas from the older
paradigms. The paradigm concept therefore loses some of its clarity and
value as a guide for research until, in the end, more and more people define
geography as what geographers do. Despite the impressions, we may get
from simplified accounts, a closer look at the history of geography reveals
that complete revolutions have not taken place' paradigms or what may be
more appropriately termed schools of thought, continue to exist side by side.

46.In the light of the above passages, which of the following claims were
made by Kuhn'?

1.Science is not a well-regulated activity.

2.Science is a process of varying tensions.

3.Science is hereditary.

4.The growth of knowledge is steady.

(a)1 and 2

(b)1, 2 and 3

(c)1, 2 and 4

(d)2, 3 and 4

[UGC J-2007]

47.To which crisis has Kuhn referred to'?

(a)Problems in finding new solutions


(b)Opposition from fellow scientists

(c)New problems

(d)New breakthroughs

[UGC J-2007]

48.Why has it been said that Kuhn's model fits the development of
geography only superficially'?

(a)A partial continuity exists between different paradigms.

(b)Paradigm revolution is not complete.

(c)Paradigm revolution has started.

(d)None of these

[UGC J-2007]

49.Is the paradigm concept irrelevant to geography'?

(a)Yes

(b)No

(c)Partially

(d)Cannot be determined

[UGC J-2007]

50.Main idea contained in the passage is

(a)that the paradigm concept has failed.


(b)that the paradigm concept continues to be applicable.

(c)that the paradigm concept is superficial.

(d)that the paradigm concept is partially applicable to geography.

[UGC J-2007]

1.Match List I with List II and select the correct answer using the codes
given.

Codes:

[UPSC-1993]

2.The energy with wavelengths between 0.4 and 0.7 mm is known as

(a)X-rays.
(b)radio waves.

(c)visible light.

(d)infrared.

3.What best explains the movement of heat from tropical areas towards the
poles?

(a)Polar fronts

(b)Orographic fronts

(c)Specific heat

(d)Advection

4.Match List I with List II and select the correct answer using the codes
given.
Codes:

[UPSC-1993]

5.Match List I with List II and select the correct answer using the codes
given.

Codes:

[UPSC-1993]
6.Match List I with List II and select the correct answer from the codes
given.

Codes:

[UGC D-2010]

7.Northern part of Australia is typified by which one of the following'?

(a)Equatorial climate

(b)Mediterranean climate

(c)Temperate climate

(d)Monsoon climate

[UPSC-2007]
8.On the banks of which river is the city Kathmandu located'?

(a)Arun

(b)Saptakosi

(c)Bagmati

(d)Sunkosi

[UPSC-2009]

9.One of the important determinants of urban land value is

(a)climate.

(b)historical factors.

(c)topography.

(d)accessibility.

[UGC D-2010]

10.Pittsburgh, as a centre of the iron and steel industry, has a great


disadvantage in respect of

(a)availability of cooking coal.

(b)supply of iron ore.

(c)transportation.

(d)market.

[UPSC-1993]
11.The `Horn of Africa' includes

(a)Namibia, Botswana and Republic of South Africa.

(b)Ethiopia, Djibouti and Somali Republic.

(c)Senegal, Guinea, Sierra Leone and Liberia.

(d)Libya, Egypt and Sudan.

[UPSC-1994]

12.The age and sex composition of the population is best represented by

(b)isopleths.

(a)choropleths.

(c)pie diagram.

(d)pyramid diagram.

[UPSC-1993]

13.The alluvial soils of the Gangetic Valley are described as

(a)intrazonal.

(b)azonal.

(c)zonal.

(d)None of the above.

[UPSC-1993]
14.The American cotton textile industry moved from New England to the
southern states because

(a)the latter have a more humid climate.

(b)New England diversified its manufacturing industries.

(c)the population growth in the south provided a major market.

(d)the south offered cheaper labour, lower taxes and more spacious sites.

[UPSC-1993]

15.The ancient `home region' of sugar beet was the land between

(a)Mediterranean Sea and Caspian Sea.

(b)Baltic Sea and North Sea.

(c)Caspian Sea and Aral Sea.

(d)Mediterranean Sea and Adriatic Sea.

[UPSC-1994]

16.The antipodes in a place situated 25°S and 80°45¢45¢¢W is

(a)25°N and 99°14¢15¢¢E.

(b)2°74¢75¢¢E.

(c)99°14¢15¢¢E.

(d)65°N and 29°74¢75¢¢E.

[UPSC-1994]
17.The central meridian indicated by a line half the length of the Equator
represents

(a)Mercator projection.

(b)polyconic projection.

(c)Mollweide's projection.

(d)Bonne's projection.

[UPSC-1994]

18.The Chindwin basin is situated in

(a)Myanmar.

(b)Thailand.

(c)Malaysia.

(d)Indonesia.

[UPSC-1993]

19.The climate of North America is influence during winter by the

(a)warm Gulf stream.

(b)cold Labrador current.

(c)cold polar air masses.

(d)tropical airmasses.

[UPSC-1993]
20.The concept of `Heartland' was propounded by

(a)Hartshorne.

(b)Haushofer.

(c)Spykman.

(d)Mackinder.

[UPSC-1994]

21.The coniferous forests have been exploited to a considerable extent


because of the

(a)presence of extensive solid stands of one or two species of timber


trees.

(b)presence of a vast network of perennial streams.

(c)existence of large markets in close proximity.

(d)availability of cheap labour.

[UPSC-1994]

22.The following factors have caused water logging in the black lava soil of
Deccan:

1.Introduction of irrigation

2.Sugarcane cultivation

3.High rainfall
Select the correct answer form the codes given.

(a)1 and 2

(b)Only 1

(c)1 and 3

(d)1, 2 and 3

[UPSC-1993]

23.The gravity and potential models in geography were the contribution of

(a)Stewart.

(b)Clark and Evans.

(c)Hagett and Chorley.

(d)Bunge.

[UPSC-1993]

24.The highest rainfall in the world is received at

(a)Cherrapunji.

(b)Mawsynram.

(c)Shillong.

(d)Calicut.

[UPSC-1993]
25.The Hkakabo Razi (5,885 m), the highest point in mainland south-East
Asia, is at the border between which of the following countries?

(a)China and Myanmar

(b)China and Vietnam

(c)Cambodia and Laos

(d)Cambodia and Thailand

[UPSC-2007]

26.The Indus and Brahmaputra rivers are examples of

(a)subsequent drainage.

(b)superimposed drainage.

(c)consequent drainage.

(d)antecedent drainage.

[UPSC-1993]

27.The loess plateau of North China is covered with a thick mantle of loess
carried here by strong winter winds from

(a)The Sahara Desert.

(b)The Gobi Desert.

(c)the Tarim basin.

(d)the Thar Desert.


[UPSC-1993]

28.The most important producer of nickel in the world is

(a)Russia.

(b)Canada.

(c)Brazil.

(d)Ven.

[UPSC-1994]

29.The most populous states of India according to the latest census were the
following:

1.Bihar

2.Uttar Pradesh

3.West Bengal

4.Maharashtra

What is their sequence arranged in the descending order?

(a)2, 1, 4, 3

(b)1,2,4,3

(c)2, 3, 4, 1

(d)1, 2, 3, 4

[UPSC-1993]
30.What kind of rocks have been exposed to great heat and pressure, altering
them into more compact, crystalline rocks'?

(a)Igneous

(b)Sedimentary

(c)Metamorphic

(d)Sima

31.Metamorphic rocks

(a)include shale.

(b)contain abundant fossils.

(c)are formed when molten material cools and solidifies.

(d)are formed when rocks are subjected to great heat and pressure.

32.At which level on the Richter scale are the most devastating earthquakes?

(a)7-8

(b)1-2

(c)3-4

(d)5-6

33.Which of the following is NOT true of magma?

(a)Magma is called lava when it reaches the surface and erupts.


(b)Magma forms sedimentary rocks upon cooling.

(c)Magma is generated by the movement within Earth and between


plates.

(d)Magma migrates towards the surface because it is less dense than the
surrounding rock.

34.Permafrost occurs in what kind of climate'?

(a)Dfa

(b)Csa

(c)Aw

(d)ET

35.If you are told you are in a BWh climate, in which of the following
locations you might be?

(a)Poland

(b)Florida

(c)Sahara Desert

(d)Iceland

36.Warm mid-latitude climates have seasonal variations of insolation that


profoundly influence temperature, which is a result of

(a)the tilt of the Earth's axis.


(b)hilly terrain.

(c)the distribution of oceans.

(d)El Nino.

37.How many time zones are shared by the 48 conterminous U.S. states and
the Canadian provinces?

(a)4

(b)5

(c)6

(d)3

38.Greenwich Mean Time (GMT) is the time at

(a)the meridian traveling through Paris, France.

(b)the Prime Meridian.

(c)the meridian traveling through Washington, DC.

(d)the International Date Line.

39.Assertion (A): Assam grows maximum tea in India.

Reason (R): A lot of cheap tribal labour is available.

(a)Both (A) and (R) are true and (R) is the correct explanation of (A).

(b)Both (A) and (R) are true, but (R) is not the correct explanation of (A).
(c)(A) is true, but (R) is false.

(d)(A) is false, but (R) is true.

[UPSC-1993]

40.Assertion (A): Japan is one of the leading industrial countries of the


world.

Reason (R): It has a large mineral resources base.

(a)Both (A) and (R) are true and (R) is the correct explanation of (A).

(b)Both (A) and (R) are true, but (R) is not the correct explanation of (A).

(c)(A) is true, but (R) is false.

(d)(A) is false, but (R) is true.

[UPSC-1993]

41.GIS refers to

(a)surveying.

(b)Landsat.

(c)a computer system that stores, organizes and analyses geographic data.

(d)an agency of the United Nations.

42.The Earth rotates about its

(a)revolution.
(b)great circle.

(c)inclination.

(d)axis.

43.The Greek word Geographica means `Earth's description' and was first
coined by, who at the time was the director of the library at Alexandria.

(a)Polymethus

(b)Hipparchus

(c)Cadmus

(d)Erastosthenes

44.The Greenwich Meridian is also known as the

(a)perihelion.

(b)Equator.

(c)aphelion.

(d)Prime Meridian.

45.Match List I with List II and select the correct answer using the code
given.
Codes:

[UPSC-1998]

If, as David Harvey suggests, the 1970s witnessed an urban crisis, an energy
crisis, an environmental crisis, then by the 1980s, the whole world appeared
to be in crisis. Within geography, a succession of challenges was launched
on some of the disciplines core concepts. Here I will focus on landscape and
nature.

One of the most effective challenges was posed by Denis Cosgrove's


critique of the idea of landscape. Cosgrove's writings represent a sustained
attempt to deconstruct the understanding of landscape that characterized an
earlier generation of cultural and historical geographers such as Carl Sauer,
Clifford Darby and Andrew H.Clark. Cosgrove rejected the simple
distinction which Sauer drew between `natural' and `cultural' landscapes,
showing

that every landscape is socially constructed. Rather than tracing a


unidirectional `human impact on the landscape' (a tradition which he and
other have parodied as the `landscape with figures' school), Cosgrove
showed how landscape and social formation were dialectically related. More
recently, he has argued that in the transition from modernity to
postmodernity biological and cybernetic models of environmental and
spatial organization (organism, system) have given way to metaphors
derived from literature and the arts (spectacle, theatre, text).

While Cosgrove has challenged the 'naturalness' of landscape, others have


begun to question the `naturalness' of nature itself. Neil Smith began the
current round of critique by adding his own materialist reading to the long
tradition of Marxist thought about the social production of nature. But the
debate has been given renewed vigour by the recent work of Margaret
Fitzsimons. While Smith develops his ideas about socially produced nature
through a reworking of Marx's theory of uneven development, Fitzsimmons
starts out from Raymond's William's essays on `The idea of nature',
extending his work via a variety of feminist sources. She explores the
parallel between man's subordination of nature and his subordination of
women.

46.Centre theme of the passage is

(a)Marxist's idea of geography.

(b)challenges made to the concept of landscape and nature.

(c)deconstruction of landscape.

(d)post-modernist idea of landscape and nature.

[UGC J-2008]

47.The traditional concept of landscape deconstructed by Cosgrove is


(a)landscape is not socially constructed.

(b)cybernetics model are fully applicable to landscapes.

(c)there is distinction between natural and cultural landscapes.

(d)how the human beings have affected the landscape.

[UGC J-2008]

48.How are the ideas of Neil Smith related with the socially produced
nature'?

(a)He uses it as a basis for his theory.

(b)He brings feminist aspect into considerations.

(c)He challenges the status quo purposed by it.

(d)He reworks and modifies it for his purpose.

[UGC J-2008]

49.The significance of literature and art in post-modernism is that

(a)they use cybernetic and biological models to explain reality.

(b)they provide the postmodernism with the required metaphors.

(c)they became basis of post-modern models.

(d)they challenge the status quo.

[UGC J-2008]

50.Fitzsimons view of nature can be explained as


(a)man's subordination of nature.

(b)man's subordination of women and nature.

(c)man's submission towards nature.

(d)man's submission towards women and nature.

[UGC J-2008]

1.The National River Conservation Directorate functions under which Union


Ministry'?

(a)Agriculture Ministry

(b)Environment and Forests Ministry

(c)Science and Technology Ministry

(d)Water Resources Ministry

[UPSC-2006]

2.The North Pole lies within the Arctic Ocean whereas the South Pole is
located within the Antarctica continent. Which of the following
hypotheses used this phenomenon as a supportive argument?

(a)Tetrahedron Theory

(b)Theory of Isostasy

(c)Theory of Continental Drift


(d)Convective Current Theory

[UPSC-1993]

3.Temperatures in cities are higher as a result of

(a)increased number of trees.

(b)pavement and buildings storing heat during the day and releasing it at
night.

(c)increased evapotranspiration in the city.

(d)greater insolation.

4.The amount of solar energy intercepted at a particular area on the Earth's


surface is

(a)convection.

(b)insolation.

(c)angle of incidence.

(d)sensible heat.

5.At which angle (of incidence) will the intensity of energy striking the
Earth be greatest?

(a)30°

(b)60-

(c)0°
(d)90-

6.The principle of homogeneity and interrelationship can be used to


delineate units for

(a)development planning.

(b)resource analysis.

(c)allocation of resources.

(d)location-specific facilities.

[UGC D-2010]

7.The process of gradual spread over space of people or ideas from the
centre of origin is known as

(a)distribution.

(b)diffusion.

(c)innovation.

(d)communication.

[UPSC-1994]

8.The projection most suitable for showing ocean routes is

(a)sinusoidal.

(b)polar zenithal.

(c)Mollweide's.
(d)Mercator's.

[UPSC-1994]

9.The proportion of the areas of the surfaces is various elevations above or


depths below a given datum. This is properly represented by

(a)Chirographic curve.

(b)Altimetry frequency curve.

(c)Hypsometric curve.

(d)Lorenz curve.

[UPSC-1993]

10.The proportion of the number of villages having population above 5,000


to the total number of villages state wise is the highest in

(a)Tamil Nadu.

(b)Uttar Pradesh.

(c)Bihar.

(d)Kerala.

[UPSC-1994]

11.The rank-size rule is useful in studies of

(a)plant communities.

(b)climatic regions.
(c)settlement systems.

(d)soil types.

[UPSC-1993]

12.The region that has a typical dispersed rural settlement pattern is the

(a)Kerala coastal plain.

(b)western Ganga plain.

(c)eastern Rajasthan.

(d)Telangana plateau.

[UPSC-1993]

13.The Rourkela Steel Plant was built on the banks of the

(a)Bhadra river.

(b)Brahmani river.

(c)Damodar river.

(d)Bhima river.

[UPSC-1993]

14.The rural settlements of Kerala are mostly

(a)compact.

(b)dispersed.
(c)linear.

(d)radial.

[UPSC-1994]

15.The shift of the cotton textile industry from New England to the Southern
Appalachians was mainly due to

(a)warmer climate.

(b)uniform working hours.

(c)abundant supply of raw cotton.

(d)low standard of living of the people.

[UPSC-1994]

16.The term `boat people' refers to the migrants from

(a)Sri Lanka to India.

(b)Taiwan to Mainland China.

(c)Vietnam to Thailand.

(d)Vietnam to the USA.

[UPSC-1993]

17.The traditional agricultural classification as presented by D.Whittlessey


was based on four major dimensions of variation-intensity, commercial
development, mobility and
(a)marketability.

(b)mechanization.

(c)crop speciality.

(d)crop productivity.

[UPSC-1994]

18.The typical area of `sal' forest in the Indian Peninsular upland occurs

(a)on the Western Ghats.

(b)between the Tapti and the Narmada.

(c)to the north-east of the Godavari.

(d)on the Malwa Plateau.

[UPSC-1993]

19.The usage of the term `ecosystem' for the whole complex of organisms-
both animals and plants-and their habitat has been proposed by

(a)Cole.

(b)Tansley.

(c)Forbes.

(d)Sukachev.

[UPSC-1993]

20.Topographical maps are distinguished from Atlas maps as they


(a)represent only relief.

(b)are large-scale maps.

(c)are small-scale maps.

(d)are drawn on a different projection.

[UPSC-1993]

21.West Bank is the name given to the land area to the west of the river

(a)Jordan.

(b)Mississippi.

(c)Nile.

(d)Tigris.

[UPSC-1993]

22.What is the correct descending order of the following states area wise?

1.Andhra Pradesh

2.Bihar

3.Madhya Pradesh

4.Uttar Pradesh

Choose the correct answer from the codes given.

(a)1, 2, 3, 4
(b)2, 3, 4, 1

(c)4, 3, 2, 1

(d)3, 4, 1, 2

[UPSC-1993]

23.Where is Mt. Waileale, one of the heaviest rain-fed places in the world,
located (11,990 mm in a year)'?

(a)Brazil

(b)Hawaii

(c)Philippines

(d)Sulawesi

[UPSC-2007]

24.Which indicator from among the following could be the best indicator of
social well-being'?

(a)Per capita income

(b)Population growth rate

(c)Crude literacy rate

(d)Life expectancy

[UGC J-2011]
25.Which of the following are the rivers at the delta mouths of which
mangroves occur in abundance?

1.Narmada

2.Subarnarekha

3.Krishna

4.Ganga

Choose the correct answer from the codes given.

(a)1 and 3

(b)3 and 4

(c)2 and 4

(d)1, 3 and 4

[UPSC-1993]

26.Which of the following cities was a city with a population of 1 million in


1981 census'?

(a)Dirugarh

(b)Kanpur

(c)Puri

(d)Patna

[UPSC-1993]
27.Which of the following countries is not a member of the SAARC'?

(a)Afghanistan

(b)Bhutan

(c)Nepal

(d)Sir Lanka

[UPSC-1993]

28.Which of the following factors are responsible for the rapid growth of
sugar production in South India as compared to North India?

1.Higher per acre field of sugarcane

2.Higher sucrose content of sugarcane

3.Lower labour cost

4.Longer crushing period

Select the correct answer form the codes given.

(a)1 and 2

(b)1, 2 and 3

(c)1, 3 and 4

(d)1, 2 and 4

[UPSC-1993]
29.Which of the following industrial towns is located on the Chhota Nagpur
plateau'?

(a)Bhilai

(b)Ranchi

(c)Asansol

(d)Durgapur

[UPSC-1993]

30.Which of the following is the correct sequence of states in decreasing


order of density of population in India according to 2001 Census'?

(a)WestBengal, Uttar Pradesh, Bihar, Kerala

(b)Uttar Pradesh, WestBengal, Kerala, Bihar

(c)WestBengal, Bihar, Kerala, Uttar Pradesh

(d)Bihar, WestBengal, Kerala, Uttar Pradesh

[UGC D-2011]

31.In what kind of plate boundary is material from the crust very slowly
forced downwards, back into the mantle'?

(a)Transform

(b)Convergent

(c)Divergent
(d)Vertical

32.Near a convergent plate boundary, rocks fracture because they are


compressed; such fractures are what kind of faults'?

(a)Reverse

(b)Normal

(c)Overturned

(d)Symmetrical

33.The greater thickness of what kind of rocks causes the continents to have
higher surface elevations than the oceanic crust'?

(a)Sima

(b)Igneous

(c)Metamorphic

(d)Sial

34.Assertion (A): Sex ration in Kerala, southernmost part of Tamil Nadu,


northeastern Madhya Pradesh and some parts of Uttar Pradesh is above
1,000.

Reason (R): The death rate among the males is high in the these regions.

(a)Both (A) and (R) are true and (R) is the correct explanation of (A).

(b)Both (A) and (R) are true, but (R) is not the correct explanation of (A).
(c)(A) is true, but (R) is false.

(d)(A) is false, but (R) is true.

[UPSC-1994]

35.Assertion (A): The atmosphere is said to be conditionally unstable when


the lapse rate is between the dry adiabatic lapse rate and the wet adiabatic
lapse rate.

Reason (R): The wet adiabatic lapse rate is always greater than the dry
adiabatic lapse rate.

(a)Both (A) and (R) are true and (R) is the correct explanation of (A).

(b)Both (A) and (R) are true, but (R) is not the correct explanation of (A).

(c)(A) is true, but (R) is false.

(d)(A) is false, but (R) is true.

[UPSC-1993]

36.Shields, which usually contain rich concentrations of minerals (such as


metallic ores), are located of large continents such as Africa, Asia and
North America.

(a)along plate boundaries

(b)along fault scarps

(c)in the core

(d)along the coast


37.The San Andreas Fault, where two plates slide past one another, is a
famous example of what kind of plate boundary?

(a)Divergent

(b)Vertical

(c)Transform

(d)Convergent

38.Potential evapotranspiration exceeds precipitation in

(a)continental climates.

(b)the tropics.

(c)the tundra.

(d)dry lands.

39.Which side of China, South America, the USA and Australia has the
driest climate'?

(a)East

(b)West

(c)North

(d)South

40.During the 20thcentury, the Earth's atmospheric temperature increased by


about°C.
(a)3

(b)0.3

(c)2

(d)0.8

41. Climate over the past 2 million years has

(a)stayed the same.

(b)become progressively warmer.

(c)shifted between warm and cool periods.

(d)become progressively cooler.

42.What meridian is opposite the Prime Meridian'?

(a)320° meridian

(b)180° meridian

(c)90° meridian

(d)Tropic of Capricorn

43.Which is NOT an example of a formal region?

(a)The area of dominance of a particular crop

(b)The area where most people belong to a particular religion

(c)The area where a particular language is spoken


(d)The market area of a supermarket

44.Tertiary sector jobs involve the

(a)extraction of materials from Earth.

(b)provision of services.

(c)the harvesting of agricultural products.

(d)manufacturing of raw materials

45.The portion of world trade that is between divisions of the same


corporation is

(a)one quarter.

(b)one third.

(c)one half.

(d)unknown.

46.International trade in services

(a)has not yet become feasible.

(b)is less profitable than trade in goods.

(c)is more profitable than trade in goods.

(d)demonstrates the same trends as trade in goods.

Spatial structure recognizes a duality between `spatial process' and `spatial


structure' that is between `movement over the Earth's surface' and the
resulting arrangement of phenomena on the Earth's surface'. In Burges' view,
spatial structure could be defined by interpreting structure as geometrical.
This became the central feature of the Quantitative Revolution in geography
wherein spatial structure became synonymous with spatial pattern. Modern
Human Geography also recognizes an essential dialectic between spatial
structure and social structure so that spatial analysis is now frequently also
social analysis and vice versa. Spatial structure is both a condition and a
consequence of human actions.

47.What do `process' and `structure' indicate in spatial analysis'?

(a)Process is movement and structure is arrangement.

(b)Process is an action, and structure is the consequence or end result of


the action.

(c)Process is the condition and structure is the consequence.

(d)Process is illusionary structure is realistic.

[UGC J-2005]

48.What is the central feature of the Quantities Revolution in Geography'?

(a)Structure becomes synonymous with spatial pattern.

(b)Duality exists between spatial process and spatial structure.

(c)Structures can be interpreted as geometrical.

(d)All of these

[UGC J-2005]
49.What is the dialectic which Modern Human Geography recognizes'?

(a)The difference between spatial structure and social structure

(b)The similarity between spatial structure and social structure

(c)The similarity between social analysis and spatial analysis

(d)The difference between social analysis and spatial analysis

[UGC J-2005]

50.Why is spatial analysis also considered as social analysis'?

(a)Because spatial structure is both a condition and a consequence of


human action.

(d)None of these

(b)Because both study the same issue from opposite point of views.

(c)Because both study the same issue from the similar point of view.

[UGC J-2005]

1.Which of the following makes the temperate forests easier or exploit than
the tropical rainforests?

1.Proximity of market

2.Softer wood

3.Taller trees
4.Grater homogeneity

Select the correct answer form he codes given.

(a)1 and 3

(b)2 and 4

(c)2 and 3

(d)3 and 4

[UPSC-1993]

2.Which of the following pairs is not correctly matched'?

(a)Kosi - Consequent river

(b)Narmada - Rift valley river

(c)Sutlej - Antecedent river

(d)Ghagghar - River with endorheic drainage

[UPSC-1993]

3.Which of the following statement is correct on the share of India's


population to world population as per 2001 census?

(a)14.7%

(b)15.7%

(c)16.7%
(d)17.7%

[UGC D-2010]

4.Which one of the following attributes is not a spatial database?

(a)Location

(b)Lines

(c)Polygon

(d)Triangle

[UGC J-2007]

5.Which area receives the most seasonal variation in incoming radiation?

(a)Equator

(b)Tropics

(c)Low latitudes

(d)High latitudes

6.During the vernal equinox, which location receives the greatest amount of
insolation?

(a)Tropic of Cancer

(b)Equator

(c)poles
(d)Tropic of Capricorn

7.In the Northern Hemisphere, the Sun is lowest in the sky with less
radiation in the

(a)spring.

(b)summer.

(c)winter.

(d)fall.

8.Which one of the following countries has Red River delta?

(a)Japan

(b)Indonesia

(c)Malaysia

(d)Vietnam

[UPSC-2007]

9.Which one of the following countries has the largest forest area in the
world?

(a)Brazil

(b)Canada

(c)Russia Federation

(d)USA
[UPSC-2008]

10.Which one of the following has the volcanic Mount St. Helens'?

(a)Aleutian Range

(b)Alaska Range

(c)Cascade Range

(d)Mohave Range

[UPSC-2007]

11.Which one of the following indicates the principle of Central Place


Theory?

(a)K3

(b)K5

(c)K7

(d)K9

[UPSC-1994]

12.Which one of the following is a satellite town?

(a)Faridabad

(b)Murshidabad

(c)Ghaziabad

(d)Moradabad
[UPSC-1993]

13.Which one of the following is generally considered to be the most


important reason for the decline of the Indus Valley civilization?

(a)Repeated droughts

(b)Epidemics

(c)Frequent floods

(d)Internal feuds

[UPSC-1994]

14.Which one of the following is most plausible solution for planning


improvement of a metropolitan city in India'?

(a)Constructing high-rise building and skyscrapers

(b)Building up suburbs and satellite towns around the city

(c)Dispersing factories and industries to the neighbouring but


independent locations

(d)Preventing immigration of people into the metropolitan city

[UPSC-1994]

15.Which one of the following is not a member of SAARC?

(a)Maldives

(b)Bhutan
(c)Sri Lanka

(d)Myanmar

[UGC D-2005]

16.Which one of the following is the correct statement? Kalahari is a desert


because it

(a)lies in the tropics.

(b)is on the leeward side of a mountain.

(c)underlies a high-pressure cell.

(d)receives only land winds.

[UPSC-2006]

17.Which one of the following major sea ports of India does not have a
natural harbour'?

(a)Bombay

(b)Cochin

(c)Marmagao

(d)Paradeep

[UPSC-1993]

18.Assertion (A): Areas along the Equator record the highest temperature
throughout the year.
Reason (R): On the Equator, days and nights are equal for the largest part
of the year.

(a)Both (A) and (R) are true and (R) is the correct explanation of (A).

(b)Both (A) and (R) are true but (R) is not the correct explanation of (A).

(c)(A) is true but (R) is false.

(d)(A) is false but (R) is true.

[UPSC-1993]

19.Assertion (A): Equatorial region is called `the region of debilitation'.

Reason (R): The Equatorial region has unit formally unfavourable


climatic conditions

(a)Both (A) and (R) are true and (R) is the correct explanation of (A).

(b)Both (A) and (R) are true but (R) is not the correct explanation of (A).

(c)(A) is true but (R) is false.

(d)(A) is false but (R) is true.

[UPSC-1993]

20.Which one of the following pairs is correctly matched'?

(a)Spatial Diffusion Theory - Hagerstrand

(b)Landscape Theory - Chorley

(c)Trade Theory - King


(d)Transportation Coast Theory - Smith

[UPSC-1994]

21.Which one of the following pairs is correctly matched'?

(a)Automobile industry - Los Angeles

(b)Ship building - Lusaka

(c)Aircraft industry - Seattle

(d)Iron and steel industry - Florence

[UPSC-1994]

22.Which one of the following pairs is NOT correctly matched?

(a)Bandung-Jawa Island

(b)Bangkok: Mekong River

(c)Denpasa: Bali Island

(d)Mandalay: Irrawady River

[UPSC-2006]

23.Which one of the following primitive people use `krall' for their shelter?

(a)Pygmy

(b)Papuan

(c)Bedouins
(d)Masai

[UPSC-2006]

24.Which one of the following South-West Asian countries has the shortest
coastline'?

(a)Iran

(b)Oman

(c)Syria

(d)Jordan

[UPSC-1993]

25.Which one of the following states does not form a part of the Narmada
basin'?

(a)Madhya Pradesh

(b)Rajasthan

(c)Gujarat

(d)Maharashtra

[UPSC-1993]

26.Which one of the following states is the leading in terms of total fish
catch?

(a)Kerala
(b)Tamil Nadu

(c)West Bengal

(d)Gujarat

[UGC D-2007]

27.Which one of the following tribes practices pastoral nomadism?

(a)Eskimo

(b)Boro

(c)Pygmy

(d)Masai

[UPSC-1993]

28.Which one of the natural regions is known as the `Bread-Basket' of the


world?

(a)The steppe region

(b)The Mediterranean region

(c)The monsoon region

(d)The Equatorial region

[UPSC-1993]

29.While traveling from north to south along the east coast of China, which
one of the following is the correct sequence of the given Chinese cities?
(a)Shanghai, Guangzhou, Harbin, Beijing

(b)Harbin, Beijing, Shanghai, Guangzhou

(c)Shanghai, Beijing, Harbin, Guangzhou

(d)Harbin, Guangzhou, Shanghai, Beijing

[UPSC-2006]

30.Who defined `Resource' as `Resources are not they become'?

(a)Van Thunen

(b)Zimmerman

(c)Hartshorne

(d)Semple

[UGC D-2006]

31.Who propounded the concept of areal differentiation'?

(a)Jean Brunhes

(b)Richard Hartshorne

(c)David M.Smith

(d)H.S. Barrows

[UGC J-2008]

32.The Appalachians, the European Alps and the Himalayas are examples of
mountain ranges formed by
(a)volcanic eruptions.

(b)divergent plate boundaries.

(c)normal faults.

(d)faulting and folding.

33.Which of the following factors does NOT influence the amount of ground
shaking caused by earthquakes?

(a)Proximity to epicentre

(b)The earthquake's intensity

(c)Time of day

(d)Unstable ground

34.Which of the following roles do streams play in shaping landforms?

(a)Weathering, transportation and deposition

(b)Erosion, transportation and deposition

(c)Erosion, weathering and deposition

(d)Erosion, transportation and weathering

35.The smallest channels eroded by the flow of water, only a few


centimetres deep, are

(a)rills.

(b)sials.
(c)moraine.

(d)alluvial fans.

36.The effect of deposition of sediment on streams is to

(a)increase slope.

(b)lower the elevation of the stream.

(c)increase depth of pools.

(d)change the shape of the channel.

37.Which of the following is NOT true of chemical weathering'?

(a)It can be a result of the acids released by decaying vegetation.

(b)It occurs faster in cooler places.

(c)Rocks are broken down in a way that changes the minerals that
compose the rock.

(d)Oxidation is an example.

38.A meandering stream does NOT

(a)erode and widen its floodplain.

(b)deposit material where current is slower and has less energy.

(c)erode material from one side of the channel, where the current is
swifter.

(d)deposit material to constrict its floodplain.


39.Alpine glaciers

(a)are found at the poles.

(b)are increasing in size each year.

(c)are thick layers of ice over 3 km in thickness.

(d)form near the peaks of individual mountains.

40.Which map would have the largest scale'?

(a)World

(b)State

(c)City

(d)Continent

41.Which of the following will NOT bring about changes in relative


location?

(a)The discovery of new trade routes

(b)A new tariff agreement between two countries

(c)The opening of a new highway

(d)Latitude and longitude

42.The key to national wealth is

(a)having a lot of natural resources.


(b)being a tourist destination.

(c)adding value to raw materials.

(d)having a large population.

43.Countries that transnational corporations have invested in and that export


globally, even if there is no market for the product within the country, are

(a)in violation of international laws.

(b)developing countries.

(c)export platforms.

(d)more developed countries.

44.A corporation that has established production facilities in other countries


primarily to meet the consumption needs of those countries is in this stage
of the evolution of a multinational corporation.

(a)First

(b)Second

(c)Third

(d)Fourth

45.In 1994, the United States, Canada and Mexico were linked through the

(a)European Union.

(b)North American Free Trade Agreement.


(c)United Nations.

(d)North Atlantic Treaty Organization.

46.German geographer Friedrich Ratzel suggested that each nation needed


Lebensraum, which means

(a)survival of the fittest.

(b)room to live.

(c)geopolitics.

(d)the strong will survive at the expense of the weak.

47.Transport route development model by Taffe is based on his studies on

(a)Ethiopia and Egypt.

(b)Sudan and Nigeria.

(c)Kenya and Nigeria.

(d)Ghana and Nigeria.

[UGC J-2010]

48.Which one of the following is an ideal combination for setting up iron


and steel industry'?

(a)Coal, electricity, market

(b)Coal, iron ore, water

(c)Electricity, water, iron ore


(d)Coal, water, cotton

[UGC J-2010]

Read the following paragraph and answer the questions.

Humanistic geography is an approach in human geography which is


distinguished by the central and active role it gives to human awareness,
agency and creativity. It is at once an attempt at `understanding meaning,
value and human significance of life events and an expansive view of what a
person is and can do. Although humanism is usually traced back to the
French school of La Geogmphie Humaine; its revival in the 1970s owed
much to a dissatisfaction with the more mechanistic models developed
during the Quantitative Revolution. It often seemed to suppress the essential
subjectivity of both the investigator and the investigated. As such it shared in
the more general critique of Positivism's claim to objectivity, and it was
represented `as a form of criticism' through which geographers can be made
more self-aware and cognizant of many of the hidden assumptions and
implications of their methods and research, rather than as a coherent
methodology for the `postbehavioural revolution' in geography'.

49.Mechanistic models were developed in which of the following phases of


geography?

(a)Quantitative geography

(b)Post-behavioural geography

(c)Radical geography

(d)Neodeterministic geography

[UGC J-2005]
50.Humanistic geography focuses on

(a)questions of inequality in society.

(b)human awareness and human agency.

(c)economic awareness and economic agency.

(d)radical changes in society.

[UGC J-2005]

ANSWER KEYS

Exercise - I

Exercise - II

Exercise - III
Exercise - IV

Exercise - V

 
1.`Ladang' is a type of cultivation characterized by

(a)dry farming.

(b)high degree of mechanization.

(c)mixed farming.

(d)shifting cultivation.

[UPSC-1993]

2.A centre of mica mining in India is located at

(a)Khetri.

(b)Kodarma.

(c)Kalahandi.

(d)Guru Mahasani.

[UPSC-1994]

3.Social affairs involving property rights, political jurisdictions, market


areas, ethnic claims to specific areas and efforts to protect traditional land
uses all involve issues of
(a)sacred spaces.

(b)territoriality.

(c)proxemics.

(d)cosmopolitanism.

4.A map that accurately portrays land shape is called a(n)

(a)conformal projection.

(b)equidistant projection.

(c)azimuthal projection.

(d)equal area projection.

5.A much fragmented or clustered pattern of population distribution is


conspicuous in parts of Asia because it is closely associated with

(a)soil and rainfall distribution.

(b)surface configuration and temperature characteristics.

(c)routes of transport and trade centres.

(d)distribution of mineral and forest resources.

[UPSC-1993]

6.A region best known for trapping of animals is the

(a)British type.
(b)Laurentian region.

(c)Taiga region.

(d)polar region.

[UPSC-1994]

7.Accumulations of shifting sand are

(a)spurs.

(b)dunes.

(c)horns.

(d)cirques.

8.The Alps mountains are spread over

(a)France, Switzerland, Italy, Romania.

(b)France, Switzerland, Italy, Austria.

(c)France, Switzerland, Italy, Spain.

(d)France, Switzerland, Italy, Belgium.

[UPSC-2009]

9.Although almost all the states of India have paper mills, half of the total
paper produced in the country comes from

(a)Orissa.
(b)Maharashtra.

(c)West Bengal.

(d)Andhra Pradesh.

[UPSC-1993]

10.Among the following, the narrowest portion of English Channel lies


between

(a)Dover and Calais.

(b)Portsmouth and Le Havre.

(c)Hull and Rotterdam.

(d)Plymouth and Brest.

[UPSC-2009]

11.Animal rearing on a commercial scale has developed well in

(a)monsoon regions.

(b)savannah regions.

(c)Prairie and Steppe regions.

(d)Sabel regions.

[UGC D-2006]

12.Assertion (A): Africa has immense natural resources and yet it is


industrially the most backward continent.
Reason (R): The continent had been under colonial rule for a long time.

(a)Both (A) and (R) are true and (R) is the correct explanation of (A).

(b)Both (A) and (R) are true but, (R) is not the correct explanation of (A).

(c)(A) is true but, (R) is false.

(d)(R) is true but, (A) is false.

[UPSC-1994]

13.Assertion (A): Cotton is graded according to staples.

Reason (R): The staples mean the degree of whiteness of threads

(a)Both (A) and (R) are true and (R) is the correct explanation of (A).

(b)Both (A) and (R) are true but, (R) is not the correct explanation of (A).

(c)(A) is true but, (R) is false.

(d)(R) is true but, (A) is false.

[UPSC-1994]

14.Assertion (A): India's cement-producing capacity is concentrated in


Madhya Pradesh, Gujarat, Tamil Nadu, Maharashtra and Karnataka.

Reason (R): Madhya Pradesh, Gujarat, Tamil Nadu, Maharashtra and


Karnataka have bulk of the limestone resources of India.

(a)Both (A) and (R) are true and (R) is the correct explanation of (A).

(b)Both (A) and (R) are true but, (R) is not the correct explanation of (A).
(c)(A) is true but, (R) is false.

(d)(R) is true but, (A) is false.

[UPSC-1994]

15.Assertion (A): The people who depend upon tropical crops suffer from
malnutrition.

Reason (R): The tropical crops are rich in carbohydrates.

(a)Both (A) and (R) are true and (R) is the correct explanation of (A).

(b)Both (A) and (R) are true but, (R) is not the correct explanation of (A).

(c)(A) is true but, (R) is false.

(d)(R) is true but, (A) is false.

[UPSC-1994]

16.Assertion (A): Rajasthan desert is covered with shifting sand dunes.

Reason (R): For want of rain and surface flowing of water, the work of
wind is felt everywhere.

(a)Both (A) and (R) are true and (R) is the correct explanation of (A).

(b)Both (A) and (R) are true but, (R) is not the correct explanation of (A).

(c)(A) is true but, (R) is false.

(d)(R) is true but, (A) is false.

[UPSC-1994]
17.The study of the principles that a country develops to organize its
economic life is known as

(a)governing laws.

(b)political economy.

(c)political geography.

(d)economic geography.

18.Investment in improved transportation and communication systems is


usually done with the intent to

(a)increase tax revenues.

(b)increase transportation costs.

(c)increase regional divides.

(d)increase economic growth.

19.Assertion (A): India has a tropical monsoon climate.

Reason (R): India is located within tropical latitudes.

(a)Both (A) and (R) are true and (R) is the correct explanation of (A).

(b)Both (A) and (R) are true but, (R) is not the correct explanation of (A).

(c)(A) is true but, (R) is false.

(d)(R) is true but, (A) is false.

[UPSC-1993]
20.Assertion (A): Areas of dense population in Asia lie between 10°N and
40°N latitudes.

Reason (R): These areas have monsoon climate.

(a)Both (A) and (R) are true and (R) is the correct explanation of (A).

(b)Both (A) and (R) are true but, (R) is not the correct explanation of (A).

(c)(A) is true but, (R) is false.

(d)(R) is true but, (A) is false.

[UPSC-1993]

21.The purpose of export substitution policies is to

(a)increase farm output.

(b)expand exports.

(c)build schools.

(d)protect domestic infant industries.

22.As a major importer of products from other countries, the USA has

(a)accumulated substantial trade deficits.

(b)improved public relations.

(c)few domestically manufactured goods.

(d)increased its economic standing with other countries.


23.Assertion (A): Physiological density is preferable to arithmetic density as
an index of population density.

Reason (R): Physiological density is based on arable land, while


arithmetic density is based on total area.

(a)Both (A) and (R) are true and (R) is the correct explanation of (A).

(b)Both (A) and (R) are true but, (R) is not the correct explanation of (A).

(c)(A) is true but, (R) is false.

(d)(R) is true but, (A) is false.

[UPSC-1994]

24.Assertion (A): Plantation agriculture is highly developed in Southeast


Asia.

Reason (R): Southeast Asia has remained under the European colonial
rule for a long time.

(a)Both (A) and (R) are true and (R) is the correct explanation of (A).

(b)Both (A) and (R) are true but, (R) is not the correct explanation of (A).

(c)(A) is true but, (R) is false.

(d)(R) is true but, (A) is false.

[UPSC-1993]

25.Infant industries are those that as new industries only manufacture small
quantities until they reach the national market and produce a larger
quantity. This is called achieving

(a)success.

(b)affordable products.

(c)economies of scale.

(d)supply and demand.

26.The purpose of import substitution policies is to

(a)expand exports.

(b)build schools.

(c)protect domestic infant industries.

(d)increase farm output.

27.Assertion (A): The eastern coast of India is affected by tropical cyclones


more than the western coast.

Reason (R): Tropical cyclones originate only in the Bay of Bengal.

(a)Both (A) and (R) are true and (R) is the correct explanation of (A).

(b)Both (A) and (R) are true but, (R) is not the correct explanation of (A).

(c)(A) is true but, (R) is false.

(d)(R) is true but, (A) is false.

[UPSC-1993]
28.Assertion (A): The population of the world has experienced an
unprecedented growth during the 20th century.

Reason (R): Death rate decreased significantly during the 20th century
due to development of hearth services.

(a)Both (A) and (R) are true and (R) is the correct explanation of (A).

(b)Both (A) and (R) are true but, (R) is not the correct explanation of (A).

(c)(A) is true but, (R) is false.

(d)(R) is true but, (A) is false.

[UPSC-1993]

29.Assertion (A): The regur soils of the Deccan plateau are black in colour.

Reason (R): They contain lot of humus.

(a)Both (A) and (R) are true and (R) is the correct explanation of (A).

(b)Both (A) and (R) are true but, (R) is not the correct explanation of (A).

(c)(A) is true but, (R) is false.

(d)(R) is true but, (A) is false.

[UPSC-1993]

30.At which place are diesel locomotives manufactured?

(a)Chittaranjan

(b)Varanasi
(c)Jamshedpur

(d)Perambur

[UPSC-1993]

31.Britishers constitute 90% of the population of

(a)Australia.

(b)New Zealand.

(c)Canada.

(d)South Africa.

[UPSC-1994]

32.By what name are the numerous depressions found in a limestone region
called'?

(a)Caverns

(b)Potholes

(c)Plungepools

(d)Sinkholes

[UPSC-1993]

33.Clear nights are cooler than cloudy nights because of

(a)condensation.

(b)radiation.
(c)insolation.

(d)conduction.

[UPSC-1993]

34.Consider the following:

Andhra Pradesh in one of the leading producers of rice because

1.it has fertile alluvial soil in the coastal plain.

2.it receives about 12. cm of rainfall per annum in the coastal tract.

3.it has a lot of tank irrigation in the coastal plain.

Select your answer from the codes given.

(a)1, 2 and 3 are correct.

(b)1 and 2 are correct.

(c)2 and 3 are correct.

(d)1 and 3 are correct.

[UPSC-1994]

35.Glaciers

(a)can move up to several hundred meters per year.

(b)have little lasting impact on geomorphology.

(c)contain more water today than they did 18,000 years ago.
(d)are more prominent at lower latitudes than higher latitudes.

36.In the Mercator projection, which piece of Earth is portrayed extremely


large in comparison to its actual size?

(a)Brazil

(b)Greenland

(c)The continental USA

(d)Low-latitude locations

37.La Nina

(a)only affects the Northern Hemisphere.

(b)always lasts for the same period of time as the previous El Nino.

(c)has nothing to do with ocean temperatures and circulation.

(d)can lead to drought in the southern USA.

38.Latent heat is

(a)heat left behind by a resting object.

(b)heat in storage in water and water vapour.

(c)detectable by sense of touch.

(d)heat reflected into the atmosphere.

39.Most of the International Date Line follows


(a)0°longitude.

(b)0° latitude.

(c)90° latitude.

(d)180° longitude.

40.Movement of a fluid when a part of it is heated is

(a)convection.

(b)conduction.

(c)advection.

(d)sublimation.

41.On a daily basis, the Sun is the most intense at

(a)12 noon.

(b)6.00 a.m.

(c)6.00 p.m.

(d)3.00 p.m.

42.The 0° meridian is also known as the

(a)geographic grid.

(b)Prime Meridian.

(c)Equator.
(d)perihelion.

43.The boundary where Earth's plates are spreading apart is what kind of
plate boundary'?

(a)Divergent

(b)Convergent

(c)Vertical

(d)Transform

44.The Hawaiian Islands

(a)are a series of shield volcanoes.

(b)formed from a series of violent volcanic eruptions.

(c)erupt lots of ash and pyroclasts.

(d)lie atop the meeting place of two major tectonic plates.

45.What kind of rocks are formed when molten crustal material cools and
solidifies?

(a)Sima

(b)Metamorphic

(c)Sedimentary

(d)Igneous

46.Which of the following is NOT considered when classifying climates?


(a)Unique weather events

(b)Terrain

(c)Weather patterns over several years

(d)Vegetation

47.Which of the following is NOT true of shield volcanoes?

(a)They are known for their violent, exploding eruptions of thick and
gassy magma.

(b)They have runny basaltic lava.

(c)They are the largest volcanoes on Earth.

(d)They include the Big Island of Hawaii.

48.Which one of the following natural regions is best known for the
production of grapes'?

(a)Hot deserts

(b)Monsoon

(c)Equatorial

(d)Mediterranean

[UPSC-2004]

49.Which one of the following natural regions is not correctly matched with
its economic base?
(a)Canadian Prairies: Spring wheat cultivation

(b)Argentine Pampas: Beef cattle ranching

(c)Australian Downs: Nomadic herding

(d)South African Veld: Maize growing

[UPSC-2001]

50.Which one of the following statements is not correct?

(a)Shifting cultivation is characterized by field rotation rather than by


crop rotation.

(b)In intensive subsistence agriculture, animal farming is more


developed.

(c)Plantation crops require heavy outlay.

(d)The economy of livestock ranchers is purely commercial.

[UPSC-2004]

51.Which method of data acquisition is NOT considered to be in the class of


remote sensing'?

(a)LANDSAT satellite data

(b)Rain gauge data

(c)Weather satellite data

(d)Aerial photography
52.Which of Earth's four physical systems is a living system?

(a)Biosphere

(b)Lithosphere

(c)Atmosphere

(d)Hydrosphere

53.Which of the following statements about regions is true'?

(a)All regions have precise boundaries.

(b)Features used to define regions must be clear and distinct on the


landscape.

(c)Geographers decide what phenomena to use to define a region.

(d)Everyone agrees on the extent of all regions.

54.The position, placement or arrangement of something throughout space is


its

(a)concentration.

(b)density.

(c)pattern.

(d)distribution.

55.The radius of the Earth is about

(a)93,000,000 miles.
(b)30,000 miles.

(c)2,30,000 miles.

(d)3,959 miles.

56.The relationship between the size of an object on a map and its size on
Earth's surface is

(a)relief.

(b)scale.

(c)geography.

(d)spatial analysis.

57.An orbiting satellite provides data for analysis by

(a)cartography.

(b)remote sensing.

(c)projection.

(d)GIS.

58.Geography is

(a)rivers and mountains.

(b)the study of places and how places interact spatially to form patterns.

(c)place names.
(d)maps.

59.GIS analysis would not be possible without

(a)cell phones.

(b)satellites.

(c)computers.

(d)government agencies.

60.A group of organisms and the non-living physical and chemical


environment in which they interact is

(a)an ecosystem.

(b)a vernacular region.

(c)a closed system.

(d)a biosphere.

61.A large-scale map represents a small portion of surface area at a high


level of detail. Which fractional unit is NOT considered to be largescale?

(a)1/10,000,000

(b)1/10,000

(c)1/10

(d)1/1,000

62.An example of the new economy is


(a)manufacturing.

(b)farming.

(c)retailing.

(d)the Internet.

63.The process of economic development means

(a)building highways.

(b)progressively increasing the value of goods and service that a place


can produce.

(c)encouraging population growth.

(d)finding more natural resources.

64.A steel manufacturing plant located next to an iron ore deposit is an


example of

(a)material-oriented manufacturing.

(b)minimal cost-distance manufacturing.

(c)raw material manufacturing.

(d)market-oriented manufacturing.

65.Over the last 40 years, men's dress shirt manufactured for the American
market has migrated around the world. The first country to engage in this
activity was
(a)Costa Rica.

(b)Bangladesh.

(c)Taiwan.

(d)Japan.

66.Most foreign direct investment (FDI) is

(a)from one rich country to one poor country.

(b)from one poor country to one rich country.

(c)from one rich country to another rich country.

(d)from one poor country to another poor country.

67.The World Trade Organization

(a)can impose trade sanctions on members who misbehave.

(b)no longer offers members most favoured nation status.

(c)does not count the USA among its members.

(d)evolved into GATT.

68.An independent political unit that claims exclusive jurisdiction over a


territory and all the people in it is a

(a)satellite.

(b)state.
(c)nationality.

(d)colony.

69.A natural resource is defined by three elements:

(a)cultural values, technology and economic system.

(b)cultural values, resource availability and economic system.

(c)resource availability, technology and economic system.

(d)cultural values, technology and resource availability.

70.Renewable energy sources include all of the following EXCEPT power.

(a)hydroelectric

(b)tidal

(c)wind

(d)nuclear

71.A group of countries that agree to control the market by limiting


production in order to drive up prices is

(a)an ecumene.

(b)a cartel.

(c)a world bank.

(d)an itcz.
72.Which of the following is NOT an evidence of global climate change?

(a)Rising sea level

(b)Rising global average temperatures

(c)Disappearing rainforests

(d)Disappearing Arctic Sea ice

73.is not a part of the hydrologic cycle.

(a)Water infiltrating into the soil and bedrock

(b)Calcium carbonate dissolving in soil water and groundwater

(c)Water moving into creeks and streams following a rainstorm

(d)Water evaporating from a lake

74.Among the core principles of the slow cities, movement is the concept of

(a)community life.

(b)visualizing geography.

(c)place commodification.

(d)pseudo place.

75.Which two chemical constituents cannot be formed by chemical


weathering of the feldspar minerals?

(a)Silica and soluble calcium bicarbonate


(b)Insoluble iron oxides and soluble magnesium bicarbonates

(c)Silica and insoluble clay minerals

(d)Soluble sodium and potassium bicarbonates

1.Consider the following coal-producing states:

1.Bihar

2.Madhya Pradesh

3.West Bengal

The correct descending order in terms of the amount of coal produced in


these states is

(a)1, 2, 3

(b)3, 1, 2

(c)1, 3, 2

(d)3, 2, 1

[UPSC-1994]

2.Assertion (A): The demand of potash fertilizers in India is met by


indigenous production.

Reason (R): India's requirement of potash fertilizers has been very small.
(a)Both (A) and (R) are true and (R) is the correct explanation of (A).

(b)Both (A) and (R) are true but, (R) is not the correct explanation of (A).

(c)(A) is true but, (R) is false.

(d)(R) is true but, (A) is false.

[UPSC-1994]

3.Assertion (A): The equatorial forest in the world is dense and impenetrable
with many types of plants.

Reason (R): The enormous growth of vegetation in this region is due to


extremely warm and humid climate.

(a)Both (A) and (R) are true and (R) is the correct explanation of (A).

(b)Both (A) and (R) are true but, (R) is not the correct explanation of (A).

(c)(A) is true but, (R) is false.

(d)(R) is true but, (A) is false.

[UPSC-1994]

4.From September to March, the Southern Hemisphere receives

(a)lesser radiation than the Northern Hemisphere.

(b)more radiation than the Northern Hemisphere.

(c)24 hours of daylight.

(d)exactly 12 hours of daylight and darkness.


5.At what date will 50°N receive the largest amount of daylight?

(a)December 21

(b)March 21

(c)September 22

(d)June 21

6.The conversion of water from vapour to liquid is the process of

(a)evaporation.

(b)convection.

(c)saturation.

(d)condensation.

7.Continental polar air masses tend to be

(a)warm and dry.

(b)cool and wet.

(c)cool and dry.

(d)warm and wet.

8.Consider the following pairs:

Area: Well-known for

1.Kinta Valley: Tin fields


2.Sudbury region: Nickel production

3.Chihuahua: Silver production

Which of the above pair(s) is/are correctly matched?

(a)1 only

(b)1 and 2 only

(c)1 and 3 only

(d)1, 2 and 3

[UPSC-2009]

9.In the USA, we tend to step back as people step closer. In the Middle East,
this can be interpreted as rude. Recognizing that different cultures view
personal space differently is to appreciate the concept of

(a)territoriality.

(b)postmodernity.

(c)modernity.

(d)proxemics.

10.Consider the following pairs:

Industrial town: Famous for

1.Kumamoto: Shipbuilding

2.Fukuoka: Chemicals
3.Hamamatsu: Musical instruments

Which of the statements given above is/ are correct'?

(a)1 only

(b)1 and 2 only

(c)1 and 3 only

(d)1, 2 and 3

[UPSC-2009]

11.is one of the most thoroughly globalized forms of trade and also one of
the largest markets in the world.

(a)Investment banking

(b)Telecommunications manufacturing

(c)Legal services

(d)Car manufacturing

12.Consider the following pairs:

1.St. Petersbury: Gulf of Finland

2.Stockholm: Baltic sea

3.Oslo: Norwegian sea

Which of the above is/are correct?


(a)1 only

(b)1 and 2 only

(c)2 and 3 only

(d)1, 2 and 3

[UPSC-2006]

13.The idea that `nature is a social creation' means that

(a)the evolutionary process concluded with the emergence of humans.

(b)humans have built the physical world and its landscapes-nothing is


natural.

(c)the physical world and its resources were made for human use.

(d)the physical world means different things to different people at


different times.

14.Consider the following statements:

1.Biwa ko is the largest lake in Japan covering about 670 km2.

2.Sichuan pendi is one of China's riceproducing areas.

Which of the statements given above is/ are correct?

(a)1 only

(b)2 only

(c)Both 1 and 2
(d)Neither 1 nor 2

[UPSC-2008]

15.Consider the following statements about Indian forestry:

1.About 40% of the Indian forests are in the inaccessible mountainous


regions which impede the speedy transfer of felled trees.

2.In India's forests except in some states, the percentage of forest cover is
precariously low.

3.Exploitation of forests is less time consuming and less costly, as almost


in every forest, a single type of tree dominates.

4.The annual productivity of the forests is 3.5 cubic meters per hectare.

(a)1 and 2 are correct

(b)1, 2 and 3 are correct

(c)3 and 4 are correct

(d)1, 2, 3 and 4 are correct

[UPSC-1994]

16.Consider the following statements:

Developing countries in the past two decades have registered increase in


population despite significant decline in fertility rates because of

1.increasing life expectancy.

2.declining infant mortality.


3.falling death rates.

4.better living conditions.

Of these statements

(a)1, 2 and 4 are correct

(b)1, 3 and 4 are correct

(c)1, 2 and 3 are correct

(d)2, 3 and 4 are correct

[UPSC-1994]

17.An extremely long ocean wave created by an underwater earthquake that


may travel hundreds of kilometres per hour is a

(a)tsunami.

(b)rill.

(c)dune.

(d)moraine.

18.are like a river carrying sediment from areas where it is eroded by waves
and depositing it where breaking waveslose energy to carry it, usually in
deeper water.

(a)Rills

(b)Long-shore currents
(c)Surfs

(d)Seismic waves

19.Consider the following states:

1.Jammu and Kashmir

2.Uttar Pradesh

3.Meghalaya

4.Arunachal Pradesh

The correct sequence of the descending order in terms of the area of


coverage of coniferous forests in these states is

(a)1, 4, 2, 3.

(b)1, 3, 2, 4.

(c)2, 3, 4, 1.

(d)1, 2, 4, 3.

[UPSC-1994]

20.Consider the following states:

1.Kerala

2.Mizoram

3.Goa
4.Tamil Nadu

As per the 1991 census, the correct sequence of the descending order of
the effective literary rates in these states is

(a)1, 2, 3, 4.

(b)1, 4, 2, 3.

(c)1, 3, 4, 2.

(d)4, 1, 2, 3.

[UPSC-1994]

21.Consider the temperature and rainfall data below:

This would be typical of


(a)an equatorial region.

(b)a monsoon region.

(c)a warm temperate region.

(d)a cool temperate region.

[UPSC-1994]

22.Contribution of forestry to national income is around

(a)2.0 percent.

(b)3.0 percent.

(c)1. 0 percent.

(d)4. 0 percent.

[UPSC-1994]

23.Copious rains in the northern coast of Tamil Nadu in October and


November are due to

(a)the south-east trade winds.

(b)easterly depression and the presence of intertropical convergence zone.

(c)westerly depressions.

(d)occlusion.

[UPSC-1994]

24.Crop combination is calculated on the basis of


(a)per centarea of net cultivated area.

(b)per centarea of gross cultivated area.

(c)ranking of crops in a region.

(d)None of the above

[UGC D-2006]

25.Dark and hard are the characteristic features of

(a)tear.

(b)pine.

(c)ebony.

(d)sal.

[UPSC-1994]

26.Development of human resources is necessary mainly because people can

(a)get higher wages.

(b)migrate for better jobs.

(c)adopt small family norms.

(d)develop the natural resources.

[UGC D-2005]

27.Diamond mines are located in


(a)Uttar Pradesh.

(b)Karnataka.

(c)Madhya Pradesh.

(d)Gujarat.

[UPSC-1994]

28.Diversified cropping system in subsistence agricultural system is


governed by

1.local demands.

2.local resource base.

3.market forces.

4.technological know how.

Select the correct codes.

(a)1, 2 and 3 are correct.

(b)1, 2 and 4 are correct.

(c)2, 3 and 4 are correct.

(d)1, 3 and 4 are correct.

[UGC J-2010]

29.Drift of material along a shore from opposite directions results in the


building of a pointed depositional feature is known as
(a)offshore bar.

(b)offshore horn.

(c)cuspate bar.

(d)tombolo.

[UPSC-1993]

30.During Pleistocene epoch, four glacial phases were identified. Which one
of the following is the correct chronological order in which they appeared
during this epoch from early times'?

(a)Gunz, Mindel, Wurm, Riss

(b)Gunz, Mindel, Riss, Wurm

(c)Wurm, Riss, Mindel, Gunz

(d)Mindel, Gunz, Riss, Wurm

[UPSC-2007]

31.Factors that contribute to the minimization of regional imbalances in


development are many. Which one of the following is the most relevant'?

(a)Increasing accessibility

(b)Strengthening forward and backward linkages of regional economies

(c)Investment in less developed regions.

(d)Planning the development of metropolitan centres.


[UGC D-2007]

32.Following sectors are important consumers of natural gas in India:

1.Power

2.Industry

3.Fertilizers

4.Production of liquefied petroleum gas

The correct sequence of the descending order in terms of the consumption


of natural gas by them is

(a)1, 3, 2, 4.

(b)1, 3, 4, 2.

(c)2, 1, 3, 4.

(d)2, 4, 3, 1.

[UPSC-1994]

33.Given below are two statements, one is labelled as Assertion (A) and the
other is labelled as Reason (R). Select your answers from the codes given.

Assertion (A): When sand arrives at a particular section of the beach more
rapidly than it is carried away, the beach is widened and built ocean ward.

Reason (R): The progradation process is everywhere common in the


coastal environment.

(a)(A) is correct, but (R) is wrong.


(b)(A) is wrong, but (R) is correct.

(c)Both (A) and (R) are correct.

(d)Both (A) and (R) are wrong.

[UGC J-2010]

34.Given below are two statements, one labelled as Assertion (A) and the
other labelled as Reason (R). Select your answer from the codes given.

Assertion (A): Ratzel's concept of `Lebensraum' was based on the idea


that `State' is similar to the `Organism'.

Reason (R): Ratzel was a determinist and was under the influence of
Darwin's theory of `survival of the fittest'.

(a)Both (A) and (R) are true and (R) is the correct explanation of (A).

(b)Both (A) and (R) are true but, (R) is not the correct explanation of (A).

(c)(A) is true but, (R) is false.

(d)(R) is true but, (A) is false.

[UGC D-2008]

35.Griffith Taylor applied the concept of `zones and strata' for the

(a)evolution of the races of mankind.

(b)evolution of cities and towns.

(c)spread of civilization.
(d)diffusion of settled agriculture.

[UPSC-1994]

36.Which of the following is NOT a common method of disposing of solid


waste'?

(a)Recycling

(b)Oceanic submergence

(c)Landfills

(d)Incineration

37.Nearly all transportation systems rely on this energy source.

(a)Oil

(b)Wood

(c)Solar energy

(d)Coal

38.OPEC

(a)is a cartel of oil-producing countries.

(b)is always in total agreement in matters of production and price of oil.

(c)has been able to take complete control over oil prices.

(d)consists of Middle Eastern countries only.


39.Hydroelectric project is located at

(a)Pykara.

(b)Korba.

(c)Debari.

(d)Haldia.

[UPSC-1994]

40.I.R. 20 and Ratna are two important high-yielding varieties of

(a)wheat.

(b)bajra.

(c)paddy.

(d)jowar.

[UPSC-1993]

41.In Brazil, iron ore is found chiefly in which one of the following Federal
Units'?

(a)Amazons

(b)Maranhao

(c)Minas Gerais

(d)Rio Grande do Norte

[UPSC-2008]
42.In respect of which one of the following crops in Green Revolution did
India have limited impact'?

(a)Maize

(b)Wheat

(c)Rice

(d)Pulses

[UPSC-1993]

43.In South-East Asia, the highest population density is found in

(a)Vietnam.

(b)Cambodia.

(c)Malaysia.

(d)Indonesia.

[UP S C - 1994]

44.In USA, which one of the following regions receives heavy rainfall
throughout the year under the influence of Westerlies?

(a)North-western

(b)North-eastern

(c)South-western

(d)South-eastern
[UPSC-2009]

45.In which geological period did the shallow sea reptile ichthyosaurus first
appear'?

(a)Silurian

(b)Devonian

(c)Triassic

(d)Ordovician

[UPSC-2007]

46.In which one of the following groups of countries are the Nordies found'?

(a)Spain, France, Romania and Turkey

(b)Sweden, Norway, England and Germany

(c)Sweden, Norway, Germany and Italy

(d)Romania, Turkey, France and Portugal

[UPSC-1994]

47.Increase of carbon dioxide in the atmosphere due to industrial pollution


will lead to

(a)increase in surface albedo.

(b)increase in surface temperature.

(c)decrease in surface temperature.


(d)increase in snow cover of the Earth.

[UPSC-1994]

48.Indus and Brahmaputra are examples of

(a)subsequent drainage.

(b)superimposed drainage.

(c)antecedent drainage.

(d)consequent drainage.

[UGC D-2006]

49.Isodopanes are associated with

(a)line of equal height.

(b)line of equal rainfall.

(c)line of inequality.

(d)line of equal transport cost.

[UGC D-2006]

50.It is necessary to know both the latitude and longitude of a place in order
to determine

(a)local time.

(b)altitude.

(c)standard time.
(d)location.

[UGC D-2006]

51.Match List I with List II and select the correct answer from the codes
given.

Codes:

[UGC J-2010]

52.Match List I with List II and select the correct answer using the codes
given.
Codes:

[UPSC-2006]

53.What kind of rocks result when rocks eroded from higher elevations
(mountains, hills, plains) accumulate at lower elevations (such as swamps
and ocean bottoms)?

(a)Igneous

(b)Sedimentary

(c)Metamorphic

(d)Sima

54.Which one of the following countries has the highest average of road
length on a 1,000 km2area basis?
(a)India

(b)Japan

(c)USA

(d)France

[UPSC-2004]

55.Which one of the following countries makes maximum use of the


geothermal energy'?

(a)New Zealand

(b)Japan

(c)Iceland

(d)Russia

[UPSC-2004]

56.Which of the following is an example of how human actions have


changed the lithosphere?

(a)Ocean waves

(b)Agriculture

(c)Earthquakes

(d)Volcanic eruptions

57.Which of the following does NOT describe endogenic processes'?


(a)Earthquakes, volcanic eruptions and formation of mountain ranges at
plate boundaries

(b)Includes the force of plate tectonics

(c)Moves portions of Earth's surface horizontally and vertically

(d)Shapes Earth's surface externally

58.The world's longest mountain range is (a) The Andes mountain Range.

(b)beneath the ocean.

(c)the north-south system of mountains in eastern Africa.

(d)the Himalayas.

59.Most of the world's tropical rainforests lie within

(a)30°S and 60°S.

(b)60°S and 80°S.

(c)30°N and 60°N.

(d)10°N and 10°S.

60.Boreal forest climates are associated with

(a)sub-Arctic climates.

(b)Mediterranean climates.

(c)humid low-latitude climates.


(d)tundras.

61.An ecosystem is where interact with each other through the exchange of
matter, energy and stimuli.

(a)rock types

(b)oceanic biota

(c)animal groups

(d)plants and animals

62.An interdependent group of items interacting in a regular way to form a


unified whole is a

(a)system.

(b)hearth.

(c)network.

(d)toponym.

63.Diffusion

(a)terminates at a hearth.

(b)is the result of relocation only.

(c)sometimes encounters barriers.

(d)has decreased greatly in recent years due to advances in technology.


64.Earth's solid portion composed of rocks and the sediments overlying
them is the

(a)biosphere.

(b)lithosphere.

(c)hydrosphere.

(d)atmosphere.

65.Which of the following is NOT an example of a positive externality?

(a)National security

(b)Transportation infrastructure

(c)Food supply

(d)Education

66.The system found in some Asian countries where the government does
not own much outright but it does plan and regulate the economy is called

(a)state-directed capitalism.

(b)positive externalities.

(c)negative externalities.

(d)chaebol.

67.There has been a transportation revolution in the USA called just-in-time


manufacturing,which saves all of the following EXCEPT
(a)transport costs.

(b)storage costs.

(c)opportunity costs on inventory.

(d)natural resources.

68.In a communist political economy, natural resources and productive


enterprises are owned by

(a)corporations.

(b)foreigners.

(c)the government.

(d)individuals.

69.is an example of a prorupt state.

(a)Thailand

(b)Chile

(c)Zimbabwe

(d)Poland

70.A nation-state

(a)is a concept that was old by the time of the Roman Empire.

(b)rules a territory that contains all members of a nation.


(c)must be recognized by the United Nations Security Council in order to
be legitimate.

(d)is very large in most cases.

71.China is the leading producer of , but the USA is also a major producer
and user.

(a)solar energy

(b)natural gas

(c)coal

(d)wood

72.Russia is the leading producer of

(a)wood.

(b)natural gas.

(c)coal.

(d)solar energy.

73.In the absence of cut off, how does a river meander loop behave over
time'?

(a)The gradient is lowered as the loop shortens, and the channel migrates
towards the cut or inner bank of the loop.

(b)The gradient is raised as the loop lengthens, and the channel migrates
towards the cut or inner bank of the loop.
(c)The gradient is lowered as the channel lengthens and migrates towards
the cut or outer bank of the loop.

(d)The gradient is raised as the loop shortens, and the channel migrates
away from the cut or outer bank of the loop.

74.A stream pattern is developed only on growing mountains like volcanoes


or where the land surface is tectonically doming upward.

(a)dendritic

(b)boreal

(c)trellis

(d)radial

75.Globalization in the form of the new production of irrigated cash crops


brought this change to the rural Sudanese village studied by geographer
Cindi Katz:

(a)The traditional roles and activities of boys and girls changed.

(b)Subsistence crops were no longer grown.

(c)Children were required to attend school.

(d)Wal-Mart

76.In which area would weathering by frost wedging probably be most


effective?

(a)In moist, temperate climates


(b)In cool high desert areas

(c)In a moist, tropical forest

(d)Where the subsoil is permanently frozen

1.Match List I with List II and select the correct answer using the codes
given.

Codes:

[UPSC-2006]

2.Match List I with List II and select the correct answer using the codes
given.
Codes:

[UPSC-2009]

3.Regions on the rain shadow (leeward) side of mountains (such as the


Rockies and Andes) are generally

(a)tundra.

(b)humid continental.

(c)Mediterranean.

(d)semi-arid.

4.China, South Korea, the United Arab Emirates and Saudi Arabia are each
buying and leasing millions of hectares of land around the world

(a)on which to grow food for their own populations.

(b)to develop colonies for their expanding populations.


(c)to explore for minerals and energy resources.

(d)as real-estate investments for their surplus money.

5.In tropical climates, the combination of withc creates intense daily


onvective storms.

(a)low temperatures, dry air

(b)high temperatures, dry air

(c)low temperatures, humidity

(d)high temperatures, humidity

6.Match List I with List II and select the correct answer using the code
given.

Codes:

[UPSC-2009]
7.Assertion (A): Paper industry is located mainly along the southern edge of
the coniferous forest belt in North America.

Reason (R): These areas have abundant raw materials and water power.

(a)Both (A) and (R) are true and (R) is the correct explanation of (A).

(b)Both (A) and (R) are true but, (R) is not the correct explanation of (A).

(c)(A) is true but, (R) is false.

(d)(R) is true but, (A) is false.

[UPSC-1994]

8.Assertion (A): The late mature stage is the last stage of city development
according to Taylor.

Reason (R): Taylor has suggested seven stages in the development of an


urban centre.

(a)Both (A) and (R) are true and (R) is the correct explanation of (A).

(b)Both (A) and (R) are true but, (R) is not the correct explanation of (A).

(c)(A) is true but, (R) is false.

(d)(R) is true but, (A) is false.

[UPSC-1994]

9.Match List I with List II and select the correct answer using the codes
given.
Codes:

[UPSC-2009]

10.Match List I with List II and select the correct answer using the codes
given.

Codes:
[UPSC-2009]

11.A ridge of material deposited at the end of a glacier is

(a)an outwash plain.

(b)a lateral moraine.

(c)a terminal moraine.

(d)an accumulation zone.

12.In which zone would temperature vary more in a single day than between
months'?

(a)Polar areas

(b)Low latitudes

(c)Highlands

(d)Mid-latitudes

13.Areas near oceans havethan areas in the interior of continents because of


the great storage capacity of water.

(a)warmer winters, warmer summers

(b)cooler winters, cooler summers


(c)cooler winters, warmer summers

(d)warmer winters, cooler summers

14.Which of the following are NOT results of post glaciation'?

(a)U-shaped valleys

(b)V-shaped valleys

(c)Sharp-edged mountain ridges

(d)New lakes and streams

15.Match List I with List II and select the correct answer using the codes
given.

Codes:

[UPSC-2009]
16.Many of the sites that are sacred for India's Hindus are located

(a)near the centres of major cities.

(b)on the border with Bangladesh.

(c)in the highlands of Tibet.

(d)near rivers.

17.Match List I with List II and select the correct answer using the codes
given.

Codes:

[UPSC-1994]
18.Match List I with List II and select the correct answer using the codes
given.

Codes:

[UPSC-1994]

19.Match List I with List II and select the correct answer using the codes
given.

Codes:
[UPSC-1994]

20.Match List I with List II and select the correct answer using the codes
given.

Codes:

[UPSC-1993]

21.Match List I with List II and select the correct answer using the codes
given.
Codes:

[UPSC-1993]

22.Match List I with List II and select the correct answer using the codes
given.

Codes:
[UPSC-1993]

23.Match List I with List II and select the correct answer using the codes
given.

Codes:

[UPSC-2006]

24.Match List I with List II and select the correct answer from the codes
given.

Codes:
[UGC D-2005]

25.Match List I with List II and select the correct answer from the codes
given.

Codes:

[UGC D-2005]

26.Match List I with List II and select the correct answer from the codes
given.
Codes:

[UGC D-2005]

27.Match List I with List II and select the correct answer from the codes
given.

Codes:
[UGC D-2009]

28.Match the association of the following ethnic groups in List I with the
List II and selectthe correct answer from the codes given.

Codes:

[UGC J-2010]

29.Match List I with List II and select the correct answer from the codes
given.
Codes:

[UGC D-2009]

30.Meghalaya is known for the following tribal combination:

(a)Khasi - Mizo

(b)Garo - Khasi

(c)Naga - Mizo

(d)Khasi - Naga

[UGC J-2008]

31.Moscow's engineering workshops and textile factories are supplied with


coal from the

(a)Valdai hills to the north.

(b)Tula coalfield to the south.


(c)Ural mountains.

(d)Caucasus mountains.

[UPSC-1994]

32.Nodal regions are delimited

(a)on the basis of spatial interaction.

(b)on the basis of uniformity.

(c)on the basis of resource base.

(d)on the basis of flow of goods.

[UGC D-2006]

33.pH value of moderately alkaline soils varies between

(a)4.5 and 5.0.

(b)5.0 and 5.5.

(c)5.8 and 6.4.

(d)7.8 and 8.4.

[UGC J-2010]

34.San Andreas Fault passes through which of the following'?

(a)Beaufort Sea and Rocky Mountains

(b)Pacific Ocean, north of San Francisco and Gulf of California


(c)Bering Strait and Alaska Range

(d)Mackenzie Mountains and Western Cordillera

[UPSC-2007]

35.The natural region produced by the creation of climate homogeneity by


the sea is

(a)humid subtropics.

(b)marine west coast.

(c)monsoon tropics.

(d)short summer humid continental.

[UPSC-1995]

36.Which one of the following activities occupies the largest areas of the
world?

(a)Commercial plantation

(b)Pastoral herding

(c)Commercial grain farming

(d)Subsistence agriculture

[UPSC-2004]

37.Seifs are produced by

(a)gravity.
(b)winds.

(c)waves.

(d)stream.

[UGC D-2005]

38.Soil erosion in India occurs in almost all the littoral states but it is most
serious along the coast of

(a)Kerala.

(b)Tamil Nadu.

(c)Orissa.

(d)Karnataka.

[UPSC-1994]

39.Soil that owes its colour to oxides of iron is

(a)regur.

(b)bangar.

(c)laterite.

(d)alkaline.

[UPSC-1994]

40.Spatial interaction is reflected in the flow of

1.people.
2.goods.

3.information.

Select the correct answer from the codes given.

(a)1, 2 and 3

(b)1 and 2

(c)1 and 3

(d)2 and 3

[UPSC-1993]

41.Summer rains in Australia broadly decreases from

(a)east to west.

(b)west to east.

(c)north to south.

(d)south to north.

[UPSC-2009]

42.Terra rossa typically develops in terrains composed of

(a)limestone.

(b)basalt.

(c)granite.
(d)red sandstone.

[UPSC-1993]

43.The `Hottentots' are the

(a)Pygmies of Congo (Zaire basin).

(b)Negritoes of Sri Lanka having brown colour.

(c)yellow-skinned people of south-west Africa.

(d)Negritoes of eastern Africa.

[UPSC-1994]

44.The `Agenda 21' was adopted in which of the following conventions?

(a)Stockholm convention

(b)Rio Earth summit

(c)Rotterdam convention

(d)Ramsar convention

[UGC J-2010]

45.The `cirque' is a French word which is known in Scotland as

(a)Corris.

(b)cors.

(c)Cwm.
(d)Corrie.

[UPSC-1994]

46.The Appalachian Highlands are economically important mainly because


of

(a)ready source of timber and water power.

(b)natural passes facilitating easy movement.

(c)rich mineral wealth forming the basis of flourishing industries.

(d)large number of both summer and winter tourist centres.

[UPSC-1994]

47.The Ban Sagar project, under construction, is on the river

(a)Mahanadi.

(b)Indravati.

(c)Son.

(d)Tapti.

[UPSC-1994]

48.The coal basin region called Donbas lies to the

(a)north of Black Sea and the sea of Azov.

(b)north of Caspian Sea.

(c)east of Gulf of Finland.


(d)south of White Sea and south-west of Kanin Peninsula.

[UPSC-2009]

49.The concept of the `natural region' was given by

(a)Jean Brunhes.

(b)Le Play.

(c)Carl Sauer.

(d)Vidal de la Blache.

[UGC D-2009]

50.Match List I with List II and select the correct answer using the codes
given.
Codes:

[UPSC-2003]

51.The longest inland waterway in the world is

(a)Mississippi river system.

(b)The Great Lakes.

(c)St, Lawrence.

(d)River Rhine.

[UPSC-2004]

52.The correct explanation of cyclogenesis is

(a)occurrence of difference in temperature resulting in abrupt lowering of


pressure.

(b)occurrence of difference in temperature resulting information of high


pressure.

(c)parallel moment of warm and cold fronts.

(d)interaction of land and sea breeze.

[UPSC-1994]
53.The place where Earth's crust actually moves is the of an earthquake.

(a)epicentre

(b)fault crust

(c)focus

(d)wave front

54.Earthquakes are most likely to occur

(a)in the centre of tectonic plates.

(b)at the poles.

(c)at the Equator.

(d)where two tectonic plates join.

55.Earthquakes are most likely to occur near

(a)areas where two tectonic plates meet.

(b)areas of volcanic activity in the interior of continents.

(c)continental shields.

(d)heavily populated areas.

56.Seafloor spreading

(a)is the result of subduction.

(b)was documented by Alfred Wegener.


(c)can occur at divergent plate boundaries.

(d)ended with the breakup of Pangaea.

57.If you were at a location that just experienced a thunderstorm with heavy
downpours, what could you predict about a location 25 miles to the
southwest'?

(a)It received more rainfall.

(b)It is not predictable.

(c)It received approximately the same rainfall.

(d)It received less rainfall.

58.Winds are deflected by caused by the rotation of Earth.

(a)the Milankovitch cycles

(b)the Coriolis effect

(c)the greenhouse effect

(d)gyres

59.Which climate is influenced by the ITCZ most of the year?

(a)Marine west coast

(b)Desert

(c)Humid tropical

(d)Polar
60.Most of the world's deserts occur in what zone?

(a)Mid-latitude low-pressure zone

(b)Subtropical High-pressure zone

(c)Intertropical convergence zone

(d)Polar high-pressure zones

61.GPS devices were originally developed

(a)for use by surveyors.

(b)as a navigational device for military uses.

(c)to create more accurate global maps.

(d)to send information to satellites.

62.If the scale of a map is 1:36,000, then 1 inch on the map equals how
many FEET on Earth's surface?

(a)3,000

(b)3,600

(c)1

(d)36,000

63.Laissezfaire capitalism

(a) is also known as state-directed capitalism.


(b)was adopted by the Soviet Union during the height of the Cold War.

(c)emphasizes the importance of a minimum of restrictions on economic


activity.

(d)maximizes the role of the government in the economy.

64.A crony is a

(a)business partner or friend of ruling elites.

(b)former elected official.

(c)really old person.

(d)ruling elite.

65.Canada's vast northern regions are almost empty of population but are
exploited for all of the following EXCEPT

(a)agriculture.

(b)mineral resources.

(c)hydroelectric power.

(d)timber resources.

66.In the USA and Canada, which general region has been losing
population?

(a)The Pacific Coast

(b)The East
(c)The South

(d)The central regions

67.A set of unwritten rules or unwritten ways in which written rules are
interpreted and actually enforced is

(a)a police force.

(b)an oligarchy.

(c)a constitution.

(d)a political culture.

68.A situation of disagreement between states, whether it includes warfare


or not, is called

(a)a war.

(b)a misunderstanding.

(c)an overture.

(d)a conflict.

69.A specific agreement that binds states to cooperate in a set of ways on


various issues are

(a)treaties.

(b)squabbles.

(c)considered diplomacy.
(d)geopolitics.

70.China is the leading producer of

(a)coal.

(b)oil.

(c)all fossil fuels.

(d)natural gas.

71.North America and Europe account for nearly three-fourths of the world's

(a)energy consumption.

(b)coal production.

(c)population.

(d)oil production.

72.Which of the following is NOT true of oil?

(a)It is deposited next to coal.

(b)It is a non-renewable resource from the human perspective.

(c)It was created over millions of years and is still being created.

(d)When burned it is actually releasing stored solar energy.

73.Which one of the following does not apply to stream turbulence and
average velocity'?
(a)Discharge; stream gradient

(b)Laminar flow; dissolved load

(c)Stream gradient; channel roughness

(d)Channel shape; bed roughness

74.The longing and ongoing attachments that a migrant may have to her
home land is an example of

(a)ethology.

(b)usufruct.

(c)topophilia.

(d)semiotics.

75.Which of the following best describes the process of eluviation'?

(a)Removal of very fine-sized silt and clay particles from the A and E soil
horizons

(b)Removal of soluble chemical constituents from the A and B soil


horizons

(c)Feldspar decomposition and leaching of the soluble products from the


C soil horizon

(d)Build-up of calcite in the B horizon of pedocal soils


1.The deep chernozen soil rich in humus is found in

(a)Amazon basin.

(b)Congo basin.

(c)Egypt.

(d)Ukraine.

[UPSC-1993]

2.The development of resources in a region depends mainly on

(a)population density.

(b)physical environment.

(c)needs and aspirations of the people.

(d)availability of capital.

[UGC D-2005]

3.The development of synthetic rubber industry in USA is the outcome of

(a)destruction of rubber trees by the early settlers.

(b)clearing of rubber-producing areas for agricultural purposes.

(c)Fist World War.

(d)Second World War.

[UPSC-1993]
4.The dominant type of irrigation in Peninsular India is irrigation by

(a)canal.

(b)tank.

(c)well.

(d)sprinkler.

[UPSC-1993]

5.The draught animal llamas are found in

(a)Tibet.

(b)Arctic.

(c)Alps.

(d)Andes.

[UPSC-2009]

6.believe that all of the creation, human and non-human, is fundamentally


related and should be treated equally.

(a)Ecotheologists

(b)Emissions traders

(c)Ecological imperialists

(d)Biospherical egalitarians
7.Thick layers of wind-blown silt are

(a)alluvium.

(b)rills.

(c)fluvium.

(d)loess.

8.Desert areas

(a)are typically not eroded by water.

(b)always contain sand dunes.

(c)are eroded by both wind and water.

(d)typically contain very little desert pavement.

9.The erstwhile USSR became one of the leading industry power of the
world because of her

(a)vast population.

(b)large number of skilled manpower.

(c)vast area.

(d)vast mineral and power resources.

[UPSC-1994]

10.Which of the following does (or did) NOT occupy the Temple Mount in
Jerusalem, one of the world's most contested sacred sites?
(a)The Church of the Holy Sepulchre

(b)The Al-Aqsa Mosque

(c)The Dome of the Rock

(d)The First Jewish Temple

11.The enormous volume of dust swept across the Steppe lands of Asia and
deposited in the north-western part of China is known as

(a)reg.

(b)loess.

(c)terra rossa.

(d)kankar.

[UPSC-1994]

12.The following states are involved in the export of fish and fish products:

1.Kerala

2.Maharashtra

3.Karnataka

4.Andhra Pradesh

The correct sequence of the descending order in terms of the export of


fish and fish product from these states is

(a)1, 2, 3, 4.
(b)1, 4, 2, 3.

(c)3, 2, 1, 4.

(d)2, 1, 4, 3.

[UPSC-1994]

13.The following states produce coal. Identify the correct order in which
they are arranged as per their importance.

(a) West Bengal, Bihar, Orissa, Madhya Pradesh

(b)Bihar, West Bengal, Orissa, Madhya Pradesh

(c)Orissa, Madhya Pradesh, West Bengal, Bihar

(d)Bihar, Orissa, Madhya Pradesh, West Bengal

[UPSC-1993]

14.The four major cultural realms according to Jan Brock comprise

(a)Anglo-American, Main Islamic, India and E.Asian.

(b)Occidental, Main Islamic, India and E.Asian.

(c)Latin American, Australian, New Zealand, main Islamic and E.Asian.

(d)Anglo-American, Mediterranean European, East Asian and main


Islamic.

[UPSC-1994]

15.The geographical importance of South Asia is mainly due to


(a)its religious diversity.

(b)its strategic location with reference to trade routes.

(c)its production of tin and oil.

(d)its exposure to Western influences.

[UPSC-1994]

16.The greats number and widest variety of plants and animals would be
found in

(a)tropical grasslands.

(b)tropical forests.

(c)temperate forests.

(d)temperate grasslands.

[UPSC-1993]

17.The Hanshin region of Japan is an industrial conurbation formed by


which one of the following groups of three cities?

(a)Tokyo, Kawasaki, Yokohama

(b)Yawata, Kokura, Mauji

(c)Osaka, Kobe, Kyoto

(d)Osaka, Tokyo, Kyoto

[UPSC-2009]
18.The heart of American agriculture and the most important agricultural
region in the world is the

(a)corn belt.

(b)cotton belt.

(c)hay and dairy belt.

(d)truck-farming belt.

[UPSC-1994]

19.The Heartland Theory explains

(a)interprovince relationship.

(b)centre-state relationship.

(c)struggle between land power and sea power.

(d)classification of states.

[UGC D-2009]

20.The highest yield of rice `per hectare in India is in

(a)West Bengal.

(b)Tamil Nadu.

(c)Punjab.

(d)Haryana.

[UPSC-1994]
21.The immediate cause for the south-east trades to cross the Equator and
blow as south-west monsoon winds over India is the

(a)intense low pressure over Tibet.

(b)heated Ganga plain.

(c)intense low-pressure area of Thar (Rajasthan) Desert.

(d)high temperature over Chhota Nagpur plateau.

[UPSC-1994]

22.Rain shadows in mountainous areas are a result of

(a)orographic uplift.

(b)adiabatic cooling.

(c)frontal uplift.

(d)convection.

23.If cold ocean currents hit the coast of a continent, it is likely that the
coastal area will

(a)have higher levels of evaporation than with warm currents.

(b)be relatively arid.

(c)have extensive precipitation.

(d)be subtropical.
24.Regional large-scale storms where winds converge on areas of low
pressure are

(a)hurricanes.

(b)fronts.

(c)cyclones.

(d)tornados.

25.The indigenous people living in Fiji, Papua and New Guinea belong to

(a)Mongolian group.

(b)Micronesian group.

(c)Polynesian group.

(d)Melanesian group.

[UPSC-1994]

26.The indigenous people living in the Steppes of Soviet Union are the

(a)Khirghiz.

(b)Tuaregs.

(c)Bushmen.

(d)Bhils.

[UPSC-1993]
27.The iron ore mined at Bailadila is mostly

(a)haematite.

(b)siderite.

(c)limonite.

(d)magnetite.

[UPSC-1994]

28.The large coffee plantation in Brazil is called

(a)estate.

(b)farm.

(c)Fazendas.

(d)Kolkhozes.

[UPSC-2009]

29.The leading sulfur-producing country in the world is

(a)USA.

(b)USSR.

(c)Japan.

(d)UK.

[UPSC-1993]
30.The main occupations of the Lapps would include

(a)herding and fishing.

(b)hunting and food gathering.

(c)postural nomadism.

(d)shifting cultivation.

[UPSC-1994]

31.The major coconut oil-producing country in South-East Asia is

(a)Laos.

(b)Cambodia.

(c)Malaysia.

(d)Philippines.

[UPSC-1994]

32.The major fishing grounds of the world are located on the

(a)continental shelves of the midlatitudes.

(b)deepsea plains of the North Pacific.

(c)equatorial waters of the Atlantic.

(d)equatorial waters of the Pacific.

[UPSC-1993]
33.Assertion (A): Brock has included South Africa in the occidental culture
realm.

Reason (R): The British and the Dutch have superimposed Western
culture in South Africa.

(a)Both (A) and (R) are true and (R) is the correct explanation of (A).

(b)Both (A) and (R) are true but, (R) is not the correct explanation of (A).

(c)(A) is true but, (R) is false.

(d)(R) is true but, (A) is false.

[UPSC-1994]

34.Assertion (A): Commercial fisheries have not developed in the tropics.

Reason (R): The demand for marine food from low-income population is
low in the tropics.

(a)Both (A) and (R) are true and (R) is the correct explanation of (A).

(b)Both (A) and (R) are true but, (R) is not the correct explanation of (A).

(c)(A) is true but, (R) is false.

(d)(R) is true but, (A) is false.

[UPSC-1994]

35.The Mesopotamia valley produces large quantities of

(a)wheat and orange.


(b)rice and date.

(c)maize and apple.

(d)rice and wheat.

[UPSC-1994]

36.The Mesopotamia valley produces large quantities of

(a)wheat and orange.

(b)rice and date.

(c)maize and apple.

(d)barley and oat.

[UPSC-1994]

37.The Moho is a density discontinuity in the

(a)Earth's crust.

(b)upper atmosphere.

(c)lower atmosphere.

(d)greater depths of the oceans.

[UPSC-1993]

38.The molten igneous material is called

(a)mantle.
(b)magma.

(c)sial.

(d)sima.

[UPSC-1993]

39.The most important fisheries in India are

(a)deepsea.

(b)offshore.

(c)natural inland.

(d)cultural inland.

[UPSC-1993]

40.The non-ecumene areas of the world are the

(a)sparsely populated areas.

(b)densely populated areas.

(c)areas of high fertility.

(d)areas of high mortality.

[UPSC-1993]

41.The plain formed due to coalescence of series of alluvial fans in the


piedmont zone is known as

(a)pediment.
(b)bajada.

(c)pediplains.

(d)hamada.

[UPSC-1993]

42.The presence of incised meanders indicates that the areas has experienced

(a)eustatic changes.

(b)glaciations in the past.

(c)rejuvenation.

(d)faulting.

[UPSC-1994]

43.The principal rubber-exporting nations in the world are

(a)India, USSR, China, UK.

(b)Malaysia, Indonesia, Thailand, Nigeria.

(c)Bangladesh, Burma, Pakistan, Sri Lanka.

(d)USA, Canada, Mexico, Chile.

[UPSC-1993]

44.The Siwalik hills have been made out of the debris coming from the

(a)Himalayas.
(b)Sutlej Valley.

(c)Ganga Valley.

(d)southern Peninsula.

[UPSC-1994]

45.The super-thermal plant located near Singareni Collieries is

(a)Farakka.

(b)Singrauli.

(c)Korba.

(d)Ramagundan.

[UPSC-1994]

46.Tuticorin, an important fishing harbour of India, is situated along the

(a)Malabar Coast.

(b)Coromandel Coast.

(c)Konkan Coast.

(d)Gulf of Kutch Coast.

[UPSC-1993]

47.Ukraine is an ideal industrial centre of eastern Europe because the region


is rich in

(a)iron ore, coal and manganese.


(b)copper, zinc and oil.

(c)natural gas, iron ore and manganese.

(d)iron ore, limestone and copper.

[UPSC-1994]

48.Vaal is a major tributary of which one of the following rivers?

(a)Limpopo

(b)Nile

(c)Orange

(d)Zambezi

[UPSC-2009]

49.Western Ghats are

(a)mountains.

(b)plateaus.

(c)escarpment of the plateaus.

(d)hills.

[UPSC-1994]

50.Match List I with List II and select the correct answer using the code
given.
Codes:

[UPSC-2004]

51.Match List I with List II and select the correct answer using the codes
given.

Codes:
[UPSC-2004]

52.What kind of fans occur when sediment that the stream can no longer
carry is deposited in a fan-shaped pattern'?

(a)Loess

(b)Alluvial

(c)Fluvium

(d)Rills

53.Soil creep is NOT

(a)the most common form of mass movement.

(b)more dangerous and dramatic when it occurs on steep slopes and in


wet conditions.

(c)common in areas of permafrost.

(d)the very slow, gradual movement of material down the slope of a hill.

54.On most of the Earth's land surface, what is the most important agent in
moving weathered rock fragments downhill?

(a)Wind
(b)Gravity

(c)Water

(d)Ice

55.During dry periods, most of the water flowing into streams is supplied by

(a)storms.

(b)overland flow.

(c)groundwater.

(d)rainfall.

56.What climate region is designated as humid and tropical?

(a)A

(b)B

(c)C

(d)D

57.In the low latitudes, the average temperature is between

(a)35°C and 37°C.

(b)65°C and 72°C.

(c)26°C and 28°C.

(d)50°C and 54°C.


58.On the Pacific Islands of Oceania, which climate would you expect to
find'?

(a)Tundra

(b)Humid tropical

(c)Semi-arid

(d)Humid continental

59.In 1402, a world map was produced which was by far the most accurate
representation for the times. This map was produced in which country?

(a)China

(b)The USA

(c)Spain

(d)Korea

60.In The Tribute of Yu, the outermost geographical zone was inhabited by

(a)the army.

(b)peasants.

(c)barbarians.

(d)the royal family.

61.Maps are

(a)exact replications of features found on the Earth.


(b)seldom used by geographers.

(c)designed to show all the information known about a location.

(d)two-dimensional representations of the Earth.

62.Regions

(a)are recognized by scholars only.

(b)are defined by one or more distinctive characteristics or features.

(c)are never political.

(d)are delineated by groups that insist on total agreement.

63.When a manufacturing facility is located near the market where the


products will be sold, it is called

(a)market-oriented manufacturing.

(b)raw material manufacturing.

(c)minimal cost-distance manufacturing.

(d)material-oriented manufacturing.

64.Japan invests in factories in less developed countries to manufacture


items that are low in the value-added scale. These countries include all
EXCEPT

(a)Vietnam.

(b)Singapore.
(c)Indonesia.

(d)Thailand.

65.The most rapidly increasing component of manufactured goods is

(a)raw materials.

(b)advertising.

(c)technology and capital.

(d)labour.

66.Which of the following is NOT an advantage that poor countries offer to


entrepreneurs wanting to build factories?

(a)Inexpensive labour

(b)Lack of pollution

(c)Hospitable regulatory environment

(d)Raw materials

67.A may have administrative divisions, but most decisions are made in the
national capital.

(a)unitary state

(b)compact state

(c)federal state

(d)nation-state
68.A government that is run according to the interests of its ruler or ruling
elite is

(a)a theocracy.

(b)an autocracy.

(c)an oligarchy.

(d)a democracy.

69.A nation is typically distinguished by the nation's

(a)history.

(b)culture.

(c)arts.

(d)desire for sovereignty.

70.Photovoltaic electric production

(a)is direct conversion of solar energy to electricity, such as in solar-


powered calculators.

(b)is carried in water or other liquids to places where it is needed.

(c)is heat collected directly from sunshine.

(d)is a result of the heat absorbed by collectors, such as those on rooftops.

71.What will most likely be the best source of new energy resources in the
future?
(a)Hydroelectric

(b)Solar Energy

(c)Conservation

(d)Oil

72.Which of the following is NOT true of developed countries?

(a)They contain nearly 80 percent of the world's oil reserves.

(b)They possess more than 60 percent of the world's coal.

(c)They contain more than 60 percent of the world's natural gas.

(d)They comprise about one-fourth of the world's population.

73.In a drainage pattern that is generally developed in areas underlain by


tilted or folded strata, tributary streams flow along outcrop areas of the
softer strata.

(a)dendritic

(b)coparallel

(c)trellis

(d)radial

74.Which of the following is usually the most important consideration for


the heritage industry when it decides how to restore and develop a given
landscape?
(a)Historical accuracy

(b)Principles of preservation

(c)UNESCO guidelines

(d)Commercial considerations

75.Which of the following statements concerning humus is not true'?

(a)It is less abundant in wet, tropical, forested areas than in temperate,


forested areas

(b)It consists of decaying and partly decayed leaves and other plant
materials.

(c)It is readily leached from the B horizon in weakly acidic, soil


solutions.

(d)It is typically found above the B soil horizon.

1.What does not cause regional imbalances?

(a)Uneven distribution of resources

(b)Sustainable development

(c)Lack of resources

(d)Lack of technology

[UGC D-2006]
2.What is Karaganda'?

(a)An animal

(b)A mountain

(c)A coalfield

(d)A deep ocean

[UPSC-1993]

3.Which of the following is NOT true'? The Columbian Exchange resulted


in the introduction of

(a)wheat, sugarcane, pigs, oxen and cattle to the New World.

(b)virgin soil epidemics to the New World.

(c)pests like dandelions and thistles, rats and starlings to the New World.

(d)None of the above isnot true. The Columbian Exchange resulted in all
of them.

4.Waves

(a)are usually formed by the effect of tides.

(b)have speeds that are proportional to wavelength.

(c)change more in deep water than in shallow water.

(d)travel with equal energy in all directions from the point of origin.

5.A natural region has the similarity of


(a)climate and natural vegetation.

(b)climate and occupation.

(c)soil and drainage.

(d)economic base and natural vegetation.

[UPSC-1995]

6.In which of the following countries, has maximum damage to the forests
been caused by acid rain?

(a)Spain

(b)Netherlands

(c)Poland

(d)Italy

[UPSC-2004]

7.The maximum amount of water vapour that air can hold is

(a)latent heat.

(b)saturation vapour pressure.

(c)supersaturated.

(d)relative humidity.

8.Which of the following typically does NOT cause summer thunderstorms'?


(a)Orographic uplift

(b)Frontal uplift

(c)Adiabatic cooling

(d)Convection

9.Off the coast of South America, as a result of a circulation change called


El Nino

(a)the fisheries off the coast increase in productivity.

(b)the normal coolwater flow is replaced by warm waters.

(c)no effect has been observed.

(d)the normal warmwater flow is often replaced by cool waters.

10.What can natural vegetation be expected in sub humid middle latitude


where evapotranspiration almost balances precipitation'?

(a)Conifers

(b)Prairies

(c)Savannah

(d)Selvas

[UPSC-1993]

11.Which among the following represents the correct sequence in increasing


height of different types of could formation'?
(a)Cirrus, cumulonimbus, stratus, cumulus

(b)Stratus, cumulus, cirrus cumulonimbus

(c)Cumulonimbus, cumulus, stratus, cirrus,

(d)Cirrus, cumulus, cumulonimbus, stratus

[UPSC-1993]

12.Which group contains the monsoon regions among the following'?

(a)USA, Canada, Mexico, Ecuador

(b)India, Bangladesh, Myanmar, Thailand

(c)Sri Lanka, Japan, Australia, Antarctica

(d)USSR, North China, Iran, Iraq

[UPSC-1993]

13.Which of the following are reasons for the location of the Australian
Desert?

1.Presence of a cold ocean current along the coast

2.Trade winds blowing over land for a considerable stretch

3.Existence of a mountain chain obstructing moisture-bearing winds

4.Occurrence of subtropical high pressure

Choose the correct answer from the codes given.


(a)1 and 2

(b)2 and 3

(c)1, 2 and 3

(d)1, 2, 3 and 4

[UPSC-1993]

14.Which of the following can be used for regional delineation?

(a)Gravity Potential Model

(b)Rank-Size Rule

(c)Losch Settlement Model

(d)Central Place Model

[UGC D-2006]

15.Which of the following characteristics is not true about frontier?

(a)Frontier is a zone.

(b)Frontier is outer-oriented.

(c)Frontier is inner-oriented.

(d)Frontier between early tribal societies was generally called a `no man's
land'.

[UGC D-2008]
16.Which of the following climatic types identified by Koppen occurs in
both China and Japan?

(a)Cwa

(b)Cfa

(c)Dfb

(d)Dwb

[UPSC-1993]

17.Which of the following countries of Europe had reached the last stage of
the demographic transition?

(a)Portugal

(b)France

(c)Spain

(d)Italy

[UPSC-1993]

18.Which of the following factors are responsible for India's failure to fully
exploit the inland fisheries during the last 5 decades?

1.Silting and pollution of the inland water bodies

2.Deforestation in the catchment area of the river

3.Lock of marketing facilities


Select the correct answer form the codes given.

(a)1, 2 and 3

(b)1 and 2

(c)1 and 3

(d)2 and 3

[UPSC-1994]

19.Which of the following factors is more important for the location of the
steel mills at Bokaro?

(a)Nearness to iron ore deposits

(b)Nearness to coal deposits

(c)Nearness to both coal and iron are deposits

(d)Nearness to the port city of Calcutta

[UPSC-1993]

20.Which of the following features is common to the Deccan Plateau, the


Colombian Plateau and Iceland?

(a)Composed of lava

(b)Extremely rugged

(c)Great height above sea level

(d)Inland drainage
[UPSC-1993]

21.Which of the following forest species is not characteristic of the


Deciduous Forests?

(a)Teak

(b)Sal

(c)Sandalwood

(d)Deodar

[UPSC-1993]

22.Which of the following has a potentiality for harnessing of tidal energy in


India?

(a)Gulf of Cambay

(b)Gulf of Mannar

(c)Backwaters of Kerala

(d)Chilka lake

[UPSC-1993]

23.Which of the following measures are effective for soil conservation in


India?

1.Avoiding crop rotation

2.Afforestation
3.Encouraging the use of chemical fertilizers

4.Limiting shifting cultivation

Select the correct answer form the codes given.

(a)1 and 2

(b)2 and 4

(c)3 and 4

(d)1, 2 and 3

[UPSC-1994]

24.Which of the following natural characteristics are associated with the dry
monsoon forests of India'?

(a)Annual rainfall is below 50 cm.

(b)The trees have short roots.

(c)Thorny shrubs and grasslands grow between the trees.

(d)Mango, mahua, sisam keekar, etc., are the prominent trees.

[UPSC-1994]

25.Which of the following pairs is NOT correctlymatched?

(a)Chicago: Lake Michigan

(b)Cleveland: Lake Erie


(c)Detroit: Lake Superior

(d)Toronto: Lake Ontario

[UPSC-2006]

26.Which of the following states does not have common border with
Bangladesh?

(a)Meghalaya

(b)Tripura

(c)Mizoram

(d)Assam

[UGC J-2005]

27.Which of the following steel plants of India has the least cost location in
the ideal location triangle conceived by Weber?

(a)Bokaro

(b)Bhilai

(c)Rourkela

(d)Durgapur

[UGC J-2007]

28.Which of the following were the main reasons for the great migrations
form Europe to USA?
1.Overpopulation of Europe

2.Economic prospects in Europe

3.Climatic similarity between Europe and USA

4.Adventurous spirit and navigational skills of Europeans

Select the correct answer from the codes given.

(a)1 and 2

(b)2 and 3

(c)3 and 4

(d)1 and4

[UPSC-1993]

29.Which one among the following natural regions occurs only in the
Northern Hemisphere'?

(a)Mediterranean

(b)Steppe

(c)Taiga

(d)Tropical desert

[UPSC-1993]

30.Which one of the following attributes is not a measure of human


development'?
(a)Literacy

(b)Life expectancy

(c)Per capita income

(d)Consumer expenditure

[UGC D-2007]

31.Which one of the following cities of China leads the rest in both trade
and population?

(a)Shanghai

(b)Dairen

(c)Tianjin

(d)Canton

[UPSC-1994]

32.Which one of the following factors is responsible for excessive soil


erosion in the Chhota Nagpur plateau'?

(a)Heavy rain throughout the year

(b)Loose sandy soil

(c)Deep ploughing by tractors

(d)Large-scale felling of trees

[UPSC-1994]
33.Which one of the following features does NOT apply to the polar culture
realms'?

(a)Mostly the people depend on animals.

(b)Hunting and herding are the main ways of obtaining livelihood.

(c)The Eskimosare hunters of the land animals.

(d)Mostly the people are Mongoloid.

[UPSC-1994]

34.Which one of the following groups is the product of erosion'?

(a)Drumlins, moraines, aretes

(b)U-shaped valleys, ventifacts, canyons

(c)Eskers, fiords, outwash plain

(d)Cirques, V-shaped valleys, levees

[UPSC-1994]

35.Which one of the following is droughtresistant?

(a)Xerophyte

(b)Halophyte

(c)Mesophyte

(d)Hydrophyte

[UPSC-1994]
36.Which one of the following is NOT a part of the Polynesia region?

(a)New Zealand

(b)Timor

(c)Tonga Islands

(d)Tuvalu

[UPSC-2006]

37.Which one of the following is the characteristic feature of Karst


topography?

(a)Coves

(b)Caves

(c)Caverns

(d)Cavities

[UPSC-1993]

38.Which one of the following is the correct order of the geological events
on the Earth from the ancient to more recent times?

(a)Hercynian orogeny, Caledonian orogeny, Alpine orogeny

(b)Caledonian orogeny, Hercynian orogeny, Alpine orogeny

(c)Hercynianorogeny, Alpine orogeny, Caledonian orogeny

(d)Alpine orogeny, Hercynian orogeny, Caledonian orogeny


[UPSC-2007]

39.Which one of the following is the main reason for the industrial
backwardness of Myanmar?

(a)Easy livelihood from agriculture

(b)Uneven surface of land

(c)Limited local market

(d)Lack of coal and iron ore

[UPSC-1994]

40.Which one of the following mountain ranges provides the most effective
regional division of China?

(a)Nanling

(b)Tienshan

(c)Qulianshan

(d)Tsingligshan

[UPSC-1994]

41.Which one of the following ocean currents is associated with the 'El
Nino' phenomenon'?

(a)Humboldt

(b)Benguela
(c)Canaries

(d)Kurushio

[UPSC-1993]

42.Which one of the following pairs is correctly matched?

(a)Gulf of Martaban: Yangon

(b)Gulf of Tongking: Shanghai

(c)Gulf of Mannar: Trincomalee

(d)Gulf of Aqaba: Cairo

[UPSC-2008]

43.Which one of the following pairs is correctly matched'?

(a)The Mediterranean Region: Summer rain

(b)The Equatorial Region: Afternoon thundershowers

(c)The Monsoon Region: Heavy rain throughout the year

(d)The Desert Region: Winter rain

[UPSC-1993]

44.Which one of the following physiographic units has been created by both
exogenic and endogenic forces'?

(a)The peninsular plateau

(b)The Thar Desert


(c)The Indo-Gangetic Plain

(d)The Himalayas

[UPSC-1994]

45.Which one of the following regions is called the 'breadbasket' of the


world'?

(a)Tropical monsoonal

(b)Mediterranean

(c)Temperate grassland

(d)Savannah grassland

[UPSC-1994]

46.Which one of the following rivers demarcates the partial boundary


between South Africa and Botswana'?

(a)Limpopo

(b)Zambezi

(c)Cubango

(d)Kwando

[UPSC-2009]

47.Which one of the following sequences of types of clouds would one meet
500 metres above the mean sea level'?
(a)Stratus, Nimbus, Cirrostratus, Cirrocumulus

(b)Nimbus, Cirrostratus, Cirrocumulus, Stratus

(c)Cirrocumulus, Stratus, Nimbus, Cirronimbus

(d)Stratus, Cirrostratus, Nimbus, Cirrocumulus

[UPSC-1994]

48.Which one of the following statements regarding south-west Asia is NOT


correct'?

(a)United Arab Emirates is made up of six Emirates.

(b)Yemen produces the best-quality coffee.

(c)Rice is cultivated in Iraq.

(d)Jordan grows cotton.

[UPSC-1994]

49.Which one of the following theories postulates that orogenicbelts have


roots?

(a)Kober's Mountain Building Theory

(b)Pratt's Theory of Isostasy

(c)Holmes' Convectional Current Theory

(d)Airy's Theory of Isostasy

[UPSC-1993]
50.Who propounded the concept of `Poly Climax"?

(a)Clement

(b)Tanslay

(c)Whittaker

(d)Haeckel

[UGC J-2010]

51.The combined length of all of the stream channels in a basin, divided by


the area of the drainage basin, is the

(a)slope.

(b)drainage density.

(c)isostatic adjustment.

(d)discharge.

52.The volume of water that a stream carries per unit time is

(a)drainage density.

(b)infiltration.

(c)discharge.

(d)runoff.

53.The seasonal reversal of pressure and wind over a large continent, known
as monsoon circulation, is characterized by
(a)wind blowing towards the poles.

(b)wind blowing towards the ocean in the winter and towards the
continental interior in the winter.

(c)wind blowing towards the Equator.

(d)wind blowing towards the continental interior in the summer and


towards the ocean in the winter.

54.When climbing a mountain, what would you predict about temperatures?

(a)The temperature will begin to drop above 5,000 feet.

(b)The temperature will drop about 3.5°F for every 1,000 feet climbed.

(c)The temperature will drop about 5.6°F for every 1,000 feet climbed.

(d)The temperature will be the same as the temperature at the base of the
mountain.

55.Where would higher levels of potential evapotranspiration occur on an


average'?

(a)Tropics

(b)Mid-latitudes

(c)Continental interiors

(d)Polar regions

56.San Diego, California, is in the Pacific Standard Time Zone, while


Philadelphia, Pennsylvania, is in the Eastern Standard Time Zone. If it is
1.00a.m. on Monday in Philadelphia, what time is it in San Diego?

(a)11.00p.m. Monday

(b)4.00a.m. Sunday

(c)10.00p.m. Sunday

(d)4.00a.m. Monday

57.Scientists found that the main cause of the diffusion of AIDS in sub-
Saharan Africa was

(a)people traveling to visit relatives.

(b)migrant workers.

(c)long-distance truck drivers.

(d)rhesus monkeys.

58.The first book titled Geography was

(a)Mayan.

(b)Chinese.

(c)Roman.

(d)Greek.

59.The geometrical arrangement of objects within an area is its

(a)distribution.
(b)concentration.

(c)pattern.

(d)density.

60.Assertion (A): Loess is a fine-grained yellow deposit.

Reason (R): It originated as wind blow dust form desert surface.

(a)Both (A) and (R) are true and (R) is the correct explanation of (A).

(b)Both (A) and (R) are true but, (R) is not the correct explanation of (A).

(c)(A) is true but, (R) is false.

(d)(R) is true but, (A) is false.

[UPSC-1994]

61.Assertion (A): On the west side of every continent between latitudes 20°
and 30°, intensely dry desert borders the ocean.

Reason (R): The cold current flows near the western coast of every
continent.

(a)Both (A) and (R) are true and (R) is the correct explanation of (A).

(b)Both (A) and (R) are true but, (R) is not the correct explanation of (A).

(c)(A) is true but, (R) is false.

(d)(R) is true but, (A) is false.

[UPSC-1994]
62.Assertion (A): Saline soils are commonly formed in poorly drained
locations such as low-lying valley floors and basins.

Reason (R): In poorly drained locations, the surface runoff (water)


containing dissolved salt gets totally evaporated.

(a)Both (A) and (R) are true and (R) is the correct explanation of (A).

(b)Both (A) and (R) are true but, (R) is not the correct explanation of (A).

(c)(A) is true but, (R) is false.

(d)(R) is true but, (A) is false.

[UPSC-1994]

63.Sacred spaces are sacred because

(a)of a miracle that occurred there.

(b)God makes it so.

(c)houses of worship (e.g., temples, mosques, churches, monasteries) are


located there.

(d)people make them so.

64.As manufactured products have become more and more complex, the
added by manufacturing has increased.

(a)cost

(b)transportation

(c)time
(d)value

65.Which country had the highest total manufacturing value-added output in


2011?

(a)United States

(b)Japan

(c)Germany

(d)China

66.Japan's principal asset in promoting development is

(a)high physiological density.

(b)extensive supplies of critical raw materials.

(c)a favourable ratio of population resources.

(d)an abundant supply of labour.

67.Willingness to join together to form a government to solve common


problems is

(a)a constitution.

(b)a state.

(c)an election.

(d)a political community.


68.A group of people who occupy a particular area and want to have their
own government is a

(a)nation.

(b)self-determination.

(c)unitary state.

(d)centripetal force.

69.The Swiss philosopher laid the foundations for the idea of the `social
contract'.

(a)Immanuel Kant

(b)Jean Paul Sarte

(c)Herbert Spencer

(d)Jean Jacques Rousseau

70.A condition where warmer air lies above cooler air, limiting vertical
circulation and trapping pollutants near the surface, is

(a)photochemical smog.

(b)source separation.

(c)temperature inversion.

(d)photovoltaic cells.
71.Sulfur oxides are a major component of acid rain; they come primarily
from the burning of

(a)peat.

(b)fossil fuels.

(c)animal waste.

(d)wood.

72.The `universal solvent', able to dissolve and transport a wide range of


substances, is

(a)air.

(b)soil.

(c)water.

(d)wind.

73.Which one of the following best describes how urbanization affects


small-stream watersheds'?

(a)Infiltration decreases; lag time between storms and peak runoff is


shortened.

(b)Infiltration is reduced; lag time between storms and peak runoff is


increased.

(c)Infiltration and lag time between storms and peak runoff increase.
(d)Infiltration increases slightly; lag time between storms and peak runoff
decreases.

74.Which of the following statements about laterites or tropical rainforests is


true?

(a)Laterite soils are very rich in nutrients required for vigorous plant
growth.

(b)Laterites, with proper management, are very suitable for commercial


agriculture.

(c)Laterite soils readily compact and harden when dried and exposed to
sunlight.

(d)Rainforests regenerate quickly following clear cutting or clearing for


agriculture.

75.This signal is consistently transmitted by shopping malls to all, regardless


of gender, age, class, or race.

(a)Buy low, sell high

(b)Observe social standards of behaviour

(c)Appreciate architecture

(d)Consume, consume

ANSWER KEYS

Exercise - I
Exercise - II

Exercise - III

Exercise - IV
Exercise - V

 
Note:This paper contains fifty (50) objectivetype questions, each question
carrying two (2) marks. Attempt all the questions.

1.The normal cycle of erosion is associated with

(a)marine erosion.

(b)wind erosion.

(c)river erosion.

(d)glacial erosion.

2.Which among the following is one of the forces responsible for continental
drift according to Wegner'?

(a)Motion of tectonic plates

(b)Convection currents

(c)Tensional force

(d)Compressional force

3.Match List I with List II and select the correct answer using the codes
given.
Codes:

4.Most of the details of world's topography belongs to the

(a)Pre-Cambrian era.

(b)Palaeozoic era.

(c)Mesozoic era.

(d)Cenozoic era.

5.Match List I with List II and select the correct answer using the codes
given.
Codes:

6.Air movement in the troposphere is caused by

(a)solar wind.

(b)convective current.

(c)gravitational force.

(d)air pressure.

7.The unit of air pressure measurement is

(a)degree.

(b)isobars.

(c)metres per second.

(d)hecta Pascal.
8.Match List I with List II and select the correct answer using the codes
given.

Codes:

9.Which one of the following is not amechanism of energy transfer?

(a)Conduction

(b)Advection

(c)Radiation

(d)Ablation

10.Given below are the two statements, one labelled as Assertion (A) and
other labelled as Reason (R). Select correct answer from the codes given.

Assertion (A): Stable air resists vertical movement and unstable air
ascends freely because of its own buoyancy.
Reason (R): When stable air is forced aloft, the clouds that form are
widespread and have little vertical thickness.

(a)(A) is correct but (R) is wrong.

(b)(A) is wrong but (R) is correct.

(c)Both (A) and (R) are correct.

(d)Both (A) and (R) are wrong.

11.The cold current flowing along the coast of Chile and Peru is known as

(a)Agulhas.

(b)EL-Nino.

(c)Humbolt.

(d)Canary.

12.The average difference in the water level between high tide and low tide
at a place is referred to as

(a)tidal bore.

(b)tidal period.

(c)tidal range.

(d)tidal wave.

13.Domestication of plants and animals was started in

(a)Palaeolithic period.
(b)Mesolithic period.

(c)Neolithic period.

(d)Pre-Palaeolithic period.

14.Which one of the following pairs is not correctly matched'?

(a)Autotrophs-Take energy from in organic sources and fix it into energy


rich organic com pound

(b)Heterotrophs-Obtain their energy from living organism

(c)Herbivores-Organisms that consume plants and animals

(d)Detrivores-Obtain their energy from dead organism

15.Flat-topped sea mounts are known as

(a)shoals.

(b)reefs.

(c)guyots.

(d)banks.

16.Which one of the following pairs does not match correctly'?

(a)P. Haggett-Geography: A Modern Synthesis

(b)P.E. James-All Possible Worlds

(c)G.P. Marsh-Explanation in Geography


(d)Abler, Adams and Gould-Spatial Organisation

17.Which one of the following statements is not correct?

(a)The concept of areal differentiation is related with ideographic


approach.

(b)The concept of areal differentiation is related with nomothetic


approach.

(c)The exponent of the concept of areal differentiation was Richard


Hartshorne.

(d)The concept of areal differentiation is related with chorology.

18.`The same environment carries different meanings to people with


different ways of living and culture'. This statement is related with the
concept of

(a)determinism.

(b)stop-and-go determinism.

(c)probabilism.

(d)possibilism.

19.Who among the following emphasized on the behavioural environment in


geography?

(a)Burton

(b)Gregory
(c)Kirk

(d)Johnston

20.Who defined geography as human ecology'?

(a)Schaefer

(b)Hartshorne

(c)Richthofen

(d)Barrows

21.Which one of the following metropolitan cities has registered the largest
population in 2011 census'?

(a)Chennai

(b)Delhi

(c)Kolkata

(d)Mumbai

22.Which of the following states in India is passing through the stage of


demographic transition which entails low fertility and low mortality?

(a)Assam

(b)Haryana

ab

(d)Tamil Nadu
23.Which one of the following is the important factor in rural out-migration
in India?

(a)Agricultural inefficiency

(b)Population growth

(c)Lack of basic services in rural areas

(d)Unemployment

24.The `Range of a Good' refers to

(a)the maximum distance which people have to travel to purchase a good


or service.

(b)the variety of goods available at a market place.

(c)the maximum demand to make the service or good viable.

(d)the maximum population necessary for survival of a function.

25.The `Administrative Principle' in Christaller's Central Place Theory refers


to

(a)K = 3.

(b)K=4.

(c)K = 7.

(d)K=9.

26.Which one of the following may be categorized as footloose industry'?


(a)Cement

(b)Ship building

(c)Electronic components

(d)Cotton textile

27.Gross cultivated area divided by net sown area is known as

(a)agricultural efficiency.

(b)crop diversification.

(c)crop concentration.

(d)crop intensity.

28.The Theory of Least Cost Location was proposed by

(a)Losch.

(b)Isard.

(c)Dicey.

(d)Weber.

29.The intensive cultivation of vegetables, fruits and flowers in the


periphery of cities in USA is called

(a)truck farming.

(b)factory farming.
(c)market gardening.

(d)agri farming.

30.Why are steel mills within Japan drawn to coastal locations?

(a)Nearness to iron ore reserves

(b)Nearness to local markets

(c)Availability of cheap labour

(d)To minimize transportation cost for imported raw materials and steel
exports

31.Which one of the following criteria is not used for the classification of
human races?

(a)Eye

(b)Nose

(c)Ear

(d)Hair

32.Match List I with List II and select the correct answer using the codes
given.
Codes:

33.Which of the following statements isnot true about a state'?

(a)It has a territorial base.

(b)It always has a social or a cultural base.

(c)It claims internal sovereignty.

(d)It claims sovereignty inexternal relations.

34.Which of the following languagesdoes not belong to the Dravidian


family'?

(a)Sinhalese

(b)Brahui

(c)Kannada

(d)Malayalam

35.The habitat of the Toda tribe is

(a)Aravalli range.

(b)Siwalik range.
(c)Kaimur range.

(d)Nilgiri Hills.

36.Which one of the followingindicators is most commonly used to measure


the economic development of a country'?

(a)Infant mortality rate

(b)Availability of potable water

(c)Female literacy

(d)Gross National Product percapita

37.Which of the following matches is not correct?

(a)Myrdal-Cumulative causation model

(b)Friedmann-Core-periphery model

(c)Boudville-Transport model

(d)Frank-Dependency model

38.Who among the followingformalized the concept that thereexists


relationship between the spatial organization and national development'?

(a)Luten

(b)Dickens

(c)Kuklinski

(d)Friedman
39.Which of the following sequences ofthe stages of economic
growthaccording to Rostow's model is correct'?

(a)Traditional society, drive tomaturity, take off, high mass consumption

(b)Traditional society, take off, drive to maturity, high mass consumption

(c)Traditional society, take off, high mass consumption, drive to maturity

(d)Traditional society, drive to maturity, high mass consumption, take off

40.Which one of the following is a `Formal' region'?

(a)Delhi metropolitan region

(b)Hyderabad-Secunderabad region

(c)Delhi-Jaipur-Agra Triangle

(d)Assam Valley

41.Between 2001 and 2011, the sex ratio (females per thousand males) has

(a)increased by 5 to 10.

(b)decreased by 5 to 10.

(c)remained constant.

(d)increased by less than 5.

42.Match List I with List II and select the correct answer using the codes
given.
Codes:

43.In India, mangrove is the most important tree of

(a)tropical moist deciduous forests.

(b)tidal forests.

(c)tropical dry deciduous forests.

(d)subtropical moist forests.

44.Which one of the following does not match correctly'?

(a)Vishakhapatnam-Ship building

(b)Titlagarh-Railway equipment

(c)Bhadravati-Iron and steel

(d)Pinjore-Machinetools
45.In which of the following decades did India experience the highest
percentage of decadal growth'?

(a)1951-61

(b)1961-71

(c)1971-81

(d)1981-91

46.Which one of the following is the most appropriate for showing


frequency distribution'?

(a)Bar graph

(b)Histogram

(c)Pie diagram

(d)Line graph

47.Which one of the following iscommonly used for computing theaverage


value of river discharge?

(a)Arithmetic mean

(b)Geometric mean

(c)Harmonic mean

(d)Weighted mean
48.Which one of the following Rn values shows the random pattern of point
distribution'?

(a)

(b)1.0

(c)1.35

(d)2.14

49.Which one of the following sampling techniques is most suitablefor the


selection of representativevillages of an area with diversified
geographical features?

(a)Systematic

(b)Stratified random

(c)Random

(d)Clustered

50.In which one of the following methods is interpolation used'?

(a)Choroschematic

(b)Chorochromatic

(c)Isopleth

(d)Choropleth

ANSWER KEY
Note: This paper contains fifty (50) objective type questions of two (2)
marks each. All questions are compulsory.

1.Landslides involve

(a)down slope movement of individual rock blocks which is usually not


perceptible.

(b)rapid perceptible movement of relatively dry mass of Earth debris or


bed rock.

(c)slow-to-very rapid movement of water-saturated rock debris.

(d)slow-flowing down slope masses of rock debris saturated with water.

2.Which of the following pairs is not a correct match of desert and country/
continent?

(a)Atacama desert - South America

(b)Gobi desert - Central Asia

(c)Sonoran desert - West Asia

(d)Great Victorian desert - Australia


3.One of the proponents of the turbidity current theory of the origin of
submarine canyons was

(a)Daly.

(b)Dana.

(c)Salis.

(d)Gregory.

4.The term `pan plane' refers to

(a)a level surface formed in the old age of fluvial cycle.

(b)a level surface formed by wind erosion.

(c)a plain formed by marine action.

(d)a plain formed by joining of floodplains.

5.In the process of erosion, the removal of material by solution is called

(a)attrition.

(b)corrasion.

(c)corrosion.

(d)sapping.

6.Arrange the solar spectrum in ascending order of its wavelength:

1.Ultraviolet
2.Microwaves

3.X-rays

4.Television waves

(a)4, 3, 2, 1

(b)1,3,4,2

(c)3, 1, 2, 4

(d)2, 3, 1, 4

7.Energy that is reflected by the atmosphere

(a)does not heat the atmosphere.

(b)has a different wavelength than the incoming energy.

(c)is usually short energy like visible light which does not penetrate the
atmosphere.

(d)heats the atmosphere by absorption.

8.Which one of the following statements about the polar jet stream is not
true?

(a)It moves northwards during summer.

(b)It supplies energy to the circulation of surface storms.

(c)Its location roughly coincides with that of the polar front.

(d)Its velocity is greater during summer.


9.The percentage of water vapour in the air in the tropical regions may be as
high as

(a)4 per cent.

(b)10 per cent.

(c)30 per cent.

(d)50 per cent.

10.The vertical air flow in an anticyclone results in

(a)diversion aloft.

(b)conversion aloft.

(c)diversion both aloft and at the surface.

(d)convergence at the surface and divergence aloft.

11.The most common salt in the ocean is

(a)calcium chloride.

(b)sodium nitrate.

(c)sodium chloride.

(d)calcium nitrate.

12.In the hydrological cycle, evaporation is preceded by

(a)clouds.
(b)precipitation.

(c)heat.

(d)condensation.

13.Which of the following occupiestrophic level I in food chain?

(a)Bacteria

(b)Carnivores

(c)Primary consumers

(d)Plants

14.Which of the following regions is known as the `carbon sink' of the


globe'?

(a)Antarctica

(b)Savannah

(c)North-west Europe

(d)Tropical rainforest

15.Ecological niche refers to

(a)interaction between man and environment.

(b)the functional role of an organism in the ecosystem.

(c)interdependence of man and plant.


(d)none of the above.

16.Who among the following expounded the concept that `the present is the
key to the past"?

(a)Buffon

(b)Lyell

(c)Hutton

(d)Playfair

17.The approach that some of the geographical facts according to Ritter,


cannot be explained scientifically, is termed as

(a)ecological.

(b)regional.

(c)teleological.

(d)vocational.

18.Whose pioneer statement on the scope and method of modem geography,


according to Hartshorne, set the direction of geographic thought for the
future'?

(a)Blache

(b)Schluter

(c)Ratzel
(d)Richthofen

19.Who among the following geographers emphasized the predominant role


of climate inhuman life'?

(a)Geddes

(b)Jefferson

(c)Huntington

(d)Brunhes

20.Who is usually given the credit for establishing the validity of the idea
that there was an ice age during which ice sheets covered much of the
northern Europe'?

(a)Agassiz

(b)Ramsay

(c)Jukes

(d)Penck

21.Given below are two statements, one labelled as Assertion (A) and the
other labelled as Reason (R). Select your answer from the codes given.

Assertion (A): The metropolitan cities are numerically dominated by male


adult population.

Reason (R): Migration is age and sex selective.

(a)Both (A) and (R) are correct and (R) is the correct explanation of (A).
(b)Both (A) and (R) are correct but (R) is not the correct explanation of
(A).

(c)(A) is correct, but (R) is false.

(d)(A) is false, but (R) is correct.

22.The percentage of aged population (60+) to the total population in India


lies between

(a)2 ands.

(b)5 and10.

(c)10 and 12.

(d)12 and15.

23.Which of the following migration streams experiences the maximum


migration in India'?

(a)Rural to rural

(b)Rural to urban

(c)Urban to urban

(d)Urban to rural

24.Which one of the following describes a projected urbanized world or


universal city by the end of 21 stcentury?

(a)Conurbation
(b)City-region

(c)Megalopolis

(d)Ecumenopolis

25.Which one of the following indicates `Random' distribution of


settlements?

(a)0.49

(b)1.00

(c)1.59

(d)2.15

26.The stretch ability of resource refers to

(a)the ability to find adequate substitution.

(b)susceptibility to destruction.

(c)ability to produce greater yields.

(d)non-exhaustibility.

27.Which one of the following sequences of land use according to Von


Thunen's model is correct'?

(a)Market gardening and fresh milk, firewood and lumber, intensive crop
farming
(b)Firewood and lumber, intensive crop farming, market gardening and
fresh milk

(c)Intensive crop farming, market gardening and fresh milk, firewood and
lumber

(d)Market gardening and fresh milk, intensive crop farming, firewood and
lumber

28.Resource creation is the result of

(a)interaction of natural and cultural processes.

(b)interaction of different natural processes.

(c)interaction of different social processes.

(d)interaction of different economic processes.

29.The final stage in the process of energy resource utilization is

(a)transport.

(b)storage.

(c)disposal of waste material.

(d)conversion.

30.Who among the following developed the sequential expansion transport


network model'?

(a)De Souza and Porter


(b)Brown and Bracey

(c)Myrdal, Seers and Rostow

(d)Taaffe, Morrill and Gould

31.The concept of the Geographical Pivot of History was proposed by

(a)Mackinder.

(b)Martin.

(c)Kropotkin.

(d)Ratzel.

32.Which of the following categories does Chile belong to, according to


shape'?

(a)Fragmented

(b)Fragmented cum elongated

(c)Elongated

(d)Compact cum elongated

33.The number of children dying in the first year of their birth per thousand
live births in a year is called

(a)death rate.

(b)standardized mortality rate.

(c)infant mortality rate.


(d)life expectancy rate.

34.Which one of the following arrangements of states in the descending


order of literacy rates according to2011 Census is correct?

(a)Kerala, Mizoram, Tripura, Goa

(b)Kerala, Goa, Mizoram, Tripura

(c)Goa, Kerala, Tripura, Mizoram

(d)Mizoram, Tripura, Kerala, Goa

35.Which one of the following is related to tribal economy'?

(a)Intensive cultivation

(b)Footloose industry

(c)Mining and quarrying

(d)Shifting cultivation

36.What are the bases of agro-climatic regionalization proposed by the


Planning Commission of India'?

(a)Relief, soil and drainage

(b)Rainfall, temperature and cropped area

(c)Vegetation, soil and land use

(d)Relief, land use and net sown area

37.Which category of regions does metropolitan region belongs to?


(a)Formal region

(b)Functional region

(c)Single-purpose region

(d)Natural region

38.Who among the following attempted the division of India into planning
regions?

(a)O.H.K. Spate

(b)L.D. Stamp

(c)R.L. Singh

(d)R.P. Misra

39.In which of the Five Year Plans was regional imbalance given
prominence for the first time?

(a)First

(b)Second

(c)Third

(d)Fourth

40.Which one of the following is not the ingredient of sustainable


development?

(a)Intergeneration transferability
(b)Social justice

(c)Inter-regional transferability

(d)Non-optimal resource utilization

41.The main species of the coniferousforests of the Himalayas between the


altitudes 1600 and 3300 metres are

(a)cedar, pine, sine, silver fir and spruce.

(b)sagon and sakhu.

(c)sandalwood and rose wood.

(d)oak, chestnut, chir and chil.

42.The Bundelkhand plateau covers partsof which two of the following


states'?

(a)Chhattisgarh and Jharkhand

(b)Uttar Pradesh and Madhya Pradesh

(c)Chhattisgarh and Uttar Pradesh

(d)Madhya Pradesh and Chhattisgarh

43.Which of the following places in Indiaranks second among the


coldestinhabited places in the world'?

(a)Dras

(b)Itanagar
(c)Manali

(d)Mana

44.Which of the following states has the highest percentage of its


agricultural area under canal irrigation'?

(a)Madhya Pradesh

(b)Jammu and Kashmir

(c)Uttarakhand

(d)Uttar Pradesh

45.Which of the following rangesseparates the Kashmir Valley from the


Indus River Valley'?

(a)Pir Panjal Range

(b)Dhauladhar Range

(c)Great Himalayan Range

(d)Siwalik Range

46.Which one of the following methodsis suitable to represent the


proportions of different land uses in a region'?

(a)Line graph

(b)Pie diagram

(c)Hythergraph
(d)Triangular diagram

47.The dot method of distribution refers to

(a)Iconic Model.

(b)Analogue Model.

(c)Symbolic Model.

(d)Graphical Model.

48.Which one of the following is a relative measure of dispersion'?

(a)Standard deviation

(b)Quartile deviation

(c)Coefficient of variation

(d)Range

49.If a map is drawn on RF 1/5,500, which one of the following RFs will be
correct if the map is reduced to half?

(a)1/2,750

(b)1/11,000

(c)1/5,000

(d)1/33,000

50.Consider the following statementsand select the correct answer using the
codes given.
1.NASA is the nationalacquisition and distributionagency for all satellite
data inIndia.

2.NASA sends the decoded data to the NNRMS whose activitiesare


guided by various standing committees.

(a)Only 1 is correct.

(b)Only 2 is correct.

(c)Both 1 and 2 are correct.

(d)Neither 1 nor 2 is correct.

ANSWER KEY

Note: This paper contains fifty (50) objectivetype questions of two (2) marks
each. All questions are compulsory.

1.Which one of the following is not aglaciofluvial depositional landform'?

(a)Esker

(b)Drumlin

(c)Kame
(d)Horn

2.Which one of the following arrangements of glacial periods of Pleistocene


Ice Age is in a correct chronological order?

(a)WUrm, Mindel, Gunz, Riss

(b)Riss, Mindel, Wiirm, Gunz

(c)Mindel, Wiirm, Gunz, Riss

(d)Gunz, Mindel, Riss, Wiirm

3.The rigid masses surrounding the geosyncline were termed by Kober as

(a)orogeny.

(b)kratogen.

(c)primarumph.

(d)zwischengebirge.

4.Given below are two statements, one labelled as Assertion (A) and the
other labelled as Reason (R). Select correct answer from the codes given.

Assertion (A): On Hawaii Islands, volcanic eruptions are effusive type.

Reason (R): Less viscous lavas erupt quietly.

(a)Both (A) and (R) are true and(R) is correct explanation of (A).

(b)Both (A) and (R) are true, but(R) is not correct explanation of (A).

(c)(A) is true, but (R) is false.


(d)(R) is true, but (A) is false.

5.Match List I with List II and select the correct answer from the codes
given.

Codes:

6.Which of the following is not true about the Taiga forests?

(a)The forest cover is dominated by coniferous trees.

(b)The forest floors are quite clear.

(c)Tree growth is very fast.

(d)The diameters of trees mostly range between 2 and 60 cm.

7.Surface ocean currents derive their energy from

(a)salinity.
(b)tectonics.

(c)wind.

(d)tsunamis.

8.Especially high and low tides that occur near the times of new and full
moons are called

(a)spring tide.

(b)neap tide.

(c)super tide.

(d)fall tide.

9.Given below are two statements, one labelled as Assertion (A) and the
other labelled as Reason (R). Select correct answer from the codes given.

Assertion (A): In the Arctic ecosystem, if lichen production were either


impaired or destroyed, the whole system would collapse because the
herbivorous and carnivorous directly and indirectly depend upon lichens.

Reason (R): The Arctic ecosystem has a very complex tropic structure.

(a)Both (A) and (R) are true and(R) is correct explanation of (A).

(b)Both (A) and (R) are true, but (R)is not correct explanation of (A).

(c)(A) is true, but (R) is false.

(d)(R) is true, but (A) is false.


10.Which one of the following is not true about the native animal life of the
rainy tropics?

(a)An immense number and variety of insects

(b)Few arboreal fauna

(c)Few grazing and carnivorous animals

(d)Abundant aquatic life

11.Which atmospheric gas filters out most of the ultraviolet radiation in


sunlight'?

(a)Argon

(b)Carbon dioxide

(c)Ozone

(d)Helium

12.Which one of the following best describes surfaces air circulation in a


Northern Hemisphere anticyclone?

(a)Divergent-clockwise

(b)Convergent-counter clockwise

(c)Stationary

(d)Inward-clockwise

13.Which type of front has the steepest slope'?


(a)Warm front

(b)Cold front

(c)Stationary front

(d)Occluded front

14.Which one of the following pairs is not correctly matched'?

(a)Evaporation - The process of converting a liquid to a gas.

(b)Condensation - The change of state from a gas to a liquid.

(c)Element - A substance that cannot be decomposed in to simpler


substances by ordinary chemical or physical means.

(d)Conduction - Tiny bits of particulate matter that serve as surfaces on


which water vapour condenses.

15.Which one of the following used 'Precipitation effectiveness' and


`Thermal efficiency' as the basis of world's climatic classification'?

(a)Trewartha

(b)Thornthwaite

(c)Koppen

(d)Kendrew

16.Which one of the following pairs is not correctly matched?

(a)Strabo - Published l7volumes work entitled as Geography


(b)Ptolemy - Published a famous text called Almagest or Geographike
,Syntax is

(c)Hipparchus - Designed orthographic and stereographic projections

(d)Hecataeus - Invented an instrument known as `Astrolabe' for the


determination of latitudes and longitudes

17.Who among the following geographers laid down the foundation of


dichotomy of General versus Special Geography?

(a)Bernhard Varenius

(b)Immanuel Kant

(c)Peter Apian

(d)Sebastian Munster

18.The concept of teleology was propounded by

(a)Alexander Von Humboldt.

(b)Carl Ritter.

(c)Oscar Peschel.

(d)Vidal de la Blache.

19.Who among the following geographers specifically emphasized on


climatic determinism'?

(a)Griffith Taylor
(b)Friedrich Ratzel

(c)Ellsworth Huntington

(d)Ellen Churchill Semple

20.The concept of `Areal Differentiation' is also termed as

(a)exceptionalism.

(b)chorological study.

(c)systematic geography.

(d)regionalism.

21.Which one of the following is the correct sequence of contributors in the


development of Greek geography'?

(a)Anaximander, Herodotus, Hipparchus, Eratosthenes

(b)Herodotus, Eratosthenes, Hipparchus, Anaximander

(c)Anaximander, Eratosthenes, Herodotus, Hipparchus

(d)Anaximander, Herodotus, Eratosthenes, Hipparchus

22.Match List I with List II and select the correct answer from the codes
given.
Codes:

23.Which one of the following pairs is not correctly matched'?

Proponent theory related with urban structure

(a)Homer Hoyt - Sector theory

(b)Sinclair - Concentric theory

(c)Christaller - Central place theory

(d)Harris and Ullman - Multiple nuclei theory

24.Given below are two statements: one labelled as Assertion (A) and the
other labelled as Reason (R). Select correct answer from the codes given.

Assertion (A): In the core periphery model, the core region dominates
whilst the periphery is dependent.
Reason (R): This dependence is structured mainly through relations of
exchange and production between core and periphery.

(a)Both (A) and (R) are true and (R) is correct explanation of (A).

(b)Both (A) and (R) are true, but (R) is not correct explanation of (A).

(c)(A) is true, but (R) is false.

(d)(A) is false, but (R) is true.

25.In the acceleration stage of urbanization, the urban population constitutes

(a)more than 70% of the total population of the state.

(b)25% to 70%of the total population of the state.

(c)10% to 25%of the total population of the state.

(d)less than 10%of the total population of the state.

26.In developing countries, the population pressure increases in some parts


due to immigration. Such parts are characterized by

(a)low cost of living.

(b)more suitable climate.

(c)large employment opportunities.

(d)accessibility.

27.Followings are the names given to estate farming in different parts of the
world. Match the correct pairs.
Codes:

28.The soils characterized by the dominant influence of climate are known


as

(a)zonal soils.

(b)intrazonal soils.

(c)interzonal soils.

(d)a zonal soils.

29.Which one of the following is the most important bases of Whittlesey's


classification of agriculture of the world?

(a)Functional forms of agriculture

(b)Distribution of climatic elements


(c)Forms of livestock grazing activities

(d)Regional distribution of the principal crops and animals of commercial


significance

30.Who among the followings used for the first time the ranking coefficient
method of agricultural productivity'?

(a)L.D. Stamp

(b)Mohammad Shafi

(c)M.G. Kendall

(d)G. Eneydi

31.Gibbs-Martin Index is used for the measurement of

(a)crop specialization.

(b)crop diversification.

(c)crop concentration.

(d)crop combination.

32.The most dominant blood group among the Mongoloid races is

(a)A.

(b)B.

(c)AB.

(d)0.
33.Given below are two statements, one labelled as Assertion (A) and the
other labelled as Reason (R). Select the correct answer from the codes
given.

Assertion (A): `Social indicator movement' was quickly developed in


1960s in the USA.

Reason (R): It grew in response to rise of a wide range of social problems


in U.S. society.

(a)Both (A) and (R) are true, but (R) is not correct explanation of(A).

(b)Both (A) and (R) are true and (R)is the correct explanation of (A).

(c)(A) is true, but (R) is false.

(d)(A) is false, but (R) is true.

34.Which one of the following sub races does not belong to the Caucasoid
race'?

(a)Eskimo

(b)Celtic

(c)Nordic

(d)East Baltic

35.`Regions are good servants but bad masters' was stated by

(a)C.C. Carter.

(b)A.J. Herbertson.
(c)O.H.K. Spate.

(d)Vidal de la Blache.

36. The regions developed along transportation lines or irrigation channels


are called

(a)transport regions.

(b)axial regions.

(c)river valley regions.

(d)transitional regions.

37.Which one of the following statements is not correct'?

(a)Regions are mental concepts formulated to arrange Earth's features in


some abstract order.

(b)Regions could either be finite or infinite/boundless three-dimensional


extending much beyond the limits of terrestriallsphere.

(c)The regional concept is a device to comprehend likenesses and


differences on the Earth' surface.

(d)A region has some kind of internal homogeneity which distinguishes it


from the neighbouring areas.

38.Which one of the following objectives of regional planning is most


important'?

(a)Cultivating habitability and community building


(b)Conservation of forests

(c)Rationalization of population distribution

(d)Perfect regional mobility system

39.Regionalism is the expression of

(a)ignorance of other regions.

(b)parochialism.

(c)sectionalism.

(d)local individuality, local loyalties and cultural traits.

40.Which one of the following sequences of the steel plants from east to
west is correct in India?

(a)Jamshedpur, Rourkela, Durgapur, Bhilai

(b)Bhilai, Rourkela, Durgapur, Jamshedpur

(c)Durgapur, Jamshedpur, Rourkela, Bhilai

(d)Durgapur, Rourkela, Bhilai, Jamshedpur

41.Match List I with List II and select the correct answer using the codes
given.
Codes:

42.Which one of the following states recorded the lowest population density
in India at 2011 Census'?

(a)Sikkim

(b)Arunachal Pradesh

(c)Nagaland

(d)Mizoram

43.The `Vale of Kashmir' lies between

(a)Pir Panjal and Karakoram range.

(b)Pir Panjal and Zaskar range.


(c)Zaskar and Ladakh range.

(d)Siwalik and Ladakh range.

44.Which one of the following migration types contributes most to


population movement in India?

(a)Rural to rural

(b)Urban to rural

(c)Rural to urban

(d)Urban to urban

45.Which of the following tribes is known as true `jungle nomads'?

(a)Pygmies

(b)Semang

(c)Masai

(d)Kirghiz

46.The oldest religion of the Middle East is

(a)Islam.

(b)Judaism.

(c)Christianity.

(d)Zoroastrianism.
47.Which one of the following methodsis best suited to show the
distributionof population with rural componentand urban centre of
varying sizes in a region'?

(a)Uniform dot method

(b)Multiple dot method

(c)Uniform dot and circle method

(d)Dot and sphere method

48.Match List I with List II and select the correctanswer from the codes
given.

Codes:

49.Nearest neighbour analysis was evolved by

(a)Henry and Raize.


(b)Clark and Evans.

(c)Zipf.

(d)Carl Pearson.

50.Spatial resolution of LISS IV Sensor is

(a)5.8 m.

(b)2.0 m.

(c)23.5 m.

(d)10.0 m.

ANSWER KEY

 
Note: This paper contains fifty (50) objectivetype questions of two (2) marks
each. All questions are compulsory.

1.The premise that present-day processes have been operating throughout


geological time is the principle of

(a)continental drift.

(b)uniformitarianism.

(c)Earth's systems.

(d)plate tectonics.

2.The concept of Pan plain is associated with

(a)W.M. Davis.

(b)W. Penk.

(c)A. Strahler.

(d)E. Huntington.

3.Match List I with List II and select the correct answer by using codes
given.
Codes:

4.Identify the order of the stream at the section marked `X'.

(a)2

(b)3

(c)4
(d)5

5.Match List I with List II and select the correct answer from the codes
given.

Codes:

6.Which of the following pairs of countries receive maximum insulation'?

(a)Indonesia and Sri Lanka

(b)Indonesia and Libya

(c)Yemen and Somalia

(d)Canada and Australia

7.Solid material passes directly in to vapour state by means of

(a)condensation.
(b)sublimation.

(c)volatalization.

(d)convection.

8.Which of the following pairs is correctly matched'?

1.Cwa and hot summers, drywinters

2.Cwb and hot dry summers

3.Csb and warm dry summers

4.Cfb and warm summers

Select the correct answer.

(a)1, 2, 3 and 4

(b)1, 3 and 4

(c)1 and 4

(d)1 and 2

9.Match List I with List II and select the correct answer using the codes
given.
Codes:

10.Radiolarian ooze are

(a)terrigenous deposits.

(b)neritic deposits.

(c)pelagic deposits.

(d)littoral deposits.

11.Certain substances are classified as water pollutants, others as air


pollutants and still others as land pollutants. Which one of the following
pollutants are three parts of our environment?

(a)Flourocarbons

(b)Smog

(c)Acid rain
(d)Ozone

12.Ria is an example of

(a)emerged upland shore.

(b)submerged upland shore.

(c)neutral shore.

(d)compound shore.

13.Flat-topped submarine mountains are called

(a)sea mounts.

(b)abyssal mounts.

(c)guyots.

(d)sea volcanoes.

14.Anacondas are largely found in

(a)Irrawady Basin.

(b)Congo Basin.

(c)Orinoco Basin.

(d)Amazon Basin.

15.Match List I with List II and select the correct answer from the codes
given.
Codes:

16.Who among the following did not support the concept of environmental
determinism?

(a)Ratzel

(b)Davis

(c)Huntington

(d)None of these

17.Match List I with List II and select the correct answer from the codes
given.
Codes:

18.Who wrote the book Airs, Water and Places'?

(a)Anaximander

(b)Hippocrates

(c)Eratosthones

(d)Hecateus

19.The Humanistic Geography aims at `Verstehn', that is

(a)lifestyle of man.

(b)interpersonal relations of men.

(c)understanding of man in his environment.


(d)objective experiences of man.

20.Geographia in 17 volumes was written by

(a)Ptolemy.

(b)Miletus.

(c)Strabo.

(d)Ritter.

21.Given below are two statements, one labelled as Assertion (A) and the
other labelled as Reason (R). Select your answer from the codes given.

Assertion (A): The newly emerged industrial cities in India are


numerically dominated by both male and female adult population.

Reason (R): Human migration is age and sex selective.

(a)Both (A) and (R) are correct and(R) is correct explanation of (A).

(b)Both (A) and (R) are correct, but (R) is not the correct explanation of
(A).

(c)(A) is correct, but (R) is false.

(d)(A) is false, but (R) is correct.

22.Which one of the following is not correctly matched'?

(a)Exurbanization - Commuter belt grows at the expense of core


(b)Counter urbanization - Population loss of ring is more than the urban
core

(c)Reurbanization - Core starts regaining population

(d)Exourbanization - Labour-intensive and export-oriented


industrialization is the base

23.International migration is no longer a factor in population distribution


mainly because

(a)people do not wish to migrate.

(b)people are afraid of racial prejudices.

(c)nations have placed restrictions on migration.

(d)employment opportunities are not bright.

24.Match List I with List II and select the correct answer from the codes
given.
Codes:

25.Who among the following used the term Ecumenopolis to describe a


projected urbanized world or universal city by the end of the 21st
century'?

(a)Geddes

(b)Gottmann

(c)Doxiades

(d)Dickinson

26.Match List I with List II and select the correct answer from the codes
given. The items in the two lists relate to the Sector Theory proposed by
Hoyt.
Codes:

27.What is generally not true of shifting cultivation'?

(a)Rotation of field

(b)Absence of draught animals

(c)More often than not the homestead is not shifted.

(d)It is a great catalytic force for community life.

28.Which one of the following states is the largest producer of tin?

(a)Rajasthan

(b)Odisha

(c)Jharkhand

(d)Chhatisgarh

29.The classical model of Industrial Location Theory in which least cost


approach is of primary consideration is recognized as the industrial
location theory of

(a)Weber.

(b)Smith.
(c)Isard.

(d)Hoover.

30.Kiel Canal connects

(a)Berring Sea and the Pacific Ocean.

(b)Red Sea and the Mediterranean Sea.

(c)Baltic Sea and North Sea.

(d)Caribbean Sea and the Pacific Ocean.

31.Special Economic Zone (SEZ) Policy of India was introduced in

(a)1999.

(b)2000.

(c)2001.

(d)2002.

32.In the contemporary context, which hone of the following countries is


most significant from geostrategic viewpoint'?

(a)Maldives

(b)Sri Lanka

(c)Afghanistan

(d)Bangladesh
33.`Carrot and stick' technique is used as a regional development policy in

(a)Sweden.

(b)the Netherlands.

(c)Germany.

(d)Great Britain.

34.The core of the Great Himalayas is made up of

(a)Dharwar rocks.

(b)Archaean rocks.

(c)quaternary rocks.

(d)Gondwana rocks.

35.Which one of the following rock systems is the main source of coal in
India'?

(a)Gondwana

(b)Cuddappa

(c)Vindhyan

(d)Dharwar

36.Which one of the following sets of commodities is imported by India


from South-West Asian Countries'?

(a)Raw wool and carpets


(b)Dates and olives

(c)Precious stones and pearls

(d)Perfumes and coffee

37.Which of the following states in India has the largest number of sugar
mills?

(a)Gujarat

(b)Maharashtra

(c)Bihar

(d)Uttar Pradesh

38.Match List I with List II and select the correct answer from the codes
given.

Codes:
39.The concept of Lebensraum became popular in the discipline of political
geography after the publication of

(a)Politische Geographic.

(b)Geographical Pivot of History.

(c)Political Geography: World Economy, Nation-State and Locality.

(d)Making Political Geography.

40.Pred criticized location theory on the following grounds:

(a)Logical inconsistency, motives and capacity

(b)Motives and capacity

(c)Logical inconsistency and capacity

(d)Logical inconsistency and motives

41.Match List I with List II and selectthe correct answer from the codes
given.

Codes:
42.Who coined the term Geopolitics'?

(a)Rudolf Kjellen

(b)Haushoffer

(c)Ratzel

(d)Hess

43.Who among the following is creditedwith incorporation of concept


ofgeographical space in the Growth Pole Theory?

(a)Bouldville

(b)Friedmann

(c)Myrdal

(d)Hagerstrand

44.Arrange the following maps indecreasing order of scale:

1.Atlas map

2.Wall map

3.Cadastral map
4.Topographical map

Select the answer from the codes given.

(a)2, 3, 4, 1

(b)1,2,3,4

(c)4, 3, 2, 1

(d)3, 4, 2, 1

45.Area of a drainage basin on a mapwith R.F. 1:50,000 is measured as200


sq. cm. What will be the basinarea of map when it is reduced at the R.F.
1:1,00,000?

(a)1,000 sq. cm.

(b)100 sq. cm.

(c)50 sq. cm.

(d)500 sq. cm.

46.On a topographic map, lines that indicate water depth are termed as

(a)isogonic lines.

(b)isobaths.

(c)isopachs.

(d)isonephs.
47.In order to identify Qutb Minar on an aerial photograph, which of the
followingelements of image interpretation would bemore expressive?

(a)Shape

(b)Shadow

(c)Tone

(d)Texture

48.Match List I with List II and selectthe correct answer by using the codes
given.

Codes:

49.Whose portrayals epitomized therelationship of political geography to


foreign policy'?

(a)Isaiah Bowman
(b)Friedrich Ratzel

(c)Karl Haushofer

(d)Adolf Hitler

50.Who for the first time emphasized theimportance of mental map of


environment in decision-making process'?

(a)Woolridge

(b)Downs

(c)Gould

(d)Lynch

ANSWER KEY

Note:This paper contains fifty (50) objectivetype questions of two (2) marks
each. All questions are compulsory.

1.Which of the following processes is responsible for the weathering of


rocks in the Karst region?

(a)Scree formation
(b)Carbonation

(c)Oxidation

(d)Hydrolysis

2.Match List I with List II and select the correct answer from the codes
given.

Codes:

3.Which one of these glacial features is believed to have formed in the bed
of a sub glacial stream?

(a)An esker

(b)A morraine
(c)Drumlin

(d)Kame

4.Indian Meteorological Department (IMD) has divided India into how


many seismic zones?

(a)Four

(b)Five

(c)Six

(d)Seven

5.Bad-land topography is the product of the combined action of

(a)wind and glacier.

(b)wind and water.

(c)water and glacier.

(d)water and temperature.

6.Increase in temperature with the increase in height is known as

(a)lapse rate.

(b)adiabatic lapse rate.

(c)inversion of temperature.

(d)normal rate.
7.Identify the correct sequence of the given processes regarding rainfall.

(a)Unsaturated air, condensation, dew point, precipitation

(b)Dew point, condensation, unsaturated air, precipitation

(c)Unsaturated air, dew point, condensation, precipitation

(d)Dew point, precipitation, condensation, unsaturated air

8.Read the following conditions:

1.Cloudy sky

2.Cold dry air

3.Strong winds

4.Long winter nights

Which of the above conditions promote inversion of temperature?

(a)1, 2 and 4

(b)2, 3 and 4

(c)1 and 4

(d)2 and 4

9.The climate of the southern Ganga plain in the Thornthwaite classification


is denoted by

(a)BA' W.
(b)CA' W.

(c)AA W.

(d)CW' W.

10.When the wind is deflected due to the rotation of Earth, it is known as

(a)planetary wind.

(b)geostrophic wind.

(c)accidental wind.

(d)forced wind.

11.Salinity in water bodies increases with

(a)the increase in evaporation and decrease in admixture of fresh water.

(b)the decrease in evaporation and increase in admixture of fresh water.

(c) the decrease in evaporation and no change in admixture of fresh water.

(d)the increase in evaporation and increase in admixture of fresh water.

12.Given below are two statements, one labelled as Assertion (A) and other
labelled as Reason (R). Select your answer from codes given.

Assertion (A): The salinity in Dead Sea is very low.

Reason (R): Dead Sea is located in tropical dry climatic region, where
evaporation is high, precipitation is low and here drainage is internal.

(a)Both (A) and (R) are right.


(b)Both (A) and (R) are wrong.

(c)(A) is wrong, but (R) is right.

(d)(A) is right, but (R) is wrong.

13.Which one of the following primary greenhouse gases is not associated


with global warming'?

(a)Water vapour

(b)Carbon dioxide

(c)Methane

(d)Hydrogen

14.Match List I with the List II and select the correct answer from the codes
given.

Codes:
15.Which one of the following is a Taiga Biome?

(a)Subtropical Biome

(b)Sub-Arctic Biome

(c)Savannah Biome

(d)Sub-Sahara Biome

16.Match List I with List II and select the correct answer from the codes
given.

Codes:
17.Who among the following defined geography as discovery of predictive
patterns during quantitative revolution period'?

(a)Haggett

(b)Harvey

(c)Stewart

(d)Bunge

18.In describing the patterns and processes of spatial interaction,


geographers are most concerned with

(a)accessibility and connectivity.

(b)density and dispersion.

(c)diffusion and pattern.

(d)pedestrian cities.

19.Who founded the Humanistic School of Geography?

(a)Wolpert

(b)Tuan

(c)Kirk
(d)Harvey

20.Who among the following correlated colour of ocean water and its
salinity?

(a)Ibn Khaldun

(b)Al Masudi

(c)Ibn Sina

(d)Al Maqdisi

21.`All history must be treated geographically and all geography must be


treated historically'. Who wrote this?

(a)Herodotus

(b)Ratzel

(c)Blache

(d)Davis

22.Which school of thoughts developed the science of astronomy'?

(a)Greeks

(b)Chinese

(c)Roman

(d)Arabs
23.Demographic transition is a framework that explores the historical
sequence of changes in

1.fertility and migration.

2.mortality and age-structure.

3.mortality and migration.

4.age-structure and sex composition.

(a)1 and 4 are correct.

(b)3 and 4 are correct.

(c)Only 1 is correct.

(d)1 and 2 are correct.

24.Name the state which employs the highest number of child labour in the
country.

(a)Tamil Nadu

(b)Bihar

(c)Andhra Pradesh

(d)Jharkhand

25.India's decadal population growth rate has been continuously declining


since

(a)1971-81.
(b)1981-91.

(c)1991-2001.

(d)2001-11.

26.Diego Garcia is an island in which of the following oceans'?

(a)Atlantic

(b)Pacific

(c)Indian

(d)Arctic

27.Read Assertion (A) and Reason (R) and find the correct answer from the
codes given.

Assertion (A): Urbanization is a defining phenomenon of this century and


the developing countries are at the locus of this transformation.

Reason (R): The urban shift has happened in the last few decades largely
due to rapid mega cities growing in the developing countries.

(a)Both (A) and (R) are correct and (R) explains (A).

(b)Both (A) and (R) are correct but (R) does not explain (A).

(c)(A) is correct, but (R) is false.

(d)(A) is false, but (R) is true.


28.Match List I with List II and select the correct answer using the codes
given.

Codes:

29.Marquette range in USA is known for

(a)uranium.

(b)copper.

(c)gold.

(d)iron ore.

30.The location of paper industries is highly influenced by which one of the


following factors'?

(a)Availability and nearness of raw material

(b)Nearness of market
(c)Availability of capital

(d)Transport connectivity

31.Which one of the following is the largest volume of import commodities


in India?

(a)Raw materials

(b)Iron and steel

(c)Petroleum and lubricants

(d)Pearls and precious stones

32.Kirkuk, one of the most important oilfields in the world, is located in

(a)Iran.

(b)Iraq.

(c)Kuwait.

(d)Russia.

33.To which racial groups do the Eskimos belong to'?

(a)The Negroids

(b)The Mongoloids

(c)The Caucasoids

(d)The Australoids
34.The concept of `cultural landscape' was promoted by

(a)Ratzel.

(b)Anne Buttimer.

(c)Carl Sauer.

(d)Wilbur Zelinsky.

35.Which one of the following Islamic countries is predominantly of Shia


sect?

(a)Iraq

(b)Iran

(c)Pakistan

(d)Afghanistan

36.Who, among the following, propounded the concept of compage?

(a)Minshull

(b)Blache

(c)Whittlesey

(d)Isard

37.Organic theory of the state was propounded by

(a)Mackinder.
(b)Ratzel.

(c)Haushofer.

(d)Isaiah Bawman.

38.Quasi federal form of government is in

(a)USA.

(b)Great Britain.

(c)Russia.

(d)India.

39.Which of the following was the earliest regional planning exercise in


India?

(a)National Capital Region Plan

(b)Dandakaranya Area Plan

(c)Damodar Valley Project

(d)Bhakra Nangal Project

40.Which of the following is an incorrect pair?

(a)Utilitarian Planning - Functional Planning

(b)Comprehensive Planning - Integrated Planning

(c)Regional Planning - Spatial Planning


(d)Sectoral Planning - Co-ordinated Planning

41.Which one of the following states has the longest coast line?

(a)Tamil Nadu

(b)Maharashtra

(c)Gujarat

(d)Kerala

42.In which of the following states of India is the concentration of late rite
soils higher?

(a)Odisha

(b)Gujarat

(c)Jammu and Kashmir

(d)Arunachal Pradesh

43.Given below are two statements, one labelled as Assertion (A) and the
other labelled as Reason (R). Select your answer from the codes given.

Assertion (A): Laterite soils are well developed in Kerala.

Reason (R): Kerala receives high rainfall during monsoon season.

(a)(A) and (R) are correct and (R) explains (A).

(b)(A) is correct, but (R) is false.

(c)(A) is false, but (R) is correct.


(d)(A) and (R) are correct, but (R) does not explain (A).

44.Given below are two statements, one labelled as Assertion (A) and the
other labelled as Reason (R). Select your answer from the codes given.

Assertion (A): Himalayan rivers are perennial.

Reason (R): The region gets rainfall from the south-west monsoon season
only.

(a)(A) and (R) are correct and (R) explains (A).

(b)(A) and (R) are correct, but (R) does not explain (A).

(c)(A) is correct, but (R) is false.

(d)(A) is false, but (R) is correct.

45.The `Valley of Kashmir' lies between which of the following ranges'?

(a)Pir Panjal and Karakoramrange

(b)Pir Panjal and Zaskar range

(c)Zaskar and Ladakh range

(d)Sulaiman and Kirthar range

46.Which one of the following regions has the highest proportion of


agricultural land under irrigation?

(a)Punjab-Haryana Plain

(b)Rajasthan-Gujarat Plain
(c)West Bengal Delta

(d)Uttar Pradesh-Bihar Plain

47.Consider the following figures:

Fig.: X

Fig.: Y

Which of the following statement is correct?

(a)Fig. X is small scale and the distance between A and B is 40metres.

(b)Fig. Y is large scale and thedistance between C and D is4 km.

(c)Fig. X is large scale and Fig. Yis small scale. The distancesbetween A
& B and C & D are 40 metres and 20 km, respectively.

(d)Fig. X is large scale and Fig. Yis small scale. The distancebetween A
& B and C & D areequal.

48.If the variability of rainfall is to bemeasured, which of the following


techniques will be used?

(a)Mean deviation

(b)Standard deviation
(c)Coefficient of variation

(d)Interquartile range

49.Which of the following is not a quantitative distribution map?

(a)Choroschematic map

(b)Isopleth map

(c)Dot map

(d)Choropleth map

50.Occupational structure of population in India at state level is best


represented by

(a)dot method.

(b)isopleth.

(c)choropleth.

(d)pie diagram.

ANSWER KEY

 
1.Landscapes produced by single dominant geomorphic processes are called
as:

(a)Simple landscape

(b)Compound landscape

(c)Monocyclic landscape

(d)Multicyclic landscape

2.Flat topped sand ridges with long dimensions extending parallel to the
prevailing winds but locking the collapsing fronts are known as

(a)Whalebacks

(b)Seifs

(c)Barchans

(d)Sand drifts

3.The river valleys whose courses are controlled by factors which are not
determinable are termed as

(a)Consequent

(b)Subsequent

(c)Insequent
(d)Resequent

4.Match List I with List II and select the correct answer from the codes
given below:

Codes:

5.The slope replacement model was propounded by

(a)Wood

(b)Davis

(c)Penck

(d)Strahler
6.Who among the following observed that summer heating of Tibetan
Plateau was the most important factor in the monsoonal circulation over
India?

(a)Fohn

(b)Koteswaram

(c)Frost

(d)Halley

7.According to Koppen's Scheme of climatic classification `E' (Polar or


Mountain) type climatic is found in

(a)Nagaland

(b)Arunachal Pradesh

(c)Sikkim

(d)Jammu & Kashmir

8.Which one of the following is not a factor that affects wind'?

(a)Pressure gradient

(b)Friction

(c)Magnetism

(d)Coriolis effect
9.Which one of the locations listed below should have highest annual
temperature range'?

(a)Equatorial Island

(b)Tropical coast

(c)Polar ice cap

(d)Mid-latitude continental center

10.Which term is used to describe the conversion of a solid directly to a gas,


without passing through the liquid State'?

(a)Evaporation

(b)Condensation

(c)Sublimation

(d)Freezing

11.The main Source of carbon monoxides, as air pollutants, is

(a)Thermal decomposition of fats

(b)Dye-making

(c)Blast furnaces

(d)Gasoline motor exhausts

12.The vertical movement of cold water from deeper oceanic layers to


replace warmer surface water is called as
(a)Turbidity

(b)Emergence

(c)Upwelling

(d)Halocline

13.Which one of the following types of sea floor sediments includes


calcareous and siliceous oozes'?

(a)Fluvial

(b)Terrigenous

(c)Biogenous

(d)Hydrogenous

14.Vast numbers of herbivorous animals and carnivorous such as lions,


Jackals and hyenas are found in

(a)Equatorial rainforest

(b)Taiga

(c)Campos of Brazil

(d)Savannas of Africa

15.Match List I with List II in the question given below:


Codes:

16.Nomothetic approach in geography emphasizes upon'?

(a)Inductive study

(b)Law making study

(c)Ideographic study

(d)Empirical study

17.Which one of the following pairs (Author & Book) does not match
correctly'?

(a)Ibn Khaldun Muqaddimah

(b)Al-Balakhi - Kitabul Ashkal

(c)Al-Masudi Routes and Realms


(d)Al-Biruni Kitab-al-Hind

18.Match List I with List II

Codes:

19.Which one of the following in correct sequence in the development of


German geographical thought.

(a)Troll, Peschel, Hettner, Ratzel

(b)Peschel, Ratzel, Hettner, Troll

(c)Peschel, Troll, Hettner, Ratzel

(d)Ratzel, Peschel, Troll, Hettner


20.Who coined the term "Cosmography" and divided it into uranography
and geography'?

(a)O. Peschel

(b)C. Ritter

(c)B. Varenius

(d)A. Humboldt

21.The book entitled Perspective on "Nature of Geography (1959)" was


authored by

(a)Ellen Churchill Semple

(b)Rudolf Kjellen

(c)Richard Hartshorne

(d)Isaih Bowman

22.Net population change is determined by

(a)Mortality

(b)Migration

(c)Fertility

(d)Both (a) and (c)

23.Rural house is an index of the

(a)Environment
(b)Racial Element

(c)Fashion and Style

(d)Modern Architectural Design

24.The stages of inception in the dynamics of urban growth refers to

(a)The factor which gives rise to a city in a particular place.

(b)The process through which separate functional districts are carved out

(c)The outward, often radial, movement of activities from the centre.

(d)The pattern of forcing activities out of an area.

25.The chief means of transportation in Tundras is:

(a)camel

(b)horse

(c)sledge

(d)donkeys

26.Development of Chennai-Bengaluru corridor is based on

(a)Automobile and IT industries

(b)Cotton textile and garment industry

(c)Pharmaceutical and chemical industries

(d)residential and retail development


27.Of late, the major focus in urban industrial development is of

(a)Nodal Development

(b)Cluster Development

(c)Port-City Oriented Development

(d)Both (a) and (c)

28.High level managerial and executive administrative position fall in the


category of

(a)Primary activities

(b)Secondary activities

(c)Tertiary activities

(d)Quaternary activities

29.Which of the following statements are correct'?

1.Lignite and low grade bituminous coal were formed during quaternary
period.

2.Dolomite is used as refractory material and flux in iron and steel


industry.

3.Solar energy, wind, hydel power and biogas are renewable resources.

Selectthe correct answer using codes given below:

(a)1, 2 and 3
(b)1 and 2

(c)2 and 3

(d)1 and 3

30.What are the three bases for spatial interaction as described in Edward
Ullman's model'?

(a)Complementarity, Transferability and intervening opportunities

(b)Complementarity, commodity specific relationship and surplus deficit


relationship

(c)Complimentarity, convenience and residential neighborhood

(d)Human behavior, convenience and transferability

31.Japan is one of the leading Industrial countries in the world because it has

1.Developed hydel power

2.Large deposits of metallic minerals

3.High level of technological capability

4.Insular location Which of these statements are correct'?

(a)1, 2, 3 and 4

(b)1, 2 and 3

(c)1 and 3

(d)2 and 4
32.Given below are two statements, one labelled as Assertion (A) and the
other labelled as Reason (R):

Assertion (A): With the development of H.Y.V. rubber trees and later
synthetic rubber, the rubber of the Amazon basin has become a little more
than memory.

Reason (R): Resources are created and destroyed by human culture.

Codes:

In the context of the above two statements, which of the following is


correct?

(a)(A) is wrong, but (R) is correct.

(b)Both (A) and (R) are correct and (R) explains (A).

(c)Both (A) and (R) are wrong.

(d)(A) is correct, but (R) is wrong.

33.Which one of the following culture realms is characterized by

1.heritage of ancient civilization and traditions of classical culture

2.rise of ancient city - states; and

3.importance of agriculture and horticulture including viticulture?

(a)East European

(b)Indian
(c)Mediterranean

(d)Chinese

34.The paradoxical situation of coexistence of patrilineal society and


dominant position of women characterize

(a)Khasi tribe

(b)Bhil tribe

(c)Gond tribe

(d)Tharu tribe

35.Geography of public policy is concerned with geographical study of

1.Creation and implementation of public policies.

2.Implementation and monitoring of public policies.

3.Creation and evaluation of public policies.

4.Evaluation of public policies.

(a)1 and 2 are correct.

(b)2 and 3 are correct.

(c)2 and 4 are correct.

(d)1 and 3 are correct.

36.Which one of the following languages belong to the Semito Hamitic


linguistic family?
(a)Gondi

(b)Hebrew

(c)Spanish

(d)Dutch

37.The statement that "Who rules East Europe, commands the Heartland
who rules the Heartland, commands the world Island" was made by

(a)Mackinder

(b)Spykman

(c)Alfred Mahan

(d)Whittlesey

38.The entire planning technique in India had been based on the model
prepared by

(a)P.C. Mahalanobis

(b)Fieldman

(c)Domar

(d)V.M. Dandekar

39.Regional development is a

(a)Single function approach

(b)Multifunctional approach
(c)Place prosperity

(d)Rationalization of population distribution

40.According to Hufschmidt, bring out the chronological sequence in the


main phases of development of Regional planning in the last 100 years of
its history.

1.Culture Regionalism Orientation

2.Metropolitan Orientation

3.Natural Resource Orientation

4.Regional Science Orientation

Codes:

(a)3, 2, 4, 1

(b)1, 3, 4, 2

(c)4, 1, 3, 2

(d)3, 1, 4, 2

41.Who has defined that "there is only one region the surface of the earth on
which mankind finds its home"?

(a)Minshull

(b)Vidal de la Blache

(c)Fenneman
(d)Herbertson

42.The agricultural region is a

(a)Homogenous region

(b)Functional region

(c)Nodal region

(d)Natural region

43.Which one of the following sequences of the Himalayan ranges from


South to North is correct'?

(a)Karakoram Ladakh Zaskar Pir Panjal

(b)Ladakh Zaskar Pir Panjal Karakoram

(c)Zaskar-Pir Panjal KarakoramLadakh

(d)Pir Panjal Zaskar

44.The oldest and the richest coal field of India with respect to quantity of
production is

(a)Bokaro

(b)Jharia

(c)Raniganj

(d)Makum-Nazira
45.The highest and the lowest population density difference during 2001 and
2011 was recorded respectively in

(a)Haryana and Arunachal Pradesh

(b)Uttar Pradesh and Tamil Nadu

(c)West Bengal and Nagaland

(d)Bihar and Nagaland

46.The most important source of irrigation in Great Indian Desert (Thar) is

(a)Canal

(b)Well

(c)Tube-well

(d)All of the above

47.The longest west flowing Peninsular river is

(a)Godavari

(b)Krishna

(c)Narmada

(d)Tapti

48.Match List I with List II and select the correct answer from the codes
given below:
Codes:

49.Flow maps are used to show the which type of distribution of following
data?

(a)Line

(b)Area

(c)Point

(d)Volume

50.Cumulative frequency distribution is shownby

(a)Histogram
(b)Frequency Curve

(c)Ogive

(d)Pie diagram

ANSWER KEY

 
Note: This paper contains seventy-five (75) objective-type questions of two
(2) marks each. All questions are compulsory.

1.The valleys which drain in the same direction as the original consequent
drainage but at the lower topographic levels and have developed with
respect to new base levels are known as

(a)resequent.

(b)obsequent.

(c)insequent.

(d)subsequent.

2.The concept of `base level erosion' was proposed by

(a)Davis.

(b)Malott.

(c)Johnson.

(d)Powell.

3.Given below are the two statements, one labelled as Assertion (A) and the
other labelled as Reason (R). Select correct answer from the codes given.
Assertion (A): The velocity of moving ice increases with steepness of
slope of the are a and thickness of glacial ice.

Reason (R): The velocity decreases to the sides owing to lesser depth of
ice and friction against the valley walls and the bottom floor.

(a)Both (A) and (R) are true and(R) is the correct explanation of (A).

(b)Both (A) and (R) are true, but(R) is not the correct explanation of (A).

(c)(A) is true, but (R) is false.

(d)(A) is false, but (R) is true.

4.The normal cycle of erosion is associated with

(a)marine erosion.

(b)wind erosion.

(c)river erosion.

(d)glacial erosion.

5.Which of the following groups of scholars stressed the role of lateral


erosion by streams in the formation of pediments?

(a)Mc Gee, Paige, Black welder

(b)Mc Gee, Black welder, Johnson

(c)Paige, Black welder, Johnson

(d)Paige, Johnson, Lawson


6.Which of the following is not a topographic evidence of rejuvenation in
landscape'?

(a)Incised meanders

(b)Structural benches

(c)Paired valley terraces

(d)Multi cyclic valley

7.Match List I with List II and select the correct answer from the codes
given.

Codes:

8.Given below are two statements, one labelled as Assertion (A) and the
other labelled as Reason (R). Select correct answer from the codes given.

Assertion (A): Ozone hole is observed only over Antarctica.


Reason (R): Ozone-depleting gases are present throughout the
stratosphere.

(a)Both (A) and (R) are true.

(b)(A) is true and (R) is wrong.

(c)Both (A) and (R) are wrong.

(d)(A) is wrong, but (R) is true.

9.Which of the following types of clouds appears at the highest level in the
sky?

(a)Altocumulus

(b)Cirrocumulus

(c)Cumulonimbus

(d)Stratocumulus

10.Atmospheric temperature decreases with height in the troposphere


because

(a)air at higher altitude is less dense.

(b)solar radiation is less at higher level.

(c)there are more atmospheric gases at higher altitudes.

(d)the atmosphere is heated by radiation from the surface.

11.Which one of the following pairs is not correctly matched?


(a)Foehn-Warm dry winds moving down the Alps

(b)Mistral-Cold wind blowing from Alps over France

(c)Santa Ana-Warm dry wind moving down Appalachian

(d)Bora-Cold winds experienced along the eastern coast of the Adriatic


Sea

12.Which one of the following gases is not a natural element of the


atmosphere'?

(a)Krypton

(b)Argon

(c)Chlorofluorocarbon

(d)Water vapour

13.In Koppen's classification of climate, the symbol Aw refers to

(a)tropical savannah climate.

(b)monsoon climate.

(c)tropical rainforest climate.

(d)steppe climate.

14.The strongest winds in a hurricane are found in

(a)the eye of the hurricane.

(b)the spiral bands of thunderstorm.


(c)the eye wall of the hurricane.

(d)the easterly waves.

15.Which one of the following characterizes Paternoster lake?

(a)A lake of volcanic origin

(b)A shallow stretch of water separated from the sea

(c)A crescent shaped lake formed due to cut-off of a river meander

(d)Lake formed on glacial stairways

16.Thornthwaite's modified climatic classification is based on the concept of

(a)effective temperature.

(b)precipitation index.

(c)potential evapotranspiration.

(d)potential precipitation.

17.Selva forests are

(a)broad leaf evergreen forests.

(b)broad leaf deciduous forests.

(c)coniferous evergreen forests.

(d)coniferous deciduous forests.

18.The specific place of an organism in an ecosystem is called


(a)niche.

(b)autotroph.

(c)trophic level.

(d)food pyramid.

19.Which of the following is referred to as producer in marine ecosystem'?

(a)Small fish

(b)Fungi

(c)Zooplanktons

(d)Phytoplankton

20.Saline soils are the results of

(a)very high precipitation and leaching of the soil.

(b)high rate of evaporation and very little leaching of the soil.

(c)high precipitation and very little leaching of the soil.

(d)all the above.

21.`There is no holiday for vegetationgrowth is rapid, uninterrupted and


continuous'. This statement applies to

(a)Taiga region.

(b)monsoon region.
(c)Mediterranean region.

(d)rainy tropics.

22.The largest variety of plants and animals are found in the

(a)temperate forests.

(b)monsoon forests.

(c)tropical forests.

(d)tropical grassland.

23.Which one of the following statements about salinity is not correct?

(a)Salinity is directly related to precipitation.

(b)There exists a direct relationship between the rate of evaporation and


salinity.

(c)Low salinity is found near the mouth of a river.

(d)The salinity is highest at the tropics and decreases towards poles and
Equator.

24.The origin of the ocean currents is related to

(a)gravitational force.

(b)winds.

(c)salinity and density.

(d)all of the above.


25.Schaefer was in favour of

(a)exceptionalism.

(b)idiography.

(c)areal differentiation.

(d)scientific generalization.

26.Which of the following principles does not belong to logical positivism'?

(a)Principle of causation

(b)Structuralism

(c)Behaviourism

(d)Functionalism

27.Which of the following matches is not correct'?

(a)William Bunge-Theoretical Geography

(b)David Harvey-Social Well being A Spatial Perspective

(c)David Smith-Human Geography A Welfare Approach

(d)R. Peet-Modern Geographical Thought

28.The statement that `Egypt is the gift of the river Nile' is attributed to

(a)Aristotle.

(b)Herodotus.
(c)Strabo.

(d)Seneca.

29.Who among the following pleaded that `history be treated geographically


and geography be treated historically"?

(a)Homer

(b)Thales

(c)Hecataeus

(d)Herodotus

30.Who among the following Arab scholars made corrections to Ptolemy's


book'?

(a)Al-Masudi

(b)Al-Idrisi

(c)Ibn-Khaldun

(d)Ibn-Batuta

31.Which one of the following is the philosophical basis of quantitative


revolution'?

(a)Existentialism

(b)Idealism

(c)Phenomenology
(d)Positivism

32.The concept of paradigm was propounded by

(a)Kant.

(b)Peet.

(c)Kuhn.

(d)Haggett.

33.Who among the following wanted to develop `Universal Science'


encompassing all aspects of knowledge'?

(a)Humboldt

(b)Ritter

(c)Recluse

(d)Guyot

34.Who among the following geographers focused on man-centred


geography'?

(a)W.H. Davis

(b)Jefferson

(c)Semple

(d)Huntington

35.Who, among the following, developed the concept of mental map'?


(a)Downs and Stea

(b)Gould and White

(c)Saarinen

(d)Boulding and Hagerstr and

36.Who among the following first initiated the concept of Second


Demographic Transition?

(a)Van de Kaa

(b)Lesthaeghe

(c)Coleman

(d)Fitzgerald

37.The age-specific fertility rate is maximum in the age group of

(a)20-24 years.

(b)25-29 years.

(c)30-34 years.

(d)35-39 years.

38.Which one of the following reasons dominates in rural to urban migration


in India'?

(a)Marriage

(b)Education
(c)Family movement

(d)Employment

39.Who argued that multiple centres of nuclei were responsible for urban
growth?

(a)Harris and Ullman

(b)Hoyt

(c)Park and Burgess

(d)Nelson

40.The largest and most dominating city in a state with respect to


commercial, industrial, educational and political activities refers to

(a)city region.

(b)urban field.

(c)primate city

(d)neckropolis.

41.Which one of the following sequences of land use in Burgess model is


correct'?

(a)Central business district, zone of workers' home, commuters' zone,


zone of better residences

(b)Central business district, zone of better residences, zone of workers'


home, commuters' zone
(c)Central business district, zone of workers' home, zone of better
residences, commuters' zone

(d)Central business district, zone of better residences, commuters' zone,


zone of workers' home

42.If a circle with 1 cm radius represents a population of 1 lakh, what will be


the radius of the circle representing a population of 4 lakhs?

(a)1 cm

(b)2 cm

(c)4 cm

(d)16 cm

43.Which one of the following market locations is optimal in the given


diagram of industrial landscape?

(a)b

(b)c

(c)a
(d)d

44.The essential feature of shifting cultivation is

(a)intensity of crops.

(b)rotation of fields.

(c)single cropping.

(d)double cropping.

45.The footloose industries are those for whom

(a)transportation costs are relatively unimportant.

(b)transport costs are very important.

(c)presence of raw material in the neighbourhood is necessary.

(d)all of the above are necessary.

46.Match List I with List II and select the correct answer from the codes
given.

Codes:
47.The three iron and steel plants at Kulti, Burnpur and Hirapur have been
merged together as

(a)Tata Iron and Steel Company.

(b)Indian Iron and Steel Company.

(c)Bengal Iron Works.

(d)Mysore Iron Works.

48.The rice producing areas are associated with

(a)low density of population.

(b)moderate density of population.

(c)high density of population.

(d)none of the above.

49.Match List I with List II and select the correct answer using the codes
given.
Codes:

50.Which of the following statements about the consequences of migration


is correct'?

(a)The population-resource relationship of the two areas involved in the


process of migration gets modified significantly as a result of
migration.

(b)The people who migrate from the rural areas to the urban industrial
areas suffer from the lack of open space, fresh air and good housing.

(c)The dietary habits of emigrants also change in new areas

(d)All of the above

51.The informal sector of economy refers to

(a)capitalist mode of economy.

(b)market type of economy or persistent economy.

(c)dynamic and stable income.

(d)permanent and productive establishment.

52.Which one of the following is not related with informal sector of


economy?

(a)Market type of economy

(b)Unskilled/semiskilled labour force

(c)Temporary and unstable income

(d)Capitalist mode of economy

53.The law of primacy in the context of urban development is most relevant


to the countries which have

(a)are latively simple economy and spatial structure.


(b)a complex economy.

(c)an integrated spatial structure.

(d)a matured economy.

54.The main occupation of the Gonds is

(a)food gathering.

(b)forestry.

(c)hunting.

(d)cultivation.

55.Which one of the following is largest in a real extent?

(a)Cultural realm

(b)Cultural region

(c)Cultural landscape

(d)Cultural point

56.What among the following is the correct sequence of the three stages
through which a new boundary under most conditions is determined?

(a)Delimitation, demarcation, allocation

(b)Allocation, delimitation, demarcation

(c)Demarcation, delimitation, allocation


(d)Allocation, demarcation, delimitation

57.Which one of the following is not usually considered an indicator of


social wellbeing?

(a)Infant mortality

(b)Female literacy

(c)Availability of potable water

(d)Crime

58.Who among the following was an advocate of export-based model?

(a)Alonso

(b)Kuklinski

(c)Perroux

(d)North

59.The Vale of Kashmir is the only level strip of land in the Himalayas.
Which river has laid its deposits to form this level plain'?

(a)Ravi

(b)Sutlej

(c)Beas

(d)Jhelum
60.In which of the following states of India have women out numbered
men'?

(a)Uttar Pradesh

(b)Karnataka

(c)Haryana

(d)Kerala

61.Which one of the following states of India records the lowest density of
population according to 2011 Census?

(a)Arunachal Pradesh

(b)Meghalaya

(c)Tripura

(d)Manipur

62.About 85 per cent of the total iron ore production of India is from

(a)Jharkhand and Orissa.

(b)Karnataka and Orissa.

(c)Bihar and Madhya Pradesh.

(d)Karnataka and Andhra Pradesh.

63.Name the Five Year Plan (FYP) in which the `Voluntary Sterilization
Population Policy' was introduced.
(a)First Five Year Plan

(b)Second Five Year Plan

(c)Third Five Year Plan

(d)Fourth Five Year Plan

64.In the subcontinent of India, the region frequently affected by tropical


cyclones is the

(a)Gujarat coast.

(b)Coromandel coast.

(c)Konkan coast.

(d)Malabar coast.

65.Which of the following regions/areas is not affected by the monsoon of


the Arabian Sea branch'?

(a)Western Ghat

(b)Deccan Plateau

(c)Madhya Pradesh

(d)Punjab plain

66.The heights of individual points on topographical maps are indicated by

(a)spot height only.

(b)benchmark only.
(c)triangulation point only.

(d)all of the above.

67.The broadest part of the Himalayas lies in

(a)Himachal Pradesh.

(b)Arunachal Pradesh.

(c)Jammu and Kashmir.

(d)Nagaland.

68.Which of the following regions of India is economically most


developed'?

(a)North-eastern hill region

(b)Eastern region

(c)North-western region

(d)Central region

69.If point A is at 230 m and point B at570 m elevations from mean sea
level with a horizontal equivalent of2 km, which one of the following
gradients is correct between these two points'?

(a)0.7 per cent

(b)7.0 per cent

(c)17.0 per cent


(d)27.0 per cent

70.Match List I with List II and select the correct answer using the codes
given.

Codes:

71.Which one of the following techniques is not suitable for showing urban
population on the map'?

(a)Circles

(b)Divided circles

(c)Spheres

(d)Hachures

72.Which one of the following statements is not correct?

(a)Contour maps are isopleth maps.

(b)Isopleths are lines of equal value.


(c)Isopleth technique always takes administrative boundaries in to
account.

(d)Isopleth technique is suitablefor continuous data.

73.Consider the following statementsand select the correct answer using the
codes given.

1.Deserts, marshes and hillytracts are known as thenegative areas of


population.

2.Dot method is the besttechnique to show thedistribution of population.

3.Dot method can be used toshow age and sex composition.

4.The dot map cannot betransferred into isopleths.

(a)Only 1 and 3 are correct.

(b)1, 2 and 3 are correct.

(c)1, 2 and 4 are correct.

(d)1, 2, 3 and 4 are correct.

74.Which of the following is mostsuitable to show the inequality in the


distribution?

(a)Lorenz curve

(b)Bar graph

(c)Circle diagram
(d)Isopleth

75.Which one of the following is thecorrect geometric mean of the dataset:


0, 50, 100, 100, 150, 250, 450, 500?

(a)125

(b)100

(c)200

(d)

ANSWER KEY

Note: This paper contains seventy-five (75) objective-type questions of two


(2) marks each. All questions are compulsory.

1.Most of the details of our present topography belong to the

(a)Pre-Cambrian era.

(b)Palaeozoic era.
(c)Mesozoic era.

(d)Cenozoic era.

2.If you are sitting on the beach facing the sea on a sunny afternoon, the
local wind will be

(a)striking your back and continuing to the sea because of the sea breeze
effect.

(b)towards you from the sea because of the sea breeze effect.

(c)towards you from the sea because of the lapse rate effect.

(d)striking your back and continuing to the sea because of the land breeze
effect.

3.The temperature at which water vapours condense is the

(a)relative humidity.

(b)condensation point.

(c)dew point.

(d)evaporation point.

4.Examine the following statements and select the correct answer using the
codes given:

1.The atmospheric air is never completely dry.

2.Water vapour can occupy as much as 4 percent of the atmospheric


volume.
3.Water vapour in the atmosphere is always invisible.

(a)1 and 2 are correct.

(b)2 and 3 are correct.

(c)1 and 3 are correct.

(d)1, 2 and 3 are correct.

5.Chemical weathering is more effective than mechanical weathering in

(a)semi-arid regions.

(b)and regions.

(c)coastal regions.

(d)cool temperate regions.

6.A drainage system which is not related to the structure of the region where
it occurs is called a

(a)radial drainage pattern.

(b)trellis drainage pattern.

(c)superimposed drainage pattern.

(d)dendritic drainage pattern.

7.Who calculated the slope based on the following formula'?


(a)Wentworth

(b)Raiz

(c)Henry

(d)Davis

8.Given below are the two statements one labelled as Assertion (A) and
other Labelled as Reason (R). Select correct answer from the codes given.

Assertion (A): The work of wind as an agent of gradation is not as


widespread as that of water.

Reason (R): It is effective only in the desert regions of the world where
rainfall is scanty and soil particles are loose.

(a)Both (A) and (R) are true and(R) is the correct explanation of (A).

(b)Both (A) and (R) are true but(R) is not the correct explanation of (A).

(c)(A) is true but (R) is false.

(d)(A) is false but (R) is true.

9.`The Two-Cycle Theory' of the origin of limestone caverns was proposed


by

(a)Davis.
(b)Swinerton.

(c)Gardner.

(d)Malott.

10.All those processes which tend to bring the surface of the lithosphere to a
common level are collectively known as

(a)gradation.

(b)degradation.

(c)aggradation.

(d)mass wasting.

11.Which one of the following atmospheric layers reflects radio waves that
are transmitted from the Earth again back to the Earth'?

(a)Mesosphere

(b)Ionosphere

(c)Troposphere

(d)Stratosphere

12.According to `Single-Cell Circulation' model proposed by Hadley, the


most important factor causing global atmospheric circulation is

(a)subsidence at the Equator.

(b)temperature contrast between Equator and poles.


(c)the jet stream.

(d)rotation of the Earth.

13.Which of the following would likely to have the lowest albedo?

(a)Mirror

(b)Fresh snow in the Arctic

(c)Moist ploughed soil

(d)Thick cloud

14.Given below are the two statements one labelled as Assertion (A) and the
other labelled as Reason (R). Select correct answer from the codes given.

Assertion (A): Hurricane cannot develop in the vicinity of the Equator.

Reason (R): Coriolis force is maximum over the Equator.

(a)Both (A) and (R) are true but(R) is not correct explanation.

(b)Both (A) and (R) are true and(R) is the correct explanation.

(c)(A) is true but (R) is false.

(d)(A) is false but (R) is true.

15.What happens when the Sun is shining vertically over the Tropic of
Capricorn?

(a)Days and nights are equal over the entire globe.

(b)Nights are longer than days in the Southern Hemisphere.


(c)Nights are equal to 24 hours over the Arctic region.

(d)Days and nights are of 12-hourduration each over Antarctica.

16.Which of the following statements is not true?

(a)Evaporation is directly proportional to temperature.

(b)Evaporation is more over land than ocean.

(c)Evaporation is higher over warm surface than on cold surface.

(d)Evaporation reduces the amount of heat available in the atmosphere,


thereby reducing the temperature.

17.Match List I with List II and select the correct answer by using codes
given.

Codes:
18.Pelagic deposits consist of matter derived from algae and are mostly in
the form of liquid mud known as

(a)blue mud.

(b)ooze.

(c)red mud.

(d)coral mud.

19.The average temperature of the surface water of the oceans is

(a)22°C.

(b)15°C.

(c)26.7°C.

(d)18.6°C.

20.Ecosystem has two components which are

(a)animals and plants.

(b)trees and weeds.

(c)frogs and men.


(d)abiotic and biotic.

21.All the ecosystems taken together in a geographical area form a bigger


unit called

(a)community.

(b)territory.

(c)biome.

(d)biosphere.

22.Which one of the following ocean currents belongs to the Indian Ocean?

(a)Canary current

(b)Antilles current

(c)Labrador current

(d)Agulhas current

23.The Western Coast of Australia records higher salinity due to

(a)ocean currents.

(b)wind direction.

(c)dry weather conditions.

(d)all the above.

24.The highest salinity is found in the


(a)Baltic Sea.

(b)Okhotsk Sea.

(c)Mediterranean Sea.

(d)Red Sea.

25.Which of the following statements about synecology is correct'?

(a)It is a study of individual species in relation to its environment.

(b)It is a study of complex interrelationship of biotic communities.

(c)It is essentially habitat ecology.

(d)It is mainly social ecology.

26.Organic deposits made by living or dead organisms that form rock


elevation like a ridge are known as

(a)corals.

(b)canyons.

(c)reefs.

(d)guyots.

27.Which one of the schools of thought first developed possibilism?

(a)German School of Geography

(b)Russian School of Geography


(c)French School of Geography

(d)British School of Geography

28.Which one of the following is/are not a supporter of the theory of


determinism?

(a)Wolfgang Hartake

(b)R. Hartshorne

(c)O.H.K. Spate

(d)All the above

29.Who among the following noted first the regularity between the sizes of
cities and their ranks?

(a)Zipf

(b)Jefferson

(c)Auerbach

(d)Christaller

30.Who among the following is an advocate of idealism in geography'?

(a)Guelke

(b)Gilbert

(c)Samuels

(d)Pocock
31.The statement that `a method by which one can rethink the thought of
those whose actions one seeks to explain'refers in human geography to
the philosophy of

(a)phenomenology.

(b)idealism.

(c)existentialism.

(d)positivism.

32.Which among the following was responsible for the radical trans
formation of spirit and purpose of geography'?

(a)Post Modernism

(b)Humanism

(c)Structuralism

(d)Quantification

33.Which among the following was one of the forces responsible for
continental drift according to Wegner?

(a)Tidal force

(b)Convection currents

(c)Tensional force

(d)Compress ional force


34.Which one of the following states/union territories recorded the highest
density of population as per census 2011?

(a)NCT Delhi

(b)West Bengal

(c)Kerala

(d)Maharashtra

35.If economic landscape of a region is dependent on `transport principle' of


central place system, with the population size of 4.0 lakhs of its highest
order town, which one of the following population size of next lower
order town is correct'?

(a)3.0 lakhs

(b)2.0 lakhs

(c)1.Olakh

(d)0.5 lakh

36.Who defined settlement geography first time incorporating the concept of


settlement pattern, house types and field system?

(a)Meitzen

(b)Stone

(c)Huntington

(d)Trewartha
37.The sparsely populated countries have slow growth of industries because

(a)of shortage of skilled labour.

(b)small population does not provide a good market.

(c)of both (a) and (b).

(d)agriculture is more developed than industries.

38.Which one of the following countries has the highest density of


population'?

(a)Bangladesh

(b)Pakistan

(c)Sri Lanka

(d)India

39.The size of population in lower age group is large in countries where

(a)birth rate is high.

(b)birth rate is low.

(c)death rate is high.

(d)death rate is low.

40.The base of the age-sex pyramid is broad and apex is narrow in

(a)developed countries.
(b)developing countries.

(c)underdeveloped countries.

(d)none of the above.

41.Which one of the following market locations is optimal in given diagram


of industrial landscape?

(a)c

(b)b

(c)d

(d)a

42.Which one of the following matches in case of shifting cultivation is not


correct?

(a)Ladang-Indonesia

(b)Ray-Vietnam

(c)Milpa-Mexico

(d)Tamrai-Brazil
43.The polar regions are sparsely populated because

(a)the growing season is short owing to low temperature.

(b)there are no mineral resources.

(c)the regions are forested.

(d)of both (b) and (c).

44.Which one of the following countries is the major producer of mica'?

(a)India

(b)USA

(c)Brazil

(d)China

45.Match List I with List II and select the correct answer using the codes
given.

Codes:
46.Match List I with List II and select the correct answer from the codes
given.

Codes:

47.The number of persons expressed in terms of unit area of agricultural


land is known as
(a)economic density.

(b)physiological density.

(c)arithmetical density.

(d)agricultural density.

48.Who among the following proposed a grand scheme of natural regions on


a global scale by combining physical determinants of plants and animals
life'?

(a)Unstead

(b)Herbertson

(c)Whittlesey

(d)Dickinson

49.Which of the following was not mentioned by Brunhes as essential fact


of human geography?

(a)Facts of unproductive occupation of soil

(b)Facts of plants and animal conquest

(c)Facts of destructive exploitation

(d)Facts of manufacturing

50.Which one of the following factors is not affecting the voting behaviour'?

(a)Religious
(b)Castes

(c)High level of literacy

(d)Size of landholdings

51.Which indicator from among the following could be the best indicator of
social well-being?

(a)Population growth rate

(b)Literacy rate

(c)Life expectancy

(d)Per capita income

52.Who among the followings gave the concept of Rimland Theory'?

(a)A. Ratzel

(b)O.H.K. Spate

(c)Spykman

(d)Griffith Taylor

53.Who propounded the Theory of Heartland in political geography?

(a)Mackinder

(b)Spykman

(c)Smith
(d)Raiz

54.A functional region is delineated on the basis of

(a)administrative boundaries.

(b)field of interactions.

(c)isolines.

(d)physical divisions.

55.Who in census of India applied the `ternary diagram' for the functional
classification of towns in India?

(a)P. Padmanabha

(b)A.R. Nanda

(c)Ashok Mitra

(d)Chandrasekhar

56.Match List I with List II and select the correct answer from the codes
given.
Codes:

57.Who authored the book entitled Growth Pole and Growth Centres for
Regional Economic Development in India'?

(a)Sen and Wanmali

(b)Sadasyuk and Sengupta

(c)Misra, Rao and Sundaram

(d)Rao and Sundaram


58.Which one of the following states recorded the highest decadal growth
rate of population between 2001 and 2011 as per census record?

(a)Bihar

(b)Uttar Pradesh

(c)Rajasthan

(d)Arunachal Pradesh

59.Match List I with List II and select the correct answer from the codes
given.

Codes:

60.Which one of the following rivers does not make a delta?

(a)Mahanadi
(b)Godavari

(c)Tapti

(d)Ganga

61.Which one of the following rivers does not flow through Himachal
Pradesh'?

(a)Jhelum

(b)Beas

(c)Chenab

(d)Ravi

62.Match List I with List II and select the correct answer from the codes
given.

Codes:
63.Match List I with List II and select the correct answer from the codes
given.

Codes:

64.Which one records the lowest normal temperature in the month of


January'?

(a)Bikaner

(b)Bangalore

(c)Jaisalmer

(d)Firozpur

65.Match List I with List II and select the correct answer from the codes
given.
Codes:

66.Which one of the following states has the lowest percentage of scheduled
tribes'?

(a)Arunachal Pradesh

(b)Manipur

(c)Mizoram

(d)Nagaland

67.Which of the following hills forms the north-eastern edge of the Chhota
Nagpur Plateau?

(a)Rajmahal Hills

(b)Mikir Hills
(c)Parasnath Hills

(d)Javadi Hills

68.Consider the following statements and select the correct answer using
codes given.

1.In a representative fraction (RF), the numerator is always one.

2.In an RF scale, the unit of denominator is always as of the unit of


numerator.

3.RF is not a suitable technique of showing the scale on a map.

4.The statement scale, e.g., 1 cm to 1 km is suitable scale on a map.

(a)1, 2 and 3 are correct.

(b)1, 3 and 4 are correct.

(c)2, 3 and 4 are correct.

(d)1, 2 and 4 are correct.

69.If RF of a map is 1/5,000 which is reduced three times, which of the


following RFs is correct for a reduced map'?

(a)1/2,500

(b)1/1,000

(c)1/15,000

(d)1/25,000
70.Consider the following statements and select the correct answer using the
codes given.

1.IRS is the mainstay of National Natural Resources Management System


(NNRMS) in India.

2.Development and operational responsibilities of Earth observation in


India are overseen by the National Remote Sensing Agency (NRSA).

(a)Only 1 is correct.

(b)Only 2 is correct.

(c)Both 1 and 2 are correct.

(d)Neither 1 nor 2 is correct.

71.Which one of the following statement is correct in the context of Sten-de-


Geer's method used for showing population distribution?

(a)Rural population is shown by the sphere method.

(b)Urban population is shown by the dot method.

(c)Scale of cuberoots isused for showing the spheres.

(d)Spheres occupy bigger space on the map.

72.Which one of the following measurements is used for inequality


distribution of farmer's income subject to their landholding size of an
area'?

(a)Standard deviation
(b)Coefficient of variation

(c)Gini index

(d)Correlation coefficient

73.Consider the following statements and select the correct answer using the
codes given.

1.Random sampling is also called probability sampling.

2.Random sampling is done withhelp of random table.

3.Landscape elements aregenerally scattered at random.

(a)1 and 2 are correct.

(b)2 and 3 are correct.

(c)1 and 3 are correct.

(d)1, 2 and 3 are correct.

74.Match List I with List II andselect the correct answer from the codes
given.
Codes:

75.According to Nearest Neighbourlndex, what would be the


maximumvalue for the perfect uniform settlement distribution'?

(a)0.00

(b)2.15

(c)1.55

(d)2.89

ANSWER KEY
Note: This paper contains seventy-five (75) objective-type questions of two
(2) marks each. All questions are compulsory.

1.Which of the following is not considered to be a characteristic feature of


the youthful stage of an ideal normal cycle of erosion?

(a)Natural levees

(b)Pot holes

(c)Gorges

(d)River captures

2.Sand dunes formed as long ridges oriented at right angles to the wind
direction are called

(a)linear dunes.

(b)Barchan dunes.

(c)transverse dunes.
(d)star dunes.

3.Match List I with List II and select the correct answer from the codes
given.

Codes:

4.Who stated that the slope profiles are convex, plane or concave according
to the circumstances of the uplifting action'?

(a)Davis

(b)Penck

(c)Johnson

(d)Wood
5.Given below are two statements, one labelled as Assertion (A) and the
other labelled as Reason (R). Select your answer from the codes given.

Assertion (A): Hawaii Island is a site of volcanic eruptions.

Reason (R): Convergent plate margins are sites of volcanic eruptions.

(a)Both (A) and (R) are correct and (R) is the correct explanation of (A).

(b)Both (A) and (R) are correct, but (R) is not the correct explanation of
(A).

(c)(A) is correct, but (R) is false.

(d)(A) is false, but (R) is correct.

6.Match List I with List II and select the correct answer from the codes
given.

Codes:
7.The statement `The present is the key to the past' is related to

(a)catastrophism.

(b)uniformitarianism.

(c)isostasy.

(d)diastrophism.

8.The theory of plate tectonics does not help explain the origin and location
of which one of the following'?

(a)Earthquakes

(b)Mountains

(c)Ocean currents

(d)Major sea floor features

9.Given below are two statements, one labelled as Assertion (A) and other
labelled as Reason (R). Select your answer from the codes given.

Assertion (A): The boundary between two air masses of different


densities is known as a front.
Reason (R): The difference in density is caused by differences in
temperature and humidity.

(a)Both (A) and (R) are true and (R) is the correct explanation of (A).

(b)Both (A) and (R) are true and (R) is not the correct explanation of (A).

(c)(A) is false, but (R) is true.

(d)(A) is true, but (R) is false.

10.Which of the following winds fall in the Zone of Hadley Cell'?

(a)Monsoon winds

(b)Trade winds

(c)Westerlies

(d)Polar winds

11.Which one of the following is correct when the combined effect of the
pressure gradient force and Coriolis force on air current is produced'?

(a)Deflection force

(b)Geostrophic force

(c)Frictional force

(d)Gravitational force

12.The term adiabatic change of temperature stands for

(a)fall of temperature after precipitation.


(b)heating and cooling of an ascending or descending air through
compression o expansion.

(c)heating of ascending air.

(d)cooling of descending air.

13.What are the effects of the low-level temperature inversion occurring in


cool nights over the urban areas'?

1.Entrapment of pollutants causing heavily smog

2.Pleasant weather conditions

3.Unusually unstable air

4.Unusually stable air

Select your answer using the codes given.

(a)1 and 3

(b)2 and 3

(c)1 and 4

(d)2 and 4

14.The `Ice Crystal Theory' related to precipitation was propounded by

(a)Boven.

(b)Muir.

(c)Peterson.
(d)Bergeron.

15.Which one of the following parts of the Indian coast is a compound coast
showing evidence of both submergence and emergence'?

(a)Orissa coast

(b)Malabar coast

(c)Konkan coast

(d)Coromandel coast

16.Darwin's subsidence theory is related to

(a) marine terraces.

(b)coral reefs.

(c)cycle of erosion.

(d)tides.

17.Fiord shorelines are formed due to

(a)submergence of sub aerially eroded coast.

(b)submergence of glaciated coast.

(c)fluvial deposition.

(d)marine erosion.

18.Tsunamis are produced by


(a)tides.

(b)cyclones.

(c)submarine earthquakes.

(d)shrinking of the Earth's crust.

19.Which one of the following sediment deposits covers the largest


percentage of the ocean floor?

(a)Terrigenous

(b)Cosmogenous

(c)Biogenous

(d)Hydrogenous

20.The term `Ecology' was first used by

(a)Arthur Tansley.

(b)Taylor.

(c)De Martonne.

(d)Jeans Brunhes.

21.Match List I with List II and select the correct answer from the codes
given.
Codes:

22.In which of the following years was the Kyoto Protocol signed?

(a)1987

(b)1997

(c)1990

(d)1985

23.Given below are two statements, one labelled as Assertion (A) and other
labelled as Reason (R). Select your answer from the codes given.

Assertion (A): Behavioural Geography is now becoming much closer to a


constructivists or transactionalist epistemology, as propounded by neo-
Kantian philosophers and psychologists.

Reason (R): Geographers like White, Kirk, Gold and Kates have made
signal contributions for behavioural movement in geography.
(a)(A) is correct, but (R) is wrong.

(b)(A) is wrong, but (R) is correct.

(c)Both (A) and (R) are correct and (R) is the correct explanation of (A).

(d)Both (A) and (R) are correct, but (R) is not the correct explanation of
(A).

24.Match List I with List II and select the correct answer from the codes
given.

Codes:

25.An Introduction to the Application of Geography to History and The


Geographical Distribution of Mankind are the works of
(a)Vidal de la Blache.

(b)Carl Ritter.

(c)Alfred Hettner.

(d)Friedrich Ratzel.

26.First quarter of 20thcentury was considered to be the Golden Age of


German geography. Among the following who were the great leaders of
this period'?

1.A. Penck

2.H. Lautensach

3.C. Ritter

4.A. Hettner

(a)1 and 3 are correct.

(b)3 and 4 are correct.

(c)1 and 4 are correct.

(d)1, 2 and 3 are correct.

27.Who one of the following determined the place of geography in the


classification of Sciences?

(a)Bernhard Varenius

(b)Philip Cluverius
(c)Immanuel Kant

(d)Sebastian Munster

28.Which of the following statements are correct relating to Ratzel's concept


of `Lebensraum'?

1.It is the geographical area with in which living organisms develop.

2.It is the economic and cultural activity of peoples outside their enclosed
settled area.

3.It deals with the relations between human society as a spatial


organization and it physical setting.

4.It speaks `geopolitik' and studies space from the viewpoint of state.

(a)1, 2 and 3 are correct.

(b)2 and 3 are correct.

(c)2, 3 and 4 are correct.

(d)1 and 3 are correct.

29.Given below are two statements, one labelled as Assertion (A) and other
labelled as Reason (R). Select your answer from the codes given.

Assertion (A): Humanistic Geography rejects positivism and


quantification in geography.

Reason (R): Humanistic Geography emphasizes the role of the human


agency and its creativity in the construction of place, space and landscape.
(a)Both (A) and (R) are true and (R) is the correct explanation of (A).

(b)Both (A) and (R) are true, but (R) is not the correct explanation of (A).

(c)(A) is true, but (R) is false.

(d)(A) is false, but (R) is true.

30.Which one of the following is correct chronological sequence of Arab


geographers in the development of geography?

(a)Al-Biruni, Al-Idrisi, Ibn Battuta, Ibn Khaldun

(b)Al-Idrisi, Ibn Khaldun, Al-Masudi, Al-Biruni

(c)Ibn Khaldun, Al-Masudi, Al-Biruni, Al-Idrisi

(d)Al-Masudi, Ibn Battuta, Ibn Khaldun, Al-Biruni

31.In India which of the following criteria applied in defining a town has
varied from census to census in recent decades?

(a)Density of population

(b)Total population

(c)Administrative status

(d)Working population

32.Match List I with List II and select the correct answer from the codes
given.
Codes:

33.`Informational city' is a term coined by

(a)David Harvey.

(b)Richard Peet.

(c)Edward Soja.

(d)Manuel Castells.

34.Which one of the following models is based on the study of human


ecology'?

(a)Hagerstrand's Innovation Diffusion Model


(b)Weber's Location Model

(c)Zelinsky's Mobility Transition Model

(d)Burgess and Parks' Concentric Zone Model

35.`Time-space compression' is a spatial concept given by

(a)Dorreen Massey.

(b)David Harvey.

(c)Daniel Bell.

(d)Edward Soja.

36.Given below are two statements, one labelled as Assertion (A) and other
labelled as Reason (R). Select your answer from the codes given.

Assertion (A): Regional disparities tend to reduce over time in a


competitive free market economy.

Reason (R): The factors of production move from one region to another to
maximize their rate of returns.

(a)Both (A) and (R) are true and (R) is the correct explanation.

(b)Both (A) and (R) are true, but (R) is not the correct explanation.

(c)(A) is true, but (R) is false.

(d)(A) is false, but (R) is true.


37.When higher income groupsre-occupy and revive older housing in
attractive inner city areas, the process is called

(a)filtering.

(b)gentrification.

(c)housing redevelopment.

(d)residential up gradation.

38.Factorial ecology is a method generally used to analyse

(a)urban socio-spatial structures.

(b)physiographic regions.

(c)agro-climatic regions.

(d)both (b) and (c)

39.To which of the following, `nomadism' is the response of man'?

(a)Large resources spread overlarge areas

(b)Large resources localized in as mall area

(c)Limited resources spread overa large area

(d)Limited resources concentrated in a small area

40.According to Von Thunen model, land rent and distance relationship of


five crops, i.e., `a', `b', `c', `d' and `e' is shown in the diagram given below.
Which one of the following pairs of crops substitute crop `c'?

(a)a - d

(b)d-e

(c)a - b

(d)b - d

41.On the basis of the given map of an area, the best suitable place for
locating a big iron and steel plant would be the site marked as
(a)a.

(b)d.

(c)b.

(d)c.

42.Consider the following statement sand select the correct answer using the
codes given.

The ships on the North Atlantic sea route are forced to the south of the
true great circle route by

1.fog associated with the contact of the cold Labrador current and the
warm Gulf stream current.

2.icebergs brought southward by the Labrador current at certain seasons


of the year.
3.the strong polar icy winds from the north.

4.the projecting landmass of northeastern North America and the


presence of warm Gulf stream current.

(a)1 and 2are correct.

(b)1, 2and3are correct.

(c)land 3are correct.

43.Which one is not the advantage of pipeline transport'?

(a)Pipelines can be laid through difficult terrain as well as underwater.

(b)Pipelines ensure steady supply.

(c)Pipeline operation involves very low consumption of energy.

(d)It is very easy to detect leakages, and in the event of damage, pipelines
can be repaired easily and quickly.

44.Industries that manufacture individual components for the final assembly


of product are known as

(a)subsidiary industries.

(b)small-scale industries.

(c)ancillary industries.

(d)cottage industries.
45.In the transport network shown in the figure given below, which one of
the locations has the maximum accessibility'?

(a)a

(b)b

(c)c

(d)d

46.Given below are two statements, one labelled as Assertion (A) and the
other labelled as Reason (R). Select your answer from the codes given.

Assertion (A): Tribal religion, where natural objects are deemed as spirits
and demons, is often characterized by animism.

Reason (R): Simplistic beliefs and practices often find a close association
with ecological niche.
(a)Both (A) and (R) are true and (R) is the correct explanation of (A).

(b)Both (A) and (R) are true, but (R) is not the correct explanation of (A).

(c)(A) is true, but (R) is false.

(d)(A) is false, but (R) is true.

47.Given below are two statements, one labelled as Assertion (A) and the
other labelled as Reason (R). Select your answer from the codes given.

Assertion (A): There are wide variations in evolution of languages and


species in the world.

Reason (R): Geographic isolation plays an important role in developing


several forms of languages and species.

(a)Both (A) and (R) are true and (R) is the correct explanation of (A).

(b)Both (A) and (R) are true, but (R) is not the correct explanation of (A).

(c)(A) is true, but (R) is false.

(d)(A) is false, but (R) is true.

48.Given below are two statements, one labelled as Assertion (A) and the
other labelled as Reason (R). Select your answer from the codes given.

Assertion (A): In spatiotemporal diffusion of the culture, the language


plays a major role.

Reason (R): Geographers of ten use language as an identification mark for


different cultures.
(a)Both (A) and (R) are true and (R) is the correct explanation of (A).

(b)Both (A) and (R) are true, but (R) is not the correct explanation of (A).

(c)(A) is true, but (R) is false.

(d)(A) is false, but (R) is true.

49.Given below are two statements. One labelled as Assertion (A) and other
labelled as Reason (R). Select your answer from the codes given.

Assertion (A): Biologically, human races do not exist, yet races do exist
as social constructs.

Reason (R): Racial hatred and prejudices are all too common in the world
today, resulting from an irrational dislike of whatever is different.

(a)Both (A) and (R) are true and (R) is the correct explanation of (A).

(b)Both (A) and (R) are true, but (R) is not the correct explanation of (A).

(c)(A) is true, but (R) is false.

(d)(A) is false, but (R) is true.

50.Given below are two statements, one labelled as Assertion (A) and other
labelled as Reason (R). Select your answer from the codes given.

Assertion (A): India is a multi religious and multi ethnic society.

Reason (R): The Constitution of India offers to all citizens, individually


and collectively, fundamental rights with out discrimination on grounds of
religion, race, caste, sex or place of birth.
(a)Both (A) and (R) are true and (R) is the correct explanation.

(b)Both (A) and (R) are true, but (R) is not the correct explanation of (A).

(c)(A) is true, but (R) is false.

(d)(A) is false, but (R) is true.

51.The statement that `A race is a valid biological concept not a valid socio-
cultural concept' was made by

(a)A.L. Kroeber.

(b)J.B. Birdsell.

(c)Vidal de la Blache.

(d)A.C. Haddon.

52.Geography of public finance can simply be defined as `Who gets what,


where, at what cost?'. Name the geographer who made this statement.

(a)R.J. Chorley

(b)Richard Peet

(c)Alaistar Bonnett

(d)R.J. Bennett

53.International aid helps poor countries due to resource transfers from rich
countries but

1.divides world into donor and recipient countries.


2.makes donor countries more powerful in international trade and power
relations.

3.forces recipient countries to carry out such economic are forms that
might leave people worse-off.

4.compels recipient countries to buy donor nation goods.

(a)1 and 4 are correct.

(b)1, 2 and 3 are correct.

(c)3 and 4 are correct.

(d)All are correct.

54.Match List I with List II and select the correct answer using the codes
given.

Codes:
55.Given below are two statements, one labelled as Assertion (A) and other
labelled as Reason (R). Select your answer from the codes given.

Assertion (A): Regions, in fact, do change over time; they change in


response to changing interrelationship of the spatial elements.

Reason (R): Planned intervention certainly proves to be of rapid


momentum within the frame of socioeconomic activities.

(a)Both (A) and (R) are correct and (R) is the correct explanation of (A).

(b)Both (A) and (R) are correct, but (R) is not the correct explanation of
(A).

(c)(A) is correct, but (R) is wrong.

(d)(A) is wrong, but (R) is correct.

56.Which technique was adopted by B.J.L. Berry in his classic study on


functional regionalization of Indian economy'?

(a)Composite Index Method

(b)Cluster Analysis

(c)Regression Analysis

(d)Multiple Factor Analysis


57.Which of the following are not the major objectives of the Hill Area
Development Programme?

1.Agro forestry

2.Large-scale industries

3.Ecopreservation

4.Major irrigation projects

(a)1 and 2 are correct.

(b)2 and 3 are correct.

(c)2 and 4 are correct.

(d)3 and 4 are correct.

58.Who among the following applied the growth pole concept to geographic
space, whereby growth poles came to be known as growth centres?

(a)Friedmann

(b)Perroux

(c)Frank

(d)Boudeville

59.In which Five Year Plan, balanced regional development was emphasized
as one of the major objectives of the planning exercise and was one full
chapter in the plan document'?
(a)Second Plan

(b)Third Plan

(c)Fourth Plan

(d)Fifth Plan

60.In ancient geography, the concept of region was first postulated by

(a)Homer.

(b)Herodotus.

(c)Strabo.

(d)Ptolemy.

61.The city region is an example of

(a)formal region.

(b)functional region.

(c)compage region.

(d)adhoc region.

62.Given below are two statements, one labelled as Assertion (A) and the
other labelled as Reason (R).Select your answer from the codes given.

Assertion (A): The task of preparing a reasonably sound district plan has
not made much headway in the states.
Reason (R): There is a lack of effective coordination at the district level
between various agencies involved in planning exercises.

(a)(A) is correct, but (R) is wrong.

(b)(A) is wrong, but (R) is correct.

(c)Both (A) and (R) are correct and (R) is the correct explanation of (A).

(d)Both (A) and (R) are correct, but (R) is not the correct explanation of
(A).

63.Which one of the following islands of India has volcanic origin?

(a)Car Nicobar

(b)Barren

(c)North Andaman

(d)Little Nicobar

64.Which of the following hills does not join the Anaimudi knot'?

(a)Elamalai hills

(b)Palni hills

(c)Anaimalai hills

(d)Kudremukh hills

65.Among the following which one is related to Blue Revolution in India'?

(a)Horticulture
(b)Floriculture

(c)Pisciculture

(d)Sericulture

66.Which one of the following groups of iron and steel plants is located
outside the Chhota Nagpur region'?

(a)Bhilai, Bokaro, Rourkela

(b)Jamshedpur, Bokaro, Durgapur

(c)Bhilai, Salem, Bhadravati

(d)Salem, Bhadravati, Jamshedpur

67.A map of R.F. 1/2.500 is enlarged by two and a half times, what will be
the scale of the enlarged map'?

(a)1/6,250

(b)1/6,250

(c)1/10,000

(d)1/12,500

68.Population density is usually shown by

(a)isopleth method.

(b)choropleth method.

(c)chorochromatic method.
(d)decimetric method.

69.What is the pattern of settlement distribution if its Rn = 2.14'?

(a)Perfect cluster

(b)Partially cluster

(c)Random

(d)Perfect uniform

70.Ajai ranks 17thin a class of 31. What is his rank from last'?

(a)13

(b)14

(c)15

(d)16

71.Given below are two statements, one labelled as Assertion (A) and the
other labelled as Reason (R). Select your answer from the codes given.

Assertion (A): Passive remote sensing systems have their own source of
energy.

Reason (R): Active remote sensing systems depend on solar energy.

(a)(A) is correct, but (R) is wrong.

(b)Both (A) and (R) are correct, but (R) is not the correct explanation of
(A).
(c)Both (A) and (R) are correct and (R) is the correct explanation of (A).

(d)Both (A) and (R) are wrong.

72.The satellite image shown in television clearly depicts

(a)temperature and rainfall.

(b)pressure and wind.

(c)clouds and snow cover.

(d)humidity and fog.

73.Match List I with List II and select the correct answer using the codes
given.

Codes:

74.Which one of the following characteristics separates GIS from the other
systems?
(a)Map making and generalized out put

(b)Linking spatial and non-spatial data

(c)Map designing and layout

(d)All of the above

75.Cephalic index, which is to classify human races, is calculated as

(a)ratio between the length and breadth of the nose.

(b)ratio between the length of head and weight of human body.

(c)ratio between flatness and length of the head.

(d)ratio between breadth and length of the head.

ANSWER KEY

 
Note: This paper contains seventy-five (75) objective-type questions of two
(2) marks each. All questions are compulsory.

1.The Drift Theory of Wegener was postulated mainly to explain

(a)Ice Age.

(b)distribution of landforms.

(c)major climatic changes.

(d)geological similarities of coastal regions.

2.Which of the following groups is produced by erosion?

(a)Drumlins, fjords, arete

(b)Eskern, outwash plains, moraines

(c)Cirque, V-shaped valley, levees

(d)Playa, pincles, swallow holes

3.As per the theory of plate tectonics, which of the following is a super
continent?

(a)Eurasia

(b)Africa
(c)Antarctica

(d)Australia

4.The direction of a horizontal line on an inclined rock strata is a

(a)dip.

(b)strike.

(c)syncline.

(d)anticline.

5.Sunda peninsula was submerged in the Pacific Ocean due to

(a)constructive activities of the plates.

(b)destructive activities of the plates.

(c)subduction of the plates into ocean.

(d)sea floor spreading.

6.Match List I with List II and select the correct answer by using the codes
given.
Codes:

7.Match List I with List II and select the correct answer by using the codes
given.

Codes:

8.The tectonic plates on which India and the adjacent portion of Asia ride
are colliding at a rate of

(a)0.1-1 millimetre per year.


(b)1-10 millimetre per year.

(c)0.1-1 centimetre per year.

(d)1-10 centimetre per year.

9.Who propounded the theory of Rural Settlement?

(a)Ritter

(b)Ratzel

(c)Meitzen

(d)Schluter

10.Who postulated the concept of plate tectonics?

(a)Le Pichon

(b)W.J. Morgan

(c)Harry Hen

(d)Tuzo Wilson

11.The following question consists of two statements. One is labelled as


Assertion (A) and the other labelled as (R). Select the correct answer
using the codes given.

Assertion (A): There is a close relationship between volcanoes and plate


margins.
Reason (R): About 80 per cent of the world's volcanoes are associated
with the convergent plate boundaries.

(a)Both (A) and (R) are true and(R) is the correct explanation of(A).

(b)Both (A) and (R) are true and(R) is not the correct explanation of (A).

(c)(A) is true, but (R) is false.

(d)(A) is false, but (R) is true.

12.Knick points are associated with

(a)river rejuvenation.

(b)glacial erosion.

(c)aeolian deposits.

(d)Karst topography.

13.When the Kaveri river as soon as it enters Tamil Nadu, which waterfall
does it create?

(a)Hoggenakka Falls

(b)Jog Falls

(c)Five Falls

(d)Paghaya Kotlam

14.Peninsulas make up most of

(a)Italy and Denmark.


(b)Poland and France.

(c)Ireland and Great Britain.

(d)Austria and the Czech Republic.

15.Tornadoes are most likely to occur during

(a)mid-morning.

(b)mid-afternoon.

(c)noon.

(d)late evening.

16.Which U.S. city was struck by the hurricane Katrina'?

(a)New York

(b)Houston

(c)New Orleans

(d)Miami, FL

17.The following question consists of two statements. One is labelled as


Assertion

(A)and the other labelled as (R). Select the correct answer using the codes
given.

Assertion (A): When the Sun is overhead on a clear sky, one can see blue
sky in all directions.
Reason (R): The longer wavelengths of visible light are scattered more
effectively than short wavelength.

(a)Both (A) and (R) are correct and (R) explains (A).

(b)Both (A) and (R) are correct, but (R) does not explain (A).

(c)(A) is correct, but (R) is false.

(d)(A) is false, but (R) is true.

18.Ozone hole is located above the

(a)Antarctica.

(b)Arctic Ocean.

(c)Equatorial Zone.

(d)Greenland.

19.A sudden, violent, gusty wind, which lasts a minute or two and then
subsides, usually accompanied by rain or hail, is called

(a)thunderstorm.

(b)hoar.

(c)blizzard.

(d)squall.

20.The following question consists of two statements. One is labelled as


Assertion (A) and the other labelled as (R). Select the correct answer
using the codes given.

Assertion (A): The thunderstorms of Bengal and the adjoining Bihar,


Orissa and Assam are known as Northwesters.

Reason (R): The squalls come predominantly from northeast.

(a)Both (A) and (R) are correct and (R) explains (A).

(b)Both (A) and (R) are correct, but (R) does not explain (A).

(c)(A) is correct, but (R) is false.

(d)(A) is false, but (R) is true.

21.The following question consists of two statements. One is labelled as


Assertion (A) and the other labelled as (R). Select the correct answer
using the codes given.

Assertion (A): The troposphere is the dense lower part of the atmosphere
in which all the weather phenomena like condensation and storms occur.

Reason (R): In troposphere, the atmospheric temperature decreases with


altitude.

(a)Both (A) and (R) are true and(R) is the correct explanation of(A).

(b)Both (A) and (R) are true, but(R) is not the correct explanation of (A).

(c)(A) is true, but (R) is false.

(d)(A) is false, but (R) is true.


22.The following question consists of two statements. One is labelled as
Assertion (A) and the other labelled as (R). Select the correct answer
using the codes given.

Assertion (A): Jet streams play important role in intensifying weather


phenomena such as cyclones, hurricanes and typhoons.

Reason (R): There are severe storms when jet streams interfere with
surface wind systems.

(a)Both (A) and (R) are true and(R) is the correct explanation of (A).

(b)Both (A) and (R) are true, but(R) is not the correct explanation of (A).

(c)(A) is true, but (R) is false.

(d)(A) is false, but (R) is true.

23.Which of the following oceans is spreading due to plate tectonic


movement'?

(a)Atlantic Ocean

(b)Pacific Ocean

(c)Indian Ocean

(d)Arctic Ocean

24.Match List I with List II and choose the correct answer using the codes
given.
Codes:

25.A reef separated from the mainlandor inland shore by a deep lagoon is
known as

(a)fringing reef.

(b)Atoll reef.

(c)barrier reef.

(d)patch reef.

26.Global change in sea level is caused by

(a) eustatic change.

(b)isostatic change.
(c)climatic change.

(d)diastrophism.

27.The following question consists of two statements. One is labelled as


Assertion (A) and the other labelled as (R). Select the correct answer
using the codes given.

Assertion (A): Western coast of Europe (UK) and Labrador of North


America are in the same latitude but Western Coast of Europe (UK)
remain sun frozen and Labrador remains frozen for 9 months.

Reason (R): Gulf stream (UK) is a warm current and Labrador is a cold
current.

(a)Both (A) and (R) are true and(R) is not the correct explanation of (A).

(b)Both (A) and (R) are true and(R) is the correct explanation of (A).

(c)(A) is true, but (R) is false.

(d)(A) is false, but (R) is true.

28.Which of the following is not true about welfare geography'?

(a)It studies the question of who gets what, where and how.

(b)It is much concerned about spatial distribution of poverty, resources


and civic amenities.

(c)It was part of the reaction against positivism in geography.

(d)It seeks to change the present economic and political set up.
29. Who can be considered as an opponent of quantitative revolution'?

(a)Stamp

(b)Chorley

(c)Haggelt

(d)Hagerstrand

30.Identify the wrong pair.

(a)A.J. Herbert son-Major natural regions

(b)H. Robert Mill-Realm of nature

(c)Peter Hagget-Locational analysis inhuman geography

(d)L.D. Stamp-Frontiers in geographical teaching

31.The concept of paradigm was propounded by

(a)Thomas Kuhn.

(b)E. Soja.

(c)P. Hall.

(d)Lewis Mumford.

32.Match List I with List II and select the correct answer from the codes
given.
Codes:

33.The concept of sustainable development rests on the following principles:

(a)Intergeneration transferability

(b)Social justice

(c)Transfrontier responsibility

(d)Intergeneration transferability, social justice, trans frontier


responsibility

34.Which of the following is not a thematic map'?

(a)Map showing population density

(b)Map showing distribution of minerals


(c)Map showing jute-producing areas

(d)Topographical map on 1:50,000.

35.Which one of the following pairs is not a correct match'?

(a)Geddes - Le Play School

(b)Perry - Neighbourhood units

(c)Howard - Garden city

(d)Le Corbusier - Satellite city

36.The plot of cities of developing countries according to their size and rank
on a normal graph paper will assume which of the following shapes'?

(a)Linear

(b)Concave

(c)Convex

(d)Curvilinear

37.Settlements located at a distance from river banks and other water bodies
are called

(a)wet point settlements

(b)dry point settlements.

(c)strong point settlements.

(d)nucleated settlements.
38.Match List I with List II and select the correct answer using the codes
given.

Codes:

39.Which one of the following distances does not fit into Christaller's
scheme of k = 3 hierarchy of central places'?

(a)7

(b)11

(c)21

(d)36

40.Match List I with List II and select the correct answer from the codes
given.
Codes:

41.Given below are the two statements, one labelled as Assertion (A) and
other labelled as Reason (R). Select your answer from the codes given.

Assertion( A): In India, large cities are growing at a very fast rate.

Reason (R): In spite of the emphasis on their planned development, the


small and medium towns have failed to come up to expected norms.

(a)Both (A) and (R) are correct and (R) is the correct explanation of (A).

(b)Both (A) and (R) are correct, but (R) is not the correct explanation of
(A).

(c)(A) is correct, but (R) is false.

(d)(A) is false, but (R) is correct.

42.Given below are the two statements, one labelled as Assertion (A) and
the other labelled as Reason (R). Select your answer from the codes
given.

Assertion (A): Annual growth rate of population in developed nations is


much lower than in developing nations.

Reason (R): Economic development results in reduction in birth rate.

(a)Both (A) and (R) are correct and (R) explains (A).

(b)Both (A) and (R) are correct, but (R) does not explain (A).

(c)(A) is true, but (R) is false.

(d)(A) is false, but (R) is true.

43.Which one of the following statements is not correct about migration?

(a)Migration varies inversely with the distance between source and


destination.

(b)The majority of migrants move in order to improve their economic


conditions.

(c)Governments create migration policies to attract the talents that they


lack.

(d)Women tend to move longer distances than men.

44.Match List I with List II and select the correct answer using the codes
given.
Codes:

45.Given below are two statements, one labelled as Assertion (A) and the
other labelled as Reason (R). Select your answer from the codes given.

Assertion (A): The distribution of population is highly uneven in the


world.

Reason (R): The environmental conditions favourable for human


settlement differ from place to place.

(a)Both (A) and (R) are correct and (R) is the correct explanation of (A).

(b)Both (A) and (R) are correct, but (R) does not explain (A) correctly.

(c)(A) is true, but (R) is false.


(d)(A) is false, but (R) is true.

46.Which of the following sets of countries does not use coal as essential
energy for industries'?

(a)Poland and Switzerland

(b)Switzerland and Holland

(c)Sweden and Italy

(d)None of these

47.The greatest share of fisheries comesfrom

(a)deep seas.

(b)continental shelves.

(c)deep ocean trenches.

(d)fresh waters.

48.Which one of the following major sea ports of India does not have natural
harbour'?

(a)Marmagao

(b)Mumbai

(c)Cochin

(d)Paradeep
49.Match List I with List II and select the correct answer using the codes
given.

Codes:

50.Match List I with List II and select the correct answer using the codes
given.

Codes:
51.Given below are the two statements, one labelled as Assertion (A) and
other labelled as Reason (R). Select your answer from the codes given.

Assertion (A): The pattern of consumption of goods and services varies


from one country to another.

Reason (R): The population distribution varies from one country to


another.

(a)Both (A) and (R) are correct and (R) explains (A).

(b)Both (A) and (R) are correct, but (R) does not explain (A).

(c)(A) is true, but (R) is false.

(d)(A) is false, but (R) is true.

52.According to Lynch, the elements which help to perceive urban


environment are

(a)path and node.

(b)path, node and landmark.

(c)boundaries and districts.

(d)path, boundaries, districts, nodes and landmark.


53.Which one of the following theories is based on the concept of economic
rent'?

(a)Agricultural land use

(b)Ranksize rule

(c)Innovation diffusion

(d)Urban land use

54.Green Revolution is a technological package, consistency of

1.high-yielding variety of seeds.

2.land reforms.

3.assured irrigation facility.

4.use of chemical fertilizers.

(a)1, 2 and 3 are correct.

(b)2, 3 and 4 are correct.

(c)1, 3 and 4 are correct.

(d)Only 3 is correct.

55.Match List I with List II and select the correct answer from the codes
given.
Codes:

56.Given below are the two statements, one labelled as Assertion (A) and
other labelled as Reason (R). Select your answer from the codes given.

Assertion (A): Most transportation net works begin as branching net


works.

Reason (R):Because the first priority of the public or private agencies that
build them is generally to get all points connected.

(a)Both (A) and (R) are correct and (R) is the correct explanation of (A).

(b)Both (A) and (R) are correct, but (R) is not the correct explanation of
(A).

(c)(A) is true, but (R) is false.

(d)(A) is false, but (R) is true.

57.Which one of the following is not a general-purpose authority?


(a)A district

(b)World Health Organization (WHO)

(c)A department

(d)A county

58.Match List I with List II and select the correct answer from the codes
given.

Codes:

59.Who defined urban geography as the study of the city as a system within
a system of cities?

(a)H.C. Carter

(b)Michael Pacione
(c)B.J.L. Berry

(d)A.E. Smails

60.Given below are the two statements, one labelled as Assertion (A), and
the other labelled as Reason (R). Select the correct answer from the codes
given.

Assertion (A): Regional development planning solves the problem of


regional disparities created by sect oral planning.

Reason (R): Sect oral planning concentrates on sect oral growth ignoring
regional balance and regional development planning takes care of the
growth and development of the delineated regions.

(a)Both (A) and (R) are true and (R) is the correct explanation of (A).

(b)Both (A) and (R) are true and but (R) is not the correct explanation of
(A).

(c)(A) is true, but (R) is false.

(d)(A) is false, but (R) is true.

61.Who gave the theory of Rim land, giving importance to sea powers due
to their ability to make a rim around `Heartland', in the book The
Geography of Peace?

(a)Spykman

(b)Truman

(c)Haushoffer
(d)Mahan

62.Mackinder's `Pivot Area', which he envisaged to be strategically very


important, was

(a)Arabia and its surroundings.

(b)central Africa.

(c)land between Volga in the west and eastern Siberia in the east.

(d)the central regions of `new world', i.e., America's.

63.The following question consists of two statements. One is labelled as


Assertion (A) and the other labelled as (R). Select the correct answer
using the codes given.

Assertion (A): Planning regions generally tally with administrative


divisions.

Reason (R): Plan execution is convenient in uniform administrative


regions.

(a)Both (A) and (R) are correct, but (R) does not explain (A).

(b)Both (A) and (R) are incorrect.

(c)(A) is correct, but (R) is incorrect.

(d)(A) is incorrect, but (R) is correct.

64.Bhakra Nangal Project is a joint venture of which one of the following


groups of states?
(a)Punjab, Haryana and Rajasthan

(b)Haryana, Punjab and Uttar Pradesh

(c)Himachal Pradesh, Uttarakhandand Punjab

(d)Punj ab, Raj asthan and Uttarakhand

65.The ranges of Western Ghats are locally called as `Sahyadris' in which of


the following states?

(a)Tamil Nadu

(b)Karnataka

(c)Kerala

(d)Maharashtra

66.Given below are two statements, one labelled as Assertion (A) and the
other labelled as Reason (R). Select your answer from the codes given.

Assertion (A): One of the major sources of irrigation in Peninsular India


is tank irrigation.

Reason (R): Most of the rivers in Peninsular region are seasonal.

(a)Both (A) and (R) are correct and (R) explains (A).

(b)Both (A) and (R) are correct, but (R) does not explain (A).

(c)(A) is correct, but (R) is false.

(d)(A) is false, but (R) is true.


67.Arrange the following events playing a vital role in evolution of town and
country planning in India in a sequential order of their taking place. Use
the codes given.

1.Appointment of Bhore Committee

2.Enactment of Sanitation Act

3.Establishment of improvement trusts in important towns

4.Formation of Town and Country Planning Organisation(TCPO)

(a)1, 2, 3, 4

(b)2, 3, 1, 4

(c)3, 1, 2, 4

(d)4, 3, 2, 1

68.Which one of the following number of million sheets representing the


world is correct'?

(a)54

(b)136

(c)152

(d)504

69.Match List I with List II and select the correct answer by using the codes
given.
Codes:

70.Match List I with List II and select the correct answer by using the codes
given.

Codes:
71.The Air Photo Scale of a flat terraindepends upon

1.height of the aircraft.

2.focal length of aerial camera.

Choose the correct answer codes given.

(a)1 is true, but 2 is false.

(b)2 is true, but 1 is false

(c)Both 1 and 2 are true.

(d)Both 1 and 2 are false.

72.Consider the following statementsand choose the correct answer from the
codes given:

1.The contours which represent aconvex slope are close togetherat the
foot and arecomparatively farther apart atthe top.

2.The closeness of contours indicates steepness of slope.

(a)Only 1 is correct.

(b)Only 2 is correct.

(c)Both 1 and 2 are correct.


(d)Neither 1 nor 2iscorrect.

73.Arrange the following maps indecreasing order of scale and selectthe


correct answer from the codes given.

1.Atlas map

2.Cadastral map

3.Topographical map

4.Wall maps

Codes:

74.Analysis of variance (f-ratio test) canonly be applied to data measured on

(a)interval scale.

(b)ratio scale.

(c)ordinal scale.

(d)nominal scale.

75.Consider the following statementsand choose the correct answer fromthe


codes given:

1.In a dot map, the value of thedots generally varies.


2.A dot map is usually used toshow a real distribution of phenomena such
as rainfall, agricultural production, population, etc.

(a)Only (a) is correct.

(b)Only (b) is correct.

(c)Both (a) and (b) are correct.

(d)Neither (a) nor (b) iscorrect

ANSWER KEY

Note: This paper contains seventy-five (75) objective-type questions of two


(2) marks each. All questions are compulsory.

1.The concept of geosynclines was given by

(a)James Hall and Dana.

(b)Hang.

(c)Holmes.
(d)Steers.

2.The Base Level concept was postulated by

(a)James Hutton.

(b)J.W. Powell.

(c)W.M. Davis.

(d)Walther Penck.

3.Which of the following is formed as a result of tectonic forces'?

(a)Hanging valley

(b)V-shaped valley

(c)Rift valley

(d)Blind valley

4.Match List I with List II and select the correct answer using the codes
given.
Codes:

5.`The present is the key to the past'. This statement was made by

(a)W.M. Davis.

(b)James Hutton.

(c)Van Ritchthofen.

(d)A. Penck.

6.Which of the following was a part of Lauratia?

(a)Anatolian Plate
(b)Chinese Plate

(c)Iranian Plate

(d)Agean Plate

7.Match List I with List II and choose the correct answer using the given
codes.

Codes:

8.When does an escarpment appear?

(a)When a land block moves horizontally.


(b)When a land block moves vertically.

(c)When a land block moves due to water logging.

(d)When a land block moves due to human intervention.

9.Identify fluvio-glacial deposits from the following:

(a)Outwash plain

(b)Flood plain

(c)Penne plain

(d)Pan plain

10.Migration from developed to less developed region is called

(a)emigration.

(b)immigration.

(c)perverse migration.

(d)reverse migration.

11.`Space is socially or culturally constructed' is the view under

(a)logical positivism.

(b)behaviouralism.

(c)post-modernism.

(d)structuralism.
12.A counter clockwise atmospheric circulation in the Northern Hemisphere
is known as

(a)pressure gradient.

(b)cyclone.

(c)anticyclone.

(d)tornado.

13.The author of the book The Unstable Earth is

(a)Paschel.

(b)G.K. Gilberk.

(c)C.A. Mallot.

(d)J.A. Steers.

14.Which of the following earthquake waves are most destructive?

(a)S-waves

(b)P-waves

(c)R-waves

(d)L-waves

15.Mediterranean climate is characterized by

(a)dry summer and humid winter.


(b)humid summer and dry winter.

(c)dry summer and dry winter.

(d)humid summer with no winter.

16.Insolation reaches the Earth's surface in the form of

(a)short waves.

(b)long waves.

(c)microwaves.

(d)Lorenz curve.

17.The range of visible wavelength on electromagnetic spectrum is

(a)0.4-0.7 micrometres.

(b)0.7-15.0 micrometres.

(c)0.3-0.9 micrometres.

(d)0.3-15.0 micrometres.

18.As per the Koppen's scheme, the Bhwh type of climate is found in

(a)Jammu and Kashmir.

(b)Rajasthan.

(c)Gujarat.

(d)Odisha.
19.Cold dry wind experienced particularly in winter along the eastern coast
of Atlantic Ocean and in northern Italy is called

(a)cyclone.

(b)boras.

(c)tornado.

(d)hurricane.

20.The atmosphere gets heated by which one of the following?

(a)Direct rays of the Sun

(b)Volcanic activity

(c)Burning of organic material

(d)Radiation from the Earth

21.A tropical cyclone was located 500 km south-west of Visakhapatnam.


The cyclone first moved 250 km in the north-west direction and then
suddenly changed the direction to the north-east. Which of the following
ports would raise the highest danger signal'?

(a)Chennai

(b)Haldia

(c)Visakhapatnam

(d)Tuticorin
22.Which one of the following regions does not receive much rainfall during
the south-west monsoon season?

(a)Tamil Nadu Coast

(b)Maharashtra

(c)Chhota Nagpur Plateau

(d)Eastern Himalayas

23.In which type of climate does equiplanation process, i.e., reduction of


relief to a plain takes place?

(a)Arid climate

(b)Cold climate

(c)Tropical climate

(d)Temperate climate

24.Winter rainfall in north-western part of India is mainly due to

(a)Western disturbance.

(b)north-east monsoon.

(c)north-west monsoon

(d)depression in the Bay of Bengal.

25.Which one of the following phenomena shown in the sketch given below
is correct'?
(a)Normal lapse rate

(b)Upper surface inversion

(c)Lower surface inversion

(d)Environmental lapse rate

26.Given below are two statements, one labelled as Assertion (A) and the
other labelled as Reason (R). Select the correct answer from the codes
given.

Assertion (A): The eastern coast of India is affected by tropical cyclones


more than the western coast.

Reason (R): Tropical cyclone originates only in the Bay of Bengal.

(a)Both (A) and (R) are correct but (R) does not explain (A).

(b)Both (A) and (R) are correct and (R) explains (A).

(c)(A) is correct but (R) is false.

(d)(A) is false but (R) is correct.

27.Match List I and List II and select the correct answer using the codes
given.
Codes:

28.The temperature that provides favourable ecological conditions for living


coral reefs is

(a)less than 10°C.

(b)10°C-15°C

(c)15°C - 20°C.

(d)more than 20°C.

29.Match List I with List II and select the correct answer using the codes
given.
Codes:

30.Who among the following has postulated the Subsidence Theory related
to coral reef?

(a)Dally

(b)Agassiz

(c)Darwin

(d)Davis

31.If a tide occurs at a place at 6.00 p.m. when will the next tide occur?

(a)0.13 a.m. of the next day

(b)6.26 a.m. of the next day


(c)12.39 p.m. of the next day

(d)06.52 p.m. of the next day

32.Match List I with List II and select the correct answer using the codes
given.

Codes:

33.Which one of the following ecosystem services is not a part of


Millennium Ecosystem Assessment (MA) report?

(a)Provisioning

(b)Promoting
(c)Supportive

(d)Regulating

34.In which one of the following places was India's first bird sanctuary set
up?

(a)Vedanthangal

(b)Kundremukha

(c)Bannarghata

(d)Keoladeo

35.Given below are two statements, one labelled as Assertion (A) and the
other labelled as Reason (R). Select your answer from the codes given.

Assertion (A): Physical factors provide only a partial and deterministic


explanation of geographical distribution of population.

Reason (R): Man is by no means passive in his choice of areas for


settlement and everywhere demonstrates an ability to exercise some
control over his environment.

(a)Both (A) and (R) are correct and (R) is the correct explanation of (A).

(b)Both (A) and (R) are correct, but (R) is not the correct explanation of
(A).

(c)(A) is correct, but (R) is false.

(d)(A) is false, but (R) is correct.


36.Which one of the following indicates the principle of transport in Central
Place Theory?

(a)K3

(b)K4

(c)K7

(d)K9

37.Assertion (A): Post-industrial city is largely characterized by horizontal


expansion.

Reason (R): Land prices are high in the central city.

(a)Both (A) and (R) are correct.

(b)Both (A) and (R) are correct but (R) does not explain (A).

(c)(A) is correct, but (R) is false.

(d)(R) is correct, but (A) is false.

38.Match List I with List II and select the correct answer using the codes
given.

Codes:
39.Read Assertion (A) and Reason (R) and find the correct answer from the
codes given.

Assertion (A): Global urban population is not uniformly distributed by


location and nearly two thirds is relatively concentrated within low
elevation coastal areas.

Reason (R): Soil fertility in the coastal areas caused concentration of rural
settlements.

(a)Both (A) and (R) are correct and (R) explains (A).

(b)Both (A) and (R) are correct but (R) does not explain (A).

(c)(A) is correct, but (R) is false.

(d)(A) is false, but (A) is true.

40.Which one of the following stages of demographic transition model


predicts `a high birth but a low death rate'?

(a)First stage

(b)Second stage

(c)Third stage

(d)Late second stage


41.Select the correct pair from the following.

42.Which one of the following is not an assumption or principle of the


concentric zone model of urban land case'?

(a)Cultural and social heterogeneity of the population

(b)Transport is equally easy, rapid and cheap in every direction with in


the city.

(c)None of districts is attractive due to differences in terrain.

(d)There is a concentration of heavy industry.

43.Who, amongst the following, defined Geography as a chorological


science?

(a)Ptolemy

(b)Richthofen

(c)Hettner

(d)P.E. James
44.Who, amongst the following, has propounded the concept of time space
geography?

(a)Hager strand

(b)Haggett

(c)Johnston

(d)Harvey

45.List I with List II and select the correct answer using the codes given

Codes:

46.Which one of the following ports has been developed on a lagoon'?

(a)Chennai

(b)Mumbai
(c)Kochi

(d)Visakhapatnam

47.Truck farming is associated with

(a)vegetables.

(b)milk.

(c)cereals.

(d)poultry.

48.The essential feature of shifting cultivation is

(a)rotation of crops.

(b)rotation of fields.

(c)single cropping.

(d)use of plenty of fertilizers.

49.What is the common practice involved in shifting cultivation'?

(a)Use of sophisticated machinery

(b)Large-scale use of fertilizers

(c)Utilization of poor soils through ploughing/fallowing

(d)Maximum use of animal power

50.Under the scope of town planning in India is covered


1.urban renewal.

2.planning of urban amenities and facilities.

3.building of new towns.

4.building of metropolitan cities.

(a)1 and 3 are correct.

(b)Only 1 is correct.

(c)1, 2 and 3 are correct.

(d)2, 3 and 4 are correct.

51.Match List I with List II and select the correct answer from the codes
given.

Codes:
52.Match List I with List II and select the correct answer using the codes
given.

Codes:

53.Who has, amongst the following geographers, defined Geography


ofPublic Finance as `who gets what, where, and at what cost"?

(a)R.J. Chorley

(b)David Harvey

(c)P. Claval

(d)R.J. Bennett

54.Match List I with List II and select the correct answer using the codes
given.
Codes:

55.Which of the following is not a supervised image classification


technique?

(a)Parallelopiped classifier

(b)Minimum distance to mean classifier

(c)Neural network analysis

(d)Gaussian maximum likelihood classifier

56.A meteor is

(a)comet without a tail.

(b)detached piece of an asteroid.

(c)tiny star.
(d)piece of matter which has entered the Earth's atmosphere from outer
space.

57.Which one of the following tribes is not correctly matched'?

(a)Batwa-Congo Basin

(b)Ruwala-Central Iran

(c)Inuit-Canada

(d)Yuklagir-Siberia

58.Which one of the following regions witnessed the growth of mighty


civilizations of Roman and Greek'?

(a)Temperate region

(b)Savannah region

(c)Mediterranean region

(d)Tundra region

59.Which of the following is not considered a geographic pattern?

(a)Centralized

(b)Distributive

(c)Linear

(d)Random

60.Machu Picchu of Inca civilization is located in


(a)Argentina.

(b)Brazil.

(c)Columbia.

(d)Peru.

61.In 1919, Mackinder renamed his `Pivot Area' as `Heartland' in the famous
book

(a)Foreign Affairs.

(b)The Round World and the Winning of the Peace.

(c)The Democratic Ideals and Reality.

(d)World War and Geography.

62.The objectives behind building the planned city of Chandigarh were

1.to build a city of modern India's dreams.

2.to settle down refugees coming to India after the partition.

3.to develop an attractive tourist centre in north-western India.

4.to provide a capital city to the truncated Punjab.

(a)2 and 4 are correct.

(b)2, 3 and 4 are correct.

(c)1, 2 and 4 are correct.


(d)Only 4 is correct.

63.Which of the following sentences is not a correct one?

(a)Utilitarian planning is a functional planning and it strives to achieve a


supreme single specific goal.

(b)Sectoral planning is a coordinated planning with planning in various


sectors in an integrated manner.

(c)Comprehensive planning has also been used with implicit connotation


of integrated planning of all elements of a single activity.

(d)Town planning is an expression of spatial planning of an urban system.

64.Several planned cities have been developed in India after Independence.


Which one of the following is not a postindependence planned city?

(a)Gandhinagar

(b)Chandigarh

(c)Jaipur

(d)Bhubaneswar

65.Which one of the following is not a measure of central tendency?

(a)Arithmetic mean

(b)Mean deviation

(c)Median
(d)Mode

66.One of the Indian Remote Sensing Satellites IRS-P4 is also known as

(a)Resourcesat.

(b)Edusat.

(c)Remotesat.

(d)Oceansat.

67.Which one of the following states has recorded the lowest human
development index'?

(a)Bihar

(b)Himachal Pradesh

(c)Punjab

(d)Haryana

68.Which one of the following group of states, which includes catchment


area of Cauvery river basin, is correct'?

(a)Tamil Nadu, Kerala, Pondicherry and Andhra Pradesh

(b)Tamil Nadu, Kerala, Andhra Pradesh and Pondicherry

(c)Tamil Nadu, Andhra Pradesh, Maharashtra and Kerala

(d)Tamil Nadu, Kerala, Karnataka and Pondicherry

69.Which one of the following is a centrographic measure?


(a)Geometric mean

(b)Arithmetic mean

(c)Mean centre

(d)Standard deviation

70.Match List I with List II andselect the correct answer using the codes
given.

Codes:

71.Which one of the following map scales is a large scale'?

(a)1:2,50,000

(b)1:25,000

(c)1:5,00,000
(d)1:50,000

72.Given below are two statements, onelabelled as Assertion (A) and


theother labelled as Reason (R). Select correct answer from the codes
given.

Assertion (A): The Raster formatdata structure provides a


greatercomputational efficiency.

Reason (R): The Raster formatdata is simple.

(a)Both (A) and (R) are correct and (R) explains (A).

(b)Both (A) and (R) are correctbut (R) does not explain (A).

(c)(A) is correct, but (R) is false

(d)(A) is false, but (R) is correct.

73.An original map has the RF 1/50,000 and RF of the new map will be
1/2,50,000. What is the correctproportion of enlargement/reduction as
given below?

(a)Reduction 1/5

(b)Enlargement (5 times)

(c)Reduction 1/10

(d)Enlargement (10 times)

74.Which one of the following values ofcorrelation coefficient (r) is


notcorrectly matched degree of relationship?
(a)+0.99 High

(b)+0.50 Moderate

(c)- 0.01 Very low

(d)- 0.99 Nil

75.In which of the following years was IRS - IA launched?

(a)1982

(b)1987

(c)1988

(d)1990

ANSWER KEY

 
Note: This paper contains seventy-five (75) objective-type questions of two
(2) marks each. All questions are compulsory.

1.Lateral Planation Theory of pediment formation was proposed by

(a)Lawson

(b)Davis

(c)Gilbert

(d)McGee

2.Match List I with List II:

Codes:
3.Which of the following is strongly presumptive faultline scrap?

(a)superimposed drainage across a fault

(b)poor correlation between rock resistance and topographic forms

(c)frequent severe earthquakes

(d)actual fault plane identified along the scrap

4.Paternoster lakes are features of:

(a)lava plains

(b)piedmonts

(c)deserts

(d)glacial troughs

5.Match List I with List II:


Codes:

6.Which one of the following processes is typical to glacial erosion?

(a)Plucking

(b)Hydraulic action

(c)Deflation

(d)Corrosion

7.Given below are two statements, one labelled as assertion (A) and the
other labelled as Reason (R).

Assertion (A): Deep chemical decay of rocks is one of the outstanding


features of humid tropical regions

Reason (R): Intensity of chemical weathering depends to a large extent on


abundance of water and higher air temperatures

Codes:

(a)Both A and R are true and A is correct explanation for R


(b)Both A and R are true and A is not correct explanation for R.

(c)A is true and R is false

(d)A is false and R is true

8.Crickmay while disagreeing with Davis supplanted peneplain with the


term:

(a)Pediplain

(b)Panplane

(c)Etchplain

(d)Structural plain

9.What is the correct sequence of the cloud types in a warm front away from
its trailing edge'?

(a)Altostratus - Nimbostratus - Cirrus - Cirrostratus

(b)Cirrostratus - Nimbostratus Altostratus Cirrus

(c)Cirrus - Nimbostratus - Altostratus - Cirrostratus

(d)Nimbostratus Alto Stratus Cirrostratus cirrus.

10.Westerlies are examples of

(a)Macro atmospheric motions

(b)Tornados

(c)Microscale atmospheric motions


(d)Gusts

11.Match the following List I with List II and select the correct answer

Codes:

12.If the elevation of the sun declines, the albedo will

(a)Decline

(b)Increase

(c)Remain the same

(d)No change

13.Seasonal contrast in pressure between land and sea gives rise to:
(a)Monsoons

(b)Westerly winds

(c)Land and Sea breezes

(d)Trade winds

14.V. Bjerknes Model is related with:

(a)Origin of anticyclones

(b)Origin of mid-latitude cyclones

(c)Origin of tropical cyclones

(d)Origin of Monsoons

15.Which of the following is correct?

(a)Cold front is marked with triangular spikes

(b)Warm front is marked by semicircles

(c)A fast advancing warm front can cause violent lifting

(d)Tornadoes develop along squall line

16.Some heat is received by the atmosphere even after Sunset from:

(a)Invisible Solar radiation

(b)Latent heat

(c)Radioactive processes
(d)Terrestrial radiation

17.Given below are two statements one labelled as Assertion (A) and the
other labelled as Reason (R). Select your answer from the codes given
below:

Assertion (A): The growth of grass and other vegetative cover is limited
to Selvas.

Reason (R): The separate crowns of trees coalesce and form a thick
canopy above the forest floor.

Codes:

(a)Both (A) and (R) are true and (R) is correct explanation of (A).

(b)Both (A) and (R) are true, but (R) is not correct explanation of (A).

(c)(A) is true but (R) is false.

(d)(R) is true but (A) is false.

18.`Montreal Protocol' is related to

(a)Global Warming

(b)Ozone Depletion

(c)Air Pollution

(d)Water Pollution

19.Objective of Kyoto protocol was to reduce greenhouse gases emission


by:
(a)An average of 10% below 1995 level by year 2010

(b)An average of 8% below 1998 level by year 2012

(c)An average of 20% below 1997 level by year 2009

(d)An average of 5% below 1990 level by year 2012

20.Xerophytes can tolerate and stand against:

(a)High temperature

(b)Severe Cold

(c)Humidity

(d)Drought conditions

21.Consider the following statements:

1.Ozone is found mostly in the stratosphere.

2.Ozone layer lie 5575 kms above the surface of the earth

3.Ozone absorbs ultraviolet rays from the Sun.

4.Ozone layer has no significance for life on the earth.

Which of the above statements are correct?

(a)1 and 3

(b)2 and 4

(c)2 and 3
(d)1 and 4

22.Given below are two statements one labelled as Assertion (A) and the
other labelled as Reason (R). Select your answer from the codes given
below:

Assertion (A): Oceanic salinity is low in equatorial regions.

Reason (R): Equatorial region is characterized by heavy rainfall,


cloudiness and humidity.

Codes:

(a)Both (A) and (R) are true and (R) is correct explanation of (A).

(b)Both (A) and (R) are true, but (R) is not correct explanation of (A).

(c)(A) is true but (R) is false.

(d)(R) is true but (A) is false.

23.The statement - "Systematic geography must move into theoretical


spheres and regional geography into search for generic and not unique
studies" was made by

(a)Richard Hartshone

(b)Issiah Bowman

(c)William Bunge

(d)Peter Haggett
24.In the development of quantitative revolution, a new filed of studies
called

"Socialphysics: stated dimensions of society are analogous to physical


dimensions and include numbers of people, distance and time" was
introduced by

(a)J. Q.Stewart

(b)Edward Ullman

(c)W. L.Garrison

(d)William Warntz

25.Given below are two statements one labelled as Assertion (A) and the
other labelled as Reason (R). Select your answer from the codes given
below:

Assertion (A): The major Thrust of Quantitative Revolution is to bring


law making, model building and theorization in Geography.

Reason (R): After the Second World War, young geographers discarded
the empirical, descriptive and gazetteer type of Geography by using the
mathematical language rather than the language of literature.

Codes:

(a)Both (A) and (R) are true and (R) is correct explanation of (A).

(b)Both (A) and (R) are true, but (R) is not correct explanation of (A).

(c)(A) is true but (R) is false.


(d)(R) is true but (A) is false.

26.Match List I with List II and select the correct answer using the codes
given below:

Codes:

27.Match List I and List II


Codes:

28.Which one of the following statements is not true about Humanism in


Geography.

(a)Humanism does not treat humans a machines

(b)Humanistic geography gives importance to human awareness, human


consciousness and human creativity

(c)Humanists accept the reduction of space and place to geometrical


concepts of surface

(d)Humanism developed as criticism against positivism and quantitative


revolution.

29.Match List I with List II


Codes:

30.Who among the following is a strong supporter of Ritter's teleological


views'?

(a)Guyot

(b)Peschel

(c)Gerland

(d)Ratzel

31.A process of population deconcentration away from large urban


settlements to non-metropolitan areas is termed as:

(a)Urban dispersal

(b)Urban sprawl
(c)Sub-urbanization

(d)Counter Urbanization

32.In the acceleration stage of urbanization the urban population constitutes:

(a)More than 70% of the total population of the state

(b)25% to 70%

(c)10% to 25%

(d)Less than 10%

33.The concept of primate city was advocated by:

(a)Jefferson

(b)Zipf

(c)Mumford

(d)Sjoberg

34.Given below are two statements one labelled as Assertion (A) and the
other labelled as Reason (R). Select your answer from the codes given
below:

Assertion (A): Losch's model is less restrictive than Christaller's.

Reason (R): Losch treated each section as having separate range,


threshold, hexagonal hinterland.

Codes:
(a)Both (A) and (R) are true and (R) is correct explanation of (A).

(b)Both (A) and (R) are true, but (R) is not correct explanation of (A).

(c)(A) is true but (R) is false.

(d)(R) is true but (A) is false.

35.Which of the following condition is characterized by second stage of


demographic transition?

(a)Low birth rate, High death rate

(b)High birth rate, High death rate

(c)High birth rate, declining death rate

(d)Low birth rate, Low death rate

36.Match List I and List II

Codes:
37.`Chain migration' is based on:

(a)Job opportunities

(b)Kinship

(c)Proximity to place of earlier Residence

(d)Combination of (a) and (c)

38.Which one of the following is not correctly matched?

(a)EW Burgess - Concept of Metropolis

(b)W J Reily - Law of retail gravitation

(c)G K Zipf - Rank size rule

(d)Patrick Geddes - Concept of conurbation

39.Consider and statements and select the correct answers:

(a)Most of the fishing grounds occur in areas where continental shelf is


wide

(b)Fishing activity is well developed in warm tropical waters

(c)Mixing of warm and cold current brings plant nutrients for fish
(d)Inland fisheries are most significant than any other fishery in India

40.Given below are two statements one labelled as Assertion (A) and the
other labelled as Reason (R). Select your answer from the codes given
below:

Assertion (A): There are regional disparities in spatial organization of


economy.

Reason (R): Interactions among inherent factors in economic activities


affect the texture of spatial organization.

Codes:

(a)Both (A) and (R) are true and (R) is correct explanation of (A).

(b)Both (A) and (R) are true, but (R) is not correct explanation of (A).

(c)(A) is true but (R) is false.

(d)(R) is true but (A) is false.

41.Which one of the following crop practices refers to crop rotation'?

(a)Cultivating more than one crop in the same field in different seasons.

(b)Cultivating the same crop in the same field in different seasons.

(c)Cultivating many crops in the same field in one season.

(d)Cultivating one crop in one season followed by a different crop in the


next season in the same field.
42.When the raw material used in an industry is gross and ubiquitous, the
location of the firm will be:

(a)Near the source of raw material

(b)Anywhere between the source of raw material and market

(c)Near the market

(d)Near the capital

43.In Great Britain a little pulp is produced but paper making is important,
while in Sweden pulp manufacture is far more important than paper
making because

(a)Great Britain is technologically advanced as compared to Sweden.

(b)Pulp making is raw material oriented, paper making is more market


oriented.

(c)Sweden is a poor country.

(d)Great Britain has plenty of forest resources for paper making.

44.Match List I with List II

Codes:
45.Break-Of-bulk point is a place:

(a)Where large sized and bulky goods are crushed into.

(b)Where goods are referred from one mode of transport to emphasize the
policy oriented another.

(c)Where packaging of the bulky and goods is carried out at large scale

(d)The place meant for transport of bulky goods

46.Consider the following road network diagram Connectivity Matrix


Diagram Connectivity road matrix diagram Select the correct connectivity
matrix from the following
47.Which of the following theories emphasizes the policy oriented concepts
such as propulsive firms, leading industries and agglomerations?

(a)Central place theory

(b)Theory of industrial location

(c)Growth pole theory

(d)Theory of spatial organization

48.The techniques usually employed for actual delineation of functional


regions are:

(a)Flow and gravitational analysis

(b)Simple correlation analysis

(c)Composite index method analysis

(d)Ranking coefficient method analysis

49.The gravity principle in identification of nodal regions states that the


interaction between two geographic points is directly related to:
(a)Their masses

(b)Their distance

(c)Their mode of transport

(d)Their size of settlements

50.Match the following:

Codes:

51.Given below are two statements one labelled as Assertion (A) and the
other labelled as Reason (R). Select your answer from the codes given
below:
Assertion (A): A planning region should be large enough to contain a
range of resources, conditions and attributes so as to serve a desired
degree of economic viability.

Reason (R): Its resource position should be such that a satisfactory level
of product combination for consumption and for exchange would be
feasible.

Codes:

(a)Both (A) and (R) are true and (R) is correct explanation of (A).

(b)Both (A) and (R) are true, but (R) is not correct explanation of (A).

(c)(A) is true but (R) is false.

(d)(R) is true but (A) is false.

52.Preparation of Sub-plans on regional basis was taken up in:

(a)Fourth plan

(b)Fifth plan

(c)Sixth plan

(d)Seventh plan

53.Who among the following presented a framework of economic regions of


different orders following the soviet concept of economic regions and
production specialization?

(a)P. Sen Gupta


(b)V. Nath

(c)S. P.Chatterjee

(d)Bhat and Prakasa Rao

54.Which one of the following states experienced the lowest growth rate of
literates during 2001-2011?

(a)Tamil Nadu

(b)Nagaland

(c)Kerala

(d)Goa

55.Which of the following places in India does not get vertical rays of the
sun?

(a)Srinagar

(b)Mumbai

(c)Chennai

(d)Thiruvananthapuram

56.Which one of the following atomic power stations is located in Tamil


Nadu? (a) Tarapur

(b)Kota

(c)Kalpakkam
(d)Narora

57.Which one of the following matches is not-correct'?

(a)Himadri - Greater Himalaya

(b)Siwaliks - Outer Himalaya

(c)Western Ghats Peninsular India

(d)Gondwanaland - Lesser Himalaya

58.Which one of the following major seaports of India does not have natural
harbor?

(a)Mumbai

(b)Cochin

(c)Marmagao

(d)Paradeep

59.The member of agro-climatic regions in India as per Planning


Commission is

(a)Twelve

(b)Thirteen

(c)Fourteen

(d)Fifteen

60.About three-fourth of manganese ore reserves of India are found in:


(a)Orissa, Madhya Pradesh, Maharashtra, Goa

(b)Karnataka, Orissa, Madhya Pradesh, Maharashtra

(c)Karnataka, Madhya Pradesh, Goa, Andhra Pradesh

(d)Madhya Pradesh, Maharashtra, Andhra Pradesh, Jharkhand

61.Match List I with List II

Codes:

62.Which of following is a GIS operation'?

(a)Image displaying

(b)Contrast stretching
(c)Map overlaying

(d)Map designing

63.Which of the following diagrams is used to show proportion of different


land uses in an area?

(a)Pie

(b)Wind rose

(c)Line

(d)Flow-Chart

64.Mean, median and mode coincide if the distribution of values is

(a)Negatively skewed

(b)Normal

(c)Positively skewed

(d)Poisson

65.The contour interval followed in SOI Toposheet with 1: 150,000 scale is:

(a)20 Metres

(b)50 Metres

(c)100 Metres

(d)150 Metres
66.Match List I with List II

Codes:

67.The most appropriate sampling technique to represent the heterogeneous


population of a region is:

(a)Cluster

(b)Random stratified

(c)Systematic

(d)Purposive

68.Which one of the following characteristic features of Indian federalism


does not have a geographical base'?
(a)Vast area size

(b)Physical and cultural diversities

(c)Regional inequalities in socio economic development

(d)Division of powers and functions between Union and states

69.Which of the following countries is known as a Unique assemblage of


human races in the world?

(a)Union of South Africa

(b)U.S.A.

(c)Indonesia

(d)India

70.`Geostrategic model', dividing the world into ten regions, was


propounded by:

(a)P. J.Taylor

(b)John Short

(c)John Agnew

(d)Saul Cohen

71.Match List I with List II


Codes:

72.For measuring the success of changes in electoral boundaries we use


concept of:

(a)Representation

(b)Malapportionment

(c)Legitimate participation

(d)Non-legitimate participation

73.Welfare approach in geographic studies emphasizes spatial variation in

(a)socio economic development

(b)Public-private partnership in development

(c)Infrastructural development
(d)Quality of life

74.The `Cultural Turn' in geography has been responsible for:

(a)Bringing social and cultural geography closer to each other

(b)Social geography taking over cultural geography

(c)Creating void between social and cultural geography.

(d)Reducing the significance of Social geography.

75.Match List I with List II and select the correct answer from the codes
given below:

Codes:

ANSWER KEYS

You might also like